Chapter: General Physiology; Topic: Sensory System; Subtopic: Mechanoreceptors and Tactile Sensation
Key Definitions & Concepts
Two-Point Discrimination: The ability to discern that two nearby objects touching the skin are truly two distinct points, rather than one. It depends on receptor density and receptive field size.
Meissner's Corpuscle: A rapidly adapting mechanoreceptor located in the dermal papillae of glabrous (hairless) skin. It has a very small receptive field, making it ideal for fine touch and low-frequency vibration.
Merkel's Disc: A slowly adapting receptor found in the basal epidermis. It also has small receptive fields and detects static pressure and texture.
Pacinian Corpuscle: A rapidly adapting receptor located deep in the dermis. It has very large receptive fields and is specialized for high-frequency vibration.
Ruffini Ending: A slowly adapting receptor in the dermis with large receptive fields, sensitive to skin stretch.
Receptive Field: The specific area of skin that, when stimulated, activates a single sensory neuron. Smaller fields allow higher resolution (better two-point discrimination).
Rapidly Adapting (Phasic): Receptors that respond only at the onset and offset of a stimulus (Meissner, Pacinian). Good for detecting motion/flutter.
Slowly Adapting (Tonic): Receptors that fire continuously as long as the stimulus is present (Merkel, Ruffini). Good for detecting shape/pressure.
Glabrous Skin: Hairless skin (palms, soles, lips) which has the highest density of Meissner's corpuscles and the finest spatial resolution.
Dorsal Column-Medial Lemniscus Pathway: The ascending tract that carries fine touch, vibration, and two-point discrimination signals to the cortex.
[Image of Cutaneous mechanoreceptors diagram]
Lead Question - 2016
Two point discrimination is mainly a function of which touch receptors?
a) Merkel's disc
b) Ruffini's end organ
c) Paccinian corpuscle
d) Meissner's corpuscle
Explanation: Two-point discrimination requires the ability to resolve fine spatial details. This ability is directly related to the size of the receptor's receptive field: the smaller the field, the higher the resolution. Both Merkel's discs and Meissner's corpuscles have small receptive fields. However, Meissner's Corpuscles are particularly abundant in the dermal papillae of glabrous skin (fingertips, lips) where spatial acuity is highest. They are rapidly adapting and extremely sensitive to light touch and dynamic deformation. While Merkel's discs (static pressure) also contribute significantly to texture perception, Meissner's corpuscles are classically associated with the highest density in areas of fine tactile discrimination. Pacinian and Ruffini receptors have large fields and poor spatial resolution. Therefore, the correct answer is d) Meissner's corpuscle.
1. Which mechanoreceptor is most sensitive to high-frequency vibration (200-300 Hz)?
a) Meissner's Corpuscle
b) Merkel's Disc
c) Pacinian Corpuscle
d) Ruffini Ending
Explanation: The Pacinian Corpuscle (Lamellar corpuscle) is a large, onion-like structure located deep in the dermis and subcutaneous tissue. Its capsule acts as a mechanical filter that allows only high-frequency dynamic changes to reach the nerve terminal. Consequently, it is exquisitely sensitive to High-frequency vibration (around 250 Hz) and deep pressure changes. It is Rapidly Adapting. Meissner's corpuscles detect low-frequency vibration ("flutter," ~50 Hz). Merkel and Ruffini are slowly adapting and do not detect vibration well. Therefore, the correct answer is c) Pacinian Corpuscle.
2. A blind person reading Braille relies heavily on the ability to detect static pressure and texture edges. This function is primarily mediated by:
a) Meissner's Corpuscles
b) Merkel's Discs
c) Pacinian Corpuscles
d) Hair Follicle Receptors
Explanation: Reading Braille requires high spatial resolution to distinguish the dots. While Meissner's corpuscles detect the initial contact and motion across the dots, the detailed analysis of the shape, edges, and texture (static pressure) is the domain of Merkel's Discs. Merkel's discs are Slowly Adapting (Type I) receptors with very small receptive fields located at the base of the epidermis. They provide a continuous signal that maps the pressure pattern of the Braille dots with high fidelity. Therefore, the correct answer is b) Merkel's Discs.
3. Which receptor is classified as Slowly Adapting Type II (SA-II) and is sensitive to skin stretch?
a) Merkel's Disc
b) Ruffini Ending
c) Meissner's Corpuscle
d) Pacinian Corpuscle
Explanation: Mechanoreceptors are classified by adaptation rate and receptive field size. SA-I (Small field): Merkel. RA-I (Small field): Meissner. RA-II (Large field): Pacinian. SA-II (Large field): Ruffini Ending. Ruffini Endings are spindle-shaped receptors located in the dermis. They are slowly adapting and are particularly sensitive to lateral skin stretch and joint position/rotation. This makes them important for proprioception (sensing hand shape) and detecting slippage of objects. Therefore, the correct answer is b) Ruffini Ending.
4. The term "Glabrous Skin" refers to skin that lacks:
a) Sweat glands
b) Hair follicles
c) Epidermis
d) Sensory receptors
Explanation: Glabrous Skin is the technical term for Hairless skin. It is found on the palms of the hands, soles of the feet, and lips. This skin type is thicker, has distinct ridges (fingerprints), and is specialized for discriminative touch. Importantly, it contains Meissner's Corpuscles and Merkel's Discs in high density but lacks Hair Follicle receptors. Hairy skin covers the rest of the body and relies more on hair movement for tactile sensitivity. Therefore, the correct answer is b) Hair follicles.
5. Which sensation is carried by the Dorsal Column-Medial Lemniscus pathway?
a) Pain and Temperature
b) Crude Touch and Pressure
c) Fine Touch, Vibration, and Proprioception
d) Itch and Tickle
Explanation: The somatosensory system has two main ascending pathways. The Anterolateral System (Spinothalamic) carries Pain, Temperature, and Crude touch. The Dorsal Column-Medial Lemniscus (DCML) pathway carries Fine (Discriminative) Touch, Vibration, and Proprioception. Signals from Meissner's, Merkel's, Pacinian, and Ruffini receptors are transmitted via large, myelinated A-beta fibers through the DCML to the cortex, preserving the high spatial and temporal fidelity required for tasks like two-point discrimination. Therefore, the correct answer is c) Fine Touch, Vibration, and Proprioception.
6. The minimal distance at which two distinct points applied to the skin are perceived as two separate points is smallest on the:
a) Back
b) Thigh
c) Fingertips
d) Forearm
Explanation: The "Two-Point Threshold" varies across the body. It is determined by the size of the receptive fields and the extent of cortical representation (cortical magnification). Areas with small receptive fields and large cortical areas have high resolution (small threshold). The Fingertips (and lips/tongue) have the highest density of Meissner/Merkel receptors and the smallest receptive fields (~2-4 mm threshold). The back has sparse receptors with large fields, resulting in a very poor threshold (~40 mm). Therefore, the correct answer is c) Fingertips.
7. Rapid adaptation in the Pacinian Corpuscle is primarily a function of its:
a) Ion channels
b) Axon diameter
c) Connective tissue capsule (Lamellae)
d) Depth in the skin
Explanation: The Pacinian corpuscle consists of a central nerve ending surrounded by concentric layers of connective tissue (lamellae) with fluid in between. This Capsule acts as a mechanical filter. When pressure is applied, the fluid redistributes, dissipating the pressure on the nerve ending within milliseconds. Thus, the nerve fires only at the onset (compression) and offset (release) of the stimulus, but not during steady pressure. If the capsule is stripped away, the nerve becomes slowly adapting. Therefore, the rapid adaptation is a property of the capsule. Therefore, the correct answer is c) Connective tissue capsule (Lamellae).
8. Which receptors respond to the bending of hairs and are rapidly adapting?
a) Hair Follicle Receptors
b) Merkel Discs
c) Ruffini Endings
d) Free Nerve Endings
Explanation: In hairy skin, the hair shaft acts as a lever arm. Nerve fibers wrap around the base of the follicle, forming the Hair Follicle Receptor (or Peritrichial ending). These are Rapidly Adapting mechanoreceptors. They fire a burst of action potentials when the hair is bent (movement) but stop firing if the hair is held in the bent position. They are exquisitely sensitive to light touch and air movement (e.g., a mosquito landing). They essentially replace Meissner's corpuscles in hairy skin. Therefore, the correct answer is a) Hair Follicle Receptors.
9. The phenomenon of "Lateral Inhibition" in the somatic sensory system serves to:
a) Increase the firing rate of all neurons
b) Sharpen the contrast and localization of the stimulus
c) Decrease the threshold for pain
d) Prolong the duration of the sensation
Explanation: To improve two-point discrimination, the nervous system must distinguish the central peak of excitation from the surrounding "fringe." Lateral Inhibition achieves this. An excited neuron inhibits its immediate neighbors via interneurons. This suppresses the weaker signals at the edge of the stimulus while allowing the strong central signal to pass. This "center-surround" antagonism Sharpens the contrast between the stimulus and the background, significantly enhancing the brain's ability to localize the exact point of contact. Therefore, the correct answer is b) Sharpen the contrast and localization of the stimulus.
10. Which fiber type transmits signals from the cutaneous mechanoreceptors (Meissner, Merkel, Pacinian, Ruffini)?
a) A-alpha (Ia)
b) A-beta (II)
c) A-delta (III)
d) C fibers (IV)
Explanation: Tactile information requires rapid transmission for timely motor adjustments and texture analysis. The axons innervating the encapsulated mechanoreceptors of the skin are large, myelinated fibers classified as A-beta (Type II) fibers. They have conduction velocities of 30-70 m/s. A-alpha fibers are for proprioceptors (muscle spindle) and somatic motor. A-delta and C fibers carry pain and temperature. Therefore, the correct answer is b) A-beta (II).
Chapter: Neurophysiology / Pharmacology; Topic: Neurotransmitters and Receptors; Subtopic: Serotonin (5-HT) Receptor Families
Key Definitions & Concepts
Metabotropic Receptors: G-Protein Coupled Receptors (GPCRs). They act indirectly via second messenger systems (cAMP, IP3/DAG) and are slower in action. Most 5-HT receptors belong to this class.
Ionotropic Receptors: Ligand-Gated Ion Channels. They act directly by opening an ion pore upon ligand binding, leading to rapid depolarization or hyperpolarization.
5-HT (Serotonin): A monoamine neurotransmitter derived from Tryptophan, involved in mood, sleep, appetite, and emesis.
5-HT1 Family (Gi/o coupled): Inhibitory GPCRs (e.g., 5-HT1A, 5-HT1B, 5-HT1D). They decrease cAMP and are often presynaptic autoreceptors.
5-HT2 Family (Gq coupled): Excitatory GPCRs (e.g., 5-HT2A, 5-HT2C). They increase IP3 and DAG, leading to calcium release. 5-HT2A is the target of atypical antipsychotics and psychedelics.
5-HT3 Receptor: The unique Ionotropic receptor in the serotonin family. It is a ligand-gated cation channel (Na+/K+) involved in rapid excitatory transmission and the vomiting reflex.
5-HT4, 6, 7 (Gs coupled): Excitatory GPCRs that increase cAMP levels.
Ondansetron: A selective 5-HT3 antagonist used as a potent antiemetic, confirming the receptor's role in the Chemoreceptor Trigger Zone (CTZ).
Buspirone: A partial agonist at 5-HT1A receptors, used as an anxiolytic.
Sumatriptan: An agonist at 5-HT1B/1D receptors, used to treat migraine headaches.
Lead Question - 2016
Which of the following is not a metabotropic receptor for serotonin?
a) 5HT1A
b) 5HT1B
c) 5HT2A
d) 5HT3
Explanation: The serotonin (5-Hydroxytryptamine, 5-HT) receptor family is large and diverse, with 7 main families (5-HT1 to 5-HT7). Almost all of these are Metabotropic receptors, meaning they are G-Protein Coupled Receptors (GPCRs) that signal via second messengers. The 5-HT1 family (A, B, D, E, F) is Gi-coupled. The 5-HT2 family (A, B, C) is Gq-coupled. The 5-HT4, 6, 7 families are Gs-coupled. The single, distinct exception is the 5-HT3 receptor. The 5-HT3 receptor is an Ionotropic receptor (Ligand-gated ion channel) that is permeable to cations (Na+, K+), mediating fast excitatory neurotransmission. Therefore, it is not a metabotropic receptor. Therefore, the correct answer is d) 5HT3.
1. The 5-HT3 receptor functions as a ligand-gated ion channel. When activated, it is primarily permeable to:
a) Chloride (Cl-)
b) Sodium (Na+) and Potassium (K+)
c) Calcium (Ca2+) only
d) Magnesium (Mg2+)
Explanation: Structure dictates function. The 5-HT3 receptor belongs to the Cys-loop superfamily of ligand-gated ion channels, structurally similar to Nicotinic ACh receptors. It is a non-selective Cation Channel. Upon binding of serotonin, the channel pore opens, allowing the influx of Sodium (Na+) and efflux of Potassium (K+), leading to rapid depolarization (excitation) of the neuron. This rapid excitation is crucial in the vomiting center and pain pathways. Inhibitory channels (like GABA-A) conduct Chloride. Therefore, the correct answer is b) Sodium (Na+) and Potassium (K+).
2. Which 5-HT receptor subtype is the primary target for the "Setron" class of antiemetic drugs (e.g., Ondansetron, Granisetron)?
a) 5-HT1A
b) 5-HT2C
c) 5-HT3
d) 5-HT4
Explanation: Chemotherapy and radiation cause the release of serotonin from enterochromaffin cells in the gut. This serotonin activates 5-HT3 receptors on vagal afferents and in the Chemoreceptor Trigger Zone (CTZ) of the area postrema, initiating the vomiting reflex. Drugs like Ondansetron are potent and selective Antagonists of the 5-HT3 receptor. By blocking this ionotropic receptor, they effectively prevent the rapid depolarization that triggers nausea and vomiting. Therefore, the correct answer is c) 5-HT3.
3. 5-HT1A receptors are Gi-coupled GPCRs. Their activation leads to neuronal inhibition primarily by opening channels for which ion?
a) Sodium
b) Calcium
c) Potassium
d) Chloride
Explanation: 5-HT1A receptors are major inhibitory autoreceptors found on the soma and dendrites of serotonergic neurons (e.g., Raphe nuclei). They are coupled to Gi/o proteins. The beta-gamma subunit of the G-protein directly activates G-protein coupled Inwardly Rectifying Potassium (GIRK) channels. Opening these channels leads to Potassium efflux, which hyperpolarizes the membrane (IPSP), thereby inhibiting neuronal firing. They also inhibit adenylyl cyclase. This mechanism provides negative feedback regulation of serotonin release. Therefore, the correct answer is c) Potassium.
4. The psychedelic effects of LSD and Psilocybin are primarily mediated by their partial agonist activity at which serotonin receptor?
a) 5-HT1A
b) 5-HT2A
c) 5-HT3
d) 5-HT7
Explanation: The 5-HT2A receptor is a Gq-coupled receptor abundant in the cortex. It increases intracellular calcium and neuronal excitability. Classic hallucinogens (LSD, Mescaline, Psilocybin) act as high-affinity partial agonists at the 5-HT2A receptor. Activation of postsynaptic 5-HT2A receptors on pyramidal neurons in the prefrontal cortex is believed to cause the distortions in perception and cognition associated with these drugs. Atypical antipsychotics (like Clozapine) block this receptor. Therefore, the correct answer is b) 5-HT2A.
5. Sumatriptan is an effective treatment for acute migraine attacks. Its mechanism involves vasoconstriction of cranial blood vessels via activation of:
a) 5-HT1B and 5-HT1D receptors
b) 5-HT2A receptors
c) 5-HT3 receptors
d) 5-HT4 receptors
Explanation: Triptans are 5-HT1B/1D agonists. 5-HT1B receptors are located on the smooth muscle of cranial blood vessels and mediate vasoconstriction. 5-HT1D receptors are located on the presynaptic terminals of the Trigeminal nerve and inhibit the release of vasoactive neuropeptides (CGRP, Substance P). By activating these Gi-coupled receptors, Sumatriptan reverses the vasodilation and neurogenic inflammation associated with migraine pain. Therefore, the correct answer is a) 5-HT1B and 5-HT1D receptors.
6. Which of the following 5-HT receptors is coupled to the Gs protein, leading to increased cAMP levels and gut motility?
a) 5-HT1
b) 5-HT2
c) 5-HT3
d) 5-HT4
Explanation: The 5-HT4 receptor is a Gs-coupled GPCR. Its activation stimulates Adenylyl Cyclase, increasing cAMP and PKA activity. In the enteric nervous system, 5-HT4 activation facilitates the release of Acetylcholine, thereby enhancing peristalsis and gut motility. Prokinetic drugs like Prucalopride (and historically Cisapride) are selective 5-HT4 agonists used to treat chronic constipation. 5-HT1 is Gi. 5-HT2 is Gq. 5-HT3 is an ion channel. Therefore, the correct answer is d) 5-HT4.
7. Atypical antipsychotics like Risperidone and Olanzapine exert their therapeutic effect partly by antagonizing the 5-HT2A receptor. This receptor signaling pathway involves:
a) Inhibition of Adenylyl Cyclase
b) Activation of Phospholipase C (PLC)
c) Opening of Chloride channels
d) Activation of Guanylyl Cyclase
Explanation: The 5-HT2 family (2A, 2B, 2C) are coupled to Gq proteins. Activation of these receptors stimulates the enzyme Phospholipase C (PLC). PLC cleaves membrane lipids to generate IP3 and DAG. IP3 triggers the release of intracellular Calcium, while DAG activates Protein Kinase C. Antagonism of this excitatory 5-HT2A pathway, combined with D2 blockade, is the hallmark mechanism of second-generation (atypical) antipsychotics, helping to reduce extrapyramidal side effects. Therefore, the correct answer is b) Activation of Phospholipase C (PLC).
8. Appetite suppression is a known effect of stimulating which specific serotonin receptor subtype (targeted by the drug Lorcaserin)?
a) 5-HT1A
b) 5-HT2C
c) 5-HT3
d) 5-HT6
Explanation: Serotonin is a key satiety signal in the hypothalamus. Specifically, activation of 5-HT2C receptors on POMC neurons in the Arcuate Nucleus promotes satiety and reduces food intake. Lorcaserin was an anti-obesity drug designed as a selective 5-HT2C agonist to suppress appetite without the cardiovascular risks associated with non-selective 5-HT agents (like Fen-Phen, which activated 5-HT2B on heart valves). Therefore, the correct answer is b) 5-HT2C.
9. 5-HT1B and 5-HT1D receptors function primarily as:
a) Postsynaptic excitatory receptors
b) Presynaptic inhibitory autoreceptors/heteroreceptors
c) Ligand-gated ion channels
d) Growth factor receptors
Explanation: While 5-HT1A is the somatodendritic autoreceptor, the 5-HT1B and 5-HT1D receptors are typically located on the axon terminals. They function as Presynaptic Inhibitory Receptors. Autoreceptors (1B): On serotonin terminals, detecting 5-HT and inhibiting further release. Heteroreceptors (1D): On non-serotonergic terminals (e.g., trigeminal nerve releasing CGRP), inhibiting the release of other neurotransmitters. This presynaptic inhibition (Gi-mediated) is the basis for their use in migraine and their regulatory role. Therefore, the correct answer is b) Presynaptic inhibitory autoreceptors/heteroreceptors.
10. Buspirone is a non-benzodiazepine anxiolytic that acts as a partial agonist at which receptor?
a) GABA-A
b) 5-HT1A
c) 5-HT2A
d) 5-HT3
Explanation: Buspirone (Buspar) is distinct from benzodiazepines (which act on GABA receptors). Its mechanism of action is functioning as a Partial Agonist at 5-HT1A receptors. By stimulating these inhibitory presynaptic autoreceptors (and some postsynaptic receptors) in the raphe and limbic system, it modulates serotonergic firing rates, reducing anxiety without significant sedation or addiction potential. Therefore, the correct answer is b) 5-HT1A.
Chapter: Neurophysiology; Topic: Motor System; Subtopic: Functional Divisions of the Cerebellum
Key Definitions & Concepts
Vestibulocerebellum (Archicerebellum): Consists of the Flocculonodular lobe; primarily responsible for the maintenance of equilibrium (balance) and the coordination of eye movements (VOR).
Spinocerebellum (Paleocerebellum): Consists of the Vermis and Paravermis; regulates muscle tone and coordinates axial and proximal limb movements by comparing intended motor commands with actual performance.
Cerebrocerebellum (Neocerebellum): Consists of the Lateral Hemispheres; involved in the planning, programming, and initiation of skilled, sequential voluntary movements.
Dysmetria: A sign of cerebellar damage (Neocerebellum) characterized by the inability to judge distance, leading to overshooting (hypermetria) or undershooting the target.
Intention Tremor: A tremor that occurs during voluntary movement and worsens as the limb approaches the target; distinct from the resting tremor of Parkinson's.
Dysdiadochokinesia: The inability to perform rapid alternating movements (like supination/pronation) smoothly.
Nystagmus: Involuntary eye movements often seen with Vestibulocerebellar lesions due to loss of oculomotor coordination.
Ataxia: Lack of coordination of voluntary muscle movements; can be truncal (vermis) or appendicular (hemispheres).
Motor Learning: The cerebellum plays a crucial role in adapting and fine-tuning motor programs through trial-and-error (synaptic plasticity at parallel fiber-Purkinje cell synapses).
Feed-forward Control: The cerebellum predicts the sensory consequences of movement and adjusts motor output in anticipation, rather than just reacting to feedback.
[Image of Cerebellar functional divisions]
Lead Question - 2016
Function of cerebellum?
a) Regulation of tone
b) Coordination of eye movement
c) Planning & initiation of movement
d) All of the above
Explanation: The cerebellum is a complex motor control center divided into three functional zones, each with distinct responsibilities. 1. Vestibulocerebellum: Controls balance and Coordination of eye movements (Option b). 2. Spinocerebellum: Receives proprioceptive input and regulates Muscle Tone and execution of movements (Option a). 3. Cerebrocerebellum: Communicates with the cortex to assist in the Planning and initiation of movement, particularly skilled, sequential tasks (Option c). Since the cerebellum performs all these functions through its different anatomical subdivisions, the most comprehensive answer is "All of the above." Therefore, the correct answer is d) All of the above.
1. Which part of the cerebellum is evolutionarily the oldest and is primarily concerned with the maintenance of equilibrium?
a) Anterior Lobe
b) Posterior Lobe
c) Flocculonodular Lobe
d) Vermis
Explanation: The cerebellum developed in stages. The oldest part (Archicerebellum) corresponds to the Flocculonodular Lobe. It has direct reciprocal connections with the Vestibular nuclei. Consequently, its primary function is the maintenance of Equilibrium (Balance) and the coordination of head and eye movements. Damage to this area results in truncal ataxia and nystagmus, often resembling vestibular dysfunction. The Anterior lobe is Paleocerebellum (tone/posture). The Posterior lobe is Neocerebellum (skilled movement). Therefore, the correct answer is c) Flocculonodular Lobe.
2. Hypotonia (decreased muscle tone) is a characteristic clinical sign of a lesion in the:
a) Basal Ganglia
b) Spinocerebellum
c) Motor Cortex
d) Thalamus
Explanation: The Spinocerebellum (Vermis and Paravermis) regulates muscle tone by modulating the activity of the descending motor pathways (Reticulospinal and Vestibulospinal tracts) and the gamma motor neurons. It facilitates extensor tone to support the body against gravity. When the cerebellum (specifically the Spinocerebellum or deep nuclei) is damaged, this facilitatory drive is lost. This results in Hypotonia (flaccidity) and pendular reflexes on the side of the lesion. In contrast, Basal Ganglia lesions often cause Rigidity (Hypertonia). Therefore, the correct answer is b) Spinocerebellum.
3. Which deep cerebellar nucleus is functionally associated with the Neocerebellum (Lateral Hemispheres) and involved in motor planning?
a) Fastigial Nucleus
b) Globose Nucleus
c) Emboliform Nucleus
d) Dentate Nucleus
Explanation: The functional divisions of the cerebellum project to specific deep nuclei. Vestibulocerebellum -> Fastigial Nucleus. Spinocerebellum -> Interposed Nuclei (Globose + Emboliform). Neocerebellum (Cerebrocerebellum) -> Dentate Nucleus. The Dentate Nucleus is the largest and most lateral nucleus. It sends fibers via the Dentato-Thalamo-Cortical tract to the premotor and motor cortex, influencing the planning and programming of voluntary movements before they are executed. Therefore, the correct answer is d) Dentate Nucleus.
4. A patient exhibits "Scanning Speech" (staccato speech) and Dysdiadochokinesia. These are manifestations of:
a) Neocerebellar Syndrome
b) Archicerebellar Syndrome
c) Paleocerebellar Syndrome
d) Parkinson's Disease
Explanation: Scanning speech (ataxic dysarthria) and Dysdiadochokinesia (impaired rapid alternating movements) are classic signs of incoordination of voluntary movement. This coordination of timing, force, and sequencing is the function of the Neocerebellum (Lateral Hemispheres). Damage here disrupts the smooth execution of skilled movements, leading to decomposition of movement, dysmetria, and speech defects. Archicerebellar syndrome causes balance issues. Paleocerebellar syndrome causes gait ataxia. Therefore, the correct answer is a) Neocerebellar Syndrome.
5. The cerebellum compares "Intended Movement" with "Actual Performance" to correct errors. Information about "Actual Performance" reaches the cerebellum primarily via the:
a) Corticospinal Tract
b) Spinocerebellar Tracts
c) Corticopontine fibers
d) Rubrospinal Tract
Explanation: The cerebellum acts as a comparator. 1. "Intended Movement" signal comes from the Motor Cortex via Corticopontine fibers. 2. "Actual Performance" signal (proprioception from muscles/joints) comes from the periphery via the Spinocerebellar Tracts (Dorsal and Ventral). The cerebellum compares these two. If there is a mismatch (error), it sends corrective signals back to the motor cortex or brainstem to adjust the movement in real-time. Therefore, the correct answer is b) Spinocerebellar Tracts.
6. Which clinical test is most specific for detecting dysfunction of the cerebellar hemispheres (Appendicular Ataxia)?
a) Romberg's Test
b) Finger-to-Nose Test
c) Testing for clasp-knife rigidity
d) Plantar reflex (Babinski)
Explanation: Cerebellar signs are ipsilateral. Hemispheric lesions cause ataxia of the limbs (appendicular ataxia). The Finger-to-Nose test (or Heel-to-Shin test) requires precise coordination of agonist and antagonist muscles to reach a target. In hemispheric disease, this reveals Dysmetria (past-pointing) and Intention Tremor. Romberg's test evaluates sensory ataxia (dorsal columns) vs. cerebellar ataxia (but is more related to balance/midline). Clasp-knife and Babinski are UMN signs. Therefore, the correct answer is b) Finger-to-Nose Test.
7. The "Damping" function of the cerebellum refers to its ability to:
a) Increase muscle tone
b) Prevent overshoot and stop movement precisely
c) Initiate movement rapidly
d) Amplify sensory input
Explanation: When a limb moves rapidly toward a target, it has momentum. The cerebellum calculates exactly when to inhibit the agonist and activate the antagonist to stop the limb precisely at the intended point. This braking action is called Damping. If the cerebellum is damaged, this damping function fails. The limb overshoots the target (Hypermetria), then overcorrects in the other direction, leading to the oscillation seen as Intention Tremor. Therefore, the correct answer is b) Prevent overshoot and stop movement precisely.
8. The cerebellum is said to act as a "Timing Device" for the brain. This function is particularly relevant for:
a) Reflexes
b) Ballistic movements
c) Slow ramp movements
d) Isometric contraction
Explanation: Ballistic movements (like typing, playing piano, or saccadic eye movements) are too fast for sensory feedback to correct them in real-time. They are pre-programmed. The cerebellum calculates the precise Duration and Sequencing of muscle activation required for these movements. It effectively "times" the onset and offset of different muscle groups to ensure fluid execution. Loss of this timing results in "Decomposition of Movement" (robotic, segmented motion). Therefore, the correct answer is b) Ballistic movements.
9. Which type of eye movement abnormality is most characteristic of a Vestibulocerebellar lesion?
a) Ptosis
b) Ocular Dysmetria and Nystagmus
c) Internuclear Ophthalmoplegia
d) Homonymous Hemianopsia
Explanation: The Vestibulocerebellum (Flocculonodular lobe) is intimately connected with the Vestibular Nuclei. It regulates the Vestibulo-Ocular Reflex (VOR) gain and gaze holding. Lesions here disrupt the ability to fixate the eyes and coordinate eye movements with head movements. This manifests as Nystagmus (typically gaze-evoked or downbeat) and Ocular Dysmetria (overshooting when looking at a target). Ptosis is CN III. INO is MLF lesion. Hemianopsia is visual cortex. Therefore, the correct answer is b) Ocular Dysmetria and Nystagmus.
10. Unlike the Cerebral Cortex, the Cerebellum controls movement on which side of the body?
a) Contralateral
b) Ipsilateral
c) Bilateral
d) Depends on the lobe
Explanation: This is a fundamental rule of neuroanatomy. The Cerebrum controls the Contralateral side (fibers cross). The Cerebellum controls the Ipsilateral side. Why? Because the cerebellar output (via SCP) crosses to the contralateral thalamus/cortex. The cortex then sends the Corticospinal tract which crosses back (at the pyramids) to the original side. This "double-crossing" means the cerebellum influences the side of the body on which it resides. A right cerebellar lesion causes ataxia of the right arm/leg. Therefore, the correct answer is b) Ipsilateral.
Chapter: Neurophysiology / Special Senses; Topic: Gustatory System; Subtopic: Taste Pathway and Innervation
Key Definitions & Concepts
Chorda Tympani: A branch of the Facial Nerve (CN VII) carrying taste sensation from the anterior 2/3 of the tongue (fungiform papillae).
Glossopharyngeal Nerve (CN IX): Carries taste sensation from the posterior 1/3 of the tongue (circumvallate and foliate papillae).
Vagus Nerve (CN X): Carries taste sensation from the epiglottis, pharynx, and extreme posterior tongue (via the Internal Laryngeal Nerve).
Nucleus Tractus Solitarius (NTS): The primary gustatory relay nucleus in the medulla where all taste afferents (VII, IX, X) converge and synapse.
Ventral Posteromedial (VPM) Nucleus: The specific thalamic nucleus that receives gustatory input from the NTS and relays it to the cortex.
Primary Gustatory Cortex: Located in the Insula and frontal operculum; responsible for the conscious perception of taste.
Geniculate Ganglion: The sensory ganglion for the taste fibers of the Facial Nerve.
Petrosal Ganglion: The sensory ganglion for the taste fibers of the Glossopharyngeal Nerve.
Nodose Ganglion: The sensory ganglion for the taste fibers of the Vagus Nerve.
Ageusia: The complete loss of taste function, often requiring damage to multiple cranial nerves or the central pathway.
[Image of Gustatory pathway cranial nerves]
Lead Question - 2016
Gustatory pathway involves which nerve?
a) Facial
b) Glossopharyngeal
c) Vagus
d) All of the above
Explanation: The sense of taste (Gustation) is mediated by taste buds located in different regions of the oral cavity and pharynx. The innervation is divided among three cranial nerves. The Facial nerve (CN VII) supplies the anterior two-thirds of the tongue (via Chorda Tympani) and the soft palate. The Glossopharyngeal nerve (CN IX) supplies the posterior one-third of the tongue (including Vallate papillae). The Vagus nerve (CN X) supplies the epiglottis and extreme posterior tongue. All three nerves project centrally to the Nucleus Tractus Solitarius. Therefore, the gustatory pathway involves all three. Therefore, the correct answer is d) All of the above.
1. The cell bodies of the primary gustatory neurons carrying taste from the anterior two-thirds of the tongue are located in the:
a) Trigeminal ganglion
b) Geniculate ganglion
c) Petrosal ganglion
d) Otic ganglion
Explanation: Taste from the anterior 2/3 of the tongue is carried by the Chorda Tympani branch of the Facial Nerve. These are special visceral afferent (SVA) fibers. Like all primary sensory neurons, their cell bodies must be located in a sensory ganglion outside the CNS. For the Facial Nerve's sensory component (nervus intermedius), this is the Geniculate Ganglion, located in the temporal bone. The Petrosal ganglion is for CN IX. The Trigeminal ganglion is for somatic sensation (touch/pain) of the tongue, not taste. The Otic ganglion is autonomic (parasympathetic). Therefore, the correct answer is b) Geniculate ganglion.
2. All primary gustatory afferent fibers (from CN VII, IX, and X) enter the brainstem and terminate in the rostral part of the:
a) Nucleus Ambiguus
b) Spinal Nucleus of Trigeminal
c) Nucleus Tractus Solitarius (NTS)
d) Superior Salivatory Nucleus
Explanation: The Nucleus Tractus Solitarius (NTS) is the major visceral sensory nucleus of the brainstem. It is functionally divided. The caudal part receives general visceral afferents (baroreceptors, gut distension). The Rostral part (Gustatory Nucleus) specifically receives special visceral afferent (taste) fibers from the Facial, Glossopharyngeal, and Vagus nerves. This convergence is the first central relay station for the taste pathway. The Nucleus Ambiguus is motor. The Spinal Trigeminal is for pain/temp. Therefore, the correct answer is c) Nucleus Tractus Solitarius (NTS).
3. Which thalamic nucleus serves as the relay station for taste sensation before it projects to the cerebral cortex?
a) Ventral Posterolateral (VPL)
b) Ventral Posteromedial (VPM)
c) Medial Geniculate Body
d) Anterior Nucleus
Explanation: Second-order neurons from the Nucleus Tractus Solitarius ascend ipsilaterally in the Central Tegmental Tract to reach the thalamus. The specific destination is the parvocellular part of the Ventral Posteromedial (VPM) nucleus. This nucleus handles sensory input from the head and face. The VPL handles the body (spinal cord). Thus, both somatic sensation of the tongue (Trigeminal) and taste sensation (VII, IX, X) relay through different sub-regions of the VPM. Therefore, the correct answer is b) Ventral Posteromedial (VPM).
4. A patient complains of loss of taste on the back of the tongue. This symptom is most likely due to a lesion of the:
a) Lingual Nerve
b) Chorda Tympani
c) Glossopharyngeal Nerve
d) Hypoglossal Nerve
Explanation: The innervation of the tongue is segmented. The anterior 2/3 is supplied by the Facial nerve (Chorda Tympani). The posterior 1/3, which contains the large Circumvallate papillae and Foliate papillae, is supplied by the Glossopharyngeal Nerve (CN IX). A lesion here (e.g., jugular foramen syndrome) causes loss of taste and somatic sensation on the posterior third of the tongue, along with loss of the gag reflex. The Hypoglossal nerve is purely motor. The Lingual nerve carries somatic sensation for the anterior 2/3. Therefore, the correct answer is c) Glossopharyngeal Nerve.
5. The Primary Gustatory Cortex is located in the:
a) Postcentral Gyrus
b) Frontal Operculum and Anterior Insula
c) Superior Temporal Gyrus
d) Occipital Lobe
Explanation: From the VPM nucleus of the thalamus, third-order neurons project to the cerebral cortex for conscious perception. The Primary Gustatory Cortex (Area 43) is located in two adjacent regions: the Frontal Operculum (the lower part of the postcentral gyrus covering the insula) and the Anterior Insula. This cortical area allows for the discrimination of different taste qualities. The postcentral gyrus handles touch. The temporal lobe handles hearing. Therefore, the correct answer is b) Frontal Operculum and Anterior Insula.
6. The Circumvallate papillae, despite being located anterior to the terminal sulcus, are embryologically derived from the third arch and thus innervated by:
a) Facial Nerve
b) Glossopharyngeal Nerve
c) Vagus Nerve
d) Trigeminal Nerve
Explanation: This is a classic anatomical exception. The V-shaped Sulcus Terminalis divides the tongue into anterior 2/3 and posterior 1/3. The large Circumvallate papillae are arranged in a row just anterior to this sulcus. However, during development, the mucosa of the posterior 1/3 overgrows slightly to include these papillae. Therefore, despite their location, the Circumvallate papillae are innervated by the nerve of the posterior 1/3, which is the Glossopharyngeal Nerve (CN IX), not the Facial nerve. Therefore, the correct answer is b) Glossopharyngeal Nerve.
7. Which cranial nerve carries taste sensation from the taste buds found on the epiglottis?
a) CN VII
b) CN IX
c) CN X
d) CN XII
Explanation: Taste buds are not limited to the tongue; they are also found on the soft palate, pharynx, and epiglottis. The taste buds on the laryngeal surface of the epiglottis and the vallecula are innervated by the Vagus Nerve (CN X), specifically its Superior Laryngeal branch (Internal Laryngeal Nerve). These taste buds are more prominent in infants (protective reflex against swallowing bad milk) and tend to atrophy in adults. Therefore, the correct answer is c) CN X.
8. The "Salty" taste is transduced primarily by the influx of which ion through open membrane channels?
a) Hydrogen (H+)
b) Potassium (K+)
c) Sodium (Na+)
d) Calcium (Ca2+)
Explanation: Taste transduction mechanisms vary by modality. Salty: Direct influx of Sodium (Na+) through amiloride-sensitive epithelial sodium channels (ENaC), causing depolarization. Sour: Influx of H+ (protons) blocking K+ channels. Sweet, Bitter, Umami: Activation of G-protein coupled receptors (T1R/T2R families) leading to second messenger cascades. Therefore, salty taste is the simplest, involving direct ion entry. Therefore, the correct answer is c) Sodium (Na+).
9. Unilateral damage to the Chorda Tympani nerve results in:
a) Loss of all taste on the ipsilateral side
b) Loss of taste on the ipsilateral anterior 2/3 of the tongue
c) Loss of general sensation (pain) on the anterior tongue
d) Paralysis of the tongue muscles
Explanation: The Chorda Tympani is purely sensory/secretomotor. It carries taste fibers from the anterior 2/3 of the tongue and parasympathetic fibers to the submandibular ganglion. Damage (e.g., during ear surgery) results in Ageusia (loss of taste) specifically on the Ipsilateral anterior 2/3 of the tongue. It does not affect the posterior 1/3 (CN IX). It does not affect general sensation (touch/pain), which is carried by the Lingual nerve (V3), although the two nerves run together. Tongue movement is CN XII. Therefore, the correct answer is b) Loss of taste on the ipsilateral anterior 2/3 of the tongue.
10. The pathway connecting the Nucleus Tractus Solitarius to the Parabrachial Nucleus and then to the Hypothalamus/Amygdala mediates the:
a) Conscious discrimination of taste
b) Motor reflex of swallowing
c) Affective (emotional) and behavioral response to taste
d) Salivary reflex only
Explanation: Taste information splits at the NTS. The thalamocortical pathway (to VPM and Cortex) is for conscious perception/discrimination. A second pathway projects to the Parabrachial Nucleus (in the pons), which then relays to the Hypothalamus and Amygdala (Limbic system). This limbic connection is responsible for the Affective and behavioral aspects of taste: whether a taste is pleasant (rewarding) or disgusting (aversive), leading to feeding or avoidance behaviors. Therefore, the correct answer is c) Affective (emotional) and behavioral response to taste.
Chapter: Autonomic Nervous System; Topic: Sympathetic Innervation; Subtopic: Innervation of Sweat Glands
Key Definitions & Concepts
Eccrine Sweat Glands: The most numerous sweat glands (distributed over the entire body) responsible for thermoregulation.
Apocrine Sweat Glands: Found in axilla and perineum; secrete a viscous fluid. They are innervated by adrenergic fibers but are less significant for general thermoregulation.
Sympathetic Cholinergic Fibers: The unique postganglionic sympathetic neurons that release Acetylcholine (instead of Norepinephrine) to innervate eccrine sweat glands.
C Fibers: Unmyelinated postganglionic autonomic fibers (both sympathetic and parasympathetic) that innervate visceral effectors, including sweat glands.
Muscarinic Receptors (M3): The receptors on eccrine sweat glands that bind Acetylcholine to stimulate sweating.
Anhidrosis: Absence of sweating, which can be a sign of autonomic failure or Horner's syndrome (sympathetic damage).
Sympathetic Chain: The location of the cell bodies of the postganglionic neurons supplying the sweat glands.
Sudosuomotor: A term referring to the autonomic control of sweat gland activity.
Adrenergic Sweating: Occurs in stress ("cold sweat") primarily on palms/soles and axilla (apocrine), mediated by Norepinephrine/Epinephrine acting on alpha/beta receptors.
Thermoregulatory Sweating: Controlled by the hypothalamus via sympathetic cholinergic pathways.
[Image of Sympathetic cholinergic innervation of sweat glands]
Lead Question - 2016
Sweat glands are supplied by all except?
a) Cholinergic neurons
b) Sympathetic neurons
c) Adrenergic neurons
d) C-fibers
Explanation: This question requires careful dissection of "Sweat Glands" generally vs. specific types. 1. Sympathetic Neurons: ALL sweat glands are innervated by the Sympathetic Nervous System. (Option b is true). 2. C-fibers: Postganglionic sympathetic fibers are unmyelinated C-fibers. (Option d is true). 3. Cholinergic Neurons: The vast majority of sweat glands (Eccrine, for thermoregulation) are innervated by Sympathetic Cholinergic fibers releasing Acetylcholine. (Option a is true). 4. Adrenergic Neurons: Apocrine glands (axilla/pubis) and some eccrine glands on palms/soles do receive adrenergic innervation (stress sweating). However, in general physiological context referring to body-wide thermoregulatory sweat glands, the exception or "least correct" standard answer is often Adrenergic, because the hallmark of human sweating is the unique *cholinergic* sympathetic output. Wait—technically Apocrine glands are Adrenergic. But the question asks "Sweat glands" (general). Standard teaching emphasizes the Cholinergic exception. Let's re-evaluate. Actually, eccrine glands are NOT supplied by Adrenergic neurons (they have receptors but no nerve endings). The innervation is Cholinergic. Thus, Adrenergic neurons do not supply the main body eccrine glands. This is the intended answer. Therefore, the correct answer is c) Adrenergic neurons.
1. The postganglionic sympathetic neurons innervating eccrine sweat glands are unique because they release:
a) Norepinephrine
b) Acetylcholine
c) Epinephrine
d) Dopamine
Explanation: The general rule of the Sympathetic Nervous System is: Preganglionic = Acetylcholine, Postganglionic = Norepinephrine. However, there is one major anatomical exception: the innervation of Eccrine Sweat Glands. These glands are innervated by sympathetic postganglionic fibers that release Acetylcholine (ACh). These are termed "Sympathetic Cholinergic" fibers. This allows sweating to be distinct from vasoconstriction during the fight-or-flight response, although they often happen together. Therefore, the correct answer is b) Acetylcholine.
2. Which type of receptor is primarily found on eccrine sweat glands to mediate thermoregulatory sweating?
a) Nicotinic (Nn)
b) Alpha-1 Adrenergic
c) Muscarinic (M3)
d) Beta-2 Adrenergic
Explanation: Since the postganglionic fiber releases Acetylcholine, the target receptor must be cholinergic. The receptors on the effector organs (sweat glands) are Muscarinic receptors, specifically the M3 subtype. Activation of M3 receptors leads to increased intracellular Calcium and secretion of sweat. This explains why antimuscarinic drugs like Atropine cause anhidrosis (dry skin) and hyperthermia as a side effect. Nicotinic receptors are at the ganglion. Therefore, the correct answer is c) Muscarinic (M3).
3. While thermoregulatory sweating is cholinergic, "emotional sweating" (palms and soles) can be stimulated by circulating catecholamines acting on:
a) Alpha adrenergic receptors
b) Muscarinic receptors only
c) Nicotinic receptors
d) Dopamine receptors
Explanation: Emotional stress causes "cold sweats," particularly on the palms, soles, and axillae. While these glands do have cholinergic innervation, they (and especially Apocrine glands) also express Alpha and Beta Adrenergic receptors. They respond to circulating Epinephrine and Norepinephrine released from the adrenal medulla during stress. This adrenergic component explains why beta-blockers can sometimes reduce "nervous sweating" (though less effectively than antiperspirants). Thermoregulatory sweating is strictly cholinergic. Therefore, the correct answer is a) Alpha adrenergic receptors.
4. Structurally, the postganglionic fibers supplying the sweat glands are classified as:
a) A-delta fibers
b) B fibers
c) C fibers
d) A-beta fibers
Explanation: The autonomic nervous system's efferent pathway consists of two neurons. The Preganglionic neuron is a myelinated B fiber. The Postganglionic neuron (from the sympathetic chain ganglion to the sweat gland) is an unmyelinated Type C fiber. These are slow-conducting fibers. This anatomical fact aligns with the general rule for all postganglionic autonomic fibers, regardless of whether they release NE or ACh. Therefore, the correct answer is c) C fibers.
5. A patient with organophosphate poisoning presents with profuse sweating (diaphoresis). This is due to the accumulation of Acetylcholine at the:
a) Sympathetic Ganglia only
b) Parasympathetic effector junctions
c) Neuroeffector junctions of eccrine sweat glands
d) Neuromuscular junction
Explanation: Organophosphates inhibit Acetylcholinesterase, leading to a massive buildup of ACh throughout the body. The symptom of Diaphoresis (sweating) is a classic Muscarinic sign ("DUMBELS" mnemonic). It occurs because ACh accumulates at the Neuroeffector junctions of sympathetic cholinergic fibers supplying the eccrine sweat glands. Although sweating is a sympathetic function, it is chemically cholinergic, so it is overstimulated in a "cholinergic crisis" just like parasympathetic functions (salivation, lacrimation). Therefore, the correct answer is c) Neuroeffector junctions of eccrine sweat glands.
6. In Cystic Fibrosis, the sweat chloride test is positive (high Cl-) because of a defect in:
a) Sweat secretion in the coil
b) Reabsorption of Cl- (and Na+) in the sweat duct
c) Cholinergic innervation
d) Aquaporin channels
Explanation: The sweat gland has two parts: the secretory coil and the reabsorptive duct. The precursor fluid secreted in the coil is isotonic. As it flows through the duct, ions (Na+ and Cl-) are reabsorbed to make the final sweat hypotonic. This reabsorption requires the CFTR (Cystic Fibrosis Transmembrane Conductance Regulator) chloride channel. In Cystic Fibrosis, the CFTR is defective. Reabsorption of Cl- (and consequently Na+) in the duct fails. The salt stays in the sweat, leading to salty skin and a positive sweat chloride test. Therefore, the correct answer is b) Reabsorption of Cl- (and Na+) in the sweat duct.
7. Which brain center is the primary integrator for thermoregulatory sweating?
a) Thalamus
b) Medulla Oblongata
c) Anterior Hypothalamus (Preoptic Area)
d) Posterior Hypothalamus
Explanation: The body's thermostat is located in the Hypothalamus. Specifically, the Anterior Hypothalamus (Preoptic Area) contains heat-sensitive neurons. When body temperature rises, these neurons fire to initiate heat-loss mechanisms, primarily Sweating and cutaneous vasodilation. The descending pathways from the anterior hypothalamus traverse the brainstem and cord to activate the sympathetic cholinergic neurons in the lateral horn (T1-L2). The posterior hypothalamus controls heat conservation (shivering). Therefore, the correct answer is c) Anterior Hypothalamus (Preoptic Area).
8. Apocrine sweat glands differ from eccrine glands in that Apocrine glands:
a) Open directly onto the skin surface
b) Are functional from birth
c) Are activated principally by Adrenergic mechanisms
d) Secrete a watery fluid for cooling
Explanation: Apocrine glands (axilla, areola, anogenital) open into hair follicles, not the skin surface. They become functional only at puberty. Their secretion is viscid and odoriferous (after bacterial action). Crucially, unlike eccrine glands, Apocrine glands are predominantly sensitive to circulating catecholamines (Adrenaline) and have Adrenergic innervation. They respond to emotional stress and sexual arousal rather than temperature. This is why "nervous sweat" smells different from "gym sweat." Therefore, the correct answer is c) Are activated principally by Adrenergic mechanisms.
9. Horner's Syndrome (Sympathetic denervation of the face) is characterized by Anhidrosis (lack of sweating) on the face. This confirms that facial sweat glands are innervated by:
a) Cranial Nerves (Parasympathetic)
b) Cervical Sympathetic Chain
c) Somatic motor nerves
d) Trigeminal Nerve directly
Explanation: Horner's syndrome results from a lesion in the sympathetic pathway (hypothalamus -> brainstem -> spinal cord -> Sympathetic Chain -> face). The classic triad is Ptosis, Miosis, and Anhidrosis (dry face). The presence of anhidrosis proves that the sweat glands of the face receive their motor supply from the Sympathetic Nervous System (specifically the Superior Cervical Ganglion). If they were parasympathetic (like salivary glands), they would be spared in a sympathetic lesion. Therefore, the correct answer is b) Cervical Sympathetic Chain.
10. Botulinum toxin injections are an effective treatment for Hyperhidrosis (excessive sweating). This works because the toxin:
a) Destroys the sweat gland
b) Blocks the release of Norepinephrine
c) Blocks the release of Acetylcholine from sympathetic nerves
d) Blocks the Muscarinic receptors
Explanation: Botulinum toxin (Botox) is a protease that cleaves SNARE proteins in nerve terminals, preventing vesicle fusion. Since eccrine sweat glands are innervated by cholinergic neurons, the toxin specifically Blocks the release of Acetylcholine from these sympathetic terminals. Without the neurotransmitter signal, the sweat glands cannot secrete fluid. The effect is localized and temporary (months) but highly effective for focal hyperhidrosis (e.g., axillary or palmar). Therefore, the correct answer is c) Blocks the release of Acetylcholine from sympathetic nerves.
Chapter: Neurophysiology; Topic: Special Senses: Vision; Subtopic: Parallel Processing in the Visual System
Key Definitions & Concepts
Parvocellular Pathway (P-pathway): The "What" pathway originating from midget ganglion cells in the retina; specialized for high spatial resolution, color vision, and fine detail.
Magnocellular Pathway (M-pathway): The "Where" pathway originating from parasol ganglion cells; specialized for high temporal resolution, motion detection, and depth perception.
Lateral Geniculate Nucleus (LGN): The thalamic relay center for vision. Layers 1-2 receive Magnocellular input; Layers 3-6 receive Parvocellular input.
Ventral Stream: The cortical continuation of the P-pathway into the temporal lobe, responsible for object recognition ("What is it?").
Dorsal Stream: The cortical continuation of the M-pathway into the parietal lobe, responsible for spatial awareness and motion ("Where is it?").
Cones: Photoreceptors concentrated in the fovea responsible for color and acuity; they provide the primary input to the P-pathway.
Spatial Summation: Low in the P-pathway (small receptive fields) allowing fine detail; High in the M-pathway (large receptive fields) increasing sensitivity to dim light/motion.
Temporal Resolution: The ability to detect rapid changes (flicker). The M-pathway has high temporal resolution; the P-pathway has low (sustained response).
Color Opponency: A feature of P-cells (Red-Green, Blue-Yellow) allowing for color discrimination; M-cells are color blind.
Koniocellular Pathway: A third, minor pathway (K-pathway) involved in blue-yellow color vision and possibly blindsight.
[Image of Visual pathways diagram P and M]
Lead Question - 2016
Parvocellular pathway for vision is concerned with?
a) Fine details of object
b) Movements of object
c) Flickering features
d) Depth of vision
Explanation: The visual system is divided into two parallel processing streams starting from the retina. The Parvocellular (P) pathway originates from small (midget) ganglion cells that receive input primarily from cones in the fovea. These cells have very small receptive fields and show a sustained response to stimuli. This physiology makes them ideally suited for analyzing the Fine details of objects (high visual acuity), form, texture, and color ("What" the object is). In contrast, the Magnocellular (M) pathway is specialized for detecting movement, flicker, and depth ("Where" the object is). Therefore, the correct answer is a) Fine details of object.
1. The Parvocellular layers of the Lateral Geniculate Nucleus (LGN) are layers:
a) 1 and 2
b) 3, 4, 5, and 6
c) 1, 4, and 6
d) 2, 3, and 5
Explanation: The Lateral Geniculate Nucleus (LGN) has a distinct laminar structure consisting of 6 principal layers. The two ventral layers (Layers 1 and 2) contain large cell bodies and are called the Magnocellular layers. The four dorsal layers (Layers 3, 4, 5, and 6) contain small cell bodies and are called the Parvocellular layers. These layers receive the axons from the P-ganglion cells of the retina. This anatomical segregation ensures that color/form information (P-pathway) remains separate from motion information (M-pathway) before reaching the cortex. Therefore, the correct answer is b) 3, 4, 5, and 6.
2. Which visual function is exclusively mediated by the Magnocellular pathway?
a) Color perception
b) High acuity form recognition
c) Motion perception
d) Face recognition
Explanation: The Magnocellular (M) pathway is "color blind" (achromatic) and has poor spatial resolution due to large receptive fields. However, it excels at detecting rapid changes in the visual scene. Its ganglion cells have transient (phasic) responses and high conduction velocities. This makes the M-pathway the exclusive system for Motion Perception and the detection of low-contrast, rapidly changing stimuli (flicker). Damage to the dorsal stream (M-pathway continuation) leads to Akinetopsia (motion blindness). Color and form are P-pathway functions. Therefore, the correct answer is c) Motion perception.
3. The "Ventral Stream" of visual processing projects to the Temporal Lobe and is primarily an extension of the:
a) Magnocellular pathway
b) Parvocellular pathway
c) Koniocellular pathway
d) Retinotectal pathway
Explanation: After V1 (Primary Visual Cortex), visual information splits. The Ventral Stream ("What" pathway) projects ventrally to the Inferior Temporal Cortex. It is involved in object recognition, face recognition, and color processing. This stream receives its dominant input from the Parvocellular pathway (P-blobs and P-interblobs in V1). Conversely, the Dorsal Stream ("Where" pathway) projects to the Parietal lobe and is dominated by the Magnocellular input for spatial awareness and motion guidance. Therefore, the correct answer is b) Parvocellular pathway.
4. Compared to Magnocellular cells, Parvocellular neurons have:
a) Larger receptive fields and faster conduction
b) Larger receptive fields and slower conduction
c) Smaller receptive fields and slower conduction
d) Smaller receptive fields and faster conduction
Explanation: Structure dictates function. P-cells are small ("Parvo"). They receive input from very few photoreceptors (sometimes 1:1 in the fovea). This results in Smaller receptive fields, which provides high spatial resolution (acuity). Their axons are thinner, leading to Slower conduction velocities compared to the thick M-cell axons. This slower speed is an acceptable trade-off for the high-detail analysis required for reading or recognizing faces, where speed is less critical than precision. M-cells are large, fast, and have large fields. Therefore, the correct answer is c) Smaller receptive fields and slower conduction.
5. A specific deficit in detecting "Flicker" at high frequencies (Critical Fusion Frequency) would suggest damage to the:
a) Parvocellular system
b) Magnocellular system
c) Rods only
d) S-cones
Explanation: The ability to resolve stimuli that change rapidly in time (Temporal Resolution) is a property of the Magnocellular system. M-cells respond transiently to stimulus onset and offset. They can follow rapid flickering lights up to high frequencies. If the M-pathway is damaged (e.g., in early Glaucoma or Dyslexia theories), the ability to detect high-frequency flicker is impaired. P-cells are "sustained" responders; they are sluggish and blur rapid flicker, making them poor at temporal resolution. Therefore, the correct answer is b) Magnocellular system.
6. Color vision defects (specifically Red-Green) are most likely to arise from dysfunction in the:
a) Magnocellular layers of LGN
b) Parvocellular layers of LGN
c) Superior Colliculus
d) Pulvinar nucleus
Explanation: Color vision relies on the comparison of signals from different cone types (L, M, S cones). This process, known as Color Opponency (e.g., Red vs. Green), is computed by the P-ganglion cells and transmitted via the Parvocellular layers of the LGN. The M-pathway sums inputs from all cones (L+M) to detect luminance (brightness) but discards the spectral difference (color) information. Therefore, the P-pathway is the exclusive carrier of Red-Green color data. (Blue-Yellow involves the Koniocellular path). Therefore, the correct answer is b) Parvocellular layers of LGN.
7. The "Blobs" (Cytochrome Oxidase Blobs) found in the Primary Visual Cortex (V1) are specially dedicated to processing:
a) Motion
b) Depth (Stereopsis)
c) Color
d) Orientation
Explanation: When V1 is stained for the metabolic enzyme Cytochrome Oxidase, a pattern of dark spots ("Blobs") and pale intervening areas ("Interblobs") appears in layers 2/3. The P-pathway projects to both. The Blobs are specifically rich in cells that are color-sensitive but not orientation-selective. They process Color information. The Interblobs contain cells that are orientation-selective (for form/edges) but not color-sensitive. M-pathway input goes to layer 4B and is distinct from the blob system. Therefore, the correct answer is c) Color.
8. Contrast sensitivity for stationary, high-spatial-frequency gratings (fine patterns) is a function of the:
a) M-pathway
b) P-pathway
c) Scotopic system
d) Vestibulo-ocular reflex
Explanation: Visual stimuli can be described by contrast and spatial frequency (detail). M-pathway: High contrast sensitivity (sees faint objects) but only for low spatial frequencies (coarse patterns). P-pathway: Low contrast sensitivity (needs distinct boundaries) but excellent for High spatial frequencies (fine detail/patterns). Therefore, perceiving a stationary, finely detailed pattern (high spatial frequency) is the domain of the P-pathway. Therefore, the correct answer is b) P-pathway.
9. Stereopsis (Depth Perception) relies heavily on disparity cues processed by the:
a) Parvocellular pathway mainly
b) Magnocellular pathway mainly
c) Koniocellular pathway
d) Auditory pathway
Explanation: Depth perception involves analyzing the small differences (disparities) between the images on the two retinas. While both pathways contribute, the Magnocellular pathway is generally considered the dominant system for stereopsis, particularly for coarse depth and depth-from-motion. M-cells project to cortical area V5/MT and the thick stripes of V2, which are rich in disparity-tuned neurons. The P-pathway contributes to fine depth, but the "Where" (Dorsal stream/Magnocellular) system is the primary locator of objects in 3D space. Therefore, the correct answer is b) Magnocellular pathway mainly.
10. Which neurotransmitter is used by the Photoreceptors (Rods and Cones) to signal the Bipolar cells at the beginning of these pathways?
a) GABA
b) Glycine
c) Glutamate
d) Dopamine
Explanation: In the dark, photoreceptors are depolarized and continuously release their neurotransmitter. This neurotransmitter is Glutamate. When light strikes the receptor, it hyperpolarizes, and Glutamate release decreases. This drop in glutamate has different effects on downstream cells: it hyperpolarizes OFF-bipolar cells (which have ionotropic glutamate receptors) and depolarizes ON-bipolar cells (which have metabotropic receptors that invert the signal). This divergence creates the ON and OFF channels that feed into the M and P pathways. Therefore, the correct answer is c) Glutamate.
Chapter: Neuroanatomy; Topic: Cerebrum (Telencephalon); Subtopic: Functional Areas of the Cerebral Cortex
Key Definitions & Concepts
Visual Cortex (V1): Also known as the Striate Cortex or Brodmann Area 17; it is the primary cortical region responsible for processing visual information.
Calcarine Sulcus: A deep sulcus on the medial surface of the occipital lobe; the primary visual cortex is located on its upper and lower banks (lips).
Cuneus: The wedge-shaped gyrus superior to the Calcarine sulcus; represents the lower quadrant of the contralateral visual field.
Lingual Gyrus: The tongue-shaped gyrus inferior to the Calcarine sulcus; represents the upper quadrant of the contralateral visual field.
Stria of Gennari: A prominent band of myelinated fibers (Line of Gennari) visible to the naked eye in the visual cortex, giving it the name "Striate Cortex."
Macular Sparing: Preservation of central vision in PCA infarcts due to dual blood supply (PCA and MCA) to the macular cortex at the occipital pole.
Precentral Gyrus: The primary motor cortex (Area 4).
Postcentral Gyrus: The primary somatosensory cortex (Areas 3, 1, 2).
Sylvian Fissure: The lateral sulcus separating the temporal lobe from the frontal/parietal lobes; contains the auditory cortex.
Retinotopic Organization: The spatial arrangement of the retina is mapped onto the visual cortex; central vision (macula) is posterior, peripheral vision is anterior.
Lead Question - 2016
Location of visual cortex?
a) Precentral gyrus
b) Postcentral gyrus
c) Sylvian fissure
d) Calcarine sulcus
Explanation: The Primary Visual Cortex (Brodmann Area 17) is located in the occipital lobe. Anatomically, it is situated on the medial surface of the hemisphere, specifically along the banks (lips) of the Calcarine Sulcus. The cortex extends superiorly into the Cuneus and inferiorly into the Lingual gyrus. The Precentral gyrus is the motor cortex. The Postcentral gyrus is the sensory cortex. The Sylvian fissure contains the auditory cortex (Heschl's gyri). Therefore, the correct answer is d) Calcarine sulcus.
1. The Primary Visual Cortex is often referred to as the "Striate Cortex" due to the presence of a distinct white line visible to the naked eye. This line is known as the:
a) Stria Terminalis
b) Stria of Gennari
c) Stria Medullaris
d) Line of Baillarger
Explanation: The cerebral cortex typically has two horizontal bands of myelinated fibers called the Lines of Baillarger (inner and outer). In the Primary Visual Cortex (Area 17), the Outer Line of Baillarger is massively thickened by the dense input of geniculocalcarine fibers (optic radiations). This thickened band is visible macroscopically in fresh brain sections as a white stripe within the gray matter. This specific structure is called the Stria of Gennari (or Line of Gennari). Because of this stripe, Area 17 is uniquely named the Striate Cortex. Therefore, the correct answer is b) Stria of Gennari.
2. Which Brodmann area corresponds to the Primary Visual Cortex?
a) Area 17
b) Area 18
c) Area 19
d) Area 41
Explanation: Cortical mapping assigns numbers to functional areas. Area 17: Primary Visual Cortex (V1), located along the calcarine sulcus. Area 18: Secondary Visual Cortex (V2), surrounding Area 17 (Parastriate). Area 19: Visual Association Cortex (V3, V4, V5), surrounding Area 18 (Peristriate). Area 41: Primary Auditory Cortex. Knowing these numbers is fundamental for neuroanatomy exams. Therefore, the correct answer is a) Area 17.
3. A patient presents with a visual field defect described as "Right Upper Quadrantanopia." This implies damage to the visual pathway in the:
a) Right Temporal Lobe (Meyer's Loop)
b) Left Temporal Lobe (Meyer's Loop)
c) Left Parietal Lobe
d) Left Lingual Gyrus
Explanation: Visual field defects are contralateral and inverted. Right field defect -> Left brain lesion. Upper quadrant defect -> Lower fibers damaged. The lower fibers of the optic radiation (representing the upper visual field) loop anteriorly into the Temporal Lobe around the lateral ventricle. This pathway is called Meyer's Loop. Therefore, a lesion in the Left Temporal Lobe damages Meyer's loop, causing a "Pie in the Sky" defect: Right Upper Quadrantanopia. Parietal lesions cause lower quadrantanopia ("Pie on the Floor"). Lingual gyrus lesions (V1) would also cause upper quadrantanopia but are occipital, not the loop itself. Therefore, the correct answer is b) Left Temporal Lobe (Meyer's Loop).
4. The Macula (central vision) is represented in which specific part of the Calcarine cortex?
a) The most anterior part
b) The most posterior part (Occipital pole)
c) The superior bank only
d) The inferior bank only
Explanation: The visual cortex has a retinotopic organization. The peripheral visual field is mapped to the anterior part of the calcarine fissure. The central visual field (Macula), which has the highest acuity and receptor density, is mapped to the Most Posterior part of the cortex, specifically at the Occipital Pole. Furthermore, due to the high density of information from the fovea, the macular representation is disproportionately large (Cortical Magnification), occupying nearly 1/3 to 1/2 of the entire visual cortex. Therefore, the correct answer is b) The most posterior part (Occipital pole).
5. Occlusion of the Posterior Cerebral Artery (PCA) typically causes Contralateral Homonymous Hemianopsia with Macular Sparing. The sparing of the macula occurs because the occipital pole receives collateral blood supply from the:
a) Anterior Cerebral Artery
b) Middle Cerebral Artery
c) Anterior Choroidal Artery
d) Posterior Communicating Artery
Explanation: The main blood supply to the visual cortex (occipital lobe) is the Posterior Cerebral Artery (PCA). Occlusion causes blindness in the contralateral field. However, the macular representation at the occipital pole is unique. It lies at the watershed zone between the PCA and the Middle Cerebral Artery (MCA). The MCA branches wrap around from the lateral surface to supply the pole. Thus, if the PCA is blocked, the MCA collateral supply keeps the macular cortex alive, preserving central vision ("Macular Sparing"). Therefore, the correct answer is b) Middle Cerebral Artery.
6. The Lingual Gyrus forms the inferior bank of the Calcarine sulcus. A lesion restricted to the Left Lingual Gyrus would result in:
a) Right Lower Quadrantanopia
b) Left Upper Quadrantanopia
c) Right Upper Quadrantanopia
d) Right Homonymous Hemianopsia
Explanation: Anatomy: The Calcarine sulcus divides the visual cortex. Cuneus (Superior bank): Receives information from the Superior Retina (Lower Visual Field). Lingual Gyrus (Inferior bank): Receives information from the Inferior Retina (Upper Visual Field). Lesion: Left Lingual Gyrus -> Damage to Left Inferior Retina representation -> Loss of Right Upper Visual Field. Therefore, the defect is a Right Superior (Upper) Quadrantanopia. Therefore, the correct answer is c) Right Upper Quadrantanopia.
7. Prosopagnosia (inability to recognize faces) is classically associated with bilateral lesions of the:
a) Primary Visual Cortex (V1)
b) Fusiform Gyrus (Occipitotemporal gyrus)
c) Angular Gyrus
d) Dorsolateral Prefrontal Cortex
Explanation: Visual processing splits into two streams. The Dorsal Stream ("Where") goes to the parietal lobe. The Ventral Stream ("What") goes to the temporal lobe for object recognition. A key specialized area in the ventral stream is the "Fusiform Face Area," located in the Fusiform Gyrus (medial occipitotemporal gyrus) on the inferior surface of the brain. Bilateral (or sometimes right-sided) damage to this area disconnects visual perception from memory/identity centers, leading to Prosopagnosia (face blindness). The patient can see parts of a face but cannot integrate them to recognize the person. Therefore, the correct answer is b) Fusiform Gyrus (Occipitotemporal gyrus).
8. The Calcarine Sulcus joins the Parieto-occipital Sulcus anteriorly to form a "Y" shape. The wedge-shaped area of cortex between these two sulci is called the:
a) Precuneus
b) Cuneus
c) Lingual Gyrus
d) Paracentral Lobule
Explanation: On the medial surface of the hemisphere: The Cuneus ("Wedge") is bounded inferiorly by the Calcarine Sulcus and anteriorly by the Parieto-occipital Sulcus. It contains the visual cortex for the lower quadrant of the visual field. The Precuneus is anterior to the Parieto-occipital sulcus (parietal lobe). The Lingual gyrus is inferior to the Calcarine sulcus. This anatomical "Y" is a critical landmark on MRI. Therefore, the correct answer is b) Cuneus.
9. Visual hallucinations (simple flashes of light or colors) are most likely to occur with irritative lesions (e.g., aura of migraine or epilepsy) arising from:
a) Area 19 (Visual Association Area)
b) Area 17 (Primary Visual Cortex)
c) Temporal Lobe
d) Frontal Eye Fields
Explanation: The complexity of a hallucination depends on the cortical level involved. Irritation of the Primary Visual Cortex (Area 17) produces unformed, simple hallucinations (photopsias) like flashes of light, stars, or lines in the contralateral visual field. Irritation of the Visual Association Cortex (Area 18/19) or Temporal lobe produces formed, complex hallucinations (objects, faces, scenes). Therefore, simple flashes point to V1 pathology. Therefore, the correct answer is b) Area 17 (Primary Visual Cortex).
10. Which subcortical nucleus projects directly to the Primary Visual Cortex via the Geniculocalcarine tract?
a) Superior Colliculus
b) Medial Geniculate Body
c) Lateral Geniculate Body
d) Pulvinar
Explanation: The visual pathway consists of: Retina -> Optic Nerve -> Chiasm -> Tract -> Lateral Geniculate Body (LGB) of the Thalamus. From the LGB, the neurons project their axons as the Optic Radiations (Geniculocalcarine tract) to the Primary Visual Cortex. The LGB is the thalamic relay for vision (L for Light). The Medial Geniculate is for hearing. The Superior Colliculus is for eye movements/reflexes but does not project directly to the primary visual cortex for conscious vision. Therefore, the correct answer is c) Lateral Geniculate Body.
Chapter: Neuroanatomy; Topic: Ventricular System and CSF; Subtopic: Distribution and Circulation of Cerebrospinal Fluid
Key Definitions & Concepts
Cerebrospinal Fluid (CSF): A clear, colorless body fluid found in the brain and spinal cord, acting as a cushion (shock absorber) and a medium for nutrient/waste exchange.
Subarachnoid Space: The anatomical space located between the arachnoid mater and the pia mater; it contains the major cerebral blood vessels and CSF.
Ventricular System: A set of four interconnected cavities (ventricles) in the brain where CSF is produced (Lateral, Third, and Fourth ventricles).
Central Canal: The cerebrospinal fluid-filled space that runs longitudinally through the entire length of the spinal cord; continuous with the ventricular system.
Choroid Plexus: A plexus of cells that produces the cerebrospinal fluid in the ventricles of the brain.
Arachnoid Granulations: Projections of the arachnoid membrane into the dural sinuses that allow CSF to exit the subarachnoid space and enter the blood stream.
Cisterns: Expanded areas of the subarachnoid space where the arachnoid spans across irregularities in the brain surface (e.g., Cisterna Magna, Lumbar Cistern).
Lumbar Cistern: The enlargement of the subarachnoid space in the lower spinal canal (L1-S2) containing the Cauda Equina; the site for lumbar puncture.
Virchow-Robin Spaces: Perivascular spaces that allow CSF to penetrate deep into the brain tissue along penetrating blood vessels (glymphatic function).
Foramina of Luschka and Magendie: The exit points in the fourth ventricle allowing CSF to flow into the subarachnoid space.
[Image of Cerebrospinal fluid circulation pathway]
Lead Question - 2016
CSF is present in which space?
a) Central canal of spinal cord
b) Ventricles of brain
c) Subarachnoid space
d) All of the above
Explanation: Cerebrospinal fluid (CSF) is a dynamic fluid that circulates through a continuous system. It is primarily produced by the choroid plexus within the Ventricles of the brain (Lateral, Third, and Fourth). From the Fourth Ventricle, it has two pathways: it can continue caudally into the Central Canal of the spinal cord, or it can exit via the median and lateral apertures to enter the Subarachnoid space. The subarachnoid space surrounds the entire brain and spinal cord. Therefore, under normal physiological conditions, CSF fills the ventricles, the central canal, and the subarachnoid space. Consequently, it is present in all the listed locations. Therefore, the correct answer is d) All of the above.
1. The subarachnoid space, which contains CSF, is located between which two meningeal layers?
a) Dura mater and Arachnoid mater
b) Arachnoid mater and Pia mater
c) Pia mater and Brain surface
d) Dura mater and Periosteum
Explanation: The meninges consist of three layers: Dura, Arachnoid, and Pia. The space between the Dura and Arachnoid (Subdural) is a potential space. The space between the Arachnoid mater and Pia mater is a real anatomical space known as the Subarachnoid Space. This space is filled with Cerebrospinal Fluid (CSF) and contains the trabeculae that connect the two layers, as well as the major arteries and veins of the brain. The Pia mater is firmly adherent to the brain surface. Hemorrhage into this space (Subarachnoid Hemorrhage) mixes blood with CSF. Therefore, the correct answer is b) Arachnoid mater and Pia mater.
2. A newborn is diagnosed with Non-Communicating Hydrocephalus. MRI reveals a blockage preventing CSF flow from the Third Ventricle to the Fourth Ventricle. The obstruction is located at the:
a) Foramen of Monro
b) Cerebral Aqueduct (of Sylvius)
c) Foramen of Magendie
d) Arachnoid Granulations
Explanation: The ventricular system is connected by specific channels. The Lateral Ventricles connect to the Third Ventricle via the Foramen of Monro. The Third Ventricle connects to the Fourth Ventricle via the Cerebral Aqueduct (of Sylvius), which passes through the midbrain. The Cerebral Aqueduct is the narrowest part of the ventricular system and is the most common site for congenital obstruction (Aqueductal Stenosis). Blockage here causes dilatation of the lateral and third ventricles while the fourth ventricle remains small, a condition known as Non-Communicating (Obstructive) Hydrocephalus. Therefore, the correct answer is b) Cerebral Aqueduct (of Sylvius).
3. Which specialized ependymal cells are primarily responsible for the secretion of Cerebrospinal Fluid?
a) Tanycytes
b) Choroid Plexus epithelium
c) Astrocytes
d) Microglia
Explanation: While the ventricles are lined by ependymal cells, the specific production of CSF is the function of the Choroid Plexus epithelium. The choroid plexus is a vascular structure covered by a specialized layer of cuboidal ependymal cells found in the lateral, third, and fourth ventricles. These cells actively transport ions (Na+, Cl-, HCO3-) from the plasma into the ventricles, creating an osmotic gradient that draws water across, thereby secreting CSF. This is an active metabolic process, distinct from passive filtration. Tanycytes are specialized ependymal cells in the third ventricle floor involved in transport. Therefore, the correct answer is b) Choroid Plexus epithelium.
4. A patient suspected of meningitis undergoes a lumbar puncture. To safely obtain CSF without damaging the spinal cord, the needle is inserted into the Lumbar Cistern at the level of:
a) T12-L1
b) L1-L2
c) L3-L4 or L4-L5
d) S1-S2
Explanation: In adults, the spinal cord typically terminates at the level of the L1 or L2 vertebra (Conus Medullaris). The subarachnoid space (Lumbar Cistern), containing CSF and the nerve roots of the Cauda Equina, continues down to S2. To avoid piercing the spinal cord proper, a lumbar puncture (spinal tap) is performed below the level of L2. The standard safe interspaces are L3-L4 or L4-L5. The nerve roots here float in the CSF and are pushed aside by the needle rather than being pierced. Therefore, the correct answer is c) L3-L4 or L4-L5.
5. The total volume of CSF in an adult is approximately 150 ml. The entire volume is replaced roughly how many times per day?
a) Once
b) 3 to 4 times
c) 10 times
d) It is not replaced
Explanation: The total volume of CSF in the ventricles and subarachnoid space is about 150 ml. However, the rate of CSF production is approximately 0.35 ml/min or roughly 500 ml/day. This means that the entire volume of CSF is produced, circulated, and reabsorbed approximately 3 to 4 times every 24 hours. This high turnover rate is crucial for flushing out metabolic waste products from the brain (the "sink" function of CSF) and maintaining a stable ionic environment for neuronal activity. Therefore, the correct answer is b) 3 to 4 times.
6. Papilledema is a clinical sign of raised Intracranial Pressure (ICP). This finding is possible because the subarachnoid space extends anteriorly to surround which cranial nerve?
a) Oculomotor Nerve (CN III)
b) Trigeminal Nerve (CN V)
c) Optic Nerve (CN II)
d) Facial Nerve (CN VII)
Explanation: The Optic Nerve (CN II) is unique among cranial nerves; embryologically, it is an outgrowth of the diencephalon (brain tract). Consequently, it is ensheathed by all three meningeal layers (dura, arachnoid, and pia). The subarachnoid space, containing CSF, extends along the optic nerve all the way to the back of the eyeball. When ICP rises, the pressure is transmitted through this CSF column to the optic nerve head, compressing the central retinal vein and axoplasmic flow. This results in swelling of the optic disc, visible on fundoscopy as Papilledema. Therefore, the correct answer is c) Optic Nerve (CN II).
7. Cerebrospinal Fluid is reabsorbed into the venous circulation primarily through which structure?
a) Choroid Plexus
b) Arachnoid Villi/Granulations
c) Foramen of Magendie
d) Cavernous Sinus walls
Explanation: While the choroid plexus produces CSF, reabsorption occurs at a different site to maintain volume balance. The primary sites of reabsorption are the Arachnoid Villi (or Granulations). These are herniations of the arachnoid membrane that protrude into the dural venous sinuses, most notably the Superior Sagittal Sinus. They function as one-way valves. When CSF pressure exceeds venous pressure, the valves open, allowing bulk flow of CSF into the venous blood. If these granulations are blocked (e.g., by blood or infection), Communicating Hydrocephalus develops. Therefore, the correct answer is b) Arachnoid Villi/Granulations.
8. A patient presents with the classic triad of gait disturbance ("magnetic gait"), urinary incontinence, and dementia. Imaging shows enlarged ventricles. This condition, Normal Pressure Hydrocephalus (NPH), involves accumulation of CSF in the:
a) Epidural space
b) Ventricles and Subarachnoid space
c) Brain parenchyma only
d) Subdural space
Explanation: Normal Pressure Hydrocephalus (NPH) is a form of communicating hydrocephalus common in the elderly. It is caused by impaired CSF absorption at the arachnoid granulations. CSF accumulates within the Ventricles, causing them to dilate (Ventriculomegaly). This expansion stretches the periventricular white matter tracts (corona radiata), leading to the triad of symptoms. Importantly, the pressure measured by lumbar puncture is often normal or high-normal (hence the name), but the volume is increased in the ventricular and communicating subarachnoid spaces. It is treated by shunting. Therefore, the correct answer is b) Ventricles and Subarachnoid space.
9. The Foramen of Magendie is a midline aperture that allows CSF to flow from the Fourth Ventricle into the:
a) Third Ventricle
b) Cisterna Magna
c) Interpeduncular Cistern
d) Central Canal only
Explanation: The CSF exits the ventricular system to enter the subarachnoid space through three openings in the Fourth Ventricle. There are two lateral apertures (Foramina of Luschka) and one median aperture known as the Foramen of Magendie. The Foramen of Magendie opens directly into the Cisterna Magna (Cerebellomedullary cistern), which is the largest subarachnoid cistern located between the cerebellum and the medulla. From there, CSF circulates around the brain and spinal cord. Blockage of these exit foramina leads to non-communicating hydrocephalus. Therefore, the correct answer is b) Cisterna Magna.
10. Xanthochromia (yellow discoloration) of the CSF is a diagnostic feature of:
a) Viral Meningitis
b) Multiple Sclerosis
c) Subarachnoid Hemorrhage
d) Guillain-Barre Syndrome
Explanation: Ideally, CSF is clear and colorless like water. Xanthochromia refers to the yellow or pink discoloration of the CSF supernatant after centrifugation. It is caused by the presence of bilirubin (from the breakdown of hemoglobin). This finding confirms that blood has been present in the CSF for several hours (typically >12 hours), indicating a Subarachnoid Hemorrhage (SAH). It helps distinguish a true hemorrhage from a "traumatic tap" (where fresh blood is introduced by the needle and the supernatant remains clear). Therefore, the correct answer is c) Subarachnoid Hemorrhage.
Chapter: General Physiology; Topic: Body Fluids and CSF; Subtopic: Factors Affecting Cerebrospinal Fluid Pressure
Key Definitions & Concepts
Cerebrospinal Fluid (CSF) Pressure: The pressure exerted by the CSF within the subarachnoid space, typically 60-150 mmH2O (in recumbent position) in adults.
Intracranial Pressure (ICP): The total pressure inside the skull, determined by brain tissue, blood volume, and CSF volume (Monro-Kellie Doctrine).
Valsalva Maneuver: Forced expiration against a closed glottis; this increases intrathoracic pressure, impeding venous return from the brain, thus raising ICP.
Queckenstedt's Sign: Compression of the jugular veins normally causes a rapid rise in CSF pressure; absence of this rise indicates a spinal block.
Intrathoracic Pressure: Pressure within the chest cavity. An increase (e.g., coughing, straining) is transmitted to the jugular veins, increasing intracranial venous pressure and CSF pressure.
Venous Return: The flow of blood back to the heart. Obstruction of venous return from the head (due to high chest pressure) increases cerebral blood volume and CSF pressure.
Forced Inspiration: A maneuver that decreases intrathoracic pressure (makes it more negative), which *facilitates* venous drainage from the head, potentially lowering CSF pressure slightly or transiently.
Crying/Coughing: Both actions involve forced expiration and increased intra-abdominal/intrathoracic pressure, leading to a spike in CSF pressure.
Monro-Kellie Hypothesis: States that the sum of volumes of brain, CSF, and blood is constant; an increase in one (e.g., venous blood) must raise pressure or displace the others.
Hydrocephalus: A condition of chronically raised CSF pressure due to obstruction or absorption failure, distinct from transient physiological spikes.
[Image of CSF flow dynamics]
Lead Question - 2016
CSF pressure is increased in all except -
a) Forced inspiration
b) Coughing
c) Valsalva manoeuvre
d) Crying
Explanation: CSF pressure is directly influenced by cerebral venous pressure. Any maneuver that increases intrathoracic or intra-abdominal pressure impedes venous return from the brain (via the jugular veins), causing congestion and raising CSF pressure. Coughing, Crying, and the Valsalva maneuver (straining) all involve forced expiration against resistance, leading to a sharp rise in intrathoracic pressure and a consequent rise in CSF pressure. In contrast, Forced Inspiration (like the Muller maneuver) makes the intrathoracic pressure more negative. This enhances venous return from the head to the heart, which tends to reduce cerebral venous pressure and thus would not increase CSF pressure (it may transiently lower it). Therefore, the correct answer is a) Forced inspiration.
1. The normal range of Cerebrospinal Fluid (CSF) pressure in a healthy adult lying in the lateral recumbent position is approximately:
a) 5-15 mm H2O
b) 60-150 mm H2O
c) 200-300 mm H2O
d) 10-20 mm Hg
Explanation: CSF pressure measurements vary by position and units. In the Lateral Recumbent position (lying on the side), the hydrostatic column of the spine is eliminated, and the pressure at the lumbar cistern equals the intracranial pressure. The normal range is 60 to 150 mm H2O (or roughly 5-15 mm Hg, using the conversion 1 mmHg ≈ 13.6 mm H2O). Pressures above 200 mm H2O are considered elevated (intracranial hypertension). Sitting up increases lumbar pressure significantly due to gravity. Therefore, the correct answer is b) 60-150 mm H2O.
2. Which physiological mechanism explains why compressing the jugular veins leads to an immediate rise in CSF pressure (Queckenstedt's test)?
a) Increased arterial blood flow to the brain
b) Decreased absorption of CSF into the venous sinuses
c) Back-pressure transmission from blocked venous outflow
d) Reflex sympathetic stimulation
Explanation: The cranial cavity is a rigid box. Venous blood leaves the brain primarily through the internal jugular veins. Compressing these veins obstructs the outflow. This causes immediate congestion (engorgement) of the intracranial venous sinuses and veins. According to the Monro-Kellie doctrine, this increase in intracranial blood volume occupies space and transmits pressure directly to the CSF compartment, raising CSF pressure. This back-pressure transmission is the basis of Queckenstedt's test; lack of a pressure rise suggests a blockage in the spinal canal (e.g., tumor). Therefore, the correct answer is c) Back-pressure transmission from blocked venous outflow.
3. Hyperventilation is clinically used to acutely lower Intracranial Pressure (ICP). This works because low PaCO2 causes:
a) Systemic hypotension
b) Cerebral vasoconstriction
c) Increased CSF absorption
d) Decreased CSF production
Explanation: Carbon dioxide (CO2) is a potent vasodilator of cerebral blood vessels. Hyperventilation "blows off" CO2, leading to hypocapnia (low PaCO2). This causes Cerebral Vasoconstriction. By constricting the cerebral arterioles, the total volume of blood within the cranium is reduced (cerebral blood volume decreases). According to the Monro-Kellie doctrine, reducing blood volume lowers the overall Intracranial Pressure (ICP). This is a rapid but temporary measure used in emergency neurosurgery or trauma. Therefore, the correct answer is b) Cerebral vasoconstriction.
4. Administration of which osmotic agent is a standard treatment to rapidly reduce CSF pressure and cerebral edema?
a) Glucose
b) Mannitol
c) Albumin
d) Normal Saline
Explanation: To pull fluid out of the brain tissue and CSF compartment, a substance is needed that remains in the plasma and creates a high osmotic gradient across the Blood-Brain Barrier (BBB). Mannitol is the agent of choice. It is an inert sugar alcohol that does not cross the intact BBB. When given intravenously, it increases plasma osmolarity, drawing water from the brain interstitium and CSF into the blood (osmotic dehydration), thereby effectively lowering ICP. Glucose enters cells and is metabolized, losing its osmotic effect. Therefore, the correct answer is b) Mannitol.
5. Cushing's Reflex (Triad) is a physiological response to dangerously high CSF pressure (ICP). It consists of:
a) Hypotension, Tachycardia, Tachypnea
b) Hypertension, Bradycardia, Irregular Respiration
c) Hypertension, Tachycardia, Hyperthermia
d) Hypotension, Bradycardia, Apnea
Explanation: When ICP rises to a level that compromises cerebral blood flow (brain ischemia), the brainstem initiates a sympathetic surge to restore perfusion. This causes systemic Hypertension (to force blood into the head). The baroreceptors detect this high pressure and reflexively cause Bradycardia (via the vagus nerve). The compression of the brainstem respiratory centers causes Irregular Respiration. This triad (Hypertension, Bradycardia, Irregular breathing) is a late, ominous sign of brainstem herniation. Therefore, the correct answer is b) Hypertension, Bradycardia, Irregular Respiration.
6. Which of the following conditions leads to Communicating Hydrocephalus (raised CSF pressure with patent ventricular pathways)?
a) Aqueductal Stenosis
b) Tumor obstructing Foramen of Monro
c) Impaired absorption by Arachnoid Granulations
d) Dandy-Walker Malformation
Explanation: Hydrocephalus is raised CSF volume/pressure. It is "Non-communicating" (Obstructive) if the blockage is within the ventricular system (e.g., Aqueductal stenosis, tumor at Monro). It is "Communicating" if the CSF can flow out of the ventricles but accumulates due to failure of absorption or overproduction. The classic cause of communicating hydrocephalus is Impaired absorption by Arachnoid Granulations (e.g., post-meningitis fibrosis or subarachnoid hemorrhage clogging the villi). The CSF communicates with the subarachnoid space but cannot exit to the venous blood. Therefore, the correct answer is c) Impaired absorption by Arachnoid Granulations.
7. The rate of CSF formation is largely independent of ICP until pressure becomes extremely high. However, the rate of CSF absorption is:
a) Constant and independent of pressure
b) Inversely proportional to CSF pressure
c) Linearly dependent on the pressure gradient between CSF and venous blood
d) Regulated by active transport pumps only
Explanation: CSF production is an active secretory process (constant rate). Absorption, however, is a passive mechanical process functioning like a one-way valve at the arachnoid villi. CSF flows into the venous sinuses when CSF pressure exceeds venous pressure. The rate of absorption is Linearly dependent on the pressure gradient. As CSF pressure rises, the gradient increases, forcing more CSF through the villi into the blood. This acts as a compensatory safety valve to limit ICP rises, up to a point where the villi collapse or are overwhelmed. Therefore, the correct answer is c) Linearly dependent on the pressure gradient between CSF and venous blood.
8. Papilledema (swelling of the optic disc) is a clinical sign of raised intracranial pressure. This occurs because:
a) The optic nerve is compressed by the tumor directly
b) The subarachnoid space extends along the optic nerve to the back of the eye
c) Retinal veins are blocked by thrombosis
d) High ICP causes arterial hypertension
Explanation: The Optic Nerve (CN II) is structurally an outgrowth of the diencephalon. As such, it is ensheathed by all three meningeal layers (dura, arachnoid, pia). The Subarachnoid space containing CSF extends along the optic nerve all the way to the back of the eyeball. When intracranial CSF pressure rises, this pressure is transmitted directly through this sheath, compressing the optic nerve and, crucially, obstructing axoplasmic flow and venous drainage from the retina. This results in the swelling of the nerve head visible as Papilledema. Therefore, the correct answer is b) The subarachnoid space extends along the optic nerve to the back of the eye.
9. Acetazolamide is a drug used to lower CSF pressure (e.g., in Idiopathic Intracranial Hypertension). It works by:
a) Osmotic diuresis
b) Inhibiting Carbonic Anhydrase in the choroid plexus
c) Constricting cerebral vessels
d) Increasing lymphatic drainage
Explanation: The production of CSF by the choroid plexus involves the active transport of ions (Na+, HCO3-). The enzyme Carbonic Anhydrase is essential for generating the bicarbonate and protons needed for these transport mechanisms. Acetazolamide is a Carbonic Anhydrase Inhibitor. By inhibiting this enzyme in the choroid plexus epithelium, it significantly reduces the rate of CSF production (by up to 50%). Decreased production helps lower the overall intracranial pressure in conditions where absorption is impaired or pressure is idiopathic. Therefore, the correct answer is b) Inhibiting Carbonic Anhydrase in the choroid plexus.
10. A lumbar puncture is generally contraindicated in a patient with raised ICP and a focal mass lesion because it can precipitate:
a) Meningitis
b) Subarachnoid hemorrhage
c) Tonsillar Herniation (Coning)
d) Status Epilepticus
Explanation: The cranial and spinal compartments are continuous. If there is high pressure in the cranium (e.g., from a tumor or abscess) and a lumbar puncture is performed, fluid is removed from the spinal column, lowering the pressure there. This creates a pressure gradient from the head (high) to the spine (low). This gradient can force the brainstem and cerebellar tonsils downwards through the foramen magnum. This event, Tonsillar Herniation or "Coning," compresses the vital respiratory and cardiac centers in the medulla, leading to sudden death. Therefore, the correct answer is c) Tonsillar Herniation (Coning).
Chapter: General Physiology / Histology; Topic: Nerve Tissue; Subtopic: Structure of the Neuron (Soma and Organelles)
Key Definitions & Concepts
Nissl Granules (Nissl Bodies): Basophilic aggregations found in the cytoplasm of neurons, composed of Rough Endoplasmic Reticulum (RER) and free ribosomes.
Soma (Cell Body/Perikaryon): The metabolic center of the neuron containing the nucleus and major organelles, including abundant Nissl granules for protein synthesis.
Dendrites: Branching processes extending from the soma that receive signals; they contain Nissl granules in their proximal parts.
Axon: The long process responsible for transmitting action potentials; characteristically lacks Nissl granules and Golgi apparatus.
Axon Hillock: The cone-shaped region of the soma where the axon originates; it is the anatomical landmark where Nissl granules disappear.
Chromatolysis: A reaction to neuronal injury (axotomy) where Nissl granules disperse and disappear (dissolution), indicating a shift from neurotransmitter synthesis to structural repair proteins.
Basophilia: The staining property of Nissl granules (blue/purple with Nissl stains like cresyl violet) due to the high RNA content of ribosomes.
Axonal Transport: Since the axon lacks protein synthesis machinery (Nissl bodies), all proteins/enzymes must be transported from the soma via microtubules.
Initial Segment: The first part of the axon just distal to the hillock, site of action potential initiation; lacks Nissl bodies.
Neurofilaments: Structural proteins abundant in the axon, distinct from the synthetic Nissl bodies.
[Image of Neuron structure labeled diagram]
Lead Question - 2016
Nissl's granules are found in which part of nerve cell -
a) Axon hillock
b) Axons
c) Node of Ranvier
d) Body
Explanation: Nissl's granules (or Nissl bodies) represent the protein-synthesizing machinery of the neuron, consisting of stacks of Rough Endoplasmic Reticulum (RER) and polyribosomes. They are abundant in the Cell Body (Soma) and the proximal parts of the dendrites, reflecting the high metabolic activity required to maintain the large volume of the neuron. A defining characteristic of neuronal anatomy is that Nissl granules are absent in the Axon and the Axon Hillock. The sharp demarcation where Nissl staining stops marks the beginning of the axon. The Node of Ranvier is part of the axon. Therefore, the correct answer is d) Body.
1. Which specific organelle constitutes the "Nissl Body" seen under a light microscope?
a) Smooth Endoplasmic Reticulum
b) Golgi Apparatus
c) Rough Endoplasmic Reticulum and Ribosomes
d) Mitochondria
Explanation: Franz Nissl utilized basic aniline dyes to stain neurons. The clumpy, basophilic structures he observed (Nissl bodies) correspond structurally to the Rough Endoplasmic Reticulum (RER) and associated Free Ribosomes (polyribosomes). These organelles contain high concentrations of RNA (specifically rRNA), which is acidic and thus binds strongly to basic dyes like Cresyl Violet or Toluidine Blue. This massive amount of RER reflects the intense protein synthetic demands of the neuron, primarily for producing neurotransmitters and membrane proteins. The Golgi and SER do not stain as Nissl bodies. Therefore, the correct answer is c) Rough Endoplasmic Reticulum and Ribosomes.
2. Following an injury to the axon (axotomy), the cell body undergoes a reactive change called "Chromatolysis." This process is characterized microscopically by:
a) Aggregation of Nissl granules
b) Disappearance or dispersion of Nissl granules
c) Shrinkage of the cell body
d) Movement of the nucleus to the center
Explanation: Chromatolysis ("color dissolving") is a sign of neuronal reaction to injury (Wallerian degeneration distally, Chromatolysis proximally). The classic triad of signs in the soma includes: 1) Swelling of the cell body, 2) Displacement of the nucleus to the periphery (eccentric nucleus), and 3) Disappearance or dispersion of Nissl granules (central chromatolysis). The Nissl bodies break down and the RER disperses to prioritize the synthesis of cytoskeletal proteins (for axon regeneration) over neurotransmitter enzymes. This results in the loss of the characteristic staining pattern. Therefore, the correct answer is b) Disappearance or dispersion of Nissl granules.
3. Proteins required for the maintenance of the axon terminal must be transported from the soma because the axon lacks:
a) Microtubules
b) Mitochondria
c) Protein synthesis machinery (Ribosomes/Nissl)
d) Vesicles
Explanation: The axon is a metabolic extension of the soma. A fundamental rule of neurobiology is that the axon lacks protein synthesis machinery (i.e., no RER, no Ribosomes, no Nissl bodies). Therefore, all structural proteins, enzymes, and peptide neurotransmitters needed at the terminal must be synthesized in the soma and shipped down the axon via Axonal Transport (using kinesin/dynein motors on microtubules). While mitochondria and SER are present in axons, the lack of ribosomes makes the axon dependent on the soma. Therefore, the correct answer is c) Protein synthesis machinery (Ribosomes/Nissl).
4. The "Axon Hillock" is visually distinguished from the rest of the soma in stained preparations by the:
a) Presence of dark pigment
b) Absence of Nissl granules
c) High concentration of Golgi bodies
d) Presence of the nucleus
Explanation: The Axon Hillock is the conical projection of the soma from which the axon emerges. In histological sections stained with basic dyes (Nissl stains), the cytoplasm of the soma is filled with dark purple clumps (Nissl bodies). However, the region of the Axon Hillock is conspicuously pale and free of stain. This is because it lacks Nissl granules (RER). This "clear zone" acts as a funnel for cytoskeletal elements (neurofilaments/microtubules) to bundle together to form the axon. This sharp transition is a key diagnostic feature. Therefore, the correct answer is b) Absence of Nissl granules.
5. Which neuronal process contains Nissl granules in its proximal portion, similar to the cell body?
a) Axon
b) Axon terminal
c) Dendrite
d) Node of Ranvier
Explanation: Neurons have two types of processes: Axons and Dendrites. While the axon is devoid of protein synthetic organelles, Dendrites are different. The cytoplasm of the large, proximal dendrites is structurally similar to the soma and contains Nissl granules, mitochondria, and microtubules. This allows for local protein synthesis (e.g., of receptor proteins) near the dendritic spines, which is crucial for synaptic plasticity and memory. The presence of Nissl substance distinguishes proximal dendrites from axons under the microscope. Therefore, the correct answer is c) Dendrite.
6. Nissl staining relies on the interaction between basic aniline dyes and which cellular component?
a) DNA in the nucleus
b) RNA in ribosomes
c) Lipids in the membrane
d) Carbohydrates in the Golgi
Explanation: Basic dyes (like Cresyl Violet, Thionin, Methylene Blue) carry a positive charge. They bind avidly to negatively charged (acidic) molecules in the cell. Nucleic acids (DNA and RNA) are strongly acidic due to phosphate groups. While the nucleus (DNA) stains, the most intense cytoplasmic staining occurs in the Nissl bodies due to the extremely high concentration of Ribosomal RNA (rRNA) found in the rough endoplasmic reticulum. Thus, Nissl staining effectively maps the distribution of RNA/protein synthesis machinery. Therefore, the correct answer is b) RNA in ribosomes.
7. Which type of neuron would be expected to have the most abundant and prominent Nissl bodies?
a) Small granular cell of cerebellum
b) Large Alpha Motor Neuron of spinal cord
c) Bipolar neuron of retina
d) Sensory neuron in DRG (small type)
Explanation: The abundance of Nissl substance correlates with the volume of cytoplasm the cell must maintain. Large projection neurons with massive axons require huge amounts of protein synthesis to maintain the structural integrity of the distant axon terminal. Therefore, large motor neurons, such as the Alpha Motor Neurons of the spinal cord anterior horn (and Betz cells of cortex), have the most prominent, coarse, and abundant Nissl bodies. Small interneurons or granule cells have very sparse, fine Nissl substance because their metabolic demands are lower. Therefore, the correct answer is b) Large Alpha Motor Neuron of spinal cord.
8. The "Initial Segment" of the axon is the site of action potential initiation. Like the axon proper, this segment is characterized by the absence of:
a) Voltage-gated Sodium channels
b) Microtubules
c) Nissl bodies and Golgi complex
d) Mitochondria
Explanation: The Initial Segment lies just distal to the Axon Hillock. It is the functional trigger zone, possessing a high density of Voltage-gated Sodium channels. Structurally, it shares the properties of the axon. It contains bundled microtubules (fascicles) and neurofilaments and a specialized membrane undercoat (ankyrin-G). Crucially, like the rest of the axon, it lacks Nissl bodies (RER) and Golgi complexes. This exclusion of synthetic organelles prevents the synthesis of proteins that might interfere with the precise excitability of this region. Therefore, the correct answer is c) Nissl bodies and Golgi complex.
9. A neuropathologist uses a stain that specifically highlights Neurofilaments (silver stain). Unlike Nissl stain, this will visualize the:
a) Nucleolus
b) Entire Axon
c) Ribosomes
d) Rough ER
Explanation: Nissl stains only show the soma and proximal dendrites (where RNA is). To visualize the Axon, one must stain the cytoskeletal elements that fill it. Neurofilaments (neuron-specific intermediate filaments) are the most abundant cytoskeletal protein in the axon. Silver impregnation techniques (like Golgi or Cajal stains) deposit silver on these neurofibrils, allowing the visualization of the Entire Axon and its arborizations, which remain invisible (unstained) with Nissl stains. Therefore, the correct answer is b) Entire Axon.
10. The function of the Golgi apparatus in the neuron is to package neurotransmitters. Like Nissl bodies, the Golgi apparatus is generally absent from the:
a) Cell body
b) Dendrites
c) Axon
d) Perikaryon
Explanation: The Golgi apparatus is responsible for post-translational modification and packaging of proteins into vesicles. Since protein synthesis (Nissl) is restricted to the soma and dendrites, the packaging machinery (Golgi) is also restricted to these areas. The Axon does not contain a Golgi apparatus. Synaptic vesicles are either transported down from the soma or locally recycled at the terminal, but the primary Golgi stacks are excluded from the axoplasm. This reinforces the concept of the soma as the metabolic center and the axon as the transmission cable. Therefore, the correct answer is c) Axon.
Chapter: General Physiology; Topic: Synaptic Transmission; Subtopic: Presynaptic Modulation and Neurotransmitters
Key Definitions & Concepts
Presynaptic Facilitation: A process where activity in a regulatory neuron increases the amount of neurotransmitter released by the presynaptic neuron, usually by prolonging the action potential or increasing Calcium influx.
Presynaptic Inhibition: The suppression of neurotransmitter release from a presynaptic terminal, typically mediated by axo-axonic synapses releasing GABA (opening Cl- channels).
Glutamate: The primary excitatory neurotransmitter in the CNS; acts on ionotropic (NMDA, AMPA) and metabotropic receptors. Presynaptic glutamate receptors can facilitate release.
GABA (Gamma-Aminobutyric Acid): The primary inhibitory neurotransmitter in the brain; the classic mediator of presynaptic inhibition.
Glycine: The primary inhibitory neurotransmitter in the spinal cord and brainstem; opens Chloride channels.
Axo-axonic Synapse: The structural basis for presynaptic modulation, where one axon terminal synapses directly onto another axon terminal.
EPSP (Excitatory Postsynaptic Potential): A graded depolarization caused by Na+ or Ca2+ influx, moving the membrane potential toward threshold.
IPSP (Inhibitory Postsynaptic Potential): A graded hyperpolarization caused by Cl- influx or K+ efflux, moving the potential away from threshold.
SNARE Complex: Proteins (Synaptobrevin, Syntaxin, SNAP-25) responsible for the fusion of synaptic vesicles with the presynaptic membrane.
Quantal Release: Neurotransmitters are released in discrete packets (quanta), each corresponding to the contents of one synaptic vesicle.
[Image of Synaptic transmission mechanism]
Lead Question - 2016
Facilitatory presynaptic neurotransmitter is?
a) GABA
b) Glycine
c) Glutamate
d) Aspartate
Explanation: Presynaptic modulation involves altering the amount of neurotransmitter released from a nerve terminal. Presynaptic Inhibition is the most common form, classically mediated by GABA (via axo-axonic synapses) which reduces calcium influx. Presynaptic Facilitation enhances release. Among the options provided, Glutamate is the primary excitatory neurotransmitter. While Serotonin is the classic example of facilitation in simple systems (like Aplysia), Glutamate acting on presynaptic receptors (specifically Kainate or NMDA autoreceptors) has been shown to facilitate further neurotransmitter release in the mammalian CNS. GABA and Glycine are strictly inhibitory in this context. Therefore, the correct answer is c) Glutamate.
1. The mechanism of Presynaptic Inhibition in the spinal cord primarily involves the release of GABA acting on an axo-axonic synapse. This leads to:
a) Increased Calcium influx into the presynaptic terminal
b) Decreased Calcium influx into the presynaptic terminal
c) Increased Sodium influx into the postsynaptic neuron
d) Blockade of postsynaptic receptors
Explanation: Presynaptic inhibition is a mechanism to selectively suppress specific inputs. An inhibitory neuron releases GABA onto the presynaptic terminal of an excitatory fiber (axo-axonic synapse). GABA activates GABA-A (increasing Cl- conductance) or GABA-B (increasing K+ conductance) receptors. This partial depolarization or hyperpolarization reduces the amplitude of the arriving action potential. The crucial consequence is a Decreased influx of Calcium through voltage-gated calcium channels. Since transmitter release is calcium-dependent, less neurotransmitter is released, inhibiting the signal. Therefore, the correct answer is b) Decreased Calcium influx into the presynaptic terminal.
2. A patient presents with muscle weakness that improves with repeated use (facilitation). This clinical picture (Lambert-Eaton Myasthenic Syndrome) is caused by antibodies attacking:
a) Postsynaptic Acetylcholine Receptors
b) Presynaptic Voltage-Gated Calcium Channels
c) Synaptic Vesicle proteins (SNAREs)
d) Acetylcholinesterase enzyme
Explanation: Lambert-Eaton Myasthenic Syndrome (LEMS) is a paraneoplastic disorder (often associated with Small Cell Lung Cancer). The pathology involves autoantibodies directed against the Presynaptic Voltage-Gated Calcium Channels (VGCCs) at the neuromuscular junction. This reduces calcium entry, preventing vesicle fusion and Acetylcholine release. Unlike Myasthenia Gravis (postsynaptic defect) where repetitive stimulation depletes the readily releasable pool leading to fatigue, in LEMS, repetitive stimulation leads to accumulation of intra-terminal calcium, eventually facilitating release and improving strength. Therefore, the correct answer is b) Presynaptic Voltage-Gated Calcium Channels.
3. The "Synaptic Delay" observed in chemical synapses (approx. 0.5 ms) is primarily due to the time required for:
a) The action potential to travel down the axon
b) Diffusion of neurotransmitter across the cleft
c) Calcium entry and vesicle fusion
d) Postsynaptic receptor activation
Explanation: Chemical transmission is slower than electrical transmission. The delay of roughly 0.5 milliseconds occurs between the arrival of the action potential at the terminal and the start of the postsynaptic potential. While diffusion takes a negligible amount of time (microseconds), the rate-limiting step is the complex biological process of Calcium entry and vesicle fusion (exocytosis). The conformational changes in the SNARE complex and the actual release of the transmitter packet consume the bulk of this delay. Therefore, the correct answer is c) Calcium entry and vesicle fusion.
4. Strychnine causes convulsions and death by antagonizing which neurotransmitter receptor in the spinal cord?
a) GABA-A receptor
b) NMDA receptor
c) Glycine receptor
d) Nicotinic ACh receptor
Explanation: Strychnine is a potent poison. Its mechanism of action is the competitive antagonism (blocking) of Glycine receptors in the spinal cord and brainstem. Glycine is the major inhibitory neurotransmitter in the spinal cord, responsible for postsynaptic inhibition (IPSPs) of motor neurons (e.g., reciprocal inhibition). By blocking these inhibitory signals, Strychnine leads to unchecked excitation, resulting in severe, painful muscle spasms and convulsions (opisthotonus). Tetanus toxin also affects glycine but by preventing its release. Therefore, the correct answer is c) Glycine receptor.
5. An Excitatory Postsynaptic Potential (EPSP) differs from an Action Potential in that an EPSP:
a) Is all-or-none
b) Propagates without decrement
c) Is a graded potential capable of summation
d) Is always followed by a refractory period
Explanation: An Action Potential is a regenerative, all-or-none event that propagates long distances. An EPSP is a local, Graded Potential generated at the synapse. Its magnitude depends on the amount of neurotransmitter released. Crucially, EPSPs are passive and decay with distance (decremental conduction). Because they are not all-or-none, multiple EPSPs can add together spatially (from different inputs) or temporally (rapid fire from one input) to reach the threshold for an action potential. This property is called Summation. Therefore, the correct answer is c) Is a graded potential capable of summation.
6. Botulinum toxin (Botox) produces flaccid paralysis by:
a) Blocking postsynaptic ACh receptors
b) Cleaving SNARE proteins to prevent ACh release
c) Inhibiting Choline Acetyltransferase
d) Blocking presynaptic Calcium channels
Explanation: Botulinum toxin enters the presynaptic nerve terminal at the neuromuscular junction. It acts as a protease that specifically cleaves proteins of the SNARE complex (Synaptobrevin, Syntaxin, or SNAP-25 depending on the toxin serotype). These proteins are essential for the docking and fusion of synaptic vesicles with the presynaptic membrane. By destroying the fusion machinery, the toxin Prevents the release of Acetylcholine. Without ACh release, the muscle cannot contract, leading to flaccid paralysis. This contrasts with Curare (receptor blocker). Therefore, the correct answer is b) Cleaving SNARE proteins to prevent ACh release.
7. Which ion is primarily responsible for the generation of an Inhibitory Postsynaptic Potential (IPSP) in the central nervous system?
a) Sodium (Na+)
b) Calcium (Ca2+)
c) Chloride (Cl-)
d) Magnesium (Mg2+)
Explanation: An IPSP acts to hyperpolarize the membrane or stabilize it away from the threshold. Inhibitory neurotransmitters like GABA and Glycine bind to their receptors (GABA-A, Glycine-R), which are ligand-gated ion channels. When these channels open, they selectively allow the influx of Chloride ions (Cl-). Since the equilibrium potential of Chloride (~-70mV) is often more negative than the resting potential (or stabilizes it), the influx of negative charge hyperpolarizes the cell, making it less likely to fire. Potassium efflux can also cause IPSPs (GABA-B). Therefore, the correct answer is c) Chloride (Cl-).
8. The phenomenon where repeated stimulation of a single presynaptic neuron leads to a cumulative increase in the postsynaptic potential is known as:
a) Spatial Summation
b) Temporal Summation
c) Long Term Potentiation
d) Occlusion
Explanation: Neurons integrate information via summation. Temporal Summation occurs when a single presynaptic neuron fires in rapid succession. The second EPSP arrives before the first has dissipated, "piggybacking" on top of it to produce a larger total depolarization. Spatial Summation involves the simultaneous firing of multiple different presynaptic neurons at different locations on the soma/dendrites. Both forms of summation are essential for a neuron to reach the threshold for firing an action potential. Therefore, the correct answer is b) Temporal Summation.
9. Miniature End Plate Potentials (MEPPs) are observed at the neuromuscular junction in the resting state. These potentials represent:
a) The opening of a single ACh receptor channel
b) The spontaneous release of a single vesicle (quantum) of ACh
c) The leakage of ACh from the cytoplasm
d) Electrical noise from the recording equipment
Explanation: Even without nerve stimulation, sensitive recording at the motor end plate reveals tiny, random depolarizations of about 0.5-1 mV. These are Miniature End Plate Potentials (MEPPs). They are caused by the random, spontaneous fusion of single synaptic vesicles with the membrane, releasing a single "packet" or quantum of Acetylcholine (containing ~10,000 molecules). A full End Plate Potential (EPP) is the result of the synchronous release of hundreds of these quanta triggered by calcium influx. Therefore, the correct answer is b) The spontaneous release of a single vesicle (quantum) of ACh.
10. Glutamate toxicity (Excitotoxicity) leading to neuronal death in stroke involves the excessive influx of which ion through NMDA receptors?
a) Sodium only
b) Potassium only
c) Calcium
d) Chloride
Explanation: Glutamate is the major excitatory transmitter. The NMDA receptor is a unique glutamate receptor that functions as a coincidence detector. When open, it allows the passage of Sodium and, critically, Calcium. In pathological states like ischemic stroke, extracellular glutamate levels rise uncontrollably. This causes overstimulation of NMDA receptors and a massive, unregulated influx of Calcium into the neuron. This calcium overload activates intracellular enzymes (proteases, lipases, endonucleases) that digest the cell components, leading to necrotic or apoptotic cell death (Excitotoxicity). Therefore, the correct answer is c) Calcium.
Chapter: General Physiology; Topic: Sensory System; Subtopic: Thermal Sensation and Nerve Fibers
Key Definitions & Concepts
Thermoreceptors: Free nerve endings located in the skin that detect changes in temperature; divided into cold and warmth receptors.
Cold Receptors: The most numerous thermoreceptors; stimulated by temperatures between 10°C and 40°C. Signals are transmitted primarily by A-delta fibers (and some C fibers).
Warmth Receptors: Less numerous than cold receptors; stimulated by temperatures between 30°C and 49°C. Signals are transmitted almost exclusively by C fibers.
A-delta Fibers (Type III): Thinly myelinated, fast-conducting fibers responsible for "Fast Pain" (sharp) and "Cold" sensation.
C Fibers (Type IV): Unmyelinated, slow-conducting fibers responsible for "Slow Pain" (aching), "Warmth," and itch.
Paradoxical Cold: The sensation of cold perceived when a cold receptor is stimulated by a noxious heat stimulus (above 45°C).
TRP Channels: Transient Receptor Potential channels on nerve endings that act as molecular thermometers (e.g., TRPM8 for cold/menthol, TRPV1 for heat/capsaicin).
Lateral Spinothalamic Tract: The ascending pathway in the spinal cord carrying pain and temperature sensations to the thalamus.
Adaptation: Thermoreceptors show rapid adaptation; they respond vigorously to changes in temperature but decrease firing during constant temperature.
Spatial Summation: The ability to detect weak thermal stimuli increases significantly when a larger area of skin is stimulated.
[Image of Nerve fiber classification diagram]
Lead Question - 2016
Warmth sensation is carried by?
a) A alpha fibers
b) A beta fibers
c) A gamma fibers
d) A delta fibers
Explanation: Thermal sensations are transmitted by specific nerve fibers. Warmth sensation is transmitted mainly by unmyelinated Type C fibers at transmission velocities of 0.4 to 2 m/s. Cold sensation is transmitted mainly by Type A-delta fibers (thinly myelinated) and to a lesser extent by Type C fibers. In the provided options, "A S" likely represents A-delta (due to OCR error), and "A (3" represents A-beta. Since Warmth is physiologically carried by C fibers (which are not listed as a clear option here, but are the correct physiological answer), one must differentiate carefully. If the question implies thermal sensation in general or "Cold," A-delta is the answer. However, for "Warmth" specifically, the strict answer is C fibers. (Note: In exams, if C is absent and A-delta is the only thermal fiber listed, it is sometimes chosen by exclusion, but C is the correct physiology). Therefore, the correct physiological answer is C fibers.
1. Which specific TRP ion channel is activated by innocuous cold temperatures (10-25°C) and cooling agents like Menthol?
a) TRPV1
b) TRPM8
c) TRPV3
d) TRPA1
Explanation: Thermoreception is mediated by Transient Receptor Potential (TRP) channels. TRPM8 (Melastatin 8) is the primary cold-sensing receptor. It opens in response to cooling temperatures (below 25°C) and chemical agents like Menthol (from mint) and Eucalyptol, producing a sensation of cold. TRPV1 responds to noxious heat (>43°C) and Capsaicin. TRPV3/TRPV4 respond to warm temperatures. TRPA1 responds to noxious cold and irritants (mustard oil). Therefore, the correct answer is b) TRPM8.
2. The sensation of "Paradoxical Cold" occurs when a cold receptor is stimulated by:
a) Extreme cold (< 0°C)
b) Hot stimulus (> 45°C)
c) Mechanical pressure
d) Chemical irritants
Explanation: Paradoxical cold is a physiological curiosity where a heat stimulus produces a sensation of cold. This occurs because Cold Receptors (A-delta fibers) can be stimulated not only by low temperatures but also by high, noxious temperatures (typically > 45°C). Because the "labeled line" for that nerve fiber communicates "cold" to the brain, the brain interprets this high-heat activation as "cold." This is often experienced when entering a very hot bath. Therefore, the correct answer is b) Hot stimulus (> 45°C).
3. Which type of nerve fiber is primarily responsible for transmitting signals from Cold Receptors?
a) A-alpha
b) A-beta
c) A-delta
d) B fibers
Explanation: There is a distinct difference in the transmission of cold vs. warmth. Cold signals are transmitted primarily by A-delta fibers (thinly myelinated, conduction velocity ~20 m/s). Some cold signals are also carried by C fibers. In contrast, warmth signals are carried almost exclusively by C fibers. This explains why the sensation of cold is perceived relatively quickly compared to the slower, building sensation of warmth. Therefore, the correct answer is c) A-delta.
4. Thermal sensations (both pain and temperature) ascend the spinal cord via which tract?
a) Dorsal Column Medial Lemniscus
b) Lateral Spinothalamic Tract
c) Anterior Spinothalamic Tract
d) Spinocerebellar Tract
Explanation: The Anterolateral System is responsible for pain and temperature. Specifically, the Lateral Spinothalamic Tract carries fibers for Pain and Temperature. These fibers enter the spinal cord, synapse in the dorsal horn, and cross (decussate) to the contralateral side via the anterior white commissure before ascending. The Anterior Spinothalamic tract carries crude touch and pressure. The Dorsal Columns carry fine touch and proprioception. Therefore, the correct answer is b) Lateral Spinothalamic Tract.
5. A lesion of the Lateral Spinothalamic tract on the right side of the spinal cord at T10 would result in loss of thermal sensation on the:
a) Right leg
b) Left leg
c) Right arm
d) Left arm
Explanation: Because the fibers carrying pain and temperature decussate (cross over) almost immediately upon entering the spinal cord (within 1-2 segments), the Lateral Spinothalamic Tract on one side carries information from the Opposite (Contralateral) side of the body. A lesion on the Right side at T10 would block signals ascending from the Left side below that level. Thus, the patient would lose temperature sensation in the Left leg. Therefore, the correct answer is b) Left leg.
6. At what temperature range do pain receptors (nociceptors) begin to be stimulated by heat, eliciting the sensation of burning pain?
a) > 30°C
b) > 37°C
c) > 45°C
d) > 60°C
Explanation: Thermal receptors function within specific ranges. Warmth receptors are active from roughly 30°C to 45°C. As the temperature rises above 45°C, the warmth receptors stop firing or are saturated, and the heat begins to cause tissue damage. At this point (>45°C), Pain receptors (polymodal nociceptors stimulating TRPV1 channels) are activated, and the sensation changes from "warm/hot" to "burning pain." Cold pain typically begins below 10-15°C. Therefore, the correct answer is c) > 45°C.
7. Which receptor is the molecular target for Capsaicin (the active component of chili peppers) and is responsible for the sensation of spicy heat?
a) TRPM8
b) TRPV1
c) TRPA1
d) Piezo2
Explanation: TRPV1 (Transient Receptor Potential Vanilloid 1) is a non-selective cation channel located on nociceptive nerve endings (C and A-delta fibers). It is activated by noxious heat (>43°C) and low pH (acid). Crucially, it is also activated by Capsaicin. When capsaicin binds to TRPV1, it lowers the channel's activation threshold to below body temperature, causing it to open at normal body heat. This results in the brain perceiving a sensation of "burning hot" despite no actual temperature rise. Therefore, the correct answer is b) TRPV1.
8. Compared to cold receptors, warmth receptors are:
a) More numerous
b) Less numerous
c) Equal in number
d) Found only in the viscera
Explanation: Mapping of thermal spots on the skin reveals a significant disparity in density. Cold receptors are far more numerous than warmth receptors. In most areas of the body, there are 3 to 10 times as many cold spots as warm spots. This evolutionary adaptation likely reflects the greater environmental threat posed by hypothermia compared to mild hyperthermia. Both receptors are free nerve endings found in the skin. Therefore, the correct answer is b) Less numerous.
9. Spatial summation is a critical feature of thermal sensation. This means that:
a) Temperature sensation persists after the stimulus is removed
b) Rapid changes in temperature are felt more intensely
c) Small changes in temperature are detected if a large area of skin is stimulated
d) Thermal signals summate with pain signals
Explanation: Thermal receptors (especially warmth) show strong Spatial Summation. While a tiny change in temperature (e.g., 0.01°C) applied to a single spot might not be felt, the same tiny change applied over a large body surface area (like the entire trunk) is easily detected. This allows the body to monitor total body heat load very precisely. This summation occurs centrally in the spinal cord and brainstem. Temporal adaptation is also a feature (feeling the bath water cool down over time). Therefore, the correct answer is c) Small changes in temperature are detected if a large area of skin is stimulated.
10. The cell bodies of the primary afferent neurons carrying temperature sensation from the body are located in the:
a) Dorsal Horn of Spinal Cord
b) Dorsal Root Ganglion
c) Ventral Posterior Nucleus of Thalamus
d) Sympathetic Chain Ganglion
Explanation: All primary somatic sensory neurons (for touch, pain, temperature, proprioception) have their cell bodies located in the Dorsal Root Ganglion (DRG) (for the body) or the Trigeminal Ganglion (for the face). These are pseudounipolar neurons. The peripheral process extends to the skin (free nerve ending), and the central process enters the spinal cord to synapse in the dorsal horn (Substantia Gelatinosa). The dorsal horn contains the second-order neuron bodies. Therefore, the correct answer is b) Dorsal Root Ganglion.
Chapter: General Physiology; Topic: Nerve-Muscle Physiology; Subtopic: Classification of Nerve Fibers
Key Definitions & Concepts
Erlanger-Gasser Classification: Classifies peripheral nerve fibers into A, B, and C groups based on diameter, myelination, and conduction velocity.
C Fibers: The smallest nerve fibers (0.5-2 micrometers) and the only group that is Unmyelinated. They conduct impulses slowly (0.5-2 m/s).
Sensory Function of C Fibers: They carry "Slow Pain" (dull, aching, burning), temperature (warmth), crude touch, and itch from the periphery to the dorsal horn (Dorsal Root C fibers).
Motor Function of C Fibers: They constitute the postganglionic autonomic fibers (Sympathetic and Parasympathetic) supplying smooth muscle, cardiac muscle, and glands.
A-Fibers: Large, myelinated fibers subdivided into alpha, beta, gamma, and delta. Responsible for somatic motor and fast sensory functions.
B-Fibers: Medium-sized, lightly myelinated fibers. They are exclusively Preganglionic Autonomic fibers.
Remak Bundles: The unique histological arrangement where multiple unmyelinated C fiber axons are enveloped by a single Schwann cell without forming myelin sheaths.
Susceptibility: C fibers are most susceptible to block by local anesthetics but least susceptible to block by pressure/hypoxia.
Polymodal Nociceptors: C fiber terminals that respond to multiple types of noxious stimuli (mechanical, thermal, and chemical).
Group IV Fibers: The equivalent term for C fibers in the Lloyd-Hunt numerical classification used for muscle afferents.
[Image of Nerve fiber classification diagram]
Lead Question - 2016
Types C nerve fibers are?
a) Sensory
b) Motor
c) Mixed
d) Any of the above
Explanation: The classification of C fibers includes two distinct functional groups. First, the Dorsal Root C fibers are sensory afferents responsible for transmitting slow pain, temperature, and crude touch. Second, the Sympathetic C fibers represent the postganglionic autonomic efferents (motor) supplying viscera, vessels, and glands. Therefore, anatomically and functionally, the category "Type C fibers" encompasses both sensory (afferent) and motor (autonomic efferent) neurons. Consequently, the correct description for the group as a whole is that they can be either sensory or motor depending on the specific nerve. Therefore, the correct answer is d) Any of the above (or functionally "Mixed" in the context of the whole body's population, but 'Any of the above' covers the fact that individual C fibers can be sensory OR motor).
1. Which physiological feature is unique to Type C nerve fibers compared to Type A and Type B fibers?
a) They have the largest diameter
b) They are unmyelinated
c) They are exclusively motor
d) They have the fastest conduction velocity
Explanation: The defining structural characteristic of Type C fibers in the Erlanger-Gasser classification is that they are Unmyelinated. Both Type A (somatic motor/sensory) and Type B (preganglionic autonomic) fibers possess a myelin sheath, which allows for faster saltatory conduction. C fibers lack this sheath; instead, multiple axons are embedded in the cytoplasm of a Schwann cell (Remak bundle), leading to continuous, slow conduction (0.5 to 2.0 m/s). This lack of myelin explains their slow transmission of "second pain." Therefore, the correct answer is b) They are unmyelinated.
2. Which sensory modality is primarily transmitted by C fibers?
a) Proprioception
b) Fine touch and vibration
c) Slow, burning pain
d) Fast, sharp pain
Explanation: Sensory information is segregated by fiber type. Proprioception travels on the fastest fibers (A-alpha, A-beta). Fine touch and vibration travel on A-beta fibers. Fast, sharp ("first") pain travels on A-delta fibers. C fibers, due to their slow conduction speed, transmit sensations that have a delayed onset or prolonged duration, specifically Slow, burning, aching pain ("second pain"), warmth, and itch (pruritus). This is why a stubbed toe hurts sharply first (A-delta), followed by a throbbing ache (C fiber). Therefore, the correct answer is c) Slow, burning pain.
3. In the autonomic nervous system, postganglionic sympathetic neurons are classified as:
a) Type A-gamma fibers
b) Type B fibers
c) Type C fibers
d) Type A-delta fibers
Explanation: The autonomic pathway consists of a two-neuron chain. The Preganglionic neuron (from CNS to ganglion) is a myelinated Type B fiber. The Postganglionic neuron (from ganglion to effector organ) is an unmyelinated Type C fiber. This applies to both the sympathetic and parasympathetic divisions (though parasympathetic postganglionic fibers are very short). Thus, the "motor" component of C fibers refers to these autonomic efferents regulating smooth muscle and glands. Therefore, the correct answer is c) Type C fibers.
4. When comparing susceptibility to nerve block, C fibers are the MOST susceptible to:
a) Hypoxia
b) Pressure
c) Local Anesthetics
d) Electrical stimulation
Explanation: Local anesthetics (like Lidocaine) block sodium channels. They penetrate small, unmyelinated fibers much more easily than large, myelinated ones (due to surface area/volume ratios and the need to block usually 3 nodes of Ranvier in myelinated fibers). Therefore, C fibers (pain/autonomic) are blocked first by Local Anesthetics. Conversely, they are the least susceptible to pressure/hypoxia, which preferentially blocks large A fibers first (causing limbs to "fall asleep" with loss of touch/motor but preservation of slow pain). Therefore, the correct answer is c) Local Anesthetics.
5. Which neurotransmitter is characteristically co-released with Glutamate by nociceptive C fibers in the spinal cord?
a) Acetylcholine
b) Substance P
c) Glycine
d) Norepinephrine
Explanation: C fibers involved in pain transmission release excitatory neurotransmitters at their synapse in the dorsal horn (substantia gelatinosa). While Glutamate mediates rapid transmission, C fibers uniquely release neuropeptides, most notably Substance P (and CGRP). Substance P binds to NK-1 receptors and mediates slow, prolonged synaptic potentials, contributing to the "wind-up" phenomenon and the lasting nature of slow pain. A-delta fibers rely mostly on Glutamate. Therefore, the correct answer is b) Substance P.
6. According to the Lloyd-Hunt numerical classification of sensory nerve fibers, Group IV fibers are equivalent to:
a) A-alpha fibers
b) A-beta fibers
c) A-delta fibers
d) C fibers
Explanation: The numerical classification (Group I-IV) is used primarily for muscle afferents. Group Ia/Ib = A-alpha (Primary spindle/GTO). Group II = A-beta (Secondary spindle/Touch). Group III = A-delta (Fast pain/Cold). Group IV = C fibers (Slow pain/Temperature). Knowing these equivalents is crucial as physiology questions often interchange the terminology. Group IV fibers are the unmyelinated free nerve endings in muscle and joints responding to noxious stimuli. Therefore, the correct answer is d) C fibers.
7. The "Triple Response of Lewis" (Red reaction, Flare, Wheal) is mediated in part by the antidromic release of substances from which nerve fibers?
a) Sympathetic postganglionic fibers
b) Sensory C fibers
c) Motor A-alpha fibers
d) Parasympathetic fibers
Explanation: When skin is injured, the "Flare" component (redness spreading beyond the injury site) is caused by an Axon Reflex. Impulses travel up sensory C fibers and then travel "backwards" (antidromically) down other branches of the same nerve fiber supplying adjacent skin. This triggers the release of vasoactive neuropeptides like Substance P and CGRP from the nerve terminals, causing vasodilation and mast cell degranulation (histamine release). This neurogenic inflammation is a specific function of peptidergic C fibers. Therefore, the correct answer is b) Sensory C fibers.
8. Which of the following values represents the typical conduction velocity range for C fibers?
a) 70-120 m/s
b) 30-70 m/s
c) 3-15 m/s
d) 0.5-2.0 m/s
Explanation: Conduction velocities are strictly stratified. A-alpha: 70-120 m/s (fastest). A-beta: 30-70 m/s. A-gamma: 15-30 m/s. A-delta: 6-30 m/s. B fibers: 3-15 m/s. C fibers: 0.5-2.0 m/s (slowest). Because they are unmyelinated, they rely on continuous conduction, which is significantly slower than saltatory conduction. This velocity is roughly equivalent to walking speed. Therefore, the correct answer is d) 0.5-2.0 m/s.
9. A-delta fibers differ from C fibers in that A-delta fibers:
a) Are unmyelinated
b) Carry visceral pain primarily
c) Have a higher threshold for activation
d) Conduct impulses faster due to thin myelination
Explanation: A-delta and C fibers are both nociceptors, but they have key structural differences. A-delta fibers are thinly myelinated, whereas C fibers are unmyelinated. This myelination allows A-delta fibers to conduct impulses faster (6-30 m/s vs 0.5-2 m/s). This speed difference is the physiological basis for the "double pain" sensation (sharp first, aching second). A-delta fibers are responsible for the initial, localizing withdrawal reflex. Visceral pain is predominantly C fiber mediated. Therefore, the correct answer is d) Conduct impulses faster due to thin myelination.
10. The receptor type most commonly found on the terminal ends of C fibers that responds to noxious heat (>43°C) and Capsaicin is:
a) Piezo2
b) TRPV1
c) Merkel disc
d) Meissner corpuscle
Explanation: The molecular transducer for heat pain located on C fibers (and some A-delta fibers) is the TRPV1 (Transient Receptor Potential Vanilloid 1) channel. This ion channel opens in response to damaging heat, low pH (acid), and Capsaicin (the hot compound in chili peppers). Activation of TRPV1 leads to the influx of Calcium and Sodium, depolarizing the C fiber and generating the sensation of burning pain. Piezo2 is for touch; Meissner/Merkel are touch receptors on A-beta fibers. Therefore, the correct answer is b) TRPV1.
Chapter: General Physiology; Topic: Sensory System; Subtopic: Ascending Tracts and Sensory Transmission
Key Definitions & Concepts
C Fibers: Small, unmyelinated nerve fibers with slow conduction velocity (0.5–2 m/s). They are responsible for transmitting slow (burning) pain, temperature (warmth), and crude touch.
Lateral Spinothalamic Tract (LSTT): The ascending pathway in the lateral white column of the spinal cord that carries pain and temperature sensations to the thalamus.
Anterior Spinothalamic Tract (ASTT): The ascending pathway in the anterior white column that carries crude touch and pressure sensations.
Posterior Column-Medial Lemniscus Pathway (PCML): The pathway responsible for fine touch, vibration, and proprioception; consists of large, myelinated A-beta fibers.
Substantia Gelatinosa: Located in Lamina II of the dorsal horn; the primary site where C fibers synapse and where pain modulation (Gate Control) occurs.
Substance P: A neuropeptide neurotransmitter released by C fibers involved in the transmission of slow pain and neurogenic inflammation.
Paleospinothalamic Pathway: The phylogenetically older division of the pain system (C fibers) that projects to the reticular formation and intralaminar nuclei, mediating the emotional aspect of pain.
Neospinothalamic Pathway: The newer division (A-delta fibers) projecting to the VPL nucleus, mediating the localization of sharp pain.
Decussation: The crossing over of nerve fibers to the opposite side. Spinothalamic fibers decussate in the spinal cord (anterior white commissure), whereas PCML fibers decussate in the medulla.
Lissauer's Tract: The dorsolateral tract where primary afferent fibers may ascend or descend 1-2 segments before entering the dorsal horn gray matter.
[Image of Pain pathways diagram]
Lead Question - 2016
'C' fibers carry sensations through which pathway?
a) Posterior column
b) Anterior spinothalamic tract
c) Lateral spinothalamic tract
d) All of the above
Explanation: Nerve fibers are classified based on myelination and diameter. 'C' fibers are unmyelinated, small-diameter fibers responsible for transmitting Slow Pain (aching/burning) and Temperature sensations. These modalities are carried centrally by the Lateral Spinothalamic Tract. The Anterior Spinothalamic Tract carries crude touch and pressure (which also utilizes C and A-delta fibers, but the classic association for "sensation" regarding C fibers in isolation is pain/temperature). The Posterior Column carries fine touch and proprioception via large A-beta fibers. Therefore, the specific pathway for the hallmark sensations of C fibers (pain/temp) is the lateral tract. Therefore, the correct answer is c) Lateral spinothalamic tract.
1. A patient presents with a cape-like distribution of loss of pain and temperature sensation across the shoulders and arms. Proprioception is preserved. This "Dissociated Sensory Loss" is characteristic of a lesion affecting the:
a) Dorsal Root Ganglia
b) Anterior White Commissure
c) Posterior Columns
d) Corticospinal Tract
Explanation: This clinical picture describes Syringomyelia, a condition where a cystic cavity (syrinx) forms in the central canal of the spinal cord. As the cyst expands, it compresses the Anterior White Commissure. This is the anatomical location where the second-order neurons of the Spinothalamic tract (carrying pain and temperature) decussate (cross over) to the opposite side. Compression here interrupts these crossing fibers bilaterally at the level of the lesion, causing segmental loss of pain and temperature. The dorsal columns (proprioception) located posteriorly are typically spared. Therefore, the correct answer is b) Anterior White Commissure.
2. Which neurotransmitter is the primary agent responsible for the slow, prolonged synaptic transmission of pain signals from C fibers in the dorsal horn?
a) Glutamate
b) Glycine
c) Substance P
d) Acetylcholine
Explanation: C fibers release excitatory neurotransmitters to activate second-order neurons. They release Glutamate (acting on AMPA/NMDA receptors) for rapid signaling, but uniquely, they also co-release the neuropeptide Substance P. Substance P acts on NK-1 receptors. It has a slow onset and a long duration of action, allowing for the temporal summation of signals. This prolonged depolarization contributes to the characteristically "dull, aching, and persistent" nature of slow pain carried by C fibers. A-delta fibers release primarily Glutamate. Therefore, the correct answer is c) Substance P.
3. In the Brown-Sequard syndrome (hemisection of the spinal cord), which sensory deficit is observed on the CONTRA-lateral side below the level of the lesion?
a) Loss of vibration sense
b) Loss of fine touch
c) Loss of pain and temperature
d) Loss of proprioception
Explanation: In a spinal cord hemisection, tracts traveling on one side are cut. The Dorsal Columns (proprioception/vibration) ascend ipsilaterally and cross in the medulla; thus, damage causes ipsilateral loss. The Corticospinal tract (motor) is also crossed above; thus, damage causes ipsilateral paralysis. However, the Lateral Spinothalamic Tract (pain/temperature) contains fibers that have already crossed (decussated) at the spinal level below. Therefore, cutting the right side of the cord interrupts pain signals coming from the Left (Contralateral) side of the body. Therefore, the correct answer is c) Loss of pain and temperature.
4. The Paleospinothalamic pathway, which carries slow pain info via C fibers, projects primarily to which non-specific nuclei to mediate arousal and emotion?
a) Ventral Posterolateral (VPL) nucleus
b) Medial Geniculate Body
c) Intralaminar nuclei of Thalamus
d) Ventral Anterior nucleus
Explanation: The pain system has two main projection targets. The Neospinothalamic tract (fast pain) goes to the VPL nucleus for localization. The Paleospinothalamic tract (slow pain/C fibers) has a more diffuse termination. It projects to the Reticular Formation (for alertness) and the Intralaminar Nuclei (e.g., Centromedian) of the thalamus. These nuclei project diffusely to the cortex and limbic system, generating the emotional, unpleasant, and alerting responses ("suffering") associated with chronic pain, rather than precise localization. Therefore, the correct answer is c) Intralaminar nuclei of Thalamus.
5. A neurosurgeon performs a Cordotomy to relieve intractable pain in a cancer patient's right leg. To achieve this, the surgeon must interrupt the spinothalamic tract in the:
a) Right dorsal column
b) Left anterolateral quadrant
c) Right anterolateral quadrant
d) Left posterior horn
Explanation: The lateral spinothalamic tract carries pain information from the contralateral side of the body because fibers cross the midline shortly after entering the cord. Therefore, pain signals from the Right leg ascend in the Left side of the spinal cord (specifically the anterolateral quadrant). To relieve pain in the right leg, the surgeon must cut the Left Anterolateral Quadrant. The cut is usually made a few segments higher than the pain level to account for the ascent of fibers in Lissauer's tract before crossing. Therefore, the correct answer is b) Left anterolateral quadrant.
6. C fibers terminate primarily in which specific lamina of the dorsal horn?
a) Lamina I (Marginal Nucleus)
b) Lamina II (Substantia Gelatinosa)
c) Lamina III and IV (Nucleus Proprius)
d) Lamina IX (Motor pools)
Explanation: The gray matter of the spinal cord is divided into Rexed Laminae. Pain fibers have specific termination sites. A-delta fibers (fast pain) terminate largely in Lamina I (Marginal Zone) and Lamina V. C fibers (slow pain) terminate almost exclusively in Lamina II, also known as the Substantia Gelatinosa. This area is rich in interneurons and opioids and is the key anatomical site for the "Gate Control" modulation of pain transmission. Therefore, the correct answer is b) Lamina II (Substantia Gelatinosa).
7. Which sensory modality is NOT carried by the Anterolateral System (Spinothalamic tracts)?
a) Thermal sensation
b) Vibration
c) Crude touch
d) Pain
Explanation: The sensory systems are broadly divided into two pathways. The Anterolateral System (Anterior + Lateral Spinothalamic) carries Pain, Temperature, Crude touch, pressure, tickle, and itch. The Posterior Column-Medial Lemniscus (PCML) pathway carries Fine (discriminative) touch, Vibration, and Proprioception (joint position sense). Therefore, Vibration is the specific modality that does NOT travel in the Anterolateral system; it travels in the dorsal columns. This distinction is vital for localizing spinal cord lesions. Therefore, the correct answer is b) Vibration.
8. A patient with a stroke in the posterior cerebral artery territory develops "Thalamic Pain Syndrome" (Dejerine-Roussy). This involves a lesion of the VPL nucleus, which is the relay station for:
a) Spinothalamic tract only
b) Medial Lemniscus only
c) Both Spinothalamic tract and Medial Lemniscus
d) Trigeminal lemniscus
Explanation: The Ventral Posterolateral (VPL) nucleus of the thalamus is the master relay station for all somatosensory information from the body (neck down). It receives input from Both the Spinothalamic tract (Pain/Temp/Crude touch) and the Medial Lemniscus (Fine touch/Vibration/Proprioception). A lesion here causes complete hemianesthesia (loss of all sensation) on the contralateral side of the body. Recovery can be complicated by the development of excruciating central neuropathic pain. Therefore, the correct answer is c) Both Spinothalamic tract and Medial Lemniscus.
9. Lateral Medullary Syndrome (Wallenberg Syndrome) results in loss of pain and temperature on the ipsilateral face and contralateral body. The body sensory loss is due to damage to the:
a) Medial Lemniscus
b) Lateral Spinothalamic Tract
c) Spinal Nucleus of Trigeminal
d) Nucleus Gracilis
Explanation: In the lateral medulla, the Lateral Spinothalamic Tract (carrying pain/temp from the contralateral body) ascends located peripherally. A PICA infarct damages this tract, causing loss of pain/temp on the Contralateral Body. Simultaneously, the infarct damages the Spinal Nucleus/Tract of Trigeminal (carrying pain/temp from the ipsilateral face), causing loss of sensation on the Ipsilateral Face. This "crossed" sensory loss is the hallmark of Wallenberg syndrome. The Medial Lemniscus is medial and spared. Therefore, the correct answer is b) Lateral Spinothalamic Tract.
10. Unlike the Dorsal Column pathway, the first-order neurons of the Spinothalamic pathway:
a) Synapse immediately in the dorsal horn
b) Ascend all the way to the medulla before synapsing
c) Are multipolar neurons
d) Cross the midline in the dorsal root ganglion
Explanation: There is a fundamental difference in the trajectory of the first-order neuron. In the Dorsal Column system, the primary axon enters the cord and ascends ipsilaterally all the way to the medulla before synapsing. In the Spinothalamic system, the first-order neuron enters the cord and Synapses immediately (or within 1-2 segments via Lissauer's tract) on a second-order neuron in the dorsal horn. It is the second-order neuron that decussates and ascends. Both have cell bodies in the DRG (pseudounipolar). Therefore, the correct answer is a) Synapse immediately in the dorsal horn.
Chapter: General Physiology; Topic: Sensory System; Subtopic: Nerve Fiber Classification and Pain Transmission
Key Definitions & Concepts
Erlanger-Gasser Classification: Classifies nerve fibers into A, B, and C types based on diameter, myelination, and conduction velocity.
A-alpha (Aα) Fibers: The largest, fastest myelinated fibers; carry somatic motor signals and proprioception (muscle spindle).
A-delta (Aδ) Fibers: Thinly myelinated fibers; responsible for "Fast Pain" (sharp, pricking, localized) and temperature (cold).
C Fibers: The smallest, unmyelinated fibers with the slowest conduction velocity; responsible for "Slow Pain" (dull, aching, burning), diffuse visceral pain, and post-ganglionic autonomic signals.
B Fibers: Medium-sized myelinated preganglionic autonomic fibers.
Visceral Pain: Typically perceived as dull, aching, cramping, and poorly localized; transmitted almost exclusively by C fibers via the sympathetic/parasympathetic nerves.
First Pain vs. Second Pain: First pain is sharp and immediate (A-delta); Second pain is delayed, burning, and longer-lasting (C fibers).
Substance P: The primary neuropeptide neurotransmitter released by C fibers in the dorsal horn (along with glutamate).
Paleospinothalamic Tract: The ascending pathway that primarily carries slow pain signals from C fibers to the brainstem and intralaminar nuclei.
Susceptibility to Anesthesia: C fibers (small, unmyelinated) are generally the most susceptible to block by local anesthetics compared to large A fibers.
[Image of Nerve fiber classification diagram]
Lead Question - 2016
Dull visceral pain is carried by which type of neurons?
a) A gamma
b) Aa
c) C fibres
d) B
Explanation: Pain sensation is transmitted by two primary types of nerve fibers: A-delta and C fibers. A-delta fibers are myelinated and fast, carrying sharp, well-localized, "pricking" pain (cutaneous fast pain). C fibers are unmyelinated, thin, and slow-conducting. They carry dull, aching, burning, and poorly localized pain. Visceral pain is characteristically dull, deep, and cramping. The vast majority of afferents from the viscera are unmyelinated C fibers traveling with autonomic nerves. Therefore, the dull quality of visceral pain is encoded by these fibers. A-alpha are motor/proprioceptive. A-gamma are motor to muscle spindles. B fibers are preganglionic autonomic. Therefore, the correct answer is c) C fibres.
1. Which nerve fiber type has the slowest conduction velocity?
a) A-beta
b) A-delta
c) B fibers
d) C fibers
Explanation: Conduction velocity depends on axon diameter and myelination. Large, myelinated fibers conduct fastest (saltatory conduction). C fibers are unique because they are Unmyelinated and have the smallest diameter (0.5-2 micrometers). Consequently, they have the slowest conduction velocity (0.5-2 meters/sec). This slow speed accounts for the delay perceived in "second pain" (the throbbing ache that follows the initial sharp injury). All A fibers and B fibers are myelinated and faster. Therefore, the correct answer is d) C fibers.
2. Which sensory modality is primarily carried by A-beta fibers?
a) Sharp Pain
b) Temperature (Warmth)
c) Fine Touch and Vibration
d) Dull Aching Pain
Explanation: A-beta fibers are large, myelinated axons with fast conduction velocities (30-70 m/s). They are responsible for transmitting non-noxious cutaneous sensations such as Fine Touch, Vibration, and Pressure (from mechanoreceptors like Pacinian and Meissner corpuscles). These fibers ascend in the Dorsal Columns. Importantly, stimulating A-beta fibers (e.g., rubbing skin) can inhibit pain transmission from C fibers in the dorsal horn, forming the basis of the "Gate Control Theory" of pain relief. Therefore, the correct answer is c) Fine Touch and Vibration.
3. When a local anesthetic is applied to a nerve bundle, which fibers are typically blocked first?
a) Large myelinated motor fibers (A-alpha)
b) Small myelinated fibers (A-delta)
c) Small unmyelinated fibers (C fibers) and B fibers
d) Proprioceptive fibers
Explanation: Susceptibility to local anesthetic block is determined by size and myelination. Generally, smaller diameter fibers are blocked before larger ones because the anesthetic penetrates the axon more easily (surface area to volume ratio) and a shorter length of nerve needs to be affected to stop conduction in unmyelinated/short internode fibers. Therefore, the sequence of block is usually: Autonomic (B/C fibers) -> Pain/Temp (C and A-delta) -> Touch/Pressure (A-beta) -> Motor (A-alpha). Thus, C fibers (pain) are among the first to be blocked. Therefore, the correct answer is c) Small unmyelinated fibers (C fibers) and B fibers.
4. Muscle Spindles, which provide information about muscle length and stretch, are innervated by sensory fibers of type:
a) Ia and II
b) Ib
c) III (A-delta)
d) IV (C)
Explanation: In the numerical classification system (Lloyd-Hunt) used for sensory afferents from muscle: Type Ia fibers (primary endings) and Type II fibers (secondary endings) innervate Muscle Spindles. Type Ib fibers innervate Golgi Tendon Organs. Type III fibers correspond to A-delta (fast pain/cold). Type IV fibers correspond to C fibers (slow pain). The Ia fibers are the largest and fastest, crucial for the stretch reflex. Therefore, the correct answer is a) Ia and II.
5. Which nerve fiber type is responsible for the transmission of "First Pain" (sharp, localized)?
a) A-alpha
b) A-beta
c) A-delta
d) C fibers
Explanation: When you accidentally touch a hot stove, you feel two distinct waves of pain. The first is an immediate, sharp, well-localized "ouch" sensation. This is First Pain (or fast pain), carried by A-delta fibers (Group III). These are thinly myelinated and conduct at 6-30 m/s. Following this is a delayed, dull, burning ache ("Second Pain"), carried by slower, unmyelinated C fibers (0.5-2 m/s). This dual transmission serves first to withdraw (reflex) and then to protect/heal (ache). Therefore, the correct answer is c) A-delta.
6. Which fibers are most susceptible to block by hypoxia (pressure)?
a) A-fibers (Large myelinated)
b) B-fibers
c) C-fibers (Unmyelinated)
d) All are equally susceptible
Explanation: Different stressors block nerve fibers in different orders. Hypoxia (Pressure/Compression): Affects Large Myelinated (A fibers) first. This is why your leg "falls asleep" (numbness/motor loss) when crossed for too long, but you might still feel a dull ache later. Local Anesthetics: Affect Small fibers (C fibers) first. Temperature (Cold): Affects myelinated fibers before unmyelinated, but intermediate sizes often blocked early. Mnemonic: "HAL" - Hypoxia blocks A-fibers (Large). Therefore, the correct answer is a) A-fibers (Large myelinated).
7. Preganglionic autonomic neurons fall under which Erlanger-Gasser classification?
a) A-gamma
b) B fibers
c) C fibers
d) A-delta
Explanation: The Erlanger-Gasser classification is: A: Somatic motor and sensory (alpha, beta, gamma, delta). B: Preganglionic Autonomic (sympathetic/parasympathetic). These are lightly myelinated, medium diameter (3-15 m/s). C: Postganglionic Autonomic (motor) and Slow Pain (sensory). These are unmyelinated. A-gamma fibers are motor to muscle spindles (intrafusal fibers). Therefore, the correct answer is b) B fibers.
8. The neurotransmitter "Substance P" is most closely associated with the synaptic transmission of:
a) Fine touch
b) Vibration
c) Slow, chronic pain
d) Fast, sharp pain
Explanation: Nociceptive C-fibers release excitatory neurotransmitters at their synapse in the spinal cord (Substantia Gelatinosa). While Glutamate is released for rapid signaling, C-fibers also co-release the neuropeptide Substance P. Substance P acts on NK-1 receptors. It is released slowly and its effects are prolonged, contributing to the temporal summation and the lasting, aching quality of Slow, chronic pain. A-delta fibers (fast pain) release primarily Glutamate. Therefore, the correct answer is c) Slow, chronic pain.
9. A-gamma (γ) motor neurons innervate:
a) Extrafusal muscle fibers (Skeletal muscle)
b) Intrafusal muscle fibers (Muscle Spindle)
c) Smooth muscle
d) Golgi tendon organs
Explanation: The motor output to muscle is divided into Alpha and Gamma systems. A-alpha motor neurons: Innervate the main force-generating muscle fibers (Extrafusal fibers). A-gamma motor neurons: Innervate the specialized muscle fibers inside the Muscle Spindle (Intrafusal fibers). Activation of gamma motor neurons contracts the ends of the spindle, keeping it taut and sensitive to stretch even when the main muscle shortens (Alpha-Gamma Coactivation). Therefore, the correct answer is b) Intrafusal muscle fibers (Muscle Spindle).
10. Which fiber type corresponds to Group IV in the numerical (Lloyd-Hunt) classification?
a) A-alpha
b) A-beta
c) A-delta
d) C fibers
Explanation: There are two classifications: Letter (Erlanger-Gasser) for all nerves, and Number (Lloyd-Hunt) specifically for sensory nerves from muscle. Group I (Ia/Ib) = A-alpha (largest). Group II = A-beta. Group III = A-delta. Group IV = C fibers (unmyelinated). Knowing the equivalence is important as questions may use either terminology interchangeably. Group IV fibers typically carry pain (nociception) from muscles. Therefore, the correct answer is d) C fibers.
Chapter: General Physiology; Topic: Sensory System; Subtopic: Pain Pathways (Ascending Tracts)
Key Definitions & Concepts
Nociception: The neural process of encoding noxious stimuli; the physiological perception of pain caused by tissue damage.
Neospinothalamic Tract: The "Specific" pain pathway. Carries fast, sharp, well-localized pain (A-delta fibers). Projects to the VPL nucleus of the thalamus and then to the Somatosensory Cortex.
Paleospinothalamic Tract: The "Non-specific" pain pathway. Carries slow, burning, aching, poorly localized pain (C-fibers). Projects to the Reticular Formation and Intralaminar nuclei of the thalamus.
A-delta Fibers: Myelinated, fast-conducting fibers responsible for "First Pain" (sharp/pricking). Release Glutamate.
C-Fibers: Unmyelinated, slow-conducting fibers responsible for "Second Pain" (dull/aching). Release Substance P and Glutamate.
Substantia Gelatinosa (Lamina II): The "gatekeeper" area in the dorsal horn of the spinal cord where pain modulation occurs (Gate Control Theory).
Lissauer’s Tract: The dorsolateral tract where pain fibers ascend or descend 1-2 segments before entering the dorsal horn.
Anterolateral System: The collective name for the Spinothalamic, Spinoreticular, and Spinomesencephalic tracts carrying pain and temperature.
VPL Nucleus (Ventral Posterolateral): The specific relay nucleus in the thalamus for body sensation (pain/temp/touch).
Intralaminar Nuclei: Non-specific thalamic nuclei that project diffusely to the cortex and limbic system, mediating the emotional and arousal aspects of pain.
[Image of Pain pathways diagram]
Lead Question - 2016
Non-specific pain pathway is for?
a) Nociceptive pain
b) Neuropathic pain
c) Idiopathic pain
d) Inflammatory pain
Explanation: The ascending pain pathways are divided into the Neospinothalamic tract (Lateral/Specific) and the Paleospinothalamic tract (Medial/Non-specific). The Neospinothalamic tract carries "fast" sharp pain (A-delta fibers) directly to the specific sensory nuclei of the thalamus (VPL) for precise localization. The Paleospinothalamic tract carries "slow" burning or aching pain (C-fibers) and projects widely to the Reticular Formation and the Non-specific Intralaminar nuclei of the thalamus. This pathway is responsible for the arousal, emotional, and unpleasant aspects of pain. Both pathways are physiological systems designed to transmit signals arising from tissue damage, which is the definition of Nociceptive pain. Neuropathic pain is pathological damage to the nerve itself. Therefore, the correct answer is a) Nociceptive pain.
1. Which neurotransmitter is primarily released by A-delta fibers at their synapse in the dorsal horn to mediate fast pain?
a) Substance P
b) Glutamate
c) Glycine
d) Enkephalin
Explanation: Primary afferent nociceptors release excitatory neurotransmitters in the spinal cord. A-delta fibers, which are responsible for fast, sharp, and localized pain (first pain), primarily release Glutamate. Glutamate acts on AMPA and NMDA receptors to produce rapid synaptic transmission (milliseconds). In contrast, C-fibers (slow pain) release both Glutamate and the neuropeptide Substance P. Substance P is released more slowly and over a longer duration, contributing to the lingering, aching quality of slow pain. Glycine and Enkephalins are inhibitory transmitters involved in pain suppression. Therefore, the correct answer is b) Glutamate.
2. A patient experiences loss of pain and temperature sensation on the right side of the body below the level of the umbilicus. This indicates a lesion in the:
a) Right Dorsal Column
b) Left Lateral Spinothalamic Tract
c) Right Lateral Spinothalamic Tract
d) Left Dorsal Column
Explanation: The pathway for pain and temperature (Spinothalamic tract) decussates (crosses over) very early, usually within 1-2 segments of entry into the spinal cord, via the anterior white commissure. It then ascends in the Contralateral Lateral Funiculus. Therefore, a lesion of the spinothalamic tract causes loss of pain and temperature sensation on the Opposite (Contralateral) side of the body below the lesion. Since the patient has symptoms on the Right side, the lesion must be in the Left Lateral Spinothalamic Tract. Dorsal column lesions cause ipsilateral loss of vibration/proprioception. Therefore, the correct answer is b) Left Lateral Spinothalamic Tract.
3. The "Gate Control Theory" of pain proposes that non-painful stimuli (like rubbing a bumped shin) can inhibit pain transmission. This modulation occurs anatomically in the:
a) Thalamus
b) Somatosensory Cortex
c) Substantia Gelatinosa (Lamina II)
d) Reticular Formation
Explanation: The Gate Control Theory states that the transmission of pain signals through the dorsal horn of the spinal cord can be modulated by the simultaneous activation of large myelinated touch fibers (A-beta fibers). These A-beta fibers stimulate inhibitory interneurons (in the Substantia Gelatinosa of Rolando, Lamina II) that release Enkephalins/GABA. These inhibitory interneurons then presynaptically inhibit the incoming C-fibers and A-delta fibers, effectively "closing the gate" to pain transmission before it ascends to the brain. Therefore, the anatomical site is the Substantia Gelatinosa (Lamina II). Therefore, the correct answer is c) Substantia Gelatinosa (Lamina II).
4. The Paleospinothalamic tract (Slow Pain pathway) terminates primarily in which thalamic nuclei?
a) Ventral Posterolateral (VPL) Nucleus
b) Ventral Posteromedial (VPM) Nucleus
c) Intralaminar and Midline Nuclei
d) Lateral Geniculate Body
Explanation: The Spinothalamic system has two main projection targets. The Neospinothalamic tract (Fast pain) projects to the Ventrobasal complex (VPL and VPM), providing somatotopic localization ("My finger hurts"). The Paleospinothalamic tract (Slow pain) projects diffusely to the Reticular Formation of the brainstem and the Intralaminar (e.g., Centromedian) and Midline nuclei of the thalamus. These non-specific nuclei project widely to the cortex and limbic system, mediating the "suffering," alerting, and emotional aspects of pain ("I feel awful"), which are poorly localized. Therefore, the correct answer is c) Intralaminar and Midline Nuclei.
5. A 50-year-old male presents with severe, burning, intractable pain on the left side of his body following a stroke two months ago. Examination reveals raised threshold to pain but exaggerated response once perceived (Hyperpathia). This condition is known as:
a) Brown-Sequard Syndrome
b) Thalamic Pain Syndrome (Dejerine-Roussy)
c) Syringomyelia
d) Tabes Dorsalis
Explanation: This is the classic presentation of Thalamic Pain Syndrome (Dejerine-Roussy Syndrome). It typically results from a lacunar stroke involving the VPL/VPM nuclei of the thalamus (posterior cerebral artery territory). The initial anesthesia is followed weeks later by the development of severe, spontaneous, burning, or crushing pain (Central Neuropathic Pain) on the contralateral side. The pain is often exacerbated by light touch (Allodynia) or emotional stress and is notoriously difficult to treat. It represents a disinhibition of pain pathways or aberrant reorganization. Therefore, the correct answer is b) Thalamic Pain Syndrome (Dejerine-Roussy).
6. Which descending pathway is primarily responsible for the endogenous analgesia system (pain suppression)?
a) Corticospinal tract
b) Periaqueductal Gray (PAG) - Raphe Magnus pathway
c) Vestibulospinal tract
d) Rubrospinal tract
Explanation: The brain has a powerful descending system to inhibit pain entry at the spinal cord level. The key control center is the Periaqueductal Gray (PAG) matter in the midbrain. The PAG receives input from the hypothalamus and cortex. It projects to the Nucleus Raphe Magnus (in the medulla) and the Locus Coeruleus. Neurons from the Raphe Magnus (Serotonergic) and Locus Coeruleus (Noradrenergic) descend in the dorsolateral funiculus to the spinal cord dorsal horn, where they activate inhibitory interneurons (enkephalinergic) to block pain transmission. Therefore, the correct answer is b) Periaqueductal Gray (PAG) - Raphe Magnus pathway.
7. Visceral pain is often "referred" to somatic structures (e.g., cardiac pain to the left arm). The physiological basis for this Referred Pain is:
a) Convergence of somatic and visceral fibers on the same second-order neurons
b) Direct connection between the heart and the arm muscles
c) Misinterpretation by the peripheral receptors
d) Crossing over of fibers in the dorsal root ganglion
Explanation: Referred pain occurs because visceral pain fibers and somatic pain fibers (from the skin/muscles) enter the spinal cord at the same segment and Converge on the same Second-Order Neurons (Projection neurons) in the dorsal horn. Since the brain is accustomed to receiving signals from the skin (somatic) much more frequently than from the viscera, it "misinterprets" the signal coming from the shared neuron as originating from the corresponding dermatome. For example, heart afferents (T1-T5) converge with arm afferents (T1-T5), causing the brain to perceive the pain in the arm. Therefore, the correct answer is a) Convergence of somatic and visceral fibers on the same second-order neurons.
8. The tract of Lissauer (Dorsolateral fasciculus) serves which important function in the pain pathway?
a) It carries pain fibers directly to the brainstem
b) It allows pain fibers to ascend or descend 1-2 segments before entering the dorsal horn
c) It contains the cell bodies of the second-order neurons
d) It is the site of decussation
Explanation: When dorsal root fibers (A-delta and C) enter the spinal cord, they do not immediately penetrate the grey matter. Instead, they enter the Tract of Lissauer (Dorsolateral fasciculus) located at the tip of the dorsal horn. Here, the fibers bifurcate and ascend or descend for 1-2 spinal segments before synapsing on second-order neurons in the dorsal horn (Substantia Gelatinosa). This anatomical arrangement explains why a spinal cord lesion results in sensory loss that may begin 1-2 segments below the actual level of the lesion. Therefore, the correct answer is b) It allows pain fibers to ascend or descend 1-2 segments before entering the dorsal horn.
9. Anterolateral Cordotomy is a surgical procedure used for intractable pain relief. To relieve pain in the right leg, the surgeon must cut the:
a) Right Anterolateral quadrant
b) Left Anterolateral quadrant
c) Right Dorsal column
d) Left Dorsal column
Explanation: The Anterolateral system (Spinothalamic tract) carries pain and temperature information. Crucially, these fibers cross over (decussate) to the opposite side of the spinal cord almost immediately upon entry (within 1-2 segments). Therefore, pain signals from the Right leg ascend in the Left side of the spinal cord. To interrupt these signals and provide pain relief for the right leg, the surgeon must perform a cordotomy on the Left Anterolateral quadrant of the spinal cord (usually a few segments above the painful level to account for Lissauer's tract ascent). Therefore, the correct answer is b) Left Anterolateral quadrant.
10. Which receptor type is responsible for transduction of noxious heat (>45°C) and is activated by Capsaicin (from chili peppers)?
a) Piezo2
b) TRPM8
c) TRPV1
d) ASIC
Explanation: Nociceptors express specific ion channels that transduce noxious stimuli. The TRPV1 (Transient Receptor Potential Vanilloid 1) channel is the classic heat-sensing receptor. It opens in response to noxious heat (>43-45°C) and protons (acid). Crucially, it is also the specific receptor for Capsaicin, the active ingredient in hot chili peppers. Binding of capsaicin lowers the heat activation threshold to body temperature, creating the sensation of "burning" heat. TRPM8 detects cold/menthol. Piezo2 detects touch. ASIC detects acid. Therefore, the correct answer is c) TRPV1.
Chapter: Neuroanatomy; Topic: Spinal Cord; Subtopic: Blood Supply of the Spinal Cord
Key Definitions & Concepts
Segmental Spinal Arteries: Feeder arteries that enter the intervertebral foramina to reinforce the longitudinal spinal arteries; they arise from regional systemic arteries (e.g., Vertebral, Intercostal, Lumbar).
Anterior Spinal Artery (ASA): Single longitudinal artery formed by the union of two branches from the Vertebral arteries; supplies the anterior 2/3 of the cord.
Posterior Spinal Arteries (PSA): Paired longitudinal arteries originating from the Vertebral arteries or PICA; supply the posterior 1/3 (Posterior Columns).
Radicular Arteries: Branches of the segmental spinal arteries that supply the nerve roots and dura but do not reach the spinal cord itself.
Segmental Medullary Arteries: Large branches of segmental arteries that reach and anastomose with the ASA/PSA to supply the cord tissue.
Artery of Adamkiewicz: The largest segmental medullary artery (Arteria Radicularis Magna), typically arising on the left side between T9-L1; crucial for supplying the lower cord.
Watershed Area: The mid-thoracic region (T4-T8) which has a precarious blood supply due to the distance between significant feeder arteries; vulnerable to ischemia.
Vasocorona: An anastomotic pial plexus connecting the anterior and posterior spinal arteries on the surface of the cord.
Vertebral Artery: The source of the spinal arteries in the cervical region; it ascends through the foramina transversaria.
Sulcal Arteries: Branches of the ASA that penetrate the anterior median fissure to supply the central gray matter.
[Image of Blood supply of spinal cord diagram]
Lead Question - 2016
Spinal segmental artery is a branch of ?
a) Ascending spinal artery
b) Basilar artery
c) Posterior spinal artery
d) Anterior spinal artery
Explanation: The spinal cord blood supply is reinforced along its length by Segmental Spinal Arteries. These arteries do not originate from the spinal cord's own longitudinal vessels (Anterior/Posterior Spinal Arteries) or the Basilar artery. Instead, they arise from regional arteries outside the spinal column. In the neck, they arise from the Vertebral Artery and the Ascending Cervical Artery. In the thorax, from Posterior Intercostals. In the lumbar region, from Lumbar arteries. The option "Ascending spinal artery" is likely a recall approximation for the Ascending Cervical Artery (a branch of the Inferior Thyroid artery) or potentially the Vertebral Artery (which ascends the spine). Since the other options are recipients (ASA/PSA) or unrelated (Basilar), option A is the intended answer representing the external source. Therefore, the correct answer is a) Ascending spinal artery (Ascending Cervical/Vertebral).
1. The Artery of Adamkiewicz (Arteria Radicularis Magna) is the principal blood supply for the lower two-thirds of the spinal cord. It most commonly arises from a posterior intercostal artery on the:
a) Right side, levels T4-T8
b) Left side, levels T9-L1
c) Right side, levels L2-L4
d) Left side, levels C5-T1
Explanation: The Great Radicular Artery of Adamkiewicz is the most clinically significant feeder vessel for the spinal cord. Its occlusion (e.g., during aortic aneurysm repair) can lead to anterior spinal artery syndrome and paraplegia. Anatomically, it shows a distinct preference for location. In approximately 75% of individuals, it arises on the Left side. Its level of origin varies but is typically between the lower thoracic and upper lumbar segments, specifically T9 to L1 (sometimes T12-L2). It reinforces the Anterior Spinal Artery to supply the lumbar enlargement. Therefore, the correct answer is b) Left side, levels T9-L1.
2. A patient presents with sudden onset of paraplegia and loss of pain and temperature sensation in both legs. Vibration and proprioception are preserved. This "Dissociated Sensory Loss" is characteristic of occlusion of the:
a) Posterior Spinal Arteries
b) Anterior Spinal Artery
c) Vertebral Artery
d) PICA
Explanation: This is the classic presentation of Anterior Spinal Artery Syndrome. The Anterior Spinal Artery supplies the anterior two-thirds of the spinal cord. This area includes the Corticospinal tracts (motor function) and the Spinothalamic tracts (pain and temperature). Consequently, ischemia here causes paralysis and loss of pain/temperature sensation. However, the posterior columns (Dorsal columns), which carry vibration and proprioception, are supplied by the Posterior Spinal Arteries. These are spared in an ASA occlusion, leading to the preservation of dorsal column modalities. Therefore, the correct answer is b) Anterior Spinal Artery.
3. The Posterior Spinal Arteries are paired vessels that run longitudinally along the posterolateral aspect of the cord. In 75% of cases, they originate directly from the:
a) Anterior Spinal Artery
b) Basilar Artery
c) Posterior Inferior Cerebellar Artery (PICA)
d) Vertebral Artery
Explanation: While the Anterior Spinal Artery is formed by the Y-shaped union of branches from the Vertebral arteries, the Posterior Spinal Arteries have a more variable origin. In about 25% of cases, they arise directly from the Vertebral arteries. However, in the majority of cases (75%), they arise from the Posterior Inferior Cerebellar Artery (PICA), which is itself a branch of the Vertebral artery. They then descend along the line of the dorsal nerve roots. Therefore, the correct answer is c) Posterior Inferior Cerebellar Artery (PICA).
4. The spinal cord is most vulnerable to ischemic injury in the "Watershed Zone" due to the tenuous blood supply. This zone is typically located in the:
a) Upper Cervical segments (C1-C4)
b) Mid-Thoracic segments (T4-T8)
c) Lumbar enlargement (L1-L5)
d) Sacral segments
Explanation: A watershed area is a region receiving dual blood supply from the most distal branches of two arterial systems, making it susceptible to hypoperfusion. In the spinal cord, the segmental supply is rich in the cervical region (vertebral arteries) and the lumbar region (Artery of Adamkiewicz). However, the Mid-Thoracic region (specifically T4-T8) has relatively few and small segmental feeders. The anterior spinal artery is narrowest here, making this the critical watershed zone prone to infarction during episodes of profound hypotension or shock. Therefore, the correct answer is b) Mid-Thoracic segments (T4-T8).
5. The venous drainage of the spinal cord communicates with the internal vertebral venous plexus. This valveless system connects with the pelvic veins and is a notorious route for the spread of metastasis from:
a) Lung Cancer
b) Prostatic Carcinoma
c) Thyroid Carcinoma
d) Breast Cancer
Explanation: The Internal Vertebral Venous Plexus (Batson's Plexus) is a network of valveless veins located in the epidural space. Because it lacks valves, blood flow can be bidirectional depending on intrathoracic or intra-abdominal pressure. It communicates inferiorly with the pelvic venous plexus and superiorly with the cranial dural sinuses. This creates a direct pathway for cancer cells from pelvic organs, most notably the Prostate, to metastasize to the vertebrae and brain without passing through the lungs (caval system). Therefore, the correct answer is b) Prostatic Carcinoma.
6. The Central (Sulcal) arteries, which supply the central gray matter of the spinal cord, are branches of the:
a) Posterior Spinal Arteries
b) Vasocorona
c) Anterior Spinal Artery
d) Radicular Arteries directly
Explanation: The blood supply to the interior of the cord is centrifugal (from center outward) and centripetal (from surface inward). The Anterior Spinal Artery lies in the anterior median fissure. It gives off numerous small Sulcal Arteries (Central arteries) that penetrate deeply into the fissure and enter the cord substance to supply the central gray matter (anterior and lateral horns) and the base of the posterior horn. These are end-arteries. The Vasocorona supplies the periphery. Therefore, the correct answer is c) Anterior Spinal Artery.
7. Which of the following statements regarding the Anterior Spinal Artery is true?
a) It is a paired structure running along the nerve roots.
b) It supplies the Posterior Columns.
c) It is formed by the union of two branches near the termination of the Basilar artery.
d) It descends in the Anterior Median Fissure.
Explanation: The Anterior Spinal Artery is a single, midline vessel. It is formed by the union of two spinal branches arising from the Vertebral arteries (not Basilar) near the foramen magnum. It descends the entire length of the spinal cord located within the Anterior Median Fissure. It supplies the anterior 2/3 of the cord. The Posterior Spinal arteries are paired. Therefore, the correct answer is d) It descends in the Anterior Median Fissure.
8. In the cervical region, the spinal cord receives segmental blood supply from branches of the Vertebral Artery and the:
a) Internal Carotid Artery
b) Ascending Pharyngeal Artery
c) Ascending Cervical Artery and Deep Cervical Artery
d) Superior Thyroid Artery
Explanation: The cervical spinal cord requires robust blood supply. This is provided by spinal branches arising from the Vertebral Artery. Additionally, segmental feeders arise from the Ascending Cervical Artery (a branch of the Inferior Thyroid artery from the Thyrocervical trunk) and the Deep Cervical Artery (from the Costocervical trunk). These vessels enter the intervertebral foramina to reinforce the longitudinal tracts. This redundancy protects the cervical cord from ischemia more effectively than the thoracic cord. Therefore, the correct answer is c) Ascending Cervical Artery and Deep Cervical Artery.
9. A patient undergoes repair of a Thoracoabdominal Aortic Aneurysm. Post-operatively, they develop paraplegia. This complication is most likely due to the interruption of blood flow from:
a) Intercostal arteries supplying the Adamkiewicz artery
b) The Subclavian artery
c) The Internal Iliac arteries
d) The Celiac Trunk
Explanation: The Artery of Adamkiewicz usually arises from a lower posterior intercostal or upper lumbar artery (T9-L2), which are direct branches of the Thoracic/Abdominal Aorta. During aortic aneurysm repair, these segmental branches may be clamped, ligated, or obstructed by a graft. If the Artery of Adamkiewicz is excluded from the circulation, the extensive lumbar spinal cord supplied by it suffers ischemia, leading to infarction and permanent paraplegia. This is a dreaded complication of aortic surgery. Therefore, the correct answer is a) Intercostal arteries supplying the Adamkiewicz artery.
10. The anastomotic chain connecting the Anterior and Posterior spinal arteries on the surface of the spinal cord is known as the:
a) Circle of Willis
b) Arterial Vasocorona
c) Cruciate Anastomosis
d) Batson's Plexus
Explanation: While the Anterior and Posterior spinal arteries supply specific internal zones, there is a fine network of pial arteries connecting them on the external surface of the cord. This is called the Arterial Vasocorona (or arterial plexus). It sends small penetrating branches to supply the superficial white matter tracts (like the spinocerebellar tracts). This anastomosis provides some collateral circulation, although it is insufficient to prevent infarction if a major longitudinal trunk like the ASA is completely occluded. Therefore, the correct answer is b) Arterial Vasocorona.
Chapter: Neuroanatomy; Topic: Cerebellum; Subtopic: Cerebellar Peduncles and Connections
Key Definitions & Concepts
Inferior Cerebellar Peduncle (ICP): The fiber bundle connecting the medulla oblongata to the cerebellum; it is the primary conduit for vestibular connections.
Juxtarestiform Body: The medial division of the ICP that carries reciprocal connections (both afferent and efferent) between the cerebellum and the vestibular nuclei.
Restiform Body: The lateral division of the ICP containing primarily afferent fibers (e.g., dorsal spinocerebellar, cuneocerebellar tracts) entering the cerebellum.
Cerebellovestibular Fibers: Efferent fibers projecting from the cerebellum (primarily Fastigial nucleus and Flocculonodular lobe) to the vestibular nuclei.
Vestibulocerebellar Fibers: Afferent fibers projecting from the vestibular apparatus and nuclei to the cerebellum (specifically the Flocculonodular lobe).
Flocculonodular Lobe: The phylogenetically oldest part of the cerebellum (Archicerebellum), primarily concerned with equilibrium and eye movements.
Fastigial Nucleus: The deep cerebellar nucleus associated with the vermis and vestibular system; its output travels via the ICP.
Middle Cerebellar Peduncle (MCP): Connects the pons to the cerebellum; contains exclusively afferent pontocerebellar fibers.
Superior Cerebellar Peduncle (SCP): The major output tract to the midbrain/thalamus; contains dentatothalamic fibers.
Truncal Ataxia: Instability of the trunk and gait caused by lesions of the midline vermis or vestibulocerebellum.
[Image of Cerebellar nuclei anatomy]
Lead Question - 2016
Cerebellovestibular fibres pass through ?
a) Superior cerebellar peduncle
b) Middle cerebellar peduncle
c) Inferior cerebellar peduncle
d) None
Explanation: The connections between the cerebellum and the vestibular system traverse the Inferior Cerebellar Peduncle. Anatomically, the Inferior Cerebellar Peduncle is divided into two parts: the Restiform body (lateral, strictly afferent) and the Juxtarestiform body (medial). The Juxtarestiform body carries the Cerebellovestibular fibers (efferent fibers from the Fastigial nucleus and direct fibers from Purkinje cells of the Flocculonodular lobe to the Vestibular nuclei) as well as the Vestibulocerebellar fibers (afferents entering the cerebellum). The Superior peduncle mainly carries output to the thalamus/red nucleus. The Middle peduncle carries input from the pons. Therefore, the correct answer is c) Inferior cerebellar peduncle.
1. Which specific component of the Inferior Cerebellar Peduncle carries the reciprocal connections between the cerebellum and the vestibular nuclei?
a) Restiform body
b) Juxtarestiform body
c) Brachium Pontis
d) Brachium Conjunctivum
Explanation: While "Inferior Cerebellar Peduncle" is the broad answer, detailed neuroanatomy distinguishes its two parts. The lateral part is the Restiform body, which carries afferents like the dorsal spinocerebellar tract. The medial part, located in the wall of the fourth ventricle, is the Juxtarestiform body. This specific bundle is responsible for the bidirectional (reciprocal) communication with the vestibular system, containing both the afferent fibers from the vestibular nuclei and the efferent Cerebellovestibular fibers. Brachium pontis is the MCP; Brachium conjunctivum is the SCP. Therefore, the correct answer is b) Juxtarestiform body.
2. A 60-year-old male presents with acute vertigo, vomiting, and nystagmus. He falls toward the left side. MRI reveals an infarction in the lateral medulla (Wallenberg Syndrome). This lesion damages which peduncle, contributing to the ataxia?
a) Superior Cerebellar Peduncle
b) Middle Cerebellar Peduncle
c) Inferior Cerebellar Peduncle
d) Cerebral Peduncle
Explanation: Lateral Medullary Syndrome (PICA syndrome) involves the dorsolateral medulla. This anatomical region contains the Inferior Cerebellar Peduncle (Restiform body) as it ascends from the medulla into the cerebellum. Damage to this peduncle interrupts the dorsal spinocerebellar tract and vestibulocerebellar connections, leading to ipsilateral hemiataxia and the tendency to fall towards the side of the lesion. The vestibular nuclei themselves are also often involved. The MCP is pontine; the SCP is midbrain. Therefore, the correct answer is c) Inferior Cerebellar Peduncle.
3. The only cerebellar peduncle that contains exclusively afferent fibers (input to the cerebellum) is the:
a) Superior Cerebellar Peduncle
b) Middle Cerebellar Peduncle
c) Inferior Cerebellar Peduncle
d) None, all are mixed
Explanation: The cerebellar connections are organized into three peduncles. The Superior Peduncle is predominantly efferent (output). The Inferior Peduncle is mixed (afferent spinal/vestibular and efferent vestibular). The Middle Cerebellar Peduncle (Brachium Pontis), however, is unique. It consists entirely of fibers arising from the pontine nuclei of the opposite side (Pontocerebellar fibers). These fibers relay information from the cerebral cortex to the cerebellum. Thus, the MCP is considered exclusively afferent. Therefore, the correct answer is b) Middle Cerebellar Peduncle.
4. A child with a medulloblastoma presents with truncal instability and inability to stand without swaying, yet has normal finger-to-nose coordination. This specific type of ataxia suggests damage to the connections of the:
a) Dentate Nucleus
b) Cerebrocerebellum
c) Vestibulocerebellum (Flocculonodular lobe)
d) Pontine nuclei
Explanation: This is the classic presentation of a midline cerebellar lesion. The Vestibulocerebellum (Flocculonodular lobe) and the vermis control the axial musculature for upright posture and equilibrium. Tumors like medulloblastomas typically arise in the midline vermis roof of the fourth ventricle. Compression of the vestibulocerebellum or its outflow (via the Fastigial nucleus and ICP) leads to Truncal Ataxia. The lateral hemispheres (Cerebrocerebellum/Dentate) control limb coordination, which is spared in this scenario. Therefore, the correct answer is c) Vestibulocerebellum (Flocculonodular lobe).
5. The "Uncinate Fasciculus of Russell" is a specific fiber tract that curves over the Superior Cerebellar Peduncle to descend to the vestibular nuclei. It is a component of which outflow?
a) Dentatothalamic tract
b) Fastigiovestibular tract
c) Pontocerebellar tract
d) Rubrospinal tract
Explanation: While most cerebellar output exits via the SCP, the Fastigial Nucleus (associated with the Vestibulocerebellum) sends fibers to the vestibular nuclei via two routes. Some pass directly through the ICP (Juxtarestiform body). Others cross the midline, loop over the Superior Cerebellar Peduncle (forming the Uncinate Fasciculus of Russell), and then descend to the contralateral vestibular nuclei. This is a specific efferent pathway from the Fastigial nucleus regulating vestibular function. Therefore, the correct answer is b) Fastigiovestibular tract.
6. A patient displays "Past-pointing" and "Dysdiadochokinesia" in the right arm. These signs indicate a lesion in the right cerebellar hemisphere, which normally sends correcting signals to the cortex via the:
a) Right Inferior Cerebellar Peduncle
b) Left Middle Cerebellar Peduncle
c) Right Superior Cerebellar Peduncle
d) Left Inferior Cerebellar Peduncle
Explanation: The lateral cerebellar hemisphere (neocerebellum) coordinates voluntary limb movements. Its output travels from the Dentate Nucleus via the Superior Cerebellar Peduncle (SCP). Since the cerebellum controls the ipsilateral body, the Right hemisphere controls the Right arm. The output leaves via the Right SCP, decussates in the midbrain to reach the Left Thalamus/Motor Cortex, which then sends signals down the corticospinal tract (decussating again) to the Right arm. This "double cross" means the cerebellar lesion is ipsilateral to the deficit. Therefore, the correct answer is c) Right Superior Cerebellar Peduncle.
7. Most efferent fibers from the cerebellar cortex synapse on the deep nuclei. However, Purkinje cell axons from which specific area bypass the deep nuclei to project directly to the vestibular nuclei?
a) Anterior Lobe
b) Posterior Lobe
c) Flocculonodular Lobe
d) Tonsils
Explanation: The general rule of cerebellar circuitry is: Cortex -> Deep Nuclei -> Brainstem/Thalamus. There is one major exception. Purkinje cells of the Flocculonodular Lobe (Archicerebellum) and parts of the vermis send their inhibitory GABAergic axons directly to the Vestibular Nuclei in the brainstem, bypassing the deep cerebellar nuclei (though some also go to the Fastigial nucleus). These direct projections pass through the Juxtarestiform body of the Inferior Cerebellar Peduncle. Therefore, the correct answer is c) Flocculonodular Lobe.
8. The Vestibulospinal tracts, which maintain upright posture against gravity, receive strong facilitatory input from the Vestibular nuclei. These nuclei, in turn, are regulated by the cerebellum via the:
a) Inferior Cerebellar Peduncle
b) Middle Cerebellar Peduncle
c) Superior Cerebellar Peduncle
d) Corticospinal tract
Explanation: The Vestibulospinal tracts are under the control of the Vestibular nuclei. The cerebellum modulates the sensitivity and output of these nuclei to maintain balance. The anatomical conduit for this regulation (both the input from the vestibular nerve to the cerebellum and the feedback from the cerebellum to the vestibular nuclei) is the Inferior Cerebellar Peduncle. Damage here results in a loss of this tonic regulation, leading to falling and gait disturbances. The SCP relates to the rubrospinal/corticospinal systems. Therefore, the correct answer is a) Inferior Cerebellar Peduncle.
9. The ventral spinocerebellar tract enters the cerebellum through which peduncle?
a) Superior Cerebellar Peduncle
b) Middle Cerebellar Peduncle
c) Inferior Cerebellar Peduncle
d) None of the above
Explanation: Most afferent tracts (Dorsal Spinocerebellar, Cuneocerebellar, Vestibulocerebellar, Olivocerebellar) enter via the Inferior Cerebellar Peduncle. The Pontocerebellar enters via the Middle. The major exception is the Ventral Spinocerebellar Tract (VSCT). This tract, which conveys information about the state of the spinal cord interneurons ("spy tract"), enters the cerebellum via the Superior Cerebellar Peduncle. It is one of the few afferent tracts in the largely efferent SCP. Therefore, the correct answer is a) Superior Cerebellar Peduncle.
10. A patient exhibits "ocular dysmetria" and gaze-evoked nystagmus. These findings indicate a failure of the cerebellum to coordinate eye movements with head movements, a function primarily involving connections through the:
a) Juxtarestiform Body
b) Restiform Body
c) Brachium Pontis
d) Decussation of SCP
Explanation: Eye movement coordination (e.g., smooth pursuit, VOR suppression) relies heavily on the Vestibulocerebellum (Flocculus/Paraflocculus). The output from these areas modulates the vestibular nuclei to control the oculomotor nuclei (III, IV, VI). These regulatory fibers travel in the Juxtarestiform Body of the Inferior Cerebellar Peduncle. The Restiform body is input. The SCP decussation is for limb control. Dysfunction here leads to the eyes "overshooting" (dysmetria) or inability to hold eccentric gaze (nystagmus). Therefore, the correct answer is a) Juxtarestiform Body.
Chapter: Embryology; Topic: Development of the Nervous System; Subtopic: Derivatives of the Neural Tube and Neural Crest
Key Definitions & Concepts
Neural Tube: The embryonic precursor to the Central Nervous System (Brain and Spinal Cord), Retina, Posterior Pituitary, and Pineal Gland.
Neural Crest Cells: Often called the "fourth germ layer," these migratory cells form the Peripheral Nervous System (DRG, Autonomic ganglia), Schwann cells, Melanocytes, and Adrenal Medulla.
Neurulation: The folding process in vertebrate embryos in which the neural plate is transformed into the neural tube.
Neuropores: Openings at the cranial and caudal ends of the neural tube; failure to close results in neural tube defects (Anencephaly, Spina Bifida).
Alar Plate: The dorsal portion of the neural tube that differentiates into sensory processing structures.
Basal Plate: The ventral portion of the neural tube that differentiates into motor neurons.
Hirschsprung Disease: A congenital condition caused by the failure of neural crest cells to migrate to the distal colon (aganglionic megacolon).
Retina: An outgrowth of the diencephalon, making it a direct derivative of the neural tube, not the neural crest or surface ectoderm.
Schwann Cells: Glial cells of the PNS derived from the Neural Crest (unlike Oligodendrocytes of the CNS, which are Neural Tube derivatives).
Adrenal Medulla: Derived from Neural Crest cells (modified sympathetic ganglia), secreting catecholamines.
Lead Question - 2016
Which of the following is derived from the neural tube except?
a) Retina
b) Brain
c) Dorsal root ganglia
d) Pineal gland
Explanation: The nervous system develops from the ectoderm. The neural plate folds to form the Neural Tube, which gives rise to the Central Nervous System (Brain, Spinal Cord), the Retina (as an optic cup outgrowth), the Posterior Pituitary (Neurohypophysis), and the Pineal Gland. As the neural tube closes, cells at the crest of the folds migrate away; these are the Neural Crest cells. Neural Crest cells give rise to the Peripheral Nervous System, including the Dorsal Root Ganglia, sympathetic chain ganglia, Schwann cells, and adrenal medulla. Therefore, the dorsal root ganglia are neural crest derivatives, not neural tube. The correct answer is c) Dorsal root ganglia.
1. A newborn presents with a tuft of hair over the lumbar spine. Radiological evaluation reveals a bony defect in the vertebral arch. This condition, Spina Bifida Occulta, results from defective closure of the:
a) Cranial Neuropore
b) Caudal Neuropore
c) Neural Crest migration
d) Notochordal process
Explanation: The neural tube closes bi-directionally. The open ends are the neuropores. The Caudal Neuropore is the last to close (around day 27-28). Failure of the caudal neuropore to close completely or induce proper formation of the overlying vertebral arch leads to Spina Bifida. The mildest form, Spina Bifida Occulta, presents with a hairy patch and a vertebral arch defect. Failure of the Cranial Neuropore leads to Anencephaly. Neural crest migration issues cause neurocristopathies like Hirschsprung's. Notochord defects affect the vertebral body/nucleus pulposus. Therefore, the correct answer is b) Caudal Neuropore.
2. Which of the following glial cells is derived from the Neural Crest?
a) Astrocytes
b) Oligodendrocytes
c) Ependymal cells
d) Schwann cells
Explanation: There is a distinct embryological divide between the supporting cells of the CNS and PNS. The glia of the Central Nervous System (Astrocytes, Oligodendrocytes, and Ependymal cells) are derived from the Neuroepithelium of the Neural Tube (Microglia are the exception, derived from Mesoderm/Monocytes). In contrast, the glia of the Peripheral Nervous System, specifically the Schwann cells (and satellite cells of ganglia), are derived from Neural Crest cells that migrate along with the growing axons. This distinction explains why Schwannomas are often associated with other neural crest disorders (e.g., Neurofibromatosis). Therefore, the correct answer is d) Schwann cells.
3. A 2-day-old infant fails to pass meconium. Contrast enema shows a "transition zone" with a dilated proximal colon and a narrow distal segment. Rectal biopsy reveals an absence of ganglion cells in the myenteric plexus. These missing cells are derived from:
a) Neural Tube
b) Surface Ectoderm
c) Neural Crest
d) Endoderm
Explanation: The clinical picture is Hirschsprung Disease (Congenital Aganglionic Megacolon). The enteric nervous system (Meissner's and Auerbach's plexuses) is formed by Neural Crest cells (specifically vagal and sacral neural crest) that migrate into the gut wall during development. In Hirschsprung disease, these cells fail to migrate to the distal-most part of the colon (rectum/sigmoid). Without these inhibitory ganglion cells, the smooth muscle cannot relax, leading to obstruction and proximal dilation. Since the ganglion cells are neural crest derivatives, this is a neurocristopathy. Therefore, the correct answer is c) Neural Crest.
4. The adrenal gland has a dual embryological origin. The cells of the Adrenal Medulla, which secrete epinephrine, are derived from:
a) Intermediate Mesoderm
b) Neural Crest
c) Surface Ectoderm
d) Endoderm
Explanation: The adrenal cortex and medulla have different origins. The Adrenal Cortex is derived from the Intermediate Mesoderm (coelomic epithelium). The Adrenal Medulla is effectively a modified sympathetic ganglion. It contains Chromaffin cells which are postganglionic sympathetic neurons that have lost their axons and secrete catecholamines directly into the blood. As sympathetic ganglia are derivatives of the Neural Crest, the adrenal medulla is also a neural crest derivative. This explains why Pheochromocytomas (tumors of the medulla) are linked to other neural crest tumors (MEN syndromes). Therefore, the correct answer is b) Neural Crest.
5. The retina of the eye is embryologically an extension of the:
a) Surface Ectoderm
b) Mesoderm
c) Diencephalon (Neural Tube)
d) Neural Crest
Explanation: The eye development involves multiple layers. The lens and corneal epithelium come from Surface Ectoderm. The choroid and sclera come from Mesoderm/Neural Crest. However, the neural sensory layer, the Retina, and the optic nerve are direct outgrowths of the forebrain (specifically the Diencephalon). They form as optic grooves, then optic vesicles, and finally optic cups. Since the diencephalon is part of the Neural Tube, the retina is considered a Neural Tube derivative (part of the CNS). This is why the optic nerve is ensheathed by meninges. Therefore, the correct answer is c) Diencephalon (Neural Tube).
6. A patient presents with a white forelock of hair, heterochromia iridis (different colored eyes), and congenital sensorineural deafness. This condition (Waardenburg Syndrome) involves a defect in cells derived from:
a) Neural Crest
b) Neural Tube
c) Otic Placode
d) Pharyngeal Endoderm
Explanation: Waardenburg Syndrome is a classic Neurocristopathy. Neural Crest cells differentiate into Melanocytes, which migrate to the skin, hair follicles, iris, and the stria vascularis of the inner ear. A defect in neural crest migration or survival leads to depigmentation (white forelock, pale/mismatched eyes) and cochlear dysfunction (deafness due to lack of melanocytes in the stria vascularis). While the inner ear sensory cells come from the Otic placode, the melanocyte defect causing the syndrome is Neural Crest. Therefore, the correct answer is a) Neural Crest.
7. The motor neurons of the spinal cord (anterior horn cells) develop from which zone of the developing neural tube?
a) Alar Plate
b) Basal Plate
c) Roof Plate
d) Floor Plate
Explanation: The neural tube is divided functionally by the Sulcus Limitans. The dorsal region is the Alar Plate, which processes sensory information (dorsal horn). The ventral region is the Basal Plate, which differentiates into motor neurons (ventral/anterior horn). The Roof plate and Floor plate are primarily signaling centers (producing BMP and SHH respectively) that guide this dorsal-ventral patterning but do not give rise to the neurons themselves. Thus, all somatic and autonomic motor neurons in the CNS arise from the Basal Plate. Therefore, the correct answer is b) Basal Plate.
8. Which of the following skeletal structures is derived from the Neural Crest (Ectomesenchyme) rather than Mesoderm?
a) Vertebrae
b) Ribs
c) Mandible
d) Femur
Explanation: Generally, bones and connective tissue are derived from Mesoderm. However, in the head and neck, Neural Crest cells migrate into the pharyngeal arches and form the "Ectomesenchyme." This ectomesenchyme gives rise to the bones of the face (viscerocranium), including the maxilla, Mandible, zygomatic bone, and the auditory ossicles. The bones of the skull base and the post-cranial skeleton (vertebrae, ribs, limbs) are derived from Paraxial Mesoderm (Somites) and Lateral Plate Mesoderm. Therefore, the correct answer is c) Mandible.
9. The Parafollicular C cells of the thyroid gland, which secrete Calcitonin, migrate into the thyroid from the Ultimobranchial body. These cells are originally derived from:
a) Thyroid diverticulum (Endoderm)
b) Neural Crest
c) Paraxial Mesoderm
d) 3rd Pharyngeal Pouch
Explanation: The thyroid follicular cells are endodermal, arising from the foramen cecum. However, the Parafollicular C cells have a different origin. They are Neural Crest derivatives that migrate into the Ultimobranchial body (from the 4th/5th pharyngeal pouch) and then become incorporated into the thyroid gland. This embryological origin is clinically significant because Medullary Thyroid Carcinoma (a tumor of C cells) is often associated with MEN syndromes, which involve other neural crest tumors like Pheochromocytoma. Therefore, the correct answer is b) Neural Crest.
10. A pregnant woman takes high doses of isotretinoin (Vitamin A) for acne. This drug is highly teratogenic primarily because it disrupts the expression of HOX genes, affecting the migration and development of:
a) Neural Tube
b) Neural Crest cells
c) Notochord
d) Intermediate Mesoderm
Explanation: Retinoic acid is a crucial signaling molecule for anterior-posterior patterning. High levels of Isotretinoin are toxic to Neural Crest cells (Cranial Neural Crest). This leads to a characteristic pattern of birth defects known as Retinoic Acid Embryopathy, which includes craniofacial dysmorphism (microtia, cleft palate), thymic aplasia, and cardiac outflow tract defects (conotruncal defects). All these structures (face, thymus stroma, conotruncal septum) rely on cranial neural crest contribution. While neural tube defects can occur with folate deficiency, retinoids specifically target the crest. Therefore, the correct answer is b) Neural Crest cells.
Chapter: Neuroanatomy; Topic: Cerebrum (Telencephalon); Subtopic: Sulci and Gyri: Classification and Anatomy
Key Definitions & Concepts
Sulcus (plural: Sulci): A groove or depression on the surface of the brain separating the gyri (ridges).
Complete Sulcus: A sulcus that is extremely deep, causing an indentation or elevation in the wall of the underlying lateral ventricle. Examples: Calcarine sulcus, Collateral sulcus.
Incomplete Sulcus: Superficial grooves that do not indent the ventricular walls (most sulci fall into this category, e.g., Paracentral).
Limiting Sulcus: A sulcus that separates two areas of cerebral cortex with different functional or histological structures (e.g., Central Sulcus separating motor and sensory cortex).
Axial Sulcus: A sulcus that develops along the long axis of a rapidly growing area (e.g., Posterior calcarine sulcus in the visual cortex).
Operculated Sulcus: A sulcus that contains a submerged gyrus within its lips (e.g., Lunate sulcus).
Calcarine Avis: The elevation in the medial wall of the posterior horn of the lateral ventricle created by the deep Calcarine sulcus.
Collateral Eminence: The elevation in the floor of the inferior horn of the lateral ventricle created by the deep Collateral sulcus.
Primary Visual Cortex (V1): Located on the superior and inferior banks (lips) of the Calcarine sulcus (Area 17).
Central Sulcus of Rolando: A limiting sulcus separating the frontal and parietal lobes; it does not cut the ventricle.
Lead Question - 2016
Which of the following is a complete sulcus in the brain?
a) Calcarine sulcus
b) Paracentral sulcus
c) Both
d) None
Explanation: Sulci of the brain are classified based on their depth and relation to the ventricles. A Complete Sulcus is one that is deep enough to produce an elevation in the wall of the adjacent lateral ventricle. There are two classic examples of complete sulci: the Calcarine Sulcus and the Collateral Sulcus. The Calcarine sulcus produces an elevation known as the Calcarine Avis in the posterior horn of the lateral ventricle. The Collateral sulcus produces the Collateral Eminence in the inferior horn. The Paracentral sulcus is an incomplete sulcus as it does not indent the ventricle. Therefore, the correct answer is a) Calcarine sulcus.
1. The Collateral Sulcus is a deep complete sulcus. The indentation it produces in the floor of the inferior horn of the lateral ventricle is known as the:
a) Hippocampus
b) Calcarine Avis
c) Collateral Eminence
d) Amygdala
Explanation: The Collateral Sulcus runs along the inferior surface of the temporal lobe, separating the Parahippocampal gyrus from the Medial Occipitotemporal (Fusiform) gyrus. Because it is a complete sulcus, it deeply invaginates the brain substance. This invagination pushes into the floor of the inferior horn of the lateral ventricle, creating a triangular elevation called the Collateral Eminence. This eminence is located lateral to the Hippocampus. The Calcarine Avis is in the posterior horn. The Hippocampus itself is formed by the Hippocampal sulcus. Therefore, the correct answer is c) Collateral Eminence.
2. The Central Sulcus of Rolando separates the primary motor cortex from the primary somatosensory cortex. Morphologically, it is classified as which type of sulcus?
a) Complete sulcus
b) Axial sulcus
c) Limiting sulcus
d) Operculated sulcus
Explanation: A Limiting Sulcus is defined as a groove that separates two areas of the cortex that differ in function and cytoarchitecture (cellular structure). The Central Sulcus is the classic example of a limiting sulcus. It separates the Precentral Gyrus (Motor cortex, Brodmann area 4, agranular cortex) from the Postcentral Gyrus (Sensory cortex, Brodmann area 3, 1, 2, granular cortex). It creates a clear biological boundary. It is not a complete sulcus because it does not indent the ventricle. Axial sulci develop within a functional area (e.g., posterior calcarine). Therefore, the correct answer is c) Limiting sulcus.
3. The Lunate sulcus, found in primate brains and occasionally in humans, is an example of an Operculated sulcus. This means:
a) It is interrupted by a gyrus crossing it
b) It separates two lobes
c) Its lips are separated by a submerged gyrus
d) It is very shallow
Explanation: An Operculated Sulcus is a distinct morphological type. In these sulci, the two "lips" or banks of the sulcus (the opercula) do not meet at the bottom of a simple groove. Instead, there is a third, submerged gyrus hidden deep within the sulcus, acting as a floor between the surface lips. The Lunate sulcus in the occipital lobe is a classic example. The Central sulcus is NOT operculated; its walls are continuous. This morphology is often a result of rapid cortical expansion in specific areas during development. Therefore, the correct answer is c) Its lips are separated by a submerged gyrus.
4. A 60-year-old patient presents with a visual field defect. MRI confirms an infarct involving the cortex lining the Calcarine sulcus. Anatomically, the Calcarine sulcus joins which other major sulcus at an acute angle to form a "Y" shape on the medial surface?
a) Cingulate sulcus
b) Parieto-occipital sulcus
c) Callosal sulcus
d) Central sulcus
Explanation: On the medial surface of the hemisphere, the Parieto-occipital sulcus descends downwards and forwards to join the Calcarine sulcus. This junction occurs posterior to the splenium of the corpus callosum. The area of cortex wedged between these two sulci is triangular and is called the Cuneus (wedge). The area anterior to the parieto-occipital sulcus is the Precuneus. The area inferior to the calcarine sulcus is the Lingual gyrus. This Y-shaped landmark is crucial for identifying the occipital lobe boundaries on MRI. Therefore, the correct answer is b) Parieto-occipital sulcus.
5. Which sulcus is considered an "Axial Sulcus" because it develops along the long axis of a rapidly growing functional area (the primary visual cortex)?
a) Anterior Calcarine Sulcus
b) Posterior Calcarine Sulcus
c) Central Sulcus
d) Superior Temporal Sulcus
Explanation: The Calcarine sulcus has two parts with different developmental classifications. The Anterior Calcarine sulcus is a Complete sulcus. The Posterior Calcarine Sulcus is classified as an Axial Sulcus. An axial sulcus develops along the long axis of a specific functional area. In this case, the striate cortex (Primary Visual Area 17) expands rapidly, and the posterior calcarine sulcus forms down the middle of this area. Consequently, the visual cortex is found on both banks (upper and lower) of the posterior calcarine sulcus. Therefore, the correct answer is b) Posterior Calcarine Sulcus.
6. A patient with a tumor in the posterior horn of the lateral ventricle has compression of the medial wall. Which anatomical structure forms the bulge on the medial wall of this horn?
a) Bulb of the Posterior Horn
b) Calcarine Avis
c) Hippocampus
d) Both a and b
Explanation: The medial wall of the posterior horn of the lateral ventricle presents two distinct longitudinal elevations. The superior elevation is the Bulb of the Posterior Horn, formed by fibers of the Forceps Major (from the splenium of the corpus callosum). The inferior elevation is the Calcarine Avis, formed by the invagination of the deep Calcarine Sulcus (a complete sulcus). The Hippocampus is in the inferior horn, not the posterior horn. Therefore, both the bulb and the calcarine avis contribute to the medial wall features. Therefore, the correct answer is d) Both a and b.
7. During a neurosurgical procedure to remove an epileptic focus, the surgeon identifies the Central Sulcus. Which MRI sign is most reliable for identifying the Central Sulcus in the axial plane?
a) The "M" sign of the frontal gyrus
b) The "Omega" or "Hand Knob" sign
c) The "S" sign of the Sylvian fissure
d) The "C" sign of the corpus callosum
Explanation: Identifying the Central Sulcus is vital to avoid damaging the motor cortex. The most reliable anatomical landmark on axial MRI is the "Omega sign" (or inverted Omega/Epsilon sign). This is a knob-like, posterior-pointing shape of the Precentral Gyrus that protrudes into the Central Sulcus. This specific area corresponds to the "Hand Knob"—the motor representation of the hand. It is a robust landmark for locating the primary motor cortex and defining the Central Sulcus adjacent to it. Therefore, the correct answer is b) The "Omega" or "Hand Knob" sign.
8. The Cingulate Sulcus runs parallel to the corpus callosum. Posteriorly, it turns upwards to terminate as the "Marginal Sulcus." This marginal portion sits immediately posterior to the:
a) Paracentral Lobule
b) Precuneus
c) Cuneus
d) Frontal Pole
Explanation: The Cingulate Sulcus separates the Cingulate Gyrus (below) from the Frontal gyri (above). As it courses posteriorly, it turns upwards (dorsally) to reach the superior margin of the hemisphere. This upturned end is called the Marginal Sulcus (or Pars Marginalis). The area of cortex bounded by the Cingulate sulcus below, the margin of the brain above, and the Marginal sulcus posteriorly is the Paracentral Lobule (motor/sensory for the leg). Thus, the Marginal sulcus defines the posterior border of the Paracentral Lobule. The Precuneus lies posterior to the Marginal sulcus. Therefore, the correct answer is a) Paracentral Lobule.
9. Uncal herniation (tentorial herniation) compresses the structures medial to the temporal lobe. Which sulcus defines the lateral boundary of the Uncus/Parahippocampal gyrus, separating it from the Fusiform gyrus?
a) Hippocampal sulcus
b) Rhinal sulcus
c) Collateral sulcus
d) Occipitotemporal sulcus
Explanation: The medial aspect of the temporal lobe contains the Parahippocampal Gyrus and its anterior hook, the Uncus. The sulcus that bounds this gyrus laterally is the Collateral Sulcus (and its anterior extension, the Rhinal Sulcus). This sulcus separates the "limbic" temporal lobe (Parahippocampal gyrus) from the neocortical temporal lobe (Fusiform gyrus). The Collateral sulcus is a complete sulcus, forming the Collateral Eminence in the ventricle. In uncal herniation, the uncus (medial to this sulcus) slides over the tentorium edge. Therefore, the correct answer is c) Collateral sulcus.
10. Which of the following sulci is typically the first to appear during fetal development?
a) Central Sulcus
b) Superior Temporal Sulcus
c) Hippocampal Sulcus / Primary fissures
d) Intraparietal Sulcus
Explanation: Brain development involves gyrification. The first indentations to appear (around 8-10 weeks gestation) are the primary fissures or grooves that fold the entire neural wall. The Hippocampal Sulcus (and the Rhinal/Collateral sulci) are among the very first to form as the hemisphere rolls inward to create the hippocampus. The Lateral (Sylvian) fossa also appears early. The Central Sulcus appears much later, around 20 weeks. This early appearance of the medial/limbic sulci underscores their fundamental role in brain architecture. Therefore, the correct answer is c) Hippocampal Sulcus / Primary fissures.
Chapter: Neuroanatomy; Topic: Diencephalon and Ventricular System; Subtopic: The Third Ventricle and Pituitary Relations
Key Definitions & Concepts
Third Ventricle: The slit-like, fluid-filled cavity located in the midline of the diencephalon, between the two thalami and hypothalami.
Infundibulum (Pituitary Stalk): A funnel-shaped hollow process extending downward from the floor of the third ventricle to the posterior pituitary.
Infundibular Recess: A specific extension of the third ventricle cavity into the infundibulum.
Tuber Cinereum: A hollow eminence of gray matter located in the floor of the third ventricle between the optic chiasm and mammillary bodies; the infundibulum attaches here.
Neurohypophysis (Posterior Pituitary): Embryologically derived from the neuroectoderm of the floor of the third ventricle (diencephalon).
Rathke's Pouch: An upward invagination of the oral ectoderm (stomodeum) that forms the Adenohypophysis (Anterior Pituitary).
Colloid Cyst: A benign tumor typically found in the anterior third ventricle that can obstruct the Foramen of Monro.
Lamina Terminalis: A thin sheet of gray matter forming the anterior wall of the third ventricle.
Supraoptic Recess: Another recess of the third ventricle located anteriorly, just above the optic chiasm.
Hypothalamus: Forms the lateral walls and floor of the third ventricle; regulates homeostasis and controls the pituitary.
[Image of Anatomy of third ventricle recesses]
Lead Question - 2016
Infundibular diverticulum is an extention of ?
a) 1st and 2th ventricles
b) 3rd ventricle
c) 4th ventricle
d) None
Explanation: The ventricular system develops from the neural canal. The Third Ventricle is the cavity of the diencephalon. Its floor is formed by hypothalamic structures arranged from anterior to posterior: the Optic Chiasm, the Tuber Cinereum, and the Mammillary bodies. The Infundibulum is a hollow, funnel-like diverticulum (outpouching) arising specifically from the Tuber Cinereum of the floor of the 3rd Ventricle. This diverticulum extends downward to form the stalk of the pituitary and the Pars Nervosa (Posterior Pituitary). The cavity of the third ventricle extends into this stalk as the "Infundibular Recess." Therefore, the correct answer is b) 3rd ventricle.
1. Which of the following is NOT a recess of the Third Ventricle?
a) Supraoptic recess
b) Lateral recess
c) Infundibular recess
d) Pineal recess
Explanation: The cavity of the third ventricle is not a simple slit; it has several evaginations or recesses projecting into surrounding structures. Anteriorly, there are the Supraoptic recess (above the optic chiasm) and the Infundibular recess (into the pituitary stalk). Posteriorly, there are the Pineal recess (into the pineal stalk) and the Suprapineal recess. However, the Lateral recess is a feature of the Fourth Ventricle, extending laterally between the inferior cerebellar peduncle and the brainstem to open as the Foramen of Luschka. The third ventricle does not have a lateral recess. Therefore, the correct answer is b) Lateral recess.
2. A 35-year-old patient presents with severe, intermittent positional headaches (worse when leaning forward) and "drop attacks." MRI reveals a round, well-circumscribed lesion in the anterior roof of the third ventricle. What is the most likely diagnosis?
a) Pituitary Adenoma
b) Craniopharyngioma
c) Colloid Cyst
d) Meningioma
Explanation: This clinical presentation is classic for a Colloid Cyst of the third ventricle. These are benign, mucin-containing cysts that are typically located in the anterior superior part of the third ventricle, immediately adjacent to the Foramen of Monro. Due to a ball-valve mechanism, the cyst can intermittently block the foramen, causing acute obstructive hydrocephalus and severe headache. Leaning forward may shift the cyst, causing sudden obstruction and "drop attacks" (sudden collapse without loss of consciousness). Pituitary adenomas are intrasellar. Craniopharyngiomas are suprasellar but usually calcified/cystic. Therefore, the correct answer is c) Colloid Cyst.
3. Which anatomical structure forms the anterior wall of the Third Ventricle?
a) Thalamus
b) Lamina Terminalis
c) Tuber Cinereum
d) Pineal Gland
Explanation: The boundaries of the third ventricle are precise. The lateral walls are formed by the Thalamus (superiorly) and Hypothalamus (inferiorly). The posterior wall is formed by the pineal gland and posterior commissure. The floor contains the hypothalamus structures. The Anterior wall is formed by the Lamina Terminalis (a thin sheet stretching from the optic chiasm to the anterior commissure), the Anterior Commissure, and the Anterior Columns of the Fornix. The Lamina Terminalis represents the cephalic end of the primitive neural tube. Therefore, the correct answer is b) Lamina Terminalis.
4. A child presents with growth retardation and bitemporal hemianopsia. Imaging shows a calcified, cystic suprasellar mass compressing the floor of the third ventricle. This tumor is derived from remnants of:
a) Neuroectoderm of the diencephalon
b) Neural Crest cells
c) Rathke's Pouch
d) The Notochord
Explanation: The diagnosis is Craniopharyngioma. These are epithelial tumors that arise from the remnants of Rathke's Pouch (the Craniopharyngeal duct). Rathke's pouch is an evagination of the oral ectoderm (stomodeum) that forms the Anterior Pituitary. Remnants can persist along the stalk or in the suprasellar region. These tumors often press upwards into the floor of the third ventricle (hypothalamus) and optic chiasm, causing visual defects and endocrine disturbances (diabetes insipidus, growth failure). They are distinct from the neuroectodermal posterior pituitary. Therefore, the correct answer is c) Rathke's Pouch.
5. The choroid plexus of the third ventricle is located in its:
a) Floor
b) Anterior Wall
c) Roof
d) Lateral Wall
Explanation: The Third Ventricle has a thin, membranous Roof composed of ependyma covered by the Tela Choroidea (a fold of pia mater). The Choroid Plexus of the third ventricle hangs downwards from this roof. It consists of two longitudinal vascular fringes. This plexus is continuous with the choroid plexus of the lateral ventricles through the Foramen of Monro. The floor and lateral walls are formed by solid brain tissue (hypothalamus/thalamus) and do not contain choroid plexus. Therefore, the correct answer is c) Roof.
6. Wernicke's Encephalopathy, often seen in chronic alcoholics due to thiamine deficiency, characteristically causes petechial hemorrhages in which structure found in the floor of the third ventricle?
a) Optic Chiasm
b) Mammillary Bodies
c) Pineal Gland
d) Anterior Commissure
Explanation: The floor of the third ventricle is formed by hypothalamic structures. From anterior to posterior, these are the optic chiasm, tuber cinereum (with infundibulum), and Mammillary bodies. The Mammillary bodies are paired spherical nuclei involved in memory (Papez circuit). They are selectively vulnerable to thiamine (Vitamin B1) deficiency. In acute Wernicke's Encephalopathy, MRI often shows atrophy, hyperintensity, or petechial hemorrhages specifically in the Mammillary Bodies and periaqueductal gray matter. This correlates with the clinical triad of confusion, ataxia, and ophthalmoplegia. Therefore, the correct answer is b) Mammillary Bodies.
7. The connection between the Third Ventricle and the Fourth Ventricle is the:
a) Foramen of Monro
b) Foramen of Luschka
c) Cerebral Aqueduct (of Sylvius)
d) Foramen of Magendie
Explanation: The ventricular system is a continuous pathway. The Lateral Ventricles connect to the Third Ventricle via the Interventricular Foramen (of Monro). The Third Ventricle connects to the Fourth Ventricle via the Cerebral Aqueduct (of Sylvius). This narrow channel traverses the midbrain (mesencephalon), surrounded by the periaqueductal gray matter. Because of its narrowness, it is the most common site of congenital obstruction (Aqueductal Stenosis), leading to non-communicating hydrocephalus. Luschka and Magendie are outflow tracts from the 4th ventricle. Therefore, the correct answer is c) Cerebral Aqueduct (of Sylvius).
8. The Tuber Cinereum, an important landmark in the floor of the third ventricle, gives attachment to the:
a) Pineal Stalk
b) Pituitary Stalk (Infundibulum)
c) Optic Nerve
d) Oculomotor Nerve
Explanation: The Tuber Cinereum is a raised area of gray matter in the interpeduncular fossa, situated between the optic chiasm anteriorly and the mammillary bodies posteriorly. It forms part of the floor of the third ventricle. Its central part projects downwards to form the Infundibulum (Pituitary Stalk), which connects the hypothalamus to the posterior pituitary. The Tuber Cinereum contains important hypothalamic nuclei (tuberal nuclei) involved in regulating the release of anterior pituitary hormones via the portal system. Therefore, the correct answer is b) Pituitary Stalk (Infundibulum).
9. A patient with a pineal region tumor presents with Parinaud's Syndrome (upward gaze palsy). This is due to compression of the posterior commissure, which forms part of the posterior wall of which ventricle?
a) Lateral Ventricle
b) Fourth Ventricle
c) Third Ventricle
d) Fifth Ventricle
Explanation: The Pineal gland projects backwards from the posterior wall of the Third Ventricle. The posterior wall of the third ventricle is formed by the Pineal gland, the Habenular Commissure (superiorly), and the Posterior Commissure (inferiorly). Tumors of the pineal region (pinealomas/germinomas) compress the superior colliculus and the adjacent Posterior Commissure of the midbrain tectum. This compression disrupts the vertical gaze centers, leading to the inability to look up (Parinaud's Syndrome), along with obstructive hydrocephalus by compressing the nearby aqueduct. Therefore, the correct answer is c) Third Ventricle.
10. The Interthalamic Adhesion (Massa Intermedia) connects the two thalami across the third ventricle. Approximately what percentage of healthy humans lack this structure?
a) 0% (Present in everyone)
b) 20-30%
c) 80-90%
d) 100% (It is pathological)
Explanation: The Interthalamic Adhesion (Massa Intermedia) is a band of gray matter that bridges the third ventricle, connecting the medial surfaces of the two thalami. It is often visualized on MRI. However, it is not a neuronal commissure (no fibers cross between sides) and has little functional significance in humans. Interestingly, it is absent in a significant portion of the population, estimated at roughly 20-30% of healthy individuals. Its absence is an anatomical variation, not a pathology. Therefore, the correct answer is b) 20-30%.
Chapter: Neuroanatomy; Topic: Brainstem; Subtopic: Fourth Ventricle and Locus Coeruleus
Key Definitions & Concepts
Substantia Ferruginea: An older anatomical term (meaning "rusty substance") describing the bluish-gray pigmented area seen in the floor of the fourth ventricle.
Locus Coeruleus (LC): The modern name for the Substantia Ferruginea; the principal site for brain synthesis of norepinephrine (noradrenaline).
Fourth Ventricle: The fluid-filled cavity located dorsal to the pons and medulla and ventral to the cerebellum; its floor is the Rhomboid Fossa.
Neuromelanin: A dark pigment formed as a byproduct of catecholamine (norepinephrine/dopamine) synthesis, giving the LC its characteristic color.
Superior Fovea: A small depression in the upper part of the floor of the fourth ventricle; the LC is located just deep to this area.
Sulcus Limitans: The groove separating motor (medial) and sensory (lateral) nuclei in the brainstem; the LC lies lateral to this sulcus.
Norepinephrine: The primary neurotransmitter of the postganglionic sympathetic nervous system and central stress response, synthesized in the LC.
Reticular Activating System (RAS): The network of neurons involved in arousal and sleep-wake transitions; the LC is a major component promoting wakefulness.
Facial Colliculus: A bump in the floor of the fourth ventricle formed by facial nerve fibers looping around the abducens nucleus; located medial to the superior fovea.
Opioid Withdrawal: A clinical state characterized by autonomic hyperactivity, largely driven by rebound firing of the Locus Coeruleus neurons.
[Image of Floor of fourth ventricle anatomy]
Lead Question - 2016
Substantia ferruginea is found in -
a) Fourth ventricle
b) Thalamus
c) Midbrain
d) Third ventricle
Explanation: The term Substantia Ferruginea is the classical anatomical name for the pigmented region known as the Locus Coeruleus ("Blue Spot"). Anatomically, this structure is located in the dorsal tegmentum of the upper pons. When viewing the gross anatomy of the brainstem, it appears as a faint bluish-gray patch on the floor of the Fourth ventricle. Specifically, it is situated in the superior fovea of the rhomboid fossa, lateral to the sulcus limitans. It is distinct from the Substantia Nigra (midbrain) and the Thalamus (diencephalon). Therefore, the correct answer is a) Fourth ventricle.
1. The neurons of the Substantia Ferruginea (Locus Coeruleus) are the primary source of which neurotransmitter in the central nervous system?
a) Dopamine
b) Serotonin
c) Norepinephrine
d) Acetylcholine
Explanation: The Locus Coeruleus is a small nucleus with a massive projection system. It contains the largest collection of noradrenergic neurons in the CNS. These neurons synthesize Norepinephrine (Noradrenaline) from tyrosine. The axons from these neurons project diffusely to almost every part of the brain, including the cortex, thalamus, hypothalamus, cerebellum, and spinal cord. This wide reach allows the LC to regulate global states such as arousal, attention, and the stress response. Dopamine comes from the Substantia Nigra/VTA. Serotonin comes from the Raphe nuclei. Therefore, the correct answer is c) Norepinephrine.
2. A 30-year-old heroin addict presents to the ER in a state of severe withdrawal, exhibiting anxiety, tremors, sweating, and tachycardia. These symptoms are primarily mediated by the hyperactivity of neurons in the:
a) Nucleus Accumbens
b) Locus Coeruleus
c) Substantia Nigra
d) Raphe Nuclei
Explanation: Opioids acutely inhibit the firing of the Locus Coeruleus (LC). With chronic use, the LC neurons undergo compensatory adaptations to maintain normal firing rates despite this inhibition (tolerance). When opioids are suddenly withdrawn, the inhibitory "brake" is removed, and the upregulated LC neurons fire excessively. This "noradrenergic storm" causes the autonomic hyperactivity symptoms of withdrawal (sweating, agitation, tachycardia). Clonidine (an alpha-2 agonist) is often used to treat these symptoms by dampening LC firing. Therefore, the correct answer is b) Locus Coeruleus.
3. The dark pigmentation of the Substantia Ferruginea, visible to the naked eye in adult brain specimens, is due to the accumulation of:
a) Lipofuscin
b) Melanin (Neuromelanin)
c) Hemosiderin
d) Bilirubin
Explanation: Similar to the Substantia Nigra in the midbrain, the Locus Coeruleus (Substantia Ferruginea) contains pigmented neurons. This pigment is Neuromelanin. Neuromelanin is a byproduct of the synthesis of catecholamines (specifically the polymerization of oxidation products of norepinephrine and dopamine). It accumulates within the neurons over age. In children, the LC is not visibly pigmented; the color becomes apparent in adolescence and deepens in adulthood. Loss of this pigment is a pathological sign in neurodegenerative diseases. Therefore, the correct answer is b) Melanin (Neuromelanin).
4. A 70-year-old patient with Alzheimer’s disease passes away. Autopsy reveals significant neuronal loss and depigmentation in the brainstem. Which nucleus is typically one of the earliest sites of tau pathology and neuronal loss in Alzheimer's?
a) Inferior Olivary Nucleus
b) Locus Coeruleus
c) Red Nucleus
d) Vestibular Nuclei
Explanation: While the hippocampus is famous for Alzheimer's pathology, neuropathological staging shows that the Locus Coeruleus is often the very first site to develop neurofibrillary tangles (hyperphosphorylated tau), even before cortical symptoms appear. As the disease progresses, there is a substantial loss of LC neurons (up to 70%). The loss of norepinephrine contributes to the cognitive decline, attention deficits, and sleep-wake disturbances seen in Alzheimer's patients. This contrasts with Parkinson's, where Substantia Nigra loss is the hallmark (though LC is affected there too). Therefore, the correct answer is b) Locus Coeruleus.
5. Anatomically, the Locus Coeruleus is located in the pontine tegmentum. It lies immediately lateral to which anatomical landmark on the floor of the fourth ventricle?
a) Facial Colliculus
b) Sulcus Limitans
c) Median Sulcus
d) Striae Medullares
Explanation: The floor of the fourth ventricle (Rhomboid Fossa) has a midline groove (Median Sulcus). Lateral to this is the medial eminence (motor area). The medial eminence is bounded laterally by the Sulcus Limitans. The Locus Coeruleus lies in the superior fovea, which is at the upper end of the Sulcus Limitans. Thus, the LC is located lateral to the motor nuclei (like the facial colliculus) and sits at the boundary between motor and sensory plates. Therefore, the correct answer is b) Sulcus Limitans.
6. A patient is prescribed a medication for narcolepsy that acts by increasing wakefulness. The mechanism involves stimulating the ascending reticular activating system. The Locus Coeruleus contributes to this system by projecting to the cortex via the:
a) Dorsal tegmental bundle and Medial forebrain bundle
b) Pyramidal tract
c) Spinothalamic tract
d) Fornix
Explanation: The Locus Coeruleus is a critical node in the "Ascending Reticular Activating System" (ARAS). Its norepinephrine output promotes arousal and wakefulness. These fibers ascend through the brainstem via the central tegmental tract, Dorsal Tegmental Bundle, and join the Medial Forebrain Bundle to reach the thalamus, hypothalamus, and the entire cerebral cortex. Silencing of LC neurons is associated with REM sleep (REM-off cells). Therefore, drugs promoting wakefulness often target noradrenergic signaling in these pathways. Therefore, the correct answer is a) Dorsal tegmental bundle and Medial forebrain bundle.
7. Which enzyme is the rate-limiting step in the synthesis of the neurotransmitter found in the Substantia Ferruginea?
a) Tryptophan Hydroxylase
b) Tyrosine Hydroxylase
c) Dopamine Beta-Hydroxylase
d) Choline Acetyltransferase
Explanation: The neurotransmitter is Norepinephrine. The synthesis pathway is: Tyrosine -> DOPA -> Dopamine -> Norepinephrine. The first step, conversion of Tyrosine to DOPA, is catalyzed by Tyrosine Hydroxylase (TH). This is the rate-limiting step for all catecholamines (Dopamine, NE, Epinephrine). While Dopamine Beta-Hydroxylase is unique to NE neurons (converting Dopamine to NE), the rate-limiting regulatory step for the whole pathway is TH. Tryptophan hydroxylase is for Serotonin. ChAT is for Acetylcholine. Therefore, the correct answer is b) Tyrosine Hydroxylase.
8. A combat veteran suffers from Post-Traumatic Stress Disorder (PTSD) with hyperarousal and startle responses. Functional imaging would likely show abnormal reactivity in the amygdala and which brainstem nucleus?
a) Edinger-Westphal Nucleus
b) Locus Coeruleus
c) Nucleus Ambiguus
d) Inferior Salivatory Nucleus
Explanation: The Locus Coeruleus is the "alarm center" of the brain. It is activated by stress and fear stimuli. It has bidirectional connections with the Amygdala. In anxiety disorders and PTSD, the Locus Coeruleus becomes sensitized and hyper-responsive. This leads to chronic hyperarousal, vigilance, and exaggerated startle responses mediated by excessive norepinephrine release in the limbic system and cortex. The other nuclei listed are parasympathetic (rest and digest) and would not drive the hyperarousal state. Therefore, the correct answer is b) Locus Coeruleus.
9. The floor of the fourth ventricle is divided into symmetrical halves by the:
a) Sulcus Limitans
b) Median Sulcus
c) Striae Medullares
d) Obex
Explanation: The floor of the fourth ventricle, the Rhomboid Fossa, is a diamond-shaped depression on the dorsal surface of the pons and medulla. It is strictly bilateral. The structure that runs longitudinally down the exact center, dividing it into right and left halves, is the Median Sulcus. The Sulcus Limitans is lateral to the median sulcus, separating motor and sensory areas. The Striae Medullares are fibers running transversely across the floor. The Obex is the inferior point of the ventricle. Therefore, the correct answer is b) Median Sulcus.
10. In REM sleep Behavior Disorder (RBD), patients physically act out their dreams due to a loss of muscle atonia. This condition involves degeneration of nuclei in the pons, including the:
a) Subcoeruleus / Locus Coeruleus complex
b) Nucleus Tractus Solitarius
c) Superior Olivary Nucleus
d) Dentate Nucleus
Explanation: Normal REM sleep involves vivid dreaming accompanied by muscle paralysis (atonia) to prevent acting out. This atonia is mediated by a circuit involving the Sublaterodorsal nucleus (SLD) and the Locus Coeruleus/Subcoeruleus complex in the pons, which project to the spinal cord to inhibit motor neurons. Degeneration of these specific pontine populations disrupts the atonia, leading to REM Sleep Behavior Disorder (RBD). RBD is a strong prodromal marker for alpha-synucleinopathies like Parkinson's disease and Lewy Body Dementia. Therefore, the correct answer is a) Subcoeruleus / Locus Coeruleus complex.
Chapter: Neuroanatomy; Topic: White Matter Tracts; Subtopic: Anatomy of the Internal Capsule
Key Definitions & Concepts
Internal Capsule: A compact band of white matter fibers (projection fibers) connecting the cerebral cortex with the brainstem and spinal cord.
Anterior Limb: Located between the Head of the Caudate Nucleus and the Lentiform Nucleus; carries frontopontine and thalamocortical fibers.
Genu: The "knee" or bend of the capsule located between the Anterior and Posterior limbs; carries the Corticobulbar tract (motor to face).
Posterior Limb: Located between the Thalamus and the Lentiform Nucleus; carries the Corticospinal tract (motor to body) and sensory radiations.
Retrolentiform Part: The portion of the capsule posterior to the Lentiform Nucleus; carries Optic Radiations (Visual pathway).
Sublentiform Part: The portion passing inferior to the Lentiform Nucleus; carries Auditory Radiations to the temporal lobe.
Lenticulostriate Arteries: Branches of the MCA supplying the upper part of the internal capsule; rupture leads to classic hypertensive strokes.
Anterior Choroidal Artery: A branch of the ICA supplying the inferior aspect of the Posterior Limb; occlusion causes contralateral hemiplegia.
Corona Radiata: The fan-shaped arrangement of fibers above the internal capsule radiating to the cortex.
Pure Motor Stroke: A lacunar infarct usually involving the Posterior Limb of the Internal Capsule, causing contralateral weakness without sensory loss.
[Image of Basal ganglia nuclei anatomy]
Lead Question - 2016
Internal capsule- All of the following are parts except ?
a) Anterior limb
b) Sublentiform part
c) Retrolentiform
d) Prelentiform
Explanation: The Internal Capsule is a V-shaped band of white matter when viewed in horizontal section. It is anatomically divided into five distinct parts based on its relationship to the Lentiform Nucleus. These are: 1) Anterior Limb (between caudate and lentiform), 2) Genu (the bend), 3) Posterior Limb (between thalamus and lentiform), 4) Retrolentiform part (behind the lentiform nucleus), and 5) Sublentiform part (beneath the lentiform nucleus). There is no anatomical division known as the "Prelentiform" part. Therefore, the correct answer is d) Prelentiform.
1. The Corticobulbar tract, which supplies motor innervation to the cranial nerve nuclei of the brainstem, descends primarily through which part of the Internal Capsule?
a) Anterior Limb
b) Genu
c) Posterior Limb
d) Retrolentiform part
Explanation: The somatotopic organization of motor fibers in the internal capsule is critical for localizing lesions. The motor fibers originate in the precentral gyrus. The fibers destined for the head and face (Corticobulbar tract) pass through the Genu (the bend) of the internal capsule. Fibers for the upper limb, trunk, and lower limb (Corticospinal tract) pass through the anterior two-thirds of the Posterior Limb. Lesions at the Genu typically cause contralateral lower facial weakness and tongue deviation. Therefore, the correct answer is b) Genu.
2. A 60-year-old hypertensive male presents with sudden onset of pure motor hemiparesis affecting the right face, arm, and leg. Sensory examination is normal. The most likely site of the lacunar infarct is:
a) Thalamus
b) Posterior Limb of Internal Capsule
c) Head of Caudate
d) Retrolentiform Internal Capsule
Explanation: A "Pure Motor Hemiparesis" is the most common lacunar syndrome. It implies damage to the pyramidal tract (motor fibers) without involving the nearby sensory pathways (thalamocortical fibers) or visual pathways. The anatomical location where all motor fibers (Face, Arm, Leg) are bundled tightly together is the Posterior Limb of the Internal Capsule. A small infarct here can knock out the entire motor output for the contralateral side. Thalamic strokes are typically sensory. Retrolentiform strokes affect vision. Therefore, the correct answer is b) Posterior Limb of Internal Capsule.
3. The Optic Radiation (Geniculocalcarine tract) fibers, carrying visual information from the LGN to the visual cortex, pass through which part of the Internal Capsule?
a) Genu
b) Sublentiform part
c) Retrolentiform part
d) Anterior Limb
Explanation: Visual fibers leaving the Lateral Geniculate Nucleus (LGN) of the thalamus must reach the occipital cortex. They travel posterior to the Lentiform nucleus. This portion of the internal capsule is called the Retrolentiform part. Damage here results in a contralateral Homonymous Hemianopsia. Note: Some fibers (Meyer's loop) loop temporally (Sublentiform), but the bulk of the radiation is classically associated with the Retrolentiform (or posterior-most) sector for exam purposes, distinguishing it from the auditory fibers. Therefore, the correct answer is c) Retrolentiform part.
4. Which artery is the primary supplier of the ventral portion of the Posterior Limb of the Internal Capsule, occlusion of which causes dense hemiplegia?
a) Recurrent Artery of Heubner
b) Anterior Choroidal Artery
c) Posterior Communicating Artery
d) Callosomarginal Artery
Explanation: The blood supply of the internal capsule is complex. The upper aspect is supplied by the Lenticulostriate branches (MCA). However, the inferior aspect of the Posterior Limb (carrying the dense motor fibers) is supplied by the Anterior Choroidal Artery (a branch of the Internal Carotid). Occlusion of this single vessel can lead to the "Anterior Choroidal Artery Syndrome," characterized by the triad of Contralateral Hemiplegia, Hemianesthesia, and Hemianopsia. Heubner's artery supplies the Anterior Limb. Therefore, the correct answer is b) Anterior Choroidal Artery.
5. The Auditory Radiation fibers connecting the Medial Geniculate Body to the auditory cortex travel via the:
a) Anterior Limb
b) Sublentiform part
c) Genu
d) External Capsule
Explanation: Auditory information is relayed from the Medial Geniculate Body (MGB) to the Primary Auditory Cortex (Heschl's gyrus) in the superior temporal lobe. These fibers must pass inferior to the Lentiform nucleus to reach the temporal lobe. This section of the internal capsule is the Sublentiform part. It is situated below the posterior part of the lentiform nucleus. Lesions here can cause cortical deafness or auditory processing deficits, though unilateral lesions are often asymptomatic due to bilateral auditory representation. Therefore, the correct answer is b) Sublentiform part.
6. Anatomically, the Anterior Limb of the Internal Capsule separates the:
a) Thalamus and Putamen
b) Caudate Nucleus and Lentiform Nucleus
c) Thalamus and Caudate Nucleus
d) Claustrum and Putamen
Explanation: On a horizontal section of the brain, the Anterior Limb of the internal capsule appears as a white band separating two large grey matter masses. Medially lies the Head of the Caudate Nucleus. Laterally lies the Lentiform Nucleus (specifically the Putamen and Globus Pallidus). The Posterior Limb separates the Thalamus (medial) from the Lentiform Nucleus (lateral). The Genu is the angle between the two limbs, pointing towards the interventricular foramen. Therefore, the correct answer is b) Caudate Nucleus and Lentiform Nucleus.
7. The "Charcot-Bouchard" microaneurysms, which are prone to rupture in chronic hypertension causing hemorrhage in the Internal Capsule, form on which arteries?
a) Cortical branches of ACA
b) Lenticulostriate branches of MCA
c) Thalamogeniculate arteries
d) Anterior Communicating Artery
Explanation: The Lenticulostriate arteries are small, perforating branches arising from the M1 segment of the Middle Cerebral Artery. They ascend to supply the basal ganglia and the superior part of the internal capsule. Due to their small size and the high pressure of the system they branch from, they are susceptible to lipohyalinosis and microaneurysm formation (Charcot-Bouchard) in chronic hypertension. Rupture of these vessels leads to a classic hypertensive Intracerebral Hemorrhage (ICH) in the putamen/internal capsule region. Therefore, the correct answer is b) Lenticulostriate branches of MCA.
8. Which descending tract passes through the Anterior Limb of the Internal Capsule?
a) Corticospinal tract
b) Frontopontine fibers
c) Parietopontine fibers
d) Optic Radiations
Explanation: The Anterior Limb contains fewer "vital" motor fibers compared to the Posterior Limb. Its primary contents are the Frontopontine fibers (connecting the frontal cortex to the pontine nuclei) and the Anterior Thalamic Radiations (connecting the medial/anterior thalamic nuclei to the frontal cortex/cingulate). Damage here produces fewer obvious motor deficits but can cause cognitive or behavioral changes (e.g., abulia) and potentially ataxia (frontocerebellar pathway). The Corticospinal tract is in the Posterior Limb. Therefore, the correct answer is b) Frontopontine fibers.
9. Sensory fibers from the Thalamus (VPL and VPM nuclei) carrying touch, pain, and temperature sensation to the Postcentral Gyrus ascend through the:
a) Anterior 1/3 of the Posterior Limb
b) Posterior 1/3 of the Posterior Limb
c) Genu
d) Anterior Limb
Explanation: Within the Posterior Limb of the internal capsule, there is a specific organization. The Anterior two-thirds contain the Corticospinal (motor) fibers. The Posterior one-third of the Posterior Limb (just anterior to the retrolentiform part) contains the Superior Thalamic Radiations. These carry the third-order sensory neurons from the VPL and VPM nuclei of the thalamus to the primary somatosensory cortex (Postcentral Gyrus). A lesion extending posteriorly in the capsule will thus cause hemianesthesia in addition to hemiplegia. Therefore, the correct answer is b) Posterior 1/3 of the Posterior Limb.
10. The white matter structure located lateral to the Lentiform Nucleus is the:
a) Internal Capsule
b) External Capsule
c) Extreme Capsule
d) Claustrum
Explanation: Moving laterally from the midline, the sequence of structures is: Thalamus/Caudate -> Internal Capsule -> Lentiform Nucleus (GP + Putamen) -> External Capsule -> Claustrum -> Extreme Capsule -> Insular Cortex. The External Capsule is a thin sheet of white matter separating the Lentiform Nucleus from the Claustrum. It contains association fibers. It is important to distinguish these layers when viewing MRI scans of the basal ganglia. Therefore, the correct answer is b) External Capsule.
Chapter: Neuroanatomy; Topic: Brainstem and Special Senses; Subtopic: Auditory Pathway and Lateral Lemniscus
Key Definitions & Concepts
Lateral Lemniscus: The primary ascending auditory tract in the brainstem, extending from the Superior Olivary Complex to the Inferior Colliculus.
Inferior Colliculus: The principal midbrain nucleus of the auditory pathway and the site of termination for the lateral lemniscus.
Medial Geniculate Body (MGB): The thalamic relay nucleus for hearing; it receives input from the Inferior Colliculus via the Brachium of the Inferior Colliculus.
Superior Olivary Complex: A collection of nuclei in the pons involved in sound localization (interaural time and intensity differences).
Trapezoid Body: A bundle of decussating fibers in the ventral pons carrying auditory information from the cochlear nuclei to the contralateral Superior Olive/Lateral Lemniscus.
Brachium of Inferior Colliculus: The fiber bundle connecting the Inferior Colliculus to the Medial Geniculate Body.
BERA (Brainstem Evoked Response Audiometry): An electrophysiological test where Wave IV corresponds to the Lateral Lemniscus and Wave V corresponds to the Inferior Colliculus.
Heschl’s Gyrus: The Primary Auditory Cortex (Brodmann Area 41, 42) located in the temporal lobe.
Tonotopic Organization: The spatial arrangement of where sounds of different frequency are processed in the brain; maintained throughout the lateral lemniscus.
Nuclei of Lateral Lemniscus: Clusters of neurons embedded within the tract responsible for processing temporal aspects of sound.
[Image of Auditory pathway diagram]
Lead Question - 2016
Lateral lemniscus terminates into ?
a) Lateral geniculate body
b) Superior colliculus
c) Inferior colliculus
d) Inferior olivary complex
Explanation: The Lateral Lemniscus is the major ascending fiber tract of the auditory system found in the brainstem (specifically the pons and midbrain). It carries auditory information from the Cochlear Nuclei and the Superior Olivary Complex. These fibers ascend through the pons and terminate in the Inferior Colliculus of the midbrain tectum. The Inferior Colliculus then relays the information to the Medial Geniculate Body of the thalamus. The Lateral Geniculate Body is for vision. The Superior Colliculus is for visual reflexes. The Inferior Olivary complex is for cerebellar motor learning. Therefore, the correct answer is c) Inferior colliculus.
1. The fibers connecting the Inferior Colliculus to the Medial Geniculate Body form which anatomical structure?
a) Lateral Lemniscus
b) Brachium of the Superior Colliculus
c) Brachium of the Inferior Colliculus
d) Acoustic Striae
Explanation: The auditory pathway is a multi-synaptic chain. After the Lateral Lemniscus terminates in the Inferior Colliculus (midbrain), the next order neurons project to the thalamus. These projecting fibers form a distinct arm-like ridge on the surface of the midbrain known as the Brachium of the Inferior Colliculus (or Inferior Brachium). This structure connects the Inferior Colliculus to the Medial Geniculate Body (MGB) of the thalamus. The Brachium of the Superior Colliculus is involved in the visual pathway (pupillary reflex). The Acoustic striae are dorsal/intermediate/ventral fibers leaving the cochlear nuclei. Therefore, the correct answer is c) Brachium of the Inferior Colliculus.
2. A patient undergoes Brainstem Evoked Response Audiometry (BERA) to assess hearing loss. The report shows a delayed latency of Wave V. Which structure is the primary generator of Wave V?
a) Cochlear Nerve
b) Superior Olivary Complex
c) Lateral Lemniscus
d) Inferior Colliculus
Explanation: BERA (or ABR) records electrical potentials generated at specific anatomical sites in the auditory pathway. The standard nomenclature identifies 5-7 waves. Wave I is the distal Cochlear nerve; Wave II is the proximal Cochlear nerve/nucleus; Wave III is the Superior Olivary Complex; Wave IV is the Lateral Lemniscus; and Wave V is generated by the Inferior Colliculus. Wave V is the most robust and clinically useful wave for determining hearing thresholds and retrocochlear pathology (like acoustic neuroma or brainstem lesions). Therefore, the correct answer is d) Inferior Colliculus.
3. Unilateral damage to the Lateral Lemniscus in the upper pons results in which clinical finding?
a) Complete deafness in the ipsilateral ear
b) Complete deafness in the contralateral ear
c) Bilateral partial hearing loss and defect in sound localization
d) Pure word deafness
Explanation: The auditory pathway is unique compared to the somatosensory system because of extensive bilateral representation. Fibers from the cochlear nuclei decussate at multiple levels (Trapezoid body, Probst's commissure, Commissure of Inferior Colliculus). Consequently, the Lateral Lemniscus contains fibers from both ears (though predominantly the contralateral ear). Therefore, a unilateral lesion of the Lateral Lemniscus does not cause complete deafness in either ear. Instead, it causes a mild bilateral hearing impairment (worse contralaterally) and a significant inability to localize sound sources (a function of binaural comparison). Therefore, the correct answer is c) Bilateral partial hearing loss and defect in sound localization.
4. The Medial Geniculate Body (MGB) projects to the Primary Auditory Cortex via the:
a) Optic Radiation
b) Auditory Radiation (Sublentiform part of Internal Capsule)
c) Anterior limb of Internal Capsule
d) Fornix
Explanation: The final link in the auditory pathway connects the thalamus to the cortex. Efferent fibers from the Medial Geniculate Body form the Auditory Radiation (also known as the Geniculotemporal tract). These fibers pass laterally through the Sublentiform part (part of the Retrolenticular limb) of the Internal Capsule. They travel ventral to the lentiform nucleus to terminate in the Transverse Temporal Gyri of Heschl (Primary Auditory Cortex) in the superior temporal lobe. Optic radiations (Geniculocalcarine) originate from the LGB. Therefore, the correct answer is b) Auditory Radiation (Sublentiform part of Internal Capsule).
5. Which nucleus of the auditory pathway is the first site where binaural (inputs from both ears) interaction occurs, enabling sound localization?
a) Cochlear Nuclei
b) Superior Olivary Complex
c) Nucleus of Lateral Lemniscus
d) Medial Geniculate Body
Explanation: Sound localization requires comparing the arrival time and intensity of sound between the two ears. The Cochlear nuclei receive input only from the ipsilateral ear. The decussation happens immediately after, via the Trapezoid Body. The Superior Olivary Complex (specifically the Medial Superior Olive for time differences and Lateral Superior Olive for intensity differences) is the first station in the brainstem to receive convergent input from both cochlear nuclei. This convergence allows for the computation of sound location in space. Therefore, the correct answer is b) Superior Olivary Complex.
6. A 55-year-old male presents with difficulty understanding speech in a noisy room. Audiometry shows normal pure tone thresholds. This "Cocktail Party Effect" deficit suggests dysfunction in central auditory processing, potentially involving the:
a) Outer Hair Cells
b) Ossicular chain
c) Olivocochlear bundle and Brainstem pathways
d) Tympanic membrane
Explanation: The ability to filter out background noise to focus on a specific speaker (Cocktail Party Effect) is a central auditory function. It relies on the efferent auditory system, specifically the Olivocochlear bundle (originating in the Superior Olive). This bundle projects back to the cochlea (Outer Hair Cells) to modulate sensitivity and suppress noise. Dysfunction in the brainstem pathways (Superior Olive, Lateral Lemniscus) or the efferent loop leads to difficulty processing speech in noise, despite normal cochlear mechanics (normal audiogram). This is Central Auditory Processing Disorder (CAPD). Therefore, the correct answer is c) Olivocochlear bundle and Brainstem pathways.
7. The Lateral Lemniscus is located in which anatomical part of the brainstem cross-section?
a) Tectum
b) Tegmentum
c) Basis Pontis
d) Pyramid
Explanation: The brainstem is divided ventro-dorsally into the Basis (ventral), Tegmentum (middle), and Tectum (dorsal, only in midbrain). The ascending sensory tracts, including the Medial Lemniscus (touch/proprioception), Spinal Lemniscus (pain/temp), and Lateral Lemniscus (auditory), travel within the Tegmentum of the pons and midbrain. The Lateral Lemniscus is the most lateral of these tracts. The Basis contains descending motor fibers (corticospinal). The Tectum contains the Colliculi. Therefore, the correct answer is b) Tegmentum.
8. The Primary Auditory Cortex (Area 41, 42) is located in the:
a) Superior Temporal Gyrus (Heschl's Gyrus)
b) Middle Temporal Gyrus
c) Angular Gyrus
d) Inferior Frontal Gyrus
Explanation: The cortical termination of the auditory pathway is the Primary Auditory Cortex. It is located on the superior surface of the temporal lobe, buried within the Lateral Sulcus (Sylvian Fissure). These transverse temporal gyri are known as Heschl's Gyri (specifically the anterior one). They correspond to Brodmann Areas 41 and 42. Surrounding this area is the Secondary Auditory Cortex (Wernicke's Area, Area 22) responsible for language comprehension. The Angular Gyrus is visual-spatial-linguistic integration. Therefore, the correct answer is a) Superior Temporal Gyrus (Heschl's Gyrus).
9. The trapezoid body, a commissural bundle of auditory fibers, is located in the:
a) Dorsal Midbrain
b) Ventral Pons
c) Closed Medulla
d) Open Medulla
Explanation: The Trapezoid Body is a prominent transverse band of fibers located in the Ventral Pons (specifically the caudal pons tegmentum). It represents the major decussation of the auditory pathway. Axons from the ventral cochlear nucleus travel ventrally and medially, crossing the midline in the trapezoid body to synapse on the contralateral Superior Olivary Complex or enter the contralateral Lateral Lemniscus. This decussation is crucial for the bilateral representation of hearing. Therefore, the correct answer is b) Ventral Pons.
10. An Acoustic Neuroma (Vestibular Schwannoma) typically arises in the Internal Acoustic Meatus. As it expands into the Cerebellopontine Angle (CPA), which auditory structure is compressed first?
a) Cochlear Nucleus
b) Cochlear Nerve (CN VIII)
c) Lateral Lemniscus
d) Medial Geniculate Body
Explanation: Acoustic Neuromas arise from the Schwann cells of the vestibular portion of the Vestibulocochlear Nerve (CN VIII). The tumor usually starts inside the Internal Acoustic Meatus. As it grows, it compresses the adjacent cochlear nerve fibers within the same bony canal. This leads to the classic symptom of progressive unilateral sensorineural hearing loss and tinnitus. As it expands further into the CPA cistern, it may compress the brainstem (and cochlear nucleus), CN V, and CN VII, but the primary and earliest compression is of the Cochlear Nerve itself. Therefore, the correct answer is b) Cochlear Nerve (CN VIII).
Chapter: Neuroanatomy & Physiology; Topic: Ventricular System; Subtopic: Cerebrospinal Fluid (CSF) Physiology and Pathology
Key Definitions & Concepts
Choroid Plexus: A network of capillaries and specialized ependymal cells located in the ventricles responsible for the production of CSF.
Arachnoid Granulations: Projections of the arachnoid membrane into the dural venous sinuses (mainly Superior Sagittal Sinus) where CSF is reabsorbed into the blood.
Xanthochromia: The yellow discoloration of the CSF supernatant seen in subarachnoid hemorrhage due to the breakdown of red blood cells (bilirubin).
Hydrocephalus: An abnormal accumulation of CSF in the ventricles, caused by obstruction (non-communicating) or impaired absorption (communicating).
Foramen of Monro: The interventricular foramen connecting the Lateral Ventricles to the Third Ventricle.
Aqueduct of Sylvius: The narrow canal connecting the Third Ventricle to the Fourth Ventricle; a common site of obstruction (Aqueductal Stenosis).
Papilledema: Swelling of the optic disc due to raised intracranial pressure; a major contraindication for lumbar puncture.
Blood-Brain Barrier (BBB): The selective barrier formed by tight junctions of capillary endothelial cells, separating blood from the brain extracellular fluid.
Lumbar Puncture: A procedure to collect CSF, typically performed at the L3-L4 or L4-L5 interspace to avoid spinal cord injury.
Queckenstedt’s Test: A clinical maneuver (compressing jugular veins) used to test for spinal block, though rarely used now with MRI availability.
[Image of Ventricular system and CSF flow]
Lead Question - 2016
Total volume of CSF is?
a) 150 ml
b) 500 ml
c) 50 ml
d) 800 ml
Explanation: The cerebrospinal fluid (CSF) is a clear, colorless body fluid found in the brain and spinal cord. In a healthy adult, the total volume of CSF typically ranges between 125 ml and 150 ml. This volume is distributed between the cranial and spinal subarachnoid spaces (~125 ml) and within the ventricles themselves (~25 ml). Despite the relatively small standing volume, the daily production is much higher (about 500 ml/day), meaning the entire CSF volume turns over approximately 3 to 4 times every 24 hours. Therefore, the correct answer is a) 150 ml.
1. Although the total volume is 150 ml, the rate of CSF production in a healthy adult is approximately:
a) 20 ml/hour
b) 0.5 ml/hour
c) 0.35 ml/min
d) 1000 ml/day
Explanation: The production of CSF is a continuous process, primarily driven by active secretion from the choroid plexus in the lateral, third, and fourth ventricles. The rate of production is remarkably constant, roughly 0.3 to 0.4 ml per minute (or about 20 ml per hour). This cumulates to a total daily production of approximately 500 ml to 600 ml. Since the total CSF volume is only about 150 ml, this high production rate ensures that the CSF is constantly renewed and flushed, allowing for the removal of metabolic waste products. Therefore, the correct answer is c) 0.35 ml/min.
2. A 75-year-old male presents with the triad of urinary incontinence, gait ataxia (magnetic gait), and cognitive dysfunction. CT scan shows dilated ventricles without significant cortical atrophy. This condition is caused by impaired absorption of CSF at the:
a) Choroid Plexus
b) Foramen of Magendie
c) Arachnoid Granulations
d) Ependymal lining
Explanation: The clinical triad of "Wet (incontinence), Wobbly (ataxia), and Wacky (dementia)" is classic for Normal Pressure Hydrocephalus (NPH). In this condition, there is a communicating hydrocephalus where CSF flow is not blocked, but its resorption is impaired. The primary site of CSF absorption into the venous system is the Arachnoid Granulations (or villi), which project into the dural venous sinuses (primarily the superior sagittal sinus). When these become fibrosed or dysfunctional (e.g., post-meningitis or idiopathic), CSF accumulates, enlarging the ventricles and stretching the corona radiata fibers. Therefore, the correct answer is c) Arachnoid Granulations.
3. Which of the following best describes the normal biochemical composition of CSF compared to blood plasma?
a) Higher protein, lower glucose
b) Lower protein, higher chloride
c) Equal protein, equal glucose
d) Higher K+, Lower Na+
Explanation: CSF is effectively an ultrafiltrate of plasma, but its ionic and organic composition differs. The most striking difference is the protein concentration; normal CSF protein (15-45 mg/dL) is much lower than plasma protein (6000-8000 mg/dL) because large proteins cannot cross the blood-CSF barrier. Glucose is roughly 60-70% of plasma levels. However, to maintain Gibbs-Donnan equilibrium, certain ions are higher in CSF to compensate for the lack of negatively charged proteins. Specifically, Chloride (and Magnesium) levels are higher in CSF than in plasma. Potassium and Calcium are slightly lower. Therefore, the correct answer is b) Lower protein, higher chloride.
4. A 25-year-old patient presents with severe headache and nuchal rigidity. Lumbar puncture reveals CSF with: low glucose, markedly elevated protein, and a neutrophil-predominant leukocytosis. The most likely etiology is:
a) Viral Meningitis
b) Bacterial Meningitis
c) Fungal Meningitis
d) Tubercular Meningitis
Explanation: CSF analysis is critical for distinguishing meningitis types. In Bacterial Meningitis, bacteria consume glucose and cause significant inflammation and protein leakage. Hence, findings typically include low glucose (hypoglycorrhachia), high protein, and a pleocytosis dominated by Polymorphonuclear cells (Neutrophils). Viral meningitis usually shows normal glucose and lymphocyte predominance. Fungal and Tubercular meningitis also show low glucose but typically present with a lymphocyte predominance, not neutrophils (except in very early TB). Therefore, the correct answer is b) Bacterial Meningitis.
5. The circulation of CSF follows a specific pathway. After leaving the Fourth Ventricle through the foramina of Luschka and Magendie, where does the CSF immediately enter?
a) Superior Sagittal Sinus
b) Cisterna Magna and Subarachnoid Space
c) Central Canal of Spinal Cord only
d) Third Ventricle
Explanation: CSF flows from the lateral ventricles -> Foramen of Monro -> 3rd Ventricle -> Aqueduct of Sylvius -> 4th Ventricle. From the 4th ventricle, it exits the ventricular system via the midline Foramen of Magendie and the two lateral Foramina of Luschka. These apertures open into the Cisterna Magna (cerebellomedullary cistern) and the pontine cistern, which are parts of the Subarachnoid Space. From there, CSF circulates around the brain and spinal cord before being absorbed. It does not go backward to the 3rd ventricle or directly into the sinus. Therefore, the correct answer is b) Cisterna Magna and Subarachnoid Space.
6. A patient is suspected of having a Subarachnoid Hemorrhage (SAH) but the CT scan is normal. A lumbar puncture is performed 12 hours after symptom onset. Which finding would confirm the diagnosis?
a) Clear CSF
b) Xanthochromia
c) Presence of Oligoclonal bands
d) Albuminocytologic dissociation
Explanation: In a traumatic tap (needle hitting a vessel), fresh red blood cells (RBCs) may be seen, but the supernatant (fluid after spinning) remains clear. In a true Subarachnoid Hemorrhage, RBCs have been present in the CSF for hours, allowing them to lyse and break down into oxyhemoglobin and bilirubin. This imparts a yellow or pink hue to the supernatant, known as Xanthochromia. This finding typically develops 2-12 hours after the bleed and is the gold standard for diagnosing SAH in CT-negative cases. Oligoclonal bands are for Multiple Sclerosis. Therefore, the correct answer is b) Xanthochromia.
7. To maintain the Blood-CSF barrier, the epithelial cells of the choroid plexus are joined together by:
a) Gap junctions
b) Desmosomes
c) Tight junctions (Zonula occludens)
d) Hemidesmosomes
Explanation: The Blood-Brain Barrier (BBB) is formed by tight junctions between capillary endothelial cells. However, the capillaries in the choroid plexus are fenestrated (leaky) to allow plasma filtration. To prevent unregulated substances from entering the CSF, the barrier function is moved to the epithelium. The cuboidal ependymal cells of the choroid plexus are connected by apical Tight Junctions (Zonula Occludens). This constitutes the Blood-CSF barrier, ensuring that transport of ions and glucose into the CSF is actively regulated rather than passive. Therefore, the correct answer is c) Tight junctions (Zonula occludens).
8. A patient with a large brain tumor presents with headache, vomiting, and blurring of vision. Fundoscopy reveals bilateral Papilledema. Why is a Lumbar Puncture contraindicated in this patient?
a) Risk of infection
b) Risk of Cerebral Herniation
c) Risk of Dural Sinus Thrombosis
d) Risk of severe hypotension
Explanation: Papilledema indicates raised Intracranial Pressure (ICP). The cranium is a closed box. If there is a mass (tumor) causing high pressure in the cranial compartment, the pressure is transmitted throughout the CSF. If a lumbar puncture is performed, it creates an area of low pressure in the spinal canal. This pressure gradient can cause the brainstem and cerebellar tonsils to be pushed ("sucked") downwards through the Foramen Magnum. This is called Coning or Tonsillar Herniation, which compresses the respiratory centers and is rapidly fatal. Therefore, the correct answer is b) Risk of Cerebral Herniation.
9. The normal specific gravity of Cerebrospinal fluid typically falls within the range of:
a) 1.003 - 1.008
b) 1.010 - 1.025
c) 1.030 - 1.040
d) Equal to water (1.000)
Explanation: Specific gravity compares the density of a fluid to water. Since CSF contains dissolved salts (NaCl), glucose, and a small amount of protein, it is slightly denser than pure water. The normal specific gravity of CSF is generally cited as 1.003 to 1.008 (at body temperature). This value is important in anesthesiology for spinal anesthesia (baricity of the anesthetic agent relative to CSF determines how the drug spreads). Plasma specific gravity is higher (~1.025) due to its high protein content. Therefore, the correct answer is a) 1.003 - 1.008.
10. A newborn is diagnosed with Non-Communicating Hydrocephalus. The obstruction is most commonly located at the:
a) Arachnoid Granulations
b) Foramen of Monro
c) Aqueduct of Sylvius
d) Foramen of Luschka
Explanation: Hydrocephalus is classified as Communicating (impairment outside the ventricles) or Non-Communicating (blockage within the ventricular system). In congenital non-communicating hydrocephalus, the most common site of obstruction is the Aqueduct of Sylvius (Aqueductal Stenosis). This narrow channel connects the third and fourth ventricles. Stenosis here prevents CSF from leaving the third and lateral ventricles, causing them to dilate while the fourth ventricle remains normal in size. Obstruction at the arachnoid granulations causes communicating hydrocephalus. Therefore, the correct answer is c) Aqueduct of Sylvius.
Chapter: Neuroanatomy; Topic: Cerebellum; Subtopic: Internal Architecture and Arbor Vitae
Key Definitions & Concepts
Arbor Vitae: Latin for "Tree of Life"; refers to the distinctive fern-like or tree-like pattern of white matter seen in a sagittal section of the cerebellum.
Folia: The numerous, narrow, leaf-like folds of the cerebellar cortex (gray matter) that surround the central core of white matter.
Vermis: The midline structure of the cerebellum connecting the two hemispheres; the arbor vitae pattern is most classically described in the midsagittal section of the vermis.
White Matter: The inner core of the cerebellum containing myelinated axons (mossy and climbing fibers) and the deep cerebellar nuclei.
Deep Cerebellar Nuclei: Four pairs of gray matter nuclei (Dentate, Emboliform, Globose, Fastigial) embedded within the white matter of the arbor vitae.
Metencephalon: The secondary brain vesicle from which the cerebellum and pons develop.
Fourth Ventricle: The fluid-filled space located anterior to the cerebellum; the arbor vitae lies dorsal to the roof of this ventricle.
Purkinje Cells: Large inhibitory neurons in the cerebellar cortex whose axons travel through the arbor vitae to reach the deep nuclei.
Truncal Ataxia: Instability of the trunk and gait, often resulting from lesions affecting the vermis (midline arbor vitae).
Peduncles: Three pairs of fiber bundles (Superior, Middle, Inferior) that connect the cerebellar white matter to the brainstem.
[Image of Cerebellar nuclei anatomy]
Lead Question - 2016
Arbor vitae are seen in ?
a) Cerebrum
b) Cerebellum
c) Pons
d) Thalamus
Explanation: The term Arbor Vitae (Latin for "Tree of Life") describes the characteristic appearance of the cerebellar white matter in a sagittal or cross-section. The cerebellum consists of an outer layer of gray matter (cortex) folded into tight ridges called folia. These folia surround a central core of white matter that branches outward, resembling the branches of a tree or a fern. This pattern is most distinct in the vermis. While the cerebrum has white matter (centrum semiovale), it does not form this specific tree-like pattern. The Pons contains transverse fibers. The Thalamus is a nuclear mass. Therefore, the correct answer is b) Cerebellum.
1. The branching pattern of the Arbor Vitae is formed by which histological component of the cerebellum?
a) The deep cerebellar nuclei only
b) The gray matter of the cortex
c) The central core of white matter
d) The Purkinje cell layer
Explanation: The cerebellum is organized into an outer cortex of gray matter and an inner core of white matter. The central core of white matter contains myelinated axons carrying afferent input (from mossy and climbing fibers) to the cortex and efferent output (from Purkinje cells) to the deep nuclei. As these white matter tracts ramify and extend outward to reach the highly folded cortex (folia), they create the visual appearance of tree branches. The gray matter forms the "leaves" of the tree. Thus, the structure defining the "arbor" itself is the white matter. Therefore, the correct answer is c) The central core of white matter.
2. A 50-year-old chronic alcoholic presents with gait instability and truncal ataxia. MRI reveals atrophy of the superior vermis. This degeneration specifically affects the anterior lobe and which part of the arbor vitae?
a) Midline white matter
b) Lateral hemispheric white matter
c) Flocculonodular lobe
d) Dentate nucleus only
Explanation: Alcoholic cerebellar degeneration typically affects the anterior lobe and the superior part of the vermis. The vermis is the midline structure responsible for axial (trunk) stability. The Midline white matter of the arbor vitae in the vermis carries the tracts necessary for maintaining upright posture. Damage here leads to truncal ataxia (drunken sailor gait) where the patient cannot stand stable with feet together but has relatively spared limb coordination (which is controlled by the lateral hemispheres). The flocculonodular lobe is usually spared in alcoholic degeneration. Therefore, the correct answer is a) Midline white matter.
3. Embedded deep within the white matter of the arbor vitae, closest to the midline roof of the fourth ventricle, is which nucleus?
a) Dentate
b) Emboliform
c) Globose
d) Fastigial
Explanation: The deep cerebellar nuclei are located within the white matter core. They are arranged from medial to lateral. The most medial nucleus, situated near the apex of the roof of the fourth ventricle within the vermal white matter, is the Fastigial nucleus. It receives input from the vermis and projects to the vestibular nuclei and reticular formation. The Globose and Emboliform are lateral to it, and the Dentate is the most lateral. This anatomical position within the medial arbor vitae correlates with its function in balance and eye movements. Therefore, the correct answer is d) Fastigial.
4. The Arbor Vitae is seen to be continuous with the brainstem via three peduncles. Which peduncle carries the massive afferent input from the pontine nuclei to the cerebellar white matter?
a) Superior Cerebellar Peduncle
b) Middle Cerebellar Peduncle
c) Inferior Cerebellar Peduncle
d) Cerebral Peduncle
Explanation: The cerebellar white matter is continuous with the peduncles. The Middle Cerebellar Peduncle (MCP), or Brachium Pontis, is the largest of the three. It is composed entirely of afferent fibers arising from the pontine nuclei of the contralateral side (Pontocerebellar fibers). These fibers enter the cerebellum and branch extensively within the arbor vitae to reach the cerebellar cortex as mossy fibers. The Superior peduncle is primarily efferent (output). The Inferior peduncle is mixed (spinocerebellar/vestibular). Therefore, the correct answer is b) Middle Cerebellar Peduncle.
5. A child presents with headache, vomiting, and ataxia. MRI shows a medulloblastoma compressing the Arbor Vitae of the vermis. This tumor most commonly arises from the roof of which ventricle?
a) Third Ventricle
b) Fourth Ventricle
c) Lateral Ventricle
d) Aqueduct of Sylvius
Explanation: The anterior surface of the cerebellum forms the roof of the Fourth Ventricle. The Arbor Vitae of the vermis lies immediately dorsal to this ventricle. Medulloblastomas are highly malignant primitive neuroectodermal tumors (PNETs) that typically arise in the midline vermis (often from the nodulus) in children. They grow into the fourth ventricle, causing obstructive hydrocephalus (headache, vomiting) and compressing the vermal white matter (truncal ataxia). The proximity of the arbor vitae to the fourth ventricle is key to understanding the symptoms of posterior fossa tumors. Therefore, the correct answer is b) Fourth Ventricle.
6. Embryologically, the cerebellum and its characteristic arbor vitae develop from the dorsal part of the:
a) Telencephalon
b) Diencephalon
c) Mesencephalon
d) Metencephalon
Explanation: The brain develops from three primary vesicles which divide into five secondary vesicles. The Rhombencephalon (Hindbrain) divides into the Metencephalon and Myelencephalon. The Metencephalon gives rise to the Pons (ventrally) and the Cerebellum (dorsally). The dorsal parts of the alar plates of the metencephalon thicken to form the rhombic lips, which fuse and grow to form the cerebellum. The Arbor Vitae structure emerges as the cortex folds and the white matter tracts develop within the metencephalic derivative. The Mesencephalon becomes the midbrain. Therefore, the correct answer is d) Metencephalon.
7. The axons of which neuronal cell type constitute the sole output from the cerebellar cortex, traveling through the arbor vitae to inhibit the deep nuclei?
a) Granule cells
b) Mossy fibers
c) Purkinje cells
d) Climbing fibers
Explanation: The circuitry within the arbor vitae involves loops. Afferent fibers (Mossy/Climbing) travel up the white matter to excite the cortex. Within the cortex, processing occurs. The result of this processing is sent out via the axons of the Purkinje cells. These are the only axons that leave the cerebellar cortex. They travel down through the white matter (arbor vitae) to reach the deep cerebellar nuclei, where they release GABA to inhibit them. Thus, Purkinje cell axons are a major component of the descending white matter tracts. Therefore, the correct answer is c) Purkinje cells.
8. Occlusion of the Posterior Inferior Cerebellar Artery (PICA) can lead to infarction of the inferior cerebellum. This vascular event (Wallenberg Syndrome) often spares the arbor vitae of the superior vermis, which is supplied by the:
a) Anterior Inferior Cerebellar Artery (AICA)
b) Superior Cerebellar Artery (SCA)
c) Basilar Artery
d) Vertebral Artery
Explanation: The vascular supply of the cerebellum is stratified. The PICA supplies the inferior surface and the tonsils. The AICA supplies the anterior inferior surface and the flocculus. The superior surface of the cerebellum, including the bulk of the white matter (arbor vitae) of the superior vermis and the deep cerebellar nuclei, is supplied by the Superior Cerebellar Artery (SCA). Therefore, PICA territory infarcts (lateral medullary syndrome) typically affect the medulla and inferior cerebellum but spare the superior structures supplied by the SCA. Therefore, the correct answer is b) Superior Cerebellar Artery (SCA).
9. A sagittal MRI of the brain is the best modality to visualize the Arbor Vitae. In this view, the "trunk" of the tree corresponds to the white matter connecting to the:
a) Spinal Cord
b) Brainstem
c) Thalamus
d) Occipital Lobe
Explanation: On a midsagittal MRI sequence (T1 weighted is excellent for anatomy), the Arbor Vitae is clearly visible as the hyperintense (white) branched structure within the vermis. The branches extend towards the periphery. The converging "trunk" of the white matter directs anteriorly and ventrally. This trunk represents the collection of peduncles (Superior, Middle, Inferior) that physically and functionally attach the cerebellum to the Brainstem (Pons, Medulla, Midbrain). It does not connect directly to the occipital lobe or thalamus (though fibers project there eventually). Therefore, the correct answer is b) Brainstem.
10. Which part of the cerebellum is evolutionarily oldest (Archicerebellum) and has its white matter connections primarily with the vestibular system?
a) Posterior Lobe
b) Anterior Lobe
c) Flocculonodular Lobe
d) Neocerebellum
Explanation: The cerebellum is divided phylogenetically. The oldest part is the Flocculonodular Lobe (Archicerebellum or Vestibulocerebellum). Its white matter connections are distinct; it receives input directly from the vestibular nerve and nuclei and projects back to the vestibular nuclei (and Fastigial nucleus). It regulates equilibrium and eye movements. The Anterior lobe is the Paleocerebellum (Spinal connections). The Posterior lobe is the Neocerebellum (Cerebral connections). The Arbor Vitae structure is present in all, but the connections define the functional lobe. Therefore, the correct answer is c) Flocculonodular Lobe.
Chapter: Neuroanatomy; Topic: Diencephalon; Subtopic: Embryology and Divisions of the Thalamus
Key Definitions & Concepts
Diencephalon: The part of the forebrain situated between the telencephalon (cerebrum) and the midbrain; it surrounds the third ventricle.
Pars Dorsalis Diencephali: The dorsal division of the diencephalon, which develops into the Thalamus proper and the Epithalamus.
Pars Ventralis Diencephali: The ventral division of the diencephalon, which develops into the Hypothalamus and Subthalamus.
Hypothalamic Sulcus: The anatomical groove on the lateral wall of the third ventricle that separates the Pars Dorsalis (Thalamus) from the Pars Ventralis (Hypothalamus).
Thalamus: The largest derivative of the Pars Dorsalis; acts as the major sensory relay station to the cortex.
Metathalamus: Consists of the Medial and Lateral Geniculate Bodies; part of the posterior thalamus involved in hearing and vision.
Epithalamus: A derivative of the Pars Dorsalis, consisting of the Pineal gland and Habenular nuclei.
Dejerine-Roussy Syndrome: Thalamic pain syndrome caused by occlusion of thalamogeniculate arteries, resulting in severe contralateral pain and hemisensory loss.
Reticular Nucleus of Thalamus: A shell of inhibitory (GABAergic) neurons surrounding the thalamus that regulates the flow of information to the cortex.
Alar Plate: The embryological structure from which the sensory relay nuclei of the thalamus (Pars Dorsalis) are derived.
[Image of Basal ganglia nuclei anatomy]
Lead Question - 2016
Pars dorsalis is a part of ?
a) Cerebrum
b) Cerebellum
c) Pons
d) Thalamus
Explanation: Embryologically, the Diencephalon is divided by the hypothalamic sulcus (sulcus limitans of the diencephalon) into a dorsal portion and a ventral portion. The dorsal portion is known as the Pars Dorsalis Diencephali, and the ventral portion is the Pars Ventralis Diencephali. The Pars Dorsalis differentiates to form the Thalamus (Thalamus proper) and the Epithalamus (Pineal body). The Pars Ventralis gives rise to the Hypothalamus and the Subthalamus. Therefore, "Pars Dorsalis" in this context refers to the developmental precursor of the Thalamus. While the Pons has a "pars dorsalis pontis" (tegmentum), strictly speaking, the term "Pars Dorsalis" alone in embryology questions points to the Thalamus. Therefore, the correct answer is d) Thalamus.
1. Which anatomical structure serves as the boundary between the Pars Dorsalis (Thalamus) and the Pars Ventralis (Hypothalamus) in the third ventricle?
a) Stria terminalis
b) Hypothalamic sulcus
c) Interthalamic adhesion
d) Lamina terminalis
Explanation: The lateral wall of the third ventricle shows a distinct groove extending from the Interventricular Foramen (of Monro) to the cerebral aqueduct. This groove is the Hypothalamic sulcus. It represents the continuation of the sulcus limitans. It serves as the landmark dividing the diencephalon into a superior (dorsal) part and an inferior (ventral) part. The superior part (Pars Dorsalis) becomes the Thalamus, and the inferior part (Pars Ventralis) becomes the Hypothalamus. The Stria terminalis lies between the thalamus and caudate. Therefore, the correct answer is b) Hypothalamic sulcus.
2. The Subthalamic Nucleus, a key component of the basal ganglia circuitry, is derived embryologically from the:
a) Pars Dorsalis Diencephali
b) Pars Ventralis Diencephali
c) Telencephalon
d) Mesencephalon
Explanation: As the diencephalon divides, the Pars Ventralis differentiates into two main structures: the Hypothalamus (involved in homeostasis) and the Subthalamus (involved in motor control). The Subthalamus contains the Subthalamic Nucleus (STN) and the Zona Incerta. Even though the STN functions functionally with the basal ganglia (which are telencephalic), its origin is diencephalic. Since it comes from the ventral division, it is a derivative of the Pars Ventralis Diencephali. The Thalamus comes from the Pars Dorsalis. Therefore, the correct answer is b) Pars Ventralis Diencephali.
3. A patient presents with hemiballismus (violent flinging of limbs). This condition is caused by a lesion in the Subthalamus. Anatomically, the Subthalamus is located:
a) Dorsal to the Thalamus
b) Ventral to the Thalamus and lateral to the Hypothalamus
c) Medial to the Hypothalamus
d) Superior to the Epithalamus
Explanation: The term "Subthalamus" literally means "below the thalamus." Anatomically, it is wedged between the Thalamus superiorly and the Tegmentum of the midbrain inferiorly. Medially, it relates to the Hypothalamus. Specifically, it is located ventral to the Thalamus and lateral to the Hypothalamus. It contains the rostral extensions of the midbrain red nucleus and substantia nigra, alongside its own Subthalamic Nucleus. Lesions here disrupt the indirect pathway of the basal ganglia, leading to hyperkinetic movements. Therefore, the correct answer is b) Ventral to the Thalamus and lateral to the Hypothalamus.
4. Which nucleus of the Thalamus (Pars Dorsalis) receives the Spinothalamic tract and Medial Lemniscus, mediating somatic sensation from the body?
a) Ventral Anterior (VA)
b) Ventral Lateral (VL)
c) Ventral Posterolateral (VPL)
d) Ventral Posteromedial (VPM)
Explanation: The Ventral group of thalamic nuclei is organized by function. The Ventral Posterior nucleus is the principal somatic sensory relay. It is subdivided into VPL and VPM. The Ventral Posterolateral (VPL) nucleus receives the major ascending sensory tracts from the body: the Spinothalamic tract (pain/temperature) and the Medial Lemniscus (touch/proprioception). The VPM receives sensory input from the face (Trigeminal). The VA and VL are motor relay nuclei receiving input from the basal ganglia and cerebellum. Therefore, the correct answer is c) Ventral Posterolateral (VPL).
5. The Epithalamus, another derivative of the Pars Dorsalis, includes which midline endocrine structure?
a) Pituitary Gland
b) Pineal Gland
c) Mammillary Body
d) Tuber Cinereum
Explanation: The Pars Dorsalis Diencephali differentiates into the large Thalamus and the smaller, posteriorly located Epithalamus. The Epithalamus consists of the Habenular nuclei (and commissure), the Stria Medullaris, and the Pineal Gland (Pineal Body). The Pineal gland synthesizes melatonin and regulates circadian rhythms. The Pituitary gland, Mammillary bodies, and Tuber Cinereum are all parts of the Hypothalamus (Pars Ventralis). Therefore, the correct answer is b) Pineal Gland.
6. A 60-year-old hypertensive patient develops severe, burning pain in the right side of the body that is exacerbated by light touch (allodynia). This "Thalamic Pain Syndrome" is typically caused by a stroke involving which arterial supply?
a) Recurrent Artery of Heubner
b) Anterior Choroidal Artery
c) Thalamogeniculate Arteries
d) Lenticulostriate Arteries
Explanation: Dejerine-Roussy Syndrome (Thalamic Pain Syndrome) is a classic lacunar stroke syndrome. It results from damage to the VPL/VPM nuclei of the thalamus. These nuclei are supplied by the Thalamogeniculate arteries, which are perforating branches arising from the P2 segment of the Posterior Cerebral Artery (PCA). Occlusion leads to infarction of the sensory relay nuclei, causing the characteristic contralateral anesthesia followed by agonizing neuropathic pain. The Anterior Choroidal supplies the LGN and Internal Capsule. Therefore, the correct answer is c) Thalamogeniculate Arteries.
7. The Medial Geniculate Body (MGB) is a specialized part of the thalamus (Metathalamus). It acts as a relay station for which sensory modality?
a) Vision
b) Audition (Hearing)
c) Olfaction
d) Taste
Explanation: The Metathalamus is located on the posterior inferior aspect of the thalamus and consists of two "bumps": the Lateral Geniculate Body (LGB) and the Medial Geniculate Body (MGB). The LGB receives input from the optic tract and relays vision. The Medial Geniculate Body (MGB) receives input from the Inferior Colliculus via the brachium of the inferior colliculus and relays Auditory information to the Primary Auditory Cortex (Heschl's Gyrus) in the temporal lobe. Mnemonic: Medial for Music (Hearing), Lateral for Light (Vision). Therefore, the correct answer is b) Audition (Hearing).
8. The "Gateway to the Cortex" is a functional description of the Thalamus. However, one specific sensory modality bypasses the Thalamus and projects directly to the cortex. Which sense is this?
a) Pain
b) Taste
c) Olfaction
d) Vestibular sense
Explanation: The Thalamus is the major relay station for almost all sensory information reaching the cerebral cortex (Vision, Hearing, Touch, Pain, Taste, Vestibular). The notable exception is Olfaction (Smell). The Olfactory nerve (CN I) projects from the olfactory bulb directly to the primary olfactory cortex (pyriform cortex, amygdala, entorhinal cortex) without first synapsing in the thalamus. Olfactory information does reach the thalamus (Medial Dorsal nucleus) eventually, but only *after* cortical processing, for conscious discrimination. Therefore, the correct answer is c) Olfaction.
9. The Reticular Nucleus of the Thalamus forms a shell around the lateral aspect of the thalamus. Unlike other thalamic nuclei, its neurons are:
a) Excitatory (Glutamatergic) and project to the cortex
b) Inhibitory (GABAergic) and project back to the thalamus
c) Cholinergic and project to the brainstem
d) Dopaminergic and project to the striatum
Explanation: The Reticular Nucleus is unique among thalamic nuclei. It does not project to the cerebral cortex. Instead, it receives collaterals from corticothalamic and thalamocortical fibers. It contains Inhibitory (GABAergic) neurons that project back into the other thalamic nuclei. This allows it to act as a "gatekeeper," regulating the gain and flow of information between the thalamus and cortex, playing a crucial role in attention and sleep spindles. All other major thalamic nuclei are excitatory (Glutamatergic). Therefore, the correct answer is b) Inhibitory (GABAergic) and project back to the thalamus.
10. The Y-shaped band of white matter that divides the Thalamus into Anterior, Medial, and Lateral nuclear groups is called the:
a) External Medullary Lamina
b) Internal Capsule
c) Internal Medullary Lamina
d) Stria Medullaris
Explanation: The gray matter of the thalamus is subdivided by a vertical sheet of white matter known as the Internal Medullary Lamina. This lamina splits anteriorly to form a Y-shape. This divides the thalamus into three main masses: the Anterior nuclear group (between the arms of the Y), the Medial nuclear group (medial to the lamina), and the Lateral nuclear group (lateral to the lamina). The lamina itself contains the Intralaminar nuclei (e.g., Centromedian). The External Medullary Lamina separates the thalamus from the Reticular nucleus laterally. Therefore, the correct answer is c) Internal Medullary Lamina.
Chapter: Neuroanatomy; Topic: Cerebrum (Telencephalon); Subtopic: Functional Anatomy of the Parietal Lobe
Key Definitions & Concepts
Supramarginal Gyrus (SMG): A horseshoe-shaped convolution of the inferior parietal lobule that surrounds the upturned posterior end of the Lateral (Sylvian) sulcus.
Brodmann Area 40: The cytoarchitectural designation for the Supramarginal gyrus; involved in phonological processing and motor planning.
Inferior Parietal Lobule (IPL): The lower part of the parietal lobe, consisting of the Supramarginal gyrus (anteriorly) and the Angular gyrus (posteriorly).
Angular Gyrus: Brodmann Area 39; located immediately posterior to the SMG, capping the Superior Temporal Sulcus.
Conduction Aphasia: A language disorder characterized by poor repetition despite relatively intact comprehension and fluency; often associated with lesions in the SMG or Arcuate Fasciculus.
Ideomotor Apraxia: The inability to carry out complex motor commands (like "wave goodbye") despite intact motor, sensory, and comprehension pathways; linked to Left SMG damage.
Lateral Sulcus (Sylvian Fissure): The deep fissure separating the frontal/parietal lobes from the temporal lobe; the SMG caps its posterior termination.
Phonological Loop: A component of working memory responsible for the temporary storage of speech sounds; functionally localized to the left SMG.
Intraparietal Sulcus: The sulcus separating the Superior Parietal Lobule from the Inferior Parietal Lobule.
Gerstmann Syndrome: A constellation of symptoms (agraphia, acalculia, finger agnosia, left-right disorientation) typically associated with the dominant Angular gyrus, adjacent to the SMG.
[Image of Lateral surface of brain gyri]
Lead Question - 2016
Superior marginal gyrus is a part of?
a) Parietal lobe
b) Frontal lobe
c) Temporal lobe
d) Occipital lobe
Explanation: The question likely refers to the Supramarginal Gyrus (often misspelled or referred to as "Superior marginal"). Anatomically, the Parietal Lobe is divided by the Intraparietal Sulcus into a Superior Parietal Lobule and an Inferior Parietal Lobule. The Inferior Parietal Lobule is further subdivided into two main gyri: the Supramarginal Gyrus (Brodmann Area 40), which arches over the posterior end of the lateral sulcus, and the Angular Gyrus (Brodmann Area 39), which arches over the superior temporal sulcus. Therefore, the Supramarginal gyrus is a key component of the Parietal lobe. It is involved in language processing (phonology) and motor planning. The correct answer is a) Parietal lobe.
1. The Supramarginal Gyrus corresponds to which Brodmann Area?
a) Area 39
b) Area 40
c) Area 44
d) Area 22
Explanation: Brodmann mapping is essential for neuroanatomy. The Inferior Parietal Lobule contains two distinct areas. The anterior portion, which caps the Sylvian fissure, is the Supramarginal Gyrus and corresponds to Brodmann Area 40. The posterior portion, which caps the Superior Temporal Sulcus, is the Angular Gyrus and corresponds to Brodmann Area 39. Area 44 is Broca's area (Frontal). Area 22 is Wernicke's area (Temporal). Understanding these numbers helps in correlating cortical location with function, particularly in language processing models where Area 40 is part of the phonological loop. Therefore, the correct answer is b) Area 40.
2. A 65-year-old male with a history of stroke presents with fluent speech and good comprehension but has a profound inability to repeat words or sentences spoken to him. He makes frequent phonemic paraphasias. The lesion is most likely involving the:
a) Left Superior Temporal Gyrus
b) Left Inferior Frontal Gyrus
c) Left Supramarginal Gyrus and Arcuate Fasciculus
d) Right Parietal Lobe
Explanation: This clinical picture represents Conduction Aphasia. The hallmark is the dissociation between intact fluency/comprehension and impaired repetition. This occurs due to a disconnection between the posterior language area (Wernicke's) and the anterior language area (Broca's). The anatomical substrate for this connection is the Arcuate Fasciculus, which runs deep to and through the Left Supramarginal Gyrus. Lesions of the Supramarginal gyrus or the underlying white matter tracts disrupt this transfer of phonological information, leading to the inability to repeat. Left Superior Temporal is Wernicke's (poor comprehension). Inferior Frontal is Broca's (non-fluent). Therefore, the correct answer is c) Left Supramarginal Gyrus and Arcuate Fasciculus.
3. Anatomically, the Supramarginal Gyrus surrounds the posterior upturned end of which sulcus?
a) Central Sulcus
b) Superior Temporal Sulcus
c) Lateral (Sylvian) Sulcus
d) Calcarine Sulcus
Explanation: The gyri of the inferior parietal lobule are defined by the sulci they "cap" or surround. The Supramarginal Gyrus is formed by the cortex that arches over the posterior, ascending termination of the Lateral (Sylvian) Sulcus. Located immediately posterior to it, the Angular Gyrus arches over the posterior end of the Superior Temporal Sulcus. The Central Sulcus separates frontal and parietal lobes. The Calcarine sulcus is in the occipital lobe. Identifying the end of the Sylvian fissure on an MRI is the standard method for locating the Supramarginal gyrus. Therefore, the correct answer is c) Lateral (Sylvian) Sulcus.
4. A patient is asked to "pretend to brush their teeth" or "salute." They understand the command and have no paralysis, but they move their hand awkwardly or use their hand as the tool itself, failing to execute the motor plan. This condition is called Ideomotor Apraxia and is associated with lesions in the:
a) Dominant Supramarginal Gyrus
b) Non-dominant Angular Gyrus
c) Primary Motor Cortex
d) Cerebellum
Explanation: Ideomotor Apraxia is a disorder of motor planning. The patient knows what to do but has lost the "praxis" or the motor formula of how to do it. The "Praxis" system is lateralized to the dominant (usually left) hemisphere. The Left Supramarginal Gyrus (Inferior Parietal Lobule) stores these skilled motor programs. A lesion here disconnects the concept of the movement (from language areas) from the motor execution (motor cortex). Thus, the patient fumbles or makes errors in transitive movements despite intact strength. Therefore, the correct answer is a) Dominant Supramarginal Gyrus.
5. The Inferior Parietal Lobule is composed of which two gyri?
a) Precentral and Postcentral gyri
b) Superior and Middle Frontal gyri
c) Supramarginal and Angular gyri
d) Lingual and Fusiform gyri
Explanation: The parietal lobe has three main parts on the lateral surface: the Postcentral Gyrus (somatosensory), the Superior Parietal Lobule (sensory association), and the Inferior Parietal Lobule (IPL). The Intraparietal sulcus separates the Superior and Inferior lobules. The IPL is further subdivided into two distinct U-shaped gyri: the Supramarginal gyrus (anteriorly) and the Angular gyrus (posteriorly). These two structures are evolutionarily recent and are critical for cross-modal integration (visual, auditory, somatic) involved in reading, writing, and calculation. Lingual/Fusiform are occipito-temporal. Therefore, the correct answer is c) Supramarginal and Angular gyri.
6. A stroke affecting the non-dominant (right) Inferior Parietal Lobule (including the Supramarginal gyrus) typically results in:
a) Gerstmann Syndrome
b) Hemispatial Neglect
c) Wernicke's Aphasia
d) Cortical Blindness
Explanation: The right parietal lobe, particularly the inferior lobule (Supramarginal and Angular gyri), plays a dominant role in spatial attention and body schema. A lesion here often leads to Hemispatial Neglect (Left-sided neglect). The patient fails to attend to, report, or respond to stimuli in the left hemifield. They may shave only the right side of their face or draw a clock with numbers only on the right. Gerstmann syndrome and Aphasia are associated with the dominant (left) parietal lobe. Cortical blindness is occipital. Therefore, the correct answer is b) Hemispatial Neglect.
7. Which artery is the primary blood supply to the Supramarginal Gyrus?
a) Anterior Cerebral Artery (ACA)
b) Posterior Cerebral Artery (PCA)
c) Middle Cerebral Artery (MCA)
d) Anterior Choroidal Artery
Explanation: The Supramarginal gyrus is located on the lateral surface of the cerebral hemisphere, surrounding the Sylvian fissure. The lateral convexity of the brain is the territory of the Middle Cerebral Artery (MCA). specifically, the Inferior division (or posterior parietal branches) of the MCA supplies the Inferior Parietal Lobule (Supramarginal and Angular gyri) and the Wernicke's area. The ACA supplies the medial surface (leg area). The PCA supplies the occipital lobe and inferior temporal lobe. MCA occlusion is the most common cause of parietal lobe syndromes like aphasia or neglect. Therefore, the correct answer is c) Middle Cerebral Artery (MCA).
8. The functional role of the left Supramarginal Gyrus in the Baddeley model of Working Memory is primarily:
a) The Central Executive
b) The Visuospatial Sketchpad
c) The Phonological Store
d) Episodic Buffer
Explanation: In cognitive neuroscience, the Supramarginal Gyrus (BA 40) is functionally mapped to the "Phonological Loop" of working memory. Specifically, it is thought to act as the Phonological Store—the "inner ear" that holds auditory-verbal information for a few seconds (e.g., remembering a phone number just long enough to dial it). The subvocal rehearsal system ("inner voice") is mapped more anteriorly to Broca's area (BA 44). This explains why lesions here cause conduction aphasia (inability to hold and repeat the phonological trace). Therefore, the correct answer is c) The Phonological Store.
9. The Supramarginal gyrus is separated from the Superior Parietal Lobule by the:
a) Central Sulcus
b) Intraparietal Sulcus
c) Postcentral Sulcus
d) Parieto-occipital Sulcus
Explanation: The parietal lobe anatomy is defined by its sulci. The Postcentral sulcus runs parallel to the central sulcus, defining the posterior border of the somatosensory cortex. Running horizontally backwards from the postcentral sulcus is the Intraparietal Sulcus. This deep sulcus acts as the horizontal divider of the parietal lobe. Everything above it is the Superior Parietal Lobule (BA 5, 7). Everything below it is the Inferior Parietal Lobule, which contains the Supramarginal and Angular gyri. This separation marks a functional division between somatosensory association (superior) and cross-modal integration (inferior). Therefore, the correct answer is b) Intraparietal Sulcus.
10. A patient presents with "Finger Agnosia" (inability to distinguish fingers) and "Acalculia" (inability to do math). These are components of Gerstmann Syndrome. While classically associated with the Angular Gyrus, the lesion often extends to or involves the adjacent:
a) Supramarginal Gyrus
b) Precentral Gyrus
c) Cingulate Gyrus
d) Fusiform Gyrus
Explanation: Gerstmann Syndrome is a tetrad of symptoms: Agraphia, Acalculia, Finger Agnosia, and Right-Left Disorientation. It is a hallmark of damage to the Dominant (Left) Inferior Parietal Lobule. While the core lesion is usually localized to the Angular Gyrus (BA 39), the Supramarginal Gyrus (BA 40) is immediately anterior and continuous with it. In clinical practice (strokes/tumors), lesions are rarely pinpoint; thus, Gerstmann syndrome often involves damage to the wider Inferior Parietal Lobule, including the Supramarginal gyrus, contributing to the complex deficits in body schema and symbolic processing. Therefore, the correct answer is a) Supramarginal Gyrus.
Chapter: Neuroanatomy; Topic: Cerebellum; Subtopic: Deep Cerebellar Nuclei
Key Definitions & Concepts
Deep Cerebellar Nuclei: Four paired masses of gray matter embedded in the white matter of the cerebellum; they are the primary output source of the cerebellum.
"Don't Eat Greasy Food": A popular mnemonic to remember the nuclei from Lateral to Medial: Dentate, Emboliform, Globose, Fastigial.
Dentate Nucleus: The largest and most lateral nucleus; it receives input from the lateral hemispheres (Cerebrocerebellum) and projects to the thalamus for motor planning.
Interposed Nuclei: A functional grouping of the Emboliform and Globose nuclei; involved in the Spinocerebellum for limb coordination.
Fastigial Nucleus: The most medial nucleus; associated with the Vermis and Flocculonodular lobe (Vestibulocerebellum) for balance and eye movements.
Superior Cerebellar Peduncle: The major output tract (efferent) of the cerebellum, carrying fibers primarily from the Dentate and Interposed nuclei to the midbrain and thalamus.
Purkinje Cells: The sole output neurons of the cerebellar cortex; they send inhibitory (GABAergic) signals to the deep cerebellar nuclei.
Intention Tremor: A hallmark sign of damage to the Dentate nucleus or lateral cerebellar hemispheres, where tremor worsens as the target is approached.
Dysdiadochokinesia: Inability to perform rapid alternating movements; a sign of lateral cerebellar dysfunction.
Truncal Ataxia: Inability to maintain upright posture/balance; associated with midline (Vermis/Fastigial) damage.
[Image of Cerebellar nuclei anatomy]
Lead Question - 2016
Most lateral nucleus of cerebellum is ?
a) Dentate
b) Globose
c) Fastigial
d) Emboliform
Explanation: The cerebellum contains four pairs of deep nuclei embedded within the white matter. From lateral to medial, the arrangement is: Dentate, Emboliform, Globose, and Fastigial. This order is crucial for understanding the functional divisions of the cerebellum. The Dentate nucleus is the largest and most lateral, appearing crumpled like a tooth (hence "dentate"). It is functionally associated with the lateral hemispheres (Neocerebellum) and is responsible for motor planning. The Emboliform and Globose (Interposed) are in the middle. The Fastigial is the most medial, near the midline vermis. Therefore, the correct answer is a) Dentate.
1. The "Interposed Nucleus" is a functional term often used in animal physiology. In humans, this corresponds to which two anatomical nuclei?
a) Dentate and Emboliform
b) Globose and Fastigial
c) Emboliform and Globose
d) Fastigial and Dentate
Explanation: Functionally, the cerebellum is divided into longitudinal zones. The intermediate zone (paravermis) projects to the Interposed Nuclei. In humans, the Interposed nucleus is anatomically distinct as two separate nuclei: the Emboliform nucleus (lateral part) and the Globose nucleus (medial part). These nuclei regulate the Rubrospinal tract and Corticospinal tract components responsible for distal limb flexor muscle tone and coordination. They are distinct from the lateral Dentate (motor planning) and medial Fastigial (axial balance). Therefore, the correct answer is c) Emboliform and Globose.
2. A 55-year-old male presents with a broad-based, staggering gait and a tendency to fall to the right. However, his finger-to-nose testing is normal. The lesion is most likely located in the:
a) Right Dentate Nucleus
b) Left Dentate Nucleus
c) Fastigial Nucleus / Vermis
d) Cerebellar hemispheres bilaterally
Explanation: This patient presents with Truncal Ataxia (gait instability) with spared limb coordination (normal finger-to-nose). This dissociation localizes the pathology to the midline structures of the cerebellum: the Vermis and the Flocculonodular lobe. The deep nucleus associated with this midline zone is the Fastigial Nucleus. It controls axial and proximal musculature via the Vestibulospinal and Reticulospinal tracts to maintain upright posture and balance. Lesions here cause truncal instability. In contrast, lesions of the Dentate nucleus (hemispheres) cause appendicular ataxia (limb issues). Therefore, the correct answer is c) Fastigial Nucleus / Vermis.
3. The Dentate Nucleus sends its efferent fibers primarily to the contralateral Thalamus (VL nucleus) via which pathway?
a) Inferior Cerebellar Peduncle
b) Middle Cerebellar Peduncle
c) Superior Cerebellar Peduncle
d) Cuneocerebellar Tract
Explanation: The cerebellum has three peduncles connecting it to the brainstem. The Inferior (Restiform body) is mainly afferent (input). The Middle (Brachium pontis) is exclusively afferent. The Superior Cerebellar Peduncle (Brachium conjunctivum) is the major efferent (output) pathway. Fibers from the Dentate nucleus leave the cerebellum via the Superior Cerebellar Peduncle, decussate in the midbrain, and synapse in the contralateral Ventrolateral (VL) nucleus of the thalamus. This forms the Dentato-Thalamo-Cortical tract, essential for motor planning. Therefore, the correct answer is c) Superior Cerebellar Peduncle.
4. Purkinje cells of the cerebellar cortex modulate the output of the deep cerebellar nuclei. The neurotransmitter released by Purkinje cells at this synapse is:
a) Glutamate
b) GABA
c) Glycine
d) Acetylcholine
Explanation: The circuitry of the cerebellum is uniform. Inputs (Mossy and Climbing fibers) are excitatory. The Granule cells are excitatory. However, the Purkinje cells, which are the sole output neurons of the cerebellar cortex, are Inhibitory. They project their axons to the deep cerebellar nuclei (Dentate, Emboliform, Globose, Fastigial) and release GABA (Gamma-Aminobutyric Acid). This inhibition sculpts and refines the constant excitatory drive that the deep nuclei receive from collateral fibers. Loss of this inhibition leads to cerebellar ataxia. Therefore, the correct answer is b) GABA.
5. A patient with a right-sided cerebellar tumor exhibits "Dysmetria" (overshooting the target) when reaching for a cup with his right hand. This symptom is primarily due to dysfunction of the:
a) Cerebrocerebellum and Dentate Nucleus
b) Vestibulocerebellum and Fastigial Nucleus
c) Spinocerebellum and Fastigial Nucleus
d) Flocculonodular lobe
Explanation: Dysmetria (past-pointing) and Intention Tremor are signs of Appendicular Ataxia, affecting the limbs. This implicates the lateral hemispheres of the cerebellum (Neocerebellum or Cerebrocerebellum). The deep nucleus serving this area is the Dentate Nucleus. Cerebellar lesions cause ipsilateral deficits because the output decussates twice (once at the SCP, and then the corticospinal tract decussates at the pyramids), or simply because the cerebellum controls the ipsilateral body. Thus, a right-sided lesion affects the right hand. The Vestibulocerebellum controls balance, not limb reaching. Therefore, the correct answer is a) Cerebrocerebellum and Dentate Nucleus.
6. Which of the deep cerebellar nuclei is shaped like a crumpled bag with an opening (hilum) directed medially?
a) Emboliform
b) Fastigial
c) Dentate
d) Globose
Explanation: The morphological appearance of the nuclei is distinct. The Dentate Nucleus is the largest and most complex. In cross-section, it appears as a thin, convoluted band of gray matter, often described as a "crumpled bag" or "flask" shape. It has a distinct hilum that is directed medially and superiorly, from which the fibers of the Superior Cerebellar Peduncle emerge. This structure is analogous to the Inferior Olivary Nucleus in the medulla, which also has a crumpled bag appearance. The other nuclei are smaller, rounded (Globose) or wedge-shaped (Emboliform). Therefore, the correct answer is c) Dentate.
7. The Fastigial nucleus receives its primary cortical input from which anatomical part of the cerebellum?
a) Lateral Hemispheres
b) Paravermis
c) Vermis
d) Tonsils
Explanation: The topographic projection from the cortex to the deep nuclei is strictly organized medial-to-lateral. The most medial zone, the Vermis (and the Flocculonodular lobe), projects to the most medial nucleus, the Fastigial Nucleus. The intermediate zone (Paravermis) projects to the Interposed Nuclei (Emboliform and Globose). The lateral zone (Hemispheres) projects to the lateral nucleus, the Dentate. This arrangement correlates with function: Vermis/Fastigial for midline balance, Paravermis/Interposed for proximal limbs, Hemispheres/Dentate for distal limb planning. Therefore, the correct answer is c) Vermis.
8. In the "H-E-A-T" mnemonic for Deep Cerebellar Nuclei defects, the 'T' stands for Tremor. This tremor is characteristically:
a) Resting Tremor
b) Intention Tremor
c) Flapping Tremor (Asterixis)
d) Wing-beating Tremor
Explanation: Cerebellar pathology presents with specific motor signs. The tremor associated with damage to the Dentate nucleus and lateral cerebellum is an Intention Tremor (also called Kinetic tremor). It is absent at rest but appears during voluntary movement and increases in amplitude as the limb approaches its target (e.g., finger-to-nose test). This contrasts with the Resting Tremor of Parkinson's (Basal Ganglia) or the Flapping Tremor of metabolic encephalopathy. The H-E-A-T mnemonic stands for Hypotonia, Equilibrium problems, Ataxia, and Tremor/Titubation. Therefore, the correct answer is b) Intention Tremor.
9. The Globose nucleus is named for its shape. "Globose" means:
a) Plug-shaped
b) Tooth-like
c) Spherical or Round
d) Roof-like
Explanation: The names of the nuclei are derived from Latin descriptions of their shape or position. Globose comes from "Globus," meaning ball or sphere, referring to its Spherical or Round shape. Emboliform comes from "Embolus," meaning a plug or wedge (shaped like a plug driven into the dentate). Dentate means "toothed" (serrated edge). Fastigial comes from "Fastigium," meaning the top of a gable or roof, referring to its position in the roof of the fourth ventricle. Therefore, the correct answer is c) Spherical or Round.
10. Which artery typically supplies the Deep Cerebellar Nuclei?
a) Posterior Inferior Cerebellar Artery (PICA)
b) Anterior Inferior Cerebellar Artery (AICA)
c) Superior Cerebellar Artery (SCA)
d) Basilar Artery direct branches
Explanation: The vascular supply of the cerebellum generally follows superior/inferior divisions. The Deep Cerebellar Nuclei are located deep within the white matter, dorsal to the roof of the fourth ventricle. Most of the deep white matter and the deep nuclei (Dentate, Interposed, Fastigial) are primarily supplied by the Superior Cerebellar Artery (SCA). The PICA supplies the inferior vermis and tonsils (and medulla). The AICA supplies the flocculus and MCP. Occlusion of the SCA can therefore lead to profound hemiataxia due to infarction of the deep nuclei and SCP. Therefore, the correct answer is c) Superior Cerebellar Artery (SCA).
Chapter: Neuroanatomy; Topic: Basal Ganglia; Subtopic: Nuclei, Connections, and Clinical Correlations
Key Definitions & Concepts
Basal Ganglia: A group of subcortical nuclei responsible for motor control, motor learning, executive functions, and emotions. Major components include the Caudate, Putamen, and Globus Pallidus.
Striatum (Neostriatum): The functional unit composed of the Caudate Nucleus and the Putamen; it is the major input zone of the basal ganglia.
Lentiform Nucleus: An anatomical grouping shaped like a lens, consisting of the Putamen (lateral) and the Globus Pallidus (medial).
Substantia Nigra: A midbrain structure functionally linked to the basal ganglia; the Pars Compacta contains dopamine neurons lost in Parkinson's disease.
Subthalamic Nucleus: A diencephalic nucleus involved in the indirect pathway; lesions here cause Hemiballismus.
Direct Pathway: Facilitates movement by disinhibiting the thalamus (Striatum -> GPi/SNr -> Thalamus).
Indirect Pathway: Inhibits movement (Striatum -> GPe -> STN -> GPi/SNr -> Thalamus).
Hemiballismus: A movement disorder characterized by wild, flinging movements of the contralateral limb, typically due to a lacunar stroke in the Subthalamic Nucleus.
Chorea: Brief, semi-directed, irregular movements that are not repetitive or rhythmic; a hallmark of Huntington's Disease.
Nigrostriatal Pathway: The dopaminergic pathway from the Substantia Nigra to the Striatum; critical for modulating motor loops.
[Image of Basal ganglia nuclei anatomy]
Lead Question - 2016
Nucleus of basal ganglia
a) Dentate
b) Thalamus
c) Caudate
d) Red nucleus
Explanation: The basal ganglia are a collection of nuclei deep within the cerebral hemispheres. The primary components are the Caudate Nucleus, the Putamen, and the Globus Pallidus. Functionally, the Subthalamic Nucleus (diencephalon) and Substantia Nigra (midbrain) are also included. The Dentate nucleus is the largest deep cerebellar nucleus. The Thalamus is a major diencephalic relay station but is distinct from the basal ganglia proper (though they interact). The Red nucleus is a midbrain structure involved in the rubrospinal tract. Therefore, the correct answer is c) Caudate.
1. Which two anatomical structures combine to form the "Striatum" (Neostriatum)?
a) Putamen and Globus Pallidus
b) Caudate Nucleus and Putamen
c) Caudate Nucleus and Thalamus
d) Globus Pallidus and Substantia Nigra
Explanation: The terminology of the basal ganglia can be anatomical or functional. The Striatum (or Neostriatum) is the major receptive or input area of the basal ganglia. Anatomically, the internal capsule separates the striatum into two masses: the Caudate Nucleus (medially) and the Putamen (laterally). Despite this separation, they are histologically identical and functionally continuous (via cellular bridges). The term "Lentiform Nucleus" refers to the Putamen and Globus Pallidus combined, but this is merely a descriptive shape, not a functional unit. Therefore, the correct answer is b) Caudate Nucleus and Putamen.
2. A 65-year-old hypertensive male presents to the ER with sudden onset of wild, involuntary, flinging movements of his right arm and leg. An MRI brain would most likely reveal a lacunar infarct in which structure?
a) Left Caudate Nucleus
b) Left Subthalamic Nucleus
c) Right Subthalamic Nucleus
d) Right Globus Pallidus
Explanation: The clinical presentation is Hemiballismus, characterized by violent, high-amplitude, flinging movements of the proximal limbs. This is a classic hyperkinetic disorder. The lesion is located in the Subthalamic Nucleus (STN). The STN is part of the indirect pathway and normally provides excitatory drive to the Globus Pallidus internus (GPi), which inhibits the thalamus. Damage to the STN removes this excitation, reducing the inhibition on the thalamus, leading to excessive movement. Since the deficits are on the right side, the lesion is contralateral. Therefore, the correct answer is b) Left Subthalamic Nucleus.
3. The "Lentiform Nucleus" is an anatomical term describing the lens-shaped mass lateral to the internal capsule. It consists of:
a) Caudate and Putamen
b) Putamen and Globus Pallidus
c) Globus Pallidus and Thalamus
d) Putamen and Claustrum
Explanation: In a coronal or horizontal section of the brain, the wedge-shaped grey matter mass located lateral to the Internal Capsule is called the Lentiform (or Lenticular) Nucleus. It is comprised of two distinct parts: the Putamen (the darker, lateral portion) and the Globus Pallidus (the paler, medial portion). While they sit together anatomically, they have different embryological origins (Putamen from telencephalon, GP largely from diencephalon) and functions. The external capsule separates the Lentiform nucleus from the Claustrum. Therefore, the correct answer is b) Putamen and Globus Pallidus.
4. A 70-year-old patient exhibits a resting tremor ("pill-rolling"), lead-pipe rigidity, and bradykinesia. Pathological examination of the brain would likely show depigmentation and neuronal loss in the:
a) Substantia Nigra Pars Reticulata
b) Locus Coeruleus
c) Substantia Nigra Pars Compacta
d) Raphe Nuclei
Explanation: This patient presents with the cardinal signs of Parkinson's Disease. The pathophysiology involves the degeneration of dopaminergic neurons in the nigrostriatal pathway. These neurons have their cell bodies in the Substantia Nigra Pars Compacta (SNc) of the midbrain. These cells contain neuromelanin, giving the structure a dark appearance. In Parkinson's, the loss of these cells leads to gross depigmentation (paleness) of the substantia nigra. The Pars Reticulata is the GABAergic output nucleus of the SN. The Locus Coeruleus (norepinephrine) is also affected but the primary motor defect is nigral. Therefore, the correct answer is c) Substantia Nigra Pars Compacta.
5. The primary output nuclei of the basal ganglia, which send inhibitory projections to the thalamus, are:
a) Caudate and Putamen
b) Globus Pallidus externus (GPe) and Subthalamic Nucleus
c) Globus Pallidus internus (GPi) and Substantia Nigra Pars Reticulata (SNr)
d) Subthalamic Nucleus and Substantia Nigra Pars Compacta (SNc)
Explanation: The basal ganglia process information through direct and indirect pathways, but they share a common final output. The output signals leave the basal ganglia to inhibit the motor thalamus (VA/VL nuclei). The two nuclei responsible for this tonic inhibitory output are the Globus Pallidus internus (GPi) and the Substantia Nigra Pars Reticulata (SNr). These two structures are histologically similar and use GABA as their neurotransmitter. The GPe and STN are intermediate relay nuclei within the circuitry. Therefore, the correct answer is c) Globus Pallidus internus (GPi) and Substantia Nigra Pars Reticulata (SNr).
6. A 40-year-old male presents with chorea (dance-like movements) and progressive dementia. His father had similar symptoms. An MRI scan reveals "boxcar" ventricles (dilated frontal horns). This appearance is due to atrophy of which structure?
a) Head of Caudate Nucleus
b) Thalamus
c) Putamen
d) Hippocampus
Explanation: This clinical picture describes Huntington's Disease, an autosomal dominant neurodegenerative disorder. The hallmark pathology is the selective degeneration of GABAergic medium spiny neurons in the striatum. The atrophy is most profound in the Head of the Caudate Nucleus. The head of the caudate normally bulges into the frontal horn of the lateral ventricle. As it atrophies and flattens, the ventricles appear dilated and squared off on neuroimaging, a sign often referred to as "boxcar ventricles." While the putamen is also involved, the caudate atrophy causes the specific radiological sign. Therefore, the correct answer is a) Head of Caudate Nucleus.
7. The principal neurotransmitter released by the medium spiny neurons of the striatum to the Globus Pallidus is:
a) Glutamate
b) Dopamine
c) GABA
d) Acetylcholine
Explanation: The striatum (caudate and putamen) is primarily composed of Medium Spiny Neurons (MSNs). These are the projection neurons of the striatum. Regardless of whether they are part of the direct pathway (projecting to GPi) or the indirect pathway (projecting to GPe), these neurons are inhibitory. The main inhibitory neurotransmitter in the CNS is Gamma-Aminobutyric Acid (GABA). Glutamate is excitatory (cortex/thalamus/STN). Dopamine is modulatory (from SNc). Acetylcholine is used by striatal interneurons. Therefore, the correct answer is c) GABA.
8. The Lenticulostriate arteries, which supply the Striatum and Internal Capsule, are branches of which major vessel?
a) Anterior Cerebral Artery
b) Middle Cerebral Artery
c) Posterior Cerebral Artery
d) Anterior Choroidal Artery
Explanation: The blood supply to the basal ganglia is critical because these deep structures are frequent sites of hypertensive hemorrhages ("Charcot-Bouchard aneurysms") and lacunar infarcts. The Lenticulostriate arteries are small perforating branches that arise from the M1 segment of the Middle Cerebral Artery (MCA). They supply the Putamen, Globus Pallidus, upper Internal Capsule, and the body of the Caudate. The Anterior Cerebral Artery supplies the head of the caudate (via Heubner's artery). Therefore, the correct answer is b) Middle Cerebral Artery.
9. A 16-year-old male presents with a wing-beating tremor, dysarthria, and elevated liver enzymes. Slit-lamp examination reveals Kayser-Fleischer rings. MRI brain shows hyperintensity in the Lentiform nuclei. This condition involves the accumulation of:
a) Iron
b) Calcium
c) Copper
d) Amyloid
Explanation: The clinical triad of liver disease, neurological symptoms (tremor, rigidity), and corneal deposits (Kayser-Fleischer rings) in a young patient is diagnostic of Wilson's Disease (Hepatolenticular Degeneration). This is an autosomal recessive disorder of Copper metabolism (ATP7B mutation). Excess copper cannot be excreted into bile and accumulates in tissues, particularly the liver and the brain. In the brain, the basal ganglia are preferentially affected, specifically the Putamen (part of the Lentiform nucleus), leading to the "Face of the Giant Panda" sign on MRI. Therefore, the correct answer is c) Copper.
10. Activation of the "Direct Pathway" of the basal ganglia results in:
a) Inhibition of the Thalamus and decreased movement
b) Disinhibition of the Thalamus and increased movement
c) Excitation of the GPi
d) Inhibition of the Motor Cortex
Explanation: The Direct Pathway functions to facilitate movement. The sequence is: Motor Cortex excites Striatum -> Striatum inhibits GPi/SNr -> GPi/SNr (which is normally inhibitory) is inhibited. This "inhibition of an inhibitor" is called disinhibition. Consequently, the Thalamus is released from tonic inhibition and can send excitatory signals back to the Motor Cortex. In contrast, the Indirect Pathway increases inhibition on the Thalamus, suppressing movement. Therefore, the correct answer is b) Disinhibition of the Thalamus and increased movement.
Chapter: Neuroanatomy; Topic: Blood Supply of the Brain; Subtopic: Anterior Cerebral Artery and Paracentral Lobule
Key Definitions & Concepts
Paracentral Lobule: A U-shaped convolution on the medial surface of the cerebral hemisphere that surrounds the central sulcus; it contains the primary motor and sensory areas for the leg and foot.
Anterior Cerebral Artery (ACA): The terminal branch of the Internal Carotid Artery that supplies the medial aspect of the frontal and parietal lobes.
Callosomarginal Artery: The major branch of the ACA that runs in the cingulate sulcus; it typically gives rise to the paracentral artery supplying the paracentral lobule.
Pericallosal Artery: The continuation of the ACA that runs directly over the corpus callosum; it supplies the corpus callosum and adjacent medial cortex.
Recurrent Artery of Heubner (Medial Striate): A branch of the ACA supplying the head of the caudate nucleus and anterior limb of the internal capsule.
Homunculus: The topographical representation of the body on the cortex; the leg and foot are mapped to the medial surface (ACA territory).
Frontopolar Artery: A branch of the ACA supplying the medial aspect of the frontal pole.
ACA Syndrome: Clinical signs resulting from ACA occlusion, characterized by contralateral leg weakness (paracentral lobule) and urinary incontinence.
Cingulate Gyrus: The curved fold covering the corpus callosum, involved in emotion formation; supplied by ACA branches.
Watershed Area: The border zone between the territories of the ACA and MCA (Middle Cerebral Artery); prone to ischemia during hypotension.
[Image of Medial surface of brain blood supply]
Lead Question - 2016
Which artery supplies the paracentral lobule?
a) Medial Striate artery
b) Calloso Marginal artery
c) Pericallosal artery
d) Frontopolar artery
Explanation: The paracentral lobule is located on the medial surface of the brain and contains the motor and sensory representations of the contralateral lower limb and perineum. It is supplied by the Anterior Cerebral Artery (ACA). Anatomically, the ACA usually divides into two main branches: the Pericallosal artery (which courses over the corpus callosum) and the Callosomarginal artery (which courses in the cingulate sulcus). The arterial branch specifically responsible for supplying the paracentral lobule is the paracentral artery, which most commonly arises from the Callosomarginal artery. If the Callosomarginal artery is absent, branches may come directly from the Pericallosal, but "Calloso Marginal" is the classic anatomical answer for the cortical supply of this region. Therefore, the correct answer is b) Calloso Marginal artery.
1. A 65-year-old male presents with sudden onset of weakness and loss of sensation in his right leg and foot. The arm and face are spared. He also reports urinary incontinence. Which artery is most likely occluded?
a) Left Middle Cerebral Artery
b) Right Anterior Cerebral Artery
c) Left Anterior Cerebral Artery
d) Left Posterior Cerebral Artery
Explanation: This is a classic presentation of Anterior Cerebral Artery (ACA) syndrome. The ACA supplies the medial surface of the cortex, specifically the paracentral lobule, which corresponds to the motor and sensory areas for the leg and foot. Occlusion leads to contralateral deficits. Since the symptoms are on the right side, the lesion must be in the Left Anterior Cerebral Artery. Urinary incontinence occurs due to involvement of the cortical center for micturition, also located in the paracentral lobule. MCA strokes typically affect the face and arm more than the leg. Therefore, the correct answer is c) Left Anterior Cerebral Artery.
2. The Recurrent Artery of Heubner (Medial Striate Artery) is a clinically significant branch of the ACA. Which vital structure does it primarily supply?
a) Thalamus
b) Occipital pole
c) Head of Caudate Nucleus and Anterior limb of Internal Capsule
d) Posterior limb of Internal Capsule
Explanation: The Recurrent Artery of Heubner is an early proximal branch of the ACA (usually arising near the Anterior Communicating Artery). It penetrates the brain substance to supply deep structures. Specifically, it supplies the head of the caudate nucleus, the anterior part of the lentiform nucleus (putamen), and the anterior limb of the internal capsule. Occlusion of this artery can cause contralateral face and arm weakness (due to internal capsule involvement) and cognitive/behavioral changes. The posterior limb of the capsule is supplied by the Anterior Choroidal artery (ICA) and Lenticulostriate arteries (MCA). Therefore, the correct answer is c) Head of Caudate Nucleus and Anterior limb of Internal Capsule.
3. Which segment of the Anterior Cerebral Artery is located proximal to the Anterior Communicating Artery (ACom)?
a) A1 segment
b) A2 segment
c) A3 segment
d) M1 segment
Explanation: The Anterior Cerebral Artery is divided into segments for radiological and surgical description. The A1 segment (pre-communicating segment) extends from the origin of the ACA at the internal carotid artery bifurcation to the Anterior Communicating Artery. The A2 segment (post-communicating segment) begins after the ACom and runs vertically in the interhemispheric fissure. A3 includes the distal branches like the pericallosal. The distinction is crucial because the ACom connects the bilateral A1 segments, allowing collateral flow if one A1 is hypoplastic or occluded. Therefore, the correct answer is a) A1 segment.
4. A patient with a ruptured aneurysm of the Anterior Communicating Artery (ACom) develops personality changes, abulia (lack of will), and memory deficits. This is due to damage to which lobes?
a) Occipital lobes
b) Temporal lobes
c) Parietal lobes
d) Frontal lobes
Explanation: The ACom is the most common site for intracranial berry aneurysms. Rupture or surgical clipping can cause vasospasm or infarction in the territory of the distal ACA, particularly affecting the medial aspects of the Frontal lobes (orbitofrontal and medial prefrontal cortex). These areas are critical for executive function, personality, motivation, and social inhibition. Damage here leads to a "frontal lobe syndrome," characterized by apathy, abulia, disinhibition, and memory loss (if fornix/septal area is involved). Temporal lobes are supplied by MCA/PCA. Therefore, the correct answer is d) Frontal lobes.
5. The Pericallosal artery, a continuation of the ACA, runs posteriorly in which anatomical space?
a) Sylvian fissure
b) Callosal sulcus
c) Cingulate sulcus
d) Calcarine sulcus
Explanation: Anatomical naming often follows location. The Pericallosal artery runs "peri" (around) the corpus callosum. It lies directly in the Callosal sulcus, which is the groove between the corpus callosum and the cingulate gyrus. In contrast, the Callosomarginal artery (the other major branch) runs in the Cingulate sulcus, which is located superior to the cingulate gyrus. The Sylvian fissure contains the Middle Cerebral Artery. The Calcarine sulcus contains the Posterior Cerebral Artery. Therefore, the correct answer is b) Callosal sulcus.
6. Bilateral occlusion of the Anterior Cerebral Arteries is rare but results in a devastating clinical picture known as akinetic mutism. This condition is primarily linked to damage of the:
a) Cingulate gyrus and supplementary motor area
b) Primary visual cortex
c) Cerebellum
d) Auditory association area
Explanation: Akinetic mutism is a state where the patient is awake and may track objects visually but does not move (akinesia) or speak (mutism) due to a severe lack of motivation or drive. This is caused by bilateral damage to the ventromedial frontal lobes, specifically the Cingulate gyrus and Supplementary Motor Area (SMA). These structures are supplied by the ACAs and are crucial for the initiation of movement and speech. While the motor system (corticospinal tract) might be intact, the "will" to move is lost. Therefore, the correct answer is a) Cingulate gyrus and supplementary motor area.
7. The "Watershed" or border zone infarcts between the ACA and MCA territories typically present with symptoms affecting which body part?
a) Face
b) Hand
c) Trunk and proximal limbs
d) Feet only
Explanation: Watershed infarcts occur at the distal boundaries of two arterial territories, usually due to systemic hypotension (hypoperfusion). The border zone between the ACA (medial supply) and MCA (lateral supply) corresponds topographically to the trunk and proximal extremities on the motor homunculus ("Man in a Barrel" syndrome). While the ACA supplies the leg/foot and the MCA supplies the face/arm, the watershed zone involves the Trunk and proximal limbs (shoulder/hip girdle). Patients may exhibit proximal weakness with sparing of distal fine motor function. Therefore, the correct answer is c) Trunk and proximal limbs.
8. Which of the following is NOT a branch of the Anterior Cerebral Artery?
a) Frontopolar artery
b) Anterior Choroidal artery
c) Pericallosal artery
d) Medial Orbitofrontal artery
Explanation: It is vital to distinguish between branches of the Internal Carotid Artery (ICA) and the ACA. The ACA branches include the Recurrent Artery of Heubner, Medial Orbitofrontal, Frontopolar, Callosomarginal, and Pericallosal arteries. However, the Anterior Choroidal artery is a direct distal branch of the Internal Carotid Artery (arising just before the bifurcation into ACA and MCA). It supplies the posterior limb of the internal capsule, choroid plexus, and optic tract. It is not a branch of the ACA. Therefore, the correct answer is b) Anterior Choroidal artery.
9. A 50-year-old female complains of loss of sensation in the perineal region. Which part of the Paracentral Lobule is specifically responsible for this sensory input?
a) The most anterior part
b) The most posterior part
c) The superior edge
d) The inferior bank of the cingulate sulcus
Explanation: The paracentral lobule surrounds the central sulcus on the medial surface. The anterior portion is the continuation of the Precentral gyrus (Motor), and the posterior portion is the continuation of the Postcentral gyrus (Sensory). The homunculus is arranged such that the leg is superior and the perineum/genitals are located in the most posterior part of the paracentral lobule (as the somatosensory map curves into the longitudinal fissure). Lesions here can cause cortical sensory loss in the genitals/perineum. Therefore, the correct answer is b) The most posterior part.
10. The Anterior Cerebral Arteries are connected across the midline by the Anterior Communicating Artery. This connection usually occurs superior to which anatomical structure?
a) Optic Chiasm
b) Pituitary Gland
c) Mammillary bodies
d) Pons
Explanation: The Circle of Willis lies at the base of the brain. The Anterior Communicating Artery (ACom) connects the two A1 segments of the ACAs. Anatomically, this connection lies superior to the Optic Chiasm (or sometimes the optic nerves). This relationship is clinically critical because an aneurysm of the ACom can expand inferiorly and compress the optic chiasm, leading to visual field defects (typically bitemporal hemianopsia, usually affecting the lower fields first). The pituitary is inferior to the chiasm. Therefore, the correct answer is a) Optic Chiasm.
Chapter: Neuroanatomy
Topic: Dural Venous Sinuses
Subtopic: Anatomy and Clinical Significance of the Straight Sinus
Key Definitions & Concepts
Straight Sinus (Sinus Rectus): An unpaired dural venous sinus found at the junction of the falx cerebri and the tentorium cerebelli.
Great Cerebral Vein of Galen: A short, thick vein that drains the deep structures of the brain; it joins the inferior sagittal sinus to form the straight sinus.
Inferior Sagittal Sinus: Runs along the inferior free margin of the falx cerebri and unites with the great cerebral vein to form the straight sinus.
Tentorium Cerebelli: The dural fold separating the cerebrum (occipital lobes) from the cerebellum; the straight sinus runs within its attachment to the falx.
Confluence of Sinuses (Torcular Herophili): The meeting point of the superior sagittal, straight, occipital, and transverse sinuses near the internal occipital protuberance.
Deep Cerebral Venous Thrombosis: A rare but severe type of stroke involving blockage of the straight sinus or vein of Galen, often causing bilateral thalamic infarcts.
Vein of Galen Malformation: A congenital arteriovenous malformation often presenting in neonates with high-output heart failure.
Falx Cerebri: The dural fold separating the two cerebral hemispheres; the straight sinus is located at its posterior base.
Internal Cerebral Veins: Two veins that unite to form the Great Cerebral Vein of Galen.
Superior Cerebellar Veins: Tributaries that drain the upper surface of the cerebellum into the straight sinus or great cerebral vein.
[Image of Dural venous sinuses anatomy]
Lead Question - 2016
Straight sinus is formed by?
a) Inferior Sagittal Sinus
b) Internal Jugular veins
c) Superior Sagittal Sinus
d) Tranverse sinus
Explanation: The straight sinus, also known as the sinus rectus, is an important component of the deep venous drainage system of the brain. It runs posteriorly and inferiorly along the midline junction where the falx cerebri meets the tentorium cerebelli. Anatomically, it is formed by the union of two specific vessels: the Inferior Sagittal Sinus (which drains the medial surfaces of the cerebral hemispheres) and the Great Cerebral Vein of Galen (which drains the deep brain structures like the thalamus and basal ganglia). It eventually terminates by emptying into the confluence of sinuses (Torcular Herophili). Therefore, among the choices provided, the correct component is the a) Inferior Sagittal Sinus.
1. The Straight Sinus runs posteriorly to drain primarily into the Confluence of Sinuses. In many individuals, the flow from the straight sinus is preferentially directed into which sinus?
a) Right Transverse Sinus
b) Left Transverse Sinus
c) Cavernous Sinus
d) Sigmoid Sinus directly
Explanation: At the Confluence of Sinuses (Torcular Herophili), the drainage patterns of the major sinuses can be variable. Typically, the Superior Sagittal Sinus, which carries a larger volume of blood from the superficial cortex, preferentially drains into the Right Transverse Sinus. In contrast, the Straight Sinus, carrying venous blood from the deep cerebral structures, typically deviates to drain into the Left Transverse Sinus. While a true confluence exists in many people where blood mixes, this anatomical asymmetry (Right for Superficial, Left for Deep) is a common and important variation to recognize in neuro-interventional radiology. Therefore, the correct answer is b) Left Transverse Sinus.
2. A neonate presents with signs of high-output cardiac failure and a cranial bruit. Imaging reveals a large midline vascular structure in the region of the pineal gland draining into a dilated straight sinus. What is the most likely diagnosis?
a) Cavernous Sinus Thrombosis
b) Vein of Galen Malformation
c) Berry Aneurysm
d) Dandy-Walker Malformation
Explanation: This clinical presentation is classic for a Vein of Galen Malformation (VGM). Despite the name, it is actually an arteriovenous fistula between the choroidal arteries and the median prosencephalic vein (the embryonic precursor to the vein of Galen). The massive arteriovenous shunting results in volume overload, leading to high-output congestive heart failure in the newborn. The "vein" and the receiving Straight Sinus become massively dilated due to the arterial pressure. A cranial bruit is often audible. Cavernous sinus thrombosis is infective. Berry aneurysms are arterial and typically asymptomatic in neonates. Therefore, the correct answer is b) Vein of Galen Malformation.
3. The Straight Sinus is situated within the junction of which two dural folds?
a) Falx Cerebri and Falx Cerebelli
b) Tentorium Cerebelli and Diaphragma Sellae
c) Falx Cerebri and Tentorium Cerebelli
d) Falx Cerebelli and Tentorium Cerebelli
Explanation: The dura mater forms septa that separate different cranial compartments. The Falx Cerebri is the vertical partition between the cerebral hemispheres, and the Tentorium Cerebelli is the horizontal partition between the cerebrum and cerebellum. The Straight Sinus is located exactly at the line of attachment where the broad posterior base of the Falx Cerebri meets the superior surface of the Tentorium Cerebelli. This anatomical positioning allows it to receive the inferior sagittal sinus running in the free edge of the falx and the great cerebral vein from beneath the splenium of the corpus callosum. Therefore, the correct answer is c) Falx Cerebri and Tentorium Cerebelli.
4. A 30-year-old female on oral contraceptives presents with progressive headache, nausea, and altered sensorium. MRI shows hyperintensity in the bilateral thalami on T2-weighted images. This finding is highly suggestive of thrombosis in which venous structure?
a) Superior Sagittal Sinus
b) Straight Sinus
c) Sigmoid Sinus
d) Cavernous Sinus
Explanation: The thalami and basal ganglia are deep brain structures drained by the internal cerebral veins, which unite to form the Great Vein of Galen. The Galenic system drains into the Straight Sinus. Blockage or thrombosis of the Straight Sinus (Deep Cerebral Venous Thrombosis) leads to venous congestion and infarction of these deep structures. Since the internal cerebral veins are paired structures draining into a single midline system, obstruction often results in symmetrical, bilateral thalamic edema or infarction. This is a hallmark radiological sign distinguishing deep system thrombosis from arterial strokes or superficial sinus thrombosis. Therefore, the correct answer is b) Straight Sinus.
5. Which of the following veins is a direct tributary to the Straight Sinus?
a) Superior Anastomotic Vein of Trolard
b) Inferior Anastomotic Vein of Labbe
c) Superior Cerebellar Veins
d) Superficial Middle Cerebral Vein
Explanation: The Straight Sinus receives blood from the Inferior Sagittal Sinus and the Great Cerebral Vein. In addition to these formative tributaries, it also receives venous drainage from the posterior cranial fossa. Specifically, the Superior Cerebellar Veins, which drain the superior aspect of the cerebellum (vermis and hemispheres), ascend to drain into the Straight Sinus or the Great Cerebral Vein. The veins of Trolard, Labbe, and the Superficial Middle Cerebral vein are part of the superficial venous system draining the cerebral cortex into the Superior Sagittal or Transverse sinuses, not the deep straight sinus. Therefore, the correct answer is c) Superior Cerebellar Veins.
6. During a neurosurgical approach to a pineal region tumor, the surgeon must be extremely careful to preserve the large venous structure located immediately superior and posterior to the pineal gland. This structure is the:
a) Basal Vein of Rosenthal
b) Great Cerebral Vein of Galen
c) Internal Cerebral Vein
d) Cavernous Sinus
Explanation: The pineal gland is located in the quadrigeminal cistern, tucked under the splenium of the corpus callosum. The Great Cerebral Vein of Galen curves around the splenium to join the inferior sagittal sinus and form the straight sinus. It is located immediately superior and dorsal to the pineal gland. Tumors of the pineal region (pinealomas) can compress the cerebral aqueduct causing hydrocephalus and can also compress or envelop the Great Cerebral Vein. Surgical damage to this vein or the receiving straight sinus can result in catastrophic venous infarction of the thalamus and midbrain. Therefore, the correct answer is b) Great Cerebral Vein of Galen.
7. The cross-sectional shape of the Straight Sinus is typically described as:
a) Circular
b) Triangular
c) Oval
d) Flat
Explanation: Dural venous sinuses are spaces between the periosteal and meningeal layers of the dura (or within duplications of the meningeal layer). The Superior Sagittal Sinus is triangular in cross-section. Similarly, the Straight Sinus is also typically triangular in cross-section. This is because it is formed at the junction of three dural sheets: the two leaves of the tentorium cerebelli (forming the base and sides) and the falx cerebri (forming the apex). This geometric shape is distinct and can be visualized on orthogonal MRI sequences or venography. Therefore, the correct answer is b) Triangular.
8. A patient with dehydration develops a severe headache. CT venogram shows a filling defect in the Torcular Herophili extending anteriorly into the midline tentorial structure. Which sinus is occluded?
a) Occipital Sinus
b) Sigmoid Sinus
c) Straight Sinus
d) Superior Petrosal Sinus
Explanation: To answer this, one must visualize the anatomy. The Torcular Herophili (Confluence of Sinuses) is the posterior hub. Extending *anteriorly* from the Torcular, along the midline of the tentorium cerebelli, is the Straight Sinus. The Occipital sinus extends inferiorly towards the foramen magnum. The Transverse sinuses extend laterally. The Superior Sagittal sinus extends superiorly and anteriorly along the skull vault. Therefore, a thrombus extending "anteriorly into the midline tentorial structure" describes the location of the Straight Sinus. Dehydration is a common risk factor for such dural sinus thromboses. Therefore, the correct answer is c) Straight Sinus.
9. The Inferior Sagittal Sinus, which contributes to the formation of the Straight Sinus, runs in the:
a) Attached superior margin of the Falx Cerebri
b) Free inferior margin of the Falx Cerebri
c) Free margin of the Tentorium Cerebelli
d) Attached margin of the Falx Cerebelli
Explanation: The Falx Cerebri has two margins. The convex, superior attached margin (attached to the skull vault) contains the Superior Sagittal Sinus. The concave, free inferior margin, which arches over the corpus callosum, contains the Inferior Sagittal Sinus. This sinus runs posteriorly in this free edge until it meets the tentorium cerebelli, where it is joined by the Great Cerebral Vein to form the Straight Sinus. The free margin of the tentorium contains the trochlear nerve but no major sinus. The attached margin of the falx cerebelli contains the occipital sinus. Therefore, the correct answer is b) Free inferior margin of the Falx Cerebri.
10. Which embryonic structure is the precursor to the Great Cerebral Vein of Galen, the thrombosis of which results in the absence of the straight sinus formation?
a) Median Prosencephalic Vein
b) Primary Head Vein
c) Ventral Longitudinal Neural Vein
d) Anterior Cardinal Vein
Explanation: The development of the cerebral venous system is complex. The Median Prosencephalic Vein (MPV) is a transient embryonic midline vein that drains the choroid plexus of the forebrain. Normally, the MPV regresses as the internal cerebral veins develop, and its caudal remnant becomes the Great Cerebral Vein of Galen. If the MPV fails to regress (often due to high flow from an AVM), it persists as a "Vein of Galen Malformation." The normal formation of the Straight Sinus is dependent on the proper development of this deep venous drainage system. Therefore, the correct answer is a) Median Prosencephalic Vein.
Chapter: Head and Neck
Topic: Lymphatic Drainage of the Neck
Subtopic: Anterior Cervical Lymph Nodes (The Delphian Node)
Key Definitions & Concepts
Delphian Node: A specific group of prelaryngeal lymph nodes located on the cricothyroid membrane in the midline of the neck.
Level VI Lymph Nodes: The central compartment lymph nodes of the neck, bounded by the hyoid bone superiorly, suprasternal notch inferiorly, and carotid arteries laterally. Delphian nodes belong to this group.
Cricothyroid Membrane: The connective tissue membrane connecting the cricoid and thyroid cartilages; the anatomical bed for the Delphian node.
Papillary Thyroid Carcinoma: The most common type of thyroid cancer; it frequently metastasizes to the central compartment (Level VI), including the Delphian node.
Pyramidal Lobe: An embryological remnant of the thyroid gland extending superiorly from the isthmus; often located adjacent to the Delphian nodes.
Subglottic Larynx: The lower portion of the voice box; lymphatic drainage from this area flows to the prelaryngeal (Delphian) and pretracheal nodes.
Oracle of Delphi: The historical origin of the name; enlargement of this node was said to "foretell" a poor prognosis (usually laryngeal cancer), much like a prophecy.
Hashimoto’s Thyroiditis: An autoimmune thyroid condition that can occasionally cause benign reactive hyperplasia of the Delphian nodes.
Pretracheal Nodes: Lymph nodes located in front of the trachea, inferior to the thyroid isthmus, distinct from the prelaryngeal (Delphian) nodes.
Thyroid Isthmus: The bridge of tissue connecting the two thyroid lobes; tumors here preferentially drain to the Delphian nodes.
[Image of Lymph nodes of the neck Level VI]
Lead Question - 2016
Delphian nodes are ?
a) Prelaryngeal nodes
b) Occipital nodes
c) Coeliac nodes
d) None of the above
Explanation: The Delphian nodes are a specific subset of the anterior cervical lymph nodes. Anatomically, they are classified as Prelaryngeal nodes because they are situated immediately in front of the larynx, resting upon the cricothyroid membrane (or ligament). They are usually one to four small nodes found in the midline. They receive lymphatic drainage from the subglottic region of the larynx, the pyriform sinus, the isthmus of the thyroid gland, and the thyroid lobes. Their enlargement is clinically significant as it may indicate metastasis from thyroid carcinoma (especially papillary) or laryngeal carcinoma. Occipital nodes are at the back of the head. Coeliac nodes are abdominal. Therefore, the correct answer is a) Prelaryngeal nodes.
1. The Delphian node is anatomically located in the midline of the neck resting on which specific structure?
a) Thyrohyoid membrane
b) Cricothyroid membrane
c) Tracheal rings
d) Hyoid bone
Explanation: Precise anatomical knowledge is required for neck dissections. The Delphian node (or nodes, as there can be up to four) is consistently found in the midline of the neck, located in the space between the cricoid cartilage and the thyroid cartilage. This space is bridged by the Cricothyroid membrane (ligament). The node lies superficial to this membrane, often between the two cricothyroid muscles. It is distinct from the pretracheal nodes which lie lower down on the tracheal rings. This location makes it a critical landmark during laryngeal and thyroid surgeries. Therefore, the correct answer is b) Cricothyroid membrane.
2. A 45-year-old female presents with a palpable midline neck mass just above the thyroid isthmus. Biopsy confirms Papillary Thyroid Carcinoma. During the total thyroidectomy, the surgeon performs a central neck dissection. Which nodal level includes the Delphian node?
a) Level I
b) Level III
c) Level VI
d) Level IV
Explanation: The lymph nodes of the neck are divided into levels I through VII for surgical classification. The Central Compartment of the neck is designated as Level VI. This compartment is bounded superiorly by the hyoid bone, inferiorly by the suprasternal notch, and laterally by the carotid arteries. It contains the prelaryngeal (Delphian), pretracheal, and paratracheal lymph nodes. Since the Delphian node is prelaryngeal, it is a key component of Level VI. Dissection of this level is standard for many thyroid cancers. Level I is submental/submandibular. Levels III and IV are lateral jugular. Therefore, the correct answer is c) Level VI.
3. Which anatomical structure of the thyroid gland is most closely associated with the Delphian lymph node and may often be found extending superiorly towards it?
a) Pyramidal lobe
b) Superior parathyroid gland
c) Tubercle of Zuckerkandl
d) Inferior thyroid artery
Explanation: The Pyramidal lobe is an embryological remnant of the thyroglossal duct that extends superiorly from the isthmus of the thyroid gland. It travels in the midline towards the hyoid bone. Due to its location, it lies in the immediate vicinity of the Delphian (prelaryngeal) nodes. During thyroidectomy, surgeons must carefully dissect this region to remove the pyramidal lobe and inspect the Delphian nodes. Failure to remove the pyramidal lobe is a common cause of recurrence in thyroid disease. The tubercle of Zuckerkandl is lateral. Parathyroids are posterior. Therefore, the correct answer is a) Pyramidal lobe.
4. A 60-year-old male with a history of heavy smoking presents with hoarseness. Laryngoscopy reveals a tumor in the subglottic region. Enlargement of the Delphian node in this patient would suggest lymphatic spread from which primary site?
a) Nasopharynx
b) Subglottic larynx
c) Oral tongue
d) Tonsil
Explanation: The lymphatic drainage of the larynx is compartmentalized. The supraglottic larynx drains to the upper deep cervical nodes (Level II/III). The glottic larynx has sparse lymphatics. However, the Subglottic larynx (the area below the vocal cords) has a distinct drainage pattern. It drains anteriorly through the cricothyroid membrane directly to the prelaryngeal (Delphian) nodes and the pretracheal nodes. Therefore, a palpable Delphian node in a smoker with hoarseness is a strong indicator of subglottic involvement or primary subglottic carcinoma. Nasopharynx drains to retropharyngeal nodes. Therefore, the correct answer is b) Subglottic larynx.
5. The term "Delphian" was coined because the enlargement of this node was historically believed to:
a) Protect the laryngeal nerves
b) Foretell a poor prognosis
c) Mimic a thyroglossal cyst
d) Indicate benign disease
Explanation: The name "Delphian" node is derived from the Oracle of Delphi in Greek mythology. The Oracle was known for making prophecies. Similarly, in the era before advanced imaging, a palpable or enlarged prelaryngeal node was considered an ominous sign that "prophesied" or foretold a poor prognosis. It typically indicated that a laryngeal or thyroid cancer had already breached its primary compartment and spread to the lymphatics, often correlating with advanced disease or higher recurrence rates. It does not protect nerves or inherently indicate benign disease. Therefore, the correct answer is b) Foretell a poor prognosis.
6. While performing a neck ultrasound for a patient with Hashimoto's thyroiditis, the radiologist notes a slightly enlarged, reactive node anterior to the cricothyroid membrane. This finding is:
a) Pathognomonic for Lymphoma
b) An indication for immediate neck dissection
c) A known benign presentation in thyroiditis
d) Impossible, as Delphian nodes never swell in benign disease
Explanation: While Delphian node enlargement is classically associated with malignancy (thyroid or laryngeal cancer), it is not exclusive to cancer. Conditions causing significant inflammation of the thyroid gland, such as Hashimoto’s thyroiditis (chronic lymphocytic thyroiditis), can lead to reactive hyperplasia of the perithyroidal lymph nodes, including the Delphian node. In this clinical context, a mildly enlarged node with benign sonographic features (preserved hilum, oval shape) is often reactive. It is not pathognomonic for lymphoma nor does it strictly require dissection without other suspicious signs. Therefore, the correct answer is c) A known benign presentation in thyroiditis.
7. A surgeon is removing a thyroglossal duct cyst in a child (Sistrunk procedure). During the dissection near the hyoid bone and thyroid membrane, they encounter lymph nodes. How can the surgeon distinguish a Delphian node from the cyst?
a) Nodes are always cystic
b) Cysts are solid and firm
c) Nodes are solid tissue, cysts are fluid-filled
d) Location; cysts never occur in the midline
Explanation: Differential diagnosis of a midline neck mass includes the Delphian node, thyroglossal duct cyst, and dermoid cyst. A key distinction is consistency and content. A Thyroglossal duct cyst is a fluid-filled remnant of the thyroglossal duct and will transilluminate or show fluid characteristics on ultrasound. A Delphian node is a solid structure composed of lymphoid tissue. Furthermore, the cyst typically moves with tongue protrusion (due to hyoid attachment), while the node does not move as distinctly with the tongue, though both move with swallowing. Therefore, the correct answer is c) Nodes are solid tissue, cysts are fluid-filled.
8. If cancer cells pass through the Delphian node, the subsequent (secondary) drainage site is typically the:
a) Submental nodes (Level I)
b) Pretracheal and Lower Deep Cervical nodes
c) Retroauricular nodes
d) Axillary nodes
Explanation: Lymphatic drainage follows a predictable stepwise pattern. The Delphian node acts as a primary sentinel node for the subglottis and thyroid isthmus. Efferent vessels from the Delphian node typically drain inferiorly and laterally. They empty into the Pretracheal nodes (lower Level VI) and the Lower Deep Cervical nodes (Level IV, and sometimes Level III). They do not drain superiorly to the submental nodes or posteriorly to the retroauricular nodes. Understanding this flow guides the extent of neck dissection required (e.g., clearing Level VI and IV). Therefore, the correct answer is b) Pretracheal and Lower Deep Cervical nodes.
9. In the context of Papillary Thyroid Carcinoma, the presence of a positive (metastatic) Delphian node is a strong predictor for:
a) Distant metastasis to the brain
b) Further metastasis to lateral neck nodes (Level III/IV)
c) Primary tumor in the parathyroid
d) Skip metastasis to the axilla
Explanation: The status of the Delphian node serves as a "barometer" for the rest of the neck. Clinical studies have shown that in patients with Papillary Thyroid Carcinoma, a metastatic Delphian node carries a high statistical correlation with significant nodal burden elsewhere. Specifically, it strongly predicts further metastasis to the lateral neck nodes (Levels III and IV) and the rest of the central compartment. Consequently, finding a positive Delphian node intraoperatively often prompts the surgeon to perform a more extensive lymph node dissection rather than just a thyroidectomy. Therefore, the correct answer is b) Further metastasis to lateral neck nodes (Level III/IV).
10. Which small artery or vascular arcade is often encountered near the Delphian node on the cricothyroid membrane?
a) Superior laryngeal artery
b) Cricothyroid artery
c) Lingual artery
d) Transverse cervical artery
Explanation: The cricothyroid membrane is not avascular. It is crossed by a small vascular anastomosis formed by the Cricothyroid artery. The cricothyroid artery is a branch of the superior thyroid artery. It runs transversely across the upper part of the cricothyroid ligament to anastomose with its fellow from the opposite side. The Delphian nodes usually lie in close proximity to this vascular arch. Surgeons performing cricothyrotomies or dissecting these nodes must be aware of this vessel to maintain hemostasis. The superior laryngeal artery pierces the thyrohyoid membrane, which is superior to this level. Therefore, the correct answer is b) Cricothyroid artery.
Chapter: Head and Neck
Topic: Deep Cervical Fascia
Subtopic: Retropharyngeal and Danger Spaces
Key Definitions & Concepts
Retropharyngeal Space (True): A potential space located immediately posterior to the pharynx, bounded anteriorly by the buccopharyngeal fascia and posteriorly by the alar fascia.
Danger Space: A potential space located posterior to the true retropharyngeal space, bounded anteriorly by the alar fascia and posteriorly by the prevertebral fascia.
Buccopharyngeal Fascia: A thin lamina of fascia that covers the exterior surface of the buccinator muscle and the pharyngeal constrictor muscles.
Alar Fascia: A subdivision of the deep layer of deep cervical fascia that separates the retropharyngeal space from the danger space; it extends from the skull base to the level of T4 (carina).
Prevertebral Fascia: The facial layer covering the vertebral column and paraspinal muscles; it forms the floor of the posterior triangle and the posterior limit of the danger space.
Mediastinitis: Inflammation of the mediastinum; a feared complication of retropharyngeal or danger space infections tracking inferiorly.
Nodes of Rouviere: Lateral retropharyngeal lymph nodes found in young children; their suppuration is a common cause of retropharyngeal abscesses in pediatrics.
Carotid Sheath: The condensation of fascia surrounding the internal jugular vein, common carotid artery, and vagus nerve; forms the lateral boundary of the retropharyngeal space.
Prevertebral Space: The space posterior to the prevertebral fascia; infections here (like Pott's disease) tend to track laterally or down the psoas sheath rather than into the mediastinum.
Grisel's Syndrome: Non-traumatic atlanto-axial subluxation secondary to inflammatory ligamentous laxity from a retropharyngeal infection.
[Image of Sagittal view of neck fascia and spaces]
Lead Question - 2016
Extension of the retropharyngeal space is between ?
a) Alar fascia and buccopharyngeal fascia
b) buccopharyngeal fascia and prevertebral fascia
c) Alar fascia and Prevertebral fascia
d) None
Explanation: The anatomy of the spaces behind the pharynx is defined by the layers of the deep cervical fascia. The "True" Retropharyngeal space is the anterior-most interfascial space. It is bounded anteriorly by the buccopharyngeal fascia (which covers the constrictor muscles) and posteriorly by the alar fascia. The alar fascia is a thin layer that separates this space from the "Danger Space" behind it. The Danger Space lies between the alar fascia and the prevertebral fascia. Therefore, strictly anatomically speaking, the extension of the true retropharyngeal space is between the alar and buccopharyngeal fascias. If the alar fascia is not considered (in older or simplified classifications), it might be described as between buccopharyngeal and prevertebral, but option (a) is the precise anatomical answer. The correct answer is a) Alar fascia and buccopharyngeal fascia.
1. The "True" Retropharyngeal space communicates inferiorly with the superior mediastinum. At which anatomical vertebral level does this space typically terminate due to the fusion of fascial layers?
a) T1
b) C7
c) T4 (Carina)
d) L1
Explanation: It is vital to distinguish the inferior extent of the retropharyngeal space from the danger space. The True Retropharyngeal space (between buccopharyngeal and alar fascia) does not extend the full length of the thorax. The alar fascia fuses with the buccopharyngeal fascia (or the visceral fascia of the esophagus) roughly at the level of the bifurcation of the trachea or the T4 vertebra. This limits the spread of infection from a true retropharyngeal abscess to the superior mediastinum only. In contrast, the Danger space extends much further down to the diaphragm. Therefore, the correct answer is c) T4 (Carina).
2. A 3-year-old child presents with high fever, difficulty swallowing (dysphagia), and drooling. A lateral neck X-ray reveals a widening of the prevertebral soft tissue shadow. This pathology is most likely due to suppuration of which specific lymph nodes?
a) Jugulodigastric nodes
b) Submental nodes
c) Retropharyngeal nodes (of Rouviere)
d) Supraclavicular nodes
Explanation: Retropharyngeal abscesses are predominantly seen in children under the age of 5. This age predilection is due to the presence of the Retropharyngeal lymph nodes (Nodes of Rouviere) located in the retropharyngeal space. These nodes drain the nasopharynx, adenoids, and posterior nasal cavity. They typically atrophy and regress by age 4-5. In young children, upper respiratory infections can spread to these nodes, leading to suppuration and abscess formation. In adults, retropharyngeal abscesses are usually secondary to trauma (e.g., fishbone ingestion) rather than nodal suppuration. Jugulodigastric nodes are in the lateral neck. Therefore, the correct answer is c) Retropharyngeal nodes (of Rouviere).
3. Which anatomical space is clinically termed the "Danger Space" because it provides a pathway for infection to spread from the skull base directly to the diaphragm?
a) Space between Buccopharyngeal and Alar fascia
b) Space between Alar and Prevertebral fascia
c) Space posterior to Prevertebral fascia
d) Pretracheal space
Explanation: The Danger Space is so named because it contains loose areolar tissue that offers little resistance to the spread of fluid or infection. It is located posterior to the Alar fascia and anterior to the Prevertebral fascia. Unlike the true retropharyngeal space which ends at T4, the Danger Space is continuous inferiorly through the posterior mediastinum all the way to the diaphragm. This allows an infection starting in the neck to rapidly cause widespread mediastinitis and empyema, which carries a high mortality rate. The space behind the prevertebral fascia is the prevertebral space. Therefore, the correct answer is b) Space between Alar and Prevertebral fascia.
4. A patient with Pott’s disease (Tuberculosis of the cervical spine) develops a cold abscess. This collection is most likely confined to which space?
a) True Retropharyngeal Space
b) Danger Space
c) Prevertebral Space
d) Carotid Sheath
Explanation: Tuberculosis of the spine affects the vertebral bodies. The pus formed (cold abscess) lifts the periosteum and the fascia immediately covering the bone. The fascia covering the vertebrae is the Prevertebral fascia. Therefore, a tuberculous abscess typically forms posterior to the prevertebral fascia, in the Prevertebral Space. An acute pyogenic abscess from a throat infection typically forms anterior to the prevertebral fascia (in the retropharyngeal or danger space). A prevertebral abscess typically creates a midline bulge but tracks laterally towards the posterior triangle rather than inferiorly into the mediastinum. Therefore, the correct answer is c) Prevertebral Space.
5. The lateral boundary of the retropharyngeal space is formed by which anatomical structure?
a) Sternocleidomastoid muscle
b) Carotid Sheath
c) Styloid process
d) Parotid gland
Explanation: The retropharyngeal space is a midline space. However, it does not extend indefinitely to the sides. Laterally, the buccopharyngeal fascia (anterior wall) and the alar/prevertebral fascia (posterior wall) fuse with the fascia surrounding the major vascular bundle of the neck. This bundle is the Carotid Sheath, containing the common carotid artery, internal jugular vein, and vagus nerve. Thus, the carotid sheath acts as the lateral partition or boundary, preventing the spread of retropharyngeal infections further laterally into the neck, unless the sheath itself is eroded. Therefore, the correct answer is b) Carotid Sheath.
6. A 40-year-old man ingests a sharp chicken bone which perforates the posterior pharyngeal wall. Two days later, he develops chest pain and signs of posterior mediastinitis. The infection most likely tracked inferiorly via the:
a) Pretracheal space
b) True Retropharyngeal space
c) Danger Space
d) Submandibular space
Explanation: While the true retropharyngeal space connects to the superior mediastinum, the Danger Space is the critical pathway for widespread posterior mediastinitis. A sharp object perforating the posterior pharyngeal wall penetrates the buccopharyngeal fascia (entering the true RPS). If it penetrates slightly deeper, passing the thin alar fascia, it enters the Danger Space. Once in the Danger Space, gravity and negative intrathoracic pressure facilitate the rapid spread of infection to the level of the diaphragm. Given the severity and rapid tracking described, involvement of the Danger Space is the primary concern. Therefore, the correct answer is c) Danger Space.
7. Which fascial layer forms the anterior boundary of the True Retropharyngeal Space?
a) Pretracheal fascia
b) Investing layer of deep cervical fascia
c) Buccopharyngeal fascia
d) Alar fascia
Explanation: The retropharyngeal space lies directly behind the pharynx and esophagus. The muscular wall of the pharynx (constrictor muscles) and esophagus is covered on its external aspect by a thin, distinct layer of fascia known as the Buccopharyngeal fascia. This fascia separates the visceral unit (pharynx/esophagus) from the potential space behind it. Consequently, the Buccopharyngeal fascia forms the anterior boundary of the retropharyngeal space. The Investing layer surrounds the whole neck. The Pretracheal fascia is anterior to the trachea. The Alar fascia is the posterior boundary. Therefore, the correct answer is c) Buccopharyngeal fascia.
8. On a lateral soft tissue radiograph of the neck, a widening of the retropharyngeal soft tissue shadow is a diagnostic sign of abscess. In an adult, the thickness of this shadow at the level of C6 should normally not exceed:
a) 7 mm
b) 10 mm
c) 22 mm
d) 30 mm
Explanation: Radiographic measurements are crucial for diagnosing retropharyngeal pathology. The "Who's Afraid of the Dark" mnemonic helps: at C2, the prevertebral soft tissue thickness should not exceed 7 mm (both kids and adults). At C6 (retro-tracheal level), the allowance is larger. In adults, it should not exceed 22 mm (some sources say 14-22mm, but 22mm is the classic cutoff for "definitely abnormal"). In children, the C6 measurement is generally < 14 mm. A measurement > 22 mm at C6 in an adult strongly suggests a retropharyngeal abscess or mass. Therefore, the correct answer is c) 22 mm.
9. A child with a retropharyngeal abscess develops a "Cock Robin" head posture (torticollis). This complication, known as Grisel's Syndrome, is caused by:
a) Spasm of the sternocleidomastoid due to nerve irritation
b) Inflammatory subluxation of the atlanto-axial joint
c) Direct infection of the cervical vertebrae
d) Meningitis spreading from the abscess
Explanation: Grisel's Syndrome is a non-traumatic subluxation of the atlanto-axial joint (C1-C2). The inflammation from the nearby retropharyngeal abscess causes hyperemia and laxity of the transverse ligament of the atlas and other periodontoid ligaments. This ligamentous weakness allows the atlas to rotatory subluxate on the axis, causing the patient to hold their head in a distinctive tilted and rotated position (torticollis) to relieve pain. It is not primarily due to muscle spasm alone or direct bone infection (osteomyelitis), but rather inflammatory ligamentous laxity. Therefore, the correct answer is b) Inflammatory subluxation of the atlanto-axial joint.
10. The Alar fascia, which separates the True Retropharyngeal space from the Danger Space, is embryologically and anatomically considered a subdivision of which layer?
a) Superficial cervical fascia
b) Investing layer of deep cervical fascia
c) Pretracheal layer of deep cervical fascia
d) Deep layer (Prevertebral) of deep cervical fascia
Explanation: The Deep Cervical Fascia is divided into three layers: Investing, Pretracheal, and Prevertebral (Deep). The Prevertebral layer (Deep layer) further splits into two laminas: an anterior lamina and a posterior lamina. The anterior lamina is the Alar fascia, and the posterior lamina is the Prevertebral fascia proper. This splitting creates the Danger Space between them. Thus, the Alar fascia is a component of the deep (prevertebral) layer of the deep cervical fascia. It is not part of the superficial, investing, or pretracheal layers. Therefore, the correct answer is d) Deep layer (Prevertebral) of deep cervical fascia.
Chapter: Head and Neck; Topic: Cranial Nerves; Subtopic: Trigeminal Nerve (CN V) Branches and Distribution
Key Definitions & Concepts
Trigeminal Nerve (CN V): The largest cranial nerve, primarily responsible for sensory innervation of the face and motor innervation of the muscles of mastication.
Gasserian Ganglion: Also known as the Trigeminal ganglion, it is the sensory ganglion of CN V located in Meckel's cave.
Ophthalmic Nerve (V1): The first division of CN V; purely sensory, supplying the scalp, forehead, orbit, and nose. Exits via Superior Orbital Fissure.
Maxillary Nerve (V2): The second division of CN V; purely sensory, supplying the mid-face, nasal cavity, and upper teeth. Exits via Foramen Rotundum.
Mandibular Nerve (V3): The third division of CN V; a mixed nerve (sensory and motor), supplying the lower jaw, tongue, and mastication muscles. Exits via Foramen Ovale.
Optic Nerve (CN II): A separate cranial nerve responsible for vision, not a branch of the Trigeminal.
Tic Douloureux: Trigeminal Neuralgia, a condition characterized by severe, shooting pain in the distribution of one or more branches of CN V.
Corneal Reflex: A protective blink reflex where the afferent limb is V1 (Nasociliary branch) and the efferent limb is CN VII.
Muscles of Mastication: Masseter, Temporalis, Medial Pterygoid, and Lateral Pterygoid; all innervated by the Mandibular nerve (V3).
Great Auricular Nerve: A branch of the cervical plexus (C2, C3), NOT the Trigeminal nerve, supplying skin over the parotid and ear.
[Image of Trigeminal nerve branches]
Lead Question - 2016
All of the following are main branches of Trigeminal nerve except ?
a) Mandibular nerve
b) Maxillary nerve
c) Ophthalmic nerve
d) Optic nerve
Explanation: The **Trigeminal nerve (CN V)** is named "tri-" because it divides into three major distinct branches coming off the trigeminal ganglion. These are the **Ophthalmic nerve (V1)**, the **Maxillary nerve (V2)**, and the **Mandibular nerve (V3)**. These provide sensory supply to the upper, middle, and lower thirds of the face respectively. The **Optic nerve** is a completely separate cranial nerve (CN II) responsible for the special sense of vision. It is not a branch of the trigeminal nerve. Confusing the names "Ophthalmic" (related to the eye) and "Optic" (vision) is a common student error. Therefore, the correct answer is d) Optic nerve.
1. A 55-year-old male complains of severe, electric-shock-like pain affecting his right cheek and upper lip. The pain is triggered by brushing his teeth. Which specific division of the Trigeminal nerve is involved?
a) Ophthalmic nerve (V1)
b) Maxillary nerve (V2)
c) Mandibular nerve (V3)
d) Facial nerve (VII)
Explanation: This clinical presentation is classic for **Trigeminal Neuralgia** (Tic Douloureux). The pain distribution is the key to localization. The cheek, upper lip, upper teeth, and side of the nose are supplied by the **Maxillary nerve (V2)**. The trigger zone (brushing teeth/touching the face) is typical. The Ophthalmic nerve supplies the forehead and eye. The Mandibular nerve supplies the lower jaw and lower teeth. The Facial nerve is motor to the face, not sensory for this type of pain. Therefore, the correct answer is b) Maxillary nerve (V2).
2. Which of the three divisions of the Trigeminal nerve contains motor fibers associated with the First Branchial Arch?
a) Ophthalmic nerve
b) Maxillary nerve
c) Mandibular nerve
d) All three divisions
Explanation: The Trigeminal nerve is the nerve of the **First Branchial Arch**. However, not all its divisions carry motor fibers. The Ophthalmic (V1) and Maxillary (V2) nerves are purely sensory. The motor root of the Trigeminal nerve bypasses the ganglion and joins only the **Mandibular nerve (V3)**. Consequently, V3 is the only division that is a "mixed" nerve, supplying the muscles of mastication (masseter, temporalis, pterygoids), mylohyoid, anterior belly of digastric, tensor veli palatini, and tensor tympani. Therefore, the correct answer is c) Mandibular nerve.
3. A patient presents with loss of sensation on the tip of the nose following a trauma. This area is supplied by the External Nasal nerve, which is a branch of:
a) Nasociliary nerve
b) Infraorbital nerve
c) Zygomatic nerve
d) Supratrochlear nerve
Explanation: The sensory supply of the nose is complex. The tip of the nose (dorsum) is supplied by the **External Nasal nerve**. This nerve is a terminal continuation of the Anterior Ethmoidal nerve, which arises from the **Nasociliary nerve**. The Nasociliary nerve is a major branch of the Ophthalmic division (V1). The Infraorbital nerve (V2) supplies the side of the nose (ala). This distinction is clinically relevant in Herpes Zoster Ophthalmicus; vesicles on the tip of the nose (Hutchinson's sign) indicate Nasociliary involvement and high risk of corneal ulcers. Therefore, the correct answer is a) Nasociliary nerve.
4. The Trigeminal Ganglion (Gasserian Ganglion) is located in a depression on the petrous temporal bone known as Meckel's Cave. This cave is formed by a splitting of which meningeal layer?
a) Pia mater
b) Dura mater
c) Arachnoid mater
d) Periosteum only
Explanation: Meckel's Cave (Trigeminal Cave) is an anatomical pouch located in the middle cranial fossa. It contains the Trigeminal ganglion and the roots of the nerve bathed in cerebrospinal fluid. Structurally, it is formed by an invagination or splitting of the **Dura mater**. The dura splits into two layers (endosteal and meningeal) to enclose the ganglion. This location is crucial for neurosurgery and understanding the spread of pathologies like meningiomas or schwannomas in this region. Therefore, the correct answer is b) Dura mater.
5. Which of the following muscles is NOT innervated by the Mandibular Nerve (V3)?
a) Tensor Tympani
b) Tensor Veli Palatini
c) Buccinator
d) Mylohyoid
Explanation: The Mandibular nerve supplies muscles derived from the first pharyngeal arch. These include the muscles of mastication, plus four others: Mylohyoid, Anterior belly of Digastric, Tensor Tympani, and Tensor Veli Palatini. The **Buccinator** muscle, despite being in the cheek and pierced by the buccal branch of V3 (which is sensory), is a muscle of facial expression. Like all muscles of facial expression (derived from the second arch), the Buccinator is innervated by the **Facial Nerve (VII)**. Therefore, the correct answer is c) Buccinator.
6. During a neurological exam, the corneal reflex is absent in the left eye when the left cornea is touched, but present in the right eye when the right cornea is touched. However, when the left cornea is touched, the right eye blinks (consensual positive). Where is the lesion?
a) Left Trigeminal Nerve (V1)
b) Left Facial Nerve
c) Right Trigeminal Nerve
d) Right Facial Nerve
Explanation: Let's analyze the reflex arc. Afferent is V1, Efferent is VII. Stimulus Left Cornea -> Sensed by Left V1 -> Signal to Brainstem. Response Left Eye -> Blinks via Left VII. Response Right Eye -> Blinks via Right VII. Since touching the Left cornea causes the Right eye to blink (consensual), the **Left sensory limb (V1) is intact**. The brain received the signal. However, the Left eye did not blink. This means the **Left motor limb (Facial Nerve)** is unable to execute the command. This is a lower motor neuron lesion of the facial nerve. Therefore, the correct answer is b) Left Facial Nerve.
7. The skin over the angle of the mandible is a specific landmark for sensory testing. This area is supplied by which nerve?
a) Mandibular nerve (Auriculotemporal branch)
b) Great Auricular nerve
c) Lesser Occipital nerve
d) Facial nerve
Explanation: While the face is predominantly supplied by the Trigeminal nerve, there is a distinct exception. The skin over the **angle of the mandible** (and the parotid gland area) is NOT supplied by the Trigeminal nerve. Instead, it is supplied by the **Great Auricular nerve**, which is a branch of the Cervical Plexus (C2, C3). This is an important distinction to remember when mapping facial anesthesia, as sparing of the angle suggests a trigeminal lesion, while involvement suggests a cervical or wider issue. Therefore, the correct answer is b) Great Auricular nerve.
8. Which parasympathetic ganglion is functionally associated with the Maxillary Nerve (V2) for the secretion of the lacrimal gland?
a) Ciliary ganglion
b) Otic ganglion
c) Pterygopalatine ganglion
d) Submandibular ganglion
Explanation: The Trigeminal nerve branches often serve as pathways for autonomic fibers from other nerves ("hitchhiking"). The **Pterygopalatine ganglion** (also called Sphenopalatine) is suspended from the Maxillary nerve (V2) in the pterygopalatine fossa. Preganglionic parasympathetic fibers come from the Facial nerve (Greater Petrosal branch). Postganglionic fibers then hitchhike on the Zygomatic branch of V2, then the Lacrimal branch of V1 to supply the **Lacrimal gland** (tearing) and nasal glands. The Ciliary is for the eye (V1). The Otic is for the parotid (V3). The Submandibular is for salivary glands (V3). Therefore, the correct answer is c) Pterygopalatine ganglion.
9. A dentist performs an inferior alveolar nerve block. Ideally, this anesthetizes the mandibular teeth. However, if the needle is placed too posteriorly, it may penetrate the parotid gland and cause transient facial paralysis. This occurs because the Facial nerve and Mandibular nerve are separated by:
a) Sphenomandibular ligament
b) Ramus of the mandible
c) Medial Pterygoid muscle
d) Styloid process
Explanation: The Inferior Alveolar nerve (branch of V3) enters the mandibular foramen on the medial side of the ramus. The parotid gland wraps around the posterior border of the mandibular ramus. The Facial nerve runs through the substance of the parotid gland. If the injection is too posterior, it goes into the gland and hits the Facial nerve. The bony landmark separating the infratemporal fossa contents (V3) from the parotid bed is the **Ramus of the mandible**. Penetrating past the posterior border of the ramus puts the facial nerve at risk. Therefore, the correct answer is b) Ramus of the mandible.
10. The Meningeal branch of the Mandibular nerve (Nervus Spinosus) re-enters the cranial cavity through which foramen to supply the dura mater?
a) Foramen Ovale
b) Foramen Spinosum
c) Foramen Rotundum
d) Foramen Lacerum
Explanation: The Mandibular nerve (V3) exits the skull through the Foramen Ovale. Immediately after exiting, it gives off a recurrent meningeal branch known as the **Nervus Spinosus**. This nerve accompanies the Middle Meningeal Artery and re-enters the middle cranial fossa through the **Foramen Spinosum**. It supplies the dura mater of the middle cranial fossa and the mastoid air cells. This is a unique pathway where a nerve exits and then immediately turns back into the skull. Therefore, the correct answer is b) Foramen Spinosum.
Chapter: Head and Neck; Topic: Orbit and Eyelid; Subtopic: Extraocular Muscles and Eyelid Innervation
Key Definitions & Concepts
Orbicularis Oculi: The sphincter muscle of the eyelids responsible for closing the eye (blinking and tight closure), innervated by the Facial Nerve (CN VII).
Levator Palpebrae Superioris (LPS): The primary elevator of the upper eyelid responsible for opening the eye, innervated by the Oculomotor Nerve (CN III).
Antagonist Muscle: A muscle that opposes the action of another; LPS (opener) is the physiological antagonist to Orbicularis Oculi (closer).
Lagophthalmos: The inability to close the eyelids completely, commonly seen in Facial Nerve paralysis (Bell's Palsy).
Ptosis: Drooping of the upper eyelid, which can result from paralysis of the LPS (CN III palsy) or Müller's muscle.
Superior Tarsal Muscle (Müller's Muscle): A smooth muscle component assisting LPS in eyelid elevation, supplied by sympathetic fibers.
Palpebral Part: The inner portion of the orbicularis oculi involved in involuntary, gentle blinking.
Orbital Part: The outer, thicker portion of the orbicularis oculi involved in forceful closure (squinting).
Corneal Reflex: An involuntary blinking reflex elicited by stimulating the cornea; Afferent limb is CN V1, Efferent limb is CN VII.
Horner's Syndrome: A condition caused by sympathetic trunk damage, leading to partial ptosis, miosis, and anhidrosis.
[Image of Muscles of the eyelid anatomy]
Lead Question - 2016
Which muscle is antagonist to orbicularis oculi that is not supplied by facial nerve?
a) Levator Palpebrae superioris
b) Orbicularis oris
c) Superior oblique
d) Inferior oblique
Explanation: The question asks for the physiological antagonist to the Orbicularis Oculi. The function of the Orbicularis Oculi is to close the eyelids (sphincter action). Therefore, its antagonist must be a muscle that opens the eyelids (elevator). The primary elevator of the upper eyelid is the Levator Palpebrae Superioris (LPS). Crucially, the question specifies a muscle "not supplied by the facial nerve." The Orbicularis Oculi is supplied by the Facial Nerve (CN VII), whereas the Levator Palpebrae Superioris is supplied by the Oculomotor Nerve (CN III). The Orbicularis oris acts on the mouth. The Superior and Inferior obliques move the eyeball, not the eyelid. Therefore, the correct answer is a) Levator Palpebrae superioris.
1. A patient presents with complete inability to close the right eye. When asked to close the eyes tight, the right eyeball rolls upwards (Bell's Phenomenon). This condition is due to paralysis of which muscle?
a) Levator Palpebrae Superioris
b) Superior Rectus
c) Orbicularis Oculi
d) Frontalis
Explanation: The clinical sign described is Lagophthalmos, the inability to close the eyelid. This function is solely performed by the Orbicularis Oculi muscle. When the Facial Nerve (CN VII) is damaged (as in Bell's Palsy), the Orbicularis Oculi is paralyzed, and the eye remains open. The upward rolling of the eyeball (Bell's phenomenon) is a normal reflex that becomes visible because the eyelid fails to cover the eye. The Levator Palpebrae Superioris opposes this muscle; if it were paralyzed, the eye would be closed (ptosis). The Superior Rectus moves the eye, not the lid. Therefore, the correct answer is c) Orbicularis Oculi.
2. The Levator Palpebrae Superioris (LPS) splits into a superficial and deep lamella anteriorly. The smooth muscle component, known as Müller's muscle, is located in the deep lamella. Loss of innervation to this specific smooth muscle results in:
a) Complete Ptosis
b) Lagophthalmos
c) Partial Ptosis
d) Retraction of the eyelid
Explanation: The eyelid elevation is primarily achieved by the skeletal muscle portion of the LPS (supplied by CN III). However, the Superior Tarsal Muscle (Müller's muscle), which is smooth muscle located underneath the LPS, provides additional tonic elevation (about 1-2 mm). This muscle is innervated by the sympathetic nervous system. Paralysis of Müller's muscle, as seen in Horner's Syndrome, leads to a mild drooping of the eyelid, known as Partial Ptosis. Complete ptosis occurs only when the main LPS or CN III is damaged. Lagophthalmos is the opposite condition. Therefore, the correct answer is c) Partial Ptosis.
3. Which part of the Orbicularis Oculi muscle is primarily responsible for the involuntary, gentle closure of the eyelids, such as during blinking?
a) Orbital part
b) Lacrimal part
c) Palpebral part
d) Temporal part
Explanation: The Orbicularis Oculi is divided into three parts: orbital, palpebral, and lacrimal. The Palpebral part is contained within the eyelids themselves and consists of thin, pale fibers. It is responsible for gentle, involuntary closure, such as reflex blinking and sleeping. The Orbital part surrounds the orbital margin and is responsible for forceful, tight closure (squinting) to protect the eye from bright light or dust. The Lacrimal part (Horner's muscle) dilates the lacrimal sac to facilitate tear drainage. Differentiation between these parts is important in facial nerve assessment. Therefore, the correct answer is c) Palpebral part.
[Image of Orbicularis oculi parts]
4. A 50-year-old diabetic patient presents with a "down and out" position of the left eye and severe drooping of the left upper eyelid. The pupil is spared. Which nerve is most likely affected?
a) Trochlear nerve
b) Abducens nerve
c) Facial nerve
d) Oculomotor nerve
Explanation: The clinical picture of an eye deviated "down and out" combined with severe ptosis (drooping eyelid) is classic for an Oculomotor Nerve (CN III) palsy. CN III innervates the Levator Palpebrae Superioris (elevation of lid), as well as the Superior Rectus, Medial Rectus, Inferior Rectus, and Inferior Oblique muscles. The unopposed action of the Lateral Rectus (CN VI) and Superior Oblique (CN IV) pulls the eye down and out. Pupil sparing is common in diabetic microvascular infarction of the nerve, whereas compressive lesions (aneurysms) often involve the pupil. Therefore, the correct answer is d) Oculomotor nerve.
5. The nerve supply to the Levator Palpebrae Superioris travels through which division of the Oculomotor nerve?
a) Superior division
b) Inferior division
c) Ganglionic branch
d) Sympathetic root
Explanation: Upon entering the orbit through the Superior Orbital Fissure, the Oculomotor nerve (CN III) divides into a superior and an inferior division. The Superior division is the smaller of the two and ascends to supply the Superior Rectus muscle and the Levator Palpebrae Superioris (LPS). The Inferior division supplies the Medial Rectus, Inferior Rectus, and Inferior Oblique (and carries parasympathetics). Knowledge of this branching is crucial in orbital trauma or surgery, where specific divisions may be injured, causing isolated muscle deficits. Therefore, the correct answer is a) Superior division.
6. During a neurological exam, the physician touches the patient's cornea with a wisp of cotton. The patient blinks bilaterally. The efferent (motor) limb of this reflex arc is mediated by which nerve?
a) Ophthalmic nerve (V1)
b) Oculomotor nerve (III)
c) Facial nerve (VII)
d) Maxillary nerve (V2)
Explanation: The corneal reflex is a vital brainstem reflex. The afferent (sensory) limb carries sensation from the cornea via the nasociliary branch of the Ophthalmic division of the Trigeminal nerve (V1). The impulse goes to the sensory nucleus of V, then connects to the motor nucleus of VII bilaterally. The efferent (motor) limb, which causes the contraction of the Orbicularis Oculi muscles to produce the blink, is carried by the Facial nerve (VII). Loss of the reflex can indicate damage to V1, VII, or the brainstem connections. Therefore, the correct answer is c) Facial nerve (VII).
7. The Levator Palpebrae Superioris arises from the lesser wing of the sphenoid. What is its primary insertion point that allows it to elevate the eyelid?
a) Superior orbital margin
b) Superior Tarsal Plate and skin of upper eyelid
c) Sclera of the eyeball
d) Conjunctival fornix only
Explanation: The aponeurosis of the Levator Palpebrae Superioris (LPS) fans out anteriorly. Its primary and most functionally significant insertion is onto the anterior surface of the Superior Tarsal Plate and into the skin of the upper eyelid (creating the eyelid crease). It also has attachments to the conjunctiva. This insertion into the rigid tarsal plate allows the muscle to pull the eyelid structure upward effectively. If the aponeurosis detaches from the tarsal plate (dehiscence), it results in senile or involutional ptosis. Therefore, the correct answer is b) Superior Tarsal Plate and skin of upper eyelid.
8. A patient with a Pancoast tumor (apical lung tumor) presents with a constricted pupil (miosis), lack of sweating (anhidrosis) on one side of the face, and a mild drooping of the eyelid. Which muscle's dysfunction is responsible for the eyelid drooping in this case?
a) Orbicularis Oculi
b) Levator Palpebrae Superioris
c) Superior Tarsal (Müller's) Muscle
d) Frontalis
Explanation: The clinical presentation is classic for Horner's Syndrome, caused by the interruption of the sympathetic chain (often by a lung tumor at the thoracic inlet). While the main elevator of the lid is the LPS (CN III), the sympathetic nerves innervate the Superior Tarsal (Müller's) Muscle. This smooth muscle assists in holding the eye open. Loss of sympathetic tone results in paralysis of Müller's muscle, leading to a mild (1-2mm) ptosis. It is distinct from the severe ptosis of CN III palsy. Therefore, the correct answer is c) Superior Tarsal (Müller's) Muscle.
9. While performing surgery on the eyelid, a surgeon must be aware of the "Gray Line" on the eyelid margin. This anatomical landmark corresponds to which structure?
a) The junction of skin and conjunctiva
b) The muscle of Riolan (Part of Orbicularis Oculi)
c) The openings of the Meibomian glands
d) The insertion of the LPS aponeurosis
Explanation: The Gray Line is a crucial surgical landmark located on the intermarginal strip of the eyelid. It marks the avascular plane between the anterior lamella (skin and orbicularis muscle) and the posterior lamella (tarsal plate and conjunctiva). Anatomically, it corresponds to the pretarsal portion of the orbicularis oculi muscle, specifically a marginal bundle known as the Muscle of Riolan. Incision along this line allows the surgeon to split the eyelid into its anterior and posterior halves without excessive bleeding. Therefore, the correct answer is b) The muscle of Riolan (Part of Orbicularis Oculi).
10. Which branch of the Facial Nerve is primarily responsible for innervating the Orbicularis Oculi muscle to ensure eye closure?
a) Cervical branch
b) Marginal Mandibular branch
c) Temporal and Zygomatic branches
d) Buccal branch
Explanation: The Facial nerve branches into five terminal divisions within the parotid gland: Temporal, Zygomatic, Buccal, Marginal Mandibular, and Cervical. The Orbicularis Oculi covers the orbit and requires innervation from the upper branches. The Temporal and Zygomatic branches cross the zygomatic arch to supply the orbicularis oculi. The temporal branch supplies the upper part, and the zygomatic branch supplies the lower/lateral part. Injury to these specific branches during face-lift surgery or trauma can lead to inability to close the eye (lagophthalmos). Therefore, the correct answer is c) Temporal and Zygomatic branches.
Chapter: Head and Neck
Topic: Osteology and Cranial Nerves
Subtopic: Hypoglossal Canal and Nerve Anatomy
Key Definitions & Concepts
Hypoglossal Canal: A bony tunnel in the occipital bone, situated superior and anterior to the occipital condyle, transmitting CN XII.
Hypoglossal Nerve (CN XII): A pure motor nerve supplying all intrinsic and extrinsic muscles of the tongue, except the palatoglossus.
Meningeal Branch of Ascending Pharyngeal Artery: A small arterial branch that enters the cranium through the hypoglossal canal to supply the dura mater.
Emissary Vein: A venous connection often found in the hypoglossal canal connecting the marginal sinus/basilar plexus to the internal jugular vein.
Genioglossus Muscle: The major extrinsic tongue muscle responsible for protrusion; acts as the "safety muscle" of the airway.
Palatoglossus: The only tongue muscle not supplied by CN XII; it is supplied by the pharyngeal plexus (Vagus nerve).
General Somatic Efferent (GSE): The functional component classification of CN XII, as it innervates somatic muscles derived from occipital myotomes.
Glossoptosis: The falling back of the tongue into the pharynx, a life-threatening risk in bilateral hypoglossal nerve paralysis.
Carotid Triangle: An anatomical region in the neck where the hypoglossal nerve loops superficially across the internal and external carotid arteries.
Bulbar Palsy: A set of signs including tongue atrophy and fasciculations resulting from Lower Motor Neuron (LMN) lesions of cranial nerves IX, X, XI, and XII.
[Image of Hypoglossal canal anatomy]
Lead Question - 2016
Which of the following pass through the Hypoglossal canal?
a) Hypoglossal nerve
b) External jugular vein
c) Facial nerve
d) Mandibular nerve
Explanation: The hypoglossal canal (also known as the anterior condylar canal) is located in the occipital bone. Its primary content is the Hypoglossal nerve (CN XII), which exits the skull here to enter the carotid space. In addition to the nerve, the canal transmits a meningeal branch of the ascending pharyngeal artery and a small emissary vein (connecting the basilar plexus with the internal jugular vein). The External Jugular Vein is superficial in the neck. The Facial nerve exits via the stylomastoid foramen. The Mandibular nerve exits via the foramen ovale. Therefore, the correct answer is a) Hypoglossal nerve.
1. A small meningeal artery accompanies the hypoglossal nerve through the hypoglossal canal. This artery is a branch of which vessel?
a) Middle Meningeal artery
b) Ascending Pharyngeal artery
c) Occipital artery
d) Vertebral artery
Explanation: While the hypoglossal nerve is the most prominent structure in the hypoglossal canal, vascular structures also traverse it. A meningeal branch arises from the Ascending Pharyngeal artery (a branch of the External Carotid Artery) and enters the posterior cranial fossa through this canal to supply the dura mater. The vertebral artery enters through the foramen magnum. The middle meningeal artery enters through the foramen spinosum. The occipital artery typically gives off meningeal branches via the mastoid foramen or jugular foramen. Therefore, the correct answer is b) Ascending Pharyngeal artery.
2. A 65-year-old patient presents with deviation of the tongue to the left side upon protrusion. There is also evident atrophy of the left side of the tongue. Where is the lesion located?
a) Right Hypoglossal nerve
b) Left Hypoglossal nerve
c) Right Corticobulbar tract
d) Left Vagus nerve
Explanation: The Genioglossus muscle protrudes the tongue. It receives innervation from the ipsilateral Hypoglossal nerve. If the left Hypoglossal nerve is damaged, the left Genioglossus is paralyzed. The intact right Genioglossus pushes the tongue forward and towards the weak side. Thus, deviation to the left indicates a lesion of the Left Hypoglossal nerve. Furthermore, atrophy is a hallmark of a Lower Motor Neuron (LMN) lesion, confirming the injury is to the nerve itself rather than the upper motor neuron (corticobulbar tract), which would cause deviation without significant atrophy. Therefore, the correct answer is b) Left Hypoglossal nerve.
3. Which of the following muscles of the tongue is NOT innervated by the Hypoglossal nerve?
a) Hyoglossus
b) Styloglossus
c) Genioglossus
d) Palatoglossus
Explanation: The Hypoglossal nerve supplies all intrinsic muscles (superior/inferior longitudinal, transverse, vertical) and three of the four extrinsic muscles (Genioglossus, Hyoglossus, Styloglossus) of the tongue. The exception is the Palatoglossus muscle. Although it acts on the tongue, it is derived from the fourth branchial arch and is functionally associated with the soft palate. Consequently, it is innervated by the Pharyngeal Plexus, specifically carrying fibers from the Vagus nerve (CN X). This is a classic "exception to the rule" in head and neck anatomy. Therefore, the correct answer is d) Palatoglossus.
4. During a Carotid Endarterectomy, the surgeon must identify the Hypoglossal nerve to avoid injury. In the carotid triangle, the nerve crosses superficially over which two arteries?
a) Internal and External Carotid Arteries
b) Facial and Lingual Arteries
c) Superior Thyroid and Lingual Arteries
d) Common Carotid and Vertebral Arteries
Explanation: As the Hypoglossal nerve descends from the skull base, it initially runs between the Internal Jugular Vein and the Internal Carotid Artery. It then sweeps anteriorly, crossing lateral (superficial) to the Internal and External Carotid Arteries. This crossing point is a critical landmark in neck surgery. Specifically, it loops around the occipital artery branch of the ECA before running forward deep to the posterior belly of the digastric muscle to enter the submandibular region. Injury here results in ipsilateral tongue paralysis. Therefore, the correct answer is a) Internal and External Carotid Arteries.
[Image of Hypoglossal nerve course in neck]
5. The Hypoglossal nerve is classified as carrying which type of functional nerve fibers?
a) General Visceral Efferent (GVE)
b) Special Visceral Efferent (SVE)
c) General Somatic Efferent (GSE)
d) Special Visceral Afferent (SVA)
Explanation: The functional classification of cranial nerves depends on the origin of the muscles they supply. The muscles of the tongue (except palatoglossus) are derived from the occipital myotomes (somites). Nerves supplying skeletal muscles derived from somites are classified as General Somatic Efferent (GSE). SVE fibers supply muscles derived from pharyngeal arches (like CN V, VII, IX, X). GVE fibers are parasympathetic. Since CN XII is purely motor to somatic tongue musculature, it contains only GSE fibers. Therefore, the correct answer is c) General Somatic Efferent (GSE).
6. Bilateral damage to the Hypoglossal nerves (e.g., from a basilar skull fracture) poses an immediate life-threatening risk due to:
a) Inability to swallow (Dysphagia)
b) Airway obstruction (Glossoptosis)
c) Loss of taste sensation
d) Vocal cord paralysis
Explanation: The Genioglossus muscles are essential for pulling the tongue forward. In a conscious state and during sleep, their tone prevents the tongue from falling backward. If both Hypoglossal nerves are paralyzed (bilateral palsy), the tongue loses this support and falls posteriorly into the oropharynx, a condition called Glossoptosis. This causes severe Airway obstruction, which can be fatal if not managed (e.g., by intubation or oral airway insertion). While swallowing is affected, the airway obstruction is the immediate acute danger. Therefore, the correct answer is b) Airway obstruction (Glossoptosis).
7. The nucleus of the Hypoglossal nerve is located in which part of the brainstem?
a) Midbrain, at the level of the superior colliculus
b) Pons, at the level of the facial colliculus
c) Medulla, in the floor of the fourth ventricle
d) Spinal cord, C1-C2 segments
Explanation: The Hypoglossal nucleus is a somatic motor nucleus. It forms a vertical column in the medulla oblongata. Specifically, it is located in the tegmentum of the Medulla, in the floor of the fourth ventricle, near the midline. This area is macroscopically visible as the "Hypoglossal trigone." The fibers emerge from the medulla in the sulcus between the pyramid and the olive. Cranial nerve nuclei location is fundamental for localizing brainstem strokes (e.g., Medial Medullary Syndrome). Therefore, the correct answer is c) Medulla, in the floor of the fourth ventricle.
8. A patient exhibits fasciculations (muscle twitches) on the right side of the tongue. This specific sign is most indicative of:
a) Upper Motor Neuron lesion
b) Neuromuscular junction disorder
c) Lower Motor Neuron lesion
d) Cerebellar disease
Explanation: Fasciculations are visible, fine, rapid twitching of muscle bundles. Along with muscle atrophy (wasting) and weakness, fasciculations are a cardinal sign of a Lower Motor Neuron (LMN) lesion. In the context of the tongue, this indicates damage to the Hypoglossal nucleus or the nerve fibers themselves. Upper Motor Neuron (UMN) lesions typically cause spasticity and weakness without significant atrophy or fasciculations. Cerebellar disease causes ataxia, not fasciculations. Therefore, the correct answer is c) Lower Motor Neuron lesion.
9. Anatomically, the Hypoglossal canal is located in which bone of the skull?
a) Temporal bone
b) Sphenoid bone
c) Occipital bone
d) Parietal bone
Explanation: The Hypoglossal canal pierces the base of the skull. It is situated in the condylar part of the Occipital bone. Specifically, it lies superior and anterior to the occipital condyles (which articulate with the atlas). This location places the nerve in close proximity to the foramen magnum and the jugular foramen (formed between the temporal and occipital bones). Fractures of the occipital condyle are a common cause of traumatic hypoglossal nerve palsy. Therefore, the correct answer is c) Occipital bone.
10. In the submandibular region, the Hypoglossal nerve rests on the Hyoglossus muscle. Which structure runs deep to the Hyoglossus muscle, separated from the nerve?
a) Lingual Nerve
b) Lingual Artery
c) Submandibular duct
d) Facial Vein
Explanation: The Hyoglossus muscle acts as a key anatomical landmark in the floor of the mouth. The Hypoglossal nerve, Lingual nerve, and Submandibular duct all pass superficial (lateral) to the Hyoglossus muscle. In contrast, the Lingual Artery passes deep (medial) to the Hyoglossus muscle. This separation is clinically important; to access the lingual artery surgically, one must retract or cut the hyoglossus muscle, whereas the nerve is exposed immediately upon reflecting the mylohyoid. Therefore, the correct answer is b) Lingual Artery.
Chapter: Head and Neck; Topic: Cranial Nerves; Subtopic: Functional Components of the Chorda Tympani
Key Definitions & Concepts
General Visceral Efferent (GVE): Autonomic fibers that provide motor innervation to glands and smooth muscle (e.g., secretomotor fibers to salivary glands).
Special Visceral Afferent (SVA): Sensory fibers carrying special senses related to the gastrointestinal tract, specifically taste (gustatory) sensation.
Superior Salivatory Nucleus: The brainstem nucleus in the pons that acts as the origin for the parasympathetic fibers of the Facial Nerve (Chorda Tympani and Greater Petrosal).
Inferior Salivatory Nucleus: The brainstem nucleus in the medulla that originates parasympathetic fibers for the Glossopharyngeal nerve (CN IX) targeting the Parotid gland.
Submandibular Ganglion: The peripheral parasympathetic ganglion where fibers from the Chorda Tympani synapse to supply the submandibular and sublingual glands.
Otic Ganglion: The ganglion associated with the Mandibular nerve (V3) but functionally related to the Glossopharyngeal nerve for Parotid innervation.
Nervus Intermedius: The smaller root of the Facial nerve containing sensory and parasympathetic fibers, distinct from the larger motor root.
Lingual Nerve: A branch of V3 that carries general sensation (touch/pain) from the tongue; Chorda Tympani fibers join it to reach their targets.
Petrotympanic Fissure: The slit in the temporal bone through which the Chorda Tympani exits the skull to enter the infratemporal fossa.
Tympanic Plexus: Located on the promontory of the middle ear, formed by the Glossopharyngeal nerve, providing innervation to the Parotid via the Lesser Petrosal nerve.
[Image of Chorda tympani nerve course]
Lead Question - 2016
Chorda-tympani does not carry which fibers?
a) Preganglionic parasympathetic fibers for sublingual glands
b) Preganglionic parasympathetic fibers for submandibular gland
c) Preganglionic parasympathetic fibers for parotid gland
d) Taste fibers from anterior two third of tongue
Explanation: The Chorda Tympani is a mixed nerve carrying two specific types of fibers: Special Visceral Afferent (SVA) and General Visceral Efferent (GVE). The SVA fibers carry taste sensation from the anterior two-thirds of the tongue. The GVE fibers are preganglionic parasympathetic secretomotor fibers. These fibers originate in the Superior Salivatory Nucleus and synapse in the Submandibular Ganglion to supply the Submandibular and Sublingual salivary glands. The Parotid gland, however, is supplied by the Glossopharyngeal nerve (CN IX). The pathway involves the Inferior Salivatory Nucleus, Tympanic branch (Jacobson’s nerve), Tympanic plexus, Lesser Petrosal nerve, and the Otic Ganglion. Therefore, the Chorda Tympani does not carry fibers for the parotid. The correct answer is c) Preganglionic parasympathetic fibers for parotid gland.
1. The cell bodies of the special visceral afferent (taste) fibers carried by the chorda tympani are located in which ganglion?
a) Submandibular ganglion
b) Geniculate ganglion
c) Otic ganglion
d) Pterygopalatine ganglion
Explanation: It is crucial to distinguish between motor (synaptic) ganglia and sensory (non-synaptic) ganglia. The taste fibers in the Chorda Tympani are sensory afferents. Like all sensory nerves, their cell bodies must be located in a sensory ganglion outside the CNS. For the Facial nerve, this is the Geniculate Ganglion, located at the genu (bend) of the facial canal. The fibers pass through this ganglion without synapsing and project centrally to the Nucleus of the Tractus Solitarius. The Submandibular, Otic, and Pterygopalatine ganglia are parasympathetic motor ganglia where synapses occur for efferent pathways. Therefore, the correct answer is b) Geniculate ganglion.
2. A patient presents with a loss of taste on the anterior two-thirds of the tongue but retains general sensation (touch and pain) in the same area. The lesion is most likely located in:
a) The Lingual nerve proximal to its junction with Chorda Tympani
b) The Lingual nerve distal to its junction with Chorda Tympani
c) The Chorda Tympani nerve before it joins the Lingual nerve
d) The Mandibular nerve trunk
Explanation: The Lingual nerve (V3) carries general sensation. The Chorda Tympani (CN VII) carries taste. These two nerves join in the infratemporal fossa. If a lesion occurs in the Chorda Tympani nerve before it joins the Lingual nerve (e.g., in the middle ear), only the functions of the Chorda Tympani (taste and secretomotor) are lost, while the Lingual nerve's somatic sensation remains intact. A lesion of the Lingual nerve distal to the junction would eliminate both taste and touch. A lesion proximal to the junction would eliminate only touch, sparing taste (though the taste fibers would have nowhere to go). Therefore, the correct answer is c) The Chorda Tympani nerve before it joins the Lingual nerve.
3. Which brainstem nucleus serves as the central termination point for the taste fibers carried by the chorda tympani?
a) Nucleus Ambiguus
b) Nucleus of the Tractus Solitarius
c) Spinal Nucleus of Trigeminal
d) Mesencephalic Nucleus
Explanation: The central processing of taste involves specific nuclei. Taste fibers from the Facial nerve (Chorda Tympani, Greater Petrosal), Glossopharyngeal nerve, and Vagus nerve all converge on the Nucleus of the Tractus Solitarius (NTS) in the medulla. Specifically, they terminate in the rostral part of this nucleus (sometimes called the Gustatory Nucleus). The Nucleus Ambiguus controls motor output to the pharynx/larynx. The Spinal Nucleus of Trigeminal handles pain and temperature from the face. The Mesencephalic nucleus is involved in proprioception. Therefore, the correct answer is b) Nucleus of the Tractus Solitarius.
4. During a radical mastoidectomy, the surgeon must be careful to identify the facial nerve. If the vertical segment of the facial nerve is sacrificed, which of the following functions will be preserved?
a) Taste on anterior 2/3 of tongue
b) Lacrimation (Tearing)
c) Salivation from submandibular gland
d) Function of Stapedius muscle
Explanation: The facial nerve gives off branches in a descending order within the facial canal. The Greater Petrosal nerve (for lacrimation) branches off at the Geniculate Ganglion (high up). The Nerve to Stapedius branches off next. The Chorda Tympani branches off last, in the vertical (mastoid) segment. If the vertical segment is sacrificed (cut), the Chorda Tympani and Nerve to Stapedius function are usually lost (unless the cut is very distal). However, the Greater Petrosal nerve arises much more proximally. Thus, Lacrimation (controlled by the Greater Petrosal nerve via the Pterygopalatine ganglion) remains intact. Therefore, the correct answer is b) Lacrimation (Tearing).
[Image of Facial nerve branches diagram]
5. The parasympathetic fibers in the chorda tympani are classified as:
a) Postganglionic cholinergic fibers
b) Preganglionic adrenergic fibers
c) Preganglionic cholinergic fibers
d) Postganglionic adrenergic fibers
Explanation: The Chorda Tympani transports parasympathetic fibers from the brainstem to the submandibular ganglion. Fibers leaving the CNS to reach a ganglion are always Preganglionic. In the parasympathetic nervous system, preganglionic neurons use Acetylcholine as their neurotransmitter, making them Cholinergic. These fibers synapse in the submandibular ganglion. The short fibers that leave the ganglion to reach the gland are Postganglionic cholinergic. Since the Chorda Tympani is the conduit *to* the ganglion, it carries preganglionic cholinergic fibers. Adrenergic fibers are typically sympathetic. Therefore, the correct answer is c) Preganglionic cholinergic fibers.
6. A 50-year-old male complains of a "metallic taste" in his mouth and dry eyes. Examination reveals no facial weakness. This constellation of symptoms might suggest a lesion affecting which nerve segment or branch?
a) Chorda Tympani only
b) Facial Nerve at the Stylomastoid foramen
c) Facial Nerve at the Geniculate Ganglion
d) Glossopharyngeal nerve
Explanation: This is a trick question requiring careful analysis of "dry eyes" vs "taste". Dry eyes (xerophthalmia) indicate a loss of lacrimation, involving the Greater Petrosal nerve. Dysgeusia (metallic taste or loss of taste) involves the taste fibers. Both the Greater Petrosal nerve (palate taste) and Chorda Tympani (tongue taste) arise from the Facial nerve. If a patient has dry eyes, the lesion must be proximal to the Greater Petrosal take-off, i.e., at or near the Geniculate Ganglion. A lesion here would affect both lacrimation and taste (if the main trunk is involved) or specifically the Greater Petrosal. However, pure Chorda Tympani involvement does not cause dry eyes. Therefore, the correct answer is c) Facial Nerve at the Geniculate Ganglion.
7. The Chorda Tympani nerve enters the infratemporal fossa by passing through the petrotympanic fissure. This fissure is located between which two bones?
a) Petrous temporal and squamous temporal
b) Petrous temporal and tympanic plate
c) Squamous temporal and sphenoid
d) Mastoid and styloid process
Explanation: The nomenclature of the fissure gives a clue to its boundaries. The Petrotympanic fissure (of Glaser) is a narrow slit. It is located between the Petrous part of the temporal bone (specifically the tegmen tympani extension) and the Tympanic part of the temporal bone (the tympanic plate). Wait—standard texts often refer to it as Squamotympanic in broader terms, but strictly the medial part is Petrotympanic. The fissure leads into the middle ear. The Chorda Tympani courses through a canal (Canal of Huguier) parallel to this fissure. Standard anatomical definition aligns with Petrous and Tympanic interaction. (Note: Some simplifications say Squamous and Tympanic, but Petrous is the deep component). Let's stick to the most accurate descriptive name: Petrotympanic. Therefore, the correct answer is b) Petrous temporal and tympanic plate.
8. A patient with a submandibular duct stone undergoes an intraoral removal. Post-operatively, the patient complains of numbness on the tip of the tongue on that side. The nerve injured carries which type of fibers responsible for this specific symptom?
a) Special Visceral Afferent (Taste)
b) General Somatic Afferent (Touch/Pain)
c) General Visceral Efferent (Secretomotor)
d) Special Visceral Efferent (Branchial Motor)
Explanation: The key here is the symptom: Numbness. Numbness refers to a loss of general sensation (touch, pain, temperature), not taste. While the Lingual nerve carries Chorda Tympani fibers (taste/secretomotor), the Lingual nerve itself provides the General Somatic Afferent (GSA) fibers for the anterior tongue. In the floor of the mouth, the Lingual nerve loops under the submandibular duct, placing it at high risk during duct surgery. Injury here affects both taste and touch, but the "numbness" is specifically due to the loss of General Somatic Afferent fibers. Therefore, the correct answer is b) General Somatic Afferent (Touch/Pain).
9. The submandibular ganglion acts as a relay station for the Chorda Tympani. Which other nerve passes through this ganglion without synapsing?
a) Sympathetic fibers from the facial artery plexus
b) Parasympathetic fibers from the Otic ganglion
c) Sensory fibers from the Hypoglossal nerve
d) Motor fibers to the Mylohyoid
Explanation: Autonomic ganglia in the head often have three "roots": parasympathetic (motor), sympathetic, and sensory. For the submandibular ganglion, the parasympathetic root comes from the Chorda Tympani (which synapses there). The Sympathetic fibers from the facial artery plexus (postganglionic from the superior cervical ganglion) pass through the ganglion without synapsing to reach the glands (providing vasoconstriction). Sensory fibers from the lingual nerve may also pass through. The key distinction is that only the parasympathetic fibers synapse. Therefore, the correct answer is a) Sympathetic fibers from the facial artery plexus.
10. An acoustic neuroma (Vestibular Schwannoma) expands in the internal acoustic meatus. As it compresses the adjacent facial nerve, which function is typically the last to be lost due to the arrangement of fibers?
a) Lacrimation
b) Taste
c) Facial facial muscle movement
d) Stapedius reflex
Explanation: This is a question of "Spatial orientation of fibers" within the Facial nerve trunk. In the internal acoustic meatus, the motor fibers to the facial muscles are distinct from the Nervus Intermedius (which carries the sensory and parasympathetic fibers: Taste, Lacrimation, Salivation). The motor fibers are generally more robust and resistant to pressure than the thinner sensory/autonomic fibers. Furthermore, the facial motor fibers are often pushed against the bone, but clinical progression varies. However, historically and clinically, Facial facial muscle movement is often the last to be fully lost (or preserved longer than the delicate sensory functions like taste and lacrimation) in slow-growing tumors, whereas sensory symptoms (Nervus Intermedius) appear early. Therefore, the correct answer is c) Facial facial muscle movement.
Chapter: Head and Neck; Topic: Cranial Nerves; Subtopic: Facial Nerve (CN VII) and Chorda Tympani
Key Definitions & Concepts
Chorda Tympani: A branch of the Facial nerve (CN VII) originating in the mastoid segment, carrying secretomotor and taste fibers.
Petrotympanic Fissure: Also known as the squamotympanic fissure; the exit point of the chorda tympani from the skull into the infratemporal fossa.
Lingual Nerve: A branch of the Mandibular nerve (V3); the chorda tympani "hitchhikes" on this nerve to reach the floor of the mouth.
Submandibular Ganglion: The parasympathetic ganglion where pre-ganglionic fibers from the chorda tympani synapse before supplying the salivary glands.
Superior Salivatory Nucleus: The brainstem nucleus providing the parasympathetic (secretomotor) origin for the chorda tympani.
Nucleus of Tractus Solitarius: The brainstem nucleus that receives special sensory (taste) fibers from the chorda tympani.
Taste Sensation (SVA): Chorda tympani provides taste to the anterior 2/3 of the tongue (except vallate papillae).
Secretomotor Function (GVE): Chorda tympani provides secretomotor innervation to the Submandibular and Sublingual salivary glands.
Infratemporal Fossa: The anatomical space where the chorda tympani joins the lingual nerve at an acute angle.
Geniculate Ganglion: The sensory ganglion of the facial nerve; the chorda tympani arises distal to this, within the facial canal.
[Image of Chorda tympani nerve course]
Lead Question - 2016
Chorda tympani is a branch of ?
a) Facial nerve
b) Trigeminal nerve
c) Greater auricular nerve
d) External laryngeal nerve
Explanation: The Chorda Tympani is a significant branch of the Facial nerve (CN VII). It arises from the vertical (mastoid) segment of the facial nerve within the facial canal, approximately 6 mm above the stylomastoid foramen. It arches upward across the posterior wall of the middle ear, crosses the tympanic membrane (hence "chorda tympani" or string of the drum), and exits the skull. It carries two types of fibers: Special Visceral Afferent (taste from the anterior 2/3 of the tongue) and General Visceral Efferent (parasympathetic secretomotor fibers to the submandibular and sublingual glands). Therefore, the correct answer is a) Facial nerve.
1. The chorda tympani nerve exits the skull to enter the infratemporal fossa through which anatomical structure?
a) Stylomastoid foramen
b) Petrotympanic fissure
c) Foramen ovale
d) Internal acoustic meatus
Explanation: After traversing the middle ear cavity between the incus and malleus, the chorda tympani nerve continues anteriorly. It exits the tympanic cavity and the skull through the Petrotympanic fissure (also known as the squamotympanic fissure or Glaserian fissure). Once it passes through this fissure, it enters the infratemporal fossa where it joins the lingual nerve. The stylomastoid foramen is the exit for the main trunk of the facial nerve. Foramen ovale transmits the Mandibular nerve. The internal acoustic meatus is the entry point for CN VII and VIII into the temporal bone. Therefore, the correct answer is b) Petrotympanic fissure.
2. A patient complains of dry mouth and loss of taste sensation on the tip of the tongue following a middle ear surgery. Which nerve was likely damaged during the procedure?
a) Glossopharyngeal nerve
b) Chorda tympani nerve
c) Greater petrosal nerve
d) Auriculotemporal nerve
Explanation: The chorda tympani nerve runs directly across the middle ear cavity, passing between the handle of the malleus and the long process of the incus. Because of this exposed course "across the drum," it is highly susceptible to injury during middle ear surgeries like myringoplasty or stapedectomy. Damage leads to loss of taste on the ipsilateral anterior 2/3 of the tongue and reduction in saliva from the submandibular and sublingual glands (dry mouth). The Glossopharyngeal nerve supplies the posterior 1/3. The Greater petrosal supplies the lacrimal gland. Therefore, the correct answer is b) Chorda tympani nerve.
3. In the infratemporal fossa, the chorda tympani nerve joins which other nerve at an acute angle to distribute its fibers?
a) Inferior alveolar nerve
b) Auriculotemporal nerve
c) Lingual nerve
d) Buccal nerve
Explanation: The chorda tympani does not reach its target organs directly. Instead, upon entering the infratemporal fossa, it joins the Lingual nerve (a branch of the Mandibular division of the Trigeminal nerve, V3) from behind at an acute angle. The chorda tympani fibers then "hitchhike" along the lingual nerve to reach the floor of the mouth. The lingual nerve provides general sensation (touch, pain, temperature) to the anterior 2/3 of the tongue, while the chorda tympani provides the special sensation (taste) and secretomotor function. Therefore, the correct answer is c) Lingual nerve.
4. A 40-year-old male presents with Bell's Palsy (facial paralysis). He also reports hyperacusis (sensitivity to loud sounds) and loss of taste on the anterior tongue. The lesion of the facial nerve is located:
a) At the stylomastoid foramen
b) In the parotid gland
c) Proximal to the origin of the chorda tympani and nerve to stapedius
d) Distal to the geniculate ganglion but proximal to chorda tympani
Explanation: Localization of a facial nerve lesion depends on the functions lost. Loss of taste (chorda tympani) places the lesion proximal to the chorda tympani's origin. Hyperacusis indicates paralysis of the stapedius muscle, placing the lesion proximal to the nerve to the stapedius. Since the nerve to the stapedius branches off before the chorda tympani in the facial canal, a lesion causing *both* symptoms must be proximal to the origin of the chorda tympani and nerve to stapedius. A lesion at the stylomastoid foramen would only cause facial paralysis without taste loss or hyperacusis. Therefore, the correct answer is c) Proximal to the origin of the chorda tympani and nerve to stapedius.
5. The pre-ganglionic parasympathetic fibers carried by the chorda tympani nerve arise from which brainstem nucleus?
a) Inferior salivatory nucleus
b) Nucleus ambiguus
c) Superior salivatory nucleus
d) Edinger-Westphal nucleus
Explanation: The secretomotor (parasympathetic) fibers of the chorda tympani stimulate the submandibular and sublingual salivary glands. These are General Visceral Efferent (GVE) fibers. Their cell bodies are located in the Superior salivatory nucleus of the pons. The Inferior salivatory nucleus is associated with the Glossopharyngeal nerve (CN IX) and the parotid gland. The Nucleus ambiguus is motor to the pharynx/larynx. The Edinger-Westphal nucleus is associated with the Oculomotor nerve (CN III). Therefore, the correct answer is c) Superior salivatory nucleus.
6. Which of the following accurately describes the relationship of the chorda tympani within the tympanic cavity?
a) Lateral to the malleus and incus
b) Medial to the incus but lateral to the malleus
c) Medial to the handle of the malleus and lateral to the long process of the incus
d) Medial to both the malleus and incus
Explanation: The course of the chorda tympani through the middle ear is anatomically precise. It enters through the posterior wall and runs forward. As it crosses the tympanic cavity, it passes medial to the handle of the malleus and lateral to the long process of the incus. This position places it directly against the upper part of the tympanic membrane (eardrum). This specific relationship is crucial for otologists to avoid nerve damage during middle ear exploration. Therefore, the correct answer is c) Medial to the handle of the malleus and lateral to the long process of the incus.
7. A patient presents with a submandibular duct stone (sialolithiasis). To relieve pain, the surgeon considers blocking the ganglion supplying the gland. Where do the pre-ganglionic fibers of the chorda tympani synapse?
a) Otic ganglion
b) Submandibular ganglion
c) Pterygopalatine ganglion
d) Geniculate ganglion
Explanation: The chorda tympani carries pre-ganglionic parasympathetic fibers. These fibers hitchhike on the lingual nerve to reach the floor of the mouth. There, they leave the lingual nerve to synapse in the Submandibular ganglion, which is suspended from the lingual nerve. The post-ganglionic fibers then arise from this ganglion to supply the submandibular and sublingual salivary glands. The Otic ganglion is for the parotid (CN IX). The Pterygopalatine is for the lacrimal/nasal glands (CN VII via Greater Petrosal). The Geniculate is a sensory ganglion, not a synapse point. Therefore, the correct answer is b) Submandibular ganglion.
8. Taste sensation from the circumvallate papillae is NOT carried by the chorda tympani. These papillae are supplied by which nerve?
a) Vagus nerve
b) Lingual nerve proper
c) Glossopharyngeal nerve
d) Hypoglossal nerve
Explanation: The chorda tympani supplies taste to the anterior two-thirds of the tongue. However, there is an anatomical exception: the Circumvallate papillae. Although these papillae are located anterior to the sulcus terminalis (the V-shaped line dividing the tongue), they are embryologically derived from the posterior tissue. Therefore, they are supplied by the Glossopharyngeal nerve (CN IX), which supplies general sensation and taste to the posterior one-third of the tongue including the vallate papillae. The Vagus supplies the extreme posterior/epiglottic region. Therefore, the correct answer is c) Glossopharyngeal nerve.
9. The functional components of the chorda tympani nerve include:
a) General Somatic Afferent (GSA) and General Visceral Efferent (GVE)
b) Special Visceral Afferent (SVA) and General Visceral Efferent (GVE)
c) Special Visceral Efferent (SVE) and General Visceral Afferent (GVA)
d) General Somatic Efferent (GSE) only
Explanation: Cranial nerves are classified by column types. The chorda tympani has two distinct functions. First, it carries Taste, which is classified as Special Visceral Afferent (SVA). Second, it carries parasympathetic motor fibers to salivary glands, which is classified as General Visceral Efferent (GVE). It does not carry SVE (branchial motor) fibers; those are in the main trunk of the facial nerve supplying facial muscles. It does not carry GSA (general touch sensation); that is the function of the lingual nerve (V3). Therefore, the correct answer is b) Special Visceral Afferent (SVA) and General Visceral Efferent (GVE).
10. A dentist accidentally damages a nerve while performing an anesthetic block for the lower teeth (Inferior Alveolar Nerve block). The patient subsequently reports a loss of taste. Which nerve was inadvertently affected?
a) Buccal nerve
b) Mental nerve
c) Lingual nerve
d) Mylohyoid nerve
Explanation: During an Inferior Alveolar Nerve (IAN) block, the needle is placed in the pterygomandibular space. The Lingual nerve lies slightly anterior and medial to the inferior alveolar nerve in this space. Because the chorda tympani travels within the sheath of the lingual nerve at this level, accidental damage or anesthesia of the lingual nerve will result in numbness of the anterior tongue (lingual nerve function) AND loss of taste (chorda tympani function). The buccal nerve is lateral. The mental nerve is far distal at the chin. Therefore, the correct answer is c) Lingual nerve.
Chapter: Neck & Thorax Anatomy; Topic: Recurrent Laryngeal Nerve; Subtopic: Course of Left Recurrent Laryngeal Nerve
Keyword Definitions:
Recurrent Laryngeal Nerve (RLN): Branch of vagus nerve supplying intrinsic laryngeal muscles except cricothyroid.
Left RLN: Loops under the arch of aorta and ascends in the tracheoesophageal groove.
Tracheoesophageal Groove: Anatomical space between trachea and esophagus where RLN runs.
Vagus Nerve: Cranial nerve X giving rise to superior and recurrent laryngeal branches.
Inferior Laryngeal Nerve: Terminal continuation of recurrent laryngeal nerve.
1) Lead Question – 2016
Left recurrent laryngeal passes between?
a) Trachea & larynx
b) Trachea & esophagus
c) Esophagus and bronchi
d) Esophagus and aorta
Answer: b) Trachea & esophagus
Explanation: The left recurrent laryngeal nerve hooks under the arch of aorta near the ligamentum arteriosum and then ascends in the tracheoesophageal groove, which lies between the trachea and esophagus. This pathway makes it vulnerable in surgeries such as esophagectomy, thyroidectomy, and mediastinal procedures. The nerve supplies all intrinsic laryngeal muscles except cricothyroid and provides sensory innervation to the infraglottic region. Its long intrathoracic course explains why left RLN palsy occurs more commonly in mediastinal tumors.
2) Which nerve supplies intrinsic laryngeal muscles except cricothyroid?
a) Superior laryngeal nerve
b) Recurrent laryngeal nerve
c) Glossopharyngeal nerve
d) Hypoglossal nerve
Answer: b) Recurrent laryngeal nerve
Explanation: RLN provides motor supply to all intrinsic laryngeal muscles except cricothyroid, which is supplied by external laryngeal nerve.
3) The left recurrent laryngeal nerve hooks around:
a) Right subclavian artery
b) Aortic arch
c) Brachiocephalic trunk
d) Pulmonary artery
Answer: b) Aortic arch
Explanation: It loops under the aortic arch near the ligamentum arteriosum before ascending.
4) Compression of the left RLN may occur in:
a) Thyroid cyst
b) Left atrial enlargement
c) Submandibular mass
d) Parotid swelling
Answer: b) Left atrial enlargement
Explanation: Left atrial enlargement (Ortner’s syndrome) compresses the left RLN causing hoarseness.
5) Sensory supply below vocal cords is via:
a) Internal laryngeal nerve
b) Inferior laryngeal nerve
c) Glossopharyngeal nerve
d) Facial nerve
Answer: b) Inferior laryngeal nerve
Explanation: Inferior laryngeal nerve is the terminal part of RLN and supplies infraglottic mucosa.
6) A patient with hoarseness and mediastinal mass most likely has involvement of:
a) Right RLN
b) Left RLN
c) Hypoglossal nerve
d) Spinal accessory nerve
Answer: b) Left RLN
Explanation: The longer thoracic course makes left RLN more prone to mediastinal compression.
7) Right RLN loops around:
a) Right subclavian artery
b) Right common carotid artery
c) Aortic arch
d) Brachiocephalic vein
Answer: a) Right subclavian artery
Explanation: The right vagus gives off RLN which loops around the right subclavian artery.
8) RLN injury during thyroid surgery is due to its relation with:
a) Superior thyroid artery
b) Inferior thyroid artery
c) Middle thyroid vein
d) Cricothyroid membrane
Answer: b) Inferior thyroid artery
Explanation: RLN crosses close to inferior thyroid artery branches, risking damage.
9) RLN supplies all except:
a) Posterior cricoarytenoid
b) Lateral cricoarytenoid
c) Cricothyroid
d) Thyroarytenoid
Answer: c) Cricothyroid
Explanation: Cricothyroid is supplied by the external laryngeal nerve.
10) Unilateral RLN palsy leads to:
a) Aphonia
b) Hoarseness
c) Breathy voice loss
d) Complete airway obstruction
Answer: b) Hoarseness
Explanation: Paralysis of one vocal cord results in hoarseness but airway remains adequate.
11) RLN ascends toward the larynx in:
a) Retropharyngeal space
b) Tracheoesophageal groove
c) Parapharyngeal space
d) Carotid sheath
Answer: b) Tracheoesophageal groove
Explanation: This anatomical pathway protects but also clinically exposes RLN during esophageal or thyroid surgery.
Chapter: Neck Anatomy; Topic: Nerves of the Larynx; Subtopic: Recurrent Laryngeal Nerve Course
Keyword Definitions:
Recurrent Laryngeal Nerve (RLN): Branch of vagus nerve that loops and ascends to supply intrinsic laryngeal muscles.
Right RLN: Loops around right subclavian artery; shorter and more oblique path.
Left RLN: Loops around aortic arch; longer intrathoracic course.
Vagus Nerve: CN X giving motor and sensory supply to pharynx, larynx, and thoracic structures.
Inferior Laryngeal Nerve: Terminal part of RLN entering larynx.
1) Lead Question – 2016
Right Recurrent laryngeal nerve loops around?
a) Right subclavian artery
b) Right axillary artery
c) Right External carotid artery
d) Right Superior thyroid artery
Answer: a) Right subclavian artery
Explanation: The right recurrent laryngeal nerve branches from the vagus in the root of the neck and loops around the right subclavian artery before ascending in the tracheoesophageal groove. This looping pattern differs from the left RLN, which loops under the aortic arch. After looping, the nerve travels superiorly to supply all intrinsic laryngeal muscles except cricothyroid. Damage may occur during thyroid or parathyroid surgery.
2) The left recurrent laryngeal nerve loops under:
a) Aortic arch
b) Pulmonary artery
c) Subclavian vein
d) Brachiocephalic vein
Answer: a) Aortic arch
Explanation: The left RLN loops beneath the aortic arch near the ligamentum arteriosum before ascending toward the larynx.
3) Injury to recurrent laryngeal nerve causes:
a) Loss of pitch control
b) Hoarseness
c) Inability to swallow solids
d) Loss of gag reflex
Answer: b) Hoarseness
Explanation: RLN injury affects intrinsic laryngeal muscles, leading to vocal cord paralysis and hoarseness.
4) A patient with mediastinal tumor compressing the left RLN will present with:
a) Nasal regurgitation
b) Hoarse voice
c) Tongue deviation
d) Facial paralysis
Answer: b) Hoarse voice
Explanation: The longer intrathoracic course of the left RLN makes it vulnerable to mediastinal masses.
5) RLN supplies all intrinsic laryngeal muscles except:
a) Cricothyroid
b) Posterior cricoarytenoid
c) Lateral cricoarytenoid
d) Thyroarytenoid
Answer: a) Cricothyroid
Explanation: Cricothyroid is supplied by external laryngeal nerve (branch of superior laryngeal nerve).
6) The RLN ascends in which anatomical groove?
a) Carotid sheath
b) Tracheoesophageal groove
c) Retropharyngeal space
d) Submandibular space
Answer: b) Tracheoesophageal groove
Explanation: This groove provides a protected pathway for RLN ascent toward the larynx.
7) Most common cause of bilateral RLN palsy:
a) Stroke
b) Thyroid surgery
c) Mastoidectomy
d) Lung cancer
Answer: b) Thyroid surgery
Explanation: Close proximity to the inferior thyroid artery makes the RLN vulnerable during thyroidectomy.
8) Recurrent laryngeal nerve provides sensory supply to:
a) Supraglottis
b) Glottis
c) Infraglottis
d) Epiglottis
Answer: c) Infraglottis
Explanation: Sensory innervation below vocal cords is via the inferior laryngeal nerve (terminal RLN branch).
9) A patient with voice fatigue and aspiration likely has injury to:
a) Glossopharyngeal
b) External laryngeal
c) Recurrent laryngeal nerve
d) Hypoglossal
Answer: c) Recurrent laryngeal nerve
Explanation: RLN injury impairs glottic closure, leading to aspiration and weak voice.
10) RLN injury during PDA ligation affects mostly:
a) Right RLN
b) Left RLN
c) Both equally
d) None
Answer: b) Left RLN
Explanation: The left RLN loops near ligamentum arteriosum and is at risk during PDA repair.
11) The nerve entering larynx below the inferior constrictor is:
a) Internal laryngeal nerve
b) External laryngeal nerve
c) Inferior laryngeal nerve
d) Glossopharyngeal nerve
Answer: c) Inferior laryngeal nerve
Explanation: This is the continuation of RLN after its ascent, supplying motor and sensory functions.
Chapter: Neuroanatomy; Topic: Brainstem Functional Columns; Subtopic: General Visceral Afferent (GVA) Nuclei
Keyword Definitions:
General Visceral Afferent (GVA): Sensory fibers that carry visceral sensations from thoracic, abdominal, and pelvic organs.
Dorsal Nucleus of Vagus: Major GVA nucleus receiving input from thoracic and abdominal viscera.
Nucleus Ambiguus: Motor nucleus (SVE), NOT GVA.
Trigeminal Nucleus: Receives somatic sensation (GSA), not visceral.
Facial Nerve Nucleus: Motor/Special sensory, not GVA.
1) Lead Question – 2016
Which of the following nuclei belong to the general visceral afferent column?
a) Facial nerve nucleus
b) Trigeminal nucleus
c) Dorsal nucleus of vagus
d) Nucleus ambiguus
Answer: c) Dorsal nucleus of vagus
Explanation: The dorsal nucleus of the vagus is the primary GVA nucleus in the brainstem. It receives visceral afferents from thoracic and abdominal organs, including heart, lungs, and gut. The facial nerve nucleus and nucleus ambiguus are motor nuclei, while the trigeminal nucleus receives somatic, not visceral, sensation. Thus, only the dorsal nucleus of vagus belongs to the GVA column.
2) GVA fibers from carotid sinus synapse in:
a) Nucleus solitarius
b) Chief sensory nucleus of V
c) Nucleus ambiguus
d) Motor nucleus of VII
Answer: a) Nucleus solitarius
Explanation: The nucleus solitarius receives taste and visceral afferent inputs from cranial nerves IX and X, including the carotid sinus baroreceptors.
3) Visceral afferent fibers from abdominal organs reach CNS mainly via:
a) Glossopharyngeal
b) Facial nerve
c) Vagus nerve
d) Hypoglossal
Answer: c) Vagus nerve
Explanation: The vagus nerve carries the majority of visceral sensations from thoracic and abdominal organs to the dorsal vagal nucleus and nucleus solitarius.
4) Lesion of dorsal vagal nucleus may cause:
a) Loss of gag reflex
b) Altered visceral reflexes
c) Facial paralysis
d) Hypoglossal palsy
Answer: b) Altered visceral reflexes
Explanation: Because this nucleus regulates visceral sensations and reflexes, its injury disrupts autonomic responses.
5) Taste fibers (special visceral afferent) from anterior tongue terminate in:
a) Nucleus solitarius
b) Dorsal vagal nucleus
c) Trigeminal motor nucleus
d) Facial motor nucleus
Answer: a) Nucleus solitarius
Explanation: The rostral part of the nucleus solitarius (gustatory nucleus) receives taste sensations.
6) Pain from heart is carried mainly by:
a) Somatic afferents
b) Sympathetic visceral afferents
c) Parasympathetic visceral afferents only
d) Spinal accessory nerve
Answer: b) Sympathetic visceral afferents
Explanation: Cardiac pain is transmitted through sympathetic GVA fibers to upper thoracic spinal segments.
7) Parasympathetic visceral inputs to nucleus solitarius come via:
a) III, IV
b) VII, IX, X
c) V, VIII
d) XI, XII
Answer: b) VII, IX, X
Explanation: These cranial nerves carry visceral and taste sensations to the nucleus solitarius.
8) A patient with bilateral dorsovagal nucleus lesion may show:
a) Loss of voluntary limb movement
b) Severe autonomic dysfunction
c) Complete facial anesthesia
d) Loss of smell
Answer: b) Severe autonomic dysfunction
Explanation: The dorsal nucleus is key to regulating autonomic visceral function; damage affects heart rate and gut motility.
9) Which nucleus is NOT part of visceral afferent pathways?
a) Nucleus solitarius
b) Dorsal nucleus of vagus
c) Nucleus ambiguus
d) Carotid sinus nerve nucleus
Answer: c) Nucleus ambiguus
Explanation: The nucleus ambiguus is a motor nucleus (SVE) supplying pharyngeal muscles.
10) Which nerve carries both taste and visceral afferents?
a) Hypoglossal
b) Vagus
c) Spinal accessory
d) Trochlear
Answer: b) Vagus
Explanation: Vagus carries taste (epiglottis) and GVA fibers from thoracoabdominal organs.
11) Visceral reflexes like vomiting are integrated in:
a) Facial motor nucleus
b) Solitary nucleus
c) Abducens nucleus
d) Spinal trigeminal nucleus
Answer: b) Solitary nucleus
Explanation: The solitary nucleus integrates visceral sensory input essential for autonomic reflexes.
Chapter: Head & Neck Anatomy; Topic: Pterygopalatine Fossa & Nerve Pathways; Subtopic: Vidian Nerve (Nerve of Pterygoid Canal)
Keyword Definitions:
Vidian Nerve: Formed by union of greater petrosal (parasympathetic) and deep petrosal (sympathetic) nerves.
Nerve of Pterygoid Canal: Another name for Vidian nerve; passes through pterygoid canal.
Greater Petrosal Nerve: Parasympathetic branch from facial nerve; NOT same as Vidian nerve.
Deep Petrosal Nerve: Sympathetic fibers contributing to Vidian nerve.
Pterygopalatine Ganglion: Parasympathetic relay ganglion receiving Vidian nerve.
1) Lead Question – 2016
Vidian nerve is also known as?
a) Nerve of Pterygoid canal
b) Greater Petrosal nerve
c) Lesser Petrosal nerve
d) Greater Auricular nerve
Answer: a) Nerve of Pterygoid canal
Explanation: The Vidian nerve is classically known as the “nerve of the pterygoid canal.” It is formed by the union of the greater petrosal nerve (parasympathetic fibers) and the deep petrosal nerve (sympathetic fibers). It passes through the pterygoid canal into the pterygopalatine fossa and carries autonomic fibers to the pterygopalatine ganglion. Greater or lesser petrosal nerves are components but not synonyms. The greater auricular nerve is unrelated.
2) Vidian nerve carries which type of fibers?
a) Pure parasympathetic
b) Pure sympathetic
c) Mixed autonomic fibers
d) Pure sensory
Answer: c) Mixed autonomic fibers
Explanation: It includes both parasympathetic (greater petrosal) and sympathetic (deep petrosal) fibers forming a mixed nerve.
3) The greater petrosal nerve is a branch of?
a) Glossopharyngeal nerve
b) Facial nerve
c) Trigeminal nerve
d) Vagus nerve
Answer: b) Facial nerve
Explanation: The greater petrosal arises from the geniculate ganglion of facial nerve.
4) Vidian nerve enters which fossa?
a) Infratemporal fossa
b) Pterygopalatine fossa
c) Parotid fossa
d) Retropharyngeal space
Answer: b) Pterygopalatine fossa
Explanation: It ends in the pterygopalatine fossa to reach its ganglion.
5) Deep petrosal nerve fibers originate from:
a) Otic ganglion
b) Stellate ganglion
c) Internal carotid plexus
d) Ciliary ganglion
Answer: c) Internal carotid plexus
Explanation: Sympathetic deep petrosal fibers arise from carotid plexus and merge to form Vidian nerve.
6) Parasympathetic fibers of Vidian nerve synapse in:
a) Otic ganglion
b) Pterygopalatine ganglion
c) Submandibular ganglion
d) Ciliary ganglion
Answer: b) Pterygopalatine ganglion
Explanation: The Vidian nerve brings preganglionic parasympathetics to the pterygopalatine ganglion.
7) A lesion of Vidian nerve causes decreased secretion in:
a) Parotid gland
b) Lacrimal gland
c) Submandibular gland
d) Thyroid gland
Answer: b) Lacrimal gland
Explanation: Parasympathetic fibers via Vidian → pterygopalatine ganglion → lacrimal gland.
8) Vidian canal lies in which bone?
a) Maxilla
b) Zygomatic
c) Sphenoid
d) Mandible
Answer: c) Sphenoid
Explanation: The pterygoid canal (Vidian canal) runs through the sphenoid bone.
9) Vidian nerve damage may result in dryness of:
a) Nasal mucosa
b) Pharynx
c) Tongue
d) External ear
Answer: a) Nasal mucosa
Explanation: Loss of parasympathetic supply through the pterygopalatine ganglion reduces nasal gland secretion.
10) Greater petrosal nerve joins deep petrosal at:
a) Foramen rotundum
b) Foramen spinosum
c) Lacerum region
d) Jugular foramen
Answer: c) Lacerum region
Explanation: The union occurs near the foramen lacerum before entering the pterygoid canal.
11) Vidian nerve does NOT supply:
a) Nasal glands
b) Palatine glands
c) Lacrimal gland (via zygomatic nerve)
d) Parotid gland
Answer: d) Parotid gland
Explanation: Parotid is supplied via glossopharyngeal → lesser petrosal → otic ganglion, unrelated to Vidian nerve.
Chapter: Head & Neck Anatomy; Topic: Cutaneous Nerve Supply of Face; Subtopic: Sensory Supply of External Nose
Keyword Definitions:
Ophthalmic division (V1): First division of trigeminal nerve supplying forehead, upper eyelid, dorsum and tip of nose.
Maxillary division (V2): Second division supplying midface including lateral nose and upper lip.
External nasal nerve: Terminal branch of anterior ethmoidal nerve (V1), supplying tip and ala of nose.
Trigeminal nerve: Main sensory nerve of face with three divisions (V1, V2, V3).
Greater auricular nerve: C2–C3 nerve supplying angle of mandible and parotid skin—not nose.
1) Lead Question – 2016
Nerve supply to the tip of the nose is from?
a) The ophthalmic division of the trigeminal nerve
b) Greater auricular nerve
c) The maxillary division of the trigeminal nerve
d) Mandibular nerve
Answer: a) The ophthalmic division of the trigeminal nerve
Explanation: The tip of the nose receives sensory innervation from the external nasal nerve, a branch of the anterior ethmoidal nerve which arises from the ophthalmic division (V1) of the trigeminal nerve. This is a key anatomical distinction because the lateral nose is supplied by V2, but the tip and dorsum are V1 territory. Clinically, this aids in diagnosing sensory deficits after nasal trauma, maxillofacial injuries, or herpetic outbreaks, where lesions limited to the tip of the nose strongly suggest involvement of V1-based nerves.
2) Sensation over the ala of the nose is supplied mainly by:
a) Infraorbital nerve
b) External nasal nerve
c) Greater auricular nerve
d) Mental nerve
Answer: b) External nasal nerve
Explanation: The ala and tip are supplied by the external nasal nerve, a V1 branch. This differentiates it from the lateral nasal wall, which receives infraorbital (V2) innervation. Clinically, trigeminal neuralgia involving V1 may cause pain over the nasal tip region. Understanding the distinction is key in nasal surgeries and local anesthesia blocks.
3) Herpes zoster eruption at the nasal tip (Hutchinson’s sign) indicates involvement of:
a) Maxillary division (V2)
b) Mandibular division (V3)
c) Ophthalmic division (V1)
d) C2–C3 nerves
Answer: c) Ophthalmic division (V1)
Explanation: Hutchinson’s sign refers to vesicles on the nasal tip, indicating involvement of the nasociliary nerve, a branch of V1. This strongly correlates with ocular involvement in herpes zoster ophthalmicus. Recognizing this sign early allows prompt antiviral therapy, preventing corneal complications.
4) Sensory loss over lateral aspect of the nose occurs due to damage of:
a) Infraorbital nerve
b) External nasal nerve
c) Buccal nerve
d) Mental nerve
Answer: a) Infraorbital nerve
Explanation: The lateral nose is supplied by the infraorbital nerve (V2). Trauma to the infraorbital foramen, often seen in maxillary fractures, can impair sensation here. This contrasts with the tip of the nose, which is V1 territory. Precise mapping of sensory territories helps localize nerve injuries.
5) During rhinoplasty, numbness at the nasal tip is expected due to injury of:
a) External nasal nerve
b) Maxillary nerve
c) Mandibular nerve
d) Greater auricular nerve
Answer: a) External nasal nerve
Explanation: The external nasal nerve runs superficially along the nasal dorsum and is frequently stretched during rhinoplasty. Temporary hypoesthesia in this region is common. Knowledge of this branch helps avoid long-term sensory complications.
6) Which nerve enters the nasal cavity through the anterior ethmoidal foramen?
a) Anterior ethmoidal nerve
b) Infraorbital nerve
c) Nasopalatine nerve
d) Buccal nerve
Answer: a) Anterior ethmoidal nerve
Explanation: The anterior ethmoidal nerve (V1 branch) enters the nasal cavity and gives rise to the external nasal nerve, supplying the nasal tip. Its course is important in sinus surgeries because inadvertent damage may cause nasal sensation loss.
7) A patient has decreased sensation over the nasal bridge. Which nerve is likely affected?
a) Supratrochlear nerve
b) Mental nerve
c) Infraorbital nerve
d) Mandibular nerve
Answer: a) Supratrochlear nerve
Explanation: The supratrochlear nerve (V1) supplies the medial upper eyelid and nasal bridge. Blunt trauma or brow surgeries may injure this nerve. Its involvement helps distinguish between V1 and V2 sensory deficits in facial trauma.
8) Sensory supply to nasal septum anteriorly is from:
a) Anterior ethmoidal nerve
b) Greater palatine nerve
c) Mandibular nerve
d) Buccal nerve
Answer: a) Anterior ethmoidal nerve
Explanation: The anterior nasal septum is supplied by the anterior ethmoidal nerve (V1). This is relevant clinically in Little’s area, where epistaxis commonly occurs, and proper anesthesia requires blocking this branch.
9) Which nerve does NOT supply the external nose?
a) V1
b) V2
c) Greater auricular nerve
d) Both V1 and V2
Answer: c) Greater auricular nerve
Explanation: V1 supplies dorsum and tip; V2 supplies lateral nose. The greater auricular nerve (C2–C3) supplies the angle of mandible and parotid skin, not the nose. This distinction helps in trauma mapping and nerve block planning.
10) Which foramen transmits the major nerve supplying the nasal tip?
a) Foramen rotundum
b) Optic canal
c) Anterior ethmoidal foramen
d) Infraorbital foramen
Answer: c) Anterior ethmoidal foramen
Explanation: The anterior ethmoidal nerve exits via the anterior ethmoidal foramen and eventually becomes the external nasal nerve that supplies the nasal tip. Thus, fractures involving the ethmoid region can compromise nasal tip sensation.
11) Pain at the nasal tip radiating to the eye is due to involvement of which nerve?
a) Nasociliary nerve
b) Greater auricular nerve
c) Maxillary nerve
d) Facial nerve
Answer: a) Nasociliary nerve
Explanation: The nasociliary nerve (V1) gives rise to the anterior ethmoidal nerve, which supplies the nasal tip. Disorders such as sinusitis or zoster affecting this nerve produce pain radiating to the eye due to shared sensory pathways. Understanding this helps differentiate ophthalmic versus maxillary nerve involvement.
Chapter: Head & Neck Anatomy; Topic: Cutaneous Nerve Supply of Face; Subtopic: Sensory Supply to Angle of Mandible
Keyword Definitions:
Greater auricular nerve: Cutaneous branch of cervical plexus (C2–C3) supplying skin over angle of mandible and parotid region.
Cervical plexus: Formed by anterior rami of C1–C4; provides sensory nerves to neck and lower face.
Mandibular nerve: Branch of trigeminal nerve (V3) supplying lower face, but not the angle of mandible.
Maxillary nerve: V2 division of trigeminal nerve supplying midface region.
Angle of mandible: Region just behind the masseter, supplied by cervical nerves, not trigeminal.
1) Lead Question – 2016
Nerve supply to the angle of the mandible is by ?
a) Posterior primary rami of C2, C3
b) Greater auricular nerve
c) Maxillary nerve
d) Mandibular nerve
Answer: b) Greater auricular nerve
Explanation: The angle of the mandible is an important landmark because, although most of the face is supplied by branches of the trigeminal nerve (V1, V2, V3), this specific region is supplied by the greater auricular nerve—a branch of the cervical plexus (C2–C3). This nerve emerges from the posterior border of the sternocleidomastoid and ascends toward the parotid gland, providing sensation to the angle of the mandible, parotid fascia, and lower auricle. Because of this, anesthesia or injury to trigeminal branches cannot block sensation here, and cervical plexus blocks may affect this zone more reliably.
2) The skin over the parotid gland receives its sensory supply from:
a) Auriculotemporal nerve
b) Buccal branch of facial nerve
c) Greater auricular nerve
d) Infraorbital nerve
Answer: c) Greater auricular nerve
Explanation: Although the auriculotemporal nerve provides innervation to the parotid gland itself (secretomotor fibers), the skin over the gland is supplied by the greater auricular nerve, a branch of C2–C3. This nerve travels vertically over the sternocleidomastoid muscle and supplies both the parotid sheath and skin overlying it. Clinically, parotid surgeries require safeguarding this nerve to avoid sensory loss over the region, and pain from parotid inflammation often radiates along its distribution.
3) A patient with cervical plexus block complains of numbness over the angle of mandible. Which nerve was anesthetized?
a) Facial nerve
b) Greater auricular nerve
c) Lingual nerve
d) Buccal nerve
Answer: b) Greater auricular nerve
Explanation: The cervical plexus block targets sensory branches of C2–C4, including the greater auricular nerve. Since the angle of the mandible is supplied by this nerve, numbness in this area indicates successful blockade. This finding differentiates it from facial nerve or trigeminal blocks, which would not anesthetize this region. Clinicians confirm block adequacy by checking sensation around the ear lobe and angle of mandible.
4) Sensation to the lower lip is carried by which nerve?
a) Mental nerve
b) Greater auricular nerve
c) Glossopharyngeal nerve
d) Facial nerve
Answer: a) Mental nerve
Explanation: The mental nerve, a branch of the inferior alveolar nerve (V3), supplies the skin of the chin and lower lip. This differs from the angle of the mandible, which is supplied by cervical nerves. Clinically, mental nerve blocks are useful for dental procedures and chin laceration repair. The mental foramen is the landmark for local anesthesia infiltration.
5) Which nerve supplies sensation to the external acoustic meatus and posterior auricle?
a) Auriculotemporal nerve
b) Greater auricular nerve
c) C3 posterior rami
d) Vestibulocochlear nerve
Answer: a) Auriculotemporal nerve
Explanation: The auriculotemporal nerve (V3) supplies the anterior part of auricle and external acoustic meatus. It carries sensory fibers from the trigeminal nerve, unlike the greater auricular nerve (C2–C3), which supplies the posterior auricle and angle of the mandible. This distribution helps differentiate referred otalgia causes, as dental and TMJ lesions may radiate pain through this nerve.
6) Which muscle lies closest to the emergence point of greater auricular nerve along the posterior border of SCM?
a) Trapezius
b) Digastric
c) Sternocleidomastoid
d) Platysma
Answer: c) Sternocleidomastoid
Explanation: The greater auricular nerve emerges at Erb’s point along the posterior border of the sternocleidomastoid muscle. It then ascends superficially across the SCM toward the parotid region. This predictable course is clinically important during cervical plexus blocks and superficial neck dissections to avoid nerve injury.
7) A patient with trauma to upper cervical roots (C2–C3) will have sensory loss over all except:
a) Angle of mandible
b) Parotid gland skin
c) Lower auricle
d) Upper lip
Answer: d) Upper lip
Explanation: The upper lip is supplied by the infraorbital nerve (V2). C2–C3 injuries affect branches of the cervical plexus, especially the greater auricular nerve, leading to sensory loss over the angle of mandible, parotid region, and lower auricle. Understanding this helps differentiate trigeminal versus cervical nerve lesions.
8) Facial nerve paralysis causes muscle weakness but sensation over the angle of mandible remains intact because it is supplied by:
a) Mandibular nerve
b) Maxillary nerve
c) Greater auricular nerve
d) Buccal branch of facial nerve
Answer: c) Greater auricular nerve
Explanation: The facial nerve is purely motor to facial muscles and does not provide cutaneous sensation (except taste). Thus, even complete facial nerve palsy spares the sensation at the angle of the mandible, as this region is supplied by the greater auricular nerve (C2–C3). This helps clinicians localize nerve injuries.
9) Pain radiating to the angle of the mandible during parotitis follows which nerve pathway?
a) Greater auricular nerve
b) Buccal nerve
c) Inferior alveolar nerve
d) Nasociliary nerve
Answer: a) Greater auricular nerve
Explanation: Parotid swelling stretches the parotid fascia, which is supplied by the greater auricular nerve. This results in referred pain toward the angle of mandible and lower auricle. Knowledge of this pathway aids diagnosis of parotid gland infections and tumors and distinguishes them from trigeminal nerve–related pain.
10) During a superficial parotidectomy, which nerve is most at risk of sensory injury?
a) Auriculotemporal nerve
b) Greater auricular nerve
c) Facial nerve
d) Glossopharyngeal nerve
Answer: b) Greater auricular nerve
Explanation: The greater auricular nerve runs superficially across the sternocleidomastoid and enters the parotid sheath, supplying the overlying skin. During parotidectomy, sacrificing this nerve may lead to numbness over the angle of mandible and lower auricle. Surgeons aim to preserve its posterior branch when possible to reduce postoperative sensory deficits.
11) Which branch of the trigeminal nerve does NOT supply any skin over the mandible?
a) V1
b) V2
c) V3
d) None of the above
Answer: a) V1
Explanation: The ophthalmic division (V1) supplies the forehead, upper eyelid, and nose—not the mandible. Sensory supply to the mandible is mainly via V3 except for the angle of mandible, which is supplied by the greater auricular nerve (C2–C3). This distinction is crucial for facial anesthesia and trauma mapping.
Chapter: Pelvis & Perineum; Topic: Pudendal Canal; Subtopic: Alcock’s Canal (Pudendal Canal)
Keyword Definitions:
Alcock’s Canal: A fascial tunnel on the lateral wall of the ischioanal fossa containing pudendal neurovascular bundle.
Pudendal Nerve: Main nerve supplying perineum.
Internal Pudendal Artery: Major arterial supply of perineum.
Ischioanal Fossa: Fat-filled space on each side of anal canal.
Inferior Rectal Nerve: Branch of pudendal nerve supplying external anal sphincter.
1) Lead Question – 2016
Content of Alcock's canal is ?
a) Internal pudendal artery
b) Internal iliac artery
c) Inferior rectal vein
d) Inferior mesenteric vein
Answer: a) Internal pudendal artery
Explanation: Alcock’s canal (pudendal canal) is a fascial canal formed by the obturator internus fascia. It contains the pudendal nerve, internal pudendal artery, and internal pudendal vein. These structures travel through the canal before giving branches to the perineum and external genitalia. It does not contain the internal iliac artery or inferior mesenteric vein. Inferior rectal vessels and nerves arise medially and usually pierce the canal walls but are not contained inside the canal. Therefore, the principal vessel within Alcock’s canal is the internal pudendal artery.
2) Pudendal nerve is a branch of?
a) L3–L4
b) S2–S4
c) L5–S1
d) S1–S2
Answer: b) S2–S4
Explanation: Pudendal nerve originates from S2–S4, supplying perineum and external genitalia.
3) Structure NOT passing through Alcock’s canal?
a) Internal pudendal vein
b) Pudendal nerve
c) Obturator nerve
d) Internal pudendal artery
Answer: c) Obturator nerve
Explanation: Obturator nerve lies in pelvis, not in pudendal canal.
4) Inferior rectal nerve supplies?
a) Internal anal sphincter
b) External anal sphincter
c) Sigmoid colon
d) Uterus
Answer: b) External anal sphincter
Explanation: Inferior rectal nerve (branch of pudendal) innervates external anal sphincter.
5) Failure of pudendal nerve block leads to loss of anaesthesia where?
a) Posterior thigh
b) Perineum
c) Chest wall
d) Foot
Answer: b) Perineum
Explanation: Pudendal nerve is the major sensory nerve of perineum.
6) Pudendal nerve exits pelvis via?
a) Lesser sciatic foramen
b) Greater sciatic foramen
c) Obturator canal
d) Femoral canal
Answer: b) Greater sciatic foramen
Explanation: It exits through greater sciatic foramen and re-enters via lesser sciatic.
7) Internal pudendal artery is a branch of?
a) External iliac artery
b) Internal iliac artery
c) Femoral artery
d) Aorta
Answer: b) Internal iliac artery
Explanation: Arises from anterior division of internal iliac artery.
8) Damage to pudendal canal during surgery can cause?
a) Foot drop
b) Fecal incontinence
c) Horner’s syndrome
d) Wrist drop
Answer: b) Fecal incontinence
Explanation: Pudendal injury affects the external anal sphincter.
9) Which muscle forms the lateral wall of Alcock’s canal?
a) Obturator internus
b) Piriformis
c) Levator ani
d) Coccygeus
Answer: a) Obturator internus
Explanation: Alcock’s canal lies within obturator internus fascia.
10) Pudendal nerve does NOT supply?
a) External urethral sphincter
b) External anal sphincter
c) Levator ani
d) Penis skin
Answer: c) Levator ani
Explanation: Levator ani is mainly supplied by S3–S4 pelvic nerves, not pudendal nerve.
11) Major blood supply to perineum is via?
a) Femoral artery
b) Internal pudendal artery
c) Inferior epigastric artery
d) Superior rectal artery
Answer: b) Internal pudendal artery
Explanation: Internal pudendal artery supplies most perineal structures.
Chapter: Thorax; Topic: Diaphragm; Subtopic: Nerve Supply, Openings & Clinical Anatomy
Keyword Definitions:
Phrenic nerve: Main motor nerve of the diaphragm arising from C3–C5 spinal roots.
Cervical roots (C3–C5): Roots contributing to phrenic nerve; “C3,4,5 keep the diaphragm alive.”
Crura of diaphragm: Musculotendinous pillars attaching diaphragm to lumbar vertebrae.
Hiatuses: Three major openings—T8 IVC, T10 esophagus, T12 aorta.
Central tendon: Strong aponeurotic part of diaphragm receiving phrenic nerve sensory fibres.
1) Lead Question – 2016
Diaphragm is supplied by ?
a) Phrenic nerve
b) C2, C3, C4 roots
c) Thoracodorsal nerve
d) Long thoracic nerve
Answer: a) Phrenic nerve
Explanation: The diaphragm receives its entire motor supply from the **phrenic nerve**, arising from cervical roots C3, C4, and C5. Although the roots are C3–C5, the nerve itself is the functional motor supply, making option (a) correct. The phrenic nerve also carries central sensory fibres supplying the central tendon, mediastinal pleura, and diaphragmatic pleura. The thoracodorsal and long thoracic nerves supply muscles of the upper limb and thorax, not the diaphragm. Damage to the phrenic nerve results in diaphragmatic paralysis and paradoxical movement, clinically important during neck trauma or thoracic surgery.
2) The sensory supply of the peripheral diaphragm is via–
a) Vagus nerve
b) Intercostal nerves
c) Glossopharyngeal nerve
d) Long thoracic nerve
Answer: b) Intercostal nerves
Explanation: Peripheral parts of the diaphragm receive sensory supply from intercostal nerves (T5–T11). This explains referred pain to the thoracic wall.
3) The diaphragm develops from all except–
a) Septum transversum
b) Pleuroperitoneal membranes
c) Dorsal mesentery of esophagus
d) Somites of cervical region (C3–C5)
Answer: d) Somites of cervical region (C3–C5)
Explanation: Cervical somites do not form diaphragm tissue; they only contribute phrenic nerve fibres.
4) A penetrating injury at the neck near the anterior scalene risks paralysis of–
a) Diaphragm
b) Deltoid
c) Pectoralis major
d) Wrist extensors
Answer: a) Diaphragm
Explanation: The phrenic nerve runs on the anterior scalene; injury causes diaphragmatic paralysis.
5) The aortic hiatus is located at–
a) T8
b) T10
c) T12
d) L1
Answer: c) T12
Explanation: Aortic hiatus at T12 transmits aorta, thoracic duct, and azygous vein.
6) The IVC passes through the diaphragm at which part?
a) Muscular portion
b) Central tendon
c) Right crus
d) Left crus
Answer: b) Central tendon
Explanation: IVC passes through the central tendon at T8, preventing compression during respiration.
7) Left phrenic nerve palsy results in–
a) Elevated left dome of diaphragm
b) Elevated right dome
c) Bilateral elevation
d) No change
Answer: a) Elevated left dome of diaphragm
Explanation: Paralysis causes paradoxical upward movement on inspiration and elevation on imaging.
8) Herniation through the esophageal hiatus most commonly leads to–
a) Sliding hiatal hernia
b) Morgagni hernia
c) Lumbar hernia
d) Umbilical hernia
Answer: a) Sliding hiatal hernia
Explanation: The esophageal hiatus enlargement leads to sliding gastroesophageal herniation.
9) The structure passing anterior to the root of the lung and close to the diaphragm is–
a) Phrenic nerve
b) Vagus nerve
c) Recurrent laryngeal nerve
d) Sympathetic chain
Answer: a) Phrenic nerve
Explanation: Phrenic nerve passes anterior to the lung root and descends to the diaphragm.
10) The right crus of diaphragm gives fibers forming the–
a) Aortic sphincter
b) Esophageal sphincter
c) Pulmonary sphincter
d) IVC sphincter
Answer: b) Esophageal sphincter
Explanation: Right crus fibers contribute to lower esophageal sphincter function.
11) Complete failure of pleuroperitoneal membrane fusion results in–
a) Bochdalek hernia
b) Sliding hernia
c) Incisional hernia
d) Paraumbilical hernia
Answer: a) Bochdalek hernia
Explanation: Posterolateral congenital diaphragmatic hernia (Bochdalek) arises from membrane fusion failure.
Chapter: Thorax; Topic: Intercostal Nerves; Subtopic: Typical and Atypical Intercostal Nerves
Keyword Definitions:
Intercostal nerve: Ventral rami of thoracic spinal nerves supplying thoracic wall.
Typical intercostal nerve: A nerve that follows the standard course and branches (T3–T6).
Atypical intercostal nerve: Nerves with special courses or distributions (T1, T2, T7–T12).
Lateral cutaneous branch: Sensory branch supplying lateral thoracic wall.
Collateral branch: Branch running below the main intercostal nerve.
1) Lead Question – 2016
Which is a typical intercostal nerve?
a) First
b) Second
c) Third
d) Seventh
Answer: c) Third
Explanation: Typical intercostal nerves are those that follow the standard course between the internal and innermost intercostal muscles, giving rise to lateral cutaneous, muscular, and collateral branches. These include **T3 to T6**. They supply the thoracic wall without taking part in plexus formation or supplying abdominal structures. In contrast, T1 contributes to the brachial plexus, T2 gives rise to the intercostobrachial nerve, and T7–T12 extend into the abdominal wall. Therefore, the **3rd intercostal nerve** is a typical intercostal nerve.
2) Which intercostal nerve gives rise to the intercostobrachial nerve?
a) T1
b) T2
c) T3
d) T4
Answer: b) T2
Explanation: The T2 intercostal nerve becomes atypical because it gives off the intercostobrachial nerve to the axilla and medial upper arm.
3) Which intercostal nerve contributes to the brachial plexus?
a) T1
b) T3
c) T4
d) T6
Answer: a) T1
Explanation: The T1 nerve root joins the brachial plexus and thus is not considered a typical intercostal nerve.
4) Intercostal nerves run between–
a) External and internal intercostals
b) Internal and innermost intercostals
c) Innermost and pleura
d) Endothoracic fascia and pleura
Answer: b) Internal and innermost intercostals
Explanation: The neurovascular bundle (vein-artery-nerve) lies in the costal groove between internal and innermost intercostals.
5) Pain from the intercostal muscles is transmitted via–
a) Dorsal rami
b) Ventral rami
c) Sympathetic fibers
d) Parasympathetic fibers
Answer: b) Ventral rami
Explanation: Intercostal nerves are ventral rami of thoracic spinal nerves T1–T11.
6) T7–T11 intercostal nerves become–
a) Lumbar nerves
b) Abdominothoracic nerves
c) Phrenic nerves
d) Cervical nerves
Answer: b) Abdominothoracic nerves
Explanation: These nerves continue beyond the costal margin to supply abdominal muscles.
7) The dermatome of the nipple corresponds to–
a) T2
b) T3
c) T4
d) T6
Answer: c) T4
Explanation: T4 dermatome marks the nipple level and is clinically important for thoracic sensory mapping.
8) Which branch of a typical intercostal nerve supplies the skin of the thorax laterally?
a) Anterior cutaneous
b) Muscular
c) Lateral cutaneous
d) Collateral
Answer: c) Lateral cutaneous
Explanation: The lateral cutaneous branch divides into anterior and posterior branches supplying lateral thorax skin.
9) Intercostal nerve block is given in relation to–
a) Superior border of rib
b) Inferior border of rib
c) Mid-shaft of rib
d) Costochondral junction
Answer: a) Superior border of rib
Explanation: The neurovascular bundle lies inferior to the rib; hence injection is given near the superior border of the rib below.
10) Which nerve supplies the skin over the umbilicus?
a) T6
b) T10
c) T12
d) L1
Answer: b) T10
Explanation: T10 dermatome corresponds to the umbilicus, used clinically for spinal cord lesion localization.
11) Which intercostal nerve is important in angina pain referral?
a) T1
b) T2
c) T4
d) T5
Answer: c) T4
Explanation: Cardiac pain is referred mainly through T1–T4 dermatomes, making T4 significant in precordial pain pathways.
Chapter: Peripheral Nerve Anatomy; Topic: Cutaneous Nerve Supply of Foot; Subtopic: Sensory Innervation of Lateral Border of Foot
Keyword Definitions:
Sural Nerve: Sensory nerve supplying posterolateral leg and lateral border of foot.
Saphenous Nerve: Longest cutaneous nerve, supplying medial leg and medial foot.
Deep Peroneal Nerve: Supplies 1st web space sensation and dorsiflexors of foot.
Superficial Peroneal Nerve: Sensory to dorsum of foot except 1st web space.
Cutaneous Dermatomes: Sensory distribution areas of peripheral nerves in the limb.
1) Lead Question – 2016
Lateral border of the foot receives its sensory supply from?
A) Saphenous nerve
B) Sural nerve
C) Deep peroneal nerve
D) Sciatic nerve
Answer: B) Sural nerve
Explanation: The sural nerve is responsible for sensory innervation of the posterolateral aspect of the leg and the lateral border of the foot including the little toe. It is formed by contributions from the tibial nerve (medial sural cutaneous nerve) and the common peroneal nerve (sural communicating branch). The saphenous nerve supplies the medial foot, not the lateral side. The deep peroneal nerve supplies only the first web space. The sciatic nerve does not supply direct cutaneous sensation to the foot. Hence, the sural nerve is the correct answer.
2) The deep peroneal nerve supplies sensation to–
A) Lateral foot
B) First web space
C) Medial arch
D) Heel
Answer: B) First web space
Explanation: Deep peroneal nerve innervates only the first dorsal web space. Thus, B is correct.
3) Saphenous nerve supplies–
A) Posterior leg
B) Lateral foot
C) Medial foot
D) Sole of foot
Answer: C) Medial foot
Explanation: Saphenous nerve is sensory to medial leg and medial foot. Thus, C is correct.
4) Sural nerve is formed by branches of–
A) Femoral + tibial
B) Tibial + common peroneal
C) Obturator + tibial
D) Superficial + deep peroneal
Answer: B) Tibial + common peroneal
Explanation: Medial sural cutaneous + sural communicating branch join to form sural nerve. Thus, B is correct.
5) A patient with injury behind the lateral malleolus may lose sensation over–
A) First web space
B) Medial malleolus
C) Lateral border of foot
D) Heel
Answer: C) Lateral border of foot
Explanation: Sural nerve runs behind lateral malleolus; injury reduces lateral foot sensation. Thus, C is correct.
6) The tibial nerve gives rise to which sensory branch in the foot?
A) Sural nerve component
B) Superficial peroneal nerve
C) Deep peroneal nerve
D) Saphenous nerve
Answer: A) Sural nerve component
Explanation: Tibial nerve contributes medial sural cutaneous branch. Thus, A is correct.
7) Sensory loss over the 5th toe indicates lesion of–
A) Deep peroneal nerve
B) Sural nerve
C) Saphenous nerve
D) Tibial nerve
Answer: B) Sural nerve
Explanation: The sural nerve supplies lateral foot and little toe. Thus, B is correct.
8) Heel sensation is supplied by–
A) Medial plantar nerve
B) Lateral plantar nerve
C) Medial calcaneal branch of tibial nerve
D) Deep peroneal nerve
Answer: C) Medial calcaneal branch of tibial nerve
Explanation: Tibial nerve gives calcaneal branches to heel. Thus, C is correct.
9) The superficial peroneal nerve supplies–
A) Medial foot
B) Heel
C) Most of dorsum of foot
D) Lateral border of foot
Answer: C) Most of dorsum of foot
Explanation: SPN supplies dorsum except 1st web space. Thus, C is correct.
10) A clinical sign of sural nerve injury is–
A) Weak dorsiflexion
B) Loss of medial foot sensation
C) Loss of lateral foot sensation
D) Loss of heel reflex
Answer: C) Loss of lateral foot sensation
Explanation: Sural nerve is purely sensory to lateral foot; injury leads to sensory deficit. Thus, C is correct.
11) Which nerve is purely sensory?
A) Deep peroneal nerve
B) Tibial nerve
C) Common peroneal nerve
D) Sural nerve
Answer: D) Sural nerve
Explanation: Sural nerve has no motor fibers and is purely sensory, supplying lateral foot. Thus, D is correct.
Chapter: Peripheral Nerve Anatomy; Topic: Cutaneous Nerves of Lower Limb; Subtopic: Longest Cutaneous Nerve
Keyword Definitions:
Saphenous Nerve: Longest cutaneous nerve in the body, branch of femoral nerve supplying medial leg and foot.
Cutaneous Nerve: Nerve supplying sensation to skin without motor function.
Femoral Nerve: Major nerve of anterior thigh giving rise to saphenous nerve.
Sural Nerve: Sensory nerve of posterior leg formed from tibial and common peroneal nerves.
Lateral Cutaneous Nerve of Thigh: Pure sensory nerve supplying lateral thigh.
1) Lead Question – 2016
Longest cutaneous nerve in body?
A) Lateral cutaneous nerve of thigh
B) Medial cutaneous nerve of thigh
C) Saphenous nerve
D) Sural nerve
Answer: C) Saphenous nerve
Explanation: The saphenous nerve is the longest cutaneous branch of the femoral nerve and the longest pure sensory nerve in the human body. It travels through the adductor canal, emerges medially at the knee, and extends down to the medial malleolus and medial border of the foot. The lateral and medial cutaneous nerves of the thigh are much shorter. The sural nerve is long but not as extensive as the saphenous nerve. Therefore, the longest cutaneous nerve is the saphenous nerve.
2) The saphenous nerve is a branch of–
A) Common peroneal nerve
B) Tibial nerve
C) Femoral nerve
D) Obturator nerve
Answer: C) Femoral nerve
Explanation: Saphenous nerve arises from femoral nerve in adductor canal. Thus, C is correct.
3) Sural nerve supplies sensory innervation to–
A) Medial foot
B) Sole of foot
C) Lateral foot
D) Lateral thigh
Answer: C) Lateral foot
Explanation: Sural nerve innervates posterolateral leg and lateral foot. Thus, C is correct.
4) Injury to saphenous nerve causes sensory loss over–
A) Posterior leg
B) Medial leg
C) Lateral foot
D) Dorsum of foot
Answer: B) Medial leg
Explanation: Saphenous nerve supplies entire medial leg. Thus, B is correct.
5) Lateral cutaneous nerve of thigh arises from–
A) Femoral nerve
B) L2–L3
C) L3–L4
D) S1–S2
Answer: B) L2–L3
Explanation: LCNT derives from L2–L3 roots. Thus, B is correct.
6) Medial cutaneous nerve of thigh is a branch of–
A) Sciatic nerve
B) Femoral nerve
C) Tibial nerve
D) Obturator nerve
Answer: B) Femoral nerve
Explanation: It arises from femoral nerve supplying skin over medial thigh. Thus, B is correct.
7) Sural nerve is formed by branches of–
A) Femoral + tibial
B) Sciatic + obturator
C) Tibial + common peroneal
D) Obturator + femoral
Answer: C) Tibial + common peroneal
Explanation: Medial sural cutaneous (tibial) + sural communicating branch (peroneal) form it. Thus, C is correct.
8) A patient with varicose vein stripping is at risk of injuring–
A) Sural nerve
B) Saphenous nerve
C) LCNT
D) Medial plantar nerve
Answer: B) Saphenous nerve
Explanation: Saphenous nerve lies close to great saphenous vein. Thus, B is correct.
9) The saphenous nerve accompanies which vessel at ankle?
A) Posterior tibial artery
B) Dorsalis pedis artery
C) Great saphenous vein
D) Popliteal artery
Answer: C) Great saphenous vein
Explanation: Saphenous nerve runs with GSV in leg. Thus, C is correct.
10) Meralgia paresthetica involves–
A) Saphenous nerve
B) LCNT
C) Sural nerve
D) Obturator nerve
Answer: B) LCNT
Explanation: Compression of LCNT causes pain, tingling in lateral thigh. Thus, B is correct.
11) Sensation over medial malleolus is supplied by–
A) Sural
B) Deep peroneal
C) Saphenous
D) Superficial peroneal
Answer: C) Saphenous
Explanation: Saphenous nerve exclusively supplies medial ankle and malleolar area. Thus, C is correct.
Chapter: Spine & Nervous System Anatomy; Topic: Sacrum – Foramina & Hiatus; Subtopic: Structures Passing Through Sacral Hiatus
Keyword Definitions:
Sacral Hiatus: An opening at the inferior end of sacral canal formed due to failure of laminae fusion of S4–S5.
Sacral Canal: Continuation of vertebral canal containing cauda equina and sacral nerves.
Sacral Nerve Roots: Ventral and dorsal roots emerging from sacral segments S1–S5.
Filum Terminale: Fibrous extension from conus medullaris that exits via sacral hiatus.
Caudal Epidural:** Site for caudal epidural anesthesia through sacral hiatus.
1) Lead Question – 2016
Structures passing through sacral hiatus are?
A) S4 nerve root
B) S2 nerve root
C) S3 nerve root
D) S5 nerve root
Answer: D) S5 nerve root
Explanation: The sacral hiatus is formed due to the non-fusion of the laminae of S4 and S5 vertebrae. It transmits the S5 nerve root, coccygeal nerve, and the filum terminale externum. Nerve roots S1–S4 exit through separate sacral foramina and do not pass through the sacral hiatus. Clinically, the sacral hiatus is used for caudal epidural anesthesia, making familiarity with its contents essential. As only the S5 nerve root passes through the sacral hiatus, the correct answer is option D.
2) Sacral hiatus is formed due to failure of fusion of–
A) S1 laminae
B) S2 laminae
C) S4–S5 laminae
D) S3 laminae
Answer: C) S4–S5 laminae
Explanation: Non-fusion of S4–S5 laminae creates sacral hiatus. Thus, C is correct.
3) The filum terminale externum attaches to–
A) L5 vertebra
B) Coccyx
C) Sacral promontory
D) Ilium
Answer: B) Coccyx
Explanation: Filum terminale anchors spinal cord to coccyx. Thus, B is correct.
4) Caudal epidural anesthesia is given through–
A) Sacral promontory
B) Sacral hiatus
C) Posterior sacral foramina
D) Intervertebral foramina
Answer: B) Sacral hiatus
Explanation: Sacral hiatus gives access to epidural space. Thus, B is correct.
5) Posterior sacral foramina transmit–
A) Ventral rami
B) Dorsal rami
C) Sympathetic chain
D) Lumbosacral trunk
Answer: B) Dorsal rami
Explanation: Dorsal rami exit posterior foramina to supply back muscles. Thus, B is correct.
6) Anterior sacral foramina transmit–
A) Cauda equina
B) Ventral rami
C) Dorsal rami
D) Ilioinguinal nerve
Answer: B) Ventral rami
Explanation: Ventral rami forming sacral plexus exit anteriorly. Thus, B is correct.
7) Coccygeal nerve emerges through–
A) Sacral hiatus
B) L5/S1 foramen
C) S2 foramen
D) Obturator canal
Answer: A) Sacral hiatus
Explanation: Coccygeal nerve exits via sacral hiatus. Thus, A is correct.
8) The sacral cornua represent–
A) Transverse processes
B) Inferior articular processes of S5
C) Fused spinous processes
D) Superior articular facets
Answer: B) Inferior articular processes of S5
Explanation: Cornua flank sacral hiatus. Thus, B is correct.
9) Caudal epidural block anesthetizes–
A) L2 dermatome
B) Sacral nerve roots
C) Brachial plexus
D) Cervical roots
Answer: B) Sacral nerve roots
Explanation: Local anesthetic spreads to S2–S5 levels. Thus, B is correct.
10) The sacral canal contains all except–
A) Cauda equina
B) Filum terminale
C) Coccygeal nerve
D) Sympathetic trunk
Answer: D) Sympathetic trunk
Explanation: Sympathetic trunk lies anterior to sacrum. Thus, D is correct.
11) Sacral hiatus is clinically palpated between–
A) Sacral cornua
B) Posterior superior iliac spines
C) Coccygeal tip
D) Ischial tuberosities
Answer: A) Sacral cornua
Explanation: Sacral cornua guide needle placement for caudal block. Thus, A is correct.
Chapter: Lower Limb Neuroanatomy; Topic: Lumbosacral Myotomes; Subtopic: Motor Supply to Dorsiflexors of Foot
Keyword Definitions:
Extensor Hallucis Longus (EHL): Muscle extending great toe and assisting ankle dorsiflexion.
L5 Nerve Root: Major root supplying muscles responsible for dorsiflexion and toe extension.
Myotome: Group of muscles supplied predominantly by a single spinal nerve root.
Foot Drop: Weakness of ankle dorsiflexors often due to L5 root or peroneal nerve lesions.
Deep Peroneal Nerve: Peripheral nerve carrying L4–L5 fibers to EHL.
1) Lead Question – 2016
Weakness of extensor hallucis longus is due to which nerve root mainly?
A) L5
B) L4
C) S1
D) S2
Answer: A) L5
Explanation: Extensor hallucis longus is the key muscle tested for the L5 myotome. It extends the great toe and is innervated mainly by L5 fibers carried through the deep peroneal nerve. Weakness of EHL is commonly seen in L5 radiculopathy, disc herniation at L4–L5, or deep peroneal nerve neuropathy. L4 contributes mainly to quadriceps and ankle inversion, while S1 supplies plantarflexors. Thus, the muscle most strongly associated with the L5 root is EHL, making option A correct.
2) The primary action of extensor hallucis longus is–
A) Plantarflexion
B) Extension of great toe
C) Foot eversion
D) Knee extension
Answer: B) Extension of great toe
Explanation: EHL extends great toe and aids dorsiflexion. Thus, B is correct.
3) L5 radiculopathy typically presents with–
A) Loss of ankle jerk
B) Weak EHL
C) Weak plantarflexion
D) Sensory loss over heel
Answer: B) Weak EHL
Explanation: L5 is best tested by EHL strength. Thus, B is correct.
4) Deep peroneal nerve supplies all except–
A) Tibialis anterior
B) Extensor hallucis longus
C) Extensor digitorum longus
D) Fibularis longus
Answer: D) Fibularis longus
Explanation: Fibularis longus is superficial peroneal nerve supply. Thus, D is correct.
5) A patient with herniation at L4–L5 will commonly show–
A) Loss of knee jerk
B) Foot drop
C) Loss of ankle jerk
D) Weak quadriceps
Answer: B) Foot drop
Explanation: L5 compression leads to dorsiflexor weakness. Thus, B is correct.
6) Sensory loss in L5 lesion is found over–
A) Medial malleolus
B) Lateral foot
C) Dorsum of foot and great toe
D) Posterior thigh
Answer: C) Dorsum of foot and great toe
Explanation: L5 dermatome covers dorsum of foot and hallux. Thus, C is correct.
7) Extensor digitorum longus is supplied mainly by–
A) L3
B) L4
C) L5
D) S1
Answer: C) L5
Explanation: EDL shares L5 dominance like EHL. Thus, C is correct.
8) S1 root lesion most strongly affects–
A) EHL
B) Gastrocnemius
C) Tibialis anterior
D) EDL
Answer: B) Gastrocnemius
Explanation: S1 is key for plantarflexion and Achilles reflex. Thus, B is correct.
9) Weak dorsiflexion of ankle is seen in–
A) Tibial neuropathy
B) L5 radiculopathy
C) S2 lesion
D) Femoral neuropathy
Answer: B) L5 radiculopathy
Explanation: Dorsiflexors share L5 fibers; radiculopathy weakens them. Thus, B is correct.
10) A positive heel-walk test indicates–
A) L4 lesion
B) L5 weakness
C) S1 lesion
D) L3 lesion
Answer: B) L5 weakness
Explanation: Heel-walking tests dorsiflexors supplied mainly by L5. Thus, B is correct.
11) Compression of common peroneal nerve affects–
A) Plantarflexors
B) Everters and dorsiflexors
C) Hip flexors
D) Knee extensors
Answer: B) Everters and dorsiflexors
Explanation: Common peroneal nerve injury weakens L5-dominant dorsiflexors and everters. Thus, B is correct.
Chapter: Upper Limb Anatomy; Topic: Clavicular Region; Subtopic: Muscles Protecting Neurovascular Structures
Keyword Definitions:
Subclavius: Small muscle beneath the clavicle protecting brachial plexus and subclavian vessels.
Brachial Plexus: Major nerve network supplying the upper limb, located inferior to clavicle.
Clavicle Fracture: Common injury where displaced fragments may threaten underlying structures.
Subclavian Vessels: Large artery and vein passing under clavicle.
Shoulder Girdle Muscles: Muscles stabilizing clavicle and scapula during upper limb motion.
1) Lead Question – 2016
Which muscle protects the brachial plexus in case of clavicle fractures?
A) Subclavius
B) Supraspinatus
C) Subscapularis
D) Teres minor
Answer: A) Subclavius
Explanation: The subclavius muscle lies directly beneath the clavicle, inserting onto its inferior surface. During clavicle fractures, especially mid-shaft injuries, the sharp bone fragments may threaten the brachial plexus and subclavian vessels below. The subclavius acts as a protective cushion, preventing direct damage. None of the other listed muscles lie in this position or serve this protective role. Supraspinatus is superior to the scapula, subscapularis lies on the anterior surface of the scapula, and teres minor is posterior. Thus, the correct answer is A.
2) Subclavius is innervated by–
A) Nerve to subclavius
B) Long thoracic nerve
C) Axillary nerve
D) Suprascapular nerve
Answer: A) Nerve to subclavius
Explanation: The muscle receives its supply from C5–C6 via the nerve to subclavius. Thus, A is correct.
3) Which vessel is most at risk in clavicular fractures?
A) Axillary artery
B) Subclavian artery
C) Radial artery
D) Ulnar artery
Answer: B) Subclavian artery
Explanation: The subclavian artery runs beneath the clavicle. Thus, B is correct.
4) Which structure runs posterior to the clavicle?
A) Brachial plexus
B) Cephalic vein
C) Median nerve
D) Basilic vein
Answer: A) Brachial plexus
Explanation: The trunks of the brachial plexus lie inferior and posterior to the clavicle. Thus, A is correct.
5) Subclavius stabilizes which joint?
A) AC joint
B) Sternoclavicular joint
C) Glenohumeral joint
D) Scapulothoracic joint
Answer: B) Sternoclavicular joint
Explanation: Subclavius anchors clavicle to first rib, stabilizing SC joint. Thus, B is correct.
6) A patient with clavicular fracture shows numbness in lateral arm. Which nerve root is likely affected?
A) C5
B) C8
C) T1
D) C7
Answer: A) C5
Explanation: Upper trunk compression affects C5, producing lateral arm sensory loss. Thus, A is correct.
7) Which muscle elevates the first rib along with sternocleidomastoid?
A) Subclavius
B) Scalenus anterior
C) Pectoralis minor
D) Trapezius
Answer: A) Subclavius
Explanation: Subclavius can assist in first rib stabilization or slight elevation. Thus, A is correct.
8) Clavicle fractures most commonly occur at–
A) Medial third
B) Middle third
C) Lateral third
D) Clavicular head
Answer: B) Middle third
Explanation: The middle third is structurally weakest. Thus, B is correct.
9) Subclavius inserts on the–
A) Clavicle (inferior surface)
B) First rib midpoint
C) Acromion
D) Coracoid process
Answer: A) Clavicle (inferior surface)
Explanation: It attaches to the subclavian groove. Thus, A is correct.
10) Which muscle depresses the clavicle?
A) Subclavius
B) Sternocleidomastoid
C) Supraspinatus
D) Levator scapulae
Answer: A) Subclavius
Explanation: Subclavius pulls clavicle downward and medially. Thus, A is correct.
11) Injury to subclavian vein may lead to–
A) Air embolism
B) Digital ischemia
C) Median nerve palsy
D) Horner syndrome
Answer: A) Air embolism
Explanation: Subclavian vein injury may suck in air under negative pressure, causing embolism. Thus, A is correct.
Chapter: Upper Limb Anatomy; Topic: Wrist Anatomy; Subtopic: Structures Passing Deep to Flexor Retinaculum
Keyword Definitions:
Flexor Retinaculum: Strong fibrous band forming the roof of the carpal tunnel.
Carpal Tunnel: Passageway containing median nerve and flexor tendons.
Median Nerve: Only nerve passing deep to the flexor retinaculum.
Ulnar Canal (Guyon’s Canal): Tunnel where ulnar nerve and artery pass superficial to flexor retinaculum.
Flexor Tendons: FDS, FDP, and FPL pass deep to the flexor retinaculum.
1) Lead Question – 2016
Structure passing deep to flexor retinaculum at the wrist:
A) Ulnar nerve
B) Median nerve
C) Radial nerve
D) Ulnar artery
Answer: B) Median nerve
Explanation: The flexor retinaculum forms the roof of the carpal tunnel, and the median nerve is the only nerve that passes deep to it. The flexor tendons—FDS, FDP, and FPL—also pass deep to the retinaculum. The ulnar nerve and ulnar artery pass superficial to the flexor retinaculum through Guyon’s canal. The radial nerve does not enter the carpal tunnel. Therefore, the correct answer is B. Injury or compression of the median nerve deep to the flexor retinaculum commonly causes carpal tunnel syndrome.
2) Which structure passes superficial to the flexor retinaculum?
A) Median nerve
B) Ulnar nerve
C) FPL tendon
D) FDP tendon
Answer: B) Ulnar nerve
Explanation: The ulnar nerve travels through Guyon’s canal superficial to the flexor retinaculum. Thus, B is correct.
3) Carpal tunnel syndrome affects which nerve?
A) Radial nerve
B) Median nerve
C) Ulnar nerve
D) Musculocutaneous nerve
Answer: B) Median nerve
Explanation: Median nerve compression causes carpal tunnel syndrome. Thus, B is correct.
4) Which tendon passes through the carpal tunnel?
A) EPL
B) ECRB
C) FDS
D) APL
Answer: C) FDS
Explanation: Flexor digitorum superficialis passes deep to flexor retinaculum. Thus, C is correct.
5) Guyon’s canal contains–
A) Median nerve
B) Ulnar nerve and artery
C) Radial artery only
D) Ulnar nerve only
Answer: B) Ulnar nerve and artery
Explanation: Both structures run superficial to the flexor retinaculum. Hence, B is correct.
6) A patient with carpal tunnel syndrome will show weakness of–
A) Interossei
B) Thenar muscles
C) Forearm extensors
D) Triceps
Answer: B) Thenar muscles
Explanation: Median nerve supplies thenar muscles affected in CTS. Thus, B is correct.
7) The flexor retinaculum attaches to–
A) Scaphoid and trapezium
B) Lunate and capitate
C) Pisiform and hook of hamate
D) Both A and C
Answer: D) Both A and C
Explanation: The flexor retinaculum spans from scaphoid/trapezium to pisiform/hamate. Thus, D is correct.
8) Median nerve injury at carpal tunnel causes loss of–
A) Wrist flexion
B) Finger flexion
C) Thumb opposition
D) Thumb adduction
Answer: C) Thumb opposition
Explanation: Opponens pollicis is median-supplied. Thus, C is correct.
9) Flexor pollicis longus tendon lies–
A) Superficial to flexor retinaculum
B) Deep to flexor retinaculum
C) In Guyon’s canal
D) In radial tunnel
Answer: B) Deep to flexor retinaculum
Explanation: FPL passes through the carpal tunnel. Thus, B is correct.
10) Superficial radial nerve supplies sensation to–
A) Thumb pulp
B) Dorsal thumb
C) Little finger
D) Palmar index
Answer: B) Dorsal thumb
Explanation: Radial nerve supplies dorsolateral hand. Thus, B is correct.
11) Compression of structures superficial to flexor retinaculum leads to–
A) Carpal tunnel syndrome
B) Guyon’s canal syndrome
C) Pronator syndrome
D) Radial tunnel syndrome
Answer: B) Guyon’s canal syndrome
Explanation: Ulnar nerve compression superficial to retinaculum leads to this syndrome. Thus, B is correct.
Chapter: Upper Limb Anatomy; Topic: Peripheral Nerve Injuries; Subtopic: Radial Nerve Lesions (Low Lesions)
Keyword Definitions:
Radial Nerve: Nerve supplying extensors of arm, forearm, wrist, fingers, and thumb.
Spiral Groove Injury: Radial nerve lesion occurring at radial groove of humerus.
Low Radial Nerve Palsy: Lesion distal to triceps branches, sparing elbow extension.
Wrist Drop: Inability to extend wrist due to extensor muscle paralysis.
Digital Extensors: Radial-innervated muscles allowing finger and thumb extension.
1) Lead Question – 2016
Low radial nerve (just after spiral groove) palsy does not produce?
A) Loss of wrist extension
B) Loss of elbow extension
C) Loss of finger extension
D) Loss of thumb extension
Answer: B) Loss of elbow extension
Explanation: A low radial nerve palsy—occurring just after the spiral groove—spares the branches to triceps because these branches arise proximally. Therefore, elbow extension remains intact. However, wrist extensors, finger extensors, and thumb extensors receive their innervation distal to this level and are therefore affected, resulting in wrist drop, inability to extend fingers, and loss of thumb extension. Hence, the only function preserved is elbow extension, making option B the correct choice.
2) Wrist drop is a classic feature of–
A) Median nerve injury
B) Ulnar nerve injury
C) Radial nerve injury
D) Axillary nerve injury
Answer: C) Radial nerve injury
Explanation: Radial nerve supplies wrist extensors; its palsy causes wrist drop. Thus, C is correct.
3) Sensation over the anatomical snuffbox is supplied by–
A) Median nerve
B) Ulnar nerve
C) Radial nerve
D) Axillary nerve
Answer: C) Radial nerve
Explanation: The superficial branch of radial nerve supplies snuffbox region. Thus, C is correct.
4) A fracture of mid-shaft humerus commonly injures–
A) Axillary nerve
B) Radial nerve
C) Median nerve
D) Musculocutaneous nerve
Answer: B) Radial nerve
Explanation: The radial nerve lies in spiral groove of humerus and is commonly injured by mid-shaft fractures. Thus, B is correct.
5) Radial nerve injury at axilla leads to loss of–
A) Elbow extension
B) Wrist extension
C) Finger extension
D) All of the above
Answer: D) All of the above
Explanation: High lesion affects triceps and all extensor muscles. Thus, D is correct.
6) Posterior interosseous nerve palsy presents with–
A) Wrist drop
B) Finger drop with preserved wrist extension
C) Thumb opposition loss
D) Clawing
Answer: B) Finger drop with preserved wrist extension
Explanation: PIN supplies finger extensors; wrist extensors remain functional via ECRL. Thus, B is correct.
7) Loss of brachioradialis reflex suggests injury to–
A) C8
B) T1
C) C6
D) C5
Answer: C) C6
Explanation: C6 root supplies brachioradialis via radial nerve. Thus, C is correct.
8) Inability to extend the thumb indicates involvement of–
A) AIN
B) Ulnar nerve
C) Radial nerve
D) Musculocutaneous nerve
Answer: C) Radial nerve
Explanation: EPL and EPB are radial-innervated. Thus, C is correct.
9) Sensory loss in dorsum of hand over lateral three fingers suggests lesion of–
A) Median nerve
B) Ulnar nerve
C) Radial nerve
D) Axillary nerve
Answer: C) Radial nerve
Explanation: Superficial radial nerve supplies dorsolateral hand. Thus, C is correct.
10) In low radial nerve palsy, which nerve branch remains functional?
A) Posterior interosseous nerve
B) Branches to triceps
C) Superficial radial nerve
D) All branches lost
Answer: B) Branches to triceps
Explanation: Triceps branches arise proximal to spiral groove and are spared. Thus, B is correct.
11) Patient with wrist drop but preserved elbow extension has lesion likely at–
A) Axilla
B) Spiral groove
C) Lateral epicondyle
D) Wrist
Answer: B) Spiral groove
Explanation: Classic finding of low radial palsy: wrist drop with normal elbow extension. Thus, B is correct.
Chapter: Upper Limb Anatomy; Topic: Cutaneous Nerve Supply of Hand; Subtopic: Digital Sensory Distribution
Keyword Definitions:
Digital Pulp: The highly sensitive fingertip area rich in sensory receptors, essential for fine touch.
Median Nerve: Supplies palmar aspect and fingertip pulps of thumb, index, middle, and lateral half of ring finger.
Radial Nerve: Supplies dorsum of hand but not pulps of digits (except a small dorsal part of thumb).
Ulnar Nerve: Supplies medial 1½ digits including little finger and medial ring finger.
Palmar Digital Branches: Terminal branches of median and ulnar nerves supplying fingertips.
1) Lead Question – 2016
Pulp of the index finger is supplied by:
A) Median nerve
B) Radial nerve
C) Ulnar nerve
D) Axillary nerve
Answer: A) Median nerve
Explanation: The palmar digital branches of the median nerve supply the pulps of the thumb, index, middle, and lateral half of the ring finger. The radial nerve supplies only the dorsal aspect of the hand and does not supply the fingertip pulp. The ulnar nerve supplies the medial 1½ digits (little finger and medial half of ring finger). The axillary nerve does not supply hand skin. Therefore, the pulp of the index finger is supplied exclusively by the median nerve, making option A correct.
2) Sensation of little finger pulp is supplied by–
A) Median nerve
B) Ulnar nerve
C) Radial nerve
D) Axillary nerve
Answer: B) Ulnar nerve
Explanation: Ulnar nerve supplies medial 1½ digits. Thus, B is correct.
3) The median nerve supplies pulp of all the following except–
A) Thumb
B) Index
C) Middle
D) Little finger
Answer: D) Little finger
Explanation: Little finger is supplied by ulnar nerve. Thus, D is correct.
4) A patient with carpal tunnel syndrome may develop sensory loss over–
A) Dorsal thumb
B) Pulp of index finger
C) Medial palm
D) Little finger
Answer: B) Pulp of index finger
Explanation: Median nerve compression affects index pulp sensation. Thus, B is correct.
5) The first dorsal web space is supplied by–
A) Ulnar nerve
B) Radial nerve
C) Median nerve
D) Axillary nerve
Answer: B) Radial nerve
Explanation: Radial nerve supplies dorsal web space. Thus, B is correct.
6) Sensory loss isolated to index fingertip suggests lesion at–
A) Axilla (radial nerve)
B) Wrist (median nerve)
C) Medial epicondyle
D) Elbow (ulnar nerve)
Answer: B) Wrist (median nerve)
Explanation: Median nerve compression at wrist affects digital pulps. Thus, B is correct.
7) Dorsal aspect of index finger is mainly supplied by–
A) Median nerve
B) Radial nerve
C) Ulnar nerve
D) Axillary nerve
Answer: B) Radial nerve
Explanation: Radial nerve supplies dorsum of lateral digits except pulps. Thus, B is correct.
8) Which nerve supplies palmar surface of thumb?
A) Median nerve
B) Radial nerve
C) Ulnar nerve
D) Musculocutaneous nerve
Answer: A) Median nerve
Explanation: Median nerve digital branches supply thumb pulp and palmar skin. Thus, A is correct.
9) Injury to ulnar nerve at the wrist results in sensory loss over–
A) Index pulp
B) Lateral palm
C) Little finger pulp
D) Dorsal thumb
Answer: C) Little finger pulp
Explanation: Ulnar nerve loss leads to sensory deficit in medial digits. Thus, C is correct.
10) A child with supracondylar humerus fracture has intact index pulp sensation. Which nerve is spared?
A) Median nerve
B) Radial nerve
C) Ulnar nerve
D) All three
Answer: A) Median nerve
Explanation: Index pulp is median-supplied; intact sensation indicates median nerve spared. Thus, A is correct.
11) Loss of lateral palm sensation indicates lesion of–
A) Ulnar nerve
B) Musculocutaneous nerve
C) Median nerve
D) Radial nerve
Answer: C) Median nerve
Explanation: Median nerve supplies lateral palmar skin. Thus, C is correct.
Chapter: Upper Limb Anatomy; Topic: Nerve Supply of Hand; Subtopic: Cutaneous Distribution of Ulnar Nerve
Keyword Definitions:
Ulnar Nerve: Major nerve of hand supplying medial digits and most intrinsic hand muscles.
Cutaneous Distribution: Area of skin supplied by a specific peripheral nerve.
Finger Tips Sensory Zones: Distal phalanx pads used to identify nerve territories.
Dorsal Branch of Ulnar Nerve: Supplies dorsomedial hand and little finger sensation.
Median Nerve Digital Supply: Supplies thumb, index, middle, and half of ring finger tips.
1) Lead Question – 2016
Sensory region of the ulnar nerve is?
A) Tip of little finger
B) Tip of index finger
C) 1st web space
D) Lateral upper aspect of arm
Answer: A) Tip of little finger
Explanation: The ulnar nerve supplies the medial 1½ digits, which includes the tip of the little finger and the medial half of the ring finger. The tip of the index finger is supplied by the median nerve; the first dorsal web space is supplied by the radial nerve; and the lateral aspect of the upper arm is innervated by the axillary nerve. Thus, the most accurate and classic sensory landmark for ulnar nerve integrity is the tip of the little finger, making option A correct.
2) The first dorsal web space is supplied by–
A) Median nerve
B) Radial nerve
C) Ulnar nerve
D) Musculocutaneous nerve
Answer: B) Radial nerve
Explanation: The radial nerve supplies dorsal thumb–index web space. Thus, B is correct.
3) Tip of the index finger is supplied by–
A) Radial nerve
B) Median nerve
C) Ulnar nerve
D) Axillary nerve
Answer: B) Median nerve
Explanation: Median nerve provides palmar digital branches to index fingertip. Thus, B is correct.
4) Sensation over medial 1½ fingers is due to–
A) Radial nerve
B) Median nerve
C) Ulnar nerve
D) Musculocutaneous nerve
Answer: C) Ulnar nerve
Explanation: Ulnar nerve supplies medial ring finger and little finger. Thus, C is correct.
5) Loss of sensation over dorsomedial hand suggests injury to–
A) Median nerve
B) Ulnar nerve
C) Radial nerve
D) Axillary nerve
Answer: B) Ulnar nerve
Explanation: Dorsal branch of ulnar nerve supplies medial dorsum. Thus, B is correct.
6) A patient with fracture of medial epicondyle may show sensory loss in–
A) Thumb
B) Middle finger
C) Little finger
D) First web space
Answer: C) Little finger
Explanation: Medial epicondyle injury affects ulnar nerve causing sensory loss in little finger. Thus, C is correct.
7) Which nerve supplies palmar aspect of ring finger (lateral half)?
A) Median nerve
B) Ulnar nerve
C) Radial nerve
D) Axillary nerve
Answer: A) Median nerve
Explanation: Median nerve supplies lateral half of ring finger. Thus, A is correct.
8) Dorsal sensory branch of ulnar nerve arises at–
A) Wrist
B) Upper arm
C) Mid forearm
D) Shoulder
Answer: C) Mid forearm
Explanation: The dorsal branch splits in distal forearm to supply dorsal digits. Thus, C is correct.
9) Loss of sensation in first web space indicates lesion of–
A) Median nerve
B) Radial nerve
C) Ulnar nerve
D) Musculocutaneous nerve
Answer: B) Radial nerve
Explanation: Radial nerve provides sensory to dorsal thumb–index web space. Thus, B is correct.
10) Loss of palmar sensation over thumb indicates damage to–
A) Median nerve
B) Radial nerve
C) Ulnar nerve
D) Spinal accessory nerve
Answer: A) Median nerve
Explanation: Median nerve supplies palmar thumb sensation. Thus, A is correct.
11) Which nerve supplies lateral cutaneous sensation of arm?
A) Axillary nerve
B) Ulnar nerve
C) Radial nerve
D) Median nerve
Answer: A) Axillary nerve
Explanation: The superior lateral cutaneous nerve of arm is a branch of axillary nerve. Thus, A is correct.
Chapter: Upper Limb Anatomy; Topic: Nerve Lesions of Hand; Subtopic: Ulnar Nerve Signs
Keyword Definitions:
Wartenberg’s Sign: Persistent abduction of little finger due to interossei weakness in ulnar nerve palsy.
Ulnar Nerve: Motor supply to interossei, medial lumbricals, adductor pollicis, and FPB deep head.
Intrinsic Plus Position: MCP flexion with IP extension produced by interossei & lumbricals.
Froment’s Sign: Flexion of thumb IP during pinch due to weak adductor pollicis.
Claw Hand: Hyperextension at MCP and flexion at IP joints due to ulnar nerve palsy.
1) Lead Question – 2016
What is Wartenberg's sign?
A) Inability to maintain intrinsic plus position
B) Inability to adduct small finger against the ring finger
C) Inability to grasp a book between the thumb and index finger
D) Inability to move the middle finger sideways
Answer: B) Inability to adduct small finger against the ring finger
Explanation: Wartenberg’s sign is characterized by persistent abduction of the little finger due to weakness of palmar interossei in ulnar nerve palsy. The patient is unable to adduct the little finger toward the ring finger. This occurs because the extensor digiti minimi is unopposed while the interossei cannot counteract it. This sign helps clinically differentiate ulnar nerve palsy from other nerve lesions. Therefore, the correct answer is B. It is commonly assessed during neurological evaluation of hand intrinsic muscle function.
2) Froment’s sign tests weakness of–
A) Opponens pollicis
B) Adductor pollicis
C) Abductor pollicis longus
D) EPL
Answer: B) Adductor pollicis
Explanation: Froment’s sign indicates ulnar nerve palsy causing weak adductor pollicis; patient flexes thumb IP to compensate. Thus, B is correct.
3) In ulnar nerve palsy, clawing is most pronounced in–
A) Index and middle fingers
B) Thumb
C) Ring and little fingers
D) All fingers equally
Answer: C) Ring and little fingers
Explanation: Medial lumbricals are ulnar-supplied; their loss leads to clawing of digits 4 and 5. Thus, C is correct.
4) A patient presents with difficulty crossing fingers. This suggests dysfunction of–
A) Lumbricals
B) Dorsal interossei
C) Thenar muscles
D) EPL
Answer: B) Dorsal interossei
Explanation: Dorsal interossei abduct fingers and help in crossing movements. Ulnar nerve injury weakens them. Thus, B is correct.
5) Ulnar nerve injury at the wrist spares which muscle?
A) Lumbrical 4
B) Interossei
C) Flexor digitorum profundus medial half
D) Adductor pollicis
Answer: C) Flexor digitorum profundus medial half
Explanation: FDP to ring/little fingers is supplied proximally; wrist injury spares it. Thus, C is correct.
6) The ‘ulnar paradox’ refers to–
A) Less clawing in proximal ulnar injury
B) More thumb involvement
C) Finger extension loss
D) Sensory sparing in digits
Answer: A) Less clawing in proximal ulnar injury
Explanation: Proximal injury weakens FDP, reducing clawing severity. Thus, A is correct.
7) Loss of adduction of fingers indicates paralysis of–
A) Palmar interossei
B) Dorsal interossei
C) Lumbricals 1&2
D) Thenar muscles
Answer: A) Palmar interossei
Explanation: Palmar interossei adduct fingers (PAD). Thus, A is correct.
8) Wartenberg’s sign is associated with lesions of–
A) Radial nerve
B) Ulnar nerve
C) Median nerve
D) Musculocutaneous nerve
Answer: B) Ulnar nerve
Explanation: Abduction of little finger is classic for ulnar palsy. Thus, B is correct.
9) A patient cannot perform intrinsic plus position. Likely muscle affected is–
A) Deltoid
B) Lumbricals
C) Brachioradialis
D) EPL
Answer: B) Lumbricals
Explanation: Lumbricals flex MCP and extend IP joints; their loss abolishes intrinsic plus. Thus, B is correct.
10) The deep branch of ulnar nerve supplies all except–
A) Interossei
B) Adductor pollicis
C) Lumbrical 1
D) FPB deep head
Answer: C) Lumbrical 1
Explanation: Lumbrical 1 is median-supplied; others are ulnar. Thus, C is correct.
11) Difficulty gripping paper between fingers occurs due to weakness of–
A) Palmar interossei
B) EPL
C) APL
D) FPL
Answer: A) Palmar interossei
Explanation: Paper grip test checks finger adduction; palmar interossei weakness indicates ulnar palsy. Thus, A is correct.
Chapter: Histology; Topic: Gastrointestinal Tract; Subtopic: Enteric Nervous System (Auerbach’s & Meissner’s Plexus)
Keyword Definitions:
Auerbach’s (Myenteric) Plexus: Neural plexus located between circular and longitudinal muscle layers; controls GI motility.
Meissner’s Plexus: Submucosal plexus controlling glandular secretion and local blood flow.
Enteric Nervous System: Intrinsic autonomic system regulating gastrointestinal functions independent of CNS input.
GI Motility: Coordinated contraction of smooth muscle mediated largely by myenteric plexus.
Muscularis Externa: GI layer containing inner circular and outer longitudinal muscle layers with Auerbach’s plexus between them.
1) Lead Question – 2016
Auerbach’s plexus is present in the–
A) Colon
B) Esophagus
C) Stomach
D) All of the above
Answer: D) All of the above
Explanation: Auerbach’s plexus (myenteric plexus) is present throughout the entire gastrointestinal tract, extending from the esophagus to the anal canal. It is located between the circular and longitudinal muscle layers of the muscularis externa. Its primary function is to regulate peristalsis and coordinated smooth muscle contraction. Because it is found in the esophagus, stomach, small intestine, and colon, the correct answer is D. Damage to this plexus, as in achalasia, significantly affects GI motility.
2) Auerbach’s plexus is located between–
A) Mucosa and submucosa
B) Serosa and adventitia
C) Circular and longitudinal muscle layers
D) Muscularis mucosae and mucosa
Answer: C) Circular and longitudinal muscle layers
Explanation: The myenteric plexus lies between the two layers of muscularis externa. Thus, C is correct.
3) Destruction of Auerbach’s plexus is characteristic of–
A) Hirschsprung disease
B) Achalasia
C) Crohn disease
D) Peptic ulcer
Answer: B) Achalasia
Explanation: Achalasia involves degeneration of myenteric plexus in esophagus, impairing peristalsis and LES relaxation. Thus, B is correct.
4) Which plexus primarily controls glandular secretion in GI tract?
A) Auerbach’s plexus
B) Meissner’s plexus
C) Subserosal plexus
D) Muscular plexus
Answer: B) Meissner’s plexus
Explanation: Meissner’s plexus lies in submucosa and regulates secretions and local blood flow. Thus, B is correct.
5) A patient with congenital aganglionic colon lacks which structure?
A) Only Meissner’s plexus
B) Only Auerbach’s plexus
C) Both Auerbach’s and Meissner’s plexus
D) Only vagal fibers
Answer: C) Both Auerbach’s and Meissner’s plexus
Explanation: Hirschsprung disease involves absence of both plexuses due to neural crest migration failure. Thus, C is correct.
6) Stomach motility is impaired in a lesion of–
A) Myenteric plexus
B) Submucosal plexus
C) Gastric glands
D) Parietal cells
Answer: A) Myenteric plexus
Explanation: Myenteric plexus orchestrates stomach peristalsis; damage reduces motility. Thus, A is correct.
7) Myenteric plexus receives input from which nerve?
A) Phrenic nerve
B) Vagus nerve
C) Accessory nerve
D) Facial nerve
Answer: B) Vagus nerve
Explanation: Parasympathetic innervation from the vagus enhances GI motility via myenteric plexus. Thus, B is correct.
8) Interstitial cells of Cajal serve as–
A) Immune cells
B) Pacemaker cells
C) Hormone-secreting cells
D) Fibroblasts
Answer: B) Pacemaker cells
Explanation: ICC generate slow waves regulating GI motility and interact with Auerbach’s plexus. Thus, B is correct.
9) In which layer of GI wall are Meissner’s and Auerbach’s plexuses respectively located?
A) Submucosa; muscularis externa
B) Muscularis mucosae; mucosa
C) Serosa; mucosa
D) Adventitia; submucosa
Answer: A) Submucosa; muscularis externa
Explanation: Meissner’s in submucosa, Auerbach’s between muscle layers. Thus, A is correct.
10) A man with long-standing achalasia shows dilation of esophagus. The underlying cause is loss of–
A) Parietal cells
B) Myenteric ganglion cells
C) Endocrine cells
D) Basal cells
Answer: B) Myenteric ganglion cells
Explanation: Loss of Auerbach’s plexus causes aperistalsis and dilation. Thus, B is correct.
11) Which GI segment has skeletal muscle but still contains Auerbach’s plexus?
A) Upper esophagus
B) Jejunum
C) Appendix
D) Rectum
Answer: A) Upper esophagus
Explanation: Upper esophagus contains skeletal muscle but retains enteric plexuses including Auerbach’s. Thus, A is correct.
Chapter: Neurophysiology; Topic: Peripheral Nerve Injury; Subtopic: Seddon Classification – Neuropraxia
KEYWORD DEFINITIONS
• Neuropraxia – Temporary conduction block without structural nerve damage
• Axonotmesis – Axonal damage with intact connective tissue
• Neurotmesis – Complete nerve disruption including connective tissue
• Endoneurium – Innermost protective nerve layer around individual axons
• Epineurium – Outermost connective tissue sheath of a peripheral nerve
Lead Question – 2015
1. Neuropraxia is?
A) Damage to axon
B) Damage to endoneurium
C) Damage to epineurium
D) No structural damage
Explanation:
Neuropraxia is the mildest form of nerve injury in Seddon’s classification. It involves a temporary block in nerve conduction without any structural damage to the axon or the surrounding connective tissue layers. Recovery is complete and usually rapid once the compressive or ischemic cause is removed. There is no Wallerian degeneration, distinguishing it from axonotmesis and neurotmesis. Therefore, the correct answer is No structural damage. Common causes include mild compression, pressure palsies, and transient ischemia.
2. Axonotmesis is characterized by:
A) No structural damage
B) Axonal damage with intact connective tissue
C) Complete nerve transection
D) Epineurial rupture
Explanation:
Axonotmesis involves disruption of the axon with intact endoneurial and epineurial sheaths, allowing guided regeneration. Wallerian degeneration occurs distal to injury. Neuropraxia has no structural injury, while neurotmesis involves complete nerve rupture. Thus, the correct answer is Axonal damage with intact connective tissue. Recovery may take months depending on regeneration speed.
3. Neurotmesis leads to:
A) Full recovery without intervention
B) Complete nerve disruption
C) Temporary conduction block
D) No Wallerian degeneration
Explanation:
Neurotmesis is the most severe nerve injury with complete disruption of axon and supporting structures. Wallerian degeneration occurs distally, and spontaneous recovery is unlikely without surgical repair. Neuropraxia causes conduction block only, and axonotmesis retains connective tissue scaffolding. Therefore, the correct answer is Complete nerve disruption. Prognosis is poor without repair.
4. A patient with wrist drop after sleeping on arm overnight likely has:
A) Neurotmesis
B) Axonotmesis
C) Neuropraxia
D) Endoneurial tear
Explanation:
Prolonged external compression of the radial nerve, such as “Saturday night palsy,” causes transient conduction block consistent with neuropraxia. Recovery occurs in days to weeks as no structural damage exists. Neurotmesis or axonotmesis would produce prolonged deficits. Thus, the correct answer is Neuropraxia. This highlights the reversible nature of mild compressive injuries.
5. Wallerian degeneration occurs in which type of nerve injury?
A) Neuropraxia
B) Axonotmesis
C) None
D) Neuropraxia and neurotmesis both
Explanation:
Wallerian degeneration occurs when axons are damaged, as in axonotmesis and neurotmesis. It does not occur in neuropraxia because the axon remains intact. Therefore, the correct answer is Axonotmesis. This process clears distal debris and allows regeneration guided by connective tissue sheaths.
6. Which protective layer surrounds individual nerve fibers?
A) Epineurium
B) Perineurium
C) Endoneurium
D) Sarcolemma
Explanation:
The endoneurium surrounds individual axons within a nerve. The perineurium encloses fascicles, and the epineurium wraps the entire nerve. Sarcolemma surrounds muscle fibers, not nerves. Thus, the correct answer is Endoneurium. These layers are crucial for structural integrity and regeneration pathways.
7. A mild ulnar nerve injury with full recovery in 2 weeks indicates:
A) Neurotmesis
B) Partial transection
C) Neuropraxia
D) Axonotmesis
Explanation:
Rapid and complete recovery within a short period indicates neuropraxia, meaning conduction block without structural injury. Axonotmesis requires axonal regeneration and takes months, while neurotmesis involves severe disruption. Therefore, the correct answer is Neuropraxia. Compression injuries at the elbow often present this way.
8. In neuropraxia, motor and sensory loss occurs because:
A) Axon is destroyed
B) Myelin compression blocks conduction
C) Endoneurium ruptures
D) Nerve is transected
Explanation:
Neuropraxia is caused by localized myelin compression or ischemia that temporarily blocks nerve conduction without damaging axons. Thus, motor and sensory deficits occur but recover fully. Endoneurium rupture and nerve transection correspond to more severe injuries. The correct answer is Myelin compression blocks conduction. This explains the reversible nature of neuropraxia.
9. Best investigation to assess peripheral nerve injury severity:
A) EEG
B) Nerve conduction study
C) PET scan
D) X-ray
Explanation:
Nerve conduction studies differentiate between neuropraxia (conduction block), axonotmesis (reduced amplitude), and neurotmesis (absent conduction). EEG, PET, and X-ray are unsuitable for peripheral nerve assessment. Therefore, the correct answer is Nerve conduction study. It provides functional insight into nerve integrity.
10. Recovery in axonotmesis occurs due to:
A) Axonal regeneration along intact sheaths
B) Immediate reconnection
C) CNS plasticity
D) Increased myelination only
Explanation:
In axonotmesis, axonal regrowth occurs along preserved endoneurial tubes at ~1–3 mm/day. Immediate reconnection does not occur, and CNS plasticity is limited to central pathways. Myelination alone cannot restore lost axons. Therefore, correct answer is Axonal regeneration along intact sheaths. This anatomical preservation allows partial to full recovery.
11. A clean laceration of the median nerve with complete loss of function suggests:
A) Neuropraxia
B) Axonotmesis
C) Neurotmesis
D) Branch block only
Explanation:
A complete nerve laceration disrupts axons and connective layers, representing neurotmesis. Spontaneous recovery is unlikely without surgical repair. Neuropraxia shows no structural damage, and axonotmesis preserves connective sheaths. Therefore, the correct answer is Neurotmesis. Early repair improves outcomes.
Chapter: Autonomic Nervous System; Topic: Sympathetic vs Parasympathetic Distribution; Subtopic: Organs with Only Sympathetic Innervation
KEYWORD DEFINITIONS
• Sympathetic system – Mediates fight-or-flight responses
• Parasympathetic system – Mediates rest-and-digest functions
• Skin – Receives sympathetic supply only (sweat glands, piloerector muscles, blood vessels)
• Sweat glands – Activated by cholinergic sympathetic fibers
• Vasoconstriction – Sympathetic control of cutaneous vessels
Lead Question – 2015
1. Which of the following has direct innervation from sympathetic system but no parasympathetic supply?
A) Heart
B) Intestine
C) Skin
D) None
Explanation:
The skin receives only sympathetic innervation, which controls sweat glands, vasoconstriction, and piloerection. Parasympathetic fibers do not supply the skin. In contrast, the heart and intestines have both sympathetic and parasympathetic innervation. Thus, structures like cutaneous blood vessels, sweat glands, and hair follicles are under exclusive sympathetic control. Therefore, the correct answer is Skin. This unique distribution enables rapid thermoregulatory and stress-related responses via sympathetic activation.
2. Sweat glands are innervated by:
A) Adrenergic sympathetic fibers
B) Cholinergic sympathetic fibers
C) Parasympathetic fibers
D) Sensory fibers
Explanation:
Sweat glands are supplied by an exceptional type of sympathetic postganglionic fiber that is cholinergic and acts on muscarinic receptors. Parasympathetic fibers do not innervate sweat glands. Sensory fibers do not control sweating. Therefore, the correct answer is Cholinergic sympathetic fibers. This mechanism supports thermoregulation during heat stress.
3. Which structure lacks parasympathetic innervation?
A) Lacrimal gland
B) Bronchi
C) Skin blood vessels
D) Salivary glands
Explanation:
Blood vessels of the skin do not receive parasympathetic supply and are controlled only by sympathetic vasoconstrictor fibers. Lacrimal, bronchial, and salivary glands are all supplied by parasympathetic fibers. Therefore, the correct answer is Skin blood vessels. This exclusive sympathetic control helps regulate body temperature and peripheral resistance.
4. A patient with sympathetic overactivity presents with excessive sweating. The hyperactivity is due to stimulation of:
A) Adrenergic α receptors
B) Muscarinic receptors
C) GABA receptors
D) Dopamine receptors
Explanation:
Sweating is mediated by cholinergic sympathetic fibers that act on muscarinic receptors. Adrenergic fibers regulate vasomotor tone but not sweating. GABA and dopamine receptors have no role in sweat gland activation. Hence, the correct answer is Muscarinic receptors. This makes sweating an unusual sympathetic cholinergic response.
5. Piloerection (goosebumps) occurs due to activation of:
A) Parasympathetic fibers
B) Sympathetic fibers
C) Somatic motor fibers
D) Enteric nervous system
Explanation:
Piloerection is produced by contraction of arrector pili muscles, which are innervated exclusively by sympathetic fibers. Parasympathetic fibers do not innervate skin structures. Somatic motor fibers do not reach these muscles. Therefore, the correct answer is Sympathetic fibers. This reaction is associated with emotional stress and cold exposure.
6. Which organ receives both sympathetic and parasympathetic supply?
A) Sweat glands
B) Blood vessels of skin
C) Heart
D) Hair follicles
Explanation:
The heart receives both sympathetic and parasympathetic innervation, allowing dual modulation of rate and contractility. Sweat glands, cutaneous vessels, and hair follicles receive only sympathetic supply. Therefore, the correct answer is Heart. This dual innervation provides balance between fight-or-flight and rest-and-digest states.
7. A comatose patient has warm, flushed skin due to loss of sympathetic tone. Which vessel type is affected?
A) Coronary arteries
B) Cutaneous blood vessels
C) Renal blood vessels
D) Pulmonary veins
Explanation:
Cutaneous blood vessels are exclusively sympathetically innervated. Loss of sympathetic tone results in vasodilation, causing warm and flushed skin. Coronary, renal, and pulmonary vessels are regulated differently and not under exclusive sympathetic control. Thus, the correct answer is Cutaneous blood vessels. This illustrates the importance of sympathetic activity in thermoregulation.
8. Which neurotransmitter is released by sympathetic fibers supplying sweat glands?
A) Norepinephrine
B) Acetylcholine
C) Epinephrine
D) Dopamine
Explanation:
Sweat glands receive cholinergic sympathetic fibers that release acetylcholine at muscarinic receptors. This is a major exception in sympathetic physiology. Norepinephrine and epinephrine are involved in other sympathetic functions but not sweating. Therefore, the correct answer is Acetylcholine. This explains paradoxical sweating even when adrenergic function is impaired.
9. Which of the following tissues has purely sympathetic vasoconstrictor control?
A) Brain
B) Heart
C) Skin
D) Liver
Explanation:
Skin blood vessels receive only sympathetic vasoconstrictor innervation with no parasympathetic influence. Brain and heart have tightly regulated local mechanisms, and liver vasculature is influenced by multiple autonomic inputs. Therefore, the correct answer is Skin. This allows rapid redistribution of blood during shock and stress.
10. During fear, skin becomes pale due to:
A) Parasympathetic activation
B) Sympathetic vasoconstriction
C) Muscarinic inhibition
D) Sweat gland paralysis
Explanation:
Sympathetic stimulation causes vasoconstriction of cutaneous vessels, reducing blood flow and producing pallor. Parasympathetic activation produces no effect on skin. Muscarinic inhibition and sweat paralysis do not explain the color change. Therefore, the correct answer is Sympathetic vasoconstriction. This physiological response prioritizes blood flow to essential organs.
11. The sympathetic nervous system increases heat loss through which mechanism?
A) Parasympathetic vasodilation
B) Sympathetic vasodilation in skin
C) Sympathetic sweating
D) Enteric reflexes
Explanation:
Sympathetic stimulation of sweat glands enhances evaporative heat loss. Vasodilation in skin is primarily a passive loss of sympathetic tone, not parasympathetic action. Enteric reflexes are unrelated. Thus, the correct answer is Sympathetic sweating. Sweat evaporation is the most effective cooling mechanism in humans.
Chapter: Autonomic Nervous System; Topic: Sympathetic Division; Subtopic: Neurotransmitter Secretion – Adrenal Medulla
KEYWORD DEFINITIONS
• Sympathetic system – Part of ANS mediating fight-or-flight responses
• Adrenal medulla – Modified sympathetic ganglion secreting epinephrine & norepinephrine
• Chromaffin cells – Cells in adrenal medulla that act like postganglionic neurons
• Catecholamines – Stress hormones (epinephrine, norepinephrine)
• Preganglionic fibers – Cholinergic fibers stimulating adrenal medulla
Lead Question – 2015
1. Part of sympathetic system which secrete chemical transmitter?
A) Cardiac ganglion
B) Cervical sympathetic chain
C) Adrenal medulla
D) Thoracic sympathetic chain
Explanation:
The adrenal medulla acts as a modified sympathetic ganglion derived from neural crest cells. Unlike other sympathetic ganglia, its chromaffin cells directly secrete catecholamines (mainly epinephrine and norepinephrine) into the bloodstream when stimulated by preganglionic sympathetic fibers. Cardiac, cervical, and thoracic sympathetic chains do not release hormones; they release neurotransmitters only at synapses. Therefore, the correct answer is Adrenal medulla. This endocrine function allows rapid systemic sympathetic activation during stress.
2. The adrenal medulla primarily releases:
A) Acetylcholine
B) Epinephrine
C) Dopamine
D) Serotonin
Explanation:
The adrenal medulla secretes mainly epinephrine (≈80%) and a smaller amount of norepinephrine (≈20%) in response to sympathetic stimulation. These catecholamines amplify fight-or-flight responses. Acetylcholine is the neurotransmitter of preganglionic sympathetic fibers, dopamine is a precursor, and serotonin is unrelated. Thus, the correct answer is Epinephrine. This hormonal release supports rapid cardiovascular and metabolic adjustments.
3. Chromaffin cells of adrenal medulla are analogous to:
A) Preganglionic neurons
B) Postganglionic sympathetic neurons
C) Sensory neurons
D) Interneurons
Explanation:
Chromaffin cells function as modified postganglionic sympathetic neurons but release catecholamines into the bloodstream instead of synapses. They are stimulated by acetylcholine from preganglionic fibers. Sensory neurons and interneurons have no such endocrine role. Therefore, the correct answer is Postganglionic sympathetic neurons. Their hormonal output provides widespread sympathetic activation.
4. A patient with pheochromocytoma has elevated blood catecholamines due to a tumor in the:
A) Adrenal cortex
B) Adrenal medulla
C) Pancreas
D) Thyroid
Explanation:
Pheochromocytoma originates from chromaffin cells of the adrenal medulla, causing excess secretion of epinephrine and norepinephrine. The adrenal cortex secretes steroid hormones, pancreas secretes insulin/glucagon, and thyroid produces T3/T4. Thus, the correct answer is Adrenal medulla. Symptoms include episodic hypertension, sweating, and tachycardia.
5. Which receptor mediates epinephrine-induced vasodilation in skeletal muscle?
A) α1 receptor
B) β2 receptor
C) β1 receptor
D) M3 receptor
Explanation:
Epinephrine activates β2 receptors in skeletal muscle vessels, causing vasodilation and enhancing blood flow during stress. α1 receptors cause vasoconstriction, β1 receptors act on the heart, and M3 receptors mediate parasympathetic effects. Therefore, the correct answer is β2 receptor. This selective dilation supports increased muscular activity.
6. Preganglionic sympathetic fibers release:
A) Norepinephrine
B) Acetylcholine
C) Dopamine
D) Epinephrine
Explanation:
All preganglionic sympathetic fibers are cholinergic, releasing acetylcholine at nicotinic receptors. Norepinephrine is mostly released by postganglionic fibers except in sweat glands. Dopamine and epinephrine are not synaptic transmitters here. Thus, the correct answer is Acetylcholine. This cholinergic activation includes stimulation of adrenal medulla.
7. A patient with sympathetic overactivity shows tachycardia and sweating. Sweating is mediated by:
A) Adrenergic fibers
B) Cholinergic sympathetic fibers
C) Parasympathetic fibers
D) Hormonal epinephrine only
Explanation:
Sweat glands are an exception in sympathetic system—they are innervated by cholinergic sympathetic postganglionic fibers acting on muscarinic receptors. Most other sympathetic targets are adrenergic. Parasympathetic pathways do not innervate sweat glands. Therefore, the correct answer is Cholinergic sympathetic fibers. This arrangement helps thermoregulation during stress.
8. Which enzyme is necessary for conversion of norepinephrine to epinephrine in adrenal medulla?
A) MAO
B) COMT
C) PNMT
D) Tyrosine hydroxylase
Explanation:
Phenylethanolamine N-methyltransferase (PNMT) converts norepinephrine to epinephrine and is highly expressed in adrenal medulla under cortisol influence. MAO and COMT degrade catecholamines; tyrosine hydroxylase is required earlier in synthesis. Thus, the correct answer is PNMT. This enzyme drives high epinephrine output.
9. Sympathetic stimulation increases blood glucose by:
A) Decreasing glucagon
B) Increasing glycogenolysis
C) Blocking lipolysis
D) Increasing insulin secretion
Explanation:
Catecholamines from adrenal medulla stimulate β receptors to increase glycogenolysis and lipolysis, raising blood glucose during stress. Insulin secretion decreases via α2 receptors. Thus, the correct answer is Increasing glycogenolysis. This ensures fuel availability for fight-or-flight activity.
10. Loss of adrenal medulla function leads to impaired response to:
A) Standing from sitting
B) Acute stress
C) Digestion
D) Vision adaptation
Explanation:
Without adrenal medullary catecholamines, the body cannot mount a full fight-or-flight response, reducing ability to handle acute stress. Standing reflex involves noradrenergic neurons, digestion is parasympathetic, and vision adaptation is unrelated. Therefore, the correct answer is Acute stress. Patients may show poor tolerance to sudden emergencies.
11. Adrenal medulla is embryologically derived from:
A) Endoderm
B) Mesoderm
C) Neural crest cells
D) Neural tube
Explanation:
Chromaffin cells of adrenal medulla originate from neural crest cells, similar to sympathetic postganglionic neurons. Cortex comes from mesoderm. Neural tube forms CNS neurons. Therefore, the correct answer is Neural crest cells. This explains its similarity to sympathetic ganglia.
Chapter: Neurophysiology; Topic: Brainstem Reflexes; Subtopic: Oculocephalic Reflex (Doll’s Eye Reflex)
KEYWORD DEFINITIONS
• Doll’s eye reflex – Brainstem reflex where eyes move opposite to head movement
• Oculocephalic reflex – Assesses integrity of midbrain and pons
• Brainstem function – Essential for basic reflexes and consciousness
• Comatose patient testing – Used when voluntary eye movement cannot be assessed
• Vestibulo-ocular reflex (VOR) – Stabilizes gaze during head movement
Lead Question – 2015
1. Doll's eye reflex is used in?
A) Hemiplegic
B) Paraplegic
C) Unconscious patient
D) Cerebral palsy
Explanation:
The Doll’s eye reflex (oculocephalic reflex) is used to assess brainstem integrity in comatose or unconscious patients. When the head is rapidly rotated, intact brainstem function causes the eyes to move in the opposite direction, indicating preserved vestibulo-ocular pathways. It is not used for hemiplegia, paraplegia, or cerebral palsy assessment. Absence of this reflex suggests severe brainstem dysfunction. Therefore, the correct answer is Unconscious patient. This reflex must not be tested in suspected cervical spine injury.
2. Doll’s eye reflex helps assess which structure?
A) Cerebellum
B) Brainstem
C) Basal ganglia
D) Spinal cord
Explanation:
The oculocephalic (Doll’s eye) reflex evaluates brainstem integrity, specifically the vestibular nuclei, pons, and midbrain centers that coordinate eye movements. Cerebellum regulates coordination, basal ganglia regulate movement initiation, and spinal cord reflexes are unrelated. Thus, the correct answer is Brainstem. The presence or absence of this reflex assists in coma evaluation.
3. A comatose patient shows absent Doll’s eye reflex. This suggests damage to:
A) Cortical motor areas
B) Brainstem gaze centers
C) Cerebellar hemispheres
D) Spinal interneurons
Explanation:
Absent Doll’s eye reflex indicates injury to the brainstem gaze centers including pontine and midbrain nuclei. Cortical damage does not abolish this reflex, nor do cerebellar or spinal injuries. Therefore, the correct answer is Brainstem gaze centers. This finding may suggest poor neurological prognosis if persistent.
4. The physiological basis of Doll's eye reflex is:
A) Spinocerebellar pathway activation
B) Vestibulo-ocular reflex
C) Corticospinal tract activity
D) Reticular formation inhibition
Explanation:
The Doll’s eye reflex is generated by the vestibulo-ocular reflex (VOR), where vestibular input causes the eyes to move opposite to head movement to maintain visual fixation. Corticospinal and reticular pathways are not directly involved. Thus, the correct answer is Vestibulo-ocular reflex. It is fundamental for gaze stabilization even without vision.
5. In a cervical spine injury patient, Doll’s eye reflex should:
A) Always be tested
B) Never be tested
C) Be tested after 24 hours
D) Be tested only during intubation
Explanation:
This reflex requires rapid passive head movement. In suspected cervical spine injury, such movement may worsen spinal damage; therefore, it must not be tested. The correct answer is Never be tested. Safer alternatives include caloric testing for assessing brainstem function.
6. Cold caloric testing stimulates which canal to mimic Doll’s eye reflex?
A) Posterior semicircular canal
B) Horizontal semicircular canal
C) Anterior semicircular canal
D) Otolith organs
Explanation:
Cold caloric testing primarily activates the horizontal semicircular canal to generate vestibulo-ocular responses similar to those seen in Doll’s eye reflex. Posterior and anterior canals are not targeted by standard caloric testing. Thus, the answer is Horizontal semicircular canal. It is used when passive head rotation is unsafe.
7. In a conscious patient, lack of Doll’s eye reflex means:
A) Normal finding
B) Brainstem failure
C) Spinal cord lesion
D) Cerebellar injury
Explanation:
In conscious individuals, Doll’s eye reflex is normally absent because voluntary eye fixation suppresses VOR. Therefore, absence is normal in awake patients. Brainstem failure is only implied when absent in comatose patients. Thus, correct answer is Normal finding. Conscious fixation overrides reflexive eye movements.
8. Active eye movement in the same direction as head movement in a comatose patient indicates:
A) Intact reflex
B) Absent brainstem reflex
C) Spinal cord injury
D) Upper motor neuron lesion
Explanation:
If a comatose patient’s eyes move with the head rather than opposite, the Doll’s eye reflex is absent, indicating impaired brainstem function. Spinal lesions or UMN lesions do not affect this reflex. Therefore, the correct answer is Absent brainstem reflex. This finding may indicate severe neurological compromise.
9. Doll’s eye reflex is mediated by which cranial nerves?
A) II and III
B) III, IV, and VI
C) V and VII
D) IX and X
Explanation:
The reflex involves extraocular movements coordinated by cranial nerves III (oculomotor), IV (trochlear), and VI (abducens). Vestibular input arises from cranial nerve VIII. Thus, the correct answer is III, IV, and VI. The coordinated response signifies intact pontine and midbrain centers.
10. A patient with metabolic coma and intact Doll's eye reflex indicates:
A) Preserved brainstem function
B) Brain death
C) Spinal shock
D) Cortical blindness
Explanation:
Presence of Doll’s eye reflex in coma signifies an intact brainstem. Brain death criteria require its absence. Spinal shock and cortical blindness do not affect this reflex. Therefore, the correct answer is Preserved brainstem function. It helps differentiate metabolic coma from structural brainstem lesions.
11. Doll’s eye reflex becomes absent in brain death because of failure of:
A) Cortical integration
B) Pontine and midbrain nuclei
C) Spinal interneurons
D) Cerebellar pathways
Explanation:
Brain death involves complete and irreversible loss of brainstem reflexes, including those mediated by pontine and midbrain nuclei responsible for the oculocephalic reflex. Therefore, the correct answer is Pontine and midbrain nuclei. Absence of this reflex is a core part of brain death testing.
Chapter: Neurophysiology; Topic: Motor System; Subtopic: Cortical Motor Areas – Suppressor Strip
KEYWORD DEFINITIONS
• Suppressor strip – Area anterior to precentral gyrus that inhibits stretch reflex
• Stretch reflex – Monosynaptic reflex causing muscle contraction when stretched
• Motor cortex – Region responsible for voluntary movement
• Upper motor neurons – Neurons controlling spinal motor circuits
• Muscle tone – Resistance to passive stretch regulated by reflex pathways
Lead Question – 2015
1. Suppressor Strip on anterior edge of pre-central gyrus has following function?
A) Increase extensor tone
B) Pain perception
C) Inhibition of stretch reflex
D) Voluntary movement
Explanation:
The suppressor strip lies just anterior to the primary motor cortex in the frontal lobe. Its major role is inhibition of the stretch reflex and modulation of muscle tone. By reducing excessive reflex activity, it prevents hyperreflexia and spasticity. Pain perception is mediated by somatosensory cortex, extensor tone by vestibulospinal pathways, and voluntary movement by the primary motor cortex itself. Therefore, the correct answer is Inhibition of stretch reflex. Lesions in this region cause exaggerated reflexes due to loss of inhibitory control.
2. The primary motor cortex corresponds to which Brodmann area?
A) Area 4
B) Area 6
C) Area 3
D) Area 22
Explanation:
The primary motor cortex, responsible for initiating voluntary movements, corresponds to Brodmann area 4. Area 6 represents the premotor/supplementary motor regions, area 3 belongs to the somatosensory cortex, and area 22 is Wernicke’s area. Therefore, the correct answer is Area 4. Damage to area 4 produces contralateral weakness and loss of fine motor control.
3. A patient with a lesion in the supplementary motor area will likely have difficulty with:
A) Planning complex movements
B) Visual processing
C) Hearing
D) Pain localization
Explanation:
The supplementary motor area (SMA) coordinates complex and bimanual movements and internally driven motor sequences. Damage produces difficulty initiating planned movements (motor apraxia). Visual processing occurs in occipital cortex, hearing in temporal cortex, and pain localization in somatosensory cortex. Thus, the correct answer is Planning complex movements. SMA lesions often cause difficulty with coordinated motor tasks.
4. Hyperreflexia following stroke is due to loss of:
A) Spinal cord reflex arcs
B) Cortical inhibition
C) Muscle fibers
D) Peripheral nerve conduction
Explanation:
Upper motor neurons, including suppressor areas, provide inhibitory control over spinal reflexes. When cortical inhibition is lost after stroke, reflexes become exaggerated. Spinal reflex arcs remain intact, muscle fibers are unaffected initially, and peripheral conduction is normal. Therefore, the correct answer is Cortical inhibition. This mechanism explains spasticity in UMN lesions.
5. The corticospinal tract decussates at the:
A) Pons
B) Midbrain
C) Medullary pyramids
D) Spinal cord dorsal horn
Explanation:
Nearly 80–90% of corticospinal fibers cross at the pyramidal decussation in the medulla to form the lateral corticospinal tract. The pons and midbrain transmit but do not primarily decussate these fibers. Dorsal horn contains sensory, not motor, pathways. Therefore, the correct answer is Medullary pyramids. This crossing explains contralateral motor deficits.
6. A patient presents with spasticity and exaggerated reflexes. Which lesion is most likely?
A) Lower motor neuron lesion
B) Upper motor neuron lesion
C) Neuromuscular junction defect
D) Myopathy
Explanation:
Spasticity and hyperreflexia occur due to upper motor neuron (UMN) lesions, which remove inhibitory influence on spinal reflexes, including suppressor strip control. LMN lesions cause flaccidity, neuromuscular junction defects cause fatigability, and myopathies cause proximal weakness. The correct answer is Upper motor neuron lesion. This presentation is typical of stroke or spinal cord injury.
7. Premotor cortex is responsible for:
A) Simple reflex movements
B) Planning movements based on external cues
C) Vision
D) Touch perception
Explanation:
The premotor cortex (Brodmann area 6) organizes movement patterns triggered by external stimuli such as visual or auditory cues. Simple reflexes occur in the spinal cord, vision in the occipital lobe, and touch in the somatosensory cortex. Therefore, the correct answer is Planning movements based on external cues. It works closely with SMA to coordinate motor output.
8. Which neurotransmitter is primarily used by corticospinal neurons?
A) GABA
B) Dopamine
C) Glutamate
D) Glycine
Explanation:
Corticospinal neurons are excitatory and use glutamate as their primary neurotransmitter. GABA and glycine are inhibitory, while dopamine is modulatory in basal ganglia circuits. Thus, the correct answer is Glutamate. Excitatory drive from motor cortex facilitates precise motor control.
9. A tumor compressing the precentral gyrus would cause:
A) Sensory loss
B) Motor weakness
C) Memory impairment
D) Visual field defect
Explanation:
The precentral gyrus contains the primary motor cortex; its compression causes contralateral weakness or paralysis. Sensory loss occurs with postcentral gyrus lesions, memory impairment involves hippocampus, and visual defects arise from occipital lobe lesions. Therefore, the correct answer is Motor weakness. Motor maps in this region allow localization of deficits.
10. The stretch reflex involves which afferent fiber type?
A) Type Ia fibers
B) C fibers
C) A-delta fibers
D) Type II taste fibers
Explanation:
Type Ia fibers from muscle spindles detect rapid stretch and initiate the monosynaptic stretch reflex. C fibers carry pain, A-delta fibers carry fast pain and temperature, and taste fibers have no role. Thus, the correct answer is Type Ia fibers. These inputs are modulated by cortical suppressor systems to maintain tone.
11. Damage to suppressor strip leads to:
A) Loss of pain sensation
B) Flaccid paralysis
C) Increased stretch reflex activity
D) Loss of vision
Explanation:
Suppressor strip damage removes inhibitory control over spinal reflexes, resulting in hyperreflexia and increased muscle tone. Pain sensation involves somatosensory cortex, flaccid paralysis reflects LMN lesions, and vision relates to occipital cortex. Therefore, the correct answer is Increased stretch reflex activity. This is a defining feature of upper motor neuron dysfunction.
Chapter: Gastrointestinal Physiology; Topic: Mineral Absorption; Subtopic: Factors Affecting Calcium Absorption
KEYWORD DEFINITIONS
• Calcium absorption – Uptake of calcium from intestine, mainly duodenum
• Phytates – Compounds in cereals that bind calcium and inhibit absorption
• Oxalates – Plant compounds forming insoluble calcium salts
• Vitamin D – Enhances calcium absorption
• Acidic pH – Improves calcium solubility and absorption
Lead Question – 2015
1. Calcium absorption is hampered by:
A) Protein
B) Lactose
C) Acid
D) Phytates
Explanation:
Phytates, found in cereals, legumes, and whole grains, bind calcium and form insoluble complexes that cannot be absorbed in the intestine. Protein and lactose actually enhance calcium absorption, while acidic pH increases calcium solubility. Therefore, the factor that hampers calcium absorption is Phytates. Individuals consuming diets high in unfermented cereals may develop calcium deficiency due to this inhibitory effect. Removing phytates through fermentation or soaking improves absorption significantly.
2. Which vitamin enhances intestinal calcium absorption?
A) Vitamin A
B) Vitamin D
C) Vitamin K
D) Vitamin E
Explanation:
Vitamin D increases the synthesis of calcium-binding proteins in the intestinal mucosa, significantly enhancing active calcium absorption. Vitamins A, K, and E do not play a primary role in calcium handling. Therefore, the correct answer is Vitamin D. Deficiency in vitamin D leads to reduced calcium uptake and conditions like rickets and osteomalacia.
3. A patient with celiac disease has low calcium levels. The cause is:
A) Excess protein intake
B) Increased phytate absorption
C) Damage to intestinal villi
D) Excess bile salts
Explanation:
Celiac disease causes villous atrophy, reducing surface area for nutrient absorption, including calcium. Protein does not impair calcium uptake, and bile salts are not directly involved. Phytate absorption is not increased in celiac disease. Therefore, the correct answer is Damage to intestinal villi. This malabsorption may manifest as low bone mineral density.
4. Calcium absorption occurs maximally in:
A) Stomach
B) Duodenum
C) Colon
D) Rectum
Explanation:
The duodenum has an acidic environment and abundant calcium transport proteins, making it the primary site for calcium absorption. The stomach has minimal absorption, while colon and rectum have little role. Thus, the correct answer is Duodenum. The duodenum’s short transit time and pH facilitate efficient uptake.
5. High dietary oxalates inhibit calcium absorption by:
A) Increasing vitamin D degradation
B) Binding calcium in intestine
C) Stimulating calcium efflux
D) Increasing renal calcium loss
Explanation:
Oxalates form insoluble calcium oxalate salts in the intestine, preventing calcium absorption. They do not affect vitamin D levels or calcium efflux. Renal calcium loss is unaffected. Thus, the correct answer is Binding calcium in intestine. Spinach and certain nuts have high oxalate content, reducing usable dietary calcium.
6. A patient with lactose intolerance may have decreased calcium absorption because:
A) Lactose enhances calcium absorption
B) Calcium is digested by lactase
C) Fat malabsorption causes calcium loss
D) Lactase stimulates vitamin D production
Explanation:
Lactose increases calcium solubility and aids its uptake. In lactose intolerance, individuals avoid dairy products, reducing calcium intake and losing lactose’s absorption-enhancing effect. Calcium is not digested by lactase, and lactase does not influence vitamin D. Therefore, the correct answer is Lactose enhances calcium absorption. This is important in dietary counseling.
7. Which hormone increases renal calcium reabsorption?
A) Calcitonin
B) Parathyroid hormone (PTH)
C) Insulin
D) Glucagon
Explanation:
PTH increases renal tubular calcium reabsorption, reduces phosphate reabsorption, and stimulates vitamin D activation. Calcitonin decreases serum calcium, while insulin and glucagon do not regulate calcium homeostasis. Therefore, the correct answer is PTH. PTH is essential for maintaining serum calcium during hypocalcemic states.
8. Phosphate binds calcium and reduces its absorption. This is most pronounced in intake of:
A) Meat
B) Whole grains
C) Fruits
D) Fats
Explanation:
Whole grains contain high levels of phytates and phosphates, which bind calcium and reduce absorption. Meat does not contain significant phytates. Fruits and fats have minimal effect on calcium binding. Thus, the correct answer is Whole grains. Dietary modification helps improve calcium bioavailability.
9. Which condition increases calcium absorption?
A) Achlorhydria
B) Low vitamin D
C) Acidic gastric pH
D) High fiber intake
Explanation:
Acidic pH increases calcium solubility, enhancing absorption. Achlorhydria reduces acid and decreases uptake. Low vitamin D also reduces absorption, and high fiber (with phytates) may bind calcium. Therefore, the correct answer is Acidic gastric pH. Elderly individuals with reduced acid secretion often have poorer calcium absorption.
10. A 55-year-old woman with osteoporosis is advised calcium supplements with meals. Reason?
A) Calcium binds fat
B) Gastric acid improves calcium solubility
C) Calcium is digested better with proteins
D) Calcium requires bile for absorption
Explanation:
Calcium dissolves better in an acidic environment created during meals, improving its absorption. Fat-binding, protein digestion, and bile are unrelated. Therefore, the correct answer is Gastric acid improves calcium solubility. Co-administration with meals maximizes uptake.
11. A patient has chronic antacid use. This reduces calcium absorption because:
A) Antacids convert calcium to free ionized form
B) Antacids decrease gastric acidity
C) Antacids degrade vitamin D
D) Antacids increase phytate levels
Explanation:
Antacids neutralize gastric acid, reducing calcium solubility and intestinal absorption. They do not degrade vitamin D or influence phytates. Thus, the correct answer is Antacids decrease gastric acidity. Long-term use can contribute to hypocalcemia and bone loss.
Chapter: Neurophysiology; Topic: Sleep Physiology; Subtopic: NREM Sleep Disorders – Parasomnias
KEYWORD DEFINITIONS
• NREM sleep – Non-rapid eye movement sleep with stages 1–4
• Slow-wave sleep – Deep sleep (Stages 3–4 NREM), associated with parasomnias
• Parasomnias – Abnormal behaviors during sleep (e.g., sleepwalking)
• REM sleep – Dreaming sleep with muscle atonia
• Arousal disorders – Occur during partial awakening from deep NREM sleep
Lead Question – 2015
1. Sleep walking is seen in which stage of sleep?
A) REM
B) Stage 1–2 NREM
C) Stage 2–3 NREM
D) Stage 3–4 NREM
Explanation:
Sleepwalking (somnambulism) is a classic NREM parasomnia occurring predominantly during deep slow-wave sleep, which corresponds to Stages 3–4 NREM. This stage is characterized by delta waves, reduced consciousness, and partial arousal phenomena. Because REM sleep involves muscle atonia, sleepwalking cannot occur during REM. Lighter NREM stages (1–2) do not typically generate complex motor behaviors. Therefore, the correct answer is Stage 3–4 NREM. It is most common in children and may be triggered by sleep deprivation, stress, or fever.
2. REM sleep is characterized by:
A) Muscle atonia
B) Increased muscle tone
C) Sleepwalking episodes
D) Violent movements
Explanation:
REM sleep features rapid eye movements, vivid dreaming, and generalized skeletal muscle atonia due to brainstem inhibition. This prevents acting out dreams. Increased muscle tone is typical of wakefulness, not REM. Sleepwalking and violent behaviors occur in NREM parasomnias, not REM. Thus, the correct answer is Muscle atonia. The atonia protects the sleeper from injury during dreams.
3. A 10-year-old child exhibits night terrors. These episodes occur most commonly in:
A) REM sleep
B) Stage 3–4 NREM sleep
C) Stage 1 sleep
D) Wakefulness
Explanation:
Night terrors occur during deep slow-wave sleep (Stages 3–4 NREM) and are classified as NREM parasomnias. They involve intense fear and autonomic activation but no dream recall. REM sleep produces nightmares with intact memory. Stage 1 sleep is light and not associated with parasomnias. Therefore, the correct answer is Stage 3–4 NREM sleep. These events typically resolve with age.
4. Which EEG pattern is characteristic of Stage 3–4 NREM sleep?
A) Alpha waves
B) Beta waves
C) Delta waves
D) Sleep spindles
Explanation:
Stages 3–4 NREM sleep, known as slow-wave sleep, are dominated by high-amplitude delta waves (0.5–2 Hz). Alpha waves occur in relaxed wakefulness, beta waves in active thinking, and sleep spindles in Stage 2 NREM. Thus, the correct answer is Delta waves. Delta activity reflects deep sleep with high arousal threshold.
5. A patient sleepwalks and has no memory of the event. The phenomenon is best classified as:
A) REM parasomnia
B) NREM parasomnia
C) Psychosis
D) Sleep apnea
Explanation:
Sleepwalking is a type of NREM parasomnia, occurring during deep slow-wave sleep, and is associated with amnesia for the episode. REM parasomnias involve dream enactment (e.g., REM behavior disorder). Psychosis is unrelated to sleep stages, and sleep apnea involves breathing disruption. The correct answer is NREM parasomnia. It is triggered by incomplete arousal from deep sleep.
6. REM Behavior Disorder (RBD) is caused by failure of:
A) REM sleep muscle atonia
B) Slow-wave sleep
C) Dream generation
D) GABAergic neurons
Explanation:
In RBD, loss of normal REM muscle atonia allows patients to act out dreams. Slow-wave sleep is intact, and dream generation remains functional. GABAergic neurons are involved but not uniquely responsible. Thus, the correct answer is REM sleep muscle atonia. This condition is associated with neurodegenerative disorders like Parkinson’s disease.
7. Sleep spindle appearance marks which sleep stage?
A) Stage 1
B) Stage 2
C) Stage 3
D) REM sleep
Explanation:
Stage 2 NREM sleep is characterized by sleep spindles (12–14 Hz) and K-complexes on EEG. These represent thalamocortical activity regulating sensory suppression. Stage 1 has theta waves, Stage 3–4 has delta waves, and REM has low-voltage mixed frequencies. Therefore, the correct answer is Stage 2. This stage comprises a large portion of total sleep time.
8. A patient with obstructive sleep apnea often experiences fragmentation of which sleep stage?
A) Stage 3–4 NREM
B) REM sleep
C) Wakefulness
D) Stage 1 sleep
Explanation:
Obstructive sleep apnea results in repeated nighttime arousals that disturb REM sleep substantially, reducing its duration and quality. Deep slow-wave sleep may also be fragmented but REM is most affected. Therefore, the correct answer is REM sleep. This contributes to daytime sleepiness and impaired memory consolidation.
9. Narcolepsy is associated with deficiency of:
A) GABA
B) Orexin (hypocretin)
C) Serotonin
D) Acetylcholine
Explanation:
Narcolepsy results from deficiency of orexin (hypocretin) produced in the lateral hypothalamus. This peptide stabilizes wakefulness. Loss of orexin leads to sleep attacks, cataplexy, and hallucinations. GABA promotes sleep but is not deficient. Serotonin and acetylcholine modulate sleep cycles but are not primary causes. Thus, the correct answer is Orexin. Narcolepsy is confirmed via CSF orexin levels and polysomnography.
10. Dreaming with vivid recall occurs mainly in:
A) Stage 3 NREM
B) REM sleep
C) Stage 1 NREM
D) Stage 2 NREM
Explanation:
Vivid, story-like dreaming occurs predominantly during REM sleep due to active cortical processing with muscle atonia. NREM dreams, when present, are brief and less structured. Therefore, the correct answer is REM sleep. This stage plays a key role in emotional regulation and memory consolidation.
11. Bruxism (teeth grinding) commonly occurs in:
A) REM sleep
B) Stage 1 NREM
C) Stage 2 NREM
D) Stage 4 NREM
Explanation:
Bruxism is most commonly observed during Stage 2 NREM sleep, which is characterized by sleep spindles and K-complexes. It is considered a parasomnia and may be triggered by stress or malocclusion. REM sleep prevents such movements due to muscle atonia. Therefore, the correct answer is Stage 2 NREM. Diagnosis may require polysomnography if severe.
Chapter: Neurophysiology; Topic: Cerebellum; Subtopic: Cerebellar Cortex – Neuronal Types
KEYWORD DEFINITIONS
• Granule cells – Only excitatory neurons in cerebellar cortex (glutamatergic)
• Purkinje cells – Large inhibitory (GABAergic) projection neurons
• Basket & Stellate cells – Inhibitory interneurons
• Golgi cells – Inhibitory interneurons regulating granule cell activity
• Mossy & climbing fibers – Excitatory afferents to cerebellum
Lead Question – 2015
1. The only excitatory neurons in cerebellar cortex are?
A) Purkinje
B) Basket
C) Golgi
D) Granule cells
Explanation:
Granule cells are the only excitatory neurons in the cerebellar cortex and use glutamate as their neurotransmitter. All other cerebellar cortical neurons—including Purkinje, basket, stellate, and Golgi cells—are inhibitory and release GABA. Granule cells receive mossy fiber input and send their axons as parallel fibers to synapse on Purkinje cells. Therefore, the correct answer is Granule cells. Their excitatory output is essential for modulating cerebellar circuitry and coordinating fine motor control.
2. Purkinje cells release which neurotransmitter?
A) Glutamate
B) GABA
C) Glycine
D) Dopamine
Explanation:
Purkinje cells are the primary output neurons of the cerebellar cortex and are purely inhibitory, releasing GABA onto deep cerebellar nuclei. They integrate inputs from climbing and mossy fiber pathways. Glutamate is excitatory, dopamine modulates basal ganglia, and glycine mediates inhibition in the spinal cord. Thus, the correct answer is GABA. Their inhibitory output is crucial for coordinated movement.
3. A patient with cerebellar degeneration shows intention tremor. Which pathway is most affected?
A) Corticospinal tract
B) Mossy fiber pathway
C) Spinothalamic tract
D) Reticulospinal tract
Explanation:
The mossy fiber pathway provides major excitatory input to granule cells, modulating Purkinje cell activity. Damage disrupts cerebellar processing, resulting in intention tremor, dysmetria, and ataxia. Corticospinal and spinothalamic tracts serve motor output and sensory pathways respectively, while reticulospinal tracts mediate posture. Therefore, the correct answer is Mossy fiber pathway. These circuits fine-tune motor execution.
4. Climbing fibers originate from which nucleus?
A) Red nucleus
B) Inferior olivary nucleus
C) Dentate nucleus
D) Substantia nigra
Explanation:
Climbing fibers arise exclusively from the inferior olivary nucleus and form powerful excitatory synapses on Purkinje cells. These fibers produce complex spikes essential for motor learning. Red nucleus, dentate, and substantia nigra project to different motor pathways. Therefore, the correct answer is Inferior olivary nucleus. Climbing fiber dysfunction impairs cerebellar learning mechanisms.
5. Which cerebellar cell type receives parallel fiber input?
A) Purkinje cell
B) Basket cell
C) Golgi cell
D) All of the above
Explanation:
Parallel fibers, which are axons of granule cells, synapse extensively on Purkinje cells, basket cells, and stellate cells, influencing both inhibitory and excitatory circuitry. Golgi cells receive mossy fiber and granule cell collaterals. Therefore, the correct answer is All of the above. Parallel fibers play a central role in cerebellar cortical integration.
6. A 50-year-old alcoholic patient presents with truncal ataxia. Which cerebellar region is most likely affected?
A) Cerebrocerebellum
B) Vestibulocerebellum (flocculonodular lobe)
C) Spinocerebellum (vermis)
D) Dentate nucleus
Explanation:
Truncal ataxia results from vermis (spinocerebellum) damage, commonly seen in chronic alcoholism. This region regulates axial muscles and gait stability. Vestibulocerebellum controls eye and balance movements, cerebrocerebellum handles limb planning, and dentate nucleus coordinates fine limb movement. Therefore, the correct answer is Spinocerebellum. Clinical findings include a wide-based gait and midline instability.
7. Basket cells in cerebellar cortex are:
A) Excitatory
B) Inhibitory
C) Glycinergic
D) Dopaminergic
Explanation:
Basket cells are inhibitory interneurons using GABA as their neurotransmitter. They synapse on Purkinje cell bodies and modulate their activity. They are not excitatory, glycinergic, or dopaminergic. Therefore, the correct answer is Inhibitory. Their role is vital in shaping cerebellar output and timing of Purkinje cell firing.
8. Which cerebellar neurons form parallel fibers?
A) Golgi cells
B) Granule cells
C) Purkinje cells
D) Stellate cells
Explanation:
Granule cells send their axons upward to bifurcate into parallel fibers that synapse widely throughout the molecular layer. Purkinje cells have dendritic trees, Golgi cells are inhibitory, and stellate cells modulate Purkinje firing. Thus, the correct answer is Granule cells. Parallel fibers enable broad integration of sensory and motor information.
9. A patient with cerebellar lesion shows dysdiadochokinesia. This signifies impairment in:
A) Rapid alternating movements
B) Muscle tone
C) Pain sensation
D) Vision
Explanation:
Dysdiadochokinesia—difficulty performing rapid alternating movements—is a hallmark sign of cerebellar dysfunction. It arises due to impaired Purkinje cell modulation of motor timing. Muscle tone, pain, and vision are not directly responsible. Therefore, the correct answer is Rapid alternating movements. This clinical sign reflects the role of cerebellum in coordinating agonist–antagonist muscle switching.
10. Stellate cells in cerebellum act as:
A) Excitatory interneurons
B) Inhibitory interneurons
C) Motor neurons
D) Sensory relay neurons
Explanation:
Stellate cells, like basket cells, are inhibitory interneurons releasing GABA. They synapse on Purkinje dendrites, modulating cortical output. They are neither excitatory nor motor or sensory relays. Therefore, the answer is Inhibitory interneurons. These cells contribute to cerebellar timing and refinement of motor signals.
11. Which afferent fiber type excites granule cells?
A) Climbing fibers
B) Mossy fibers
C) Parallel fibers
D) Purkinje axons
Explanation:
Mossy fibers provide the primary excitatory input to granule cells, conveying sensory and cortical information. Climbing fibers excite Purkinje cells directly, parallel fibers arise from granule cells, and Purkinje axons are inhibitory outputs. Thus, the correct answer is Mossy fibers. This pathway forms the fundamental feed-forward circuit of the cerebellar cortex.
Chapter: Neurophysiology; Topic: Synaptic Transmission; Subtopic: Excitatory Postsynaptic Potentials (EPSP)
KEYWORD DEFINITIONS
• EPSP – Depolarizing postsynaptic potential making neuron more likely to fire
• Na⁺ influx – Major ion movement causing EPSP
• Glutamate – Primary excitatory neurotransmitter
• Ligand-gated ion channels – Open when neurotransmitter binds
• Depolarization – Membrane potential becomes less negative
Lead Question – 2015
1. EPSP is due to?
A) K⁺ influx
B) Na⁺ efflux
C) Na⁺ influx
D) Ca²⁺ influx
Explanation:
An excitatory postsynaptic potential (EPSP) is generated when neurotransmitters like glutamate open ligand-gated channels, allowing Na⁺ ions to flow into the postsynaptic neuron. This inward Na⁺ current causes depolarization, bringing the membrane closer to the threshold for action potential firing. K⁺ influx and Na⁺ efflux would hyperpolarize the cell, while Ca²⁺ influx is typically associated with presynaptic neurotransmitter release rather than postsynaptic depolarization. Therefore, the correct answer is Na⁺ influx. EPSPs form the fundamental basis of excitatory synaptic integration in the CNS.
2. The main excitatory neurotransmitter in the CNS is:
A) GABA
B) Glycine
C) Glutamate
D) Serotonin
Explanation:
Glutamate is the principal excitatory neurotransmitter in the central nervous system, responsible for generating EPSPs by opening Na⁺ and Ca²⁺ permeable channels. GABA and glycine are inhibitory, while serotonin modulates mood and cognition. Therefore, the correct answer is Glutamate. Glutamatergic excitatory synapses are essential for learning, memory, and neural plasticity.
3. A patient develops seizures due to excessive excitatory activity. Which receptor is most likely overactivated?
A) NMDA receptor
B) GABA-A receptor
C) Glycine receptor
D) Serotonin receptor
Explanation:
Excessive activation of NMDA receptors permits prolonged Na⁺ and Ca²⁺ influx, greatly enhancing excitatory neurotransmission and potentially leading to seizures. GABA-A and glycine receptors mediate inhibition, and serotonin is modulatory. Thus, the correct answer is NMDA receptor. NMDA overactivation contributes to neuronal excitotoxicity seen in stroke and epilepsy.
4. IPSP is typically caused by which ion movement?
A) Na⁺ influx
B) K⁺ efflux
C) Ca²⁺ influx
D) Cl⁻ efflux
Explanation:
An inhibitory postsynaptic potential (IPSP) results from K⁺ efflux or Cl⁻ influx, both of which hyperpolarize the membrane, making action potential generation less likely. Na⁺ and Ca²⁺ influx are associated with depolarization. Therefore, the answer is K⁺ efflux. IPSPs counterbalance EPSPs, ensuring controlled neural activity.
5. Summation of multiple EPSPs occurring close in time at a single synapse refers to:
A) Spatial summation
B) Temporal summation
C) Inhibitory summation
D) After-hyperpolarization
Explanation:
Temporal summation occurs when successive EPSPs arrive rapidly at the same synapse, increasing the likelihood of reaching threshold. Spatial summation involves simultaneous EPSPs from different synapses. Therefore, the correct answer is Temporal summation. Summation allows neurons to integrate incoming signals and regulate firing patterns.
6. A patient with hypoxic injury shows decreased EPSP formation. Which mechanism is impaired?
A) Na⁺ channel opening
B) GABA release
C) Cl⁻ influx
D) K⁺ efflux
Explanation:
EPSPs depend on the opening of ligand-gated Na⁺ channels activated by glutamate. Hypoxia impairs ATP-dependent ion gradients and synaptic transmission, reducing Na⁺ influx and decreasing excitatory drive. GABA release, Cl⁻ influx, and K⁺ efflux are inhibitory mechanisms. Therefore, the correct answer is Na⁺ channel opening. Hypoxia often depresses CNS excitatory function first.
7. Which receptor subtype mediates fast EPSP?
A) AMPA receptor
B) NMDA receptor
C) GABA-B receptor
D) Glycine receptor
Explanation:
AMPA receptors mediate fast excitatory postsynaptic potentials by allowing rapid Na⁺ influx when glutamate binds. NMDA receptors are slower, voltage-dependent, and require depolarization to relieve Mg²⁺ block. GABA-B receptors are inhibitory metabotropic receptors. Thus, the correct answer is AMPA receptor. Fast EPSPs are crucial for rapid synaptic communication.
8. A 65-year-old with Alzheimer’s disease shows decreased cholinergic EPSPs. Which area degenerates first?
A) Substantia nigra
B) Basal forebrain nucleus (Nucleus basalis of Meynert)
C) Cerebellum
D) Red nucleus
Explanation:
Alzheimer’s disease features degenerative loss of cholinergic neurons in the nucleus basalis of Meynert, reducing acetylcholine-mediated EPSPs in the cortex and undermining cognition. Substantia nigra degeneration causes Parkinsonism. Cerebellum and red nucleus are unrelated to cortical cholinergic projections. Therefore, the correct answer is Basal forebrain nucleus. This deficit contributes to memory impairment in Alzheimer’s.
9. At the neuromuscular junction, the major excitatory ion responsible for depolarization is:
A) Cl⁻ influx
B) K⁺ efflux
C) Na⁺ influx
D) Ca²⁺ efflux
Explanation:
Acetylcholine binds nicotinic receptors at the neuromuscular junction, opening channels permeable mainly to Na⁺, which rapidly enters the muscle cell to produce an end-plate potential. Cl⁻ and K⁺ fluxes do not initiate NMJ depolarization. Thus, the correct answer is Na⁺ influx. End-plate potential triggers muscle action potential and contraction.
10. Which neurotransmitter is most closely associated with generating EPSPs in the hippocampus?
A) GABA
B) Glutamate
C) Dopamine
D) Glycine
Explanation:
Glutamate is the primary excitatory neurotransmitter in hippocampal circuits, essential for learning and memory. It activates AMPA and NMDA receptors to produce EPSPs. GABA and glycine are inhibitory, while dopamine modulates reward pathways. Therefore, the correct answer is Glutamate. Synaptic plasticity in the hippocampus relies heavily on glutamatergic EPSPs.
11. EPSPs bring membrane potential closer to threshold because they cause:
A) Hyperpolarization
B) Depolarization
C) Inhibition
D) After-potential
Explanation:
EPSPs are depolarizing events generated by inward Na⁺ currents, making the membrane potential less negative and more likely to trigger an action potential. Hyperpolarization and inhibition move the membrane away from threshold. After-potentials follow action potentials but are not part of EPSP formation. Therefore, the correct answer is Depolarization. EPSPs integrate until threshold is reached, initiating neuronal firing.
Chapter: Neurophysiology; Topic: Proprioception; Subtopic: Muscle Spindles – Distribution & Function
KEYWORD DEFINITIONS
• Muscle spindle – Stretch receptor sensing muscle length and rate of stretch
• Intrafusal fibers – Specialized fibers inside muscle spindles
• Gamma motor neurons – Regulate spindle sensitivity
• Proprioception – Awareness of body position
• Fine motor muscles – Muscles requiring precise control, having high spindle density
Lead Question – 2015
1. Maximum density of muscle spindle is found in?
A) Calf muscle
B) Lumbricals
C) Quadriceps muscle
D) Triceps
Explanation:
Muscle spindles are most densely distributed in muscles requiring fine, precise movements. Lumbricals of the hand perform delicate actions such as finger flexion–extension balance, making them rich in muscle spindles. Large postural muscles like calf, quadriceps, and triceps have fewer spindles per gram, as they focus more on power than precision. Therefore, the correct answer is Lumbricals. High spindle density ensures accurate proprioceptive feedback essential for coordinated hand movements and fine motor tasks.
2. Muscle spindle primarily detects:
A) Muscle tension
B) Muscle length
C) Vibration
D) Temperature
Explanation:
Muscle spindles respond to changes in muscle length and the rate of lengthening. They contain intrafusal fibers innervated by gamma motor neurons, allowing dynamic regulation. Golgi tendon organs detect tension, Pacinian corpuscles detect vibration, and temperature is sensed by thermoreceptors. Therefore, the correct answer is Muscle length. This mechanism plays a crucial role in stretch reflexes, posture maintenance, and coordinated movement.
3. A patient loses proprioception in the lower limb. Which structure is most likely affected?
A) Golgi tendon organ
B) Pacinian corpuscle
C) Muscle spindle
D) Free nerve endings
Explanation:
Proprioception depends largely on muscle spindles, which monitor muscle length and movement. Damage to these structures impairs the ability to sense limb position, leading to ataxic gait. Golgi tendon organs detect muscle tension, Pacinian corpuscles detect vibration, and free nerve endings mediate pain and temperature. Therefore, the answer is Muscle spindle. Clinical tests include joint position sense and Romberg’s sign evaluation.
4. Gamma motor neurons regulate which function?
A) Muscle contraction strength
B) Spindle sensitivity
C) Tendon tension
D) Skin stretch
Explanation:
Gamma motor neurons adjust intrafusal fiber tension, thereby regulating muscle spindle sensitivity. This ensures spindles remain responsive during both contraction and relaxation. Alpha motor neurons drive muscle contraction, Golgi tendon organs monitor tension, and Ruffini endings detect skin stretch. Therefore, the correct answer is Spindle sensitivity. Gamma activation is essential for fine motor coordination and maintaining posture.
5. Which muscle type has the highest muscle spindle-to-muscle fiber ratio?
A) Postural muscles
B) Fine control muscles
C) Power muscles
D) Respiratory muscles
Explanation:
Fine control muscles, such as extraocular muscles and lumbricals, require precise regulation of movement and thus have the highest density of muscle spindles. Postural muscles have moderate numbers, while power muscles prioritize force over precision. Respiratory muscles rely more on central drive. Therefore, the correct answer is Fine control muscles. High spindle ratio ensures accurate proprioceptive signaling during delicate motor tasks.
6. A 55-year-old diabetic patient shows impaired position sense in toes. Which pathway is likely affected?
A) Spinothalamic tract
B) Dorsal column pathway
C) Corticospinal tract
D) Vestibulospinal tract
Explanation:
The dorsal column–medial lemniscal pathway transmits proprioception, vibration, and fine touch from muscle spindles and mechanoreceptors. Diabetic neuropathy commonly damages large myelinated fibers, leading to proprioceptive loss. Spinothalamic tract carries pain and temperature, corticospinal tract mediates voluntary motor control, and vestibulospinal tract maintains balance. Hence, the answer is Dorsal column pathway. Testing includes vibration sense and joint position assessment.
7. Stretch reflex is mediated by:
A) Pacinian corpuscles
B) Golgi tendon organs
C) Muscle spindles
D) Free nerve endings
Explanation:
The stretch reflex is initiated when muscle spindles detect rapid stretch, sending Ia afferent signals to spinal cord motor neurons, causing reflex contraction. Golgi tendon organs mediate inverse stretch reflex, reducing tension. Pacinian corpuscles detect vibration, and free nerve endings detect pain. Therefore, the correct answer is Muscle spindles. This reflex is clinically tested with tendon taps such as knee jerk.
8. Golgi tendon organ differs from muscle spindle by detecting:
A) Joint position
B) Muscle tension
C) Muscle velocity
D) Skin stretch
Explanation:
Golgi tendon organs respond to muscle tension, particularly during strong contractions, preventing damage by inhibiting excessive force. Muscle spindles detect length changes, not tension. Skin stretch is sensed by Ruffini endings, and joint position involves multiple proprioceptors. Therefore, the correct answer is Muscle tension. Golgi-mediated inhibition protects muscles from overload injuries.
9. A patient with posterior column degeneration has difficulty standing with eyes closed. This is due to loss of:
A) Pain sensation
B) Temperature sense
C) Proprioception
D) Touch-pressure sense
Explanation:
Posterior column degeneration, as seen in B12 deficiency or tabes dorsalis, impairs proprioception from muscle spindles. With eyes closed, visual compensation is lost, leading to a positive Romberg’s sign. Pain and temperature are transmitted through spinothalamic pathways and remain intact early. Therefore, the answer is Proprioception. This confirms reliance on spindle-mediated feedback for posture.
10. Muscle spindles are most active during:
A) Muscle contraction only
B) Muscle relaxation only
C) Both contraction and stretch
D) Tendon injury
Explanation:
Muscle spindles remain active during both stretch and contraction due to gamma motor neuron control, maintaining sensitivity across muscle states. They detect dynamic and static length changes. Golgi tendon organs monitor tension during contraction. Tendon injury does not preferentially activate spindles. Therefore, the correct answer is Both contraction and stretch. This dual activity ensures stable posture and coordinated movement.
11. Fine motor muscles like lumbricals have dense spindles primarily to improve:
A) Power generation
B) Precision of movement
C) Endurance
D) Reflex inhibition
Explanation:
High spindle density in fine motor muscles enhances proprioceptive accuracy for delicate movements. These receptors provide continuous feedback about muscle length and position, enabling precise control of finger motion. Power and endurance depend on muscle fiber type, not spindle density. Reflex inhibition is mediated by Golgi tendon organs. Therefore, the correct answer is Precision of movement. This anatomical specialization supports skilled hand functions like writing and typing.
Chapter: Neurophysiology; Topic: Higher Cortical Functions; Subtopic: Brodmann Areas – Motor Speech Area
KEYWORD DEFINITIONS
• Brodmann areas – Cortical regions mapped based on cytoarchitecture
• Broca’s area – Motor speech area responsible for word formation
• Area 44/45 – Inferior frontal gyrus regions involved in speech output
• Wernicke’s area – Language comprehension region
• Arcuate fasciculus – Connects Broca’s and Wernicke’s areas for repetition
Lead Question – 2015
1. Broadman's area for motor speech?
A) Area 1,2,3
B) Area 4,6
C) Area 28,29
D) Area 44
Explanation:
Broca’s area, the motor speech area responsible for word formation and articulation, corresponds mainly to Brodmann area 44 and partly area 45 in the dominant hemisphere. It coordinates motor planning for expressive speech. Areas 1,2,3 form the primary somatosensory cortex, area 4 and 6 belong to the motor and premotor cortex, and areas 28/29 belong to the limbic cortex. Therefore, the correct answer is Area 44. Damage to this region results in Broca’s aphasia, characterized by non-fluent, effortful speech with preserved comprehension.
2. Which Brodmann area is associated with language comprehension?
A) Area 44
B) Area 6
C) Area 22
D) Area 17
Explanation:
Wernicke’s area, responsible for comprehension of spoken and written language, corresponds to Brodmann area 22 in the dominant hemisphere. Broca’s area (44/45) manages speech production, area 6 is premotor cortex, and area 17 is the primary visual cortex. Thus, the correct answer is Area 22. Lesions to this area lead to fluent but nonsensical speech, characteristic of Wernicke’s aphasia, where comprehension is severely impaired.
3. A patient presents with fluent speech but poor comprehension. Which region is most likely damaged?
A) Area 4
B) Area 22
C) Area 44
D) Area 7
Explanation:
Fluent but meaningless speech with impaired comprehension is the hallmark of Wernicke’s aphasia, caused by lesions in Brodmann area 22. Area 4 handles voluntary motor control, area 44 controls motor speech, and area 7 is involved in visuospatial processing. Therefore, the correct answer is Area 22. Such patients often produce “word salad” and are unaware of their deficits, unlike Broca’s aphasia patients.
4. Broca’s aphasia typically occurs due to a lesion in which cerebral artery territory?
A) ACA
B) PCA
C) MCA – superior division
D) MCA – inferior division
Explanation:
Broca’s area is supplied predominantly by the superior division of the middle cerebral artery (MCA). Infarction here results in non-fluent aphasia with preserved comprehension. The inferior division of MCA supplies Wernicke’s area, PCA supplies occipital lobe and inferotemporal regions, and ACA supplies medial frontal areas. Therefore, the correct answer is MCA – superior division. Speech becomes effortful and telegraphic, but understanding remains intact.
5. Which Brodmann area corresponds to primary motor cortex?
A) Area 1
B) Area 6
C) Area 4
D) Area 44
Explanation:
The primary motor cortex is located in Brodmann area 4, situated in the precentral gyrus. It is responsible for voluntary motor control. Area 6 is the premotor cortex, area 1 is part of the sensory cortex, and area 44 corresponds to Broca’s area. Therefore, the correct answer is Area 4. Damage here leads to contralateral weakness or paralysis, demonstrating strict somatotopic organization.
6. A patient understands speech but struggles severely with articulation. Most likely defect?
A) Area 22 lesion
B) Area 4 lesion
C) Area 44 lesion
D) Area 17 lesion
Explanation:
Intact comprehension with difficulty in articulation strongly suggests Broca’s aphasia, caused by damage to Brodmann area 44. Area 22 lesions impair comprehension, area 4 lesions impair motor control of limbs, and area 17 lesions affect vision. Therefore, the correct answer is Area 44. Speech becomes non-fluent and effortful, but patients retain insight and attempt to correct their mistakes.
7. Which area connects Broca’s and Wernicke’s regions for repetition?
A) Angular gyrus
B) Arcuate fasciculus
C) Uncinate fasciculus
D) Corpus callosum
Explanation:
The arcuate fasciculus is the white matter tract that links Broca’s and Wernicke’s areas, enabling repetition of spoken words. Damage causes conduction aphasia, characterized by impaired repetition with preserved comprehension and fluency. Angular gyrus is involved in reading/writing, uncinate fasciculus connects frontal–temporal poles, and corpus callosum coordinates interhemispheric exchange. Thus, the answer is Arcuate fasciculus.
8. The region responsible for converting written language into spoken form is:
A) Area 17
B) Angular gyrus
C) Area 44
D) Area 4
Explanation:
The angular gyrus integrates visual information (letters/words) and converts it into linguistic form, enabling reading comprehension. Area 17 is primary visual cortex, area 44 handles speech production, and area 4 controls motor output. Therefore, the correct answer is Angular gyrus. Damage leads to alexia with agraphia, where reading and writing abilities are lost.
9. A patient cannot repeat words but speaks fluently with preserved comprehension. Diagnosis?
A) Broca’s aphasia
B) Wernicke’s aphasia
C) Conduction aphasia
D) Global aphasia
Explanation:
Conduction aphasia is caused by damage to the arcuate fasciculus connecting Broca’s and Wernicke’s areas. This results in selective impairment of repetition while fluency and comprehension remain intact. Broca’s aphasia affects speech production, Wernicke’s affects comprehension, and global aphasia affects all aspects. Hence, the correct answer is Conduction aphasia. Patients often produce phonemic substitution errors.
10. Brodmann area 45 is primarily associated with:
A) Motor planning
B) Speech articulation
C) Auditory comprehension
D) Visual processing
Explanation:
Area 45, along with area 44, forms Broca’s complex and plays an essential role in speech planning and articulation. It works with motor cortex to orchestrate expressive language. Comprehension occurs in area 22, motor planning in area 6, and vision in area 17. Thus, the correct answer is Speech articulation. Lesions here contribute to expressive aphasia and impaired verbal fluency.
11. Global aphasia results from damage to:
A) Area 22 exclusively
B) Area 44 exclusively
C) Perisylvian cortex
D) Occipital cortex
Explanation:
Global aphasia occurs when both Broca’s and Wernicke’s areas, along with surrounding perisylvian cortex, are damaged—usually due to extensive MCA infarction. This causes profound impairment in fluency, comprehension, naming, and repetition. Damage to area 22 or 44 alone cannot produce global aphasia. Therefore, the correct answer is Perisylvian cortex. This represents the most severe form of aphasia with widespread cortical involvement.
Chapter: Neurophysiology; Topic: Higher Cortical Functions; Subtopic: Language Areas – Motor Speech
KEYWORD DEFINITIONS
• Broca’s area – Motor speech area responsible for word formation
• Wernicke’s area – Comprehension of spoken and written language
• Arcuate fasciculus – Connects Broca’s and Wernicke’s areas for repetition
• Aphasia – Impairment of language production or comprehension
• Dominant hemisphere – Usually left hemisphere controlling language functions
Lead Question – 2015
1. Broca's area is concerned with:
A) Word formation
B) Comprehension
C) Repetition
D) Reading
Explanation:
Broca’s area, located in the dominant frontal lobe (usually left), is the motor speech area responsible for word formation and articulation. It coordinates the movements required for verbal expression. Damage to this region results in Broca’s aphasia, characterized by slow, effortful speech with preserved comprehension. Wernicke’s area manages comprehension, while repetition requires the arcuate fasciculus. Reading involves multiple visual and language centers. Thus, the correct answer is Word formation. Clinically, Broca’s aphasia patients understand language but cannot produce fluent speech.
2. Damage to Broca’s area causes which condition?
A) Fluent aphasia
B) Broca’s aphasia
C) Conduction aphasia
D) Global aphasia
Explanation:
Lesions in Broca’s area lead to Broca’s aphasia, a non-fluent type of aphasia in which comprehension remains relatively intact but speech output becomes slow, effortful, and fragmented. Fluent aphasia occurs in Wernicke’s lesions, conduction aphasia in arcuate fasciculus damage, and global aphasia in extensive perisylvian lesions. Therefore, the correct answer is Broca’s aphasia. Patients struggle with forming words but often retain awareness of their deficits.
3. A patient speaks with slow, effortful speech but comprehension is intact. Most likely lesion?
A) Wernicke’s area
B) Angular gyrus
C) Broca’s area
D) Primary auditory cortex
Explanation:
Slow, effortful, and non-fluent speech with preserved comprehension is the hallmark of Broca’s aphasia, resulting from damage to Broca’s area in the inferior frontal gyrus. Wernicke’s area lesions impair comprehension, angular gyrus lesions impair reading and writing, and primary auditory cortex lesions impair sound perception. Thus, the correct answer is Broca’s area. These patients produce telegraphic speech but retain the ability to understand spoken language.
4. Repetition is specifically impaired in which type of aphasia?
A) Broca’s aphasia
B) Conduction aphasia
C) Wernicke’s aphasia
D) Transcortical motor aphasia
Explanation:
Conduction aphasia is characterized by disproportionately impaired repetition despite preserved comprehension and relatively fluent speech. It arises from damage to the arcuate fasciculus, which links Broca’s and Wernicke’s areas. In Broca’s aphasia speech is non-fluent, in Wernicke’s aphasia comprehension is impaired, and repetition is preserved in transcortical aphasias. Therefore, the answer is Conduction aphasia. Patients often make phonemic errors and struggle to repeat even simple phrases.
5. Which artery supplies Broca’s area?
A) Anterior cerebral artery
B) Middle cerebral artery
C) Posterior cerebral artery
D) Basilar artery
Explanation:
Broca’s area is supplied by the superior division of the middle cerebral artery (MCA). Infarction in this region produces non-fluent aphasia. The anterior cerebral artery supplies medial frontal regions, while the posterior cerebral artery supplies occipital and inferotemporal lobes. Basilar artery involvement affects brainstem structures. Therefore, the correct answer is Middle cerebral artery. MCA strokes are among the most common causes of cortical language deficits.
6. A patient understands language but cannot speak fluently. This describes:
A) Wernicke’s aphasia
B) Broca’s aphasia
C) Global aphasia
D) Dysarthria
Explanation:
Patients with Broca’s aphasia have intact comprehension but reduced fluency due to impaired language motor planning. Wernicke’s aphasia features fluent but nonsensical speech with poor comprehension. Global aphasia affects all language modalities, and dysarthria results from motor weakness rather than language planning deficits. Therefore, the correct answer is Broca’s aphasia. Awareness of the deficit often leads to patient frustration during communication attempts.
7. A right-handed patient with a left inferior frontal gyrus lesion will MOST likely present with:
A) Fluent speech with poor comprehension
B) Impaired repetition only
C) Slow, non-fluent speech
D) Impaired naming with intact fluency
Explanation:
Left inferior frontal gyrus lesions (Broca’s area) cause slow, non-fluent, effortful speech with intact comprehension. Wernicke’s lesions cause fluent speech with poor comprehension, arcuate fasciculus lesions impair repetition, and naming deficits with fluency suggest anomic aphasia. The correct answer is Slow, non-fluent speech. This highlights the role of Broca’s area in expressive language.
8. Which cortical area is primarily involved in language comprehension?
A) Broca’s area
B) Wernicke’s area
C) Prefrontal cortex
D) Supplementary motor area
Explanation:
Wernicke’s area, located in the superior temporal gyrus of the dominant hemisphere, is crucial for understanding spoken and written language. Broca’s area handles speech production, the prefrontal cortex manages executive functions, and the supplementary motor area coordinates motor planning. Therefore, the answer is Wernicke’s area. Lesions here cause fluent but meaningless speech with impaired comprehension.
9. Inability to read despite normal vision is called:
A) Alexia
B) Apraxia
C) Dysarthria
D) Prosopagnosia
Explanation:
Alexia, often called word blindness, results from lesions in the left occipitotemporal region, disrupting visual–language integration. Prosopagnosia affects face recognition, apraxia affects motor planning, and dysarthria affects articulation due to motor weakness. Therefore, the correct answer is Alexia. Patients can see text but cannot interpret printed words, indicating higher-order processing deficits.
10. A patient cannot repeat simple phrases but has intact comprehension and fluent speech. Diagnosis?
A) Wernicke’s aphasia
B) Conduction aphasia
C) Broca’s aphasia
D) Transcortical sensory aphasia
Explanation:
Conduction aphasia is caused by damage to the arcuate fasciculus, disrupting communication between Broca’s and Wernicke’s areas. As a result, repetition is disproportionately impaired while fluency and comprehension remain largely intact. Wernicke’s aphasia impairs comprehension, Broca’s impairs fluency, and transcortical sensory aphasia preserves repetition. Thus, the correct answer is Conduction aphasia. Patients often substitute similar-sounding words during speech.
11. A patient with global aphasia has lesions affecting:
A) Broca’s and Wernicke’s areas
B) Only Broca’s area
C) Only Wernicke’s area
D) Arcuate fasciculus only
Explanation:
Global aphasia occurs when both Broca’s and Wernicke’s areas, along with surrounding perisylvian cortex, are damaged—most commonly due to extensive MCA infarction. All language functions are severely impaired including fluency, comprehension, and repetition. Damage to one area alone does not produce this profound deficit. Therefore, the correct answer is Broca’s and Wernicke’s areas. This is the most severe form of aphasia and has a poor prognosis.
Chapter: Neurophysiology; Topic: Higher Cortical Functions; Subtopic: Visual Association Cortex – Face Recognition
KEYWORD DEFINITIONS
• Prosopagnosia – Inability to recognize familiar faces
• Visual association cortex – Processes complex visual information
• Fusiform gyrus – Brain region responsible for face perception
• Agnosia – Failure to recognize despite intact sensory function
• Temporal lobe – Critical for memory and recognition functions
Lead Question – 2015
1. "Prosopagnosia" is characterized by:
A) Inability to read
B) Inability to identify faces
C) Inability to write
D) Inability to speak
Explanation:
Prosopagnosia is characterized by the inability to recognize familiar faces despite intact vision and intellectual abilities. The condition results from damage to the fusiform face area in the temporal lobe, especially on the right side. It is a form of visual agnosia specific to facial identity, not general object recognition. Reading, writing, and speech are not primarily impaired. Therefore, the correct answer is Inability to identify faces. Clinically, patients may identify others using secondary cues like voice or gait, highlighting the selective nature of this deficit.
2. Damage to the fusiform face area results in:
A) Pure alexia
B) Prosopagnosia
C) Agraphia
D) Anomia
Explanation:
The fusiform face area, located in the inferior temporal cortex, specializes in processing facial recognition. Lesions here cause prosopagnosia, a selective inability to recognize familiar faces. Pure alexia arises from left occipitotemporal lesions, agraphia involves parietal lobe damage, and anomia results from disruptions in language areas. Thus, the correct answer is Prosopagnosia. Clinically, these patients can still recognize individuals using auditory or contextual cues, confirming the specificity of the deficit.
3. A patient recognizes objects but not faces. This condition is called:
A) Aphasia
B) Agnosia
C) Prosopagnosia
D) Apraxia
Explanation:
Prosopagnosia is a subtype of visual agnosia where facial recognition alone is impaired, while recognition of objects remains intact. Aphasia affects language, apraxia affects motor planning, and general agnosia involves broader recognition deficits. Therefore, the correct answer is Prosopagnosia. This condition commonly results from bilateral temporal-occipital damage and may be seen in stroke, trauma, or degenerative diseases.
4. A 60-year-old stroke patient cannot recognize family members by face but can identify their voices. Most likely lesion?
A) Wernicke’s area
B) Fusiform gyrus
C) Precentral gyrus
D) Postcentral gyrus
Explanation:
The fusiform gyrus in the temporal lobe specialized for face recognition is the primary site affected in prosopagnosia. Voices are processed separately, therefore remain intact in these patients. Wernicke’s area affects language comprehension, while precentral and postcentral gyri manage motor and sensory functions respectively. Thus, the correct answer is Fusiform gyrus. Recognition of individuals through non-visual cues helps distinguish this disorder from more global cognitive deficits.
5. Inability to identify familiar faces despite normal vision is called:
A) Visual agnosia
B) Prosopagnosia
C) Agraphesthesia
D) Autotopagnosia
Explanation:
Prosopagnosia represents a specialized form of visual agnosia affecting face perception. Visual acuity and fields remain intact. Agraphesthesia involves inability to recognize symbols traced on skin, while autotopagnosia refers to inability to localize body parts. The correct answer is Prosopagnosia. This deficit can be congenital or acquired, with acquired cases often following posterior cerebral artery strokes.
6. Which lobe is primarily involved in facial recognition?
A) Frontal
B) Temporal
C) Parietal
D) Occipital
Explanation:
The temporal lobe, particularly the inferior temporal cortex and fusiform gyrus, plays a central role in facial recognition. While the occipital lobe handles initial visual processing, detailed identification occurs in temporal association areas. Frontal and parietal lobes contribute to higher cognitive and sensory processing, but not specifically facial identity. Therefore, the correct answer is Temporal. Lesions here cause prosopagnosia, highlighting its functional importance.
7. A patient with right temporal lobe damage may show:
A) Difficulty reading
B) Prosopagnosia
C) Difficulty writing
D) Aphasia
Explanation:
Right temporal lobe lesions often lead to deficits in face recognition due to involvement of the right fusiform face area. Left temporal lesions more commonly cause language disturbances like aphasia. Reading and writing are mainly parietal–occipitotemporal functions. Thus, the correct answer is Prosopagnosia. Patients often rely on voice or contextual clues to identify known individuals.
8. Prosopagnosia is best classified as:
A) Motor disorder
B) Language disorder
C) Visual agnosia
D) Attention deficit
Explanation:
Prosopagnosia is a selective visual agnosia in which patients cannot recognize faces despite intact basic visual function. It is not related to motor or language deficits, nor is it an attention deficit disorder. It specifically reflects impaired higher-order processing in the temporo-occipital association cortex. Thus, the correct answer is Visual agnosia. Patients may still identify individuals using other sensory cues.
9. A patient identifies objects but cannot recognize his own reflection. This is due to damage in:
A) Prefrontal cortex
B) Primary visual cortex
C) Inferior temporal cortex
D) Angular gyrus
Explanation:
The inferior temporal cortex, particularly the fusiform face area, is essential for recognizing faces including one’s own. Primary visual cortex damage causes blindness, angular gyrus injury produces reading/writing disorders, and prefrontal lesions affect executive functions. Therefore, the correct answer is Inferior temporal cortex. This condition illustrates precise cortical localization of complex recognition functions.
10. Prosopagnosia is commonly associated with infarction of which artery?
A) Anterior cerebral artery
B) Middle cerebral artery
C) Posterior cerebral artery
D) Basilar artery
Explanation:
The posterior cerebral artery supplies the occipital and inferotemporal regions including the fusiform gyrus. Infarction here can impair facial recognition and produce prosopagnosia. MCA strokes affect language and motor areas, while ACA strokes affect medial frontal lobes. Basilar artery strokes produce brainstem deficits. Therefore, the correct answer is Posterior cerebral artery. Diagnosis is supported by neuroimaging and neuropsychological testing.
11. A patient with degenerative dementia shows progressive facial recognition impairment. Most likely diagnosis?
A) Alzheimer’s disease
B) Parkinson’s disease
C) Huntington’s disease
D) ALS
Explanation:
Alzheimer’s disease frequently affects temporo-parietal association cortices, including areas related to visual recognition such as the fusiform gyrus. As degeneration progresses, patients may develop prosopagnosia. Parkinson’s and Huntington’s primarily affect motor circuits, and ALS affects motor neurons. Thus, the correct answer is Alzheimer’s disease. Prosopagnosia in dementia highlights involvement of higher-order visual processing circuits. Chapter: Neurophysiology; Topic: Sensory Receptors; Subtopic: Slowly Adapting Cutaneous Receptors
KEYWORD DEFINITIONS
• Mechanoreceptors – Receptors responding to mechanical deformation
• Slowly adapting receptors – Detect sustained pressure and continuous stimuli
• Ruffini endings – Slowly adapting receptors sensing stretch and sustained pressure
• Merkel discs – Slowly adapting receptors sensing texture and steady pressure
• Meissner & Pacinian corpuscles – Rapidly adapting receptors for light touch/vibration
Lead Question – 2015
1. Which of the following receptor is stimulated by sustained pressure?
A) Ruffini's end organ
B) Merkel's disc
C) Hair cells
D) Meissner Corpuscles
Explanation:
Ruffini’s end organs are slowly adapting mechanoreceptors that respond to sustained pressure and skin stretch. They continue firing as long as the stimulus persists, unlike rapidly adapting receptors such as Meissner and Pacinian corpuscles, which respond mainly at stimulus onset and offset. Merkel discs also respond to steady pressure but are more involved in texture discrimination. Hair cells function in hearing and balance rather than somatosensation. Therefore, the correct answer is Ruffini’s end organ. These receptors play an important role in proprioception and grip modulation during sustained mechanical loading.
2. Which receptor is MOST sensitive to low-frequency vibration?
A) Ruffini ending
B) Meissner corpuscle
C) Merkel disc
D) Pacinian corpuscle
Explanation:
Meissner corpuscles are rapidly adapting receptors sensitive to low-frequency vibration (30–50 Hz) and light touch. They are abundant in glabrous skin such as fingertips and are essential for grip control and tactile discrimination. Pacinian corpuscles detect high-frequency vibration, Merkel discs detect texture and steady pressure, and Ruffini endings detect stretch. Thus, the correct answer is Meissner corpuscle. Clinical testing of vibration sense using a tuning fork helps identify dorsal column or peripheral nerve dysfunction.
3. A patient with posterior column lesion presents with impaired vibration sense. Which receptor’s input is mainly affected?
A) Meissner corpuscle
B) Pacinian corpuscle
C) Ruffini ending
D) Free nerve endings
Explanation:
Pacinian corpuscles provide high-frequency vibration sensation and transmit signals through large A-beta fibers into the dorsal column–medial lemniscal pathway. Damage to this pathway results in loss of vibration, proprioception, and fine touch. Meissner corpuscles also contribute but mainly to low-frequency vibration. Free nerve endings transmit pain and temperature, not vibration. Thus, the correct answer is Pacinian corpuscle. Posterior column pathology is characteristic in conditions like tabes dorsalis and vitamin B12 deficiency.
4. Which receptor detects fine texture and detailed form discrimination?
A) Merkel disc
B) Ruffini ending
C) Meissner corpuscle
D) Pacinian corpuscle
Explanation:
Merkel discs are slowly adapting receptors specialized for detecting texture, shape, and fine spatial details. They respond to sustained pressure and are located in the basal epidermis. Meissner corpuscles detect low-frequency vibration, Pacinian corpuscles detect high-frequency vibration, and Ruffini endings detect stretch. Hence, the correct answer is Merkel disc. These receptors are crucial for reading Braille and recognizing object surfaces during tactile exploration.
5. A diabetic patient shows early loss of vibration sense. Which fibers are damaged first?
A) A-delta fibers
B) C fibers
C) A-beta fibers
D) B fibers
Explanation:
A-beta fibers are large, myelinated sensory fibers that carry vibration and fine touch information from mechanoreceptors such as Pacinian and Meissner corpuscles. These fibers are commonly affected early in diabetic neuropathy, leading to diminished vibration perception. A-delta fibers carry fast pain, C fibers carry slow pain and temperature, and B fibers are autonomic. Therefore, the correct answer is A-beta fibers. Tuning fork testing is a sensitive way to detect early large-fiber neuropathy.
6. Which type of receptor is MOST likely to show rapid adaptation during continuous stimulation?
A) Merkel disc
B) Ruffini ending
C) Pacinian corpuscle
D) Nociceptor
Explanation:
Pacinian corpuscles exhibit extremely rapid adaptation, responding only to the onset and offset of mechanical stimuli. This feature allows them to detect transient changes such as vibration. Merkel discs and Ruffini endings are slowly adapting, maintaining continuous firing during sustained stimulation, while nociceptors respond to painful stimuli rather than vibration or pressure. Thus, the correct answer is Pacinian corpuscle. Rapidly adapting receptors are essential in motion detection and discriminating dynamic mechanical changes.
7. A patient with difficulty perceiving object slippage in the hand likely has dysfunction of which receptor?
A) Pacinian corpuscle
B) Meissner corpuscle
C) Merkel disc
D) Ruffini ending
Explanation:
Meissner corpuscles are rapidly adapting receptors that detect low-frequency vibration and slip sensations, providing crucial feedback for grip modulation. Dysfunction impairs the ability to adjust finger pressure, leading to poor handling of objects. Pacinian corpuscles detect high-frequency vibration, Merkel discs detect texture, and Ruffini endings detect stretch. Therefore, the correct answer is Meissner corpuscle. Damage to these receptors significantly affects hand dexterity.
8. Which receptor primarily detects stretching of skin and joints?
A) Merkel disc
B) Ruffini ending
C) Meissner corpuscle
D) Pacinian corpuscle
Explanation:
Ruffini endings are slowly adapting mechanoreceptors that respond to sustained stretch of the skin and joint capsules. They provide important proprioceptive information regarding finger position and hand configuration. Merkel discs detect steady pressure, Meissner corpuscles detect light touch and low-frequency vibration, and Pacinian corpuscles detect high-frequency vibration. The correct answer is Ruffini ending. These receptors are essential in maintaining grip force and coordinated movement.
9. A patient retains pain and temperature sensation but loses vibration and joint position sense. Which tract is intact?
A) Dorsal column
B) Spinothalamic tract
C) Spinocerebellar tract
D) Corticospinal tract
Explanation:
The spinothalamic tract carries pain and temperature sensations. Because these sensations are preserved while vibration and proprioception are lost, the dorsal column pathway is affected while the spinothalamic tract remains intact. Spinocerebellar and corticospinal tracts carry proprioceptive and motor signals respectively. Thus, the correct answer is Spinothalamic tract. This sensory dissociation helps localize spinal lesions clinically.
10. Which receptor responds best to high-frequency vibration applied over bony prominences?
A) Merkel disc
B) Meissner corpuscle
C) Pacinian corpuscle
D) Free nerve ending
Explanation:
Pacinian corpuscles detect high-frequency vibration (200–300 Hz) and are located deep in fascia, joints, and subcutaneous tissues, making them ideal for vibration detection near bones. Meissner corpuscles detect low-frequency vibration, Merkel discs detect texture, and free nerve endings detect pain/temperature. Therefore, the correct answer is Pacinian corpuscle. Clinical tuning fork tests mainly assess Pacinian-mediated pathways.
11. A 70-year-old with B12 deficiency shows severe loss of vibration sense. Which pathway is damaged?
A) Spinothalamic tract
B) Reticulospinal tract
C) Dorsal column–medial lemniscal pathway
D) Vestibulospinal tract
Explanation:
The dorsal column–medial lemniscal pathway carries proprioception, fine touch, and vibration. Vitamin B12 deficiency causes demyelination of this pathway, producing deficits like vibration loss, ataxia, and impaired joint position sense. Spinothalamic tract carries pain and temperature, vestibulospinal modulates posture, and reticulospinal influences muscle tone. Therefore, the correct answer is Dorsal column–medial lemniscal pathway. Early detection is essential to prevent irreversible neurological damage.
Chapter: Neurophysiology; Topic: Sensory Receptors; Subtopic: Adaptation & Vibration Receptor Physiology
KEYWORD DEFINITIONS
• Mechanoreceptors – Sensory receptors responding to mechanical deformation
• Rapidly adapting receptors – Respond quickly to stimulus onset; detect vibration
• Slowly adapting receptors – Detect sustained pressure and stretch
• Pacinian corpuscle – Deep, rapidly adapting receptor for high-frequency vibration
• Meissner corpuscle – Rapidly adapting receptor for low-frequency vibration
Lead Question – 2015
1. Vibrations are detected by which types of receptors?
A) Slowly adapting
B) Rapidly adapting
C) Non-adapting
D) None of the above
Explanation:
Vibration is detected by rapidly adapting mechanoreceptors, primarily Pacinian and Meissner corpuscles. These receptors respond quickly at the start and end of a stimulus, making them ideal for sensing vibratory frequencies. Slowly adapting receptors detect sustained pressure and stretch, while non-adapting receptors are not typical in the somatosensory system. Therefore, the correct answer is Rapidly adapting. Clinically, vibration testing using a tuning fork evaluates large-fiber dorsal column integrity, which is crucial in diagnosing neuropathies and posterior column disorders such as tabes dorsalis and vitamin B12 deficiency.
2. Which receptor detects high-frequency vibration (≈250 Hz)?
A) Merkel disc
B) Ruffini ending
C) Pacinian corpuscle
D) Free nerve endings
Explanation:
Pacinian corpuscles are deep, rapidly adapting mechanoreceptors specialized for detecting high-frequency vibration around 200–300 Hz. Their lamellated structure permits rapid response to transient mechanical stimuli. Merkel discs detect pressure and texture, Ruffini endings detect skin stretch, and free nerve endings respond to pain and temperature. Therefore, the correct answer is Pacinian corpuscle. High-frequency vibration assessment is vital in early detection of peripheral neuropathies, especially diabetic neuropathy.
3. A patient with dorsal column damage will MOST likely lose which sensation first?
A) Pain
B) Temperature
C) Vibration
D) Crude touch
Explanation:
The dorsal column–medial lemniscal pathway carries fine touch, vibration, and proprioception. Among these, vibration sense is often the earliest to be impaired, especially in neuropathies or spinal cord lesions. Pain and temperature are carried by the spinothalamic tract and remain intact initially. Crude touch can also be preserved due to overlapping pathways. Thus, the correct answer is Vibration. Testing with a 128 Hz tuning fork is a crucial bedside neurological examination tool.
4. Low-frequency vibration (30–50 Hz) is detected by which receptor?
A) Pacinian corpuscle
B) Merkel disc
C) Meissner corpuscle
D) Ruffini ending
Explanation:
Meissner corpuscles are rapidly adapting receptors located in glabrous skin, particularly fingertips, and are sensitive to low-frequency vibration and light touch. Pacinian corpuscles detect high-frequency vibration, Merkel discs detect texture and sustained pressure, and Ruffini endings detect skin stretch. Therefore, the correct answer is Meissner corpuscle. These receptors are essential for tasks involving fine tactile discrimination and grip modulation.
5. A diabetic patient presents with decreased vibration sense. Which fibers are most likely affected?
A) C fibers
B) A-delta fibers
C) A-beta fibers
D) B fibers
Explanation:
A-beta fibers are large, myelinated sensory fibers that transmit vibration and fine touch from mechanoreceptors such as Pacinian and Meissner corpuscles. In diabetic neuropathy, these fibers are affected early, causing loss of vibration sense before pain and temperature deficits appear. C fibers and A-delta fibers carry nociceptive and thermal sensations, while B fibers belong to the autonomic system. The correct answer is A-beta fibers. Vibration sense testing helps detect early large-fiber neuropathy.
6. Which type of receptor exhibits very rapid adaptation and is ideal for detecting sudden mechanical changes?
A) Slowly adapting receptor
B) Non-adapting receptor
C) Rapidly adapting receptor
D) Thermoreceptors
Explanation:
Rapidly adapting receptors discharge quickly at the onset and offset of a stimulus, making them highly effective for detecting dynamic changes such as vibration and movement. Pacinian corpuscles and Meissner corpuscles fall into this category. Slowly adapting receptors detect sustained pressure, while thermoreceptors respond to temperature. Non-adapting receptors are not typical in somatosensory physiology. Therefore, the correct answer is Rapidly adapting receptor. Their function is essential in evaluating dorsal column sensory integrity.
7. A 65-year-old presents with inability to perceive vibration on toes but intact pain sensation. Which pathway is preserved?
A) Dorsal column pathway
B) Spinocerebellar pathway
C) Spinothalamic pathway
D) Vestibulospinal pathway
Explanation:
Pain and temperature sensations remain intact through the spinothalamic pathway. Loss of vibration sense indicates dorsal column involvement. Since pain sensation is preserved, the spinothalamic tract must be intact. Therefore, the correct answer is Spinothalamic pathway. This dissociation is clinically useful in diagnosing neuropathies and spinal cord lesions affecting selective sensory modalities.
8. Which receptor detects texture discrimination and form recognition?
A) Pacinian corpuscle
B) Meissner corpuscle
C) Merkel disc
D) Ruffini ending
Explanation:
Merkel discs are slowly adapting receptors specialized for detecting texture, form, and fine spatial details. They respond to sustained pressure rather than vibration. Meissner corpuscles detect low-frequency vibration, Pacinian corpuscles detect high-frequency vibration, and Ruffini endings detect stretch. Hence, the correct answer is Merkel disc. These receptors are vital for tactile precision tasks such as Braille reading.
9. A patient cannot detect high-frequency vibration on the ankle. Which receptor is most likely impaired?
A) Meissner corpuscle
B) Ruffini ending
C) Pacinian corpuscle
D) Free nerve ending
Explanation:
Pacinian corpuscles are located deep in subcutaneous tissues, joints, and fascia and are responsible for detecting high-frequency vibration. Meissner corpuscles detect low-frequency vibration, Ruffini endings detect stretch, and free nerve endings detect pain and temperature. Therefore, the correct answer is Pacinian corpuscle. Clinically, reduced vibration over bony prominences indicates large-fiber neuropathy.
10. Which receptor type is MOST closely associated with grip modulation and tactile feedback during object handling?
A) Ruffini ending
B) Merkel disc
C) Meissner corpuscle
D) Pacinian corpuscle
Explanation:
Meissner corpuscles provide rapid feedback during grip adjustments because of their sensitivity to low-frequency vibration and slip detection. They are crucial for maintaining precise hand control. Ruffini endings detect stretch, Merkel discs detect form and texture, and Pacinian corpuscles detect deep high-frequency vibration. Therefore, the correct answer is Meissner corpuscle. Dysfunction of these receptors compromises fine motor control.
11. A patient with severe B12 deficiency shows complete loss of vibration sense. Which anatomical pathway is affected?
A) Spinothalamic tract
B) Dorsal column–medial lemniscal pathway
C) Reticulospinal tract
D) Corticospinal tract
Explanation:
Vitamin B12 deficiency leads to subacute combined degeneration, affecting the dorsal columns, which carry vibration, proprioception, and fine touch. Spinothalamic pathway carries pain and temperature, while reticulospinal and corticospinal tracts are motor pathways. Therefore, the correct answer is Dorsal column–medial lemniscal pathway. Loss of vibration sense is an early and sensitive sign of dorsal column dysfunction in B12 deficiency.
Chapter: Neurophysiology; Topic: Sensory Receptors; Subtopic: Mechanoreceptors – Vibration Sensation
KEYWORD DEFINITIONS
• Mechanoreceptors – Detect mechanical deformation
• Deep receptors – Located in deeper tissues; detect vibration and pressure
• Pacinian corpuscle – Primary receptor for vibration
• Meissner corpuscle – Detects light touch and low-frequency vibration
• Posterior column pathway – Carries vibration and proprioception
Lead Question – 2015
1. Vibration sense is detected by?
A) Nociceptors
B) Deep receptors
C) Superficial receptors
D) None of the above
Explanation:
Vibration sense is primarily detected by deep receptors, especially Pacinian corpuscles located in deeper layers of the skin, joints, and fascia. These receptors are rapidly adapting mechanoreceptors sensitive to high-frequency vibration. Nociceptors detect pain, not vibration, while superficial receptors detect touch and texture rather than vibration. Therefore, the correct answer is Deep receptors. In clinical practice, vibration sense assessment helps evaluate the integrity of the dorsal column–medial lemniscal pathway, which is often impaired in conditions such as diabetic neuropathy and vitamin B12 deficiency.
2. Which receptor is MOST responsible for high-frequency vibration detection?
A) Merkel disc
B) Ruffini ending
C) Pacinian corpuscle
D) Free nerve endings
Explanation:
Pacinian corpuscles are deep, rapidly adapting mechanoreceptors specifically tuned to detect high-frequency vibration around 200–300 Hz. Their onion-like capsule structure enables rapid response to changes in mechanical pressure. Merkel discs detect texture, Ruffini endings detect stretch, and free nerve endings detect pain and temperature. Thus, the correct answer is Pacinian corpuscle. This receptor plays a key clinical role in assessing vibration sense during neurological examination using a tuning fork, especially in neuropathies and dorsal column lesions.
3. A patient with posterior column damage loses vibration sense. Which tract is affected?
A) Spinothalamic tract
B) Spinocerebellar tract
C) Dorsal column–medial lemniscal pathway
D) Reticulospinal tract
Explanation:
The dorsal column–medial lemniscal pathway carries vibration, fine touch, and proprioception. Damage to this pathway leads to loss of vibration sense, gait imbalance, and impaired position sense. Spinothalamic tract carries pain and temperature, spinocerebellar tract carries unconscious proprioception, and reticulospinal tract mediates motor modulation. Therefore, the correct answer is Dorsal column–medial lemniscal pathway. This distinction is vital in diagnosing neurological disorders such as tabes dorsalis or B12 deficiency.
4. Low-frequency vibration (30–50 Hz) is best detected by?
A) Pacinian corpuscle
B) Meissner corpuscle
C) Ruffini ending
D) Merkel disc
Explanation:
Meissner corpuscles are rapidly adapting superficial mechanoreceptors highly sensitive to low-frequency vibration and light touch. They are abundant in fingertips and glabrous skin, enabling tactile discrimination. Pacinian corpuscles detect high-frequency vibration, Ruffini endings detect stretch, and Merkel discs detect sustained pressure and texture. Thus, the correct answer is Meissner corpuscle. These receptors play a central role in fine motor tasks and early detection of surface changes.
5. A diabetic patient presents with impaired vibration sense. Which structure is most likely damaged?
A) Spinothalamic tract fibers
B) A-beta fibers
C) A-delta fibers
D) C fibers
Explanation:
A-beta fibers are large, myelinated sensory fibers responsible for transmitting vibration and fine touch sensation from mechanoreceptors such as Pacinian and Meissner corpuscles. These fibers are highly vulnerable in diabetic neuropathy, leading to early loss of vibration sense. Spinothalamic fibers carry pain and temperature, A-delta fibers carry fast pain, and C fibers carry slow pain. Therefore, the correct answer is A-beta fibers. Assessment using a tuning fork is a crucial clinical tool for early neuropathy detection.
6. Which receptor detects sustained pressure and contributes to proprioception?
A) Pacinian corpuscle
B) Ruffini ending
C) Meissner corpuscle
D) Hair follicle receptor
Explanation:
Ruffini endings are slowly adapting receptors that detect skin stretch and sustained pressure. They provide continuous feedback regarding finger position and grip modulation. Pacinian corpuscles detect vibration, Meissner corpuscles detect light touch and low-frequency vibration, and hair follicle receptors detect hair movement. Thus, the correct answer is Ruffini ending. These receptors are essential for proprioceptive accuracy during tasks requiring hand stability.
7. A 55-year-old alcoholic patient has loss of vibration and position sense. What deficiency is most likely?
A) Folate
B) Vitamin A
C) Vitamin B12
D) Vitamin C
Explanation:
Vitamin B12 deficiency causes demyelination of the dorsal columns, resulting in loss of vibration sense, proprioceptive deficits, and ataxic gait. Folate deficiency affects red cell production but not dorsal columns, while vitamins A and C do not cause such neurological deficits. Hence, the correct answer is Vitamin B12. Early diagnosis is essential because prolonged deficiency leads to irreversible neurological damage.
8. Which receptor is responsible for detecting texture and form discrimination?
A) Merkel disc
B) Pacinian corpuscle
C) Ruffini ending
D) Meissner corpuscle
Explanation:
Merkel discs are slowly adapting mechanoreceptors specialized for fine touch and texture discrimination. Located in the basal epidermis, they respond to sustained pressure and help identify object shape. Pacinian corpuscles detect vibration, Ruffini endings detect skin stretch, and Meissner corpuscles detect light touch. Thus, the correct answer is Merkel disc. These receptors are essential for reading Braille and performing detailed tactile tasks.
9. A patient cannot detect vibration but has intact pain and temperature. Which tract remains functional?
A) Dorsal column pathway
B) Corticospinal tract
C) Spinothalamic tract
D) Vestibulospinal tract
Explanation:
Pain and temperature sensations are carried by the spinothalamic tract, whereas vibration and proprioception are carried by the dorsal column pathway. If vibration sense is absent but pain and temperature remain intact, the spinothalamic tract is functioning. Therefore, the correct answer is Spinothalamic tract. This sensory dissociation helps localize lesions within the spinal cord.
10. Which receptor is primarily responsible for detecting high-frequency vibration applied to joints and deep tissues?
A) Meissner corpuscle
B) Pacinian corpuscle
C) Ruffini ending
D) Free nerve endings
Explanation:
Pacinian corpuscles are deep mechanoreceptors located in ligaments, joints, and subcutaneous tissues, making them ideal for detecting high-frequency vibration. Their rapid adaptation allows them to respond immediately to mechanical changes. Meissner corpuscles detect low-frequency vibration, Ruffini endings detect stretch, and free nerve endings detect pain and temperature. Thus, the correct answer is Pacinian corpuscle. These receptors are essential in proprioceptive feedback and vibration sense testing.
11. Which receptor is most affected when high-frequency vibration (e.g., tuning fork) cannot be perceived on examination?
A) Meissner corpuscle
B) Merkel disc
C) Pacinian corpuscle
D) Ruffini ending
Explanation:
High-frequency vibration is detected by Pacinian corpuscles, which are located deep within tissues and respond rapidly to mechanical changes. Meissner corpuscles detect low-frequency vibration, Merkel discs detect pressure and texture, and Ruffini endings detect stretch. Therefore, the correct answer is Pacinian corpuscle. In clinical practice, loss of vibration sense indicates dorsal column or peripheral nerve pathology, making this receptor highly relevant in neurological assessment.
Chapter: Neurophysiology; Topic: Sensory Receptors; Subtopic: Mechanoreceptors
KEYWORD DEFINITIONS
• Mechanoreceptors – Detect mechanical deformation
• Pacinian corpuscle – Responds to vibration and deep pressure
• Meissner corpuscle – Detects light touch
• Merkel disc – Detects pressure and texture
• Ruffini ending – Detects stretch and sustained pressure
Lead Question – 2015
1. Vibration sense is detected by which type of receptor?
A) Merkel's disc
B) Ruffini's end organ
C) Pacinian corpuscle
D) Meissner's corpuscle
Explanation:
Pacinian corpuscles are rapidly adapting mechanoreceptors specialized for detecting high-frequency vibration and deep pressure. Their onion-like lamellar structure allows them to respond quickly to changes in mechanical stimuli. In contrast, Merkel discs detect texture, Meissner corpuscles detect light touch, and Ruffini endings detect skin stretch. Therefore, the correct answer is Pacinian corpuscle. These receptors play an essential role in proprioception and fine manipulation tasks requiring vibration sensing. Damage to deep sensory pathways can reduce vibration sense, making this receptor clinically relevant in neurological examinations.
2. Which receptor adapts the fastest among cutaneous mechanoreceptors?
A) Merkel disc
B) Ruffini ending
C) Pacinian corpuscle
D) Free nerve endings
Explanation:
Pacinian corpuscles show extremely rapid adaptation due to their layered lamellar structure, enabling them to detect high-frequency vibration and abrupt mechanical changes. Merkel discs and Ruffini endings are slowly adapting, while free nerve endings are polymodal and variable in adaptation. Because Pacinian corpuscles stop firing quickly once the stimulus stabilizes, they are ideal for detecting transient mechanical events. Thus, the correct answer is Pacinian corpuscle. Understanding receptor adaptation helps clinicians evaluate sensory deficits related to neuropathy or dorsal column lesions.
3. A patient with posterior column lesion loses vibration sense. Which fiber type carries this sensation?
A) A-delta fibers
B) C fibers
C) A-beta fibers
D) B fibers
Explanation:
Vibration sense is transmitted via large-diameter, heavily myelinated A-beta fibers that rapidly conduct mechanosensory information from receptors such as Pacinian and Meissner corpuscles. These fibers ascend in the dorsal columns to the medulla. A-delta fibers carry fast pain, C fibers carry slow pain and temperature, and B fibers are autonomic. Therefore, the correct answer is A-beta fibers. Loss of vibration sense is a classical feature of dorsal column dysfunction, as seen in vitamin B12 deficiency or tabes dorsalis.
4. Which receptor is primarily responsible for detecting skin stretch during grasping movements?
A) Meissner corpuscle
B) Ruffini ending
C) Pacinian corpuscle
D) Hair follicle receptor
Explanation:
Ruffini endings are slowly adapting mechanoreceptors sensitive to skin stretch and joint movement. They provide critical feedback during handgrip control, helping maintain steady pressure. Meissner corpuscles detect light touch, Pacinian corpuscles detect vibration, and hair follicle receptors detect hair movement. Thus, the correct answer is Ruffini ending. These receptors contribute significantly to proprioception and are often assessed indirectly during neurological examinations of deep sensory functions.
5. A diabetic patient presents with reduced vibration sense in the feet. Which pathway is compromised?
A) Spinothalamic tract
B) Corticospinal tract
C) Dorsal column–medial lemniscal pathway
D) Reticulospinal tract
Explanation:
The dorsal column–medial lemniscal pathway carries fine touch, vibration, and proprioception using large myelinated fibers. In diabetic neuropathy, early damage to these fibers leads to reduced vibration perception. The spinothalamic tract carries pain and temperature, while corticospinal and reticulospinal tracts are motor pathways. Therefore, the correct answer is Dorsal column–medial lemniscal pathway. Assessing vibration sense using a tuning fork is a key clinical test in diabetic neuropathy evaluation.
6. Which receptor is MOST sensitive to low-frequency vibration (e.g., 30–50 Hz)?
A) Pacinian corpuscle
B) Meissner corpuscle
C) Merkel disc
D) Ruffini ending
Explanation:
Meissner corpuscles are rapidly adapting receptors responsible for detecting low-frequency vibration and light touch, especially in glabrous skin. Pacinian corpuscles detect high-frequency vibration, Merkel discs detect sustained pressure, and Ruffini endings detect stretch. Thus, the correct answer is Meissner corpuscle. These receptors are abundant in fingertips and play a major role in tactile discrimination and grip control.
7. A 40-year-old presents with impaired proprioception and vibration sense. Which vitamin deficiency is most likely?
A) Vitamin A
B) Vitamin B12
C) Vitamin D
D) Vitamin K
Explanation:
Vitamin B12 deficiency leads to demyelination of the dorsal columns, causing loss of vibration sense, proprioception, and gait disturbances. Vitamins A, D, and K do not typically affect dorsal column pathways. Therefore, the correct answer is Vitamin B12. Early detection is essential because neurological deficits can become irreversible if treatment is delayed.
8. Merkel discs are best known for detecting which sensation?
A) Vibration
B) Skin stretch
C) Deep pressure
D) Texture and form
Explanation:
Merkel discs are slowly adapting mechanoreceptors specialized for detecting texture, shape, and fine spatial details, enabling high-resolution tactile discrimination. They respond to sustained pressure but not to vibration. Thus, the correct answer is Texture and form. Their dysfunction can impair fine touch perception, affecting tasks requiring precision such as reading Braille.
9. A patient reports inability to detect vibration but retains crude touch. Which spinal tract is intact?
A) Dorsal column
B) Spinothalamic tract
C) Spinoreticular tract
D) Dorsal spinocerebellar tract
Explanation:
Crude touch is carried by the spinothalamic tract, while vibration sense is transmitted via the dorsal columns. Loss of vibration with preserved crude touch suggests dorsal column dysfunction with intact spinothalamic function. Therefore, the correct answer is Spinothalamic tract. This distinction is clinically valuable in assessing sensory loss patterns in spinal cord lesions.
10. Which receptor is MOST abundant in the fingertips and essential for fine tactile discrimination?
A) Pacinian corpuscle
B) Meissner corpuscle
C) Ruffini ending
D) Free nerve endings
Explanation:
Meissner corpuscles are concentrated in glabrous skin, especially fingertips, where they detect light touch and fine tactile detail. Pacinian corpuscles detect vibration, Ruffini endings detect stretch, and free nerve endings detect pain and temperature. Thus, the correct answer is Meissner corpuscle. Their density makes fingertips the most sensitive region for performing delicate tactile tasks.
11. A patient cannot perceive high-frequency vibration applied to the skin. Which receptor is dysfunctional?
A) Merkel disc
B) Meissner corpuscle
C) Pacinian corpuscle
D) Hair follicle receptor
Explanation:
High-frequency vibration (around 250 Hz) is detected primarily by Pacinian corpuscles, which are rapidly adapting deep mechanoreceptors. Meissner corpuscles detect low-frequency vibration, Merkel discs detect pressure, and hair follicle receptors detect hair movement. Therefore, the correct answer is Pacinian corpuscle. Their loss signals dysfunction in deep mechanoreceptor pathways, often associated with peripheral neuropathy.
Topic: Nervous System; Subtopic: Pain Pathways and Nerve Fiber Classification
Keyword Definitions:
• Pain: An unpleasant sensory and emotional experience associated with tissue injury.
• Aδ fibers: Thinly myelinated nerve fibers responsible for fast, sharp, localized pain.
• C fibers: Unmyelinated fibers transmitting slow, dull, burning pain.
• Spinothalamic tract: Ascending pathway transmitting pain and temperature sensations.
• Polymodal nociceptors: Sensory receptors responding to mechanical, thermal, and chemical stimuli.
• Referred pain: Pain perceived at a site distant from its origin due to shared neural pathways.
Lead Question - 2015
Pain is carried by which nerve fibers?
a) Aα, Aβ
b) Aα, Aγ
c) Aδ, C
d) Aγ, C
Explanation (Answer: c) Aδ, C)
Pain sensation is transmitted by two types of nerve fibers — Aδ and C fibers. Aδ fibers are thinly myelinated and convey fast, sharp, well-localized pain, while C fibers are unmyelinated and carry slow, burning, and poorly localized pain. These fibers synapse in the dorsal horn of the spinal cord, primarily in laminae I and II (substantia gelatinosa). Aδ fibers activate the neospinothalamic tract, whereas C fibers activate the paleospinothalamic tract. Together, they ensure both immediate withdrawal and prolonged awareness of pain.
1. Which tract carries fast pain sensations to the brain?
a) Neospinothalamic tract
b) Spinocerebellar tract
c) Dorsal column
d) Spinoreticular tract
Explanation (Answer: a) Neospinothalamic tract)
The neospinothalamic tract transmits fast pain impulses from Aδ fibers. These signals ascend directly to the thalamus, providing sharp, well-localized pain sensations. This pathway allows quick withdrawal from harmful stimuli and contributes to the conscious perception of acute pain.
2. Which of the following fibers are unmyelinated?
a) Aα
b) Aβ
c) C fibers
d) Aδ fibers
Explanation (Answer: c) C fibers)
C fibers are unmyelinated and conduct impulses at 0.4–2 m/s. They are responsible for transmitting dull, aching, and burning pain. Due to lack of myelin, these fibers conduct more slowly than Aδ fibers. Their prolonged activation contributes to the persistence of chronic pain sensations.
3. Aδ fibers carry which type of pain?
a) Burning pain
b) Sharp, pricking pain
c) Diffuse pain
d) Chronic pain
Explanation (Answer: b) Sharp, pricking pain)
Aδ fibers transmit fast, pricking, and well-localized pain sensations through rapid conduction. They are thinly myelinated fibers that allow the brain to perceive pain quickly and accurately, helping initiate protective withdrawal reflexes against harmful stimuli like pinpricks or burns.
4. The neurotransmitter released by Aδ fibers in the dorsal horn is:
a) Serotonin
b) Glutamate
c) Substance P
d) GABA
Explanation (Answer: b) Glutamate)
Glutamate is the primary excitatory neurotransmitter released by Aδ fibers in the dorsal horn. It produces fast excitatory postsynaptic potentials (EPSPs), mediating acute, localized pain. In contrast, Substance P released by C fibers causes prolonged depolarization and sustains slow pain responses.
5. Which of the following tracts carries slow, burning pain?
a) Neospinothalamic
b) Paleospinothalamic
c) Spinocerebellar
d) Dorsal column
Explanation (Answer: b) Paleospinothalamic)
The paleospinothalamic tract conveys slow, burning, and aching pain through unmyelinated C fibers. This pathway involves multiple synapses, projecting to the brainstem, reticular formation, and limbic system, contributing to the emotional and autonomic responses to pain stimuli.
6. A 30-year-old man experiences dull, diffuse abdominal pain due to appendicitis. The fiber type involved is:
a) Aα
b) Aβ
c) Aδ
d) C fibers
Explanation (Answer: d) C fibers)
Visceral pain, like the dull, aching pain of appendicitis, is transmitted by unmyelinated C fibers. These fibers carry slow pain signals to the brain via the spinoreticular pathway, resulting in diffuse, poorly localized sensations associated with autonomic responses like nausea.
7. The lamina of the spinal cord that receives pain fibers is:
a) Lamina I and II
b) Lamina III
c) Lamina V only
d) Lamina VII
Explanation (Answer: a) Lamina I and II)
Pain fibers synapse in Lamina I (marginal zone) and Lamina II (substantia gelatinosa) of the dorsal horn. These regions process nociceptive information before it ascends via the spinothalamic tracts. Neurotransmitters such as glutamate and substance P play vital roles here.
8. Loss of pain and temperature sensation on the contralateral side of the body is due to lesion in:
a) Spinocerebellar tract
b) Dorsal column
c) Lateral spinothalamic tract
d) Corticospinal tract
Explanation (Answer: c) Lateral spinothalamic tract)
A lesion in the lateral spinothalamic tract results in contralateral loss of pain and temperature below the level of injury because the tract crosses within one or two segments of the spinal cord. This tract carries impulses transmitted by Aδ and C fibers to the thalamus.
9. Which ion influx is responsible for depolarization in pain fibers?
a) Calcium
b) Sodium
c) Chloride
d) Potassium
Explanation (Answer: b) Sodium)
Depolarization in pain fibers occurs due to the influx of sodium ions through voltage-gated sodium channels. This initiates the action potential that transmits pain impulses to the spinal cord. Many analgesic drugs (like local anesthetics) work by blocking these sodium channels.
10. A patient with diabetic neuropathy has burning foot pain. The affected fibers are:
a) Aδ fibers
b) C fibers
c) Aβ fibers
d) Aα fibers
Explanation (Answer: b) C fibers)
Burning neuropathic pain in diabetic neuropathy results from injury to unmyelinated C fibers. These fibers are sensitive to metabolic damage from chronic hyperglycemia, leading to hyperalgesia, allodynia, and spontaneous burning sensations characteristic of neuropathic pain syndromes.
11. Which of the following neurotransmitters mediates prolonged, slow pain?
a) Dopamine
b) GABA
c) Substance P
d) Serotonin
Explanation (Answer: c) Substance P)
Substance P is released by C fibers in response to noxious stimuli and mediates slow, burning pain. It acts on neurokinin-1 (NK1) receptors, causing sustained depolarization and enhanced pain perception. Chronic pain conditions often involve increased Substance P activity in the dorsal horn.
Topic: Nervous System; Subtopic: Pain Pathways and Transmission
Keyword Definitions:
• Pain: An unpleasant sensory and emotional experience due to actual or potential tissue damage.
• Unspecified pain pathway: A slower, polysynaptic pain pathway responsible for deep, diffuse pain sensations.
• Visceral pain: Pain originating from internal organs, poorly localized and often referred.
• Spinoreticular tract: Ascending tract carrying diffuse pain sensations to the brainstem.
• Neuropathic pain: Pain resulting from nerve injury, burning or shooting in nature.
• Somatic pain: Pain arising from skin, muscles, and joints, usually sharp and well localized.
Lead Question - 2015
Unspecified pain pathway is for?
a) Neuropathic pain
b) Trauma
c) Visceral pain
d) Psychogenic pain
Explanation (Answer: c) Visceral pain)
The unspecified pain pathway, also known as the paleospinothalamic or spinoreticular pathway, transmits deep, dull, and poorly localized pain such as visceral pain. These fibers are slow-conducting C fibers that relay signals through multiple synapses in the brainstem before reaching the thalamus. This type of pain is often associated with emotional and autonomic responses such as nausea or sweating and is difficult to pinpoint anatomically due to widespread convergence of visceral afferents.
1. Which tract carries the impulses of fast, well-localized pain?
a) Spinothalamic tract
b) Spinoreticular tract
c) Spinomesencephalic tract
d) Spinocerebellar tract
Explanation (Answer: a) Spinothalamic tract)
The neospinothalamic tract carries fast, sharp, and well-localized pain through Aδ fibers to the thalamus. It allows precise localization of pain stimuli, such as a pinprick. This contrasts with the paleospinothalamic (unspecified) tract, which transmits dull, aching, and poorly localized pain sensations.
2. The neurotransmitter mainly involved in transmission of slow pain is:
a) Acetylcholine
b) Substance P
c) GABA
d) Dopamine
Explanation (Answer: b) Substance P)
Substance P is the primary neurotransmitter released by C fibers in the dorsal horn of the spinal cord during slow (unspecified) pain transmission. It activates second-order neurons in the spinoreticular and paleospinothalamic tracts, conveying dull, throbbing pain often linked with tissue injury and inflammation.
3. Which fibers transmit slow, burning pain sensation?
a) Aα
b) Aβ
c) Aδ
d) C fibers
Explanation (Answer: d) C fibers)
C fibers are unmyelinated, small-diameter fibers responsible for transmitting slow, burning, and poorly localized pain. They conduct at a velocity of about 0.5–2 m/s and are responsible for the second wave of pain sensation following acute injury, mediated via the unspecified pain pathway.
4. A 45-year-old patient presents with dull abdominal pain and nausea. Which pathway transmits this type of pain?
a) Spinothalamic
b) Spinoreticular
c) Spinocerebellar
d) Corticospinal
Explanation (Answer: b) Spinoreticular)
Visceral pain, which is dull, diffuse, and accompanied by autonomic symptoms, is transmitted through the spinoreticular (unspecified) tract. The pain fibers synapse in the reticular formation before ascending to the thalamus and limbic system, contributing to the emotional and autonomic components of pain perception.
5. Which of the following is true regarding the unspecified pain pathway?
a) Mediates fast pain
b) Uses Aδ fibers
c) Polysynaptic with multiple relays
d) Allows precise localization of pain
Explanation (Answer: c) Polysynaptic with multiple relays)
The unspecified (paleospinothalamic) pathway is polysynaptic, involving multiple relays in the spinal cord, reticular formation, and thalamus. It conveys dull, aching, and poorly localized pain. This pathway also interacts with the limbic system, explaining the emotional distress associated with chronic visceral pain.
6. Which structure modulates the perception of slow pain in the brain?
a) Cerebellum
b) Limbic system
c) Hypothalamus
d) Amygdala only
Explanation (Answer: b) Limbic system)
The limbic system modulates emotional and behavioral responses to slow or visceral pain transmitted by the unspecified pathway. It associates pain with anxiety, fear, or distress. This is why visceral pain often evokes strong emotional reactions compared to localized somatic pain.
7. Referred pain occurs because:
a) Shared spinal segments between somatic and visceral afferents
b) Crossed corticospinal fibers
c) Unmyelinated fibers’ slow conduction
d) Pain receptor desensitization
Explanation (Answer: a) Shared spinal segments between somatic and visceral afferents)
Referred pain occurs due to convergence of visceral and somatic afferents on the same spinal neurons. The brain misinterprets visceral pain as originating from a somatic region. For example, cardiac pain is referred to the left arm or jaw due to overlapping thoracic segments (T1–T5).
8. Which of the following is not true regarding visceral pain?
a) Poorly localized
b) Often referred to body surface
c) Transmitted by Aδ fibers
d) Associated with autonomic symptoms
Explanation (Answer: c) Transmitted by Aδ fibers)
Visceral pain is transmitted mainly by unmyelinated C fibers via the unspecified pain pathway. It is dull, poorly localized, and often accompanied by autonomic symptoms like nausea or sweating. The pain may be referred due to shared spinal cord pathways with somatic afferents.
9. A patient with chronic pancreatitis experiences deep, aching abdominal pain. The tract involved is:
a) Spinothalamic
b) Spinoreticular
c) Spinomesencephalic
d) Dorsal column
Explanation (Answer: b) Spinoreticular)
In chronic visceral conditions such as pancreatitis, pain is transmitted via the spinoreticular tract (unspecified pathway). This pathway’s diffuse, polysynaptic nature leads to poorly localized, persistent pain often associated with emotional distress, characteristic of visceral pathology.
10. The main difference between neospinothalamic and paleospinothalamic tracts is:
a) Fiber type and speed
b) Number of synapses
c) Localization accuracy
d) All of the above
Explanation (Answer: d) All of the above)
The neospinothalamic tract uses myelinated Aδ fibers, conducts rapidly, and provides precise localization of pain, while the paleospinothalamic (unspecified) tract uses unmyelinated C fibers, has multiple synapses, conducts slowly, and produces diffuse, emotional pain perception — typically visceral in origin.
11. Which of the following best describes the unspecified pain pathway?
a) Monosynaptic, localized
b) Polysynaptic, diffuse, emotional
c) Olfactory in origin
d) Linked to proprioception
Explanation (Answer: b) Polysynaptic, diffuse, emotional)
The unspecified pain pathway is polysynaptic and transmits dull, poorly localized pain often associated with emotion and autonomic responses. It involves the reticular formation and limbic system, contributing to the diffuse and affective nature of visceral pain sensations.
Topic: Nervous System; Subtopic: Spinal Reflexes and Motor Control
Keyword Definitions:
• Reflex: An involuntary, automatic, and stereotyped response to a specific stimulus.
• Withdrawal reflex (Flexor reflex): A polysynaptic spinal reflex that causes withdrawal of a limb from a painful stimulus.
• Reciprocal inhibition: Mechanism that relaxes antagonist muscles during a reflex action.
• Crossed extensor reflex: A reflex causing extension of the opposite limb to maintain balance during withdrawal.
• Golgi tendon reflex: Protective reflex preventing muscle damage due to excessive tension.
• Stretch reflex: Monosynaptic reflex maintaining muscle tone and posture (e.g., knee jerk reflex).
Lead Question - 2015
Withdrawal reflex is also known as ?
a) Extension reflex
b) Stretch reflex
c) Golgitendon reflex
d) Flexor reflex
Explanation (Answer: d) Flexor reflex)
The withdrawal reflex, also called the flexor reflex, is a protective spinal reflex that withdraws a body part from a painful or noxious stimulus. It involves activation of flexor muscles and inhibition of extensors through polysynaptic pathways in the spinal cord. This reflex is ipsilateral and rapid, essential for minimizing tissue injury. The reflex is often accompanied by a contralateral crossed extensor reflex to maintain posture and balance.
1. The receptor involved in the withdrawal reflex is:
a) Muscle spindle
b) Golgi tendon organ
c) Nociceptor
d) Pacinian corpuscle
Explanation (Answer: c) Nociceptor)
Nociceptors are pain receptors that detect damaging stimuli like heat, pressure, or injury. They initiate the withdrawal reflex by sending afferent impulses to the spinal cord, activating interneurons that stimulate flexor muscles and inhibit extensors. This automatic response protects the body from potential harm by removing the affected limb from danger.
2. Which of the following is an example of a monosynaptic reflex?
a) Flexor reflex
b) Crossed extensor reflex
c) Stretch reflex
d) Withdrawal reflex
Explanation (Answer: c) Stretch reflex)
The stretch reflex is the only monosynaptic spinal reflex. It involves direct synaptic transmission between a sensory neuron and a motor neuron without interneurons. Examples include the knee jerk (patellar) reflex. It maintains muscle tone and posture, contrasting with the polysynaptic nature of the withdrawal reflex.
3. The efferent pathway of withdrawal reflex involves:
a) Alpha motor neurons
b) Gamma motor neurons
c) Interneurons only
d) Sympathetic fibers
Explanation (Answer: a) Alpha motor neurons)
In the withdrawal reflex, afferent pain signals activate spinal interneurons that synapse with alpha motor neurons. These motor neurons cause contraction of flexor muscles and inhibition of antagonistic extensors through reciprocal inhibition. This rapid response ensures immediate withdrawal from harmful stimuli like heat or sharp objects.
4. Crossed extensor reflex occurs:
a) On the same side of stimulation
b) On the opposite side of stimulation
c) In both sides simultaneously
d) Only in upper limbs
Explanation (Answer: b) On the opposite side of stimulation)
The crossed extensor reflex occurs contralaterally. When one limb withdraws due to pain, the opposite limb extends to maintain balance. For example, stepping on a sharp object causes withdrawal of one leg and extension of the other. This is mediated by interneurons crossing the spinal cord midline.
5. Which statement is true about the flexor reflex?
a) Monosynaptic
b) Always accompanied by reciprocal inhibition
c) Mediated by muscle spindle
d) Controlled by cerebellum only
Explanation (Answer: b) Always accompanied by reciprocal inhibition)
The flexor reflex involves reciprocal inhibition—activation of flexor muscles and simultaneous inhibition of extensors. This coordination ensures effective limb withdrawal. It is a polysynaptic reflex involving interneurons and does not require supraspinal control, although descending pathways can modulate it.
6. A patient with spinal cord injury above the lumbar level shows exaggerated withdrawal reflexes. The reason is:
a) Hyperpolarization of neurons
b) Loss of inhibitory descending control
c) Absence of pain receptors
d) Muscle fatigue
Explanation (Answer: b) Loss of inhibitory descending control)
After spinal cord injury, descending inhibitory influences from higher centers are lost. This leads to hyperexcitability of spinal reflex circuits, producing exaggerated or hyperactive withdrawal reflexes. Clinically, this manifests as spasticity or exaggerated flexion in response to minimal stimuli.
7. The main neurotransmitter in the withdrawal reflex arc is:
a) Dopamine
b) Acetylcholine
c) Glutamate
d) Serotonin
Explanation (Answer: c) Glutamate)
Glutamate is the principal excitatory neurotransmitter mediating synaptic transmission in the withdrawal reflex. It activates spinal interneurons and alpha motor neurons. Glycine and GABA act as inhibitory neurotransmitters for antagonist muscles, ensuring coordinated movement during reflex withdrawal.
8. Golgi tendon reflex causes:
a) Muscle contraction
b) Muscle relaxation
c) Flexor withdrawal
d) Joint extension
Explanation (Answer: b) Muscle relaxation)
The Golgi tendon reflex protects muscles and tendons from excessive tension by causing muscle relaxation. Golgi tendon organs sense force and send inhibitory signals to alpha motor neurons, reducing contraction strength. This reflex ensures prevention of muscle or tendon damage during heavy load bearing.
9. Which type of reflex is lost first in peripheral nerve injury?
a) Flexor withdrawal reflex
b) Crossed extensor reflex
c) Deep tendon reflex
d) Pain reflex
Explanation (Answer: c) Deep tendon reflex)
In peripheral neuropathies, loss of deep tendon (stretch) reflexes occurs first due to involvement of large myelinated fibers. The flexor withdrawal reflex, being polysynaptic and slower, may persist longer. Reflex testing helps localize the level and type of neural damage in clinical practice.
10. In a burn injury, a rapid withdrawal of hand occurs due to activation of:
a) Proprioceptors
b) Nociceptors
c) Chemoreceptors
d) Thermoreceptors
Explanation (Answer: b) Nociceptors)
Nociceptors are specialized pain receptors that trigger the withdrawal reflex in response to harmful stimuli like burns. Their afferent signals activate spinal interneurons that initiate immediate muscle contraction for limb withdrawal. This rapid response prevents deeper tissue damage and is purely protective.
11. The contralateral component of the withdrawal reflex helps in:
a) Reducing pain sensation
b) Maintaining posture and balance
c) Inhibiting ipsilateral flexors
d) Decreasing muscle tone
Explanation (Answer: b) Maintaining posture and balance)
The crossed extensor reflex complements the withdrawal reflex by extending the opposite limb, thereby maintaining posture and equilibrium during withdrawal. This coordinated motor response allows the body to shift weight and prevent falls when one limb withdraws rapidly from a painful stimulus.
Topic: Nervous System Physiology; Subtopic: Wallerian Degeneration and Nerve Regeneration
Keyword Definitions:
• Wallerian degeneration: Process of degeneration of the distal segment of a nerve fiber after axonal injury.
• Axon: Long nerve fiber that transmits impulses away from the neuronal cell body.
• Neurolemma: Schwann cell sheath aiding in regeneration of peripheral nerves.
• Chromatolysis: Reversible cellular change in the neuron after axonal injury.
• Regeneration: Repair of injured nerve fibers, mainly in the peripheral nervous system.
• Myelin sheath: Lipid-rich covering that insulates nerve fibers and enhances conduction speed.
Lead Question - 2015
Wallerian degeneration is for ?
a) Nerve degeneration
b) Muscle degeneration
c) Nerve regeneration
d) Muscle regeneration
Explanation (Answer: a) Nerve degeneration)
Wallerian degeneration refers to the process of degeneration of the distal portion of a nerve fiber after it is severed from the cell body. It involves breakdown of the axon and myelin sheath distal to the injury site, while Schwann cells and macrophages clear debris. This degeneration facilitates subsequent nerve regeneration in the peripheral nervous system but not in the central nervous system due to absence of neurilemma and inhibitory factors.
1. Wallerian degeneration occurs in:
a) Axon distal to site of injury
b) Axon proximal to site of injury
c) Cell body of neuron
d) Dendrites only
Explanation (Answer: a) Axon distal to site of injury)
Wallerian degeneration affects the segment of the axon distal to the site of injury. The distal axon and its myelin sheath disintegrate due to lack of nutrient support from the neuronal cell body. This process begins within 24–36 hours after injury and is essential for clearance before regeneration in peripheral nerves.
2. Which cells are responsible for myelin removal during Wallerian degeneration?
a) Microglia
b) Schwann cells and macrophages
c) Astrocytes
d) Oligodendrocytes
Explanation (Answer: b) Schwann cells and macrophages)
In the peripheral nervous system, Schwann cells and recruited macrophages clear the myelin debris during Wallerian degeneration. Schwann cells also secrete neurotrophic factors that promote axonal regrowth. In contrast, the central nervous system lacks efficient macrophage activity, hindering regeneration due to myelin-associated inhibitors and absence of neurolemma.
3. Regeneration of axons in the CNS is limited due to:
a) Lack of Schwann cells
b) Presence of myelin inhibitory proteins
c) Glial scar formation
d) All of the above
Explanation (Answer: d) All of the above)
Axonal regeneration in CNS fails due to absence of Schwann cells, presence of inhibitory proteins such as Nogo-A, and formation of glial scars by astrocytes. These factors block axonal sprouting and remyelination. In contrast, peripheral nerves regenerate efficiently due to supportive Schwann cells and permissive extracellular environment.
4. Which of the following occurs first in Wallerian degeneration?
a) Myelin fragmentation
b) Axonal swelling and disintegration
c) Macrophage infiltration
d) Schwann cell proliferation
Explanation (Answer: b) Axonal swelling and disintegration)
The first morphological change in Wallerian degeneration is axonal swelling and breakdown distal to injury, usually within 24 hours. This is followed by myelin fragmentation, macrophage infiltration, and Schwann cell proliferation to clear debris and prepare the nerve for potential regeneration.
5. Chromatolysis refers to:
a) Breakdown of axon
b) Degeneration of distal nerve segment
c) Dissolution of Nissl bodies in neuron cell body
d) Myelin sheath regeneration
Explanation (Answer: c) Dissolution of Nissl bodies in neuron cell body)
Chromatolysis occurs in the cell body proximal to axonal injury. Nissl bodies disperse and the nucleus moves to the periphery, reflecting increased protein synthesis for regeneration. It is a reversible adaptive response aiding recovery unless the injury is severe or the neuron is in the CNS.
6. In peripheral nerve injury, regeneration begins:
a) Immediately after injury
b) Within 24 hours
c) After clearance of myelin debris
d) Only if CNS neurons are involved
Explanation (Answer: c) After clearance of myelin debris)
Regeneration begins only after myelin debris is cleared by Schwann cells and macrophages. Clean endoneurial tubes guide new axonal sprouts from the proximal stump toward target organs. Delay in debris clearance impairs regeneration, highlighting the crucial role of immune and glial cell cooperation in the process.
7. The first sign of Wallerian degeneration in light microscopy is:
a) Myelin beading
b) Axonal swelling
c) Nuclear pyknosis
d) Endoneurial edema
Explanation (Answer: a) Myelin beading)
Myelin beading appears as irregular varicosities in the myelin sheath soon after axonal transection. It signifies disintegration of the axolemma and onset of degeneration. This precedes fragmentation and phagocytosis. These changes are identifiable within 24–48 hours in the distal segment of the injured nerve.
8. A 35-year-old patient with crush injury of radial nerve recovers slowly because:
a) Myelin debris clearance is delayed
b) Schwann cells are destroyed
c) Axon regeneration is blocked by fibrosis
d) All of the above
Explanation (Answer: d) All of the above)
In crush injuries, extensive damage delays debris clearance, may destroy Schwann cells, and cause fibrosis, all slowing axonal regrowth. Regeneration rate averages 1–3 mm/day. Proper alignment and intact neurolemma enhance recovery, but severe damage results in incomplete functional restoration.
9. Successful nerve regeneration in PNS requires:
a) Intact endoneurial tube
b) Presence of Schwann cells
c) Healthy neuron cell body
d) All of the above
Explanation (Answer: d) All of the above)
Peripheral nerve regeneration depends on an intact endoneurial tube guiding axonal sprouts, viable Schwann cells producing neurotrophic factors, and a healthy neuronal cell body for new axon synthesis. Disruption of any element limits recovery, emphasizing the coordinated role of structural and biochemical support in regeneration.
10. Wallerian degeneration does not occur in:
a) Peripheral nerve
b) Central nervous system
c) Spinal root injury
d) Cranial nerve distal to ganglion
Explanation (Answer: b) Central nervous system)
True Wallerian degeneration with functional regeneration occurs only in the peripheral nervous system due to Schwann cell activity. In the CNS, degeneration is incomplete and non-reparative due to oligodendrocyte inhibition and glial scarring. Thus, CNS neurons rarely regenerate after injury.
11. A patient with transected sciatic nerve shows regeneration over months. Which process preceded regeneration?
a) Chromatolysis
b) Wallerian degeneration
c) Gliosis
d) Synaptic inhibition
Explanation (Answer: b) Wallerian degeneration)
In peripheral nerve injury, regeneration follows Wallerian degeneration, which removes distal axonal and myelin debris. This cleanup allows Schwann cells to proliferate, align, and guide new axonal sprouts from the proximal stump toward the target organ. Without prior degeneration, regeneration cannot proceed effectively.
Topic: Nervous System; Subtopic: Motor Evoked Potentials and Neural Conduction
Keyword Definitions:
• Motor evoked potential (MEP): Electrical response recorded from muscles following stimulation of motor pathways, usually by transcranial magnetic or electrical stimulation.
• Central motor pathways: Pathways from motor cortex to spinal cord controlling voluntary movements.
• Peripheral motor pathways: Motor nerves transmitting impulses from spinal cord to skeletal muscles.
• Corticospinal tract: Major descending pathway responsible for fine motor control.
• Latency: Time interval between stimulus and response indicating conduction speed.
• Demyelination: Loss of myelin slowing nerve conduction and altering evoked potentials.
Lead Question - 2015
Motor evoked potential assess ?
a) Peripheral motor pathways
b) Central motor pathways
c) Both of the above
d) Regeneration in muscles
Explanation (Answer: b) Central motor pathways)
Motor Evoked Potentials (MEPs) are used to assess the integrity of central motor pathways, especially the corticospinal tract. They are elicited by transcranial magnetic stimulation over the motor cortex, and recorded from target muscles. Latency and amplitude reflect conduction through brain, spinal cord, and upper motor neuron pathways. Peripheral nerves contribute minimally, making MEPs useful in diagnosing multiple sclerosis, spinal cord lesions, and intraoperative monitoring of central motor integrity.
1. MEPs are most useful in detecting lesions of:
a) Lower motor neuron
b) Upper motor neuron
c) Neuromuscular junction
d) Muscle fibers
Explanation (Answer: b) Upper motor neuron)
MEPs primarily test upper motor neuron (UMN) pathways by stimulating the motor cortex and recording muscle responses. Damage in the corticospinal tract increases latency or abolishes MEPs. Lower motor neuron or muscle diseases affect compound muscle action potentials instead. Thus, MEPs are key for identifying central motor conduction defects in UMN disorders like stroke or multiple sclerosis.
2. During spinal surgery, MEPs help in:
a) Monitoring sensory integrity
b) Monitoring motor pathway integrity
c) Measuring muscle fatigue
d) Recording EEG activity
Explanation (Answer: b) Monitoring motor pathway integrity)
Intraoperative MEPs provide real-time monitoring of motor pathway function during spinal or neurosurgical procedures. Changes in amplitude or latency warn of potential corticospinal injury. Somatosensory evoked potentials (SSEPs) monitor sensory tracts, not motor. MEPs help prevent permanent paralysis during surgeries involving spinal cord or brainstem manipulation.
3. Latency of MEP is increased in:
a) Myasthenia gravis
b) Multiple sclerosis
c) Myopathy
d) Motor neuron disease
Explanation (Answer: b) Multiple sclerosis)
In multiple sclerosis, demyelination of central motor tracts delays conduction, increasing MEP latency. Amplitude may also decrease due to conduction block. Myasthenia gravis affects neuromuscular transmission, not central conduction. Myopathy affects muscle responses, not latency. Thus, prolonged latency in MEPs is diagnostic of demyelinating CNS disorders.
4. The stimulus used to elicit MEPs in clinical testing is usually:
a) Direct electrical stimulation
b) Transcranial magnetic stimulation
c) Infrared light pulses
d) Ultrasound stimulation
Explanation (Answer: b) Transcranial magnetic stimulation)
Transcranial magnetic stimulation (TMS) induces an electrical current in the motor cortex using magnetic fields. This activates descending corticospinal fibers and evokes motor responses in peripheral muscles. It is safe, noninvasive, and ideal for assessing central motor conduction. Electrical stimulation is used for peripheral nerve studies, not cortical activation.
5. Absence of MEPs in a patient with spinal cord injury indicates:
a) Peripheral neuropathy
b) Corticospinal tract disruption
c) Myasthenic crisis
d) Hypokalemia
Explanation (Answer: b) Corticospinal tract disruption)
Loss of MEP response indicates complete interruption of corticospinal pathways within the spinal cord. This helps localize the level of damage and predict prognosis. Peripheral nerves remain intact, ruling out peripheral neuropathy. Myasthenic crisis affects neuromuscular junction but not cortical conduction. MEP absence is an important prognostic marker in spinal injuries.
6. Clinical application of MEP in multiple sclerosis is:
a) Measuring muscle contraction
b) Detecting conduction block in central motor tracts
c) Estimating oxygen uptake
d) Evaluating sensory threshold
Explanation (Answer: b) Detecting conduction block in central motor tracts)
In multiple sclerosis, demyelination causes slowed conduction or complete block in corticospinal pathways. MEPs identify these changes by increased latency or absent motor responses. This helps in diagnosis and monitoring disease progression. It complements MRI in functional assessment, as it evaluates actual neural transmission integrity rather than structural changes alone.
7. In MEP testing, stimulation site for lower limb response is:
a) Parietal cortex
b) Occipital cortex
c) Vertex (Cz region)
d) Temporal lobe
Explanation (Answer: c) Vertex (Cz region)
For lower limb MEPs, stimulation is applied over the vertex region corresponding to leg representation in the motor cortex. Electrodes record responses from tibialis anterior or quadriceps muscles. This setup assesses corticospinal conduction from cortex to lumbosacral spinal segments. Parietal and occipital cortices are not involved in motor activation.
8. MEP amplitude primarily reflects:
a) Cortical excitability
b) Synaptic inhibition
c) Sensory input
d) Vascular perfusion
Explanation (Answer: a) Cortical excitability)
Amplitude of MEP correlates with excitability of motor cortex neurons and efficiency of corticospinal transmission. Decreased amplitude may indicate cortical suppression or conduction block. It can vary with anesthesia, fatigue, or disease. Sensory input and perfusion do not directly influence amplitude in MEP recordings.
9. Which anesthetic agent suppresses MEPs most markedly?
a) Ketamine
b) Propofol
c) Isoflurane
d) Nitrous oxide
Explanation (Answer: c) Isoflurane)
Isoflurane and other volatile anesthetics significantly depress cortical excitability, reducing MEP amplitude or abolishing responses. Intravenous agents like propofol and ketamine have less effect, making them preferable for intraoperative MEP monitoring. Nitrous oxide also suppresses but less intensely. Anesthesia choice is crucial for reliable MEP recording during surgery.
10. A patient with amyotrophic lateral sclerosis (ALS) shows MEP findings of:
a) Normal latency and amplitude
b) Prolonged latency and reduced amplitude
c) Increased amplitude
d) Absent sensory potentials
Explanation (Answer: b) Prolonged latency and reduced amplitude)
In ALS, both upper and lower motor neurons degenerate, causing prolonged MEP latency and decreased amplitude due to impaired central conduction. Sensory pathways remain intact. These changes help in distinguishing ALS from peripheral neuropathies. MEP testing assists in early detection of corticospinal tract dysfunction before overt clinical weakness.
11. MEPs are absent but sensory evoked potentials are normal in:
a) Spinal cord transection
b) Peripheral neuropathy
c) Myasthenia gravis
d) Cerebellar degeneration
Explanation (Answer: a) Spinal cord transection)
In spinal cord transection, motor pathways are completely interrupted, abolishing MEPs. Sensory pathways may remain intact initially, resulting in normal sensory evoked potentials (SEPs). This dissociation confirms motor pathway damage. Peripheral neuropathy or neuromuscular diseases affect both motor and sensory conduction, while cerebellar lesions spare MEPs.
Chapter: Neuroanatomy; Topic: Nervous Tissue; Subtopic: Myelination in the Peripheral and Central Nervous System
Keyword Definitions:
• Myelin sheath: A fatty insulating layer that increases the speed of nerve impulse conduction.
• Schwann cells: Glial cells responsible for myelination in the peripheral nervous system.
• Oligodendrocytes: Cells that myelinate axons in the central nervous system.
• Astrocytes: Star-shaped glial cells that maintain the blood-brain barrier and nutrient supply.
• Ependymal cells: Line ventricles of the brain and central canal of the spinal cord; produce cerebrospinal fluid.
• Nodes of Ranvier: Gaps between myelin segments allowing saltatory conduction.
Lead Question - 2015
Myelination in peripheral nervous system is done by:
a) Astrocytes
b) Oligodendrocytes
c) Ependymal cells
d) Schwann cells
Explanation (Answer: d) Schwann cells
In the peripheral nervous system, Schwann cells produce the myelin sheath around axons. Each Schwann cell myelinates one segment of a single axon. In contrast, oligodendrocytes myelinate several axons in the CNS. Schwann cells also assist in nerve regeneration, unlike CNS glial cells that inhibit axonal regrowth.
1. Myelination in the central nervous system is performed by which type of cell?
a) Schwann cells
b) Astrocytes
c) Oligodendrocytes
d) Microglia
Explanation (Answer: c) Oligodendrocytes
Oligodendrocytes are responsible for myelination of axons in the central nervous system (CNS). One oligodendrocyte can myelinate several axons simultaneously. Damage to these cells, as seen in multiple sclerosis, leads to demyelination and impaired nerve conduction. They differ from Schwann cells that myelinate peripheral nerves.
2. A 30-year-old woman presents with blurred vision and limb weakness. MRI shows demyelination in the CNS. The affected cells are:
a) Schwann cells
b) Oligodendrocytes
c) Astrocytes
d) Microglia
Explanation (Answer: b) Oligodendrocytes)
In multiple sclerosis, there is autoimmune destruction of oligodendrocytes, leading to CNS demyelination. This causes conduction block and neurological symptoms like visual loss and muscle weakness. Peripheral nerves remain unaffected because their myelin is formed by Schwann cells, not oligodendrocytes.
3. Which of the following statements regarding Schwann cells is true?
a) Each Schwann cell myelinates multiple axons
b) Schwann cells are found only in the CNS
c) They aid in axonal regeneration after injury
d) They produce cerebrospinal fluid
Explanation (Answer: c) They aid in axonal regeneration after injury)
Schwann cells not only form the myelin sheath in peripheral nerves but also assist in axonal regeneration following injury by creating a guiding tube. They release neurotrophic factors promoting regrowth. In contrast, CNS regeneration is limited due to inhibitory molecules from oligodendrocytes and glial scarring by astrocytes.
4. Which cell forms myelin around multiple axons in the central nervous system?
a) Microglia
b) Oligodendrocytes
c) Schwann cells
d) Astrocytes
Explanation (Answer: b) Oligodendrocytes)
Oligodendrocytes extend cytoplasmic processes to wrap myelin around several axons within the CNS. Each cell may myelinate up to 30 axons. This arrangement contrasts with Schwann cells, which myelinate only one segment of one axon in the PNS. Oligodendrocyte injury leads to CNS demyelination.
5. A patient with Guillain-Barré syndrome shows segmental demyelination. Which cell type is primarily affected?
a) Oligodendrocytes
b) Microglia
c) Schwann cells
d) Astrocytes
Explanation (Answer: c) Schwann cells)
Guillain-Barré syndrome (GBS) is an acute demyelinating disorder of the peripheral nervous system caused by immune-mediated destruction of Schwann cells. It leads to ascending muscle weakness and areflexia. CNS remains unaffected because oligodendrocytes are not involved. Early management with plasmapheresis aids recovery.
6. Which of the following glial cells forms the blood-brain barrier by connecting neurons to capillaries?
a) Oligodendrocytes
b) Astrocytes
c) Schwann cells
d) Microglia
Explanation (Answer: b) Astrocytes)
Astrocytes are star-shaped glial cells that provide metabolic support to neurons and form part of the blood-brain barrier via their end-feet processes. They regulate ionic balance and neurotransmitter clearance. Though not involved in myelination, they are crucial in maintaining CNS homeostasis and repair after injury.
7. Myelin sheath primarily facilitates which of the following functions?
a) Neurotransmitter synthesis
b) Rapid saltatory conduction
c) Receptor activation
d) Axonal branching
Explanation (Answer: b) Rapid saltatory conduction)
The myelin sheath insulates axons and allows action potentials to jump between the Nodes of Ranvier, a process called saltatory conduction. This greatly increases the velocity of nerve impulses. Loss of myelin, as seen in demyelinating diseases, causes conduction slowing or block.
8. A child with congenital absence of Schwann cells will most likely present with:
a) Hyperreflexia
b) Slowed peripheral nerve conduction
c) Enhanced synaptic transmission
d) Increased CNS myelination
Explanation (Answer: b) Slowed peripheral nerve conduction)
Absence of Schwann cells results in loss of peripheral myelin and markedly reduced nerve conduction velocity. Schwann cells are essential for saltatory conduction and nerve regeneration. Such a condition leads to weakness, sensory loss, and reduced reflexes typical of peripheral neuropathy.
9. Which of the following correctly pairs the glial cell with its function?
a) Ependymal cell – Blood-brain barrier
b) Oligodendrocyte – PNS myelination
c) Schwann cell – PNS myelination
d) Astrocyte – Cerebrospinal fluid secretion
Explanation (Answer: c) Schwann cell – PNS myelination)
Schwann cells myelinate peripheral nerve fibers and help in axonal regeneration. In contrast, oligodendrocytes myelinate CNS fibers, ependymal cells secrete cerebrospinal fluid, and astrocytes form the blood-brain barrier. Thus, Schwann cells are critical for peripheral nerve repair and conduction efficiency.
10. A 40-year-old male develops progressive limb weakness following Campylobacter infection. The pathology involves:
a) Destruction of oligodendrocytes
b) Degeneration of Schwann cells
c) Axonal sprouting in CNS
d) Astrocyte proliferation
Explanation (Answer: b) Degeneration of Schwann cells)
In post-infectious Guillain-Barré syndrome, immune-mediated damage to Schwann cells causes segmental demyelination of peripheral nerves. The loss of myelin slows conduction and produces ascending paralysis. Recovery occurs as Schwann cells remyelinate axons, highlighting their vital role in nerve repair.
Chapter: Embryology; Topic: Ectodermal Derivatives; Subtopic: Neural Crest Cells – Migration and Derivatives
Key Definitions:
• Neural crest cells: A population of specialized cells derived from the ectoderm at the margins of the neural folds that migrate extensively and differentiate into diverse tissues throughout the body.
• Ectoderm: The outermost germ layer of the embryo, giving rise to the nervous system, epidermis, and various glands.
• Adrenal medulla: The inner region of the adrenal gland derived from neural crest cells, producing catecholamines like epinephrine and norepinephrine.
• Pia and arachnoid mater: Meningeal layers of neural crest origin surrounding the brain and spinal cord.
Lead Question (NEET PG 2015):
1. Structures derived from the neural crest are?
a) Pia
b) Dental papillae
c) Adrenal medulla
d) All of the above
Answer: d) All of the above
Explanation: Neural crest cells arise from the lateral edges of the neural folds during neurulation. They migrate throughout the body and give rise to multiple tissues, including melanocytes, dorsal root ganglia, sympathetic ganglia, Schwann cells, adrenal medulla, pia-arachnoid mater, and connective tissues of the head and neck. The dental papilla, which contributes to dentin formation, also originates from neural crest mesenchyme. Thus, all listed structures — pia, dental papilla, and adrenal medulla — are derived from neural crest cells, illustrating their remarkable multipotency and migratory capacity.
Guessed Questions (Related to Neural Crest Derivatives):
2. Which of the following is not derived from neural crest cells?
a) Melanocytes
b) Schwann cells
c) Anterior pituitary
d) Adrenal medulla
Answer: c) Anterior pituitary
Explanation: The anterior pituitary (adenohypophysis) is derived from ectoderm of Rathke’s pouch, not neural crest cells. The posterior pituitary arises from neuroectoderm. Neural crest cells, however, form melanocytes, Schwann cells, and adrenal medulla.
3. Clinical: A child presents with a tumor of the adrenal medulla. This tumor is derived from neural crest cells and is called:
a) Neuroblastoma
b) Wilms tumor
c) Nephroblastoma
d) Ependymoma
Answer: a) Neuroblastoma
Explanation: Neuroblastoma arises from neural crest-derived cells of the adrenal medulla or sympathetic ganglia. It secretes catecholamines and presents with hypertension, abdominal mass, and elevated urinary VMA levels.
4. The melanocytes of the skin are derived from which embryonic origin?
a) Surface ectoderm
b) Neural crest
c) Mesoderm
d) Endoderm
Answer: b) Neural crest
Explanation: Melanocytes arise from neural crest cells that migrate to the basal layer of the epidermis and produce melanin pigment. Defective migration results in hypopigmented conditions such as piebaldism or Waardenburg syndrome.
5. Clinical: Hirschsprung’s disease results from failure of migration of neural crest cells to form which structure?
a) Enteric ganglia
b) Dorsal root ganglia
c) Adrenal cortex
d) Motor neurons
Answer: a) Enteric ganglia
Explanation: Hirschsprung’s disease (congenital aganglionic megacolon) results from failure of neural crest cells to populate the wall of the distal colon, leading to absence of Auerbach’s and Meissner’s plexuses and functional obstruction.
6. Neural crest cells contribute to all of the following except:
a) Parafollicular cells of thyroid
b) Chromaffin cells of adrenal medulla
c) Odontoblasts
d) Hepatocytes
Answer: d) Hepatocytes
Explanation: Hepatocytes are derived from endoderm of the foregut. Neural crest cells give rise to parafollicular cells, chromaffin cells, odontoblasts, and various craniofacial connective tissues.
7. Clinical: A child with congenital absence of ganglion cells in the gut wall is likely to have defective migration of neural crest cells from:
a) Rhombencephalon
b) Telencephalon
c) Diencephalon
d) Mesencephalon
Answer: a) Rhombencephalon
Explanation: Neural crest cells originating from the rhombencephalon migrate caudally to form enteric ganglia. Defective migration results in Hirschsprung’s disease, leading to megacolon and constipation.
8. The meninges derived from neural crest cells are:
a) Dura mater
b) Pia and arachnoid mater
c) Only pia mater
d) All three layers
Answer: b) Pia and arachnoid mater
Explanation: The pia and arachnoid mater (leptomeninges) are derived from neural crest cells, while the dura mater arises from mesoderm. These layers protect the brain and spinal cord and contain cerebrospinal fluid within the subarachnoid space.
9. Neural crest cells contribute to formation of which cardiac structure?
a) Endocardial cushions
b) Aorticopulmonary septum
c) Myocardium
d) Pericardium
Answer: b) Aorticopulmonary septum
Explanation: Neural crest cells migrate into the truncus arteriosus and bulbus cordis to form the aorticopulmonary septum. Abnormal development leads to congenital heart defects such as Tetralogy of Fallot and transposition of great arteries.
10. Clinical: Waardenburg syndrome, characterized by heterochromia iridis, white forelock, and deafness, is caused by abnormal migration of:
a) Mesodermal cells
b) Neural crest cells
c) Endodermal cells
d) Neuroectodermal cells
Answer: b) Neural crest cells
Explanation: Waardenburg syndrome results from defective neural crest cell migration affecting melanocytes, craniofacial mesenchyme, and cochlear ganglion development. The disorder causes pigmentation anomalies and sensorineural deafness.
11. Neural crest derivatives include all except:
a) Schwann cells
b) Odontoblasts
c) Pia mater
d) Anterior horn motor neurons
Answer: d) Anterior horn motor neurons
Explanation: Motor neurons of the anterior horn are derived from neuroectoderm of the neural tube, not neural crest. Neural crest derivatives include Schwann cells, pia mater, odontoblasts, and sympathetic ganglia.
Chapter: Embryology; Topic: Early Embryonic Development; Subtopic: Notochord – Formation, Function, and Fate
Key Definitions:
• Notochord: A midline rod-like structure formed from mesodermal cells that serves as the primitive axis of the embryo and plays a key inductive role in neural development.
• Primary inductor: The notochord induces overlying ectoderm to form the neural plate, initiating neurulation.
• Hypoblast: The lower layer of the bilaminar embryonic disc, contributing to extraembryonic endoderm but not to the notochord.
• Nucleus pulposus: The central gelatinous core of the intervertebral disc, representing the remnant of the notochord in adults.
Lead Question (NEET PG 2015):
1. True about notochord are all except?
a) Defines axis of embryo
b) Serves as primary inductor
c) Derived from hypoblast
d) Remains as nucleus pulposus
Answer: c) Derived from hypoblast
Explanation: The notochord is derived from mesodermal cells migrating from the primitive node (Hensen’s node), not from the hypoblast. It defines the longitudinal axis of the embryo and induces the formation of the neural plate, marking it as the primary inductor in embryogenesis. Later in development, most of the notochord degenerates, but its remnants persist as the nucleus pulposus of intervertebral discs. The notochord also provides signals for differentiation of surrounding mesoderm into vertebral bodies and contributes to body plan organization.
Guessed Questions (Related to Notochord and Neural Induction):
2. The notochord develops from which embryonic structure?
a) Primitive node
b) Hypoblast
c) Amnion
d) Yolk sac
Answer: a) Primitive node
Explanation: The notochord arises from mesodermal cells migrating cranially from the primitive node. These cells form the notochordal process, which eventually develops into the solid notochordal rod that defines the embryonic midline.
3. Clinical: A congenital defect due to persistence of the notochord is known as:
a) Chordoma
b) Teratoma
c) Meningocele
d) Glioma
Answer: a) Chordoma
Explanation: Chordoma is a rare malignant tumor derived from remnants of the notochord, usually found at the base of the skull or sacrococcygeal region. It is slow-growing and locally invasive, arising from residual notochordal tissue that fails to regress completely.
4. The notochord induces the overlying ectoderm to form:
a) Neural plate
b) Neural crest
c) Somites
d) Myotome
Answer: a) Neural plate
Explanation: The notochord functions as a primary inductor by releasing signaling molecules that induce the overlying ectoderm to thicken and form the neural plate. This marks the beginning of neurulation and the development of the central nervous system.
5. Clinical: If notochordal induction fails, which of the following may result?
a) Neural tube defects
b) Craniofacial abnormalities
c) Hemivertebra
d) All of the above
Answer: d) All of the above
Explanation: Failure of notochordal induction disrupts neural tube formation, leading to neural tube defects such as spina bifida or anencephaly. It may also result in defective vertebral and craniofacial development due to its influence on mesodermal differentiation.
6. The notochord is located between:
a) Neural tube and endoderm
b) Ectoderm and mesoderm
c) Mesoderm and endoderm
d) Ectoderm and yolk sac
Answer: a) Neural tube and endoderm
Explanation: The notochord lies ventral to the neural tube and dorsal to the endoderm. This central position allows it to serve as an organizing center, influencing the development of both neural and mesodermal structures.
7. Clinical: A midline mass at the base of the skull extending into the nasopharynx could represent:
a) Chordoma
b) Meningocele
c) Craniopharyngioma
d) Lipoma
Answer: a) Chordoma
Explanation: Chordomas arise from persistent notochordal remnants and are often located in the clivus or sacrococcygeal region. They may present as midline masses with compressive symptoms in the cranial base or spine.
8. The notochord disappears during development except in which structure?
a) Vertebral body
b) Nucleus pulposus
c) Spinal cord
d) Neural crest
Answer: b) Nucleus pulposus
Explanation: Most of the notochord degenerates as vertebral bodies form, but remnants persist as the nucleus pulposus of intervertebral discs. This gelatinous core maintains flexibility and cushioning between vertebrae.
9. Which germ layer gives rise to the notochord?
a) Ectoderm
b) Mesoderm
c) Endoderm
d) None of the above
Answer: b) Mesoderm
Explanation: The notochord is derived from mesodermal cells that migrate from the primitive node. This axial mesodermal structure is crucial for patterning the surrounding tissues during gastrulation and neurulation.
10. Clinical: A child presents with a dorsal midline sinus discharging mucoid material since birth. This may indicate persistence of:
a) Notochordal canal
b) Neural crest
c) Primitive streak
d) Neurenteric canal
Answer: d) Neurenteric canal
Explanation: Persistence of the neurenteric canal, which transiently connects the amniotic cavity with the yolk sac during notochord formation, may result in midline fistulae or cysts connecting spinal and gastrointestinal structures.
11. The notochordal process communicates transiently with the yolk sac via:
a) Neurenteric canal
b) Allantois
c) Cloacal membrane
d) Amniotic canal
Answer: a) Neurenteric canal
Explanation: During notochordal development, the notochordal process temporarily opens to the yolk sac through the neurenteric canal, maintaining continuity between the amniotic and yolk sac cavities until closure during normal development.
Chapter: Anatomy; Topic: General Principles of Nerve Supply; Subtopic: Hilton’s Law and Its Clinical Significance
Key Definitions:
• Hilton’s Law: The nerve supplying a joint also supplies the muscles that move the joint and the skin overlying the insertions of those muscles.
• Nerve innervation: The process by which nerves provide motor and sensory supply to muscles, joints, and skin.
• Articular branches: Small nerve branches that supply sensory fibers to the joint capsule and ligaments.
• Proprioception: The sensory function of nerves that detect joint position and movement, maintaining coordination and balance.
Lead Question (NEET PG 2015):
1. Hilton’s law is related to:
a) Venous drainage
b) Blood supply
c) Nerve innervation
d) All of the above
Answer: c) Nerve innervation
Explanation: Hilton’s law states that the same nerves which supply the muscles acting on a joint also supply the joint itself and the skin overlying the insertions of those muscles. This anatomical principle ensures coordinated sensory and motor function for protection and efficient movement. For example, the femoral nerve supplies the quadriceps muscle, the knee joint, and the skin over the anterior thigh. Clinically, joint pain can radiate to the overlying skin due to shared nerve supply, explaining referred pain patterns in joint diseases.
Guessed Questions (Related to Hilton’s Law and Nerve Supply):
2. According to Hilton’s law, the nerve to a muscle acting on a joint also supplies:
a) The overlying skin
b) The bone only
c) The opposite limb
d) The lymphatics
Answer: a) The overlying skin
Explanation: Hilton’s law emphasizes that the nerve supplying a joint innervates the muscles moving that joint and the skin overlying the muscle insertion. This coordinated innervation helps integrate sensory feedback and motor control for joint movement.
3. The knee joint receives articular branches from which nerve according to Hilton’s law?
a) Femoral nerve
b) Obturator nerve
c) Tibial nerve
d) All of the above
Answer: d) All of the above
Explanation: The knee joint is supplied by articular branches from the femoral, obturator, and tibial nerves, which also supply muscles acting across the joint such as quadriceps, hamstrings, and gastrocnemius. This perfectly illustrates Hilton’s law.
4. Clinical: A patient with shoulder joint inflammation experiences pain over the deltoid region. This is explained by:
a) Hilton’s law
b) Bell’s law
c) Law of Laplace
d) Boyle’s law
Answer: a) Hilton’s law
Explanation: The axillary nerve supplies the deltoid muscle, the shoulder joint, and the skin over the deltoid region. Hence, inflammation of the shoulder joint causes referred pain over the deltoid region according to Hilton’s law.
5. The hip joint is supplied by branches of which of the following nerves?
a) Femoral, obturator, and sciatic
b) Obturator only
c) Pudendal only
d) Gluteal only
Answer: a) Femoral, obturator, and sciatic
Explanation: The hip joint receives articular branches from the femoral, obturator, and sciatic nerves, which also innervate muscles acting on the hip (such as iliopsoas, adductors, and hamstrings). This aligns with Hilton’s law regarding shared innervation.
6. Clinical: Pain from inflammation of the elbow joint radiating to the forearm and hand is due to shared innervation by which nerve?
a) Median nerve
b) Ulnar nerve
c) Radial nerve
d) All of the above
Answer: d) All of the above
Explanation: The elbow joint is supplied by branches from the median, ulnar, and radial nerves, which also innervate muscles acting across it (biceps, triceps, brachialis). Therefore, inflammation or injury to the joint can produce referred pain along these nerve distributions.
7. Which of the following best describes the functional purpose of Hilton’s law?
a) Coordination of muscle action with joint sensation
b) Control of blood flow in joints
c) Equalization of venous drainage
d) Lymphatic flow regulation
Answer: a) Coordination of muscle action with joint sensation
Explanation: Hilton’s law provides a neurophysiological basis for coordinated muscle contraction and joint proprioception. It ensures that the same nerve controlling movement also senses joint position and pain, optimizing protection and movement precision.
8. Clinical: In osteoarthritis of the knee, pain is often felt in the anterior thigh region. This occurs because:
a) Femoral nerve supplies both regions
b) Referred pain from sciatic nerve
c) Tibial nerve entrapment
d) Vascular compression
Answer: a) Femoral nerve supplies both regions
Explanation: The femoral nerve supplies the quadriceps (which move the knee joint) and provides articular branches to the knee and cutaneous branches to the anterior thigh. Thus, knee pain may be referred to the thigh following Hilton’s law.
9. The ankle joint receives articular branches from which nerves?
a) Tibial and deep peroneal nerves
b) Femoral and obturator nerves
c) Sural and pudendal nerves
d) Only tibial nerve
Answer: a) Tibial and deep peroneal nerves
Explanation: The ankle joint is supplied by branches from the tibial, deep peroneal, and superficial peroneal nerves, which also supply muscles that move the ankle. This innervation pattern is consistent with Hilton’s law of shared motor and sensory supply.
10. Clinical: During arthroscopic surgery, knowledge of Hilton’s law is important because:
a) It helps identify nerves responsible for referred joint pain
b) It guides blood vessel ligation
c) It predicts lymphatic drainage
d) It determines joint cartilage thickness
Answer: a) It helps identify nerves responsible for referred joint pain
Explanation: Hilton’s law allows clinicians to understand and predict patterns of referred pain and choose appropriate nerve blocks during joint surgeries. Knowing which nerves supply a joint and its associated muscles helps minimize postoperative pain and optimize anesthesia.
11. Clinical: Pain over the heel in an inflamed ankle joint can be explained by shared innervation through which nerve?
a) Tibial nerve
b) Common peroneal nerve
c) Sural nerve
d) Femoral nerve
Answer: a) Tibial nerve
Explanation: The tibial nerve supplies both the ankle joint and skin over the heel through its articular and cutaneous branches. Therefore, inflammation of the ankle joint can cause referred pain in the heel, illustrating Hilton’s law.
Chapter: Neuroanatomy; Topic: Dural Venous Sinuses; Subtopic: Cavernous Sinus and Its Relations
Key Definitions:
• Cavernous sinus: A paired dural venous sinus located on either side of the body of the sphenoid bone, surrounding the pituitary gland and internal carotid artery.
• Internal carotid artery: A major artery that passes through the cavernous sinus and supplies blood to the brain and orbit.
• Cranial nerves in cavernous sinus: Nerves passing through or within the wall of the sinus, including CN III, IV, V1, V2, and VI.
• Venous communication: The cavernous sinus receives blood from facial and ophthalmic veins, making it a potential site for the spread of facial infections.
Lead Question (NEET PG 2015):
1. Which of the following structures is seen in the cavernous sinus?
a) Maxillary division of V nerve
b) Mandibular division of V nerve
c) Internal carotid artery
d) Facial nerve
Answer: c) Internal carotid artery
Explanation: The cavernous sinus contains the internal carotid artery and the abducent nerve (CN VI) within its lumen. The lateral wall of the sinus houses the oculomotor (III), trochlear (IV), ophthalmic (V1), and maxillary (V2) nerves. The mandibular nerve (V3) and facial nerve do not pass through this sinus. Because of its close association with facial veins via the ophthalmic veins, infections from the “danger area” of the face can spread to the cavernous sinus, leading to thrombosis, ophthalmoplegia, and loss of corneal reflex.
Guessed Questions (Related to Cavernous Sinus and Its Relations):
2. Which cranial nerve lies within the cavernous sinus alongside the internal carotid artery?
a) Oculomotor nerve
b) Trochlear nerve
c) Abducent nerve
d) Maxillary nerve
Answer: c) Abducent nerve
Explanation: The abducent nerve (CN VI) travels within the cavernous sinus, lateral to the internal carotid artery. It is the only cranial nerve running free inside the sinus, making it highly vulnerable to compression during cavernous sinus thrombosis or aneurysmal dilation of the internal carotid artery, leading to lateral rectus palsy and diplopia.
3. The lateral wall of the cavernous sinus contains all of the following except:
a) Oculomotor nerve
b) Trochlear nerve
c) Abducent nerve
d) Ophthalmic division of trigeminal nerve
Answer: c) Abducent nerve
Explanation: The lateral wall of the cavernous sinus houses the oculomotor (III), trochlear (IV), and the first two divisions of the trigeminal nerve (V1, V2). The abducent nerve (VI), however, lies within the sinus along with the internal carotid artery. This anatomical distinction is clinically important in localizing lesions within the sinus.
4. Cavernous sinus thrombosis may result in paralysis of all the following muscles except:
a) Lateral rectus
b) Superior oblique
c) Levator palpebrae superioris
d) Superior pharyngeal constrictor
Answer: d) Superior pharyngeal constrictor
Explanation: Cavernous sinus thrombosis can involve cranial nerves III, IV, V1, V2, and VI, leading to ophthalmoplegia (paralysis of extraocular muscles) and sensory loss over the forehead and face. The superior pharyngeal constrictor is supplied by the vagus nerve, which is not related to the cavernous sinus, hence remains unaffected.
5. Which of the following venous structures drains into the cavernous sinus?
a) Inferior sagittal sinus
b) Superior ophthalmic vein
c) Sigmoid sinus
d) Great cerebral vein
Answer: b) Superior ophthalmic vein
Explanation: The superior ophthalmic vein drains blood from the orbit into the cavernous sinus. This connection allows retrograde spread of facial infections from the “danger area” of the face (upper lip, nose) to intracranial structures, leading to cavernous sinus thrombosis—a life-threatening condition.
6. Clinical: A patient with cavernous sinus thrombosis presents with inability to abduct the eye. Which nerve is affected first?
a) Oculomotor
b) Abducent
c) Trochlear
d) Ophthalmic
Answer: b) Abducent
Explanation: The abducent nerve runs through the central part of the cavernous sinus, making it the most susceptible to compression in cases of thrombosis or internal carotid aneurysm. Paralysis of the lateral rectus results in an inability to abduct the affected eye, causing medial deviation and diplopia.
7. The cavernous sinus drains posteriorly into which structure?
a) Sigmoid sinus
b) Superior sagittal sinus
c) Superior petrosal sinus
d) Straight sinus
Answer: c) Superior petrosal sinus
Explanation: The cavernous sinus drains posteriorly into the superior and inferior petrosal sinuses, which in turn drain into the transverse and internal jugular veins respectively. These sinuses provide a major route for venous outflow from the cavernous sinus toward the posterior cranial fossa.
8. Clinical: Cavernous sinus infection can spread from the face through which venous connection?
a) Pterygoid venous plexus
b) Emissary veins
c) Ophthalmic veins
d) Transverse sinus
Answer: c) Ophthalmic veins
Explanation: The superior and inferior ophthalmic veins connect the facial vein with the cavernous sinus. Because these veins lack valves, retrograde spread of infection from the “danger area” of the face can occur, resulting in cavernous sinus thrombosis and ophthalmoplegia.
9. Which cranial nerve does NOT pass through the cavernous sinus at any point?
a) Trochlear nerve
b) Oculomotor nerve
c) Optic nerve
d) Ophthalmic nerve
Answer: c) Optic nerve
Explanation: The optic nerve passes through the optic canal, not through the cavernous sinus. Cranial nerves III, IV, V1, V2, and VI are associated with the cavernous sinus, either within its wall or through its lumen. The optic nerve lies superior and medial to the sinus near the sphenoid bone.
10. The internal carotid artery within the cavernous sinus is accompanied by which nerve closely adhering to its lateral side?
a) Abducent nerve
b) Oculomotor nerve
c) Trochlear nerve
d) Maxillary nerve
Answer: a) Abducent nerve
Explanation: Inside the cavernous sinus, the abducent nerve runs lateral to the internal carotid artery. This close relationship makes the nerve vulnerable during carotid aneurysms, leading to lateral rectus palsy. Other nerves (III, IV, V1, V2) are located in the lateral wall of the sinus, not within the lumen.
11. Clinical: A patient with cavernous sinus thrombosis exhibits loss of corneal reflex. Which nerve carries the afferent limb of this reflex?
a) Maxillary nerve
b) Ophthalmic nerve
c) Trochlear nerve
d) Oculomotor nerve
Answer: b) Ophthalmic nerve
Explanation: The ophthalmic division (V1) of the trigeminal nerve carries the afferent limb of the corneal reflex, while the facial nerve provides the efferent limb via the orbicularis oculi. In cavernous sinus thrombosis, involvement of V1 leads to loss of corneal sensation and absence of the blink reflex, an important clinical sign for diagnosis.
Chapter: Neuroanatomy; Topic: Brainstem Pathways; Subtopic: Decussations in the Midbrain (Cerebral Peduncle)
Key Definitions:
• Cerebral peduncle: The ventral portion of the midbrain containing descending corticospinal, corticobulbar, and corticopontine tracts along with tegmental structures.
• Ventral tegmental decussation: The crossing of fibers of the rubrospinal tract within the midbrain tegmentum, connecting the red nucleus to the opposite side of the spinal cord.
• Rubrospinal tract: A descending motor pathway originating from the red nucleus that facilitates flexor muscle tone and inhibits extensor tone.
• Red nucleus: A large midbrain nucleus involved in motor coordination, located in the tegmentum at the level of the superior colliculus.
Lead Question (NEET PG 2015):
1. Ventral tegmental decussation in cerebral peduncle is due to -
a) Tectospinal tract
b) Tectobulbar tract
c) Vestibulospinal tract
d) Rubrospinal tract
Answer: d) Rubrospinal tract
Explanation: The ventral tegmental decussation, also known as the Forel’s decussation, is formed by the crossing fibers of the rubrospinal tract. These fibers arise from the red nucleus in the midbrain tegmentum, cross to the opposite side ventral to the oculomotor nucleus, and descend in the lateral funiculus of the spinal cord. The rubrospinal tract facilitates flexor muscles and inhibits extensor tone, contributing to motor coordination. Damage above the decussation produces contralateral motor deficits, while lesions below cause ipsilateral effects due to the crossing at the ventral tegmental level.
Guessed Questions (Related to Midbrain Decussations and Motor Pathways):
2. The dorsal tegmental decussation (of Meynert) is formed by fibers of which tract?
a) Tectospinal tract
b) Rubrospinal tract
c) Vestibulospinal tract
d) Reticulospinal tract
Answer: a) Tectospinal tract
Explanation: The dorsal tegmental decussation (Meynert’s decussation) is formed by the fibers of the tectospinal tract arising from the superior colliculus. These fibers cross dorsally and descend to influence head and neck movements in response to visual and auditory stimuli, coordinating reflex orientation of the head toward sensory cues.
3. The red nucleus, origin of the rubrospinal tract, is located at which level of the midbrain?
a) Inferior colliculus
b) Superior colliculus
c) Interpeduncular fossa
d) Pontomedullary junction
Answer: b) Superior colliculus
Explanation: The red nucleus lies in the tegmentum of the midbrain at the level of the superior colliculus. It receives afferents from the cerebellum and motor cortex, and gives rise to the rubrospinal tract that decussates ventrally to control flexor muscle tone on the opposite side.
4. The rubrospinal tract primarily facilitates which type of muscles?
a) Extensors
b) Flexors
c) Both equally
d) Neither flexors nor extensors
Answer: b) Flexors
Explanation: The rubrospinal tract enhances flexor muscle activity and inhibits extensor muscle tone, helping in the fine regulation of limb movements. It acts mainly on upper limb flexors and plays a compensatory role in motor control when corticospinal input is diminished.
5. Clinical: A lesion involving the red nucleus leads to which movement disorder?
a) Chorea
b) Tremor
c) Decerebrate rigidity
d) Athetosis
Answer: c) Decerebrate rigidity
Explanation: Lesions involving the red nucleus or below it (removing rubrospinal influence) produce decerebrate rigidity characterized by extension and pronation of limbs due to unopposed vestibulospinal and pontine reticulospinal activity. Above the red nucleus, decorticate rigidity occurs due to preserved rubrospinal facilitation of flexors.
6. Which of the following tracts crosses at the pyramidal decussation in the medulla?
a) Rubrospinal tract
b) Corticospinal tract
c) Tectospinal tract
d) Vestibulospinal tract
Answer: b) Corticospinal tract
Explanation: The corticospinal (pyramidal) tract decussates at the pyramidal decussation in the lower medulla, where approximately 80–90% of fibers cross to form the lateral corticospinal tract, controlling voluntary movement on the opposite side of the body.
7. The rubrospinal tract descends in which part of the spinal cord white matter?
a) Anterior funiculus
b) Lateral funiculus
c) Posterior funiculus
d) Central gray matter
Answer: b) Lateral funiculus
Explanation: After crossing at the ventral tegmental decussation, the rubrospinal tract descends in the lateral funiculus of the spinal cord, just anterior to the lateral corticospinal tract, modulating motor neurons of flexor muscles.
8. The rubrospinal tract terminates mainly in which spinal region?
a) Cervical region
b) Thoracic region
c) Lumbar region
d) Sacral region
Answer: a) Cervical region
Explanation: The rubrospinal tract predominantly influences cervical spinal levels, coordinating upper limb flexor muscles. It is less significant in humans compared to other mammals but aids in recovery of motor control after corticospinal lesions.
9. Clinical: A patient with a lesion above the red nucleus exhibits which posture?
a) Decerebrate rigidity
b) Decorticate rigidity
c) Flaccid paralysis
d) Spastic paraplegia
Answer: b) Decorticate rigidity
Explanation: A lesion above the red nucleus preserves rubrospinal activity, causing flexion of upper limbs and extension of lower limbs (decorticate rigidity). In contrast, lesions below the red nucleus eliminate rubrospinal input, resulting in decerebrate posture with extension of all limbs.
10. The tectospinal tract primarily mediates reflex movements of the head in response to which stimuli?
a) Visual and auditory stimuli
b) Somatosensory input
c) Olfactory stimuli
d) Pain sensation
Answer: a) Visual and auditory stimuli
Explanation: The tectospinal tract originates from the superior and inferior colliculi, integrating visual and auditory information to mediate reflexive head and neck movements toward stimuli. Its fibers decussate dorsally in the midbrain via Meynert’s decussation and descend to the cervical spinal cord.
11. Clinical: A lesion affecting the ventral tegmental decussation would most likely produce deficits on which side of the body?
a) Ipsilateral
b) Contralateral
c) Bilateral
d) Alternating
Answer: b) Contralateral
Explanation: Since fibers of the rubrospinal tract cross at the ventral tegmental decussation in the midbrain, a lesion at or above this decussation causes contralateral motor deficits, particularly affecting limb flexor tone. Below the decussation, effects are ipsilateral due to prior crossing of fibers.
Chapter: Neuroanatomy; Topic: Brainstem Nuclei and 4th Ventricle Anatomy; Subtopic: Nuclei Related to the Floor (Rhomboid Fossa) of the Fourth Ventricle
Key Definitions:
• Fourth ventricle (rhomboid fossa): A diamond-shaped cavity on the dorsal aspect of the pons and medulla; its floor (rhomboid fossa) is formed by the dorsal surfaces of the brainstem and contains visible surface landmarks for underlying cranial nerve nuclei.
• Abducens nucleus (VI): Motor nucleus located in the dorsal pons; its fibers supply the lateral rectus muscle and its surface landmark contributes to the facial colliculus (with facial nerve looping over it).
• Facial nucleus (VII): Motor nucleus located in the ventrolateral pons (not exposed on the midline floor of the fourth ventricle); its fibers loop dorsally around the abducens nucleus to form the facial colliculus before exiting laterally.
• Hypoglossal nucleus (XII): Motor nucleus in the medulla that lies adjacent to the midline and is represented on the floor of the fourth ventricle by the hypoglossal trigone.
• Dorsal vagal (dorsal motor nucleus of vagus, X): Parasympathetic motor nucleus located in the medulla, seen on the floor of the fourth ventricle as the vagal trigone lateral to the hypoglossal trigone.
Lead Question (NEET PG 2015):
1. Which of the following is NOT seen in the floor of the 4th ventricle?
a) Abducens nucleus
b) Facial nucleus
c) Dorsal vagal nucleus
d) Hypoglossal nucleus
Answer: b) Facial nucleus
Explanation (≈100 words): The floor of the fourth ventricle (rhomboid fossa) shows surface elevations that correspond to underlying nuclei. The hypoglossal nucleus lies beneath the hypoglossal trigone near the midline of the medullary portion; the dorsal vagal (dorsal motor nucleus of X) forms the vagal trigone lateral to it. The abducens nucleus lies in the dorsal pons and contributes to the facial colliculus where facial nerve fibres loop over it. The facial nucleus itself is located ventrolaterally in the pons and is not directly visible as a surface landmark on the midline floor. Thus, the facial nucleus is not seen in the rhomboid fossa surface anatomy.
Guessed Questions (Related to Brainstem Nuclei, Floor of 4th Ventricle and Clinical Correlates):
2. The hypoglossal trigone on the floor of the fourth ventricle corresponds to which nucleus?
a) Hypoglossal nucleus (XII)
b) Dorsal motor nucleus of vagus (X)
c) Nucleus ambiguus
d) Facial nucleus (VII)
Answer: a) Hypoglossal nucleus (XII)
Explanation (≈100 words): The hypoglossal trigone is a midline elevation on the medullary portion of the floor of the fourth ventricle that directly overlies the hypoglossal nucleus. The hypoglossal nucleus contains lower motor neurons for the intrinsic and extrinsic muscles of the tongue. Lesions affecting this nucleus or its exiting rootlets produce an ipsilateral flaccid paralysis of the tongue, causing deviation toward the side of the lesion on protrusion and atrophy over time. Clinically, hypoglossal palsy impairs speech articulation (dysarthria) and swallowing. The trigone thus is an important surface marker for this motor nucleus.
3. The vagal trigone on the floor of the fourth ventricle overlies which nucleus responsible for parasympathetic output to thoracic and abdominal viscera?
a) Nucleus ambiguus
b) Dorsal motor nucleus of vagus (X)
c) Solitary nucleus
d) Abducens nucleus
Answer: b) Dorsal motor nucleus of vagus (X)
Explanation (≈100 words): The vagal trigone is a lateral elevation on the rhomboid fossa corresponding to the dorsal motor nucleus of the vagus nerve. This nucleus contains preganglionic parasympathetic neurons whose fibers exit as the vagus nerve to provide parasympathetic innervation to thoracic and most abdominal viscera, influencing heart rate, bronchoconstriction, and gastrointestinal motility and secretion. Damage to this nucleus can disrupt autonomic control leading to cardiac dysregulation, impaired gastric motility, and other visceral dysfunctions. The nucleus ambiguus supplies branchiomotor fibers to pharyngeal and laryngeal muscles and contributes to vagal motor function but is not the primary parasympathetic nucleus seen as the vagal trigone.
4. Which nucleus forms the facial colliculus on the floor of the fourth ventricle due to the looping of facial nerve fibers over it?
a) Abducens nucleus (VI)
b) Facial nucleus (VII)
c) Vestibular nucleus
d) Hypoglossal nucleus (XII)
Answer: a) Abducens nucleus (VI)
Explanation (≈100 words): The facial colliculus is a rounded elevation on the pontine portion of the rhomboid fossa formed by fibers of the facial nerve (VII) looping dorsally and medially around the abducens nucleus (VI). Although commonly associated with the facial nerve, the protrusion itself reflects the underlying abducens nucleus and internal genu of the facial nerve. Clinically, lesions affecting the facial colliculus region can impair lateral gaze (abducens dysfunction) and facial expression (facial nerve fibers), causing ipsilateral horizontal diplopia and facial weakness. The facial nucleus proper lies ventrolaterally and is not directly visible on the midline floor.
5. The solitary nucleus, involved in visceral sensation and taste, is located relative to the floor of the fourth ventricle as:
a) A midline elevation (trigone)
b) A column of neurons lateral to the vagal trigone
c) Ventrolateral to the facial nucleus
d) In the pontine tegmentum only
Answer: b) A column of neurons lateral to the vagal trigone
Explanation (≈100 words): The nucleus tractus solitarius (nucleus of the solitary tract) is a longitudinal column extending through the medulla and lower pons, positioned lateral to the vagal trigone on the floor of the fourth ventricle. It receives visceral afferent fibers conveying taste (from CN VII, IX, X) and general visceral sensory information (baroreceptors, chemoreceptors). Dysfunction leads to impaired visceral sensation and altered autonomic reflexes, such as aberrant baroreceptor reflexes and taste disturbance. Because it is lateral and not a midline trigone, its surface landmarking differs from the hypoglossal and vagal trigones but it is still readily associated with the rhomboid fossa anatomy.
6. Clinical: A lesion affecting the dorsal aspect of caudal pons involving the facial colliculus is most likely to produce which combination?
a) Ipsilateral horizontal gaze palsy and ipsilateral facial paralysis
b) Contralateral facial paralysis only
c) Ipsilateral tongue weakness and dysarthria
d) Loss of gag reflex only
Answer: a) Ipsilateral horizontal gaze palsy and ipsilateral facial paralysis
Explanation (≈100 words): The facial colliculus overlies the abducens nucleus and the internal genu of the facial nerve. A lesion here can disrupt abducens nucleus function (causing impaired ipsilateral lateral gaze) and the traversing facial nerve fibers (producing ipsilateral lower motor neuron facial weakness). Clinically this produces inability to abduct the eye on the affected side and paralysis of ipsilateral facial muscles including inability to close eyelid, smile, or wrinkle forehead. Because both motor pathways are affected at the same dorsal pontine location, the combined gaze palsy plus facial palsy is characteristic of lesions involving the facial colliculus region.
7. Straight anatomical question: The nucleus ambiguus contributes motor fibres to which cranial nerves that are related to the floor of the fourth ventricle region?
a) Glossopharyngeal (IX) and Vagus (X) and accessory (XI) branchial motor fibers
b) Facial (VII) and Hypoglossal (XII)
c) Trigeminal (V) motor root only
d) Abducens (VI) only
Answer: a) Glossopharyngeal (IX) and Vagus (X) and accessory (XI) branchial motor fibers
Explanation (≈100 words): The nucleus ambiguus contains branchial motor neurons whose axons exit the brainstem as part of cranial nerves IX and X and contribute fibers to the cranial accessory component. These motor fibers innervate muscles of the pharynx, larynx, and soft palate, critical for swallowing, gag reflex, and phonation. Though not a surface midline trigone on the floor like hypoglossal or vagal trigones, its functional importance is evident clinically: lesions produce dysphagia, dysphonia, and loss of gag reflex. The nucleus ambiguus is located in the ventrolateral medulla adjacent to the dorsal vagal nucleus and solitary tract nuclei.
8. Clinical: A patient with a dorsal medullary infarct presents with dysphagia, hoarseness, and loss of gag reflex; which nucleus is most likely involved?
a) Dorsal motor nucleus of vagus (X)
b) Nucleus ambiguus
c) Hypoglossal nucleus
d) Abducens nucleus
Answer: b) Nucleus ambiguus
Explanation (≈100 words): Dysphagia, hoarseness, and impaired gag reflex indicate dysfunction of the branchial motor supply to pharyngeal and laryngeal muscles. The nucleus ambiguus provides branchiomotor fibers for cranial nerves IX and X that innervate these musculatures. Lesions in the dorsolateral medulla (e.g., lateral medullary/Wallenberg infarct) may damage adjacent nuclei and tracts, producing bulbar palsy-like symptoms. While the dorsal motor nucleus of X contributes parasympathetic outputs, the motor deficits described point more specifically to nucleus ambiguus involvement. Prompt recognition is critical as these lesions compromise airway protection and swallowing safety.
9. Which nucleus is the primary sensory nucleus for visceral afferents (taste and baroreceptor input) and lies adjacent to the floor of the fourth ventricle?
a) Spinal trigeminal nucleus
b) Nucleus tractus solitarius (solitary nucleus)
c) Vestibular nucleus
d) Inferior olivary nucleus
Answer: b) Nucleus tractus solitarius (solitary nucleus)
Explanation (≈100 words): The nucleus tractus solitarius receives visceral afferent fibers from cranial nerves VII, IX, and X carrying taste and visceral sensory signals including baroreceptor and chemoreceptor inputs. Located dorsolaterally in the medulla and extending into the pons, it lies lateral to the vagal trigone on the floor of the fourth ventricle. It integrates visceral sensory data and relays to autonomic and respiratory centers, influencing heart rate, respiration, and reflexes like gag and vomiting. Lesions can disrupt autonomic reflexes and taste sensation, producing clinically relevant cardiovascular and gastrointestinal dysregulation.
10. Straight anatomical: Which of the following nuclei is located most medially on the floor of the fourth ventricle?
a) Hypoglossal nucleus
b) Dorsal motor nucleus of vagus
c) Vestibular nuclei
d) Facial nucleus
Answer: a) Hypoglossal nucleus
Explanation (≈100 words): On the medullary portion of the rhomboid fossa, the hypoglossal trigone is the most medially placed surface landmark, overlying the hypoglossal nucleus near the midline. Lateral to it lies the vagal trigone (dorsal motor nucleus of X), and still more lateral and dorsal are vestibular nuclei. The facial nucleus is ventrolateral in the pons and not a midline floor landmark. Recognizing this medial-to-lateral arrangement is vital for localizing lesions: midline medullary lesions affect hypoglossal function (tongue weakness), while more lateral lesions affect vagal or vestibular functions.
11. Clinical: In a dorsal pontine lesion producing an apparent facial nerve LMN palsy with preserved forehead movement, which site is most likely affected?
a) Facial nucleus in the ventrolateral pons
b) Corticobulbar fibers in the internal capsule
c) Upper motor neuron supply in the cortex
d) Abducens nucleus only
Answer: a) Facial nucleus in the ventrolateral pons
Explanation (≈100 words): Lower motor neuron (LMN) facial palsy produces paralysis of both upper and lower facial muscles ipsilaterally, while upper motor neuron (UMN) lesions spare forehead (due to bilateral cortical innervation). A dorsal pontine lesion affecting the facial nerve nucleus or its intrapontine fibers (e.g., ventrolateral region) yields LMN signs including complete ipsilateral facial paralysis, hyperacusis (if stapedius affected), and possible taste disturbance (anterior two-thirds). Preservation of forehead movement would suggest an UMN lesion, but the presence of LMN pattern with other brainstem signs points to nucleus/fascicle pathology in the pons rather than cortical damage.
Chapter: Neuroanatomy; Topic: Cerebral Circulation; Subtopic: Blood Supply of Cerebral Hemispheres
Key Definitions:
• Cerebral hemisphere: The largest part of the brain divided into right and left halves, responsible for higher cognitive and motor functions.
• Anterior cerebral artery (ACA): A branch of the internal carotid artery supplying the medial surfaces of the frontal and parietal lobes.
• Middle cerebral artery (MCA): The largest branch of the internal carotid artery supplying the lateral surfaces of the cerebral hemispheres.
• Posterior cerebral artery (PCA): A branch of the basilar artery supplying the occipital lobe and inferomedial temporal lobe.
Lead Question (NEET PG 2015):
1. Major supply of medial surface of cerebral hemisphere:
a) Anterior cerebral artery
b) Posterior cerebral artery
c) Middle cerebral artery
d) Posterior inferior cerebellar artery
Answer: a) Anterior cerebral artery
Explanation: The anterior cerebral artery (ACA) primarily supplies the medial surface of the cerebral hemisphere, particularly the medial frontal and parietal lobes. It provides branches to the leg and foot areas of the motor and sensory cortices through the paracentral lobule. The ACA runs in the longitudinal fissure, anastomosing with its counterpart via the anterior communicating artery. Lesions of the ACA result in contralateral weakness and sensory loss predominantly affecting the lower limb, due to the topographic arrangement of the motor cortex (homunculus pattern).
Guessed Questions (Related to Cerebral Arterial Supply):
2. The lateral surface of the cerebral hemisphere is mainly supplied by:
a) Anterior cerebral artery
b) Middle cerebral artery
c) Posterior cerebral artery
d) Basilar artery
Answer: b) Middle cerebral artery
Explanation: The middle cerebral artery supplies the lateral convexity of the cerebral hemisphere, including areas for speech, upper limb, and face motor control. Occlusion causes contralateral hemiplegia affecting face and arm more than leg.
3. The visual cortex is supplied by which artery?
a) Middle cerebral artery
b) Posterior cerebral artery
c) Anterior cerebral artery
d) Basilar artery
Answer: b) Posterior cerebral artery
Explanation: The visual cortex in the occipital lobe receives blood from the posterior cerebral artery. Infarction leads to contralateral homonymous hemianopia with macular sparing if the macular region has collateral MCA supply.
4. Occlusion of the anterior cerebral artery causes weakness primarily in:
a) Contralateral upper limb
b) Contralateral lower limb
c) Ipsilateral lower limb
d) Face and tongue
Answer: b) Contralateral lower limb
Explanation: The ACA supplies the paracentral lobule, which controls motor and sensory functions of the lower limb. Occlusion results in contralateral paralysis and sensory loss mainly affecting the leg.
5. The artery supplying Broca’s and Wernicke’s areas is:
a) Anterior cerebral artery
b) Middle cerebral artery
c) Posterior cerebral artery
d) Internal carotid artery
Answer: b) Middle cerebral artery
Explanation: Broca’s and Wernicke’s speech areas are located on the lateral surface of the left hemisphere, supplied by the superior and inferior divisions of the middle cerebral artery, respectively. MCA stroke may cause aphasia.
6. The posterior cerebral artery is a branch of:
a) Internal carotid artery
b) Basilar artery
c) Middle cerebral artery
d) Vertebral artery
Answer: b) Basilar artery
Explanation: The posterior cerebral artery arises from the terminal bifurcation of the basilar artery. It supplies the occipital lobe, inferomedial temporal lobe, and thalamus through perforating branches.
7. A patient with right middle cerebral artery occlusion will present with:
a) Left-sided leg weakness
b) Left-sided face and arm weakness
c) Bilateral visual loss
d) Right-sided sensory loss
Answer: b) Left-sided face and arm weakness
Explanation: The MCA supplies the motor and sensory cortices for the face and upper limb. Contralateral weakness and sensory loss of these regions are classic features of MCA stroke.
8. The circle of Willis connects which arterial systems?
a) Internal carotid and vertebrobasilar systems
b) External carotid and vertebral systems
c) Jugular and vertebral systems
d) Dural sinuses and carotid system
Answer: a) Internal carotid and vertebrobasilar systems
Explanation: The circle of Willis provides an anastomotic connection between the carotid and vertebrobasilar circulations, ensuring collateral flow to cerebral tissue in case of vessel occlusion.
9. The paracentral lobule is mainly supplied by:
a) Anterior cerebral artery
b) Posterior cerebral artery
c) Middle cerebral artery
d) Basilar artery
Answer: a) Anterior cerebral artery
Explanation: The paracentral lobule lies on the medial surface of the hemisphere, supplied by the ACA. It represents the motor and sensory cortical areas for the lower limb in the somatotopic map.
10. Which artery supplies the thalamus?
a) Anterior cerebral artery
b) Posterior cerebral artery
c) Middle cerebral artery
d) Vertebral artery
Answer: b) Posterior cerebral artery
Explanation: The thalamus receives its arterial supply mainly from perforating branches of the posterior cerebral artery (PCA) and posterior communicating artery, which form part of the circle of Willis.
11. A lesion of the posterior cerebral artery typically causes which visual defect?
a) Bitemporal hemianopia
b) Contralateral homonymous hemianopia
c) Monocular blindness
d) Quadrantanopia
Answer: b) Contralateral homonymous hemianopia
Explanation: Damage to the visual cortex in the occipital lobe supplied by the posterior cerebral artery leads to contralateral homonymous hemianopia, often with macular sparing due to collateral MCA supply.
Chapter: Neuroanatomy; Topic: Venous Drainage of Brain; Subtopic: Superficial and Deep Cerebral Veins
Key Definitions:
• Superficial middle cerebral vein: A prominent vein on the lateral surface of the brain that drains the lateral aspect of the cerebral hemispheres into the cavernous sinus or sphenoparietal sinus.
• Cavernous sinus: A paired dural venous sinus located on either side of the sella turcica that receives venous blood from the orbit and superficial brain veins.
• Internal cerebral vein: A deep cerebral vein formed by the union of thalamostriate and choroidal veins that drains deep brain structures.
• Venous sinuses: Channels between the layers of the dura mater that collect venous blood from the brain and drain into the internal jugular veins.
Lead Question (NEET PG 2015):
1. Superficial middle cerebral vein drains into -
a) Internal cerebral vein
b) Cavernous sinus
c) Great cerebral vein of Galen
d) Straight sinus
Answer: b) Cavernous sinus
Explanation: The superficial middle cerebral vein runs along the lateral sulcus, draining blood from the lateral surface of the cerebral hemispheres. It usually empties into the cavernous sinus either directly or through the sphenoparietal sinus. Occasionally, it communicates with the superior sagittal sinus through the superior anastomotic vein (of Trolard) and with the transverse sinus through the inferior anastomotic vein (of Labbé). This venous network provides important collateral drainage pathways that protect against venous obstruction or increased intracranial pressure.
Guessed Questions (Related to Cerebral Venous Drainage):
2. The superior sagittal sinus drains primarily into which sinus?
a) Left transverse sinus
b) Right transverse sinus
c) Cavernous sinus
d) Sigmoid sinus
Answer: b) Right transverse sinus
Explanation: The superior sagittal sinus runs along the superior margin of the falx cerebri and usually drains posteriorly into the right transverse sinus, carrying venous blood from the cerebral cortex and meninges.
3. The great cerebral vein of Galen drains into which structure?
a) Inferior sagittal sinus
b) Straight sinus
c) Cavernous sinus
d) Sigmoid sinus
Answer: b) Straight sinus
Explanation: The great cerebral vein of Galen is formed by the union of the two internal cerebral veins and drains into the straight sinus, which continues posteriorly to the confluence of sinuses.
4. The inferior sagittal sinus joins with which vein to form the straight sinus?
a) Internal cerebral vein
b) Great cerebral vein of Galen
c) Superficial middle cerebral vein
d) Basal vein
Answer: b) Great cerebral vein of Galen
Explanation: The straight sinus is formed by the union of the inferior sagittal sinus and the great cerebral vein. It lies at the junction of the falx cerebri and tentorium cerebelli and drains deep cerebral venous blood.
5. The superior anastomotic vein (of Trolard) connects which two venous sinuses?
a) Cavernous and straight sinuses
b) Superior sagittal and superficial middle cerebral veins
c) Transverse and sigmoid sinuses
d) Inferior sagittal and occipital sinuses
Answer: b) Superior sagittal and superficial middle cerebral veins
Explanation: The vein of Trolard serves as a major communicating channel between the superficial middle cerebral vein and the superior sagittal sinus, assisting in collateral venous drainage of the cerebral cortex.
6. The inferior anastomotic vein (of Labbé) connects which venous structures?
a) Superficial middle cerebral vein and transverse sinus
b) Superior sagittal and straight sinuses
c) Cavernous and sigmoid sinuses
d) Internal cerebral vein and great vein of Galen
Answer: a) Superficial middle cerebral vein and transverse sinus
Explanation: The vein of Labbé connects the superficial middle cerebral vein to the transverse sinus, providing an alternate route for venous drainage from the lateral surface of the brain.
7. The cavernous sinus receives blood from all of the following except:
a) Superior ophthalmic vein
b) Superficial middle cerebral vein
c) Inferior petrosal sinus
d) Sphenoparietal sinus
Answer: c) Inferior petrosal sinus
Explanation: The inferior petrosal sinus drains the cavernous sinus into the internal jugular vein. It does not drain into it. The cavernous sinus receives blood from ophthalmic veins, superficial middle cerebral, and sphenoparietal sinuses.
8. The basal vein of Rosenthal drains into which vein?
a) Superior sagittal sinus
b) Cavernous sinus
c) Great cerebral vein of Galen
d) Straight sinus
Answer: c) Great cerebral vein of Galen
Explanation: The basal vein of Rosenthal drains deep cerebral structures including the medial temporal lobe and basal forebrain, joining the great cerebral vein before entering the straight sinus.
9. A thrombosis in the cavernous sinus may affect which cranial nerve first?
a) Optic nerve
b) Abducent nerve
c) Facial nerve
d) Hypoglossal nerve
Answer: b) Abducent nerve
Explanation: The abducent nerve (cranial nerve VI) lies freely within the cavernous sinus, close to the internal carotid artery, making it most vulnerable to compression or thrombosis of the sinus.
10. The vein of Galen malformation in infants can cause which of the following clinical findings?
a) Hydrocephalus
b) Cerebral ischemia
c) Hemianopia
d) Cerebellar ataxia
Answer: a) Hydrocephalus
Explanation: A vein of Galen malformation is a congenital arteriovenous communication that causes increased venous pressure and hydrocephalus due to impaired CSF absorption and high-output cardiac failure.
11. The internal cerebral vein is formed by the union of which two veins?
a) Thalamostriate and choroidal veins
b) Great cerebral and basal veins
c) Superior and inferior sagittal sinuses
d) Superficial middle and sphenoparietal veins
Answer: a) Thalamostriate and choroidal veins
Explanation: The internal cerebral vein is formed near the interventricular foramen by the joining of the thalamostriate and choroidal veins, which drain deep structures like the thalamus and caudate nucleus.
Chapter: Neuroanatomy; Topic: Venous Drainage of Brain; Subtopic: Great Cerebral Vein of Galen and Its Connections
Key Definitions:
• Great cerebral vein of Galen: A short midline vein formed by the union of two internal cerebral veins beneath the splenium of the corpus callosum that drains deep structures of the brain.
• Internal cerebral veins: Paired veins that drain deep cerebral structures including thalamus, caudate nucleus, and choroid plexus, joining to form the great cerebral vein.
• Straight sinus: A dural venous sinus formed by the union of the inferior sagittal sinus and the great cerebral vein, draining into the confluence of sinuses.
• Venous sinuses: Endothelial-lined spaces between layers of the dura mater that collect venous blood from the brain and drain into the internal jugular veins.
Lead Question (NEET PG 2015):
1. Great cerebral vein of Galen drains into -
a) Cavernous sinus
b) Basal vein
c) Internal cerebral vein
d) Straight sinus
Answer: d) Straight sinus
Explanation: The great cerebral vein of Galen is a short median vein formed by the union of the two internal cerebral veins beneath the splenium of the corpus callosum. It courses posteriorly to join the inferior sagittal sinus, forming the straight sinus. The straight sinus continues posteriorly to drain into the confluence of sinuses. The great vein drains deep structures of the cerebrum, such as thalami, basal ganglia, and deep white matter. Its blockage may cause venous congestion, hydrocephalus, or intracranial hypertension, particularly in neonates with vein of Galen malformations.
Guessed Questions (Related to Venous Drainage of Brain):
2. The internal cerebral veins are formed by the union of:
a) Thalamostriate and choroidal veins
b) Superior cerebral and inferior sagittal veins
c) Great cerebral and basal veins
d) Transverse and cavernous sinuses
Answer: a) Thalamostriate and choroidal veins
Explanation: Each internal cerebral vein is formed by the union of the thalamostriate vein and the choroidal vein near the interventricular foramen. These veins drain the thalamus, caudate nucleus, and choroid plexus before joining to form the great cerebral vein.
3. The straight sinus is formed by the union of:
a) Superior sagittal and transverse sinuses
b) Inferior sagittal sinus and great cerebral vein
c) Cavernous and petrosal sinuses
d) Sigmoid and occipital sinuses
Answer: b) Inferior sagittal sinus and great cerebral vein
Explanation: The straight sinus runs along the junction of the falx cerebri and tentorium cerebelli. It is formed by the union of the inferior sagittal sinus and the great cerebral vein, draining into the confluence of sinuses.
4. The basal vein of Rosenthal drains into which venous structure?
a) Cavernous sinus
b) Straight sinus
c) Great cerebral vein
d) Transverse sinus
Answer: c) Great cerebral vein
Explanation: The basal vein of Rosenthal, formed by the union of deep middle cerebral and anterior cerebral veins, drains the medial temporal lobe and basal forebrain structures and empties into the great cerebral vein of Galen.
5. A newborn with a vein of Galen malformation presents with heart failure. The underlying cause is:
a) Arteriovenous shunt increasing venous return
b) Obstruction of venous drainage
c) Thrombosis of superior sagittal sinus
d) Malformation of choroid plexus
Answer: a) Arteriovenous shunt increasing venous return
Explanation: Vein of Galen malformation is an arteriovenous fistula between cerebral arteries and the median prosencephalic vein, leading to increased venous return, high-output cardiac failure, and hydrocephalus in infants.
6. The superior sagittal sinus drains primarily into which sinus?
a) Sigmoid sinus
b) Straight sinus
c) Right transverse sinus
d) Cavernous sinus
Answer: c) Right transverse sinus
Explanation: The superior sagittal sinus drains posteriorly into the right transverse sinus at the confluence of sinuses, carrying venous blood from the superior cerebral veins and meninges.
7. The confluence of sinuses receives blood from all of the following except:
a) Superior sagittal sinus
b) Straight sinus
c) Transverse sinus
d) Cavernous sinus
Answer: d) Cavernous sinus
Explanation: The cavernous sinus does not drain into the confluence of sinuses. It drains via the superior and inferior petrosal sinuses into the sigmoid sinus and internal jugular vein.
8. In which dural fold is the straight sinus located?
a) Falx cerebri
b) Tentorium cerebelli
c) Falx cerebelli
d) Diaphragma sellae
Answer: b) Tentorium cerebelli
Explanation: The straight sinus lies at the junction of the falx cerebri and tentorium cerebelli. It drains posteriorly into the confluence of sinuses, conveying blood from deep cerebral structures.
9. The cavernous sinus receives blood directly from which vein?
a) Superior ophthalmic vein
b) Great cerebral vein
c) Straight sinus
d) Inferior sagittal sinus
Answer: a) Superior ophthalmic vein
Explanation: The superior ophthalmic vein drains blood from the orbit and communicates with the cavernous sinus, providing a route for infections to spread from the face or orbit to intracranial structures.
10. Obstruction of the straight sinus will primarily affect venous drainage from:
a) Cerebral cortex
b) Deep structures of brain
c) Orbit
d) Cerebellum only
Answer: b) Deep structures of brain
Explanation: The straight sinus receives blood from the great cerebral vein, which drains deep brain regions such as the thalamus, basal ganglia, and internal capsule. Its blockage leads to deep venous congestion and intracranial hypertension.
11. The internal cerebral veins drain which of the following structures?
a) Thalamus and caudate nucleus
b) Cerebral cortex
c) Cerebellum
d) Pons
Answer: a) Thalamus and caudate nucleus
Explanation: The internal cerebral veins drain deep gray matter structures, including the thalamus and caudate nucleus, as well as parts of the choroid plexus, converging posteriorly to form the great cerebral vein of Galen.
Chapter: Neuroanatomy; Topic: Cranial Nerves; Subtopic: Functional Classification of Cranial Nerves
Key Definitions:
• Sensory nerve: A nerve that carries afferent fibers transmitting impulses from sensory receptors toward the central nervous system.
• Motor nerve: A nerve that carries efferent fibers responsible for stimulating muscles and glands.
• Mixed nerve: A nerve containing both sensory and motor fibers, performing dual functions such as sensation and movement.
• Olfactory nerve (CN I): A pure sensory cranial nerve responsible for the sense of smell, composed of specialized bipolar neurons.
Lead Question (NEET PG 2015):
1. Which of the following is pure sensory nerve?
a) Trigeminal
b) Abducent
c) Trochlear
d) Olfactory
Answer: d) Olfactory
Explanation: The olfactory nerve (cranial nerve I) is a pure sensory nerve that carries impulses for the sense of smell. It consists of bipolar sensory neurons whose axons pass through the cribriform plate of the ethmoid bone to reach the olfactory bulb. From there, secondary neurons transmit signals to the olfactory cortex in the temporal lobe. In contrast, the trigeminal nerve is mixed, while the abducent and trochlear nerves are purely motor. The olfactory nerve’s sensory fibers are unique because they regenerate throughout life, unlike most other central nervous system neurons.
Guessed Questions (Related to Cranial Nerves):
2. Which of the following cranial nerves is purely motor?
a) Facial nerve
b) Oculomotor nerve
c) Abducent nerve
d) Trigeminal nerve
Answer: c) Abducent nerve
Explanation: The abducent nerve (cranial nerve VI) is a purely motor nerve that supplies the lateral rectus muscle, responsible for abducting the eye. Lesions lead to medial deviation (strabismus) and diplopia on lateral gaze.
3. Which of the following cranial nerves is mixed in function?
a) Glossopharyngeal nerve
b) Optic nerve
c) Trochlear nerve
d) Abducent nerve
Answer: a) Glossopharyngeal nerve
Explanation: The glossopharyngeal nerve (cranial nerve IX) carries both sensory and motor fibers — providing taste from the posterior third of the tongue, parasympathetic fibers to the parotid gland, and motor fibers to the stylopharyngeus muscle.
4. The optic nerve is a continuation of which structure embryologically?
a) Retina
b) Thalamus
c) Diencephalon
d) Cerebellum
Answer: c) Diencephalon
Explanation: The optic nerve is an outgrowth of the diencephalon and is covered by meninges. It transmits visual information from the retina to the lateral geniculate body in the thalamus.
5. Which cranial nerve does not arise from the brainstem?
a) Optic nerve
b) Trochlear nerve
c) Abducent nerve
d) Trigeminal nerve
Answer: a) Optic nerve
Explanation: The optic nerve originates from the diencephalon, not the brainstem. It is part of the central nervous system and carries visual impulses from retinal ganglion cells to the thalamus.
6. The sense of taste from the anterior two-thirds of the tongue is carried by which nerve?
a) Glossopharyngeal nerve
b) Facial nerve
c) Trigeminal nerve
d) Vagus nerve
Answer: b) Facial nerve
Explanation: The chorda tympani branch of the facial nerve carries special visceral afferent fibers for taste sensation from the anterior two-thirds of the tongue, synapsing in the nucleus tractus solitarius.
7. Damage to the abducent nerve results in which clinical manifestation?
a) Inability to move the eye laterally
b) Ptosis
c) Dilated pupil
d) Diplopia on upward gaze
Answer: a) Inability to move the eye laterally
Explanation: The abducent nerve supplies the lateral rectus muscle. Its palsy results in inability to abduct the eye, leading to convergent strabismus and diplopia during lateral gaze.
8. The trochlear nerve supplies which muscle of the eye?
a) Superior rectus
b) Superior oblique
c) Inferior rectus
d) Lateral rectus
Answer: b) Superior oblique
Explanation: The trochlear nerve (cranial nerve IV) is a pure motor nerve that supplies the superior oblique muscle, which depresses and intorts the eyeball when the eye is adducted.
9. Loss of smell (anosmia) occurs due to lesion in which nerve?
a) Optic nerve
b) Oculomotor nerve
c) Olfactory nerve
d) Trigeminal nerve
Answer: c) Olfactory nerve
Explanation: Damage to the olfactory nerve fibers or olfactory bulb can result in anosmia, often due to head trauma causing fracture of the cribriform plate or neurodegenerative conditions like Parkinson’s disease.
10. Which cranial nerve passes through the superior orbital fissure?
a) Olfactory nerve
b) Trochlear nerve
c) Optic nerve
d) Facial nerve
Answer: b) Trochlear nerve
Explanation: The trochlear nerve enters the orbit via the superior orbital fissure along with the oculomotor, abducent, and ophthalmic division of the trigeminal nerve to supply the superior oblique muscle.
11. A patient presents with loss of corneal sensation but normal blink reflex. Which nerve is affected?
a) Ophthalmic division of trigeminal
b) Facial nerve
c) Trochlear nerve
d) Abducent nerve
Answer: a) Ophthalmic division of trigeminal
Explanation: The ophthalmic division (V1) of the trigeminal nerve provides sensory fibers to the cornea. Its lesion causes loss of corneal sensation, while the blink reflex remains intact if the facial nerve (motor limb) is functional.
Chapter: Neuroanatomy; Topic: Cranial Nerve Nuclei; Subtopic: Oculomotor Nucleus and Its Location
Key Definitions:
• Oculomotor nerve (III): The third cranial nerve, responsible for most of the eye movements, eyelid elevation, and pupillary constriction.
• Oculomotor nucleus: The motor nucleus located in the midbrain that gives rise to the somatic motor fibers of the oculomotor nerve.
• Edinger–Westphal nucleus: The parasympathetic nucleus associated with the oculomotor nerve, controlling pupillary constriction and accommodation.
• Midbrain: The uppermost part of the brainstem containing nuclei for cranial nerves III and IV and serving as a pathway for motor and sensory tracts.
Lead Question (NEET PG 2015):
1. Oculomotor nucleus is located in -
a) Forebrain
b) Midbrain
c) Pons
d) Medulla
Answer: b) Midbrain
Explanation: The oculomotor nucleus is located in the midbrain at the level of the superior colliculus, near the midline, anterior to the cerebral aqueduct. It gives rise to fibers that form the oculomotor nerve, which exits the brainstem through the interpeduncular fossa. The associated Edinger–Westphal nucleus provides parasympathetic fibers to the sphincter pupillae and ciliary muscle. Together, these nuclei coordinate eye movements, eyelid elevation, and pupillary constriction, essential for visual focus and light reflexes.
Guessed Questions (Related to Oculomotor Nerve and Brainstem Nuclei):
2. The Edinger–Westphal nucleus is responsible for which of the following functions?
a) Eye abduction
b) Pupillary constriction
c) Eye depression
d) Eye elevation
Answer: b) Pupillary constriction
Explanation: The Edinger–Westphal nucleus provides preganglionic parasympathetic fibers that travel via the oculomotor nerve to the ciliary ganglion, which in turn innervates the sphincter pupillae for pupillary constriction.
3. The oculomotor nerve exits the brainstem through which area?
a) Pons
b) Interpeduncular fossa
c) Olive of medulla
d) Middle cerebellar peduncle
Answer: b) Interpeduncular fossa
Explanation: The oculomotor nerve emerges on the ventral aspect of the midbrain from the interpeduncular fossa, located between the cerebral peduncles, before entering the cavernous sinus and orbit.
4. A lesion in the oculomotor nerve results in which clinical feature?
a) Ptosis, mydriasis, and eye deviation down and out
b) Miosis, enophthalmos, and anhidrosis
c) Loss of corneal reflex
d) Nystagmus
Answer: a) Ptosis, mydriasis, and eye deviation down and out
Explanation: Oculomotor nerve palsy leads to paralysis of most extraocular muscles (except lateral rectus and superior oblique), causing the eye to deviate downward and outward, along with ptosis and a dilated pupil.
5. Which of the following cranial nerves emerges from the dorsal aspect of the brainstem?
a) Oculomotor nerve
b) Trochlear nerve
c) Abducent nerve
d) Trigeminal nerve
Answer: b) Trochlear nerve
Explanation: The trochlear nerve (cranial nerve IV) is the only cranial nerve to emerge dorsally from the brainstem, specifically from the dorsal midbrain below the inferior colliculus.
6. The oculomotor nerve supplies all extraocular muscles except:
a) Superior rectus
b) Lateral rectus
c) Inferior rectus
d) Medial rectus
Answer: b) Lateral rectus
Explanation: The oculomotor nerve innervates superior rectus, inferior rectus, medial rectus, and inferior oblique muscles. The lateral rectus is supplied by the abducent nerve (VI).
7. The oculomotor nerve nucleus lies at the level of which structure?
a) Superior colliculus
b) Inferior colliculus
c) Pons
d) Medulla oblongata
Answer: a) Superior colliculus
Explanation: The oculomotor nucleus lies in the midbrain at the level of the superior colliculus, adjacent to the midline, and anterior to the cerebral aqueduct.
8. Compression of the oculomotor nerve due to posterior communicating artery aneurysm leads to:
a) Miosis and enophthalmos
b) Ptosis and mydriasis
c) Diplopia only
d) Loss of corneal reflex
Answer: b) Ptosis and mydriasis
Explanation: Aneurysm of the posterior communicating artery compresses the oculomotor nerve, affecting parasympathetic fibers and causing pupillary dilation (mydriasis) and eyelid drooping (ptosis).
9. Which nucleus is involved in the accommodation reflex of the eye?
a) Oculomotor nucleus
b) Edinger–Westphal nucleus
c) Abducent nucleus
d) Trochlear nucleus
Answer: b) Edinger–Westphal nucleus
Explanation: The Edinger–Westphal nucleus mediates the parasympathetic component of the accommodation reflex by controlling ciliary muscle contraction and pupillary constriction for near vision.
10. A lesion in the midbrain affecting the oculomotor nucleus results in:
a) Contralateral lateral rectus paralysis
b) Ipsilateral oculomotor palsy
c) Bilateral visual loss
d) Horner’s syndrome
Answer: b) Ipsilateral oculomotor palsy
Explanation: A lesion involving the oculomotor nucleus or its fibers in the midbrain produces ipsilateral paralysis of the extraocular muscles supplied by CN III and loss of parasympathetic control to the pupil.
11. The parasympathetic fibers of the oculomotor nerve synapse in which ganglion?
a) Pterygopalatine ganglion
b) Otic ganglion
c) Submandibular ganglion
d) Ciliary ganglion
Answer: d) Ciliary ganglion
Explanation: The Edinger–Westphal nucleus sends preganglionic fibers via the oculomotor nerve to the ciliary ganglion, where they synapse before supplying the sphincter pupillae and ciliary muscle.
Chapter: Neuroanatomy; Topic: Brain and Ventricular System; Subtopic: Third Ventricle and Its Embryological Derivation
Key Definitions:
• Ventricular system: A series of interconnected cavities within the brain filled with cerebrospinal fluid (CSF).
• Diencephalon: The part of the forebrain containing the thalamus, hypothalamus, epithalamus, and subthalamus; surrounds the third ventricle.
• Third ventricle: A midline cavity between the two halves of the diencephalon that communicates with the lateral ventricles and cerebral aqueduct.
• Cerebrospinal fluid (CSF): A clear fluid produced by the choroid plexuses that cushions the brain and spinal cord, circulating through the ventricles and subarachnoid space.
Lead Question (NEET PG 2015):
1. Third ventricle is the cavity of -
a) Mesencephalon
b) Rhombencephalon
c) Diencephalon
d) Telencephalon
Answer: c) Diencephalon
Explanation: The third ventricle is the cavity of the diencephalon. It lies in the midline between the two halves of the thalamus and hypothalamus. The lateral walls are formed by the thalami, the floor by the hypothalamus, and the roof by the tela choroidea. The third ventricle communicates with each lateral ventricle through the interventricular foramen (of Monro) and with the fourth ventricle via the cerebral aqueduct. It plays an essential role in CSF circulation and acts as a narrow passage connecting higher and lower brain centers.
Guessed Questions (Related to Brain Ventricles and Embryology):
2. The lateral ventricles are cavities of which part of the brain?
a) Telencephalon
b) Diencephalon
c) Mesencephalon
d) Rhombencephalon
Answer: a) Telencephalon
Explanation: The lateral ventricles are paired cavities within the cerebral hemispheres (telencephalon). They communicate with the third ventricle through the interventricular foramina (of Monro) and contain choroid plexuses that secrete CSF.
3. The fourth ventricle is the cavity of which embryonic structure?
a) Mesencephalon
b) Rhombencephalon
c) Diencephalon
d) Telencephalon
Answer: b) Rhombencephalon
Explanation: The fourth ventricle develops from the rhombencephalon (hindbrain) and lies between the pons and medulla anteriorly and the cerebellum posteriorly. It communicates with the subarachnoid space via the foramina of Luschka and Magendie.
4. The cerebral aqueduct connects which two ventricles?
a) Lateral and third ventricles
b) Third and fourth ventricles
c) Fourth ventricle and central canal
d) Lateral ventricles
Answer: b) Third and fourth ventricles
Explanation: The cerebral aqueduct (aqueduct of Sylvius), located in the midbrain, connects the third ventricle of the diencephalon to the fourth ventricle of the hindbrain, allowing CSF to flow downward.
5. The choroid plexus of the third ventricle is formed by:
a) Pia mater and ependyma
b) Arachnoid mater and dura mater
c) Ependyma only
d) Pia mater only
Answer: a) Pia mater and ependyma
Explanation: The choroid plexus is a vascular structure formed by invagination of pia mater covered by ependymal cells. It secretes CSF into the ventricles, maintaining intracranial pressure and nutrient balance.
6. Obstruction of the cerebral aqueduct results in which condition?
a) Communicating hydrocephalus
b) Non-communicating hydrocephalus
c) Subdural effusion
d) CSF leak
Answer: b) Non-communicating hydrocephalus
Explanation: Obstruction of the cerebral aqueduct prevents CSF drainage from the third to fourth ventricle, leading to enlargement of lateral and third ventricles — a form of non-communicating (obstructive) hydrocephalus.
7. The floor of the third ventricle is formed by:
a) Thalamus
b) Hypothalamus
c) Epithalamus
d) Corpus callosum
Answer: b) Hypothalamus
Explanation: The hypothalamus forms the floor and lower part of the lateral walls of the third ventricle, containing important structures like the optic chiasma, tuber cinereum, and mammillary bodies.
8. A lesion causing blockage of the foramina of Luschka and Magendie would lead to accumulation of CSF in:
a) Third ventricle only
b) Fourth ventricle
c) Subarachnoid space
d) Spinal canal only
Answer: b) Fourth ventricle
Explanation: Blockage of the foramina of Luschka and Magendie prevents CSF outflow from the fourth ventricle into the subarachnoid space, leading to dilation of the fourth ventricle and raised intracranial pressure.
9. Which part of the brain surrounds the lateral walls of the third ventricle?
a) Midbrain
b) Thalamus
c) Pons
d) Cerebellum
Answer: b) Thalamus
Explanation: The thalamus forms the lateral walls of the third ventricle, serving as a major relay station for sensory and motor pathways and enclosing the midline cavity of the diencephalon.
10. The central canal of the spinal cord is a continuation of which ventricular structure?
a) Third ventricle
b) Fourth ventricle
c) Lateral ventricle
d) Cerebral aqueduct
Answer: b) Fourth ventricle
Explanation: The central canal of the spinal cord continues downward from the fourth ventricle and carries CSF through the spinal cord, maintaining communication between the cranial and spinal compartments.
11. In a case of pineal tumor, obstruction of the aqueduct of Sylvius can cause dilation of which ventricle?
a) Fourth ventricle
b) Third and lateral ventricles
c) Only third ventricle
d) Lateral ventricles only
Answer: b) Third and lateral ventricles
Explanation: Pineal tumors compress the cerebral aqueduct, blocking CSF flow from the third to fourth ventricle. This causes accumulation and dilation of the third and both lateral ventricles, leading to obstructive hydrocephalus.
Chapter: Neuroanatomy; Topic: Cranial Nerves; Subtopic: Facial Nerve and Its Parasympathetic Branches
Key Definitions:
• Facial nerve (VII): A mixed cranial nerve with motor, sensory, and parasympathetic components that supplies muscles of facial expression, lacrimal glands, and salivary glands (except parotid).
• Geniculate ganglion: A sensory ganglion located at the sharp bend (genu) of the facial canal that gives rise to the greater petrosal nerve.
• Greater petrosal nerve: A branch of the facial nerve carrying preganglionic parasympathetic fibers to the pterygopalatine ganglion for lacrimal and nasal gland secretion.
• Pterygopalatine ganglion: A parasympathetic ganglion in the pterygopalatine fossa where fibers from the greater petrosal nerve synapse before supplying lacrimal and nasal glands.
Lead Question (NEET PG 2015):
1. Greater petrosal nerve is formed from?
a) Geniculate ganglion
b) Plexus around ICA
c) Plexus around middle meningeal artery
d) None of the above
Answer: a) Geniculate ganglion
Explanation: The greater petrosal nerve arises from the geniculate ganglion of the facial nerve within the facial canal. It carries preganglionic parasympathetic fibers originating from the superior salivatory nucleus. These fibers pass through the foramen lacerum, join with the deep petrosal nerve (sympathetic fibers) to form the nerve of the pterygoid canal (Vidian nerve), and reach the pterygopalatine ganglion. Postganglionic fibers then innervate the lacrimal gland and nasal mucosal glands. The geniculate ganglion, therefore, gives rise to this crucial branch involved in lacrimation and nasal secretion.
Guessed Questions (Related to Facial Nerve and Parasympathetic Pathways):
2. The greater petrosal nerve carries which type of fibers?
a) Somatic motor
b) Special sensory
c) Preganglionic parasympathetic
d) Postganglionic sympathetic
Answer: c) Preganglionic parasympathetic
Explanation: The greater petrosal nerve carries preganglionic parasympathetic fibers from the facial nerve to the pterygopalatine ganglion for lacrimal and nasal gland secretion, facilitating moisture of eye and nasal mucosa.
3. The deep petrosal nerve carries which type of fibers?
a) Sympathetic
b) Parasympathetic
c) Sensory
d) Motor
Answer: a) Sympathetic
Explanation: The deep petrosal nerve carries postganglionic sympathetic fibers from the internal carotid plexus. It joins the greater petrosal nerve to form the nerve of the pterygoid canal (Vidian nerve), influencing vasoconstriction in nasal mucosa.
4. The nerve of the pterygoid canal (Vidian nerve) is formed by the union of:
a) Lesser petrosal and auriculotemporal nerves
b) Greater petrosal and deep petrosal nerves
c) Chorda tympani and glossopharyngeal nerves
d) Lingual and maxillary nerves
Answer: b) Greater petrosal and deep petrosal nerves
Explanation: The Vidian nerve is formed by joining the greater petrosal (parasympathetic) and deep petrosal (sympathetic) nerves within the foramen lacerum, transmitting mixed autonomic fibers to the pterygopalatine ganglion.
5. Preganglionic parasympathetic fibers from the greater petrosal nerve synapse in which ganglion?
a) Otic ganglion
b) Submandibular ganglion
c) Pterygopalatine ganglion
d) Ciliary ganglion
Answer: c) Pterygopalatine ganglion
Explanation: The greater petrosal nerve synapses in the pterygopalatine ganglion, from which postganglionic fibers innervate the lacrimal gland and mucous glands of the nasal cavity and palate.
6. A lesion at the geniculate ganglion would result in loss of:
a) Taste sensation and lacrimation
b) Hearing
c) Salivation from parotid gland
d) Smell perception
Answer: a) Taste sensation and lacrimation
Explanation: The geniculate ganglion gives rise to both the greater petrosal nerve (for lacrimation) and chorda tympani (for taste). Damage at this level results in loss of taste from anterior tongue and dry eyes.
7. The lacrimal gland receives its secretomotor fibers via:
a) Auriculotemporal nerve
b) Zygomatic nerve
c) Maxillary nerve
d) All of the above
Answer: d) All of the above
Explanation: Postganglionic fibers from the pterygopalatine ganglion reach the lacrimal gland through branches of the maxillary and zygomatic nerves, finally joining the lacrimal branch of the ophthalmic nerve.
8. Which cranial nerve provides preganglionic fibers to the pterygopalatine ganglion?
a) Oculomotor nerve
b) Glossopharyngeal nerve
c) Facial nerve
d) Vagus nerve
Answer: c) Facial nerve
Explanation: The facial nerve provides preganglionic parasympathetic fibers to the pterygopalatine ganglion via its greater petrosal branch, mediating secretion from lacrimal and nasal glands.
9. A patient presents with dry eyes due to lack of tear secretion. The lesion is most likely in which nerve?
a) Greater petrosal nerve
b) Chorda tympani
c) Glossopharyngeal nerve
d) Deep petrosal nerve
Answer: a) Greater petrosal nerve
Explanation: The greater petrosal nerve provides parasympathetic supply to the lacrimal gland. Damage to it leads to decreased tear secretion and dryness of the conjunctiva (xerophthalmia).
10. The chorda tympani nerve joins which other nerve in the infratemporal fossa?
a) Auriculotemporal nerve
b) Lingual nerve
c) Inferior alveolar nerve
d) Buccal nerve
Answer: b) Lingual nerve
Explanation: The chorda tympani joins the lingual nerve in the infratemporal fossa to carry taste fibers from the anterior two-thirds of the tongue and parasympathetic fibers to the submandibular and sublingual glands.
11. The superior salivatory nucleus gives rise to parasympathetic fibers for which glands?
a) Parotid gland
b) Lacrimal, submandibular, and sublingual glands
c) Thyroid gland
d) Sebaceous glands
Answer: b) Lacrimal, submandibular, and sublingual glands
Explanation: The superior salivatory nucleus in the pons provides preganglionic parasympathetic fibers through the facial nerve to control secretions from lacrimal, submandibular, and sublingual glands.
Chapter: Neuroanatomy; Topic: Cranial Nerves; Subtopic: Trigeminal Nerve – Anatomy and Functions
Key Definitions:
• Cranial nerves: Twelve pairs of nerves arising directly from the brain or brainstem, supplying structures of the head, neck, and thorax.
• Trigeminal nerve (V): The largest cranial nerve with both sensory and motor components, responsible for facial sensation and mastication.
• Gasserian (trigeminal) ganglion: A sensory ganglion located in Meckel’s cave, containing cell bodies of the trigeminal sensory neurons.
• Divisions of trigeminal nerve: Ophthalmic (V1), maxillary (V2), and mandibular (V3) divisions that transmit sensory information from face and oral cavity.
Lead Question (NEET PG 2015):
1. Largest cranial nerve is:
a) Trochlear
b) Trigeminal
c) Oculomotor
d) Vagus
Answer: b) Trigeminal
Explanation: The trigeminal nerve (cranial nerve V) is the largest of all cranial nerves. It is both sensory and motor in nature. The sensory root conveys sensation from the face, scalp, cornea, nasal and oral cavities, while the motor root supplies the muscles of mastication. It emerges from the pons as two roots — a large sensory and a smaller motor root. The trigeminal ganglion (Gasserian) lies in Meckel’s cave in the middle cranial fossa. Its extensive distribution explains its large size compared to other cranial nerves.
Guessed Questions (Related to Trigeminal Nerve and Cranial Nerves):
2. The trigeminal nerve is primarily responsible for which of the following functions?
a) Movement of the tongue
b) Facial sensation and mastication
c) Eye movement
d) Hearing
Answer: b) Facial sensation and mastication
Explanation: The trigeminal nerve carries sensory information from the face, scalp, and oral cavity, and motor fibers to muscles of mastication, such as masseter and temporalis, enabling chewing movements.
3. The sensory ganglion of the trigeminal nerve is located in:
a) Foramen ovale
b) Meckel’s cave
c) Cavernous sinus
d) Middle ear
Answer: b) Meckel’s cave
Explanation: The trigeminal (Gasserian) ganglion resides in a dural recess called Meckel’s cave, located in the middle cranial fossa near the apex of the petrous temporal bone, containing sensory neuron cell bodies.
4. Which branch of the trigeminal nerve has both sensory and motor components?
a) Ophthalmic
b) Maxillary
c) Mandibular
d) Zygomatic
Answer: c) Mandibular
Explanation: The mandibular division (V3) of the trigeminal nerve carries both sensory fibers (to lower face, jaw, and oral mucosa) and motor fibers to muscles of mastication and tensor tympani.
5. The sensory root of the trigeminal nerve originates from which part of the brainstem?
a) Midbrain
b) Pons
c) Medulla oblongata
d) Cerebellum
Answer: b) Pons
Explanation: The trigeminal nerve arises from the lateral aspect of the pons by a large sensory root and a small motor root. Its sensory fibers terminate in the trigeminal sensory nuclei extending through the brainstem.
6. A patient presents with loss of corneal reflex but normal vision. Which nerve is affected?
a) Optic nerve
b) Oculomotor nerve
c) Trigeminal nerve
d) Trochlear nerve
Answer: c) Trigeminal nerve
Explanation: The corneal reflex afferent limb is mediated by the ophthalmic division (V1) of the trigeminal nerve. Its loss indicates a lesion of V1 or its nucleus, even when vision remains intact.
7. Which of the following muscles is supplied by the mandibular division of the trigeminal nerve?
a) Orbicularis oculi
b) Masseter
c) Buccinator
d) Sternocleidomastoid
Answer: b) Masseter
Explanation: The mandibular nerve supplies muscles of mastication — masseter, temporalis, medial and lateral pterygoids — through its motor root, enabling closure and movement of the jaw.
8. The mandibular nerve exits the skull through which foramen?
a) Foramen rotundum
b) Foramen ovale
c) Foramen spinosum
d) Jugular foramen
Answer: b) Foramen ovale
Explanation: The mandibular division of the trigeminal nerve exits the cranial cavity via the foramen ovale in the sphenoid bone to enter the infratemporal fossa, where it branches extensively.
9. Which branch of the trigeminal nerve passes through the superior orbital fissure?
a) Ophthalmic
b) Maxillary
c) Mandibular
d) None
Answer: a) Ophthalmic
Explanation: The ophthalmic division (V1) of the trigeminal nerve traverses the superior orbital fissure to enter the orbit, supplying sensory innervation to the forehead, cornea, and upper eyelid.
10. In trigeminal neuralgia, pain is most commonly felt in which branch?
a) Ophthalmic
b) Maxillary
c) Mandibular
d) Maxillary and mandibular
Answer: d) Maxillary and mandibular
Explanation: Trigeminal neuralgia is characterized by intense, paroxysmal facial pain, commonly involving the maxillary (V2) and mandibular (V3) divisions due to compression or irritation of the trigeminal root.
11. A lesion in the mandibular nerve would result in which of the following clinical findings?
a) Loss of jaw jerk reflex
b) Ptosis
c) Loss of taste
d) Loss of hearing
Answer: a) Loss of jaw jerk reflex
Explanation: The jaw jerk reflex depends on the motor and sensory fibers of the mandibular nerve (V3). A lesion results in absence of this reflex and weakness of masticatory muscles on the affected side.
Chapter: Neuroanatomy; Topic: Cranial Nerves; Subtopic: Trochlear Nerve and Its Anatomical Course
Key Definitions:
• Trochlear nerve (IV): The smallest cranial nerve that supplies the superior oblique muscle of the eye and is the only nerve to emerge dorsally from the brainstem.
• Intracranial course: The distance traveled by a cranial nerve within the cranial cavity before it exits through a foramen.
• Superior oblique muscle: An extraocular muscle responsible for depression and intorsion of the eyeball.
• Brainstem exit: The trochlear nerve uniquely emerges from the dorsal aspect of the midbrain and decussates completely before doing so.
Lead Question (NEET PG 2015):
1. The nerve which has the longest intracranial course is:
a) Fourth cranial nerve
b) Third cranial nerve
c) Sixth cranial nerve
d) Fifth cranial nerve
Answer: a) Fourth cranial nerve
Explanation: The trochlear nerve (cranial nerve IV) has the longest intracranial course among all cranial nerves. It arises from the dorsal aspect of the midbrain, decussates completely within the midbrain, and courses anteriorly around the brainstem to enter the cavernous sinus. It then enters the orbit through the superior orbital fissure to supply the superior oblique muscle. Despite being the thinnest cranial nerve, its long intracranial course makes it highly susceptible to trauma, especially in head injuries involving shearing forces.
Guessed Questions (Related to Trochlear and Other Cranial Nerves):
2. Which cranial nerve is the only one to emerge from the dorsal aspect of the brainstem?
a) Oculomotor nerve
b) Trochlear nerve
c) Abducent nerve
d) Optic nerve
Answer: b) Trochlear nerve
Explanation: The trochlear nerve is unique as it emerges dorsally from the midbrain, below the inferior colliculus. It completely decussates before exiting, supplying the contralateral superior oblique muscle.
3. Which cranial nerve has the longest extracranial course?
a) Facial nerve
b) Vagus nerve
c) Abducent nerve
d) Glossopharyngeal nerve
Answer: b) Vagus nerve
Explanation: The vagus nerve (cranial nerve X) has the longest extracranial course, extending from the medulla oblongata to the thoracic and abdominal organs, influencing heart rate, digestion, and respiratory function.
4. A lesion of the trochlear nerve leads to which of the following clinical features?
a) Diplopia on looking up
b) Diplopia on looking down and in
c) Loss of lateral gaze
d) Ptosis and mydriasis
Answer: b) Diplopia on looking down and in
Explanation: The trochlear nerve supplies the superior oblique muscle, which depresses and intorts the eyeball. Lesion results in vertical diplopia, especially noticeable when the patient looks down and in (e.g., reading or walking downstairs).
5. The trochlear nerve supplies which extraocular muscle?
a) Superior rectus
b) Inferior oblique
c) Superior oblique
d) Medial rectus
Answer: c) Superior oblique
Explanation: The trochlear nerve supplies only the superior oblique muscle, which acts to depress, abduct, and intort the eyeball, especially when the eye is adducted.
6. Which cranial nerve nucleus is located at the level of the inferior colliculus?
a) Oculomotor nucleus
b) Trochlear nucleus
c) Abducent nucleus
d) Facial nucleus
Answer: b) Trochlear nucleus
Explanation: The trochlear nucleus is situated in the midbrain at the level of the inferior colliculus. Fibers from this nucleus decussate in the midbrain before emerging dorsally to form the trochlear nerve.
7. A patient presents with head tilt to the opposite side and vertical diplopia. Which nerve is most likely affected?
a) Oculomotor nerve
b) Trochlear nerve
c) Abducent nerve
d) Trigeminal nerve
Answer: b) Trochlear nerve
Explanation: In trochlear nerve palsy, patients tilt their head to the side opposite the lesion to minimize diplopia caused by paralysis of the superior oblique muscle, which normally intorts the eye.
8. Which of the following cranial nerves decussates completely before emerging from the brainstem?
a) Optic nerve
b) Trochlear nerve
c) Abducent nerve
d) Facial nerve
Answer: b) Trochlear nerve
Explanation: The trochlear nerve is the only cranial nerve that undergoes complete decussation within the brainstem. Each trochlear nucleus supplies the contralateral superior oblique muscle.
9. Which nerve passes through the cavernous sinus along with the internal carotid artery?
a) Trochlear nerve
b) Abducent nerve
c) Optic nerve
d) Olfactory nerve
Answer: b) Abducent nerve
Explanation: The abducent nerve (VI) passes through the cavernous sinus close to the lateral wall of the internal carotid artery, making it vulnerable to injury in cavernous sinus thrombosis.
10. Which cranial nerve lesion produces both ptosis and mydriasis?
a) Trochlear nerve
b) Oculomotor nerve
c) Abducent nerve
d) Optic nerve
Answer: b) Oculomotor nerve
Explanation: The oculomotor nerve supplies the levator palpebrae superioris (eyelid elevation) and carries parasympathetic fibers to the sphincter pupillae. A lesion causes ptosis and pupil dilation (mydriasis).
11. The thinnest cranial nerve is:
a) Trochlear nerve
b) Oculomotor nerve
c) Abducent nerve
d) Optic nerve
Answer: a) Trochlear nerve
Explanation: The trochlear nerve is the smallest in diameter among cranial nerves but has the longest intracranial course. Despite its small size, it plays a crucial role in eye movement control, particularly in depression during adduction.
Chapter: Neuroanatomy; Topic: Cranial Nerves; Subtopic: Facial Nerve and Its Lesions
Key Definitions:
• Facial nerve (VII): A mixed cranial nerve with motor, sensory, and parasympathetic fibers supplying muscles of facial expression, taste to anterior tongue, and secretomotor fibers to salivary and lacrimal glands.
• Chorda tympani: A branch of the facial nerve carrying taste fibers from the anterior two-thirds of the tongue and secretomotor fibers to submandibular and sublingual glands.
• Greater petrosal nerve: Parasympathetic branch of the facial nerve supplying the lacrimal gland.
• Facial palsy: Paralysis of facial muscles due to lesion of the facial nerve, leading to asymmetry of the face and loss of expression.
Lead Question (NEET PG 2015):
1. Features of facial nerve palsy are all except -
a) Loss of salivation
b) Loss of lacrimation
c) Facial muscle paralysis
d) Loss of taste sensation from posterior tongue
Answer: d) Loss of taste sensation from posterior tongue
Explanation: The facial nerve supplies taste fibers to the anterior two-thirds of the tongue via the chorda tympani. Taste from the posterior one-third of the tongue is supplied by the glossopharyngeal nerve (IX). Therefore, facial nerve palsy leads to loss of taste from the anterior tongue, not the posterior. Other features include paralysis of muscles of facial expression, loss of salivation from submandibular and sublingual glands, and loss of lacrimation due to greater petrosal nerve involvement. Hence, the correct answer is “Loss of taste sensation from posterior tongue.”
Guessed Questions (Related to Facial Nerve and Its Lesions):
2. The facial nerve exits the skull through which foramen?
a) Foramen ovale
b) Stylomastoid foramen
c) Jugular foramen
d) Internal acoustic meatus
Answer: b) Stylomastoid foramen
Explanation: After passing through the facial canal in the temporal bone, the facial nerve exits the skull via the stylomastoid foramen and enters the parotid gland, where it divides into its five terminal branches supplying facial muscles.
3. Which branch of the facial nerve carries taste sensation?
a) Greater petrosal nerve
b) Chorda tympani
c) Auriculotemporal nerve
d) Lingual nerve
Answer: b) Chorda tympani
Explanation: The chorda tympani branch carries taste sensation from the anterior two-thirds of the tongue and parasympathetic fibers to submandibular and sublingual glands via the lingual nerve.
4. A patient presents with inability to close his eye and drooping of the mouth on one side. Which nerve is affected?
a) Trigeminal nerve
b) Facial nerve
c) Glossopharyngeal nerve
d) Accessory nerve
Answer: b) Facial nerve
Explanation: Damage to the facial nerve causes paralysis of orbicularis oculi and orbicularis oris muscles, leading to inability to close the eye and drooping of the mouth corner — classic features of facial nerve palsy.
5. Bell’s palsy involves a lesion of which structure?
a) Upper motor neuron
b) Lower motor neuron of facial nerve
c) Glossopharyngeal nerve
d) Trigeminal nerve
Answer: b) Lower motor neuron of facial nerve
Explanation: Bell’s palsy is a lower motor neuron lesion of the facial nerve, resulting in flaccid paralysis of all muscles of facial expression on the affected side, including the forehead.
6. Which of the following structures is not supplied by the facial nerve?
a) Lacrimal gland
b) Submandibular gland
c) Parotid gland
d) Sublingual gland
Answer: c) Parotid gland
Explanation: The parotid gland is supplied by parasympathetic fibers from the glossopharyngeal nerve, not the facial nerve. The facial nerve passes through but does not innervate it.
7. A lesion of the facial nerve proximal to the geniculate ganglion will result in:
a) Only facial paralysis
b) Loss of lacrimation, salivation, and taste
c) Loss of corneal reflex only
d) Loss of hearing
Answer: b) Loss of lacrimation, salivation, and taste
Explanation: Lesion proximal to the geniculate ganglion affects all branches — greater petrosal (lacrimation), chorda tympani (taste and salivation), and motor fibers, leading to multiple deficits.
8. The greater petrosal nerve carries parasympathetic fibers to:
a) Parotid gland
b) Lacrimal gland
c) Submandibular gland
d) Sublingual gland
Answer: b) Lacrimal gland
Explanation: The greater petrosal nerve, a branch of the facial nerve, carries preganglionic parasympathetic fibers to the pterygopalatine ganglion, from where postganglionic fibers supply the lacrimal gland to stimulate tear secretion.
9. Which muscle of the middle ear is supplied by the facial nerve?
a) Tensor tympani
b) Stapedius
c) Tensor veli palatini
d) Levator veli palatini
Answer: b) Stapedius
Explanation: The stapedius muscle, supplied by the facial nerve, dampens the vibrations of the stapes to protect the inner ear from loud sounds. Paralysis causes hyperacusis (increased sound sensitivity).
10. In upper motor neuron facial palsy, which part of the face is spared?
a) Forehead
b) Lower face
c) Both sides
d) Eye muscles
Answer: a) Forehead
Explanation: In UMN facial palsy, the upper face (forehead) is spared due to bilateral cortical representation, while the contralateral lower facial muscles are affected.
11. After a temporal bone fracture, a patient presents with facial paralysis, loss of taste, and hyperacusis. The lesion is likely at which site?
a) At the stylomastoid foramen
b) At the facial canal proximal to chorda tympani
c) At the geniculate ganglion
d) At the internal acoustic meatus
Answer: b) At the facial canal proximal to chorda tympani
Explanation: A lesion at this site affects motor fibers (facial paralysis), chorda tympani (loss of taste), and stapedius (hyperacusis). The location explains the combination of these clinical findings.
Chapter: Head and Neck Anatomy; Topic: Parotid Gland; Subtopic: Parasympathetic (Secretomotor) Innervation of the Parotid Gland
Key Definitions:
• Parotid gland: The largest salivary gland situated in front of the ear, secreting serous saliva that aids digestion.
• Secretomotor fibers: Parasympathetic fibers that stimulate secretion from glands such as salivary and lacrimal glands.
• Tympanic plexus: A network of nerves in the middle ear formed by the tympanic branch of the glossopharyngeal nerve, giving rise to the lesser petrosal nerve.
• Otic ganglion: A parasympathetic ganglion related to the mandibular nerve (V3) that relays secretomotor fibers to the parotid gland via the auriculotemporal nerve.
Lead Question (NEET PG 2015):
1. Secretomotor fibers to parotid glands are through -
a) Tympanic plexus
b) Geniculate ganglion
c) Greater petrosal nerve
d) None
Answer: a) Tympanic plexus
Explanation: The secretomotor (parasympathetic) pathway to the parotid gland originates in the inferior salivatory nucleus of the medulla. Preganglionic fibers pass through the glossopharyngeal nerve (CN IX) and its tympanic branch, which forms the tympanic plexus in the middle ear. From this plexus arises the lesser petrosal nerve, which carries fibers to the otic ganglion. After synapsing there, postganglionic fibers reach the parotid gland via the auriculotemporal nerve (a branch of the mandibular nerve, V3). Thus, the tympanic plexus is the route for secretomotor fibers to the parotid gland.
Guessed Questions (Related to Parotid Gland and its Nerve Supply):
2. Which nerve carries preganglionic parasympathetic fibers to the parotid gland?
a) Facial nerve
b) Glossopharyngeal nerve
c) Vagus nerve
d) Trigeminal nerve
Answer: b) Glossopharyngeal nerve
Explanation: The glossopharyngeal nerve carries preganglionic fibers from the inferior salivatory nucleus to the parotid gland through the tympanic branch and lesser petrosal nerve, making it the main secretomotor nerve of the gland.
3. The otic ganglion is functionally associated with which cranial nerve?
a) Trigeminal nerve
b) Glossopharyngeal nerve
c) Facial nerve
d) Vagus nerve
Answer: b) Glossopharyngeal nerve
Explanation: Although anatomically attached to the mandibular division of the trigeminal nerve (V3), the otic ganglion receives its preganglionic fibers from the glossopharyngeal nerve and is thus functionally linked to CN IX.
4. Postganglionic fibers from the otic ganglion reach the parotid gland through:
a) Auriculotemporal nerve
b) Lingual nerve
c) Inferior alveolar nerve
d) Buccal nerve
Answer: a) Auriculotemporal nerve
Explanation: The auriculotemporal nerve, a branch of the mandibular nerve, carries postganglionic parasympathetic fibers from the otic ganglion to the parotid gland, completing the secretomotor pathway.
5. The lesser petrosal nerve arises from which structure?
a) Tympanic plexus
b) Geniculate ganglion
c) Superior cervical ganglion
d) Pterygopalatine ganglion
Answer: a) Tympanic plexus
Explanation: The lesser petrosal nerve originates from the tympanic plexus, carrying preganglionic parasympathetic fibers from the glossopharyngeal nerve to the otic ganglion before reaching the parotid gland.
6. A lesion at the otic ganglion will cause:
a) Dryness of mouth due to decreased parotid secretion
b) Facial paralysis
c) Loss of taste on anterior tongue
d) Decreased lacrimation
Answer: a) Dryness of mouth due to decreased parotid secretion
Explanation: The otic ganglion mediates parasympathetic transmission to the parotid gland. Damage interrupts this pathway, resulting in reduced salivation and a dry mouth.
7. Frey’s syndrome following parotidectomy results from regeneration of fibers of:
a) Auriculotemporal nerve
b) Glossopharyngeal nerve
c) Facial nerve
d) Lingual nerve
Answer: a) Auriculotemporal nerve
Explanation: During healing after parotidectomy, parasympathetic fibers from the auriculotemporal nerve aberrantly regenerate to sweat glands, causing gustatory sweating (Frey’s syndrome) when eating.
8. The parotid duct opens opposite which tooth in the oral cavity?
a) Upper first molar
b) Upper second molar
c) Lower first molar
d) Upper canine
Answer: b) Upper second molar
Explanation: The Stensen’s (parotid) duct passes horizontally across the masseter muscle, pierces the buccinator, and opens into the vestibule of the mouth opposite the upper second molar tooth.
9. Inflammation of the parotid gland (mumps) is painful due to involvement of which nerve?
a) Auriculotemporal nerve
b) Facial nerve
c) Glossopharyngeal nerve
d) Mandibular nerve
Answer: a) Auriculotemporal nerve
Explanation: The parotid capsule, derived from deep cervical fascia, is supplied by the auriculotemporal nerve. Swelling of the gland stretches this capsule, causing severe pain radiating to the ear.
10. Which cranial nerve nucleus gives rise to the preganglionic fibers for the parotid gland?
a) Superior salivatory nucleus
b) Inferior salivatory nucleus
c) Dorsal motor nucleus of vagus
d) Nucleus ambiguus
Answer: b) Inferior salivatory nucleus
Explanation: The inferior salivatory nucleus in the medulla oblongata gives rise to preganglionic parasympathetic fibers that travel via the glossopharyngeal nerve to supply the parotid gland.
11. Which ganglion mediates the secretomotor fibers for the parotid gland?
a) Pterygopalatine ganglion
b) Otic ganglion
c) Submandibular ganglion
d) Ciliary ganglion
Answer: b) Otic ganglion
Explanation: The otic ganglion is the parasympathetic relay center for the parotid gland. It receives fibers from the lesser petrosal nerve and sends postganglionic fibers to the gland through the auriculotemporal nerve.
Chapter: Head and Neck Anatomy; Topic: Parotid Gland; Subtopic: Parasympathetic Innervation of the Parotid Gland
Key Definitions:
• Parotid gland: The largest salivary gland located anteroinferior to the external acoustic meatus, secreting serous saliva to aid digestion.
• Glossopharyngeal nerve (CN IX): Cranial nerve providing preganglionic parasympathetic fibers to the parotid gland through the lesser petrosal nerve.
• Auriculotemporal nerve: A branch of the mandibular nerve (V3) carrying postganglionic parasympathetic fibers from the otic ganglion to the parotid gland.
• Otic ganglion: A small parasympathetic ganglion situated below the foramen ovale that relays fibers to the parotid gland.
Lead Question (NEET PG 2015):
1. Postganglionic fibres to parotid gland is supplied by?
a) Glossopharyngeal nerve
b) Auriculotemporal nerve
c) Both of the above
d) None of the above
Answer: b) Auriculotemporal nerve
Explanation: The parasympathetic secretomotor pathway to the parotid gland begins in the inferior salivatory nucleus. Preganglionic fibers travel via the glossopharyngeal nerve (CN IX) and its branch, the lesser petrosal nerve, to the otic ganglion, where they synapse. Postganglionic fibers then travel through the auriculotemporal nerve, a branch of the mandibular nerve (V3), to reach the parotid gland. This pathway stimulates salivary secretion. Thus, the auriculotemporal nerve provides postganglionic fibers, while the glossopharyngeal nerve provides preganglionic fibers.
Guessed Questions (Related to Parotid Gland Innervation):
2. Preganglionic parasympathetic fibers to the parotid gland are carried by:
a) Facial nerve
b) Glossopharyngeal nerve
c) Vagus nerve
d) Trigeminal nerve
Answer: b) Glossopharyngeal nerve
Explanation: The glossopharyngeal nerve provides preganglionic parasympathetic fibers via its tympanic branch and the lesser petrosal nerve to the otic ganglion, which relay to the parotid gland for salivation.
3. The otic ganglion is functionally related to which cranial nerve?
a) Trigeminal nerve
b) Glossopharyngeal nerve
c) Facial nerve
d) Vagus nerve
Answer: b) Glossopharyngeal nerve
Explanation: Though anatomically associated with the mandibular division of the trigeminal nerve, the otic ganglion is functionally connected to the glossopharyngeal nerve, which supplies its preganglionic fibers.
4. Frey’s syndrome (gustatory sweating) occurs due to injury to:
a) Auriculotemporal nerve
b) Lingual nerve
c) Facial nerve
d) Glossopharyngeal nerve
Answer: a) Auriculotemporal nerve
Explanation: In Frey’s syndrome, injury to the auriculotemporal nerve during parotid surgery causes parasympathetic fibers to regrow abnormally to sweat glands, leading to sweating and flushing during eating.
5. A lesion in the otic ganglion would lead to decreased secretion from which gland?
a) Submandibular gland
b) Sublingual gland
c) Parotid gland
d) Lacrimal gland
Answer: c) Parotid gland
Explanation: The otic ganglion relays parasympathetic fibers from the glossopharyngeal nerve to the parotid gland; damage to it impairs salivary secretion from this gland.
6. Parasympathetic stimulation of the parotid gland causes:
a) Vasoconstriction and thick secretion
b) Vasodilation and watery secretion
c) Decreased blood flow
d) None of the above
Answer: b) Vasodilation and watery secretion
Explanation: Parasympathetic stimulation increases blood flow to the parotid gland and promotes secretion of watery, enzyme-rich saliva for digestion.
7. Which nerve is preserved during parotid gland surgery to avoid facial paralysis?
a) Glossopharyngeal nerve
b) Facial nerve
c) Auriculotemporal nerve
d) Lingual nerve
Answer: b) Facial nerve
Explanation: The facial nerve divides the parotid gland into superficial and deep lobes; careful preservation of this nerve during surgery prevents facial muscle paralysis.
8. Postganglionic sympathetic fibers to the parotid gland arise from:
a) Superior cervical ganglion
b) Otic ganglion
c) Submandibular ganglion
d) Pterygopalatine ganglion
Answer: a) Superior cervical ganglion
Explanation: Sympathetic fibers arise from the superior cervical ganglion, travel via the external carotid plexus, and reach the parotid gland to cause vasoconstriction and reduce salivary flow.
9. The auriculotemporal nerve is a branch of which cranial nerve division?
a) Maxillary nerve
b) Mandibular nerve
c) Ophthalmic nerve
d) Facial nerve
Answer: b) Mandibular nerve
Explanation: The auriculotemporal nerve originates from the posterior division of the mandibular nerve (V3) and carries postganglionic parasympathetic fibers to the parotid gland along with sensory fibers to the temporal region.
10. Inflammation of the parotid gland (mumps) causes pain due to stretching of its capsule, which is innervated by:
a) Auriculotemporal nerve
b) Facial nerve
c) Glossopharyngeal nerve
d) Lingual nerve
Answer: a) Auriculotemporal nerve
Explanation: The parotid fascia and capsule are supplied by the auriculotemporal nerve. In mumps, the gland swells within its tight capsule, causing pain referred to the ear region.
11. A patient with injury to the glossopharyngeal nerve will have decreased secretion from which gland?
a) Submandibular gland
b) Sublingual gland
c) Parotid gland
d) Lacrimal gland
Answer: c) Parotid gland
Explanation: The glossopharyngeal nerve provides preganglionic parasympathetic fibers to the parotid gland. Damage leads to loss of salivary secretion from the parotid, though other glands remain unaffected.
Chapter: Head and Neck Anatomy; Topic: Lymphatic Drainage of Head and Neck; Subtopic: Lymphatic Drainage of Lips
Key Definitions:
• Lymph nodes: Small, bean-shaped structures that filter lymph and help the body fight infection.
• Submandibular nodes: Nodes located beneath the jaw, receiving lymph from upper lip and lateral lower lip.
• Submental nodes: Nodes situated below the chin, draining the central lower lip and chin.
• Preauricular (parotid) nodes: Nodes located in front of the ear, receiving lymph from the eyelids and lateral face, not the lips.
Lead Question (NEET PG 2015):
1. Lips does not drain into which group of lymph nodes?
a) Submandibular nodes
b) Sublingual nodes
c) Preauricular parotid
d) None of the above
Answer: c) Preauricular parotid
Explanation: The lymphatic drainage of the lips is divided as follows: the upper lip and lateral parts of the lower lip drain into the submandibular nodes, while the central part of the lower lip and chin drain into the submental nodes. The preauricular (parotid) nodes receive lymph from the lateral eyelids, forehead, and scalp, not from the lips. Therefore, the lips do not drain into the preauricular group of lymph nodes. This knowledge is clinically important in oral cancer metastasis and infections of the oral region.
Guessed Questions (Related to Lymphatic Drainage of Head and Neck):
2. The central part of the lower lip drains into which lymph nodes?
a) Submandibular nodes
b) Submental nodes
c) Deep cervical nodes
d) Preauricular nodes
Answer: b) Submental nodes
Explanation: The submental lymph nodes receive lymph from the central lower lip, chin, and anterior floor of the mouth. These nodes lie in the submental triangle between the anterior bellies of digastric muscles.
3. The upper lip drains mainly into which lymph nodes?
a) Parotid nodes
b) Submandibular nodes
c) Submental nodes
d) Retropharyngeal nodes
Answer: b) Submandibular nodes
Explanation: The submandibular nodes receive lymph from the upper lip, lateral lower lip, and upper oral cavity. These nodes lie superficial to the submandibular gland and drain into deep cervical nodes.
4. A carcinoma at the midline of the lower lip will first spread to which nodes?
a) Submandibular
b) Submental
c) Parotid
d) Deep cervical
Answer: b) Submental
Explanation: The central lower lip and chin region drain to submental nodes; hence, metastasis from a carcinoma in this region will first involve submental nodes before spreading further.
5. Which of the following lymph node groups receives lymph from the scalp and forehead?
a) Submental nodes
b) Parotid (preauricular) nodes
c) Submandibular nodes
d) Deep cervical nodes
Answer: b) Parotid (preauricular) nodes
Explanation: The preauricular nodes drain the lateral scalp, forehead, and eyelids. They do not receive lymph from the lips or oral cavity, distinguishing them from submandibular nodes.
6. The deep cervical lymph nodes receive direct drainage from all of the following except:
a) Palatine tonsil
b) Tongue
c) Lips
d) Scalp
Answer: c) Lips
Explanation: The lips first drain into submental and submandibular nodes before reaching the deep cervical group. Other structures like the tonsil and tongue can have direct drainage to the deep cervical chain.
7. A 40-year-old patient presents with swelling under the chin after a dental infection of the central incisor. Which lymph node is enlarged?
a) Submandibular
b) Submental
c) Jugulodigastric
d) Parotid
Answer: b) Submental
Explanation: Infections from the central lower incisor region drain into the submental nodes due to their anatomic connection with the central lower lip and chin area.
8. Which lymph node group is referred to as the "tonsillar node"?
a) Submental node
b) Jugulodigastric node
c) Jugulo-omohyoid node
d) Parotid node
Answer: b) Jugulodigastric node
Explanation: The jugulodigastric node, a deep cervical node located near the posterior belly of digastric, is known as the tonsillar node as it drains the palatine tonsil and pharyngeal regions.
9. Lymph from the tip of the tongue drains first into:
a) Submandibular nodes
b) Submental nodes
c) Deep cervical nodes
d) Parotid nodes
Answer: b) Submental nodes
Explanation: The tip of the tongue and anterior floor of the mouth drain into submental nodes, while lateral borders of the anterior tongue drain into submandibular nodes.
10. In carcinoma of the lateral border of the tongue, the first group of lymph nodes involved are:
a) Submental nodes
b) Submandibular nodes
c) Jugulodigastric nodes
d) Parotid nodes
Answer: b) Submandibular nodes
Explanation: The anterior two-thirds of the tongue, especially its lateral borders, primarily drain into the submandibular lymph nodes before reaching the deep cervical nodes.
11. Infections of the upper lip and cheek are most likely to cause swelling in which lymph nodes?
a) Submandibular
b) Submental
c) Parotid
d) Buccal
Answer: a) Submandibular
Explanation: The submandibular lymph nodes drain the upper lip, lateral lower lip, and cheek area. Infection or inflammation in these regions commonly leads to enlargement of these nodes.
Chapter: Neuroanatomy; Topic: Cranial Nerves; Subtopic: Parasympathetic Supply of the Eye
Key Definitions:
• Intrinsic ocular muscles: Muscles within the eyeball that control pupil size and lens shape — sphincter pupillae and ciliary muscle.
• Parasympathetic fibers: Autonomic fibers responsible for constriction of the pupil and accommodation of the lens.
• Edinger–Westphal nucleus: Midbrain nucleus providing parasympathetic fibers via the oculomotor nerve to the eye.
• Ciliary ganglion: Parasympathetic ganglion located behind the eyeball, relaying fibers to the sphincter pupillae and ciliary muscle.
Lead Question (NEET PG 2015):
1. Which intrinsic ocular muscle is supplied by parasympathetic innervation?
a) Superior rectus
b) Superior oblique
c) Constrictor pupillae
d) Dilator pupillae
Answer: c) Constrictor pupillae
Explanation: The constrictor (sphincter) pupillae is supplied by parasympathetic fibers originating from the Edinger–Westphal nucleus. These fibers travel via the oculomotor nerve to the ciliary ganglion, and from there through short ciliary nerves to reach the sphincter pupillae. Activation of these fibers causes pupillary constriction (miosis). In contrast, the dilator pupillae is supplied by sympathetic fibers from the superior cervical ganglion. Thus, constrictor pupillae represents the parasympathetic innervation of the eye.
Guessed Questions (Related to Ocular Nerves and Muscles):
2. Parasympathetic fibers to the ciliary muscle are carried by which nerve?
a) Trochlear nerve
b) Abducent nerve
c) Oculomotor nerve
d) Optic nerve
Answer: c) Oculomotor nerve
Explanation: The oculomotor nerve (III) carries preganglionic parasympathetic fibers from the Edinger–Westphal nucleus to the ciliary ganglion, which then supply the ciliary muscle for lens accommodation.
3. The dilator pupillae muscle is supplied by which type of fibers?
a) Parasympathetic
b) Sympathetic
c) Somatic motor
d) Sensory
Answer: b) Sympathetic
Explanation: The dilator pupillae receives sympathetic postganglionic fibers from the superior cervical ganglion via the long ciliary nerves, leading to pupillary dilation (mydriasis).
4. Which ganglion is associated with the oculomotor nerve?
a) Ciliary ganglion
b) Otic ganglion
c) Pterygopalatine ganglion
d) Submandibular ganglion
Answer: a) Ciliary ganglion
Explanation: The ciliary ganglion is the parasympathetic ganglion linked to the oculomotor nerve. It relays fibers that control the sphincter pupillae and ciliary muscle functions.
5. Which muscle of the eye causes accommodation for near vision?
a) Superior rectus
b) Inferior oblique
c) Ciliary muscle
d) Superior oblique
Answer: c) Ciliary muscle
Explanation: The ciliary muscle, controlled by parasympathetic fibers from the oculomotor nerve, contracts to reduce tension on the suspensory ligaments, thickening the lens for near focus.
6. A lesion of the Edinger–Westphal nucleus results in:
a) Miosis
b) Mydriasis
c) Ptosis
d) Diplopia
Answer: b) Mydriasis
Explanation: Destruction of the Edinger–Westphal nucleus interrupts parasympathetic supply to the sphincter pupillae, leading to unopposed sympathetic activity and dilated pupil (mydriasis).
7. Which of the following muscles is not an intrinsic muscle of the eye?
a) Ciliary muscle
b) Sphincter pupillae
c) Superior rectus
d) Dilator pupillae
Answer: c) Superior rectus
Explanation: The superior rectus is an extrinsic ocular muscle that moves the eyeball upward. Intrinsic muscles include the ciliary, sphincter pupillae, and dilator pupillae, which control lens and pupil function.
8. The light reflex (pupillary constriction in response to light) is mediated through which nucleus?
a) Red nucleus
b) Edinger–Westphal nucleus
c) Solitary nucleus
d) Trochlear nucleus
Answer: b) Edinger–Westphal nucleus
Explanation: The afferent limb of the light reflex is via the optic nerve, and the efferent limb is mediated by parasympathetic fibers from the Edinger–Westphal nucleus via the oculomotor nerve to the sphincter pupillae.
9. Damage to the short ciliary nerves results in loss of:
a) Accommodation and light reflex
b) Corneal reflex
c) Lacrimation
d) Eye movement
Answer: a) Accommodation and light reflex
Explanation: Short ciliary nerves carry postganglionic parasympathetic fibers to the ciliary muscle and sphincter pupillae. Their damage leads to paralysis of accommodation and loss of pupillary light reflex.
10. Which of the following best describes the effect of sympathetic stimulation on the eye?
a) Pupillary constriction
b) Lens accommodation
c) Pupillary dilation
d) Convergence of eyeballs
Answer: c) Pupillary dilation
Explanation: Sympathetic stimulation activates the dilator pupillae via fibers from the superior cervical ganglion, resulting in dilation of the pupil (mydriasis) as part of the fight-or-flight response.
11. A patient has a dilated, non-reactive pupil after head trauma. Which structure is likely damaged?
a) Ciliary ganglion
b) Superior cervical ganglion
c) Abducent nucleus
d) Trochlear nerve
Answer: a) Ciliary ganglion
Explanation: Injury to the ciliary ganglion disrupts parasympathetic postganglionic fibers to the sphincter pupillae, causing a fixed, dilated pupil unresponsive to light, a classic sign of parasympathetic failure.
Chapter: Neuroanatomy; Topic: Autonomic Nervous System; Subtopic: Sympathetic System – Central Control
Key Definitions:
• Sympathetic system: The division of the autonomic nervous system responsible for “fight or flight” responses such as increased heart rate, pupil dilation, and vasoconstriction.
• Hypothalamus: The central regulatory area of the brain that integrates autonomic and endocrine functions.
• Edinger–Westphal nucleus: Parasympathetic nucleus associated with the oculomotor nerve controlling pupil constriction and lens accommodation.
Lead Question (NEET PG 2015):
1. Major central nucleus of sympathetic system is:
A) Nucleus ambiguus
B) Nucleus tractus solitarius
C) Edinger–Westphal nucleus
D) Hypothalamus
Answer: D) Hypothalamus
Explanation: The hypothalamus acts as the major central control nucleus for the sympathetic division of the autonomic nervous system. It integrates inputs from higher cortical centers and regulates autonomic outflow via descending hypothalamospinal tracts to the brainstem and spinal cord. The posterior and lateral regions of the hypothalamus primarily influence sympathetic activity, while the anterior region governs parasympathetic control.
Related (Guessed) Questions with Explanations:
2. The Edinger–Westphal nucleus is related to which function?
A) Accommodation reflex
B) Lacrimation
C) Salivation
D) Swallowing
Answer: A) Accommodation reflex
Explanation: The Edinger–Westphal nucleus provides parasympathetic fibers through the oculomotor nerve to the ciliary muscle and sphincter pupillae, mediating accommodation and pupillary constriction.
3. Which of the following nuclei is parasympathetic in nature?
A) Nucleus ambiguus
B) Edinger–Westphal nucleus
C) Nucleus tractus solitarius
D) Hypothalamic nucleus
Answer: B) Edinger–Westphal nucleus
Explanation: The Edinger–Westphal nucleus belongs to the cranial parasympathetic nuclei, controlling ocular parasympathetic functions.
4. The sympathetic preganglionic neurons are located in which region of the spinal cord?
A) Cervical
B) Thoracolumbar
C) Lumbosacral
D) Sacral
Answer: B) Thoracolumbar
Explanation: Sympathetic preganglionic neurons arise from the intermediolateral cell column of spinal segments T1 to L2, giving the system its thoracolumbar origin.
5. Which structure integrates autonomic and endocrine responses?
A) Cerebellum
B) Thalamus
C) Hypothalamus
D) Pons
Answer: C) Hypothalamus
Explanation: The hypothalamus connects the nervous and endocrine systems via the pituitary gland, coordinating homeostatic autonomic and hormonal responses.
6. The nucleus tractus solitarius primarily receives which type of fibers?
A) Somatic motor
B) Visceral sensory
C) Somatic sensory
D) Visceral motor
Answer: B) Visceral sensory
Explanation: The nucleus tractus solitarius receives afferent visceral sensory input, including baroreceptor and chemoreceptor signals from cranial nerves VII, IX, and X.
7. The lateral horn of the spinal cord (T1–L2) contains:
A) Parasympathetic preganglionic neurons
B) Sympathetic preganglionic neurons
C) Somatic motor neurons
D) Sensory interneurons
Answer: B) Sympathetic preganglionic neurons
Explanation: The intermediolateral cell column located in the lateral horn of T1–L2 spinal cord segments houses sympathetic preganglionic neurons.
8. The hypothalamus exerts its autonomic influence mainly via:
A) Corticospinal tract
B) Reticulospinal tract
C) Hypothalamospinal tract
D) Rubrospinal tract
Answer: C) Hypothalamospinal tract
Explanation: The hypothalamus sends descending fibers through the hypothalamospinal tract to regulate autonomic centers in the brainstem and spinal cord.
9. The “fight or flight” response is mediated primarily by which neurotransmitter?
A) Acetylcholine
B) Dopamine
C) Noradrenaline
D) Serotonin
Answer: C) Noradrenaline
Explanation: Noradrenaline (norepinephrine) is the principal neurotransmitter released by postganglionic sympathetic neurons to produce the fight-or-flight response.
10. The parasympathetic control of the pupil arises from:
A) Edinger–Westphal nucleus
B) Nucleus tractus solitarius
C) Nucleus ambiguus
D) Hypoglossal nucleus
Answer: A) Edinger–Westphal nucleus
Explanation: The Edinger–Westphal nucleus sends preganglionic fibers through the oculomotor nerve to constrict the pupil via the ciliary ganglion.
11. The posterior hypothalamus primarily regulates:
A) Parasympathetic activity
B) Sympathetic activity
C) Endocrine secretion only
D) Circadian rhythm
Answer: B) Sympathetic activity
Explanation: The posterior and lateral parts of the hypothalamus are concerned with sympathetic outflow, while the anterior part controls parasympathetic tone.
Chapter: Neuroanatomy; Topic: Cranial Nerves; Subtopic: Oculomotor Nerve (III Cranial Nerve)
Key Definitions:
• Oculomotor nerve: The third cranial nerve responsible for most extraocular movements, eyelid elevation, and pupillary constriction.
• Extraocular muscles: Muscles that control eye movement; include superior, inferior, medial recti, lateral rectus, superior oblique, and inferior oblique.
• Abducent nerve: The sixth cranial nerve that supplies the lateral rectus muscle, responsible for abduction of the eye.
Lead Question (NEET PG 2015):
1. Oculomotor nerve supplies all extraocular muscles except:
A) Superior rectus
B) Inferior rectus
C) Lateral rectus
D) Medial rectus
Answer: C) Lateral rectus
Explanation: The oculomotor nerve (cranial nerve III) supplies the superior rectus, inferior rectus, medial rectus, and inferior oblique muscles, along with levator palpebrae superioris. The lateral rectus is supplied by the abducent nerve (cranial nerve VI), and the superior oblique is supplied by the trochlear nerve (cranial nerve IV). Thus, among the given options, the muscle not innervated by the oculomotor nerve is the lateral rectus.
Related (Guessed) Questions with Explanations:
2. Which nerve supplies the superior oblique muscle?
A) Oculomotor nerve
B) Trochlear nerve
C) Abducent nerve
D) Optic nerve
Answer: B) Trochlear nerve
Explanation: The superior oblique muscle is supplied by the trochlear nerve (cranial nerve IV), the only cranial nerve emerging dorsally from the brainstem.
3. A patient with right oculomotor nerve palsy will show which of the following signs?
A) Medial deviation of eyeball
B) Lateral deviation of eyeball
C) Downward and inward gaze
D) Upward deviation of eyeball
Answer: B) Lateral deviation of eyeball
Explanation: Oculomotor nerve palsy causes paralysis of most extraocular muscles, leaving the lateral rectus (abducent) and superior oblique (trochlear) unopposed, producing a “down and out” eye position.
4. Which of the following muscles is responsible for abduction of the eyeball?
A) Medial rectus
B) Lateral rectus
C) Superior rectus
D) Inferior oblique
Answer: B) Lateral rectus
Explanation: The lateral rectus, innervated by the abducent nerve, abducts the eyeball away from the midline.
5. The levator palpebrae superioris is innervated by:
A) Trochlear nerve
B) Oculomotor nerve
C) Abducent nerve
D) Facial nerve
Answer: B) Oculomotor nerve
Explanation: The superior division of the oculomotor nerve supplies the levator palpebrae superioris, responsible for eyelid elevation.
6. Which cranial nerve emerges from the dorsal aspect of the brainstem?
A) Oculomotor nerve
B) Trochlear nerve
C) Abducent nerve
D) Trigeminal nerve
Answer: B) Trochlear nerve
Explanation: The trochlear nerve is the only cranial nerve to emerge dorsally from the midbrain and decussates before innervating the contralateral superior oblique.
7. Damage to the oculomotor nerve results in which of the following?
A) Ptosis and mydriasis
B) Miosis and proptosis
C) Ptosis and miosis
D) Proptosis and diplopia only
Answer: A) Ptosis and mydriasis
Explanation: Oculomotor nerve palsy causes ptosis due to levator palpebrae paralysis and mydriasis from sphincter pupillae involvement.
8. Which of the following nerves carries parasympathetic fibers to the sphincter pupillae?
A) Abducent nerve
B) Trochlear nerve
C) Oculomotor nerve
D) Optic nerve
Answer: C) Oculomotor nerve
Explanation: The parasympathetic fibers from the Edinger–Westphal nucleus travel via the oculomotor nerve to the ciliary ganglion, innervating the sphincter pupillae.
9. The nucleus of the oculomotor nerve is located in which part of the brainstem?
A) Pons
B) Midbrain
C) Medulla oblongata
D) Thalamus
Answer: B) Midbrain
Explanation: The oculomotor nucleus is located in the midbrain at the level of the superior colliculus, near the midline.
10. A lesion in the right abducent nerve will result in inability to:
A) Elevate the eye
B) Depress the eye
C) Adduct the eye
D) Abduct the eye
Answer: D) Abduct the eye
Explanation: The abducent nerve supplies the lateral rectus muscle, which abducts the eye; its palsy causes inability to move the eye laterally.
11. Which of the following statements about the oculomotor nerve is true?
A) It carries only motor fibers
B) It carries motor and sensory fibers
C) It carries motor and parasympathetic fibers
D) It carries sensory and sympathetic fibers
Answer: C) It carries motor and parasympathetic fibers
Explanation: The oculomotor nerve contains somatic motor fibers to extraocular muscles and parasympathetic fibers to the sphincter pupillae and ciliary muscle.
Chapter: Abdomen; Topic: Posterior Relations of Kidneys; Subtopic: Surface Anatomy and Relations
Keyword Definitions:
Kidney: A retroperitoneal organ responsible for urine formation and excretion of metabolic wastes.
Posterior relation: Structures located behind an organ, important in surgical and anatomical understanding.
Subcostal nerve: The nerve running below the 12th rib, providing motor and sensory supply to the abdominal wall.
Ilioinguinal nerve: A branch of the lumbar plexus supplying the skin over the groin and scrotum/labia.
Diaphragm: The muscular sheet separating the thoracic and abdominal cavities, aiding in respiration.
Lead Question (2015)
Posterior relation of right kidney are all except -
a) Diaphragm
b) Subcostal nerve
c) 11th rib
d) Ilioinguinal nerve
Explanation: The posterior relations of the right kidney include the diaphragm, 12th rib, and muscles like psoas major, quadratus lumborum, and transversus abdominis, along with nerves like subcostal, iliohypogastric, and ilioinguinal. The 11th rib does not relate posteriorly to the right kidney. Hence, the answer is (c) 11th rib.
1. Posterior relations of left kidney include all except -
a) 11th rib
b) 12th rib
c) Psoas major
d) Descending colon
Explanation: The left kidney is related posteriorly to both 11th and 12th ribs, psoas major, quadratus lumborum, and transversus abdominis. The descending colon lies anterior to the kidney, not posteriorly. Hence, the answer is (d) Descending colon.
2. The right kidney lies opposite which vertebral levels?
a) T10–L1
b) T11–L2
c) T12–L3
d) L1–L4
Explanation: The right kidney typically lies lower due to the presence of the liver, extending from T12 to L3 vertebral levels, while the left kidney lies slightly higher (T11–L2). Hence, the answer is (c) T12–L3.
3. Which muscle forms the medial relation of the kidney posteriorly?
a) Transversus abdominis
b) Quadratus lumborum
c) Psoas major
d) Latissimus dorsi
Explanation: The psoas major muscle forms the medial posterior relation of the kidney and plays a role in flexing the thigh at the hip joint. Hence, the answer is (c) Psoas major.
4. The nerve located posterior to both kidneys is -
a) Obturator nerve
b) Iliohypogastric nerve
c) Phrenic nerve
d) Femoral nerve
Explanation: The iliohypogastric nerve runs posterior to both kidneys along with subcostal and ilioinguinal nerves. It arises from L1 and supplies the abdominal wall. Hence, the answer is (b) Iliohypogastric nerve.
5. The structure anterior to the right kidney is -
a) Liver
b) Spleen
c) Stomach
d) Pancreas
Explanation: The anterior relation of the right kidney includes the liver, duodenum, and right colic flexure, while the spleen and stomach relate to the left kidney. Hence, the answer is (a) Liver.
6. A stab injury at the right costovertebral angle injures which structure first?
a) Diaphragm
b) 12th rib
c) Kidney
d) Liver capsule
Explanation: The costovertebral angle corresponds to the posterior relation of the kidney. A stab wound here usually affects the lower part of the kidney first, making it vulnerable to injury. Hence, the answer is (c) Kidney.
7. The upper pole of the right kidney is related to -
a) Liver
b) Spleen
c) Diaphragm
d) Stomach
Explanation: The upper pole of the right kidney lies beneath the diaphragm and is in contact with the right suprarenal gland and the liver. Hence, the answer is (a) Liver.
8. In renal surgeries, posterior approach is preferred because -
a) Avoids peritoneal cavity
b) Provides better exposure of renal hilum
c) Easy access to renal vein
d) Prevents injury to adrenal gland
Explanation: Posterior approach avoids opening the peritoneal cavity, thus minimizing risk of contamination and complications. It also allows direct access to the kidney's posterior surface. Hence, the answer is (a) Avoids peritoneal cavity.
9. During nephrectomy, which nerve may be damaged posteriorly?
a) Obturator
b) Iliohypogastric
c) Phrenic
d) Sciatic
Explanation: The iliohypogastric nerve lies close to the posterior surface of the kidney and can be damaged during posterior incisions or retraction in nephrectomy. Hence, the answer is (b) Iliohypogastric.
10. In a CT scan, the posterior relation of left kidney at the level of 12th rib includes -
a) Diaphragm
b) Liver
c) Duodenum
d) Stomach
Explanation: At the level of the 12th rib, the posterior surface of the left kidney is covered by the diaphragm, quadratus lumborum, and psoas major. Hence, the answer is (a) Diaphragm.
Chapter: Lower Limb; Topic: Venous System of Lower Limb; Subtopic: Great Saphenous Vein
Keyword Definitions:
Great Saphenous Vein: The longest vein in the body, originating from the medial end of the dorsal venous arch of the foot and draining into the femoral vein.
Femoral Vein: A deep vein in the thigh that receives blood from the great saphenous vein just below the inguinal ligament.
Inguinal Ligament: A fibrous band extending from the anterior superior iliac spine to the pubic tubercle, marking the lower border of the abdomen.
Venous Valves: Structures that prevent backflow of blood, ensuring one-way flow toward the heart, especially in lower limb veins.
Lead Question - 2015
True about the anatomy of great saphenous vein:
a) Starts as a continuation of medial marginal vein
b) Ends of femoral vein 2.5 cm below the inguinal ligament
c) There are 2 - 5 valves below the knee
d) Ascends 2.5 - 3 cm behind tibial malleolus
Explanation: The great saphenous vein begins as a continuation of the medial marginal vein of the foot. It passes anterior to the medial malleolus, ascends along the medial side of the leg and thigh, and drains into the femoral vein approximately 3.5 cm below the inguinal ligament. It has 10–20 valves preventing venous reflux.
1. The great saphenous vein terminates in which structure?
a) Popliteal vein
b) Femoral vein
c) External iliac vein
d) Deep femoral vein
Explanation: The femoral vein receives the great saphenous vein at the saphenofemoral junction, located approximately 3.5 cm below and lateral to the pubic tubercle. This site is clinically important in varicose vein surgery, where the vein is often ligated to prevent venous reflux and recurrence.
2. The great saphenous vein passes in front of which bony landmark?
a) Lateral malleolus
b) Medial malleolus
c) Tibial tuberosity
d) Fibular head
Explanation: The great saphenous vein passes anterior to the medial malleolus before ascending along the medial aspect of the leg. This relationship helps in identifying the vein for venipuncture or bypass graft harvesting, as it remains superficial and consistent in location.
3. Which of the following veins is commonly used in coronary artery bypass grafting (CABG)?
a) Small saphenous vein
b) Great saphenous vein
c) Femoral vein
d) Popliteal vein
Explanation: The great saphenous vein is most commonly used in CABG because of its suitable length, diameter, and accessibility. It can be easily harvested without causing major circulatory compromise in the limb due to collateral venous drainage via perforators and deep veins.
4. The saphenous opening is located in which fascia?
a) Cribriform fascia
b) Deep fascia of thigh
c) Superficial fascia
d) Fascia lata
Explanation: The cribriform fascia, a part of the fascia lata, covers the saphenous opening through which the great saphenous vein pierces to join the femoral vein. This opening allows communication between superficial and deep venous systems and is a key landmark during varicose vein surgery.
5. Which of the following clinical conditions is associated with incompetence of great saphenous vein valves?
a) Varicose veins
b) Deep vein thrombosis
c) Phlebitis
d) Lymphedema
Explanation: Varicose veins occur due to valve incompetence in the great saphenous vein or perforator veins. This leads to venous hypertension, dilation, and tortuosity. Common symptoms include heaviness, pain, and swelling in the legs, and treatment includes compression, sclerotherapy, or surgical stripping of the vein.
6. Which perforator connects the great saphenous vein with deep veins in the lower leg?
a) Cockett’s perforator
b) Boyd’s perforator
c) Dodd’s perforator
d) Sherman’s perforator
Explanation: The Cockett’s perforators connect the great saphenous vein with the posterior tibial veins in the lower leg. These perforators play a key role in maintaining unidirectional venous flow; their incompetence results in venous stasis ulcers and varicosities of the lower limb.
7. Which structure accompanies the great saphenous vein throughout its course?
a) Saphenous nerve
b) Femoral nerve
c) Obturator nerve
d) Tibial nerve
Explanation: The saphenous nerve, a branch of the femoral nerve, accompanies the great saphenous vein in the leg. It provides sensory innervation to the medial aspect of the leg and foot, and care must be taken during vein harvesting to avoid nerve injury that can cause numbness.
8. During coronary artery bypass surgery, the great saphenous vein is reversed before grafting because:
a) It improves arterial flow
b) To prevent valve obstruction
c) To match lumen diameter
d) To reduce thrombosis
Explanation: The great saphenous vein is reversed before grafting to ensure that the valves do not obstruct the flow of blood when used as an arterial conduit. In its normal orientation, the valves allow only upward venous flow, hence reversal prevents blockage during coronary artery bypass grafting.
9. Which of the following statements about saphenous vein harvesting is true?
a) It can cause lymphedema
b) It leads to deep vein obstruction
c) It rarely affects limb circulation
d) It is contraindicated in elderly patients
Explanation: Removal of the great saphenous vein rarely affects limb circulation because deep veins carry the majority of venous return. The presence of numerous perforating veins ensures collateral drainage, making it a safe choice for bypass grafting or vascular surgeries without significant circulatory compromise.
10. The great saphenous vein is located in which compartment of the leg?
a) Deep posterior compartment
b) Superficial fascia
c) Anterior compartment
d) Deep fascia
Explanation: The great saphenous vein runs in the superficial fascia of the leg and thigh. It lies between the two layers of superficial fascia, making it easily visible and accessible for venous cannulation, bypass surgery, and varicose vein management.
Chapter: Abdomen; Topic: Inguinal Region; Subtopic: Cremasteric Muscle and its Nerve Supply
Keyword Definitions:
Cremasteric muscle: A thin layer of skeletal muscle covering the spermatic cord and testis, derived from the internal oblique muscle.
Genitofemoral nerve: A mixed nerve from the lumbar plexus (L1-L2) that divides into genital and femoral branches.
Pudendal nerve: Main somatic nerve of the perineum originating from S2-S4 roots.
Ilioinguinal nerve: Arises from L1 and supplies skin over the superomedial thigh and genital area.
Lead Question - 2015
Nerve supply of cremasteric muscle?
a) Pudendal nerve
b) Femoral branch of genitofemoral
c) Genital branch of genitofemoral nerve
d) Ilioinguinal nerve
Explanation: The genital branch of the genitofemoral nerve supplies the cremasteric muscle. This branch passes through the inguinal canal and innervates the muscle derived from the internal oblique. The cremasteric reflex involves this nerve for the motor limb and the ilioinguinal nerve for the sensory limb, crucial in testicular elevation and diagnosis of spinal lesions.
1. Cremasteric reflex is mediated by which spinal segment?
a) L1-L2
b) S2-S4
c) T12-L1
d) L3-L4
Explanation: The L1-L2 spinal segments mediate the cremasteric reflex. The sensory input is via the ilioinguinal nerve and the motor output via the genital branch of the genitofemoral nerve. This reflex is an indicator of integrity of the L1-L2 spinal cord segment and is often tested in neurological examination.
2. Absence of cremasteric reflex may indicate lesion in:
a) Femoral nerve
b) L1 spinal segment
c) Pudendal nerve
d) Obturator nerve
Explanation: Absence of the reflex may suggest a lesion in the L1 spinal segment or damage to the genitofemoral or ilioinguinal nerve. It may also be absent in upper and lower motor neuron lesions, testicular torsion, or during anesthesia affecting the related dermatomes.
3. Which nerve is responsible for the sensory limb of the cremasteric reflex?
a) Genitofemoral nerve
b) Pudendal nerve
c) Ilioinguinal nerve
d) Femoral nerve
Explanation: The ilioinguinal nerve provides the sensory limb of the cremasteric reflex by sensing touch from the upper medial thigh, while the genital branch of genitofemoral nerve provides the motor response, lifting the testis. Both nerves originate from the lumbar plexus and coordinate the reflex response.
4. Clinical absence of cremasteric reflex is most commonly tested for:
a) Appendicitis
b) Testicular torsion
c) Inguinal hernia
d) Hydrocele
Explanation: The cremasteric reflex is absent in testicular torsion, a urological emergency. Its absence helps distinguish torsion from epididymitis, where the reflex remains intact. Loss of reflex indicates disruption of the genitofemoral or ilioinguinal nerve pathway, often due to torsion or upper motor neuron lesion.
5. In which type of hernia is the cremasteric reflex usually absent?
a) Direct inguinal hernia
b) Indirect inguinal hernia
c) Femoral hernia
d) Obturator hernia
Explanation: The indirect inguinal hernia may involve compression or stretching of the genital branch of the genitofemoral nerve, leading to an absent cremasteric reflex. Since this nerve passes through the inguinal canal, it can be affected during the herniation process or following surgical repair procedures.
6. Which of the following nerves arises from the lumbar plexus and enters the inguinal canal through the deep ring?
a) Ilioinguinal nerve
b) Genitofemoral nerve
c) Pudendal nerve
d) Obturator nerve
Explanation: The genitofemoral nerve (L1-L2) enters the inguinal canal via the deep inguinal ring. Its genital branch accompanies the spermatic cord to supply the cremasteric muscle and scrotal skin. This pathway is crucial for maintaining reflex function and sensation in the inguinal region.
7. Cremasteric muscle is derived from which abdominal muscle?
a) External oblique
b) Internal oblique
c) Transversus abdominis
d) Rectus abdominis
Explanation: The internal oblique muscle gives rise to the cremasteric muscle fibers, which form a loop around the spermatic cord and testis. These fibers help elevate the testis in response to temperature and touch stimuli, an essential function in thermoregulation and protection of the testis.
8. Which of the following best describes the function of the cremasteric muscle?
a) Elevation of penis
b) Elevation of testis
c) Retraction of scrotum
d) Constriction of urethra
Explanation: The cremasteric muscle is responsible for elevation of the testis toward the body. This action regulates testicular temperature for optimal spermatogenesis and protects from external trauma. It acts reflexively to cold and touch stimuli, mediated via genitofemoral and ilioinguinal nerves.
9. Cremasteric reflex is absent in which spinal cord injury level?
a) Above T6
b) At L1-L2
c) Below S2
d) C5-C6
Explanation: The reflex is lost in L1-L2 spinal cord injury, as these segments mediate both afferent and efferent limbs of the reflex. The ilioinguinal and genitofemoral nerves are derived from these levels, and any lesion interrupts the sensory-motor arc, eliminating the testicular elevation response.
10. Which of the following reflexes shares a similar spinal level as cremasteric reflex?
a) Anal reflex
b) Abdominal reflex
c) Plantar reflex
d) Biceps reflex
Explanation: The abdominal reflex shares a similar spinal level (T7–L2) with the cremasteric reflex (L1–L2). Both are superficial reflexes involving cutaneous stimulation and muscle contraction, commonly assessed in neurological exams to localize lesions in the thoracolumbar spinal cord.
Chapter: Pelvis and Perineum; Topic: Nerve Supply of Pelvic Organs; Subtopic: Innervation of Uterus
Keyword Definitions:
Labour pain: Intense uterine contractions during childbirth transmitted through visceral afferent fibers.
Sympathetic nerves: Fibers arising from thoracolumbar segments that carry pain from the uterine body during labour.
Parasympathetic nerves: Fibers from sacral outflow (S2–S4) mainly supplying cervix and vagina.
Pudendal nerve: Somatic nerve supplying perineum and external genitalia, involved in somatic pain.
Splanchnic nerves: Visceral nerves carrying sympathetic and sensory fibers from abdominal and pelvic organs.
Lead Question (2015):
Labour pain in uterus is carried by
a) Parasympathetic nerves
b) Sympathetic nerves
c) Pudendal nerve
d) Splanchnic nerve
Explanation: The correct answer is b) Sympathetic nerves. Labour pain originates from uterine contractions and cervical dilation. Pain from the uterine body is transmitted via sympathetic fibers through the hypogastric plexus and enters the spinal cord at T10–L1 levels. Pain from the cervix and perineum, however, is carried by parasympathetic and pudendal nerves respectively.
1) Pain from uterine contractions during first stage of labour is transmitted via:
a) T10–L1 sympathetic fibers
b) S2–S4 parasympathetic fibers
c) Pudendal nerve
d) Femoral nerve
Explanation: The answer is a) T10–L1 sympathetic fibers. During the first stage of labour, pain arises from stretching of the uterine wall and is transmitted through sympathetic afferents accompanying the hypogastric nerves to T10–L1 spinal segments. This explains why epidural blocks at these levels effectively relieve early labour pain.
2) Pain during the second stage of labour is mainly transmitted by:
a) Pudendal nerve
b) Sympathetic nerves
c) Pelvic splanchnic nerves
d) Ilioinguinal nerve
Explanation: The correct answer is a) Pudendal nerve. In the second stage of labour, pain is somatic due to stretching of perineal tissues and vaginal walls. This pain is transmitted by the pudendal nerve (S2–S4). Pudendal block provides effective anesthesia for perineal delivery procedures and episiotomy.
3) Which nerve fibers carry pain from the cervix during labour?
a) Parasympathetic fibers from S2–S4
b) Sympathetic fibers from T12–L1
c) Pudendal nerve
d) Iliohypogastric nerve
Explanation: The answer is a) Parasympathetic fibers from S2–S4. Cervical dilation pain is transmitted by parasympathetic afferents through pelvic splanchnic nerves (S2–S4). This explains why a caudal block that anesthetizes these sacral roots helps relieve cervical pain in later stages of labour.
4) In epidural anesthesia for labour, which spinal segments are targeted to block uterine contraction pain?
a) T10–L1
b) L4–S1
c) S2–S4
d) T4–T6
Explanation: The correct answer is a) T10–L1. Blocking these segments relieves pain from uterine contractions during the first stage of labour. For complete perineal analgesia during the second stage, the block is extended to include S2–S4 roots that carry pudendal nerve fibers.
5) A woman in early labour complains of pain in the lower abdomen radiating to the back. The nerve pathway involved is:
a) Sympathetic afferents via hypogastric plexus
b) Pudendal nerve
c) Pelvic splanchnic nerves
d) Somatic fibers from iliohypogastric nerve
Explanation: The answer is a) Sympathetic afferents via hypogastric plexus. Visceral pain from uterine contractions is referred to the lower abdomen and back due to shared T10–L1 dermatomes. These sympathetic afferents travel through the hypogastric and aortic plexuses to the spinal cord.
6) Which of the following statements about pudendal nerve is TRUE?
a) It carries somatic pain from perineum
b) It supplies the uterine body
c) It originates from T10–L1
d) It is purely parasympathetic
Explanation: The correct answer is a) It carries somatic pain from perineum. The pudendal nerve is derived from sacral spinal nerves (S2–S4) and supplies motor and sensory innervation to the perineum, external anal sphincter, and genitalia. It is crucial in transmitting somatic pain during the second stage of labour.
7) Pain from the uterine fundus is referred to which dermatome level?
a) T10–L1
b) L4–L5
c) S1–S3
d) T4–T6
Explanation: The correct answer is a) T10–L1. The uterine fundus is supplied by sympathetic afferents that enter the spinal cord at T10–L1. Therefore, pain from uterine contractions is referred to the lower abdomen and back corresponding to these dermatomes, a typical presentation during early labour.
8) Which type of nerve fibers transmit visceral pain from the uterus?
a) Unmyelinated C fibers
b) Myelinated A-beta fibers
c) Myelinated A-delta fibers
d) Somatic motor fibers
Explanation: The correct answer is a) Unmyelinated C fibers. Labour pain is transmitted via unmyelinated visceral C fibers associated with sympathetic afferents. These fibers conduct slow, dull, and diffuse pain sensations that are poorly localized, typical of visceral pain from uterine contractions.
9) During caudal anesthesia, which nerve fibers are blocked to relieve perineal pain?
a) Pudendal and pelvic splanchnic nerves
b) Femoral and obturator nerves
c) Iliohypogastric nerve
d) Sympathetic chain fibers
Explanation: The answer is a) Pudendal and pelvic splanchnic nerves. Caudal anesthesia blocks sacral nerve roots (S2–S4), providing effective analgesia for perineal pain during childbirth. It targets both pudendal (somatic) and pelvic splanchnic (visceral parasympathetic) nerves, useful in second-stage labour pain relief.
10) A multiparous woman in labour experiences pain not relieved by epidural block at T10–L1. The cause is likely unblocked:
a) Pudendal nerve
b) Iliohypogastric nerve
c) Femoral nerve
d) Genitofemoral nerve
Explanation: The correct answer is a) Pudendal nerve. An epidural block covering T10–L1 levels relieves visceral pain from uterine contractions but not somatic pain from perineal distension. The pudendal nerve (S2–S4) carries perineal pain; thus, a supplementary pudendal block is required for complete analgesia during delivery.
Chapter: Hepatobiliary System; Topic: Gall Bladder; Subtopic: Nerve Supply and Clinical Anatomy
Keyword Definitions:
Gall Bladder: A pear-shaped organ beneath the liver that stores and concentrates bile.
Cystic Duct: Connects the gall bladder to the common bile duct, allowing bile flow.
Vagus Nerve: The tenth cranial nerve that supplies parasympathetic fibers to thoracic and abdominal organs.
Phrenic Nerve: A mixed nerve that carries sensory fibers to the diaphragm and peritoneum.
Lead Question - 2015
Sensory nerve supply of gall bladder is through -
a) Vagus nerve
b) Trigeminal nerve
c) Parasympathetic nerve
d) Facial nerve
Explanation:
Answer: a) Vagus nerve
The gall bladder receives sensory innervation primarily via the right phrenic nerve and vagus nerve. The vagus nerve mediates visceral sensations such as distension or inflammation. Pain from gall bladder disease is often referred to the right shoulder due to shared sensory pathways with the diaphragm via the phrenic nerve. Both autonomic and sensory components interact to coordinate gall bladder function.
1. Which of the following nerves carries referred pain from the inflamed gall bladder to the right shoulder?
a) Vagus nerve
b) Phrenic nerve
c) Hypoglossal nerve
d) Glossopharyngeal nerve
Explanation:
Answer: b) Phrenic nerve
Referred pain from gall bladder inflammation (cholecystitis) is transmitted via the phrenic nerve, whose sensory fibers supply the diaphragm. The brain interprets pain from the diaphragm as originating in the shoulder region, a phenomenon known as referred pain. This link explains right shoulder discomfort in gall bladder disease.
2. Parasympathetic stimulation of gall bladder causes -
a) Relaxation
b) Contraction
c) No effect
d) Pain
Explanation:
Answer: b) Contraction
Parasympathetic fibers from the vagus nerve stimulate gall bladder contraction, particularly during digestion. The release of cholecystokinin from the duodenum enhances this action, leading to bile ejection into the duodenum. Coordinated contraction and relaxation of the sphincter of Oddi ensure efficient bile flow and fat digestion.
3. Clinical: A 40-year-old woman with gallstones presents with right shoulder pain. Which nerve carries this referred pain?
a) Vagus nerve
b) Phrenic nerve
c) Intercostal nerve
d) Thoracodorsal nerve
Explanation:
Answer: b) Phrenic nerve
Gallstones cause irritation of the gall bladder and diaphragm, which share sensory input via the phrenic nerve. This results in referred pain to the right shoulder. The vagus nerve mediates visceral sensations but not the somatic referral pathway. This pattern helps distinguish biliary pain from other abdominal causes.
4. Which artery supplies the gall bladder?
a) Hepatic artery proper
b) Cystic artery
c) Gastroduodenal artery
d) Inferior mesenteric artery
Explanation:
Answer: b) Cystic artery
The cystic artery, usually a branch of the right hepatic artery, supplies the gall bladder. It divides into superficial and deep branches, nourishing both surfaces. Knowledge of its anatomical variation is essential during cholecystectomy to prevent hemorrhage or ischemia. Proper ligation of the cystic artery is a crucial surgical step.
5. Clinical: In cholecystitis, which nerve is responsible for localized pain in the right upper quadrant?
a) Phrenic nerve
b) Intercostal nerves (T7–T9)
c) Vagus nerve
d) Sympathetic chain
Explanation:
Answer: b) Intercostal nerves (T7–T9)
Localized right upper quadrant pain in cholecystitis arises due to somatic afferents carried by intercostal nerves T7–T9 supplying the parietal peritoneum. When the inflamed gall bladder irritates the overlying peritoneum, pain becomes sharp and localized. Deeper visceral pain remains dull and referred to the epigastrium.
6. The motor supply to the gall bladder comes from -
a) Sympathetic nerves
b) Parasympathetic nerves via vagus
c) Phrenic nerve
d) Somatic nerves
Explanation:
Answer: b) Parasympathetic nerves via vagus
Motor innervation to the gall bladder is provided by parasympathetic fibers from the vagus nerve. These fibers cause contraction of the gall bladder and relaxation of the sphincter of Oddi during digestion. This mechanism is essential for bile release in response to fatty meal intake.
7. Clinical: During laparoscopic cholecystectomy, which nerve may cause referred shoulder pain due to CO₂ insufflation?
a) Vagus nerve
b) Phrenic nerve
c) Intercostal nerve
d) Sciatic nerve
Explanation:
Answer: b) Phrenic nerve
CO₂ insufflation irritates the diaphragm, stimulating the phrenic nerve, which refers pain to the shoulder. This transient shoulder pain is common after laparoscopic procedures involving the upper abdomen. Awareness of this mechanism helps reassure patients postoperatively and differentiate it from surgical complications.
8. Sympathetic fibers to gall bladder arise from -
a) T5–T9 via greater splanchnic nerves
b) T10–T12 via lesser splanchnic nerves
c) L1–L2 via hypogastric plexus
d) None
Explanation:
Answer: a) T5–T9 via greater splanchnic nerves
Sympathetic fibers to the gall bladder originate from T5–T9 spinal segments through greater splanchnic nerves. These fibers reach the celiac plexus and regulate vasoconstriction, reducing bile flow during stress. Sympathetic activation inhibits gall bladder contraction and modulates pain perception from visceral organs.
9. Clinical: A patient undergoing vagotomy may experience -
a) Increased gall bladder contraction
b) Decreased gall bladder contraction
c) Increased bile secretion
d) Pain relief
Explanation:
Answer: b) Decreased gall bladder contraction
Vagotomy disrupts parasympathetic supply to the gall bladder, reducing its contractile response to cholecystokinin. Consequently, bile ejection into the duodenum decreases, impairing fat digestion. This effect is rarely significant clinically but demonstrates the vagus nerve’s role in biliary motility control.
10. Pain in the epigastric region from gall bladder disease is mediated by -
a) Vagus and sympathetic nerves
b) Phrenic nerve only
c) Glossopharyngeal nerve
d) Somatic nerves
Explanation:
Answer: a) Vagus and sympathetic nerves
Visceral pain from gall bladder distension or inflammation is transmitted via sympathetic and vagal afferent fibers. These converge in the spinal cord at thoracic levels, resulting in poorly localized epigastric pain. The combined action of these nerves explains the diffuse character of early gall bladder pain before peritoneal irritation develops.
11. Clinical: A patient has pain radiating from right hypochondrium to shoulder during deep inspiration. The affected nerve is -
a) Phrenic nerve
b) Vagus nerve
c) T12 intercostal nerve
d) Lumbar nerve
Explanation:
Answer: a) Phrenic nerve
Inflammation of the gall bladder during inspiration causes stretching of the diaphragm, stimulating the phrenic nerve. This nerve carries sensory fibers to the shoulder via cervical spinal nerves C3–C5. The characteristic radiation of pain to the right shoulder is an important diagnostic feature of acute cholecystitis.
Chapter: Head and Neck; Topic: Tongue Anatomy; Subtopic: Nerve Supply of Tongue
Keyword Definitions:
• Lingual nerve: A branch of the mandibular nerve (V3) providing general sensation to the anterior two-thirds of the tongue.
• Glossopharyngeal nerve: Cranial nerve IX, supplying both general and special (taste) sensation to the posterior one-third of the tongue.
• Vagus nerve: Cranial nerve X, contributing to sensation near the epiglottis and root of the tongue.
• Taste sensation: Perception of taste mediated by special visceral afferent fibers.
Lead Question - 2015
Sensory supply to tongue is by all, EXCEPT?
a) Lingual nerve
b) Vagus nerve
c) Glossopharyngeal nerve
d) None of the above
Explanation: The sensory innervation of the tongue is complex. The lingual nerve supplies general sensation to the anterior two-thirds, the chorda tympani (via facial nerve) carries taste fibers for the same region, the glossopharyngeal nerve supplies both taste and general sensation to the posterior one-third, and the vagus nerve supplies the epiglottic region. Therefore, all the listed nerves supply the tongue. Correct answer: (d) None of the above.
1. Which nerve provides taste sensation to the anterior two-thirds of the tongue?
a) Lingual nerve
b) Chorda tympani
c) Glossopharyngeal nerve
d) Vagus nerve
Explanation: Taste sensation from the anterior two-thirds of the tongue is carried by the chorda tympani, a branch of the facial nerve (VII). These fibers hitchhike along the lingual nerve, reaching the tongue through the mandibular division of the trigeminal nerve. Correct answer: (b) Chorda tympani.
2. A lesion of the lingual nerve before joining chorda tympani causes loss of:
a) Taste and general sensation anterior 2/3
b) Taste only
c) General sensation only
d) Taste posterior 1/3
Explanation: The lingual nerve carries general sensation, while the chorda tympani carries taste fibers. A lesion of the lingual nerve before it joins the chorda tympani leads to loss of general sensation only in the anterior two-thirds of the tongue. Taste remains intact. Correct answer: (c) General sensation only.
3. Taste buds at the posterior one-third of the tongue are supplied by:
a) Glossopharyngeal nerve
b) Lingual nerve
c) Facial nerve
d) Hypoglossal nerve
Explanation: The glossopharyngeal nerve (cranial nerve IX) provides both general and taste sensation to the posterior one-third of the tongue, including vallate papillae. It plays an important role in swallowing and taste reflexes. Correct answer: (a) Glossopharyngeal nerve.
4. Loss of gag reflex occurs due to lesion of:
a) Hypoglossal nerve
b) Vagus nerve
c) Glossopharyngeal nerve
d) Lingual nerve
Explanation: The glossopharyngeal nerve provides the sensory limb, and the vagus nerve provides the motor limb of the gag reflex. Damage to either may cause absence of the reflex. However, sensory loss is primarily due to glossopharyngeal nerve involvement. Correct answer: (c) Glossopharyngeal nerve.
5. Which cranial nerve does NOT carry taste fibers?
a) Facial nerve
b) Glossopharyngeal nerve
c) Vagus nerve
d) Trigeminal nerve
Explanation: The trigeminal nerve (V) carries general sensory fibers but does not transmit taste sensation. Taste fibers are carried by the facial (VII), glossopharyngeal (IX), and vagus (X) nerves. Hence, loss of taste is not associated with trigeminal nerve lesions. Correct answer: (d) Trigeminal nerve.
6. A patient with right hypoglossal nerve palsy will show:
a) Deviation of tongue to left
b) Deviation of tongue to right
c) Deviation upward
d) No deviation
Explanation: The hypoglossal nerve supplies all intrinsic and extrinsic muscles of the tongue except palatoglossus. A lesion causes ipsilateral tongue weakness. On protrusion, the tongue deviates toward the affected side due to unopposed action of contralateral genioglossus. Correct answer: (b) Deviation to right.
7. Injury to chorda tympani leads to loss of taste in:
a) Anterior 2/3 of tongue
b) Posterior 1/3 of tongue
c) Base of tongue
d) Tip only
Explanation: The chorda tympani carries taste sensation from the anterior two-thirds of the tongue via the facial nerve. Damage results in loss of taste and reduced salivation from submandibular and sublingual glands. Correct answer: (a) Anterior 2/3 of tongue.
8. Which nerve supplies the vallate papillae taste buds?
a) Facial nerve
b) Glossopharyngeal nerve
c) Lingual nerve
d) Vagus nerve
Explanation: Although vallate papillae are located in the anterior two-thirds of the tongue, their taste buds are supplied by the glossopharyngeal nerve, not by the facial nerve. This is an important clinical exception. Correct answer: (b) Glossopharyngeal nerve.
9. Sensory supply of epiglottis is through:
a) Lingual nerve
b) Glossopharyngeal nerve
c) Superior laryngeal branch of vagus
d) Hypoglossal nerve
Explanation: The internal laryngeal branch of the vagus nerve supplies both general and taste sensation to the epiglottis and region near the root of the tongue. It plays a protective role during swallowing. Correct answer: (c) Superior laryngeal branch of vagus.
10. A tumor compressing the glossopharyngeal nerve at the jugular foramen will cause:
a) Loss of taste posterior 1/3
b) Absent gag reflex
c) Difficulty swallowing
d) All of the above
Explanation: The glossopharyngeal nerve carries taste fibers, sensory fibers for the gag reflex, and motor fibers to stylopharyngeus. Compression at the jugular foramen leads to loss of taste in posterior one-third, absent gag reflex, and dysphagia. Correct answer: (d) All of the above.
Chapter: Thorax; Topic: Intercostal Nerves; Subtopic: Thoracic Nerve Distribution
Keyword Definitions:
• Intercostal nerves: Ventral rami of thoracic spinal nerves supplying muscles and skin between ribs.
• Thoracic spinal nerves: Twelve pairs emerging from thoracic segments of the spinal cord.
• Brachial plexus: Nerve network supplying upper limb.
• Ventral rami: Branches of spinal nerves that supply anterior and lateral trunk.
• Dorsal rami: Branches supplying deep back muscles and skin of the back.
Lead Question – 2015
Intercostal nerve is a branch of?
a) Brachial plexus
b) Dorsal rami of thoracic spinal nerves
c) Ventral rami of thoracic spinal nerves
d) Ventral rami of cervical spinal nerves
Answer: c) Ventral rami of thoracic spinal nerves
Explanation: Intercostal nerves are the anterior primary rami of thoracic spinal nerves T1–T11, supplying intercostal muscles and overlying skin. Each runs in the costal groove with blood vessels. Unlike other spinal nerves, they do not form plexuses. The subcostal nerve (T12) lies below the 12th rib. Hence, option c is correct.
1) Which of the following nerves supply intercostal muscles?
a) Sympathetic nerves
b) Intercostal nerves
c) Phrenic nerve
d) Vagus nerve
Answer: b) Intercostal nerves
Explanation: The intercostal muscles—external, internal, and innermost—are innervated by intercostal nerves derived from the ventral rami of thoracic spinal nerves. These nerves control muscle contraction during respiration, facilitating rib cage expansion and contraction. Sympathetic nerves regulate visceral functions, not skeletal muscles.
2) The subcostal nerve corresponds to which thoracic spinal nerve?
a) T11
b) T12
c) T10
d) T9
Answer: b) T12
Explanation: The subcostal nerve is the ventral ramus of the T12 spinal nerve. It lies below the 12th rib and supplies abdominal wall muscles and overlying skin. It communicates with the iliohypogastric and ilioinguinal nerves, forming part of the innervation to the anterior abdominal wall and lateral trunk.
3) A patient has numbness over the skin between the 5th and 6th ribs. Which nerve is affected?
a) T4
b) T5
c) T6
d) T7
Answer: b) T5
Explanation: The intercostal nerve corresponding to the rib space supplies the overlying skin. The 5th intercostal nerve, derived from the T5 ventral ramus, supplies the intercostal space between the 5th and 6th ribs. Damage to this nerve causes sensory loss and mild intercostal muscle weakness at that segmental level.
4) The first intercostal nerve differs from others because:
a) It gives a lateral cutaneous branch
b) It supplies the skin of the arm
c) It has no anterior cutaneous branch
d) It joins the brachial plexus
Answer: d) It joins the brachial plexus
Explanation: The first intercostal nerve largely contributes to the brachial plexus and does not give a typical lateral cutaneous branch. It mainly helps innervate upper limb muscles through communication with the lower trunk of the brachial plexus. Only a small intercostal segment remains in the thorax.
5) The intercostobrachial nerve is derived from:
a) 1st thoracic nerve
b) 2nd thoracic nerve
c) 3rd thoracic nerve
d) 4th thoracic nerve
Answer: b) 2nd thoracic nerve
Explanation: The intercostobrachial nerve originates from the lateral cutaneous branch of the 2nd intercostal nerve (T2). It supplies the skin of the axilla and medial side of the upper arm. Damage during axillary dissection can cause sensory loss in this region. It’s an important landmark in breast surgery.
6) A thoracic wall injury damaging the 3rd intercostal nerve would cause weakness in:
a) Scalene muscles
b) Intercostal muscles between 3rd and 4th ribs
c) Diaphragm
d) Serratus anterior
Answer: b) Intercostal muscles between 3rd and 4th ribs
Explanation: The 3rd intercostal nerve supplies the intercostal muscles and overlying skin between the 3rd and 4th ribs. Injury causes weakness in chest wall expansion and localized sensory loss. Diaphragm and serratus anterior are innervated by phrenic and long thoracic nerves respectively, not intercostal nerves.
7) The lateral cutaneous branch of an intercostal nerve supplies:
a) Skin over the back
b) Skin of the chest and lateral thoracic wall
c) Skin of abdomen only
d) Diaphragm
Answer: b) Skin of the chest and lateral thoracic wall
Explanation: Each intercostal nerve gives a lateral cutaneous branch that pierces the intercostal muscles to supply the skin of the lateral thoracic and abdominal wall. It further divides into anterior and posterior branches. The skin over the back is supplied by dorsal rami of spinal nerves.
8) Clinical: During thoracotomy, which structure is most at risk if incision is made along the upper border of a rib?
a) Intercostal vein
b) Intercostal artery
c) Intercostal nerve
d) All of the above
Answer: d) All of the above
Explanation: Intercostal vein, artery, and nerve lie together in the costal groove along the inferior border of each rib, arranged as vein–artery–nerve (VAN). Therefore, incisions are made along the superior border of a rib to prevent injury to this neurovascular bundle during surgical procedures like thoracotomy or chest tube insertion.
9) Clinical: A stab wound in the 9th intercostal space along the midaxillary line may damage nerves supplying:
a) Skin of the upper arm
b) Abdominal wall
c) Diaphragm
d) Shoulder
Answer: b) Abdominal wall
Explanation: Lower intercostal nerves (T7–T11) continue beyond the costal margin to supply the abdominal wall muscles and overlying skin. A stab injury in the 9th intercostal space damages these nerves, resulting in weakness of abdominal wall muscles and loss of sensation in the corresponding skin region.
10) Clinical: A patient with herpes zoster along the T8 dermatome experiences pain along:
a) Shoulder region
b) Umbilical level
c) Above umbilicus
d) Below umbilicus
Answer: c) Above umbilicus
Explanation: The T8 dermatome corresponds to the skin area just above the umbilicus. Herpes zoster virus affects sensory ganglia, producing painful vesicular eruptions along the affected dermatome. Thoracic dermatomes help in clinical localization of spinal nerve involvement. Hence, pain above the umbilicus corresponds to T8 dermatome.
Chapter: Upper Limb Anatomy; Topic: Nerves of Upper Limb; Subtopic: Medial Cutaneous Nerve of Arm
Keyword Definitions:
Radical neck dissection: A surgical procedure for removing lymph nodes and surrounding tissue to treat cancer in the neck.
Medial cutaneous nerve of arm: A branch of the medial cord of the brachial plexus that supplies skin sensation on the medial side of the arm.
Brachial plexus: A network of nerves supplying the upper limb, formed by the anterior rami of C5–T1 spinal nerves.
Thoracodorsal nerve: Nerve supplying the latissimus dorsi muscle, aiding in shoulder adduction and extension.
Long thoracic nerve: Nerve supplying the serratus anterior muscle, important for scapular movement.
Lead Question – 2015
A nerve injured in radical neck dissection leads to loss of sensation in medial side of the arm, nerve injured is?
a) Long thoracic nerve
b) Thoracodorsal nerve
c) Dorsal scapular nerve
d) Medial cutaneous nerve of arm
Explanation: The medial cutaneous nerve of arm (C8–T1) is a branch of the medial cord of the brachial plexus. It supplies sensory innervation to the medial surface of the arm. Injury during radical neck dissection leads to loss of sensation over this region. Other nerves like long thoracic and thoracodorsal are motor nerves, not sensory.
Guessed Questions for NEET PG:
1. Injury to the long thoracic nerve leads to –
a) Winged scapula
b) Loss of arm sensation
c) Shoulder dislocation
d) Loss of elbow flexion
Explanation: The long thoracic nerve supplies the serratus anterior muscle. Its injury results in winging of the scapula due to loss of scapular fixation. Sensory loss does not occur as it is a motor nerve. The correct answer is a) Winged scapula.
2. Which muscle is supplied by the thoracodorsal nerve?
a) Latissimus dorsi
b) Deltoid
c) Trapezius
d) Rhomboid major
Explanation: The thoracodorsal nerve arises from the posterior cord of the brachial plexus and innervates the latissimus dorsi muscle. It helps in shoulder adduction, internal rotation, and extension. Hence, the correct answer is a) Latissimus dorsi.
3. Dorsal scapular nerve injury affects which muscle?
a) Rhomboid major
b) Deltoid
c) Biceps brachii
d) Subscapularis
Explanation: The dorsal scapular nerve supplies rhomboid major, rhomboid minor, and levator scapulae. Injury leads to weak scapular retraction and elevation. The correct answer is a) Rhomboid major.
4. Which nerve is derived from the medial cord of brachial plexus?
a) Medial cutaneous nerve of arm
b) Axillary nerve
c) Musculocutaneous nerve
d) Suprascapular nerve
Explanation: The medial cutaneous nerve of arm is a direct branch of the medial cord (C8–T1). It provides cutaneous sensation to the medial arm. Thus, the correct answer is a) Medial cutaneous nerve of arm.
5. Loss of shoulder adduction indicates injury to –
a) Thoracodorsal nerve
b) Axillary nerve
c) Suprascapular nerve
d) Musculocutaneous nerve
Explanation: Shoulder adduction is primarily by the latissimus dorsi, innervated by the thoracodorsal nerve. Hence, injury to this nerve causes weakness in adduction. The correct answer is a) Thoracodorsal nerve.
6. Clinical Case: A patient develops winged scapula after axillary lymph node dissection. Which nerve is likely injured?
a) Long thoracic nerve
b) Axillary nerve
c) Median nerve
d) Radial nerve
Explanation: The long thoracic nerve supplies the serratus anterior muscle. Damage during axillary surgery causes winging of the scapula, as the muscle can no longer hold the scapula against the thoracic wall. The correct answer is a) Long thoracic nerve.
7. Clinical Case: A patient presents with loss of sensation over the medial side of the arm following trauma to the axilla. Which nerve is injured?
a) Medial cutaneous nerve of arm
b) Lateral cutaneous nerve of forearm
c) Ulnar nerve
d) Musculocutaneous nerve
Explanation: The medial cutaneous nerve of arm provides sensation to the medial aspect of the arm. Injury in the axilla or during lymph node dissection can cause sensory loss in that region. Hence, the correct answer is a) Medial cutaneous nerve of arm.
8. Clinical Case: A patient after trauma to the neck shows difficulty in raising the shoulder. Which nerve is likely involved?
a) Spinal accessory nerve
b) Axillary nerve
c) Dorsal scapular nerve
d) Suprascapular nerve
Explanation: The spinal accessory nerve supplies the trapezius muscle, responsible for shoulder elevation. Injury during neck surgery causes shoulder droop and difficulty elevating the shoulder. The correct answer is a) Spinal accessory nerve.
9. Clinical Case: Injury to the posterior cord of the brachial plexus causes paralysis of which muscle?
a) Latissimus dorsi
b) Deltoid
c) Both a and b
d) None
Explanation: The posterior cord gives rise to the axillary and thoracodorsal nerves, which supply deltoid and latissimus dorsi respectively. Hence, both muscles are affected. The correct answer is c) Both a and b.
10. Clinical Case: Following breast surgery, a woman complains of difficulty in reaching forward and scapular winging. Which nerve was injured?
a) Long thoracic nerve
b) Thoracodorsal nerve
c) Axillary nerve
d) Median nerve
Explanation: The long thoracic nerve injury leads to paralysis of the serratus anterior muscle. This causes winging of the scapula and difficulty in forward arm movement. The correct answer is a) Long thoracic nerve.
Chapter: Lower Limb Anatomy; Topic: Medial Compartment of Thigh; Subtopic: Adductor Muscles and Their Nerve Supply
Keyword Definitions:
• Adductor muscles: Group of muscles in the medial compartment of the thigh responsible for adduction of the thigh, including adductor longus, brevis, magnus, gracilis, and obturator externus.
• Adductor magnus: The largest adductor muscle, consisting of adductor and hamstring parts.
• Profunda femoris artery: The main blood supply of the adductor compartment.
• Adductor tubercle: A bony prominence on the medial condyle of the femur where the hamstring part of adductor magnus inserts.
Lead Question - 2015
What is true about adductors of thigh –
a) Ischial head of adductor magnus is an adductor
b) Profunda femoris artery is the main blood supply
c) Ischial head of adductor magnus originates from adductor tubercle
d) Adductor magnus is the largest muscle
Explanation (Answer: b) Profunda femoris artery is the main blood supply
The profunda femoris artery provides the chief vascular supply to the adductor muscles of the thigh. The adductor magnus has two parts — adductor (obturator nerve) and hamstring (tibial nerve). The adductor part arises from the inferior pubic ramus, while the hamstring part arises from the ischial tuberosity and inserts into the adductor tubercle. It’s the largest adductor muscle.
1. Which nerve supplies the adductor longus muscle?
a) Femoral nerve
b) Obturator nerve
c) Sciatic nerve
d) Tibial nerve
2. Which artery mainly supplies the adductor muscles of the thigh?
a) Femoral artery
b) Profunda femoris artery
c) Popliteal artery
d) Obturator artery
3. Clinical case: A patient has difficulty bringing thighs together and loss of sensation on the medial thigh. Which nerve is affected?
a) Obturator nerve
b) Femoral nerve
c) Sciatic nerve
d) Pudendal nerve
4. The hamstring part of adductor magnus is supplied by:
a) Tibial part of sciatic nerve
b) Obturator nerve
c) Common peroneal nerve
d) Femoral nerve
5. Which of the following is not part of the adductor group of the thigh?
a) Adductor longus
b) Gracilis
c) Sartorius
d) Adductor brevis
6. Clinical case: A 40-year-old runner complains of pain in the medial thigh after sprinting. Which muscle is most likely strained?
a) Adductor longus
b) Biceps femoris
c) Tensor fasciae latae
d) Rectus femoris
7. Adductor magnus is supplied by:
a) Obturator and tibial nerves
b) Femoral and sciatic nerves
c) Tibial and femoral nerves
d) Common peroneal nerve
8. The opening in the adductor magnus for passage of femoral vessels is called:
a) Adductor canal
b) Adductor hiatus
c) Saphenous opening
d) Obturator foramen
9. Clinical case: Paralysis of the obturator nerve results in:
a) Loss of adduction of the thigh
b) Loss of knee extension
c) Loss of hip flexion
d) Foot drop
10. Which muscle acts both as an adductor and extensor of the thigh?
a) Adductor magnus
b) Adductor longus
c) Gracilis
d) Pectineus
11. The adductor tubercle serves as an attachment for:
a) Hamstring part of adductor magnus
b) Adductor longus
c) Adductor brevis
d) Pectineus
Chapter: Lower Limb Anatomy; Topic: Gluteal Region; Subtopic: Nerve Supply of Gluteal Muscles
Keyword Definitions:
• Superior gluteal nerve: A branch of the sacral plexus (L4–S1) that supplies the gluteus medius, gluteus minimus, and tensor fasciae latae muscles.
• Gluteus maximus: The largest muscle of the body, responsible for hip extension, supplied by the inferior gluteal nerve.
• Tensor fasciae latae: A muscle that assists in abduction and medial rotation of the thigh.
• Gluteus medius and minimus: Muscles that stabilize the pelvis during walking.
Lead Question - 2015
Superior gluteal nerve does not supply?
a) Tensor fasciae latae
b) Gluteus medius
c) Gluteus minimus
d) Gluteus maximus
Explanation (Answer: d) Gluteus maximus
The superior gluteal nerve (L4–S1) supplies the gluteus medius, gluteus minimus, and tensor fasciae latae. It exits through the greater sciatic foramen above the piriformis. The gluteus maximus is supplied by the inferior gluteal nerve (L5–S2), which provides motor control for hip extension and rising from a sitting position. Injury leads to difficulty climbing stairs or standing up.
1. Which nerve supplies the gluteus maximus muscle?
a) Superior gluteal nerve
b) Inferior gluteal nerve
c) Sciatic nerve
d) Pudendal nerve
2. Which muscles are supplied by the superior gluteal nerve?
a) Gluteus medius and minimus
b) Gluteus maximus and medius
c) Tensor fasciae latae and gluteus maximus
d) Piriformis and obturator internus
3. Clinical case: A patient has a waddling gait and pelvic drop on the contralateral side during walking. The lesion involves which nerve?
a) Superior gluteal nerve
b) Inferior gluteal nerve
c) Obturator nerve
d) Femoral nerve
4. The superior gluteal nerve emerges from which spinal segments?
a) L2–L4
b) L4–S1
c) L5–S2
d) S1–S3
5. Clinical case: A patient has difficulty abducting the thigh at the hip. Which muscle is likely paralyzed?
a) Gluteus medius
b) Gluteus maximus
c) Adductor longus
d) Pectineus
6. Which of the following exits through the greater sciatic foramen above the piriformis?
a) Superior gluteal nerve
b) Inferior gluteal nerve
c) Pudendal nerve
d) Sciatic nerve
7. Clinical case: After intramuscular injection in the superomedial quadrant of the gluteal region, a patient develops hip drop. Which nerve was injured?
a) Superior gluteal nerve
b) Inferior gluteal nerve
c) Sciatic nerve
d) Posterior cutaneous nerve of thigh
8. Which muscle acts as both a flexor and abductor of the hip joint?
a) Tensor fasciae latae
b) Gluteus maximus
c) Piriformis
d) Obturator externus
9. Clinical case: A 35-year-old athlete presents with inability to climb stairs after a fall. Which nerve is most likely affected?
a) Inferior gluteal nerve
b) Superior gluteal nerve
c) Obturator nerve
d) Femoral nerve
10. Which muscle, if paralyzed, will result in difficulty maintaining a level pelvis during walking?
a) Gluteus medius
b) Gluteus maximus
c) Adductor magnus
d) Psoas major
11. Trendelenburg sign is due to paralysis of which nerve?
a) Superior gluteal nerve
b) Inferior gluteal nerve
c) Obturator nerve
d) Sciatic nerve
Chapter: Lower Limb Anatomy; Topic: Femoral Nerve; Subtopic: Muscular and Sensory Distribution
Keyword Definitions:
• Femoral nerve: The largest branch of the lumbar plexus (L2–L4), supplying the anterior thigh muscles and skin of the anterior and medial leg.
• Pectineus: A flat muscle of the upper thigh that flexes and adducts the hip; mainly supplied by the femoral nerve.
• Obturator externus: A muscle responsible for lateral rotation of the thigh, innervated by the obturator nerve.
• Vastus medialis: A part of the quadriceps group that stabilizes the patella and extends the knee.
Lead Question - 2015
Femoral nerve supplies all except?
a) Pectineus
b) Sartorius
c) Vastus medialis
d) Obturator externus
Explanation (Answer: d) Obturator externus
The femoral nerve (L2–L4) supplies the muscles of the anterior thigh including quadriceps femoris, pectineus (partially), and sartorius. The obturator externus is an exception; it is supplied by the posterior division of the obturator nerve. Understanding these innervations is important in diagnosing nerve injuries affecting hip movements and knee extension strength.
1. Which root values contribute to the formation of the femoral nerve?
a) L1–L3
b) L2–L4
c) L3–L5
d) L4–S1
2. Which of the following is NOT a branch of the femoral nerve in the thigh?
a) Saphenous nerve
b) Muscular branches
c) Obturator branch
d) Articular branches
3. Clinical case: A patient cannot extend the knee after pelvic surgery. Which nerve is most likely injured?
a) Femoral nerve
b) Obturator nerve
c) Sciatic nerve
d) Lateral femoral cutaneous nerve
4. Which of the following muscles receives dual nerve supply from femoral and obturator nerves?
a) Pectineus
b) Sartorius
c) Adductor longus
d) Vastus lateralis
5. Clinical case: A 50-year-old man presents with loss of sensation over the anterior thigh and medial leg. The likely lesion involves?
a) Femoral nerve
b) Obturator nerve
c) Tibial nerve
d) Common peroneal nerve
6. Which of the following is a sensory continuation of the femoral nerve beyond the adductor canal?
a) Saphenous nerve
b) Obturator nerve
c) Lateral femoral cutaneous nerve
d) Ilioinguinal nerve
7. Femoral nerve emerges from the lateral border of which muscle in the abdomen?
a) Psoas major
b) Iliacus
c) Quadratus lumborum
d) Obturator internus
8. Clinical case: During catheterization, the patient feels sharp pain radiating down the anterior thigh. Which structure might be irritated?
a) Femoral nerve
b) Obturator nerve
c) Genitofemoral nerve
d) Iliohypogastric nerve
9. Which of the following statements about the femoral nerve is TRUE?
a) Lies outside the femoral sheath
b) Lies inside the femoral sheath
c) Lies within the femoral canal
d) Passes behind the femoral vein
10. Clinical case: Following trauma to the inguinal region, the patient develops weakness in hip flexion and knee extension. Which nerve is injured?
a) Femoral nerve
b) Obturator nerve
c) Sciatic nerve
d) Lateral cutaneous nerve of thigh
11. Which nerve supplies the skin over the anterior and medial aspects of the thigh?
a) Femoral nerve
b) Obturator nerve
c) Tibial nerve
d) Pudendal nerve
Topic: Lower Limb; Subtopic: Obturator Nerve and Hip Joint Movements
Keyword Definitions:
• Obturator Nerve: A nerve arising from the lumbar plexus (L2–L4) that supplies the medial thigh muscles responsible for adduction of the hip joint.
• Hip Joint Adduction: Movement of the thigh toward the midline, primarily performed by adductor muscles.
• Femoral Nerve: A major nerve of the anterior thigh supplying extensor muscles of the knee and sensation to the anterior thigh.
• Saphenous Nerve: A sensory branch of the femoral nerve that supplies the medial leg and foot.
• Sciatic Nerve: The largest nerve in the body, supplying the posterior thigh, leg, and foot.
Lead Question – 2015
A patient presents with defective adduction of the hip joint and pains in the hip and knee joint. Which nerve is involved?
a) Obturator nerve
b) Femoral nerve
c) Saphenous nerve
d) Sciatic nerve
Explanation: The obturator nerve (L2–L4) supplies the adductor muscles of the medial thigh. Injury to this nerve results in loss of hip adduction and referred pain to the medial thigh and knee, due to its articular branches. This may occur during pelvic surgery or childbirth trauma. Answer: (a) Obturator nerve.
1. The obturator nerve emerges from which part of the lumbar plexus?
a) L1–L2
b) L2–L4
c) L4–S1
d) L5–S2
Explanation: The obturator nerve originates from the anterior divisions of L2–L4 roots of the lumbar plexus. It descends through the pelvis, exiting via the obturator foramen to supply adductor muscles. Answer: (b) L2–L4.
2. Which muscle is supplied by the posterior division of the obturator nerve?
a) Adductor longus
b) Adductor brevis
c) Adductor magnus (adductor part)
d) Gracilis
Explanation: The posterior division of the obturator nerve supplies the adductor magnus (adductor part) and obturator externus muscles. The anterior division supplies adductor longus, adductor brevis, and gracilis. Answer: (c) Adductor magnus (adductor part).
3. In case of obturator nerve injury, which movement is primarily affected?
a) Flexion of the knee
b) Adduction of the thigh
c) Abduction of the thigh
d) Extension of the hip
Explanation: The obturator nerve controls adductor muscles. Damage to it causes weakness in adduction of the thigh, making crossing legs difficult. Sensory loss may occur on the medial thigh. Answer: (b) Adduction of the thigh.
4. The sensory branch of the obturator nerve supplies:
a) Lateral thigh
b) Medial thigh
c) Posterior thigh
d) Anterior leg
Explanation: The obturator nerve provides sensory innervation to the medial aspect of the thigh and articular branches to the hip and knee joints. Damage causes pain in these regions. Answer: (b) Medial thigh.
5. A pelvic tumor compressing the obturator nerve will cause weakness in:
a) Knee flexion
b) Thigh abduction
c) Thigh adduction
d) Knee extension
Explanation: Compression of the obturator nerve affects adductor muscles of the thigh, leading to weakness in adduction and instability during walking. Answer: (c) Thigh adduction.
6. Which of the following is not a muscle supplied by the obturator nerve?
a) Gracilis
b) Adductor longus
c) Pectineus
d) Obturator externus
Explanation: The pectineus muscle is mainly supplied by the femoral nerve and occasionally by the obturator nerve. However, it is not consistently innervated by it. Answer: (c) Pectineus.
7. During childbirth, compression of which nerve may lead to difficulty in thigh adduction?
a) Obturator nerve
b) Pudendal nerve
c) Femoral nerve
d) Sciatic nerve
Explanation: Prolonged labor or forceps delivery can compress the obturator nerve against the pelvic wall, leading to weakness in adduction and pain in the medial thigh. Answer: (a) Obturator nerve.
8. Which of the following tests is used clinically to assess obturator nerve integrity?
a) Knee jerk reflex
b) Adductor reflex
c) Achilles reflex
d) Patellar reflex
Explanation: The adductor reflex tests the integrity of the obturator nerve (L2–L4). The examiner taps the medial thigh while observing adductor muscle contraction. Answer: (b) Adductor reflex.
9. A patient with obturator nerve injury may also complain of referred pain to:
a) Lateral thigh
b) Medial thigh and knee
c) Posterior leg
d) Anterior thigh
Explanation: Obturator nerve injury often causes referred pain to the medial thigh and knee because of its articular branches to these joints. Answer: (b) Medial thigh and knee.
10. In total hip arthroplasty, inadvertent damage to which nerve causes loss of thigh adduction and medial thigh sensation?
a) Obturator nerve
b) Femoral nerve
c) Sciatic nerve
d) Superior gluteal nerve
Explanation: During hip replacement surgery, the obturator nerve can be injured as it runs along the pelvic wall. This leads to weakness in adduction and sensory loss on the medial thigh. Answer: (a) Obturator nerve.
Chapter: Upper Limb Anatomy; Topic: Nerve Supply of Hand; Subtopic: Innervation of Nail Bed
Keyword Definitions:
• Median Nerve: Supplies most flexor muscles of forearm and lateral hand including palmar surface and nail beds of thumb, index, and middle fingers.
• Radial Nerve: Supplies extensor compartment of forearm and dorsal surface of hand except nail beds of lateral three and a half fingers.
• Ulnar Nerve: Supplies intrinsic hand muscles and medial one and a half fingers including their nail beds.
• Nail Bed: The skin beneath the nail plate supplied by cutaneous branches of digital nerves.
Lead Question – 2015
The nerve supply of nail bed of index finger is ?
a) Superficial br of radial nerve
b) Deep br of radial nerve
c) Median nerve
d) Ulnar nerve
Explanation: The median nerve supplies the nail beds of the index, middle, and lateral half of the ring finger via its palmar digital branches. Although the radial nerve supplies the dorsum of these fingers, the nail beds are derived from palmar side sensory supply. Hence, the correct answer is c) Median nerve. (100 words)
1. A patient presents with loss of sensation over the tip of the index finger. The most likely affected nerve is:
a) Ulnar nerve
b) Median nerve
c) Radial nerve
d) Musculocutaneous nerve
Explanation: The median nerve provides sensory supply to the palmar surface and tips (nail beds) of the thumb, index, middle, and half of the ring finger. Loss of sensation over the index fingertip suggests median nerve injury, typically seen in carpal tunnel syndrome. Hence, the correct answer is b) Median nerve. (100 words)
2. The dorsal surface of the hand is mainly supplied by:
a) Median nerve
b) Ulnar nerve
c) Radial nerve
d) Axillary nerve
Explanation: The radial nerve supplies sensation to the dorsum of the hand, especially the lateral two-thirds and dorsal aspects of proximal phalanges, excluding the nail beds. The ulnar nerve supplies the medial one-third of the dorsum. Median nerve’s contribution is minimal dorsally. Therefore, the correct answer is c) Radial nerve. (100 words)
3. Which of the following nerves is compressed in carpal tunnel syndrome?
a) Ulnar nerve
b) Median nerve
c) Radial nerve
d) Musculocutaneous nerve
Explanation: Carpal tunnel syndrome involves compression of the median nerve beneath the flexor retinaculum at the wrist. Symptoms include tingling, pain, and numbness in the lateral three and a half fingers and weakness of thenar muscles. Hence, the correct answer is b) Median nerve. (100 words)
4. In a patient with ulnar nerve injury at the wrist, which sensory area is affected?
a) Palmar surface of thumb
b) Nail bed of middle finger
c) Medial one and half fingers
d) Dorsum of hand laterally
Explanation: The ulnar nerve supplies the skin of the medial one and half fingers and their nail beds. Injury causes sensory loss in these areas, especially over the little finger and medial half of the ring finger. Therefore, the correct answer is c) Medial one and half fingers. (100 words)
5. Clinical test for median nerve injury involves checking for loss of sensation in:
a) Tip of thumb
b) Dorsum of hand
c) Medial palm
d) Lateral forearm
Explanation: Loss of sensation at the tip of the thumb indicates median nerve impairment since its digital branches supply this area. The test is useful for assessing sensory loss in carpal tunnel syndrome or distal median neuropathy. Hence, the correct answer is a) Tip of thumb. (100 words)
6. (Clinical) A typist complains of tingling in the thumb, index, and middle fingers, worsening at night. The probable diagnosis is:
a) Ulnar nerve palsy
b) Radial nerve palsy
c) Carpal tunnel syndrome
d) Thoracic outlet syndrome
Explanation: The described symptoms are classic for carpal tunnel syndrome, due to compression of the median nerve at the wrist. Nocturnal paresthesia and hand weakness are key features. Treatment includes splinting or surgical decompression. The correct answer is c) Carpal tunnel syndrome. (100 words)
7. (Clinical) A cut over the medial wrist results in loss of sensation over the little finger nail bed. Which nerve is injured?
a) Median nerve
b) Ulnar nerve
c) Radial nerve
d) Musculocutaneous nerve
Explanation: The ulnar nerve supplies the little finger and medial half of ring finger including their nail beds through its digital branches. Injury near the wrist leads to sensory loss in these regions. Hence, the correct answer is b) Ulnar nerve. (100 words)
8. (Clinical) A patient after humeral shaft fracture cannot extend the wrist. Which nerve is injured?
a) Median nerve
b) Ulnar nerve
c) Radial nerve
d) Axillary nerve
Explanation: Wrist drop following humeral shaft fracture indicates radial nerve injury. This nerve supplies the extensor compartment of the forearm, and its loss leads to inability to extend the wrist and fingers. Hence, the correct answer is c) Radial nerve. (100 words)
9. (Clinical) Loss of flexion at the distal interphalangeal joint of the index finger is due to lesion of:
a) Median nerve
b) Ulnar nerve
c) Radial nerve
d) Axillary nerve
Explanation: The anterior interosseous branch of median nerve supplies flexor digitorum profundus for index and middle fingers. Injury causes inability to flex the distal interphalangeal joints of these fingers, producing a characteristic “pointing index.” The correct answer is a) Median nerve. (100 words)
10. (Clinical) A patient has loss of sensation on dorsum of hand except fingertips. Which nerve is affected?
a) Median nerve
b) Radial nerve
c) Ulnar nerve
d) Musculocutaneous nerve
Explanation: The superficial branch of the radial nerve provides sensation to most of the dorsal hand, except the fingertips (nail beds) which are supplied by palmar digital branches of median and ulnar nerves. Thus, loss of dorsal hand sensation with sparing of fingertips suggests radial nerve lesion. Correct answer: b) Radial nerve. (100 words)
Chapter: Neuroanatomy; Topic: Spinal Nerve Roots; Subtopic: Cervical Nerve Root Lesions
Keyword Definitions:
• Herniated Intervertebral Disc: Protrusion of nucleus pulposus through the annulus fibrosus, compressing adjacent nerve roots.
• Cervical Nerve Roots: Nerves arising from the cervical spinal cord that control upper limb muscles.
• C5 Nerve Root: Supplies deltoid and biceps brachii muscles responsible for shoulder abduction and elbow flexion.
• Elbow Flexion: Bending the elbow joint mainly by biceps brachii and brachialis.
• Disc Herniation Symptoms: Include pain, numbness, and weakness in corresponding myotomal distribution.
Lead Question – 2015
A patient has a herniated intervertebral disc impinging on the right C5 nerve roots. Which of the following movements would most likely be affected?
a) Extension of the fingers
b) Extension of the shoulder
c) Flexion of the elbow
d) Flexion of the wrist
Explanation: The C5 nerve root primarily innervates the deltoid and biceps brachii muscles, which control shoulder abduction and elbow flexion. Herniation compressing C5 results in weakness of elbow flexion and shoulder movement, along with diminished biceps reflex. The correct answer is c) Flexion of the elbow as it represents the key function affected by C5 root involvement.
Guessed Questions:
1. Which spinal nerve root is primarily responsible for wrist extension?
a) C5
b) C6
c) C7
d) C8
Explanation: The C6 nerve root innervates muscles such as the extensor carpi radialis longus and brevis that extend the wrist. C7 mainly controls finger extension, while C8 influences finger flexion. Compression of the C6 root causes weakness in wrist extension and loss of biceps reflex. Hence, the correct answer is b) C6.
2. Which reflex is affected by C5 nerve root compression?
a) Triceps reflex
b) Biceps reflex
c) Brachioradialis reflex
d) Knee jerk reflex
Explanation: The biceps reflex is mediated by the C5 and C6 spinal roots, primarily C5. Compression of C5 impairs this reflex and weakens elbow flexion. The triceps reflex (C7–C8) and brachioradialis reflex (C6) remain intact. Hence, the correct answer is b) Biceps reflex.
3. A patient with C6 radiculopathy may have weakness in which movement?
a) Finger abduction
b) Wrist extension
c) Shoulder adduction
d) Elbow extension
Explanation: The C6 nerve root controls wrist extension via the extensor carpi radialis and contributes to elbow flexion. Compression causes weakness in wrist extension, sensory loss in the thumb, and diminished brachioradialis reflex. Hence, the correct answer is b) Wrist extension.
4. C7 nerve root lesion results in difficulty performing which action?
a) Elbow flexion
b) Elbow extension
c) Shoulder abduction
d) Wrist flexion
Explanation: The C7 root supplies the triceps brachii, responsible for elbow extension. Compression causes weakness in extending the elbow and fingers, and loss of triceps reflex. Sensory loss occurs in the middle finger region. Hence, the correct answer is b) Elbow extension.
5. Which intervertebral disc herniation most likely affects the C5 nerve root?
a) C3–C4 disc
b) C4–C5 disc
c) C5–C6 disc
d) C6–C7 disc
Explanation: In the cervical spine, the nerve root exiting above the herniated disc is usually affected. Thus, herniation at the C4–C5 disc compresses the C5 nerve root, leading to pain and weakness in the shoulder and elbow flexion. Hence, the correct answer is b) C4–C5 disc.
6. A patient presents with loss of biceps reflex and shoulder abduction. Which nerve root is most likely involved?
a) C5
b) C6
c) C7
d) C8
Explanation: The C5 root controls shoulder abduction (deltoid) and elbow flexion (biceps). Injury results in weakness in both movements and absent biceps reflex. C6–C8 control lower arm functions. Hence, the correct answer is a) C5.
7. Pain radiating over the lateral shoulder and upper arm suggests involvement of which nerve root?
a) C4
b) C5
c) C6
d) C7
Explanation: The C5 dermatome covers the lateral shoulder and upper arm. Herniation at C4–C5 compressing the C5 root causes pain, tingling, and weakness in this region. The pattern helps differentiate it from lower cervical lesions. Hence, the correct answer is b) C5.
8. A 40-year-old patient has difficulty abducting the shoulder and diminished biceps reflex. Which disc level is affected?
a) C3–C4
b) C4–C5
c) C5–C6
d) C6–C7
Explanation: The C4–C5 disc herniation compresses the C5 root, impairing deltoid and biceps function. Symptoms include shoulder abduction weakness and reduced biceps reflex. Hence, the correct answer is b) C4–C5.
9. Which muscle would show weakness in C5 root compression?
a) Deltoid
b) Triceps
c) Extensor digitorum
d) Flexor carpi ulnaris
Explanation: The Deltoid muscle, innervated by the axillary nerve (C5–C6), is primarily affected in C5 compression. The patient cannot abduct the arm efficiently. Hence, the correct answer is a) Deltoid.
10. A patient with cervical disc prolapse presents with weakness in shoulder abduction and elbow flexion, sensory loss over lateral arm, and absent biceps reflex. Which root is compressed?
a) C4
b) C5
c) C6
d) C7
Explanation: This clinical presentation is classical of C5 root compression. It affects the deltoid and biceps muscles, leading to weakness of shoulder abduction and elbow flexion, sensory loss over the lateral arm, and absent biceps reflex. Hence, the correct answer is b) C5.
Chapter: Upper Limb Anatomy; Topic: Shoulder Muscles; Subtopic: Teres Minor and Its Nerve Supply
Keyword Definitions:
• Teres minor: A small rotator cuff muscle located on the posterior aspect of the scapula that assists in lateral rotation of the arm.
• Axillary nerve: A branch of the posterior cord of the brachial plexus (C5–C6) that supplies the deltoid and teres minor muscles.
• Rotator cuff: A group of four muscles (supraspinatus, infraspinatus, teres minor, subscapularis) that stabilize the shoulder joint.
Lead Question - 2015
Teres minor is supplied by:
a) Suprascapular nerve
b) Infrascapular nerve
c) Thoracodorsal nerve
d) Axillary nerve
Explanation:
The teres minor muscle is supplied by the axillary nerve (C5, C6) from the posterior cord of the brachial plexus. This muscle assists in the lateral rotation of the arm and stabilizes the humeral head in the glenoid cavity. Injury to the axillary nerve can weaken abduction and lateral rotation. Hence, the correct answer is d) Axillary nerve.
1) The teres minor muscle is part of which muscle group?
a) Rotator cuff muscles
b) Pectoral muscles
c) Axial muscles
d) Scapular stabilizers
Explanation:
The teres minor belongs to the rotator cuff group along with supraspinatus, infraspinatus, and subscapularis. These muscles surround the shoulder joint, providing stability and allowing precise rotational movements. The teres minor specifically aids in lateral rotation and prevents humeral head dislocation. Correct answer: a) Rotator cuff muscles.
2) Which nerve supplies both deltoid and teres minor muscles?
a) Radial nerve
b) Axillary nerve
c) Musculocutaneous nerve
d) Suprascapular nerve
Explanation:
The axillary nerve innervates both the deltoid and teres minor muscles. It originates from the posterior cord of the brachial plexus (C5, C6). The nerve also gives rise to the superior lateral cutaneous nerve of the arm, providing sensation to the deltoid region. Answer: b) Axillary nerve.
3) Which of the following actions is performed by the teres minor?
a) Medial rotation
b) Lateral rotation
c) Flexion of the arm
d) Abduction of the arm
Explanation:
The teres minor functions mainly in lateral rotation of the arm at the shoulder joint. It acts synergistically with the infraspinatus muscle to stabilize the humeral head during movement. This helps in controlled shoulder motion and joint protection. The correct answer is b) Lateral rotation.
4) Clinical Case: A patient presents with difficulty in abducting the arm beyond 15° and weakness in lateral rotation. Which nerve is injured?
a) Axillary nerve
b) Radial nerve
c) Suprascapular nerve
d) Long thoracic nerve
Explanation:
Loss of abduction beyond 15° (deltoid) and weakness in lateral rotation (teres minor) suggest an axillary nerve injury. This nerve supplies both muscles and is vulnerable during shoulder dislocation or surgical neck fracture of the humerus. Correct answer: a) Axillary nerve.
5) The axillary nerve passes through which anatomical space?
a) Triangular interval
b) Quadrangular space
c) Triangular space
d) Subscapular space
Explanation:
The axillary nerve travels through the quadrangular space along with the posterior circumflex humeral artery. The boundaries are teres minor (superior), teres major (inferior), long head of triceps (medial), and surgical neck of humerus (lateral). Injury here may cause deltoid paralysis. Answer: b) Quadrangular space.
6) Clinical Case: During shoulder surgery, a small branch near the posterior circumflex humeral artery is injured. The patient later develops weakness in lateral rotation. Which muscle is affected?
a) Subscapularis
b) Teres major
c) Teres minor
d) Infraspinatus
Explanation:
The teres minor receives its branch from the axillary nerve in the quadrangular space near the posterior circumflex humeral artery. Injury to this branch results in weakness in lateral rotation. Teres major is supplied by the lower subscapular nerve. Correct answer: c) Teres minor.
7) Which muscle is NOT part of the rotator cuff?
a) Teres minor
b) Supraspinatus
c) Infraspinatus
d) Teres major
Explanation:
The rotator cuff includes supraspinatus, infraspinatus, teres minor, and subscapularis. The teres major is not part of the cuff; it adducts and medially rotates the arm and is supplied by the lower subscapular nerve. Hence, the correct answer is d) Teres major.
8) Clinical Case: A patient with posterior shoulder dislocation develops weakness in abduction and lateral rotation. Which nerve is involved?
a) Radial nerve
b) Axillary nerve
c) Suprascapular nerve
d) Musculocutaneous nerve
Explanation:
The axillary nerve is injured in posterior shoulder dislocation due to its close relation to the humeral neck. It causes paralysis of the deltoid and teres minor, leading to loss of abduction and lateral rotation. Correct answer: b) Axillary nerve.
9) Which artery accompanies the axillary nerve in the quadrangular space?
a) Subscapular artery
b) Posterior circumflex humeral artery
c) Anterior circumflex humeral artery
d) Thoracodorsal artery
Explanation:
The posterior circumflex humeral artery accompanies the axillary nerve through the quadrangular space. This vessel arises from the third part of the axillary artery and supplies the deltoid and teres minor. Injury may cause hemorrhage and nerve damage. Correct answer: b) Posterior circumflex humeral artery.
10) Damage to the axillary nerve affects which sensory area?
a) Lateral arm
b) Medial forearm
c) Posterior forearm
d) Medial arm
Explanation:
The axillary nerve gives rise to the superior lateral cutaneous nerve of the arm, supplying the skin over the deltoid (regimental badge area). Damage causes numbness or paresthesia here. This finding helps in clinical diagnosis of axillary nerve injury. Correct answer: a) Lateral arm.
Chapter: Upper Limb Anatomy; Topic: Brachial Plexus; Subtopic: Thoracodorsal Nerve
Keyword Definitions:
• Thoracodorsal nerve: Nerve arising from the posterior cord of the brachial plexus that supplies the latissimus dorsi muscle.
• Brachial plexus: A network of nerves formed by the anterior rami of C5–T1 spinal nerves, responsible for innervating the upper limb.
• Latissimus dorsi: A broad back muscle involved in extension, adduction, and medial rotation of the arm.
Lead Question - 2015
Root value of thoracodorsal nerve?
a) C5, C6, C7
b) C8, T1
c) C6, C7, C8
d) T1, T2
Explanation:
The thoracodorsal nerve arises from the posterior cord of the brachial plexus with root values of C6, C7, and C8. It supplies the latissimus dorsi muscle, which helps in arm adduction, extension, and medial rotation. Injury to this nerve can cause weakness in pulling movements and climbing. Hence, the correct answer is c) C6, C7, C8.
1) Which nerve supplies the latissimus dorsi muscle?
a) Axillary nerve
b) Thoracodorsal nerve
c) Suprascapular nerve
d) Long thoracic nerve
Explanation:
The thoracodorsal nerve (middle subscapular nerve) supplies the latissimus dorsi muscle. This muscle extends, adducts, and medially rotates the arm. The nerve originates from the posterior cord of the brachial plexus (C6–C8). Damage affects activities like climbing or swimming that require strong shoulder adduction and extension. Answer: b) Thoracodorsal nerve.
2) The thoracodorsal nerve is a branch of:
a) Medial cord
b) Posterior cord
c) Lateral cord
d) Upper trunk
Explanation:
The thoracodorsal nerve arises from the posterior cord of the brachial plexus, along with the upper and lower subscapular nerves. It carries fibers from C6–C8 roots and supplies the latissimus dorsi. The posterior cord primarily gives nerves that innervate the posterior compartment of the upper limb. Answer: b) Posterior cord.
3) Which muscle action is lost if the thoracodorsal nerve is injured?
a) Abduction of arm
b) Extension and adduction of arm
c) Flexion of forearm
d) Supination of forearm
Explanation:
The latissimus dorsi, innervated by the thoracodorsal nerve, performs arm extension, adduction, and medial rotation. Injury to this nerve leads to weakness in these movements, especially during climbing or rowing. Abduction and supination are unaffected as they involve other muscles. The correct answer is b) Extension and adduction of arm.
4) A patient has weakness in pulling the body upward while climbing. Which nerve is likely injured?
a) Axillary nerve
b) Thoracodorsal nerve
c) Musculocutaneous nerve
d) Radial nerve
Explanation:
The thoracodorsal nerve supplies the latissimus dorsi, crucial for pulling actions like climbing or swimming. Injury causes difficulty in raising the body upward by the arms. The axillary nerve affects deltoid; musculocutaneous affects forearm flexors; radial affects wrist extensors. Hence, the answer is b) Thoracodorsal nerve.
5) The thoracodorsal nerve accompanies which artery?
a) Subscapular artery
b) Circumflex scapular artery
c) Posterior circumflex humeral artery
d) Thoracoacromial artery
Explanation:
The thoracodorsal nerve accompanies the thoracodorsal artery, a branch of the subscapular artery. Both supply the latissimus dorsi muscle. They travel together along the lateral border of the scapula towards the posterior axillary wall. This anatomical association is important during axillary dissections. Correct answer: a) Subscapular artery.
6) Clinical Case: A 35-year-old man undergoes axillary lymph node dissection for carcinoma breast. He later develops weakness in adduction of the arm. Which nerve is most likely injured?
a) Axillary nerve
b) Thoracodorsal nerve
c) Long thoracic nerve
d) Median nerve
Explanation:
During axillary dissection, the thoracodorsal nerve may be injured because it runs close to lymph nodes and vessels. This causes weakness in adduction and extension of the arm due to paralysis of the latissimus dorsi. The long thoracic nerve causes winged scapula. Correct answer: b) Thoracodorsal nerve.
7) In brachial plexus anatomy, thoracodorsal nerve lies between:
a) Upper and lower subscapular nerves
b) Medial and lateral pectoral nerves
c) Suprascapular and axillary nerves
d) Radial and musculocutaneous nerves
Explanation:
The thoracodorsal nerve lies between the upper and lower subscapular nerves, arising from the posterior cord. These three nerves innervate posterior axillary wall muscles—subscapularis and latissimus dorsi. Their anatomical proximity is vital in axillary surgery to avoid injury. Correct answer: a) Upper and lower subscapular nerves.
8) Clinical Case: A surgeon performing mastectomy notes a nerve running with subscapular vessels to the latissimus dorsi. Preserving this nerve is crucial because:
a) It controls shoulder abduction
b) It supplies latissimus dorsi
c) It supplies serratus anterior
d) It controls biceps brachii
Explanation:
The nerve described is the thoracodorsal nerve, which accompanies the subscapular vessels to supply the latissimus dorsi. Preservation is essential to maintain arm adduction and extension during shoulder movement, vital for postoperative limb function. Hence, the answer is b) It supplies latissimus dorsi.
9) Injury to the thoracodorsal nerve leads to weakness in:
a) Abduction and lateral rotation
b) Extension and medial rotation
c) Flexion and supination
d) Pronation and adduction
Explanation:
The latissimus dorsi muscle, innervated by the thoracodorsal nerve, is primarily responsible for arm extension, adduction, and medial rotation. Damage results in weakness of these movements, especially while performing activities requiring strong shoulder depression and pulling. The correct answer is b) Extension and medial rotation.
10) Which root value combination is incorrect for the thoracodorsal nerve?
a) C6
b) C7
c) C8
d) T1
Explanation:
The thoracodorsal nerve arises from the posterior cord of the brachial plexus, with root values from C6, C7, and C8. It has no contribution from T1. The incorrect option is d) T1. Understanding root values is vital for correlating nerve injuries with muscle weakness patterns.
Chapter: Nervous System; Topic: Brachial Plexus Lesions; Subtopic: Erb’s Palsy – Nerve Roots and Clinical Manifestations
Keyword Definitions:
• Brachial Plexus: A network of spinal nerves from C5–T1 that supplies the upper limb.
• Erb’s Point: Junction of C5–C6 roots where upper trunk of the brachial plexus is formed.
• Erb’s Palsy: Paralysis caused by injury to the upper trunk (C5–C6) resulting in the “waiter’s tip” deformity.
• Waiter’s Tip Posture: The arm hangs adducted, internally rotated, with the forearm extended and pronated due to muscle paralysis.
Lead Question - 2015
Nerve roots involved in Erb's palsy:
a) C5, C6
b) C6, C7
c) C7, C8, T1
d) C5, C6, C7, C8, T1
Explanation: The correct answer is (a) C5, C6. Erb’s palsy results from injury to the upper trunk of the brachial plexus formed by the C5 and C6 nerve roots. This affects muscles like deltoid, biceps, brachialis, and supraspinatus. The characteristic deformity is “waiter’s tip” posture, with the arm adducted, internally rotated, elbow extended, and forearm pronated. Common causes include shoulder dystocia during childbirth or trauma.
1) Which part of the brachial plexus is affected in Erb’s palsy?
a) Upper trunk
b) Middle trunk
c) Lower trunk
d) Posterior cord
Explanation: The correct answer is (a) Upper trunk. Erb’s palsy involves the upper trunk (C5–C6 roots) of the brachial plexus. This lesion causes paralysis of shoulder abductors and elbow flexors leading to the “waiter’s tip” deformity. Middle and lower trunks involve different nerve root levels and cause different clinical presentations such as Klumpke’s palsy.
2) (Clinical) A newborn with shoulder dystocia presents with inability to flex the elbow and arm adduction. The most likely diagnosis is:
a) Klumpke’s palsy
b) Erb’s palsy
c) Radial nerve injury
d) Median nerve injury
Explanation: The correct answer is (b) Erb’s palsy. In difficult labor, traction on the infant’s neck stretches the C5–C6 roots. This results in paralysis of the deltoid, biceps, and supraspinatus muscles. The arm remains adducted, internally rotated, and the elbow extended. Early physiotherapy helps in preventing contractures and promoting recovery.
3) Which of the following muscles is paralyzed in Erb’s palsy?
a) Deltoid
b) Trapezius
c) Latissimus dorsi
d) Pectoralis minor
Explanation: The correct answer is (a) Deltoid. The deltoid, supplied by the axillary nerve (C5–C6), is affected in Erb’s palsy. Its paralysis results in loss of shoulder abduction. Trapezius is supplied by the spinal accessory nerve and remains unaffected. Other affected muscles include supraspinatus, biceps brachii, and brachialis.
4) (Clinical) A 25-year-old biker sustains trauma to the neck and shoulder. He presents with an adducted, internally rotated arm and extended elbow. Diagnosis is:
a) Klumpke’s palsy
b) Erb’s palsy
c) Radial nerve palsy
d) Axillary nerve lesion
Explanation: The correct answer is (b) Erb’s palsy. Traction injury to the upper trunk of the brachial plexus (C5–C6) leads to Erb’s palsy. The posture described is classical—“waiter’s tip.” The patient cannot abduct the shoulder or flex the elbow. Motor and sensory loss correspond to the C5–C6 dermatomes.
5) The “waiter’s tip” posture in Erb’s palsy is due to paralysis of which muscles?
a) Deltoid, Biceps, Brachialis, Supraspinatus
b) Triceps, Anconeus, Extensor carpi radialis
c) Flexor carpi ulnaris, Palmaris longus
d) Pectoralis minor, Subscapularis
Explanation: The correct answer is (a) Deltoid, Biceps, Brachialis, Supraspinatus. These muscles are supplied by C5–C6 roots, which are injured in Erb’s palsy. Their paralysis causes arm adduction, internal rotation, and extended elbow, resulting in the “waiter’s tip” deformity.
6) (Clinical) Which nerve is responsible for loss of shoulder abduction in Erb’s palsy?
a) Axillary nerve
b) Radial nerve
c) Median nerve
d) Ulnar nerve
Explanation: The correct answer is (a) Axillary nerve. The axillary nerve arises from the posterior cord (C5–C6) and supplies the deltoid and teres minor muscles. Damage to the C5–C6 roots causes deltoid paralysis, leading to loss of shoulder abduction in Erb’s palsy.
7) In Erb’s palsy, which movement remains intact?
a) Shoulder abduction
b) Elbow flexion
c) Wrist flexion
d) Forearm supination
Explanation: The correct answer is (c) Wrist flexion. Wrist flexors are supplied by C7–C8 via the median and ulnar nerves and are unaffected in Erb’s palsy. Shoulder abduction and elbow flexion are lost due to C5–C6 root injury, leading to a limp, pronated arm.
8) (Clinical) A child presents with inability to lift the arm overhead and flex the elbow following a fall. The most probable nerve roots affected are:
a) C3–C4
b) C5–C6
c) C7–C8
d) T1–T2
Explanation: The correct answer is (b) C5–C6. Erb’s palsy results from C5–C6 injury. These roots supply the axillary, musculocutaneous, and suprascapular nerves responsible for shoulder abduction and elbow flexion. Hence, trauma near the neck or shoulder region often causes this characteristic motor deficit.
9) Sensory loss in Erb’s palsy is observed over which area?
a) Medial forearm
b) Lateral arm
c) Posterior forearm
d) Palm
Explanation: The correct answer is (b) Lateral arm. The C5 dermatome covers the lateral upper arm supplied by the axillary and musculocutaneous nerves. Therefore, injury to C5–C6 roots results in sensory deficit over this area in Erb’s palsy, along with characteristic motor weakness.
10) (Clinical) Which of the following is an important part of Erb’s palsy rehabilitation?
a) Early physiotherapy and passive movements
b) Complete immobilization
c) Surgical nerve repair immediately
d) Use of corticosteroids
Explanation: The correct answer is (a) Early physiotherapy and passive movements. Early physiotherapy helps prevent contractures and maintain muscle tone. Gentle exercises improve the range of motion during recovery. Surgery is reserved for severe or non-recovering cases after 6–9 months of conservative treatment.
Chapter: Nervous System; Topic: Brachial Plexus Injuries; Subtopic: Erb’s Palsy – Nerve Root Lesions and Upper Limb Deformities
Keyword Definitions:
• Erb’s Palsy: A paralysis caused by injury to the upper trunk (C5–C6) of the brachial plexus, often during difficult childbirth or trauma.
• Brachial Plexus: A network of nerves formed by ventral rami of C5–T1 spinal nerves supplying the upper limb.
• Waiter’s Tip Posture: Typical deformity of Erb’s palsy involving arm adduction, internal rotation, and forearm pronation.
• Nerve Roots: Spinal nerve origins where sensory and motor fibers emerge from the spinal cord.
Lead Question - 2015
Upper limb deformity in Erb's palsy?
a) Adduction and lateral rotation of arm
b) Adduction and medial rotation of arm
c) Abduction and lateral rotation of arm
d) Abduction and medial rotation of arm
Explanation: The correct answer is (b) Adduction and medial rotation of arm. Erb’s palsy results from injury to the upper trunk (C5–C6) affecting suprascapular, musculocutaneous, and axillary nerves. The patient shows “waiter’s tip” posture—arm adducted and medially rotated, elbow extended, forearm pronated. It occurs due to loss of abduction, external rotation, and flexion caused by paralysis of deltoid, biceps, and supraspinatus muscles.
1) Which nerve roots are primarily affected in Erb’s palsy?
a) C3–C4
b) C5–C6
c) C7–C8
d) T1–T2
Explanation: The correct answer is (b) C5–C6. Erb’s palsy occurs due to injury at Erb’s point involving upper trunk (C5–C6). Muscles supplied by these roots—deltoid, biceps, brachialis, and supraspinatus—are paralyzed, resulting in arm adduction, internal rotation, and loss of elbow flexion. Sensory loss occurs over the lateral aspect of the arm.
2) (Clinical) A newborn presents with an internally rotated arm and inability to flex the elbow after a difficult delivery. The diagnosis is:
a) Klumpke’s palsy
b) Erb’s palsy
c) Radial nerve injury
d) Musculocutaneous nerve palsy
Explanation: The correct answer is (b) Erb’s palsy. During shoulder dystocia or breech delivery, traction on the neck can stretch or tear C5–C6 roots, leading to Erb’s palsy. The “waiter’s tip” deformity is pathognomonic, characterized by arm adduction, internal rotation, and pronated forearm. Early physiotherapy prevents contractures and improves recovery.
3) Which of the following muscles is NOT affected in Erb’s palsy?
a) Deltoid
b) Biceps
c) Brachioradialis
d) Trapezius
Explanation: The correct answer is (d) Trapezius. Trapezius is supplied by the spinal accessory nerve (cranial nerve XI) and remains intact in Erb’s palsy. In contrast, muscles innervated by C5–C6 roots—deltoid, biceps, brachialis, and supraspinatus—are affected, leading to loss of shoulder abduction and elbow flexion with characteristic “waiter’s tip” posture.
4) (Clinical) A 5-year-old boy presents with inability to abduct his shoulder and flex the elbow after a fall on the shoulder. The most likely lesion is:
a) Radial nerve injury
b) Erb’s palsy
c) Klumpke’s palsy
d) Ulnar nerve injury
Explanation: The correct answer is (b) Erb’s palsy. Traction or blow to the shoulder can stretch the upper trunk of the brachial plexus. Weakness in deltoid and biceps results in loss of abduction and elbow flexion. The limb hangs in internal rotation and pronation—classically seen as “waiter’s tip” deformity of Erb’s palsy.
5) Which of the following nerves is not derived from the C5–C6 roots?
a) Suprascapular nerve
b) Musculocutaneous nerve
c) Axillary nerve
d) Ulnar nerve
Explanation: The correct answer is (d) Ulnar nerve. The ulnar nerve originates from the C8–T1 roots, supplying the intrinsic hand muscles. C5–C6 roots give rise to nerves like suprascapular, musculocutaneous, and axillary, all of which are affected in Erb’s palsy, causing weakness in shoulder and elbow movements.
6) (Clinical) In Erb’s palsy, which movement of the upper limb is preserved?
a) Shoulder abduction
b) Elbow flexion
c) Wrist flexion
d) Shoulder external rotation
Explanation: The correct answer is (c) Wrist flexion. Wrist flexors are supplied by C7–C8 roots via the median and ulnar nerves, which are not affected in Erb’s palsy. Movements like shoulder abduction and elbow flexion are lost due to paralysis of deltoid and biceps, respectively, resulting in the classical deformity.
7) Sensory loss in Erb’s palsy typically involves which region?
a) Medial arm
b) Lateral arm
c) Posterior forearm
d) Medial forearm
Explanation: The correct answer is (b) Lateral arm. Sensory deficit occurs along the C5 dermatome, which covers the lateral upper arm region. This corresponds to areas supplied by the axillary and musculocutaneous nerves, both affected in upper trunk lesions of the brachial plexus seen in Erb’s palsy.
8) (Clinical) A patient with Erb’s palsy will have difficulty performing which activity?
a) Grasping small objects
b) Raising the arm to comb hair
c) Flexing fingers
d) Extending wrist
Explanation: The correct answer is (b) Raising the arm to comb hair. This action requires shoulder abduction and external rotation, functions of deltoid and supraspinatus (C5–C6). Their paralysis in Erb’s palsy prevents the patient from lifting the hand overhead, causing significant impairment in daily activities.
9) Which of the following best describes the posture of Erb’s palsy?
a) Claw hand
b) Ape thumb
c) Waiter’s tip
d) Wrist drop
Explanation: The correct answer is (c) Waiter’s tip. The “waiter’s tip” posture results from paralysis of C5–C6 muscles. The arm lies adducted and internally rotated, the elbow extended, and the forearm pronated, as if the person is waiting for a tip—classic sign of Erb’s palsy.
10) (Clinical) In an adult with Erb’s palsy, which treatment is most beneficial in early stages?
a) Immediate surgical repair
b) Immobilization of limb
c) Active and passive physiotherapy
d) Corticosteroid therapy
Explanation: The correct answer is (c) Active and passive physiotherapy. Early physiotherapy prevents muscle atrophy and joint contractures. Gentle range-of-motion exercises maintain flexibility while nerves recover. Surgical nerve grafting is reserved for severe cases with persistent deficits after 6–9 months of conservative management.
Topic: Sensory Physiology; Subtopic: Sensory Perception and Laws
Keyword Definitions:
• Weber–Fechner Law: States that the perceived intensity of a stimulus increases as the logarithm of the actual stimulus intensity.
• Stimulus: Any detectable change in the environment that can evoke a response in a sensory receptor.
• Sensation: The subjective experience resulting from sensory receptor stimulation.
• Phantom Limb: Sensation perceived in a limb that has been amputated.
• Cortical Plasticity: The brain’s ability to reorganize neural connections based on experience or injury.
• Receptor Potential: Graded electrical change generated by a sensory receptor in response to a stimulus.
• Threshold Stimulus: The minimum intensity required to evoke a detectable sensation.
Lead Question - 2014
Weber Fechner law is related to?
a) Phantom limb
b) Force of contraction in heart
c) Intensity of stimulus and sensation felt
d) Cortical plasticity
Explanation: The Weber–Fechner law describes the relationship between the magnitude of a physical stimulus and the perceived intensity of that stimulus. It states that the perceived sensation grows as the logarithm of stimulus intensity, meaning larger increments are needed for higher intensity detection. Thus, option (c) is correct — it relates to intensity and perception.
1) Which of the following best represents Weber’s law?
a) ΔI / I = k
b) I = k log S
c) S = k log I
d) ΔI = k log S
Explanation: Weber’s law states that the just noticeable difference (ΔI) is a constant fraction (k) of the original intensity (I). This ratio remains constant for a given sensory modality. Thus, the correct expression is ΔI/I = k, making option (a) correct.
2) Which sensory modality follows the Weber–Fechner law most accurately?
a) Vision
b) Smell
c) Hearing
d) Pain
Explanation: The Weber–Fechner law applies best to hearing and vision, where changes in sound or brightness are perceived logarithmically. Among them, hearing demonstrates a strong correlation between intensity and perceived loudness. Therefore, the correct answer is (c) hearing.
3) A patient reports that doubling the intensity of sound does not double its perceived loudness. This best illustrates?
a) Weber–Fechner law
b) Bell–Magendie law
c) Boyle’s law
d) Ohm’s law
Explanation: The phenomenon where perceived intensity increases less rapidly than actual intensity is governed by the Weber–Fechner law. It reflects the logarithmic relationship between stimulus and perception. Thus, option (a) is correct.
4) According to Weber’s law, the constant (k) varies with?
a) Type of receptor
b) Type of sensation
c) Intensity of light
d) Environmental temperature
Explanation: The constant k in Weber’s law is specific to the type of sensation such as sound, light, or touch. Each sensory system has a unique sensitivity threshold. Therefore, option (b) is correct.
5) Fechner extended Weber’s law by proposing that?
a) Sensation is proportional to log stimulus intensity
b) Sensation is proportional to stimulus intensity squared
c) Sensation is independent of stimulus intensity
d) Sensation decreases with intensity
Explanation: Fechner’s extension stated that perceived sensation (S) = k log (I/I₀), where I is stimulus intensity. This describes a logarithmic increase in sensation with intensity. Hence, (a) is correct.
6) In clinical practice, Weber–Fechner law helps explain?
a) Adaptation to constant stimuli
b) Threshold of pain
c) Visual accommodation
d) Muscle fatigue
Explanation: Clinically, the Weber–Fechner law explains how patients adapt to constant sensory input—such as getting used to bright light or loud sound—through gradual desensitization. It is vital for understanding sensory adaptation. Therefore, (a) is correct.
7) Which of the following is an example of Weber–Fechner law in daily life?
a) Adjusting to room brightness
b) Feeling of hunger
c) Reflex arc activation
d) Salivary secretion to food
Explanation: The gradual adjustment of visual perception in varying light levels is an example of Weber–Fechner law. When entering a bright room from darkness, the eyes adapt logarithmically to light intensity. Hence, (a) is correct.
8) In audiology, the relationship between sound intensity and loudness follows?
a) Weber–Fechner law
b) Laplace’s law
c) Boyle’s law
d) Ohm’s law
Explanation: Sound perception does not increase linearly with physical intensity. The perceived loudness increases logarithmically with intensity, following the Weber–Fechner law. This explains the decibel scale in audiology. Hence, (a) is correct.
9) A 45-year-old man with neuropathy perceives weak touch as intense pain. This phenomenon violates which principle?
a) Weber–Fechner law
b) Bell’s law
c) Henry’s law
d) Dalton’s law
Explanation: The Weber–Fechner law assumes a proportional relationship between stimulus and perception. In neuropathy, damaged sensory pathways distort this proportionality, causing allodynia (pain from non-painful stimuli). Thus, option (a) is correct.
10) In psychophysics, the smallest detectable change in a stimulus is termed?
a) Just noticeable difference
b) Minimal perceptible unit
c) Perceptual threshold
d) Response magnitude
Explanation: The just noticeable difference (JND) is the minimum change in stimulus required to detect a difference. It’s the basis of Weber’s law (ΔI/I = k). Therefore, (a) is correct.
Chapter: Central Nervous System; Topic: Olfactory Pathways; Subtopic: Cellular Components of Olfactory Bulb
Keyword Definitions:
• Mitral Cells: Principal neurons of the olfactory bulb that receive input from olfactory receptor neurons.
• Periglomerular Cells: Interneurons surrounding glomeruli that modulate synaptic transmission.
• Olfactory Bulb: Brain structure involved in processing smell information.
• Glomeruli: Spherical structures where olfactory nerve fibers synapse.
• Medulla: Part of brainstem controlling vital reflexes.
• Geniculate Body: Thalamic nuclei for auditory and visual processing.
• Primary Visual Cortex: Area in occipital lobe for visual perception.
Lead Question - 2014
Mitral and periglomerular cells are seen in?
a) Medulla
b) Olfactory bulb
c) Primary visual cortex
d) Geniculate body
Explanation: The olfactory bulb contains both mitral cells and periglomerular cells. Mitral cells serve as the main relay neurons transmitting olfactory signals to higher centers like the olfactory cortex. Periglomerular cells regulate these synapses within glomeruli. Together, they help refine odor detection and processing, making option (b) the correct answer.
1) The first relay station for olfactory impulses is?
a) Olfactory bulb
b) Olfactory tract
c) Olfactory cortex
d) Hypothalamus
Explanation: The olfactory bulb acts as the first relay station where olfactory receptor neurons synapse with mitral and tufted cells. These signals are then transmitted to the olfactory tract and higher centers. Thus, option (a) is correct.
2) Which type of neurons are found in the olfactory epithelium?
a) Bipolar neurons
b) Unipolar neurons
c) Multipolar neurons
d) Pseudounipolar neurons
Explanation: The sensory neurons of the olfactory epithelium are bipolar neurons. They have a dendrite that detects odorant molecules and an axon that transmits signals to the olfactory bulb. Hence, the correct answer is (a).
3) Olfactory tract fibers project mainly to?
a) Medulla
b) Thalamus
c) Temporal lobe
d) Occipital lobe
Explanation: The olfactory tract primarily projects to the temporal lobe, particularly the piriform cortex and amygdala, which are involved in smell perception and emotion. Option (c) is correct.
4) Which neurotransmitter is predominantly used by periglomerular cells?
a) Glutamate
b) Dopamine
c) Serotonin
d) GABA
Explanation: Periglomerular cells are inhibitory interneurons that mainly use GABA as their neurotransmitter to modulate mitral cell activity. This inhibitory control helps fine-tune olfactory signal processing. Thus, the answer is (d).
5) Mitral cells send axons through which structure?
a) Olfactory tract
b) Optic nerve
c) Internal capsule
d) Mammillothalamic tract
Explanation: The axons of mitral cells form the olfactory tract, which conveys processed olfactory information from the bulb to the olfactory cortex. Therefore, option (a) is correct.
6) A patient with anosmia likely has a lesion in which area?
a) Olfactory bulb
b) Medulla
c) Occipital cortex
d) Pons
Explanation: Loss of smell, or anosmia, often results from damage to the olfactory bulb or the olfactory tract. Such injury interrupts odor signal transmission, making option (a) correct.
7) Damage to which cells reduces odor discrimination ability?
a) Mitral cells
b) Purkinje cells
c) Pyramidal cells
d) Betz cells
Explanation: Mitral cells in the olfactory bulb play a crucial role in odor discrimination by transmitting refined signals to higher olfactory areas. Their damage leads to difficulty differentiating odors. Hence, (a) is correct.
8) Which structure directly receives olfactory input without thalamic relay?
a) Piriform cortex
b) Thalamus
c) Hypothalamus
d) Cerebellum
Explanation: Unlike other senses, smell bypasses the thalamus and directly reaches the piriform cortex for initial processing. This makes option (a) correct.
9) In olfactory pathways, lateral inhibition is mainly mediated by?
a) Periglomerular cells
b) Mitral cells
c) Tufted cells
d) Pyramidal cells
Explanation: Periglomerular cells provide lateral inhibition, enhancing contrast between activated and inactive glomeruli. This improves odor discrimination and clarity. The correct answer is (a).
10) A 45-year-old man loses his sense of smell after head trauma. Which structure is likely damaged?
a) Cribriform plate
b) Thalamus
c) Hypothalamus
d) Cerebellum
Explanation: Trauma can shear olfactory nerve fibers as they pass through the cribriform plate of the ethmoid bone, leading to anosmia. Therefore, (a) is correct.
Chapter: Nervous System; Topic: Reflexes; Subtopic: Spinal Segmental Reflexes and Clinical Correlations
Keyword Definitions:
• Reflex: An involuntary, rapid, and predictable motor response to a stimulus.
• Spinal Segment: A region of the spinal cord giving rise to a pair of spinal nerves.
• Knee Jerk: A monosynaptic reflex testing L2–L4 spinal segments.
• Calcaneal Reflex: Also called ankle jerk, testing S1–S2 integrity.
• Patellar Reflex: Stretch reflex involving quadriceps contraction.
• Deep Tendon Reflexes (DTRs): Test spinal cord and peripheral nerve integrity.
• Monosynaptic Reflex: Reflex arc involving one synapse between sensory and motor neurons.
• Polysynaptic Reflex: Reflex involving one or more interneurons.
• Motor Neuron: Nerve cell carrying impulses from CNS to muscles.
• Spinal Shock: Transient loss of reflex activity after spinal cord injury.
Lead Question – 2014
S1 S2 is checked by which reflex?
a) Knee jerk
b) Patellar reflex
c) Calcaneal reflex
d) None
Explanation: The calcaneal reflex (ankle jerk) tests the integrity of the S1–S2 spinal segments, primarily S1. When the Achilles tendon is tapped, it stimulates the gastrocnemius and soleus muscles to contract, causing plantar flexion. This reflex assesses the health of the tibial nerve and corresponding spinal roots. Answer: (c) Calcaneal reflex.
1. Which spinal segment is tested by the knee jerk reflex?
a) L2–L4
b) L4–S1
c) S1–S2
d) C5–C6
Explanation: The knee jerk or patellar reflex assesses the L2–L4 spinal segments. Tapping the patellar tendon stretches the quadriceps muscle, causing contraction and leg extension. It evaluates the integrity of the femoral nerve and associated spinal segments. Answer: (a) L2–L4.
2. The biceps jerk tests which spinal level?
a) C5–C6
b) C7–C8
c) L2–L3
d) S1–S2
Explanation: The biceps jerk evaluates the integrity of C5–C6 spinal segments through the musculocutaneous nerve. Tapping the biceps tendon causes muscle contraction and forearm flexion. Absence indicates possible C5–C6 root lesion. Answer: (a) C5–C6.
3. The triceps reflex primarily checks which spinal root?
a) C4
b) C5
c) C6
d) C7
Explanation: The triceps reflex tests the C7 spinal segment via the radial nerve. Tapping the triceps tendon elicits elbow extension. Weak or absent response suggests a lesion in the C7 root or radial nerve. Answer: (d) C7.
4. The ankle jerk reflex involves which efferent nerve?
a) Femoral
b) Obturator
c) Tibial
d) Sciatic
Explanation: The ankle jerk or calcaneal reflex involves the tibial nerve, a branch of the sciatic nerve. It carries motor impulses to gastrocnemius and soleus muscles for plantar flexion. Impaired reflex suggests tibial or S1 root dysfunction. Answer: (c) Tibial.
5. Loss of ankle jerk is commonly seen in which condition?
a) Upper motor neuron lesion
b) Peripheral neuropathy
c) Parkinson’s disease
d) Multiple sclerosis
Explanation: Loss or reduction of ankle jerk reflex occurs in peripheral neuropathy, especially in diabetes mellitus. Damage to peripheral nerves or S1 root impairs the reflex arc, leading to diminished plantar flexion response. Answer: (b) Peripheral neuropathy.
6. A 55-year-old diabetic patient presents with absent ankle jerk but normal knee jerk. The lesion is likely at:
a) L2–L4
b) S1–S2
c) C5–C6
d) L5–S1
Explanation: Absence of ankle jerk with preserved knee jerk indicates S1–S2 root involvement. The calcaneal reflex tests the S1 segment. Diabetic neuropathy often affects distal sensory and motor fibers first. Answer: (b) S1–S2.
7. A lesion at S1 spinal root leads to loss of which reflex?
a) Knee jerk
b) Biceps jerk
c) Ankle jerk
d) Supinator jerk
Explanation: The S1 spinal root supplies the gastrocnemius and soleus muscles responsible for the ankle jerk reflex. Damage to S1 abolishes this reflex, while knee jerk remains unaffected. Answer: (c) Ankle jerk.
8. A patient with herniated L5–S1 disc complains of reduced plantar flexion. Which reflex is affected?
a) Knee jerk
b) Ankle jerk
c) Biceps jerk
d) Triceps jerk
Explanation: Herniation at L5–S1 compresses the S1 nerve root, leading to weakness in plantar flexion and loss of ankle jerk reflex. This clinical sign helps localize the lesion to S1. Answer: (b) Ankle jerk.
9. Which of the following best describes a monosynaptic reflex?
a) Withdrawal reflex
b) Stretch reflex
c) Crossed extensor reflex
d) Pupillary light reflex
Explanation: The stretch reflex (like patellar reflex) is monosynaptic, involving direct communication between a sensory and a motor neuron without interneurons. It helps maintain muscle tone and posture. Answer: (b) Stretch reflex.
10. A 45-year-old patient with spinal shock after trauma shows loss of all reflexes below the injury. This loss is due to:
a) Irreversible spinal cord damage
b) Transient suppression of reflex arcs
c) Motor neuron hyperactivity
d) Autonomic overactivity
Explanation: In spinal shock, there is temporary loss of all reflexes below the injury due to sudden interruption of descending facilitatory pathways. Reflexes gradually return as spinal neurons regain excitability. Answer: (b) Transient suppression of reflex arcs.
Chapter: Central Nervous System; Topic: Sensory Pathways; Subtopic: Thalamic Processing of Sensory Information
Keyword Definitions:
• Thalamus: Major relay center for sensory information before reaching the cerebral cortex.
• Proprioception: Sense of body position and movement.
• Pain and Temperature: Sensations transmitted via spinothalamic tract to thalamus.
• Tactile Sensation: Touch and pressure perception mediated by dorsal column system.
• Somatosensory Cortex: Region that interprets sensory signals from thalamus.
• Spinothalamic Tract: Pathway for pain, temperature, and crude touch sensations.
• Ventral Posterior Nucleus: Thalamic nucleus relaying somatic sensations to cortex.
Lead Question – 2014
Sensations which are appreciated in thalamus
a) Proprioception
b) Pain & temperature
c) Tactile sensations
d) Pressure
Explanation: Pain and temperature sensations are primarily appreciated in the thalamus, even without cortical input. The thalamus acts as a sensory gateway, receiving inputs via the spinothalamic tract. While tactile and proprioceptive sensations reach the cortex for full perception, crude pain and temperature are recognized at thalamic level. Answer: (b) Pain & temperature.
1. Which thalamic nucleus is primarily involved in the relay of somatic sensations?
a) Ventral anterior nucleus
b) Ventral posterior nucleus
c) Medial geniculate body
d) Pulvinar
Explanation: The ventral posterior nucleus (VPN) of the thalamus receives somatic sensory input from the spinothalamic and medial lemniscal tracts. It relays sensations like pain, temperature, and touch to the somatosensory cortex. VPN damage leads to contralateral sensory loss. Answer: (b) Ventral posterior nucleus.
2. Crude pain sensation can be perceived even without cerebral cortex because:
a) Medulla perceives pain
b) Thalamus has pain receptors
c) Thalamus can appreciate crude sensations
d) Brainstem acts independently
Explanation: Crude pain and temperature sensations are appreciated at the thalamic level. The thalamus, particularly the ventral posterior nucleus, receives spinothalamic input. Even if cortical areas are damaged, crude sensations persist due to thalamic processing. Answer: (c) Thalamus can appreciate crude sensations.
3. Fine touch and proprioception reach the thalamus through which pathway?
a) Spinothalamic tract
b) Corticospinal tract
c) Dorsal column–medial lemniscus pathway
d) Reticulospinal tract
Explanation: The dorsal column–medial lemniscus pathway transmits fine touch, vibration, and proprioception to the thalamus. Fibers ascend ipsilaterally in dorsal columns, decussate in medulla, and terminate in the ventral posterior nucleus of thalamus. Answer: (c) Dorsal column–medial lemniscus pathway.
4. A patient with right thalamic stroke experiences left-sided loss of pain and temperature sensation. The tract involved is:
a) Dorsal column
b) Corticospinal tract
c) Spinothalamic tract
d) Spinocerebellar tract
Explanation: The spinothalamic tract carries pain and temperature sensations from the contralateral side of the body to the thalamus. A right thalamic lesion causes left-sided sensory deficits. This tract decussates soon after entering the spinal cord. Answer: (c) Spinothalamic tract.
5. Which of the following sensations is lost in a lesion of the ventral posterior nucleus of the thalamus?
a) Motor control
b) Sensation from opposite side of body
c) Vision
d) Hearing
Explanation: The ventral posterior nucleus relays sensory input from the contralateral body via the spinothalamic and medial lemniscal systems. Lesions cause loss of pain, temperature, touch, and proprioception on the opposite side. Answer: (b) Sensation from opposite side of body.
6. A 52-year-old man presents with burning pain following a thalamic infarct. This condition is known as:
a) Thalamic syndrome
b) Brown-Séquard syndrome
c) Guillain–Barré syndrome
d) Horner’s syndrome
Explanation: Thalamic syndrome (Dejerine–Roussy syndrome) occurs after thalamic infarction. It manifests as contralateral burning pain, sensory loss, and hyperesthesia due to disrupted sensory relay from thalamus to cortex. Answer: (a) Thalamic syndrome.
7. Which sensory modality is least affected by thalamic lesions?
a) Crude touch
b) Pain
c) Vision
d) Smell
Explanation: Olfactory sensation bypasses the thalamus and directly reaches the cerebral cortex via the olfactory tract. Thus, smell remains unaffected in thalamic lesions, unlike other sensory modalities that relay through thalamic nuclei. Answer: (d) Smell.
8. A 40-year-old woman has loss of proprioception but intact pain and temperature. The likely site of lesion is:
a) Thalamus
b) Medial lemniscus
c) Spinothalamic tract
d) Dorsal column
Explanation: Loss of proprioception with preserved pain and temperature indicates a dorsal column lesion. The dorsal column–medial lemniscus pathway carries proprioceptive information, while the spinothalamic tract conveys pain and temperature. Answer: (d) Dorsal column.
9. Which thalamic nucleus is connected with pain perception?
a) Ventral posterior nucleus
b) Lateral geniculate body
c) Medial geniculate body
d) Pulvinar
Explanation: The ventral posterior nucleus processes pain and temperature inputs received from the spinothalamic tract. It relays these sensations to the primary somatosensory cortex for interpretation. Answer: (a) Ventral posterior nucleus.
10. A 60-year-old stroke patient reports increased sensitivity to mild touch on the opposite side. This exaggerated pain response is due to:
a) Thalamic hyperactivity
b) Brainstem ischemia
c) Spinal cord lesion
d) Peripheral neuropathy
Explanation: Post-thalamic stroke, neurons in the thalamus become hyperexcitable, leading to exaggerated pain or allodynia. This abnormal processing of sensory signals, termed thalamic pain syndrome, causes severe discomfort from even light stimuli. Answer: (a) Thalamic hyperactivity.
```
Chapter: Nervous System; Topic: Neurotrophic Factors; Subtopic: Brain-Derived Neurotrophic Factor (BDNF) and its Receptors
Keyword Definitions:
• BDNF: Brain-Derived Neurotrophic Factor, a protein promoting neuron survival and synaptic plasticity.
• TrK Receptors: Tyrosine kinase receptors mediating neurotrophin signaling.
• Neurotrophins: Family of growth factors essential for nervous system development.
• Synaptic Plasticity: Ability of synapses to strengthen or weaken over time.
• Neuronal Survival: Maintenance of neurons by growth factors like BDNF.
• Neurodegeneration: Progressive loss of neuron function and structure.
• TrK-B: Specific receptor for BDNF mediating neuronal growth and differentiation.
Lead Question – 2014
Receptor for BDNF?
a) TrK-A
b) TrK-B
c) TrK-C
d) None
Explanation: Brain-Derived Neurotrophic Factor (BDNF) primarily binds to TrK-B receptors, initiating intracellular signaling that promotes neuronal survival, differentiation, and synaptic plasticity. Unlike NGF that binds to TrK-A or NT-3 that binds to TrK-C, BDNF is specific for TrK-B. This receptor plays a vital role in learning, memory, and neuroprotection in the central nervous system. Answer: (b) TrK-B.
1. Which of the following neurotrophins primarily binds to TrK-A receptor?
a) Nerve Growth Factor (NGF)
b) BDNF
c) Neurotrophin-3
d) Neurotrophin-4
Explanation: NGF (Nerve Growth Factor) binds specifically to TrK-A receptor. This interaction promotes neuronal differentiation and maintenance of sympathetic and sensory neurons. TrK-A activation triggers MAP kinase and PI3K pathways, enhancing cell survival and neurite growth. BDNF and NT-3 act on TrK-B and TrK-C respectively. Answer: (a) NGF.
2. BDNF plays a crucial role in which of the following processes?
a) Liver regeneration
b) Bone mineralization
c) Synaptic plasticity
d) Hormone synthesis
Explanation: BDNF is essential for synaptic plasticity—the ability of synapses to adapt during learning and memory formation. It strengthens neuronal connections by enhancing neurotransmitter release and dendritic growth. This function underlies cognitive functions such as memory consolidation and learning adaptability. Answer: (c) Synaptic plasticity.
3. TrK-C receptor mainly binds with which neurotrophin?
a) NGF
b) NT-3
c) NT-4
d) BDNF
Explanation: TrK-C receptor has high affinity for Neurotrophin-3 (NT-3), which regulates neuronal survival and axonal growth in both central and peripheral nervous systems. It also cross-reacts weakly with TrK-A and TrK-B. Activation of TrK-C triggers signaling cascades crucial for neuron differentiation. Answer: (b) NT-3.
4. A 45-year-old patient with depression shows decreased hippocampal volume. Which factor deficiency is most likely responsible?
a) NGF
b) BDNF
c) Dopamine
d) ACTH
Explanation: Decreased BDNF levels are linked with depression and reduced hippocampal neurogenesis. BDNF supports survival and growth of neurons in the hippocampus, enhancing memory and emotional regulation. Antidepressant therapy increases BDNF expression, promoting neuroplasticity and recovery. Answer: (b) BDNF.
5. Which receptor mediates the effect of Nerve Growth Factor (NGF)?
a) TrK-A
b) TrK-B
c) TrK-C
d) TrK-D
Explanation: NGF exerts its biological effects mainly through TrK-A receptor. Activation of TrK-A promotes neuronal differentiation, maintenance, and survival in sympathetic and sensory neurons. This receptor is vital for pain perception and autonomic nervous system function. Answer: (a) TrK-A.
6. A patient recovering from stroke demonstrates improved motor recovery due to increased neurotrophin release. Which receptor is primarily involved?
a) TrK-A
b) TrK-B
c) TrK-C
d) p75 receptor
Explanation: TrK-B receptor mediates BDNF’s neuroprotective effects after stroke, promoting neuronal repair and synaptic remodeling. Enhanced BDNF–TrK-B signaling contributes to neurogenesis and recovery of motor functions through activation of intracellular cascades like MAPK and PI3K pathways. Answer: (b) TrK-B.
7. Which neurotrophin deficiency is most commonly linked to Alzheimer’s disease?
a) NGF
b) BDNF
c) NT-3
d) NT-4
Explanation: BDNF deficiency contributes to synaptic loss and neuronal degeneration in Alzheimer’s disease. Reduced BDNF-TrK-B signaling impairs synaptic plasticity, leading to cognitive decline. Restoring BDNF levels improves neuronal survival and cognitive function, highlighting its therapeutic potential in neurodegenerative disorders. Answer: (b) BDNF.
8. TrK-B receptor activation primarily stimulates which intracellular signaling pathway?
a) JAK-STAT
b) MAPK and PI3K-Akt
c) TGF-beta
d) Wnt-beta catenin
Explanation: TrK-B receptor activation triggers MAPK and PI3K-Akt pathways, promoting cell survival, synaptic plasticity, and neuronal differentiation. These cascades mediate neurotrophic effects of BDNF, ensuring neuronal health and adaptive brain function. Answer: (b) MAPK and PI3K-Akt.
9. Which of the following receptors binds both BDNF and NT-4?
a) TrK-A
b) TrK-B
c) TrK-C
d) p75 receptor
Explanation: Both BDNF and NT-4 share the same high-affinity receptor, TrK-B. Binding activates neuroprotective signaling pathways that enhance synaptic function and plasticity. These pathways play key roles in development and maintenance of the nervous system. Answer: (b) TrK-B.
10. A 32-year-old woman with long-term antidepressant use shows increased hippocampal BDNF expression. Which mechanism best explains this?
a) Reduced cortisol binding
b) Enhanced BDNF-TrK-B signaling
c) Decreased dopamine reuptake
d) Blocked serotonin degradation
Explanation: Chronic antidepressant use enhances BDNF-TrK-B signaling, improving neurogenesis and synaptic remodeling in the hippocampus. This molecular adaptation restores neural circuitry, alleviating depressive symptoms and improving cognitive performance. Answer: (b) Enhanced BDNF-TrK-B signaling.
Chapter: Nervous System Physiology; Topic: Autonomic Nervous System; Subtopic: Neurotransmitters of Sympathetic and Parasympathetic Systems
Keyword Definitions:
Noradrenaline (Norepinephrine): A catecholamine neurotransmitter released mainly by postganglionic sympathetic fibers, except in sweat glands.
Postganglionic Fibers: Neurons extending from autonomic ganglia to target organs.
Sympathetic Nervous System: Division of the autonomic nervous system responsible for “fight or flight” responses.
Parasympathetic Nervous System: Division promoting “rest and digest” functions using acetylcholine as neurotransmitter.
Lead Question - 2014
Noradrenaline is major neurotransmitter in?
a) Postganglionic parasympathetic fibres
b) Postganglionic sympathetic fibres except in sweat glands
c) Autonomic ganglia
d) Preganglionic autonomic fibres
Explanation: Noradrenaline is the principal neurotransmitter released by postganglionic sympathetic fibers, except those innervating sweat glands, which release acetylcholine. It acts via adrenergic receptors to increase heart rate, blood pressure, and glucose availability during stress. Hence, the correct answer is b) Postganglionic sympathetic fibres except in sweat glands.
1) Which neurotransmitter is released by preganglionic sympathetic neurons?
a) Dopamine
b) Acetylcholine
c) Noradrenaline
d) Serotonin
Explanation: Preganglionic neurons of both sympathetic and parasympathetic systems release acetylcholine, which acts on nicotinic receptors in autonomic ganglia. This cholinergic signaling ensures transmission before the effector organ response. Hence, the correct answer is b) Acetylcholine.
2) Sweat glands are unique because their postganglionic sympathetic fibers release:
a) Dopamine
b) Acetylcholine
c) Noradrenaline
d) Serotonin
Explanation: Sweat glands are an exception in the sympathetic system as their postganglionic fibers release acetylcholine rather than noradrenaline. This neurotransmitter acts on muscarinic receptors to promote sweating, crucial for thermoregulation. Therefore, the correct answer is b) Acetylcholine.
3) Which enzyme converts dopamine to noradrenaline in sympathetic nerve endings?
a) Dopamine decarboxylase
b) Monoamine oxidase
c) Dopamine β-hydroxylase
d) Tyrosine hydroxylase
Explanation: Dopamine β-hydroxylase catalyzes the conversion of dopamine into noradrenaline within vesicles of sympathetic neurons. This enzyme is essential for catecholamine synthesis and proper sympathetic neurotransmission. Thus, the correct answer is c) Dopamine β-hydroxylase.
4) Which receptor subtype does noradrenaline primarily act on in the heart?
a) α1 receptors
b) β1 receptors
c) β2 receptors
d) α2 receptors
Explanation: Noradrenaline predominantly acts on β1 adrenergic receptors in the heart, increasing heart rate, contractility, and cardiac output. This sympathetic activation supports the body’s fight-or-flight response. The correct answer is b) β1 receptors.
5) Which neurotransmitter is deficient in a patient with postganglionic sympathetic denervation?
a) Acetylcholine
b) Noradrenaline
c) Dopamine
d) Glutamate
Explanation: Postganglionic sympathetic denervation leads to a deficiency of noradrenaline, resulting in loss of sympathetic tone, hypotension, and impaired stress response. This condition affects adrenergic receptor-mediated activities. Therefore, the correct answer is b) Noradrenaline.
6) A patient with pheochromocytoma will show elevated levels of:
a) Noradrenaline
b) Acetylcholine
c) GABA
d) Serotonin
Explanation: Pheochromocytoma, a tumor of the adrenal medulla, causes excessive secretion of catecholamines, mainly noradrenaline and adrenaline. It results in hypertension, palpitations, and sweating. Elevated plasma or urinary metanephrines confirm the diagnosis. Thus, the correct answer is a) Noradrenaline.
7) Which neurotransmitter acts on nicotinic receptors in autonomic ganglia?
a) Acetylcholine
b) Noradrenaline
c) Adrenaline
d) Dopamine
Explanation: Acetylcholine is the neurotransmitter acting on nicotinic receptors within autonomic ganglia, facilitating synaptic transmission between preganglionic and postganglionic neurons. This cholinergic signal is crucial for both sympathetic and parasympathetic systems. The correct answer is a) Acetylcholine.
8) A patient treated with reserpine shows decreased sympathetic activity because it:
a) Inhibits acetylcholine release
b) Depletes noradrenaline from vesicles
c) Blocks adrenergic receptors
d) Enhances dopamine synthesis
Explanation: Reserpine inhibits vesicular monoamine transporter (VMAT), preventing storage of noradrenaline in synaptic vesicles. This depletion reduces sympathetic transmission and lowers blood pressure. The correct answer is b) Depletes noradrenaline from vesicles.
9) Which of the following drugs increases noradrenaline levels by inhibiting its reuptake?
a) Cocaine
b) Atropine
c) Clonidine
d) Propranolol
Explanation: Cocaine inhibits the reuptake of noradrenaline and dopamine at synaptic clefts, leading to increased sympathetic stimulation, euphoria, and hypertension. This pharmacologic effect explains cocaine’s sympathomimetic actions. Therefore, the correct answer is a) Cocaine.
10) Which of the following symptoms is NOT mediated by noradrenaline release?
a) Increased heart rate
b) Pupil dilation
c) Bronchodilation
d) Salivation
Explanation: Noradrenaline mediates sympathetic effects like increased heart rate and pupil dilation but does not cause salivation, which is a parasympathetic (acetylcholine-mediated) response. Hence, the correct answer is d) Salivation.
Chapter: Neurophysiology; Topic: Somatosensory System; Subtopic: Cortical Representation (Sensory Homunculus)
Keyword Definitions:
Somatosensory cortex: The region of the cerebral cortex responsible for processing sensory input from various parts of the body.
Sensory homunculus: A cortical map showing how much of the somatosensory cortex is devoted to sensations from each body region.
Proprioception: The sense of position and movement of body parts.
Cortical representation: The area of the cerebral cortex that corresponds to sensory input from specific body parts, proportional to sensitivity rather than size.
Lead Question - 2014
Which of the following has small representation in somatosensory area of cerebral cortex?
a) Lips
b) Thumb/fingers
c) Tongue
d) Trunk
Explanation: The somatosensory cortex (Brodmann’s areas 3, 1, and 2) has disproportionate representation of different body parts, depending on their sensory precision. Highly sensitive areas like lips, tongue, and fingers occupy large cortical areas, while less sensitive parts like the trunk and thigh have small representation. Hence, the correct answer is (d) Trunk.
1) Which lobe of the brain contains the primary somatosensory cortex?
a) Frontal lobe
b) Parietal lobe
c) Temporal lobe
d) Occipital lobe
Explanation: The primary somatosensory cortex lies in the postcentral gyrus of the parietal lobe, corresponding to Brodmann’s areas 3, 1, and 2. It processes tactile, proprioceptive, and nociceptive sensations from the body surface and deeper tissues. The correct answer is (b) Parietal lobe.
2) A 45-year-old patient with a lesion in the right postcentral gyrus may show loss of sensation in:
a) Right hand
b) Left hand
c) Both hands
d) Lower limbs only
Explanation: The somatosensory cortex receives contralateral sensory input; hence, a lesion in the right postcentral gyrus results in sensory deficits on the left side of the body, typically affecting the hand and face depending on lesion location. Therefore, the correct answer is (b) Left hand.
3) Which area of the body has the largest cortical representation in the sensory homunculus?
a) Trunk
b) Lips
c) Forearm
d) Thigh
Explanation: The sensory homunculus reflects density of sensory receptors rather than body size. The lips and tongue, rich in tactile receptors, occupy large cortical representation, allowing fine discrimination. In contrast, areas like the trunk and thigh have limited sensory input. Hence, the correct answer is (b) Lips.
4) Which Brodmann areas correspond to the primary somatosensory cortex?
a) 17
b) 4
c) 3, 1, 2
d) 22
Explanation: Brodmann’s areas 3, 1, and 2, located on the postcentral gyrus, constitute the primary somatosensory cortex. These areas receive projections from the thalamus and process tactile, pain, temperature, and proprioceptive sensations. Hence, the correct answer is (c) 3, 1, 2.
5) Which of the following body parts has bilateral cortical representation for sensation?
a) Face
b) Lips
c) Genitalia
d) Hand
Explanation: While most somatosensory input is contralateral, certain midline structures like the genitalia, pharynx, and parts of the oral cavity have bilateral cortical representation to preserve sensory awareness on both sides. The correct answer is (c) Genitalia.
6) A 50-year-old man with a parietal lobe stroke is unable to recognize objects by touch (astereognosis). The lesion likely involves:
a) Primary motor cortex
b) Secondary somatosensory area
c) Visual cortex
d) Cerebellum
Explanation: Astereognosis—loss of tactile object recognition—occurs when the secondary somatosensory cortex (area 5 and 7) in the parietal lobe is affected. Primary sensory input remains intact, but higher processing for object interpretation is lost. Hence, the correct answer is (b) Secondary somatosensory area.
7) The thalamic nucleus that projects to the primary somatosensory cortex is:
a) Ventral posterior nucleus
b) Medial geniculate body
c) Lateral geniculate body
d) Ventral anterior nucleus
Explanation: The ventral posterior nucleus of the thalamus transmits somatosensory information to Brodmann’s areas 3, 1, and 2. The ventral posterior lateral (VPL) handles body input, and ventral posterior medial (VPM) handles face input. Thus, the correct answer is (a) Ventral posterior nucleus.
8) Lesion of the right parietal association cortex causes which clinical sign?
a) Aphasia
b) Left-sided neglect
c) Right-sided neglect
d) Agraphia
Explanation: Damage to the right parietal association area causes left-sided spatial neglect due to impaired perception of the contralateral environment. Patients may ignore the left side of their body or surroundings. The correct answer is (b) Left-sided neglect.
9) A patient cannot localize the source of pain after cortical injury. This indicates involvement of:
a) Thalamus
b) Primary somatosensory cortex
c) Cerebellum
d) Hippocampus
Explanation: Localization of pain depends on cortical processing in the primary somatosensory area. A lesion here impairs the ability to identify and localize painful stimuli, though the pain perception itself remains intact. Thus, the correct answer is (b) Primary somatosensory cortex.
10) In the sensory homunculus, which body part lies most medially on the cortex?
a) Face
b) Hand
c) Leg
d) Tongue
Explanation: In the cortical sensory map, the leg and foot are represented medially on the paracentral lobule, while the face and hand are located more laterally. This somatotopic organization corresponds to the body layout on the sensory cortex. The correct answer is (c) Leg.
Chapter: Neurophysiology; Topic: Synaptic Transmission; Subtopic: Post-Tetanic Potentiation
Keyword Definitions:
Post-tetanic potentiation: A short-term increase in synaptic strength following high-frequency stimulation, due to residual calcium accumulation in the presynaptic terminal.
Synaptic transmission: Process by which neurons communicate across synapses through neurotransmitter release.
Calcium ions (Ca++): Essential ions that trigger neurotransmitter vesicle fusion with the presynaptic membrane.
Neurotransmitter release: The exocytosis of chemical messengers like acetylcholine or glutamate from nerve terminals.
Lead Question - 2014
Post-tetanic potentiation is due to -
a) Hyperpolarization of muscle fibres
b) Rapid K+ efflux
c) Increased availability of Ca++
d) Rapid Na+ influx
Explanation: Post-tetanic potentiation occurs when repetitive stimulation leads to an increased influx of calcium ions into the presynaptic terminal. The residual calcium enhances neurotransmitter release during subsequent stimuli, resulting in stronger postsynaptic responses. This mechanism is important in short-term memory and synaptic plasticity. Hence, the correct answer is (c) Increased availability of Ca++.
1) The primary site of calcium accumulation during post-tetanic potentiation is:
a) Postsynaptic membrane
b) Presynaptic terminal
c) Synaptic cleft
d) Dendritic spines
Explanation: Post-tetanic potentiation is caused by accumulation of calcium within the presynaptic terminal after high-frequency stimulation. This calcium facilitates additional neurotransmitter release with subsequent impulses, enhancing synaptic transmission transiently. The postsynaptic response increases due to more transmitter molecules binding to receptors. Thus, the correct answer is (b) Presynaptic terminal.
2) Which of the following best describes the time course of post-tetanic potentiation?
a) Lasts milliseconds
b) Lasts seconds to minutes
c) Lasts hours
d) Permanent
Explanation: Post-tetanic potentiation typically lasts for several seconds to a few minutes after the tetanic stimulation ends. It reflects short-term synaptic plasticity rather than long-term potentiation, which lasts hours or days. The transient increase in neurotransmitter release fades as presynaptic calcium levels return to normal. The correct answer is (b) Lasts seconds to minutes.
3) A patient with Lambert-Eaton myasthenic syndrome shows decreased post-tetanic potentiation. This is due to:
a) Defect in acetylcholine receptors
b) Defect in presynaptic calcium channels
c) Excessive acetylcholine degradation
d) Overactive sodium channels
Explanation: Lambert-Eaton myasthenic syndrome (LEMS) involves autoantibodies against presynaptic voltage-gated calcium channels, impairing calcium influx and thus neurotransmitter release. This reduces post-tetanic potentiation. Unlike myasthenia gravis, LEMS primarily affects presynaptic mechanisms. Hence, the correct answer is (b) Defect in presynaptic calcium channels.
4) During post-tetanic potentiation, enhanced neurotransmitter release is primarily due to:
a) Increased vesicle number
b) Increased presynaptic calcium
c) Increased postsynaptic receptors
d) Reduced potassium conductance
Explanation: The accumulation of calcium in the presynaptic terminal increases the probability of synaptic vesicle fusion with the presynaptic membrane, thereby increasing neurotransmitter release. This transient enhancement of synaptic efficacy defines post-tetanic potentiation. Hence, the correct answer is (b) Increased presynaptic calcium.
5) Which ion’s accumulation is responsible for post-tetanic potentiation?
a) Na+
b) K+
c) Cl−
d) Ca++
Explanation: Calcium ions (Ca++) accumulate in the presynaptic terminal during repeated stimulation, enhancing neurotransmitter release. This transient elevation causes stronger postsynaptic potentials upon subsequent impulses, which is the hallmark of post-tetanic potentiation. The correct answer is (d) Ca++.
6) A 42-year-old man with chronic fatigue exhibits improved muscle contraction after repetitive nerve stimulation. This is characteristic of:
a) Myasthenia gravis
b) Lambert-Eaton syndrome
c) Botulism
d) Duchenne muscular dystrophy
Explanation: In Lambert-Eaton syndrome, repetitive nerve stimulation transiently improves muscle strength due to facilitation of calcium entry and enhanced acetylcholine release—reflecting post-tetanic potentiation. In contrast, myasthenia gravis shows fatigability without facilitation. Hence, the correct answer is (b) Lambert-Eaton syndrome.
7) Post-tetanic potentiation is an example of:
a) Long-term potentiation
b) Short-term synaptic plasticity
c) Synaptic depression
d) Neural adaptation
Explanation: Post-tetanic potentiation represents short-term synaptic plasticity where transient increases in neurotransmitter release enhance synaptic strength for seconds to minutes. It differs from long-term potentiation (LTP), which involves gene expression and structural changes. The correct answer is (b) Short-term synaptic plasticity.
8) Which process terminates post-tetanic potentiation?
a) Calcium sequestration by mitochondria
b) Increased neurotransmitter release
c) Sodium influx
d) Potassium efflux
Explanation: The decline of post-tetanic potentiation occurs as intracellular calcium is pumped back into the endoplasmic reticulum or sequestered by mitochondria, restoring baseline neurotransmitter release rates. The temporary nature of calcium elevation explains why potentiation is short-lived. The correct answer is (a) Calcium sequestration by mitochondria.
9) Which synapse commonly exhibits post-tetanic potentiation?
a) Neuromuscular junction
b) Retinal synapses
c) Cerebellar synapses
d) Autonomic ganglia
Explanation: Post-tetanic potentiation is well-documented in cerebellar and hippocampal synapses, where repetitive activation strengthens transmission for seconds to minutes. This phenomenon underlies short-term learning and coordination adjustments. Hence, the correct answer is (c) Cerebellar synapses.
10) A patient’s EMG shows enhanced amplitude after rapid nerve stimulation. This finding suggests:
a) Myasthenia gravis
b) Lambert-Eaton myasthenic syndrome
c) Multiple sclerosis
d) Motor neuron disease
Explanation: Enhanced EMG amplitude after rapid stimulation is a hallmark of post-tetanic potentiation, typical in Lambert-Eaton syndrome. It reflects increased calcium entry into presynaptic terminals during repetitive activity, improving acetylcholine release and muscle contraction. The correct answer is (b) Lambert-Eaton myasthenic syndrome.
Chapter: Neurophysiology; Topic: Cerebellum; Subtopic: Cerebellar Neuronal Connections
Keyword Definitions:
Cerebellum: Part of the hindbrain that coordinates voluntary movements, posture, and balance.
Climbing fibres: Afferents from the inferior olivary nucleus that synapse directly with Purkinje cells.
Mossy fibres: Afferents from the spinal cord, vestibular nuclei, and pontine nuclei that excite granule cells.
Purkinje cells: Large inhibitory neurons of the cerebellar cortex projecting to deep cerebellar nuclei.
Lead Question - 2014
True about cerebellar neuronal connections?
a) Climbing fibres from inferior olivary nucleus
b) Mossy fibres from inferior olivary nucleus
c) Climbing fibres are inhibitory to Purkinje cells
d) Mossy fibres are inhibitory to Purkinje cells
Explanation: Climbing fibres originate exclusively from the inferior olivary nucleus and form powerful excitatory synapses on Purkinje cells, each Purkinje cell receiving input from a single climbing fibre. Mossy fibres arise from multiple sources such as the spinal cord and pontine nuclei and excite granule cells indirectly. Hence, the correct answer is (a) Climbing fibres from inferior olivary nucleus.
1) The neurotransmitter released by Purkinje cells is:
a) GABA
b) Glutamate
c) Dopamine
d) Serotonin
Explanation: Purkinje cells are inhibitory neurons that release GABA (gamma-aminobutyric acid) onto deep cerebellar nuclei. They play a crucial role in modulating cerebellar output, ensuring smooth and coordinated motor activity. Their inhibitory nature helps prevent overactivation of motor pathways. The correct answer is (a) GABA.
2) Which structure acts as the major output of the cerebellum?
a) Purkinje cells
b) Deep cerebellar nuclei
c) Inferior olivary nucleus
d) Granule cells
Explanation: The cerebellum’s output is mediated through the deep cerebellar nuclei, which receive inhibitory input from Purkinje cells and send excitatory signals to motor centers in the brainstem and thalamus. These nuclei are dentate, emboliform, globose, and fastigial. Hence, the correct answer is (b) Deep cerebellar nuclei.
3) Lesion of the cerebellar hemisphere leads to:
a) Intention tremor
b) Resting tremor
c) Rigidity
d) Flaccidity
Explanation: Lesions of the cerebellar hemisphere affect coordination on the ipsilateral side of the body, causing intention tremor, dysmetria, and dysdiadochokinesia. These are hallmark signs of cerebellar dysfunction distinct from extrapyramidal disorders that cause resting tremor or rigidity. The correct answer is (a) Intention tremor.
4) Mossy fibres synapse with which cells?
a) Purkinje cells
b) Golgi cells
c) Granule cells
d) Stellate cells
Explanation: Mossy fibres synapse on granule cells in the cerebellar cortex, which then project via parallel fibres to Purkinje cells. This indirect pathway modulates cerebellar output and integrates sensory and motor information. Therefore, the correct answer is (c) Granule cells.
5) Climbing fibre activity produces:
a) Simple spikes
b) Complex spikes
c) EPSPs
d) IPSPs
Explanation: Climbing fibres from the inferior olivary nucleus generate “complex spikes” in Purkinje cells due to multiple excitatory synapses. These spikes differ from the “simple spikes” produced by mossy fibre–granule cell pathways. Complex spikes are essential for motor learning. Hence, the correct answer is (b) Complex spikes.
6) A 32-year-old patient presents with loss of coordination and inability to perform rapid alternating movements. Which cerebellar area is most likely affected?
a) Vestibulocerebellum
b) Spinocerebellum
c) Cerebrocerebellum
d) Flocculonodular lobe
Explanation: The cerebrocerebellum, mainly the lateral hemispheres, coordinates skilled and sequential movements. Its lesion leads to dysdiadochokinesia and loss of fine motor control. Vestibulocerebellum deals with balance, and spinocerebellum with posture. The correct answer is (c) Cerebrocerebellum.
7) Which tract conveys proprioceptive information to the cerebellum?
a) Corticospinal tract
b) Spinothalamic tract
c) Spinocerebellar tract
d) Rubrospinal tract
Explanation: The spinocerebellar tracts (dorsal and ventral) transmit unconscious proprioceptive information to the cerebellum for coordination and posture control. These tracts enable real-time correction of movement errors. Therefore, the correct answer is (c) Spinocerebellar tract.
8) In cerebellar lesions, which side of the body is affected?
a) Contralateral
b) Bilateral
c) Ipsilateral
d) Alternate
Explanation: Each cerebellar hemisphere controls coordination of movements on the same (ipsilateral) side of the body due to double crossing of pathways. Therefore, cerebellar lesions produce ipsilateral deficits. The correct answer is (c) Ipsilateral.
9) In a patient with cerebellar damage, eye movements are erratic. Which area is most likely involved?
a) Vermis
b) Flocculonodular lobe
c) Dentate nucleus
d) Fastigial nucleus
Explanation: The flocculonodular lobe, part of the vestibulocerebellum, maintains equilibrium and coordinates eye movements via vestibular nuclei. Lesions cause nystagmus and postural instability. Hence, the correct answer is (b) Flocculonodular lobe.
10) A patient exhibits ataxia after chronic alcoholism. Which cerebellar region is typically damaged?
a) Vermis
b) Lateral hemisphere
c) Flocculonodular lobe
d) Dentate nucleus
Explanation: Chronic alcoholism often causes degeneration of the cerebellar vermis, leading to truncal ataxia and gait instability. The vermis regulates posture and balance. Therefore, the correct answer is (a) Vermis.
Chapter: Physiology Topic: Nervous System; Subtopic: Pain Pathways and Nerve Fiber Types
Keyword Definitions:
Pain: An unpleasant sensory and emotional experience associated with actual or potential tissue damage.
Nociceptors: Specialized sensory receptors that detect damaging or potentially damaging stimuli.
Aδ fibers: Thinly myelinated fibers transmitting sharp, localized “first pain.”
C fibers: Unmyelinated fibers transmitting dull, burning “second pain.”
Myelination: Fatty insulation of axons increasing conduction velocity.
Dorsal horn: Region in spinal cord receiving sensory input from nociceptors.
Spinothalamic tract: Major ascending pathway conveying pain and temperature sensations to the brain.
Lead Question – 2014
Sharp pain is transmitted by which type of fibres?
a) Aα
b) Aβ
c) Aδ
d) C
Explanation: The correct answer is Aδ fibers. These are thinly myelinated nerve fibers that transmit sharp, localized, and fast pain sensations. They conduct impulses at a velocity of 5–30 m/s. The pain they mediate is often called “first pain.” In contrast, unmyelinated C fibers transmit dull, burning, and poorly localized “second pain” at slower speeds.
1) Which fibers transmit slow, burning pain?
a) Aα
b) Aβ
c) Aδ
d) C
Explanation: The correct answer is C fibers. These are unmyelinated, small-diameter fibers that conduct impulses at 0.4–2 m/s. They carry dull, throbbing, and persistent pain known as “second pain.” Their slow conduction allows prolonged perception of tissue injury and contributes to protective behavior and healing awareness.
2) Which part of the brain perceives pain intensity and localization?
a) Thalamus
b) Cerebellum
c) Hypothalamus
d) Hippocampus
Explanation: The correct answer is Thalamus. The thalamus acts as a relay center for sensory impulses, including pain, directing them to the somatosensory cortex. It helps localize and interpret pain intensity before emotional components are processed in the limbic system.
3) Which neurotransmitter is primarily involved in transmission of pain in the spinal cord?
a) Dopamine
b) Glutamate
c) Acetylcholine
d) GABA
Explanation: The correct answer is Glutamate. It is the main excitatory neurotransmitter in nociceptive pathways, particularly in Aδ fiber synapses within the dorsal horn. Substance P is also involved, particularly in chronic pain transmission via C fibers and secondary sensory neurons.
4) Which fibers are responsible for tactile sensations like touch and pressure?
a) Aα
b) Aβ
c) Aδ
d) C
Explanation: The correct answer is Aβ fibers. These are large, heavily myelinated fibers that conduct sensory impulses rapidly. They are primarily responsible for transmitting non-noxious stimuli like touch, pressure, and vibration, helping differentiate pain from normal tactile sensations.
5) Which tract transmits pain and temperature sensations to the brain?
a) Dorsal column
b) Corticospinal tract
c) Spinothalamic tract
d) Vestibulospinal tract
Explanation: The correct answer is Spinothalamic tract. This ascending pathway originates in the dorsal horn of the spinal cord and carries pain and temperature sensations to the thalamus and somatosensory cortex, enabling conscious pain perception and localization.
6) A patient reports sharp pain immediately after a needle prick. Which fibers are involved?
a) Aδ fibers
b) C fibers
c) Aβ fibers
d) B fibers
Explanation: The correct answer is Aδ fibers. These fibers rapidly transmit the initial sharp pain from mechanical or thermal stimuli. Their conduction speed allows quick withdrawal reflexes, helping prevent further tissue damage before slow C fiber pain begins.
7) A patient with peripheral neuropathy loses sharp pain sensation but retains dull pain. Which fibers are affected?
a) C fibers
b) Aδ fibers
c) Aβ fibers
d) Aα fibers
Explanation: The correct answer is Aδ fibers. Damage to thinly myelinated Aδ fibers causes loss of sharp pain and temperature discrimination, while unmyelinated C fibers still transmit burning or aching sensations, preserving slow pain response.
8) Which of the following is true regarding pain transmission by Aδ fibers?
a) They are unmyelinated
b) They transmit dull pain
c) They conduct impulses rapidly
d) They mediate visceral pain
Explanation: The correct answer is They conduct impulses rapidly. Aδ fibers are thinly myelinated, allowing faster conduction (5–30 m/s) compared to unmyelinated C fibers. They are responsible for acute, localized pain sensations from somatic structures like skin and muscles.
9) A burn patient reports persistent throbbing pain hours after injury. Which fibers are responsible?
a) Aδ fibers
b) C fibers
c) Aβ fibers
d) Aα fibers
Explanation: The correct answer is C fibers. They mediate slow, prolonged pain that persists after initial injury. This continuous pain aids in wound protection. C fibers are unmyelinated, slow-conducting, and important in chronic pain and inflammatory responses.
10) A patient with spinal cord injury loses pain and temperature sensation but retains touch and vibration. Which tract is damaged?
a) Dorsal column
b) Spinothalamic tract
c) Corticospinal tract
d) Vestibulospinal tract
Explanation: The correct answer is Spinothalamic tract. It carries pain and temperature sensations from the opposite side of the body. Its damage results in loss of these sensations below the lesion, while touch and vibration (carried by dorsal columns) remain intact.
Chapter: Physiology; Topic: Hypothalamic Regulation of Appetite; Subtopic: Neuroendocrine Control of Feeding Behavior
Keyword Definitions:
Appetite: Desire for food regulated by hypothalamic centers, influenced by hormones and neurotransmitters.
Arcuate nucleus: A hypothalamic region that contains neurons regulating hunger and satiety.
NPY (Neuropeptide Y): A potent appetite stimulant found in the hypothalamus.
AGRP (Agouti-related peptide): A neuropeptide that increases food intake by antagonizing melanocortin receptors.
CART (Cocaine- and amphetamine-regulated transcript): A peptide that suppresses appetite.
α-MSH (Alpha-melanocyte-stimulating hormone): A peptide that decreases appetite via MC4 receptors.
Insulin: Hormone that decreases appetite by acting on hypothalamic centers.
Lead Question – 2014
Which of the following increases appetite?
a) CART
b) α - MSH
c) AGPP
d) Insulin
Explanation: The correct answer is AGRP (Agouti-related peptide). AGRP stimulates appetite by inhibiting melanocortin receptors (MC3 and MC4) in the hypothalamus, promoting feeding behavior. It acts synergistically with Neuropeptide Y, both secreted from arcuate nucleus neurons. CART and α-MSH suppress appetite, while insulin also inhibits food intake via satiety signaling pathways.
1) Which hormone secreted from the stomach stimulates appetite?
a) Ghrelin
b) Leptin
c) Insulin
d) CCK
Explanation: The answer is Ghrelin. It is a peptide hormone produced by the stomach that increases appetite by acting on hypothalamic neurons, especially the NPY/AGRP group. Ghrelin levels rise before meals and fall afterward, signaling hunger to maintain energy balance through hypothalamic stimulation of feeding behavior.
2) Which center of the hypothalamus is known as the hunger center?
a) Ventromedial nucleus
b) Lateral hypothalamic area
c) Arcuate nucleus
d) Paraventricular nucleus
Explanation: The correct answer is Lateral hypothalamic area. It contains neurons that stimulate feeding when activated. Damage to this region leads to anorexia. It integrates peripheral signals like ghrelin and glucose levels, modulating neuronal activity to promote hunger sensations and maintain energy homeostasis in the body.
3) Which hypothalamic nucleus acts as the satiety center?
a) Lateral hypothalamus
b) Ventromedial nucleus
c) Arcuate nucleus
d) Posterior hypothalamus
Explanation: The correct answer is Ventromedial nucleus. It inhibits feeding behavior when stimulated. Lesions in this area cause hyperphagia and obesity. It receives leptin and insulin signals, reducing food intake by suppressing hunger-promoting neurons in the lateral hypothalamus and maintaining energy balance.
4) Which neurotransmitter promotes satiety and reduces appetite?
a) Dopamine
b) Serotonin
c) Norepinephrine
d) Glutamate
Explanation: The correct answer is Serotonin. It acts on hypothalamic receptors to reduce appetite and food intake, particularly carbohydrates. Serotonin increases satiety through activation of POMC neurons and inhibition of NPY/AGRP neurons. Drugs enhancing serotonin activity can help reduce appetite and support weight management.
5) Which of the following is an anorexigenic hormone?
a) Ghrelin
b) Neuropeptide Y
c) Leptin
d) AGRP
Explanation: The correct answer is Leptin. Secreted by adipose tissue, it signals energy sufficiency to the hypothalamus, reducing appetite. It inhibits NPY and AGRP neurons while stimulating POMC/CART neurons, promoting satiety. Leptin deficiency or resistance is associated with obesity and altered energy homeostasis in humans.
6) A 45-year-old obese patient shows leptin resistance. Which mechanism explains his persistent hunger?
a) Increased CART activity
b) Impaired hypothalamic leptin signaling
c) Enhanced α-MSH sensitivity
d) Decreased ghrelin secretion
Explanation: The answer is Impaired hypothalamic leptin signaling. Leptin resistance prevents satiety signaling, causing continued food intake despite energy sufficiency. It leads to altered neuronal responses in the arcuate nucleus, persistent hunger, and weight gain due to failure of leptin-mediated appetite suppression mechanisms.
7) A person deprived of sleep for several days reports increased hunger. Which hormone likely increased?
a) Leptin
b) Ghrelin
c) Insulin
d) CART
Explanation: The answer is Ghrelin. Sleep deprivation increases ghrelin and decreases leptin, stimulating appetite and preference for high-calorie foods. This imbalance promotes weight gain. Ghrelin acts on hypothalamic centers, enhancing NPY and AGRP activity, which trigger hunger signals during energy deficit or sleep loss.
8) Which hypothalamic peptide increases appetite during fasting?
a) CART
b) AGRP
c) CRH
d) Somatostatin
Explanation: The correct answer is AGRP. Fasting increases AGRP expression in arcuate neurons, stimulating appetite. AGRP inhibits melanocortin receptors that normally suppress feeding. This compensatory mechanism helps restore energy balance during caloric restriction or prolonged fasting conditions.
9) A patient with hypothalamic tumor affecting the lateral hypothalamus presents with weight loss and loss of appetite. What is the likely mechanism?
a) Overactivation of AGRP neurons
b) Damage to hunger center
c) Hyperleptinemia
d) Ghrelin excess
Explanation: The answer is Damage to hunger center. The lateral hypothalamus is responsible for stimulating appetite. Tumor-induced damage leads to anorexia, weight loss, and decreased feeding behavior, demonstrating its critical role in energy intake regulation.
10) A child presents with hyperphagia and obesity due to a mutation in the leptin receptor gene. What happens to appetite control?
a) Increased leptin sensitivity
b) Impaired satiety signaling
c) Reduced ghrelin secretion
d) Enhanced α-MSH response
Explanation: The correct answer is Impaired satiety signaling. Mutation in leptin receptor prevents leptin from inhibiting appetite-regulating neurons in the hypothalamus, leading to uncontrolled hunger, hyperphagia, and early-onset obesity despite high leptin levels in circulation.
Chapter: Physiology; Topic: Thermoregulation; Subtopic: Mechanism of Fever
Keyword Definitions:
• Endogenous Pyrogens: Cytokines like IL-1, IL-6, and TNF-α produced by immune cells that mediate fever.
• Hypothalamic Set Point: Temperature level regulated by the preoptic area of the hypothalamus.
• PGE2: Prostaglandin E2, a lipid mediator that raises hypothalamic temperature set point during fever.
• Thermoregulation: Physiological process maintaining internal temperature through heat production and loss mechanisms.
Lead Question - 2014
Endogenous pyrogens act by ?
a) Increasing heat generation
b) Raising thermostat point of hypothalamus
c) Causing vasoconstriction
d) By Non-shivering thermogenesis
Answer & Explanation: Correct answer is b) Raising thermostat point of hypothalamus. Endogenous pyrogens such as IL-1, IL-6, and TNF-α trigger the synthesis of PGE2 in the hypothalamic preoptic area, elevating the temperature set point. The body perceives its normal temperature as low and activates heat-generating mechanisms like shivering and vasoconstriction to raise temperature. This process continues until the body temperature reaches the new set point, resulting in fever. This mechanism aids immune function by inhibiting microbial growth. When pyrogenic signals subside, antipyretic mechanisms like sweating and vasodilation restore normal temperature.
1) Which cytokine is the most potent endogenous pyrogen?
a) IL-1
b) IL-10
c) Interferon-gamma
d) TNF-beta
Answer & Explanation: Correct answer is a) IL-1. IL-1 is a major endogenous pyrogen released by macrophages. It stimulates hypothalamic PGE2 production, elevating the set point for temperature regulation, thereby initiating the febrile response.
2) Fever differs from hyperthermia because:
a) Both are caused by external heat
b) Set-point increases in fever
c) Set-point decreases in fever
d) Fever has no hypothalamic control
Answer & Explanation: Correct answer is b) Set-point increases in fever. In fever, hypothalamic set-point rises due to PGE2, while in hyperthermia, body temperature rises above the set-point due to external or metabolic causes without hypothalamic involvement.
3) The main site of PGE2 synthesis during fever is:
a) Hippocampus
b) Preoptic area of hypothalamus
c) Cerebellum
d) Medulla oblongata
Answer & Explanation: Correct answer is b) Preoptic area of hypothalamus. PGE2 acts here to elevate the set-point temperature, resulting in activation of heat conservation and production mechanisms that induce fever.
4) During fever, the body initiates heat conservation by:
a) Vasodilation
b) Vasoconstriction
c) Perspiration
d) Sweating
Answer & Explanation: Correct answer is b) Vasoconstriction. To conserve heat, cutaneous vessels constrict, reducing blood flow to skin and minimizing heat loss, which helps elevate core temperature to the new hypothalamic set-point.
5) Exogenous pyrogens cause fever by:
a) Directly raising hypothalamic set-point
b) Stimulating endogenous cytokine release
c) Activating sweat glands
d) Inhibiting IL-6 production
Answer & Explanation: Correct answer is b) Stimulating endogenous cytokine release. Exogenous pyrogens like bacterial lipopolysaccharides induce immune cells to produce IL-1 and TNF-α, which in turn raise the hypothalamic set-point through PGE2 synthesis.
6) A patient with infection develops chills before fever. The chills are due to:
a) Decreased body temperature
b) Set-point rising above body temperature
c) Sweating
d) Vasodilation
Answer & Explanation: Correct answer is b) Set-point rising above body temperature. The hypothalamus perceives current temperature as low and activates shivering and vasoconstriction to generate heat, causing chills before the actual rise in temperature.
7) Which of the following is not an endogenous pyrogen?
a) IL-6
b) TNF-α
c) IL-10
d) IL-1
Answer & Explanation: Correct answer is c) IL-10. IL-10 is an anti-inflammatory cytokine that suppresses immune activation and downregulates proinflammatory mediators, thereby reducing fever and inflammation.
8) Which antipyretic drug inhibits prostaglandin synthesis?
a) Paracetamol
b) Atropine
c) Dopamine
d) Epinephrine
Answer & Explanation: Correct answer is a) Paracetamol. Paracetamol (acetaminophen) inhibits cyclooxygenase enzymes in the CNS, thereby reducing PGE2 synthesis in the hypothalamus and lowering the elevated set-point temperature.
9) Which organ detects and controls body temperature changes?
a) Hypothalamus
b) Thalamus
c) Medulla
d) Pituitary gland
Answer & Explanation: Correct answer is a) Hypothalamus. The hypothalamus integrates signals from thermoreceptors and controls autonomic responses to maintain body temperature within physiological limits during fever or cold exposure.
10) Clinical case: A 45-year-old male develops fever after bacterial infection. The fever subsides after taking ibuprofen. What is the mechanism?
a) Increased vasoconstriction
b) Inhibition of PGE2 synthesis
c) Decreased IL-6 secretion
d) Activation of sweat glands
Answer & Explanation: Correct answer is b) Inhibition of PGE2 synthesis. Ibuprofen inhibits COX enzymes, reducing PGE2 production in the hypothalamus, thereby lowering the elevated temperature set-point and promoting heat loss through sweating and vasodilation.
Chapter: Physiology; Topic: Thermoregulation; Subtopic: Fever Mechanism
Keyword Definitions:
• Fever: Elevation of body temperature due to pyrogenic response to infection or inflammation.
• Prostaglandins (PGs): Lipid mediators derived from arachidonic acid, involved in inflammation, fever, and vascular tone.
• PGE2: Key prostaglandin produced in hypothalamus, mediates fever by elevating set-point.
• PGF2α, PGI2, PGD2: Other prostaglandins with roles in smooth muscle contraction, vasodilation, platelet function, and sleep regulation.
Lead Question - 2014
Fever is produced by ?
a) PGF2α
b) PGE2
c) PGI2
d) PGD2
Answer & Explanation: Correct answer is b) PGE2. Fever occurs when pyrogens (endogenous or exogenous) trigger prostaglandin E2 synthesis in the preoptic area of the anterior hypothalamus. PGE2 elevates the hypothalamic set-point, causing heat-conserving mechanisms like vasoconstriction and shivering, which raise body temperature. Other prostaglandins, such as PGF2α, PGI2, and PGD2, have distinct physiological roles including smooth muscle contraction, vasodilation, platelet inhibition, and sleep regulation, and do not directly mediate fever. Understanding PGE2’s role is crucial for therapeutic interventions with antipyretics like NSAIDs that inhibit cyclooxygenase, thereby reducing PGE2 and fever.
1) Endogenous pyrogens include:
a) IL-1, IL-6, TNF-α
b) PGE2 only
c) Histamine
d) Bradykinin
Answer & Explanation: Correct answer is a) IL-1, IL-6, TNF-α. These cytokines are produced by activated leukocytes in infection or inflammation. They stimulate hypothalamic PGE2 production, raising the temperature set-point and causing fever.
2) NSAIDs reduce fever by inhibiting:
a) Lipoxygenase
b) Cyclooxygenase
c) Phospholipase A2
d) Adenylate cyclase
Answer & Explanation: Correct answer is b) Cyclooxygenase. NSAIDs block COX enzymes, preventing conversion of arachidonic acid to prostaglandins, including PGE2. Reduced PGE2 synthesis lowers hypothalamic set-point, thus decreasing fever.
3) Pyrogen-induced PGE2 acts on which hypothalamic region?
a) Posterior hypothalamus
b) Preoptic anterior hypothalamus
c) Suprachiasmatic nucleus
d) Lateral hypothalamus
Answer & Explanation: Correct answer is b) Preoptic anterior hypothalamus. PGE2 in this region increases the thermoregulatory set-point, leading to heat conservation and generation mechanisms that elevate core body temperature during fever.
4) Fever helps the body by:
a) Reducing metabolic rate
b) Enhancing immune response
c) Causing hypothermia
d) Decreasing leukocyte activity
Answer & Explanation: Correct answer is b) Enhancing immune response. Moderate fever improves leukocyte mobility, enhances phagocytosis, and inhibits growth of certain pathogens, serving as a protective physiological response.
5) Exogenous pyrogens are:
a) Bacterial toxins
b) Cytokines
c) PGE2
d) Histamine
Answer & Explanation: Correct answer is a) Bacterial toxins. Lipopolysaccharide (LPS) and other bacterial products act as exogenous pyrogens, stimulating cytokine release which induces hypothalamic PGE2 synthesis, leading to fever.
6) Which prostaglandin is involved in vasodilation but not fever?
a) PGE2
b) PGI2
c) PGF2α
d) None
Answer & Explanation: Correct answer is b) PGI2. PGI2 (prostacyclin) is mainly produced by endothelial cells, inhibits platelet aggregation, and causes vasodilation. It does not directly mediate hypothalamic set-point elevation or fever.
7) Shivering during fever is triggered by:
a) Decreased set-point
b) Increased hypothalamic set-point
c) Peripheral vasodilation
d) Reduced PGE2
Answer & Explanation: Correct answer is b) Increased hypothalamic set-point. When PGE2 raises the hypothalamic set-point above body temperature, heat-generating mechanisms like shivering and vasoconstriction are activated to reach the new set-point.
8) Antipyretics target fever by:
a) Reducing IL-1 production
b) Inhibiting PGE2 synthesis
c) Stimulating TNF-α
d) Activating PGF2α
Answer & Explanation: Correct answer is b) Inhibiting PGE2 synthesis. Drugs like aspirin and ibuprofen inhibit cyclooxygenase, preventing PGE2 formation, thereby lowering the hypothalamic set-point and alleviating fever.
9) Fever-producing prostaglandin is synthesized from:
a) Cholesterol
b) Arachidonic acid
c) Phosphatidylcholine
d) Linoleic acid
Answer & Explanation: Correct answer is b) Arachidonic acid. Arachidonic acid is released from membrane phospholipids and converted by COX enzymes to PGE2, the key mediator elevating hypothalamic set-point and causing fever.
10) Which prostaglandin is mainly involved in sleep regulation, not fever?
a) PGD2
b) PGE2
c) PGI2
d) PGF2α
Answer & Explanation: Correct answer is a) PGD2. PGD2 is synthesized in the brain, particularly by leptomeninges, promoting sleep and modulating circadian rhythm. Unlike PGE2, it does not induce hypothalamic fever response.
Chapter: Nervous System; Topic: Special Senses; Subtopic: Gustation (Taste)
Keyword Definitions:
• Papillae: Small projections on the tongue surface; some contain taste buds. • Fungiform papillae: Mushroom-shaped, mostly at tip and lateral margins of tongue, contain taste buds. • Circumvallate papillae: Large, dome-shaped, located at the posterior tongue in V-shaped row, contain many taste buds. • Foliate papillae: Leaf-shaped, at lateral borders of posterior tongue, contain taste buds. • Filiform papillae: Conical, cover most of tongue surface, primarily tactile, do not contain taste buds.
Lead Question - 2014
False regarding papillae of tongue ?
a) Fungiform papillae at tip
b) Circumvallate papillae at base
c) Foliate papillae at back edge
d) Filiform papillae have taste buds at tip
Answer & Explanation: Correct answer is d) Filiform papillae have taste buds at tip. Filiform papillae are keratinized, conical structures covering the anterior two-thirds of the tongue, specialized for mechanical sensation (touch, texture) rather than gustation. They lack taste buds entirely. Fungiform papillae at tip contain taste buds, circumvallate papillae at base have multiple taste buds in trench, and foliate papillae at lateral posterior edges contain taste buds. Thus, only statement d is false. Understanding papilla distribution is critical for clinical assessment of taste disorders and tongue pathology.
1) Which papillae contain the most taste buds?
a) Fungiform
b) Circumvallate
c) Foliate
d) Filiform
Answer & Explanation: Correct answer is b) Circumvallate. Circumvallate papillae are few but large, located at the posterior tongue, each surrounded by a trench with hundreds of taste buds. They are key for bitter taste detection and are innervated by glossopharyngeal nerve (cranial nerve IX).
2) Taste buds in fungiform papillae are innervated by:
a) Glossopharyngeal nerve
b) Facial nerve
c) Vagus nerve
d) Hypoglossal nerve
Answer & Explanation: Correct answer is b) Facial nerve. Fungiform papillae at anterior tongue tip transmit taste via chorda tympani branch of facial nerve (cranial nerve VII), conveying sweet, salty, and umami sensations.
3) Foliate papillae are most prominent in:
a) Children
b) Adults
c) Elderly
d) Newborns
Answer & Explanation: Correct answer is a) Children. Foliate papillae are leaf-like folds at lateral posterior tongue edges, more prominent in children. They regress with age and contain taste buds mainly sensitive to sour tastes.
4) Filiform papillae function primarily in:
a) Bitter taste
b) Mechanical sensation
c) Sweet taste
d) Saliva secretion
Answer & Explanation: Correct answer is b) Mechanical sensation. Filiform papillae are keratinized, conical, covering most anterior tongue, aiding friction and texture detection. They do not contain taste buds, distinguishing them from gustatory papillae.
5) Circumvallate papillae are innervated by:
a) Facial nerve
b) Glossopharyngeal nerve
c) Vagus nerve
d) Trigeminal nerve
Answer & Explanation: Correct answer is b) Glossopharyngeal nerve. Posterior tongue circumvallate papillae transmit taste via cranial nerve IX, particularly bitter taste, and contain many taste buds arranged in trenches.
6) Which papillae detect sour taste predominantly?
a) Fungiform
b) Foliate
c) Circumvallate
d) Filiform
Answer & Explanation: Correct answer is b) Foliate. Foliate papillae are lateral folds at the posterior tongue with numerous taste buds, particularly sensitive to sour taste. They regress with age, being most functional in children.
7) Tongue papillae without taste buds are:
a) Fungiform
b) Filiform
c) Circumvallate
d) Foliate
Answer & Explanation: Correct answer is b) Filiform. Filiform papillae cover the anterior tongue, specialized for mechanical tasks like texture perception and gripping food. They do not contain taste buds, unlike fungiform, circumvallate, and foliate papillae.
8) Papillae with mushroom-shaped appearance are:
a) Circumvallate
b) Fungiform
c) Foliate
d) Filiform
Answer & Explanation: Correct answer is b) Fungiform. Fungiform papillae are scattered on anterior tongue tip and sides, dome-shaped, contain taste buds, and are innervated by the facial nerve via chorda tympani.
9) Which cranial nerve innervates taste buds in posterior tongue?
a) Facial
b) Glossopharyngeal
c) Vagus
d) Hypoglossal
Answer & Explanation: Correct answer is b) Glossopharyngeal. Cranial nerve IX supplies circumvallate papillae and posterior one-third of tongue, transmitting bitter and other taste sensations to the brain.
10) The majority of tongue surface is covered by:
a) Filiform papillae
b) Fungiform papillae
c) Circumvallate papillae
d) Foliate papillae
Answer & Explanation: Correct answer is a) Filiform papillae. Filiform papillae cover most of anterior tongue, providing mechanical function, friction for food handling, and do not contain taste buds, unlike scattered fungiform papillae.
Chapter: Nervous System; Topic: Sensory Systems; Subtopic: Special Senses and Proprioception
Keyword Definitions:
• Sense organ: Specialized structure that detects specific stimuli and transmits sensory information to the central nervous system.
• Efferent supply: Nerve fibers that carry signals away from the CNS to peripheral organs or muscles.
• Golgi tendon organ: Proprioceptive receptor in tendons, senses muscle tension, primarily afferent but some efferent modulation via gamma motor fibers.
• Organ of Corti: Sensory epithelium in cochlea detecting sound; has efferent innervation from olivocochlear bundle.
• Retina: Light-sensitive layer in the eye, primarily afferent via optic nerve.
• Taste bud: Receptor for gustatory stimuli, primarily afferent via cranial nerves VII, IX, X.
Lead Question - 2014
Sense organ which is having efferent supply?
a) Golgi tendon organ
b) Organ of Corti
c) Retina
d) Taste bud
Answer & Explanation: The correct answer is b) Organ of Corti. The Organ of Corti, located in the cochlea of the inner ear, receives efferent fibers from the olivocochlear bundle, which modulate outer hair cell activity to fine-tune cochlear sensitivity and protect from loud sounds. This is unique among sensory organs as most sense organs, like retina and taste buds, receive purely afferent signals. Golgi tendon organs have some efferent gamma modulation, but main efferent supply in sense organs is classically described in the Organ of Corti. Efferent fibers help in auditory gain control and sound discrimination.
1) The efferent fibers to the cochlea originate from:
a) Superior olivary complex
b) Inferior colliculus
c) Cochlear nucleus
d) Lateral geniculate body
Answer & Explanation: Correct answer is a) Superior olivary complex. Efferent fibers of the olivocochlear bundle arise from the superior olivary complex and synapse on outer hair cells, regulating cochlear amplification and protecting against noise-induced damage.
2) Golgi tendon organs detect:
a) Muscle length
b) Muscle tension
c) Joint position
d) Vibration
Answer & Explanation: Correct answer is b) Muscle tension. Golgi tendon organs are proprioceptors embedded in tendons, sensing the force generated by muscle contraction. They prevent excessive tension and potential injury by initiating inhibitory reflexes.
3) Efferent innervation in the Organ of Corti primarily modulates:
a) Inner hair cells
b) Outer hair cells
c) Tectorial membrane
d) Spiral ganglion
Answer & Explanation: Correct answer is b) Outer hair cells. Efferent fibers from the olivocochlear bundle synapse on outer hair cells, controlling their motility. This modulation enhances frequency selectivity and reduces cochlear damage from loud sounds.
4) Retina is supplied by efferent fibers from:
a) Edinger-Westphal nucleus
b) Olivocochlear bundle
c) Facial nerve
d) None
Answer & Explanation: Correct answer is a) Edinger-Westphal nucleus. While retina itself is sensory, parasympathetic efferents from Edinger-Westphal nucleus control pupillary constriction via ciliary ganglion. Direct efferent supply to photoreceptors is absent; main transmission is afferent through optic nerve.
5) Taste buds transmit information via efferent fibers?
a) True
b) False
Answer & Explanation: Correct answer is b) False. Taste buds are purely afferent receptors, sending gustatory information to the brainstem via cranial nerves VII, IX, and X. There is no efferent modulation at the receptor level.
6) Organ of Corti efferent system helps in:
a) Enhancing sound localization
b) Muscle tone regulation
c) Color vision
d) Taste discrimination
Answer & Explanation: Correct answer is a) Enhancing sound localization. The olivocochlear efferent system fine-tunes cochlear output, suppresses background noise, and improves signal-to-noise ratio, aiding precise localization of sound sources.
7) Which of the following has both afferent and efferent innervation?
a) Retina
b) Organ of Corti
c) Taste bud
d) Muscle spindle
Answer & Explanation: Correct answer is b) Organ of Corti. The Organ of Corti uniquely receives afferent signals via spiral ganglion neurons and efferent fibers via the olivocochlear bundle, modulating outer hair cell activity. Other organs like retina and taste buds primarily transmit afferent signals.
8) Lesion of olivocochlear bundle causes:
a) Loss of light touch
b) Increased susceptibility to noise-induced hearing loss
c) Loss of taste
d) Loss of pupillary reflex
Answer & Explanation: Correct answer is b) Increased susceptibility to noise-induced hearing loss. Efferent fibers reduce cochlear amplifier gain and protect outer hair cells. Damage to olivocochlear bundle diminishes this protective mechanism, increasing vulnerability to acoustic trauma.
9) Gamma motor fibers modulate which receptor?
a) Golgi tendon organ
b) Pacinian corpuscle
c) Muscle spindle
d) Organ of Corti
Answer & Explanation: Correct answer is c) Muscle spindle. Gamma motor fibers adjust intrafusal fiber tension, maintaining sensitivity of muscle spindles during muscle contraction, allowing accurate proprioception.
10) Which sense organ has purely afferent function?
a) Organ of Corti
b) Golgi tendon organ
c) Retina
d) Both a and b
Answer & Explanation: Correct answer is c) Retina. The retina converts light into neural signals sent to the brain via optic nerve. Unlike the Organ of Corti, it lacks direct efferent modulation at the receptor level.
Chapter: Nervous System; Topic: Somatosensory System; Subtopic: Mechanoreceptors
Keyword Definitions:
• Vibration sensation: The perception of oscillatory stimuli applied to the skin, mediated by specialized mechanoreceptors.
• Pacinian corpuscles: Large, encapsulated mechanoreceptors located in deep dermis and subcutaneous tissue, sensitive to high-frequency vibration and pressure.
• Meissner's corpuscles: Superficial dermal mechanoreceptors, detect low-frequency vibration and light touch.
• Merkel's discs: Slowly adapting mechanoreceptors, sensitive to sustained pressure and texture.
• Ruffini endings: Slowly adapting mechanoreceptors detecting skin stretch and joint position.
• Mechanoreceptors: Sensory receptors that respond to mechanical stimuli like pressure, touch, stretch, and vibration.
Lead Question - 2014
Vibrations are felt by ?
a) Meissner's corpuscle
b) Merkel's disc
c) Pacinian corpuscle
d) Ruffini's end organ
Answer & Explanation: The correct answer is c) Pacinian corpuscle. Pacinian corpuscles are large, onion-like encapsulated receptors located in the deep dermis and subcutaneous tissue. They are highly sensitive to high-frequency vibrations (about 250 Hz) and deep pressure. Meissner’s corpuscles detect light touch and low-frequency vibrations, Merkel discs sense pressure and texture, and Ruffini endings respond to skin stretch. Pacinian corpuscles rapidly adapt to stimuli, making them ideal for detecting transient vibrations. Clinically, loss of vibration sense can indicate peripheral neuropathy or dorsal column lesions, affecting proprioception and fine touch perception.
1) Low-frequency vibration is detected by:
a) Pacinian corpuscle
b) Meissner's corpuscle
c) Merkel's disc
d) Ruffini's ending
Answer & Explanation: The correct answer is b) Meissner's corpuscle. Meissner’s corpuscles are superficial dermal receptors that respond to light touch and low-frequency vibrations (30–50 Hz). They allow discrimination of fine textures and small object movements across the skin, important for tactile acuity.
2) Merkel's discs primarily detect:
a) Deep pressure
b) Vibration
c) Sustained pressure and texture
d) Skin stretch
Answer & Explanation: The correct answer is c) Sustained pressure and texture. Merkel’s discs are slowly adapting receptors in the basal epidermis. They provide high-resolution spatial information about objects in contact with the skin and play a key role in tactile discrimination.
3) Ruffini endings sense:
a) High-frequency vibration
b) Skin stretch
c) Light touch
d) Temperature
Answer & Explanation: The correct answer is b) Skin stretch. Ruffini endings, found in dermis and joint capsules, respond to sustained stretch, contributing to the perception of finger position and hand shape. They are slowly adapting mechanoreceptors, unlike Pacinian corpuscles which rapidly adapt to vibration.
4) Which receptor adapts rapidly to stimuli?
a) Merkel's disc
b) Pacinian corpuscle
c) Ruffini ending
d) Free nerve ending
Answer & Explanation: The correct answer is b) Pacinian corpuscle. Pacinian corpuscles respond quickly to changes in stimulus and stop firing if stimulus is constant. Rapid adaptation makes them ideal for detecting vibrations and transient pressure but not sustained pressure.
5) Loss of vibration sense suggests lesion in:
a) Corticospinal tract
b) Dorsal column-medial lemniscus pathway
c) Spinothalamic tract
d) Corticobulbar tract
Answer & Explanation: The correct answer is b) Dorsal column-medial lemniscus pathway. Vibration, fine touch, and proprioception travel via dorsal columns to the brain. Lesions here, due to neuropathy or spinal cord injury, cause contralateral loss of vibration and joint position sense.
6) Pacinian corpuscles are most abundant in:
a) Fingertips
b) Palms
c) Deep dermis of hands and feet
d) Hair follicles
Answer & Explanation: The correct answer is c) Deep dermis of hands and feet. Pacinian corpuscles are concentrated in areas where detection of vibration and pressure changes is critical, allowing precise manipulation of objects and detection of surface textures.
7) Slowly adapting mechanoreceptors include:
a) Pacinian corpuscle
b) Meissner's corpuscle
c) Ruffini endings and Merkel's discs
d) Free nerve endings
Answer & Explanation: The correct answer is c) Ruffini endings and Merkel's discs. These receptors maintain firing during sustained stimuli, enabling detection of skin stretch and pressure. Rapidly adapting receptors like Pacinian corpuscles only respond at onset of stimulus.
8) Which receptor is involved in object manipulation and grip control?
a) Pacinian corpuscle
b) Merkel's disc
c) Meissner's corpuscle
d) Ruffini ending
Answer & Explanation: The correct answer is d) Ruffini ending. Ruffini endings detect skin stretch, providing proprioceptive feedback about finger and hand positions, essential for adjusting grip force during manipulation.
9) High-frequency vibrations are transmitted via:
a) Meissner’s corpuscle
b) Pacinian corpuscle
c) Merkel disc
d) Ruffini ending
Answer & Explanation: The correct answer is b) Pacinian corpuscle. Pacinian corpuscles detect high-frequency vibrations (~250 Hz). Their layered structure filters mechanical changes efficiently, rapidly transducing pressure and vibration signals to sensory neurons.
10) Which receptor is primarily responsible for tactile acuity?
a) Pacinian corpuscle
b) Meissner's corpuscle
c) Ruffini ending
d) Golgi tendon organ
Answer & Explanation: The correct answer is b) Meissner's corpuscle. Meissner corpuscles are densely packed in fingertips and lips, allowing discrimination of fine textures and low-frequency vibration. They support precise tactile perception essential for tasks like reading Braille or handling small objects.
Chapter: Nervous System; Topic: Cerebral Cortex; Subtopic: Sensory Perception
Keyword Definitions:
• Sensory perception: The process by which the brain receives and interprets sensory input from the environment via specialized receptors.
• Brodmann's areas: Cytoarchitectonic regions of the cerebral cortex defined by Korbinian Brodmann, each with specific functional roles.
• Primary somatosensory cortex: Located in postcentral gyrus (areas 3,1,2), responsible for tactile, proprioceptive, and pressure sensation.
• Motor cortex: Areas 4 and 6, responsible for voluntary movement.
• Broca's area: Areas 44 and 45, involved in speech production.
• Primary auditory cortex: Areas 41 and 42, responsible for hearing perception.
Lead Question - 2014
Sensory perception involves Brodmann's area ?
a) 3, 1, 2
b) 4, 6
c) 44, 45
d) 41, 42
Answer & Explanation: The correct answer is a) 3, 1, 2. Brodmann areas 3, 1, and 2 comprise the primary somatosensory cortex located in the postcentral gyrus of the parietal lobe. These areas receive input from the thalamus corresponding to tactile, pressure, pain, and proprioceptive signals from the contralateral side of the body. Damage to these areas results in sensory deficits such as numbness or loss of tactile discrimination. Areas 4 and 6 are motor cortices, 44 and 45 are Broca’s speech areas, and 41 and 42 are primary auditory areas involved in hearing perception.
1) Lesion in Brodmann areas 3,1,2 causes:
a) Hemiplegia
b) Loss of tactile sensation
c) Aphasia
d) Cortical deafness
Answer & Explanation: The correct answer is b) Loss of tactile sensation. Lesions in areas 3,1,2 disrupt processing of somatosensory information, causing contralateral deficits in touch, vibration, proprioception, and stereognosis, while motor and auditory functions remain unaffected.
2) Primary motor cortex corresponds to which Brodmann area?
a) 3,1,2
b) 4
c) 6
d) 41,42
Answer & Explanation: The correct answer is b) 4. Area 4, located in the precentral gyrus, is the primary motor cortex responsible for voluntary movement initiation. Area 6 is premotor/supplementary motor cortex aiding planning, while 3,1,2 is sensory and 41,42 is auditory.
3) Broca’s area is involved in:
a) Sensory perception
b) Motor planning
c) Speech production
d) Hearing
Answer & Explanation: The correct answer is c) Speech production. Broca’s area (44,45) located in the inferior frontal gyrus of the dominant hemisphere coordinates speech articulation. Lesions cause expressive aphasia, while sensory perception remains intact.
4) Lesion in area 41,42 causes:
a) Cortical deafness
b) Hemiplegia
c) Apraxia
d) Agnosia
Answer & Explanation: The correct answer is a) Cortical deafness. Areas 41 and 42 constitute the primary auditory cortex in the superior temporal gyrus. Damage here results in inability to perceive sound despite intact peripheral auditory apparatus.
5) Somatosensory association cortex corresponds to:
a) 5,7
b) 4
c) 44,45
d) 41,42
Answer & Explanation: The correct answer is a) 5,7. Areas 5 and 7 integrate somatosensory information to recognize object size, shape, and spatial relations, facilitating perception and motor planning.
6) Lesion in postcentral gyrus may lead to:
a) Contralateral hemiplegia
b) Contralateral sensory loss
c) Broca’s aphasia
d) Cortical blindness
Answer & Explanation: The correct answer is b) Contralateral sensory loss. Postcentral gyrus houses primary somatosensory cortex (3,1,2). Lesion affects contralateral tactile, proprioceptive, and vibratory sensations without affecting motor function.
7) Primary gustatory cortex is located in:
a) Insula and frontal operculum
b) Postcentral gyrus
c) Precentral gyrus
d) Superior temporal gyrus
Answer & Explanation: The correct answer is a) Insula and frontal operculum. Gustatory cortex processes taste, while primary somatosensory cortex (3,1,2) processes touch and proprioception.
8) Damage to area 3 alone may cause:
a) Loss of vibration sense
b) Loss of voluntary movement
c) Aphasia
d) Cortical deafness
Answer & Explanation: The correct answer is a) Loss of vibration sense. Area 3 receives direct thalamic input, critical for discriminative touch and vibration. Damage impairs these sensations without affecting motor or auditory functions.
9) Which area is involved in proprioception?
a) 3,1,2
b) 4
c) 44,45
d) 41,42
Answer & Explanation: The correct answer is a) 3,1,2. Primary somatosensory cortex interprets proprioceptive input from muscles and joints, enabling body position awareness. Lesions result in impaired joint position sense.
10) Integration of sensory input for spatial orientation occurs in:
a) Areas 5 and 7
b) Areas 3,1,2
c) Areas 4 and 6
d) Areas 41,42
Answer & Explanation: The correct answer is a) Areas 5 and 7. Somatosensory association cortex integrates tactile and proprioceptive information to form spatial awareness, essential for coordinated motor actions and object recognition.
Chapter: Nervous System; Topic: Peripheral Nervous System; Subtopic: Myelination
Keyword Definitions:
• Myelination: Process of forming a myelin sheath around axons to increase conduction velocity of nerve impulses.
• Peripheral Nervous System (PNS): Part of the nervous system outside brain and spinal cord, including cranial and spinal nerves.
• Schwann cells: Glial cells in PNS responsible for forming myelin sheaths around peripheral axons.
• Oligodendrocytes: CNS glial cells that myelinate multiple axons in the brain and spinal cord.
• Astrocytes: Star-shaped CNS glial cells supporting neurons, regulating extracellular environment, but not myelinating axons.
• Ependymal cells: CNS glial cells lining ventricles, involved in cerebrospinal fluid production, not myelination.
Lead Question - 2014
Myelination in peripheral nervous system is done by ?
a) Astrocytes
b) Oligodendrocytes
c) Ependymal cells
d) Schwann cells
Answer & Explanation: The correct answer is d) Schwann cells. Schwann cells are the primary glial cells of the PNS responsible for myelinating peripheral nerves. Each Schwann cell wraps around a single axon segment, forming a myelin sheath that increases conduction speed of action potentials. Unlike oligodendrocytes in the CNS, which can myelinate multiple axons, Schwann cells provide one-to-one myelination. Damage to Schwann cells, as in peripheral neuropathies, leads to slowed nerve conduction, muscle weakness, and sensory deficits. Astrocytes and ependymal cells are CNS glia and do not myelinate axons.
1) Which CNS cell myelinates multiple axons?
a) Schwann cells
b) Oligodendrocytes
c) Astrocytes
d) Microglia
Answer & Explanation: The correct answer is b) Oligodendrocytes. Oligodendrocytes myelinate multiple CNS axons simultaneously, increasing conduction speed. In contrast, Schwann cells myelinate individual PNS axons. Astrocytes provide support and maintain extracellular environment, while microglia act as immune cells in CNS.
2) Which disease involves demyelination in PNS?
a) Multiple sclerosis
b) Guillain-Barré syndrome
c) Alzheimer’s disease
d) Parkinson’s disease
Answer & Explanation: The correct answer is b) Guillain-Barré syndrome. This autoimmune disorder targets Schwann cells, causing demyelination of peripheral nerves, leading to muscle weakness, paresthesia, and areflexia. Multiple sclerosis affects CNS myelin formed by oligodendrocytes, while Alzheimer’s and Parkinson’s are neurodegenerative but not primarily demyelinating.
3) Node of Ranvier is associated with:
a) Schwann cells
b) Oligodendrocytes
c) Astrocytes
d) Microglia
Answer & Explanation: The correct answer is a) Schwann cells. Nodes of Ranvier are gaps between Schwann cell myelin segments along PNS axons, allowing saltatory conduction. These nodes facilitate rapid action potential propagation. CNS nodes are similarly associated with oligodendrocyte myelin.
4) Which PNS glial cell also aids in axon regeneration?
a) Oligodendrocytes
b) Schwann cells
c) Astrocytes
d) Ependymal cells
Answer & Explanation: The correct answer is b) Schwann cells. Schwann cells produce growth factors and form bands of Büngner to guide regenerating axons after peripheral nerve injury. CNS oligodendrocytes inhibit regeneration, while astrocytes and ependymal cells are CNS supportive cells without regenerative function.
5) Which statement is true?
a) Schwann cells myelinate CNS
b) Oligodendrocytes myelinate PNS
c) Schwann cells myelinate PNS
d) Astrocytes myelinate both CNS and PNS
Answer & Explanation: The correct answer is c) Schwann cells myelinate PNS. Schwann cells wrap single axons in the PNS, whereas oligodendrocytes myelinate multiple CNS axons. Astrocytes do not form myelin; they maintain extracellular ionic balance and support neurons.
6) Which protein is major in PNS myelin?
a) Myelin basic protein (MBP)
b) Proteolipid protein (PLP)
c) Myelin protein zero (P0)
d) Neurofilament
Answer & Explanation: The correct answer is c) Myelin protein zero (P0). P0 is a structural glycoprotein in Schwann cell myelin, essential for compaction and stability. MBP and PLP are predominant in CNS myelin, while neurofilaments form the axonal cytoskeleton.
7) Peripheral neuropathy due to diabetes affects:
a) Oligodendrocytes
b) Schwann cells
c) Astrocytes
d) Microglia
Answer & Explanation: The correct answer is b) Schwann cells. Chronic hyperglycemia damages Schwann cells, leading to demyelination and impaired nerve conduction. Symptoms include distal sensory loss, weakness, and slowed reflexes. CNS glial cells remain largely unaffected.
8) Which factor stimulates Schwann cell myelination?
a) High neuronal activity
b) Low axonal diameter
c) Absence of laminin
d) Astrocytic signaling
Answer & Explanation: The correct answer is a) High neuronal activity. Active axons induce Schwann cells to myelinate via electrical activity and neuregulin signaling. Large-diameter axons are preferentially myelinated. Laminin supports but absence inhibits myelination; astrocytes are CNS glia.
9) Which PNS structure is unmyelinated but surrounded by Schwann cells?
a) Large motor axons
b) Small pain fibers
c) Oligodendrocyte processes
d) Corticospinal axons
Answer & Explanation: The correct answer is b) Small pain fibers. Unmyelinated C fibers are enveloped by non-myelinating Schwann cells forming Remak bundles, allowing support and trophic function without fast conduction. Large motor axons are myelinated for rapid transmission.
10) In PNS regeneration, Schwann cells:
a) Inhibit axon growth
b) Form myelin only
c) Guide regenerating axons
d) Replace oligodendrocytes
Answer & Explanation: The correct answer is c) Guide regenerating axons. After injury, Schwann cells proliferate, form bands of Büngner, and secrete neurotrophic factors, facilitating axon regrowth. CNS oligodendrocytes cannot guide regeneration, leading to limited CNS repair.
Chapter: Central Nervous System; Topic: Motor Systems; Subtopic: Posture and Balance Control
Keyword Definitions:
• Basal Ganglia: Group of subcortical nuclei regulating voluntary motor control, procedural learning, routine behaviors, and posture.
• Cerebellum: Brain structure controlling coordination, precision, balance, and posture of voluntary movements.
• Hypothalamus: Brain region controlling autonomic functions, endocrine activity, and homeostasis.
• Amygdala: Limbic system structure involved in emotion processing and fear responses.
• Posture: The position in which the body is held while standing, walking, or resting.
Lead Question - 2014
While walking or standing, posture is maintained by ?
a) Basal ganglia
b) Hypothalamus
c) Cerebellum
d) Amygdala
Answer & Explanation: The correct answer is c) Cerebellum. The cerebellum integrates sensory inputs from vestibular, visual, and proprioceptive systems to fine-tune muscle activity for balance and posture. It ensures smooth coordination during standing and walking, preventing falls. Basal ganglia regulate initiation and amplitude of movements, but maintenance of upright posture and real-time adjustments primarily depend on cerebellar circuits. Damage to the cerebellum results in ataxia, unsteady gait, and difficulty maintaining posture. Hypothalamus and amygdala do not directly control posture but are involved in homeostasis and emotional processing, respectively.
1) Lesion in cerebellum results in which of the following?
a) Rigidity
b) Ataxia
c) Tremor only
d) Hemiplegia
Answer & Explanation: The correct answer is b) Ataxia. Cerebellar lesions disrupt coordination and balance, causing ataxia, unsteady gait, dysmetria, and inability to maintain posture. Muscle strength may remain normal, differentiating it from pyramidal tract lesions. Both medial and lateral cerebellar damage can affect axial control and limb coordination, leading to pronounced difficulty in standing or walking.
2) Which part of the cerebellum primarily controls posture and balance?
a) Lateral hemispheres
b) Vermis
c) Flocculonodular lobe
d) Dentate nucleus
Answer & Explanation: The correct answer is b) Vermis. The vermis receives input from the vestibular system and proprioceptors to control axial muscles and maintain posture. Lesions here cause truncal ataxia and imbalance, affecting standing and walking. The flocculonodular lobe also contributes to balance via vestibular connections, while lateral hemispheres fine-tune limb coordination for voluntary movements.
3) Which sensory system provides feedback for posture maintenance?
a) Visual
b) Vestibular
c) Proprioceptive
d) All of the above
Answer & Explanation: The correct answer is d) All of the above. Posture control integrates visual, vestibular, and proprioceptive inputs. Visual system detects environmental orientation, vestibular system senses head position and motion, and proprioceptors monitor muscle and joint status. The cerebellum processes these signals to maintain balance during standing and walking.
4) Which pathway is critical for postural reflexes?
a) Corticospinal tract
b) Vestibulospinal tract
c) Spinothalamic tract
d) Optic tract
Answer & Explanation: The correct answer is b) Vestibulospinal tract. The vestibulospinal tract originates in vestibular nuclei and projects to spinal interneurons controlling axial and proximal limb muscles. It mediates postural reflexes in response to changes in head position, working with cerebellar input to maintain stability during walking and standing.
5) Damage to basal ganglia affects posture primarily by:
a) Causing rigidity and bradykinesia
b) Causing ataxia
c) Causing tremor only
d) Impairing sensory feedback
Answer & Explanation: The correct answer is a) Causing rigidity and bradykinesia. Basal ganglia lesions, as in Parkinson’s disease, lead to rigidity, decreased movement amplitude, and postural instability. While the cerebellum adjusts balance dynamically, basal ganglia lesions indirectly impair posture by limiting smooth initiation and scaling of movements.
6) Which cerebellar disorder leads to broad-based, unsteady gait?
a) Spinocerebellar ataxia
b) Stroke in motor cortex
c) Parkinson’s disease
d) Amyotrophic lateral sclerosis
Answer & Explanation: The correct answer is a) Spinocerebellar ataxia. Degenerative cerebellar diseases, such as spinocerebellar ataxia, impair coordination and trunk stability, producing a broad-based, staggering gait. Patients often sway while standing, with difficulty maintaining balance during ambulation, highlighting the cerebellum’s key role in posture maintenance.
7) Which lobe of the cerebellum is primarily connected to vestibular nuclei?
a) Anterior lobe
b) Posterior lobe
c) Flocculonodular lobe
d) Lateral hemisphere
Answer & Explanation: The correct answer is c) Flocculonodular lobe. The flocculonodular lobe receives direct vestibular input to coordinate eye movements and balance. Lesions here result in truncal ataxia, impaired standing, and tendency to fall, emphasizing its role in posture and equilibrium.
8) Postural adjustments during walking are integrated mainly by:
a) Hypothalamus
b) Amygdala
c) Cerebellum and brainstem nuclei
d) Spinal cord only
Answer & Explanation: The correct answer is c) Cerebellum and brainstem nuclei. The cerebellum processes sensory input from vestibular, proprioceptive, and visual systems, while brainstem nuclei like vestibular and reticular formations implement rapid postural reflexes. Together, they ensure dynamic balance during locomotion, preventing falls.
9) Which clinical sign indicates cerebellar involvement affecting posture?
a) Positive Romberg sign
b) Hemiplegia
c) Hyperreflexia
d) Spasticity
Answer & Explanation: The correct answer is a) Positive Romberg sign. A positive Romberg test, showing instability when eyes are closed, reflects impaired integration of proprioceptive and cerebellar signals needed for posture maintenance. Cerebellar lesions or sensory deficits compromise standing balance, confirming involvement in postural control.
10) During standing, rapid corrective movements to prevent falling are controlled primarily by:
a) Corticospinal tract
b) Vestibulospinal and reticulospinal tracts
c) Spinothalamic tract
d) Hypothalamospinal tract
Answer & Explanation: The correct answer is b) Vestibulospinal and reticulospinal tracts. These extrapyramidal tracts, guided by cerebellar input, activate axial and proximal limb muscles to maintain posture. Vestibulospinal tract responds to head movement, while reticulospinal tract adjusts tone and reflexes. Damage to these pathways causes instability and postural imbalance.
Chapter: Central Nervous System; Topic: Motor Systems; Subtopic: Extrapyramidal vs Pyramidal Tracts
Keyword Definitions:
• Extrapyramidal Tracts: Motor pathways that control involuntary movements, posture, and muscle tone, not directly via the corticospinal (pyramidal) tract.
• Pyramidal Tract: Direct motor pathway (corticospinal and corticobulbar tracts) responsible for voluntary fine movements.
• Reticulospinal Tract: Extrapyramidal pathway modulating voluntary and reflex motor activity and muscle tone.
• Rubrospinal Tract: Extrapyramidal tract originating in the red nucleus; facilitates flexor muscles and inhibits extensors.
• Tectospinal Tract: Extrapyramidal tract mediating reflex postural movements in response to visual and auditory stimuli.
Lead Question - 2014
Which is not an extrapyramidal tract ?
a) Reticulospinal tract
b) Rubrospinal tract
c) Corticospinal tract
d) Tectospinal tract
Answer & Explanation: The correct answer is c) Corticospinal tract. The corticospinal tract is a pyramidal tract that directly connects the cerebral cortex to spinal motor neurons to control voluntary fine movements. Extrapyramidal tracts, including reticulospinal, rubrospinal, and tectospinal tracts, regulate involuntary movements, posture, and muscle tone indirectly through brainstem motor centers. Damage to extrapyramidal pathways leads to movement disorders such as rigidity, tremor, and abnormal posturing, while corticospinal tract lesions cause weakness and spasticity.
1) Which extrapyramidal tract primarily facilitates flexor muscles of the upper limb?
a) Reticulospinal tract
b) Rubrospinal tract
c) Vestibulospinal tract
d) Tectospinal tract
Answer & Explanation: The correct answer is b) Rubrospinal tract. Originating in the red nucleus, the rubrospinal tract facilitates upper limb flexors and inhibits extensors, contributing to postural control and fine motor coordination. Lesions produce weakness in flexor muscles, affecting voluntary limb movements, while gross motor functions are preserved through corticospinal pathways and other extrapyramidal tracts.
2) Which extrapyramidal tract originates from the pontine and medullary reticular formation?
a) Reticulospinal tract
b) Rubrospinal tract
c) Corticospinal tract
d) Vestibulospinal tract
Answer & Explanation: The correct answer is a) Reticulospinal tract. This tract originates in the pontine and medullary reticular formation and projects to spinal interneurons. It modulates voluntary movement, reflex activity, and muscle tone, facilitating automatic posture and locomotion. Damage leads to abnormal tone, spasticity, and impaired postural adjustments.
3) Which tract is responsible for reflex postural movements in response to visual stimuli?
a) Rubrospinal tract
b) Reticulospinal tract
c) Tectospinal tract
d) Corticospinal tract
Answer & Explanation: The correct answer is c) Tectospinal tract. Originating from the superior colliculus in the midbrain, the tectospinal tract mediates reflex head and neck movements toward visual and auditory stimuli. It coordinates posture with sensory input, contributing to orientation and protective reflexes, whereas pyramidal tracts are involved in voluntary motor control.
4) Lesion of the corticospinal tract typically results in:
a) Rigidity
b) Tremor
c) Spastic paralysis
d) Abnormal posture
Answer & Explanation: The correct answer is c) Spastic paralysis. The corticospinal (pyramidal) tract directly transmits motor signals from the cortex to spinal motor neurons. Lesions lead to spasticity, hyperreflexia, and weakness of voluntary movements, mainly affecting fine motor control. Extrapyramidal lesions, by contrast, produce rigidity, tremor, and abnormal postural changes without direct weakness.
5) The vestibulospinal tract primarily influences:
a) Fine finger movements
b) Axial and proximal limb muscles for posture
c) Reflexes in cranial nerves
d) Sensory perception
Answer & Explanation: The correct answer is b) Axial and proximal limb muscles for posture. The vestibulospinal tract originates in vestibular nuclei and controls balance and posture by influencing axial and proximal limb muscles. It is part of the extrapyramidal system and works in coordination with other tracts to maintain upright stance and compensate for perturbations.
6) Which tract is primarily responsible for facilitating voluntary fine movements?
a) Corticospinal tract
b) Rubrospinal tract
c) Reticulospinal tract
d) Tectospinal tract
Answer & Explanation: The correct answer is a) Corticospinal tract. The corticospinal tract is a pyramidal pathway critical for precise voluntary movements, especially of distal muscles in the hands and fingers. Extrapyramidal tracts support gross movements, posture, and reflex modulation, but fine dexterity depends primarily on corticospinal projections.
7) A patient exhibits rigidity, tremor, and abnormal posture with preserved voluntary strength. Which system is likely affected?
a) Pyramidal system
b) Extrapyramidal system
c) Cerebellar system
d) Sensory system
Answer & Explanation: The correct answer is b) Extrapyramidal system. Extrapyramidal lesions impair involuntary movement control, leading to rigidity, tremor, bradykinesia, and abnormal posture. Voluntary strength remains relatively preserved because the pyramidal tract (corticospinal) is intact. Parkinson’s disease is a classical example of extrapyramidal dysfunction affecting motor control indirectly.
8) Which tract is not involved in involuntary postural adjustments?
a) Reticulospinal tract
b) Rubrospinal tract
c) Corticospinal tract
d) Vestibulospinal tract
Answer & Explanation: The correct answer is c) Corticospinal tract. While corticospinal fibers mediate voluntary movement, extrapyramidal tracts like reticulospinal, rubrospinal, and vestibulospinal tracts regulate involuntary adjustments, muscle tone, and posture. Damage to extrapyramidal tracts impairs balance and reflexive movements but spares voluntary motor power.
9) Which tract primarily projects from the red nucleus in the midbrain?
a) Rubrospinal tract
b) Reticulospinal tract
c) Corticospinal tract
d) Vestibulospinal tract
Answer & Explanation: The correct answer is a) Rubrospinal tract. The rubrospinal tract originates from the red nucleus and modulates flexor muscle activity while inhibiting extensors, mainly affecting upper limb movements. It complements the corticospinal tract and is a key component of the extrapyramidal motor system.
10) Which statement differentiates pyramidal from extrapyramidal tracts?
a) Pyramidal controls involuntary movements; extrapyramidal controls voluntary fine movements
b) Pyramidal originates in brainstem; extrapyramidal originates in cortex
c) Pyramidal directly connects cortex to motor neurons; extrapyramidal modulates movement indirectly
d) Pyramidal is part of basal ganglia; extrapyramidal is part of cortex
Answer & Explanation: The correct answer is c) Pyramidal directly connects cortex to motor neurons; extrapyramidal modulates movement indirectly. The pyramidal system (corticospinal and corticobulbar tracts) allows direct voluntary control, especially of distal muscles. Extrapyramidal tracts, including rubrospinal, reticulospinal, vestibulospinal, and tectospinal, influence posture, tone, and reflexive movements indirectly through brainstem and spinal interneurons. Damage to either system produces distinct motor deficits.
Chapter: Central Nervous System; Topic: Limbic System; Subtopic: Reward and Motivation Pathways
Keyword Definitions:
• Reward Center: A group of brain structures that produce pleasure or motivation when stimulated, mainly located in the hypothalamus and limbic system.
• Hypothalamus: A diencephalic structure regulating hunger, thirst, emotion, and endocrine activity.
• Limbic System: A set of interconnected structures involved in emotion, memory, and motivation including hippocampus, amygdala, and cingulate gyrus.
• Dopamine Pathway: Neurotransmitter system responsible for pleasure, motivation, and reinforcement learning.
Lead Question - 2014
Reward center is located in ?
a) Cerebellum
b) Amygdala
c) Hippocampus
d) Hypothalamus
Answer & Explanation: The correct answer is d) Hypothalamus. The reward center, primarily located in the lateral hypothalamus and medial forebrain bundle, produces sensations of pleasure and motivation when stimulated. This dopaminergic pathway connects to the nucleus accumbens and ventral tegmental area (VTA). It plays a major role in reinforcement behavior, addiction, and learning mechanisms.
1) The neurotransmitter most closely associated with the reward system is:
a) Acetylcholine
b) Dopamine
c) Serotonin
d) GABA
Answer & Explanation: The correct answer is b) Dopamine. Dopamine released from the ventral tegmental area (VTA) to the nucleus accumbens reinforces rewarding stimuli. It is crucial in motivation, addiction, and reinforcement learning. Drugs like cocaine and amphetamines increase dopamine levels, leading to strong activation of the brain’s reward system and reinforcing addictive behavior.
2) Which structure forms a part of the mesolimbic dopamine pathway?
a) Substantia nigra
b) Nucleus accumbens
c) Locus coeruleus
d) Red nucleus
Answer & Explanation: The correct answer is b) Nucleus accumbens. The mesolimbic pathway originates in the ventral tegmental area (VTA) and projects to the nucleus accumbens, amygdala, and prefrontal cortex. It mediates reward, pleasure, and addiction-related behavior. Overstimulation of this circuit contributes to substance dependence and motivational disorders.
3) Which brain structure is primarily responsible for fear and aggression?
a) Amygdala
b) Hippocampus
c) Hypothalamus
d) Pons
Answer & Explanation: The correct answer is a) Amygdala. The amygdala, part of the limbic system, regulates emotional responses such as fear, aggression, and anxiety. It works with the hypothalamus and prefrontal cortex to coordinate autonomic and behavioral reactions. Lesions reduce fear responses, whereas hyperactivity contributes to anxiety and panic disorders.
4) A patient with bilateral damage to the hippocampus is most likely to have:
a) Loss of balance
b) Memory impairment
c) Speech difficulty
d) Visual field defect
Answer & Explanation: The correct answer is b) Memory impairment. The hippocampus plays a central role in converting short-term memories into long-term storage. Bilateral damage, as seen in conditions like Alzheimer’s disease or hypoxic injury, causes anterograde amnesia. However, previously stored memories may remain intact, highlighting its selective role in memory consolidation.
5) Which of the following best describes the function of the limbic system?
a) Coordination of voluntary movements
b) Regulation of emotion and behavior
c) Control of reflexes
d) Maintenance of balance
Answer & Explanation: The correct answer is b) Regulation of emotion and behavior. The limbic system integrates emotional experiences with behavior, motivation, and memory. Structures such as the amygdala, hippocampus, and hypothalamus work together to influence emotional learning, decision-making, and social responses through neurotransmitters like dopamine and serotonin.
6) A 28-year-old man experiences euphoria and pleasure after drug use. Which neural pathway is primarily activated?
a) Mesocortical pathway
b) Mesolimbic pathway
c) Nigrostriatal pathway
d) Tuberoinfundibular pathway
Answer & Explanation: The correct answer is b) Mesolimbic pathway. This dopaminergic pathway originates in the ventral tegmental area (VTA) and projects to the nucleus accumbens. Activation produces sensations of pleasure and reinforcement. Chronic overstimulation by addictive substances leads to tolerance and craving through neural adaptation mechanisms.
7) Which part of the hypothalamus is associated with satiety?
a) Lateral hypothalamus
b) Ventromedial hypothalamus
c) Posterior hypothalamus
d) Anterior hypothalamus
Answer & Explanation: The correct answer is b) Ventromedial hypothalamus. The ventromedial nucleus acts as the satiety center, inhibiting feeding behavior. Lesions cause hyperphagia and obesity, whereas stimulation reduces appetite. The lateral hypothalamus, in contrast, functions as the feeding center and is involved in reward-driven eating behavior.
8) Which neurotransmitter imbalance is implicated in depression?
a) Increased dopamine
b) Decreased serotonin and norepinephrine
c) Increased GABA
d) Increased glutamate
Answer & Explanation: The correct answer is b) Decreased serotonin and norepinephrine. Depression is associated with reduced levels of serotonin, norepinephrine, and dopamine. Antidepressants like SSRIs increase serotonin levels in synaptic clefts, improving mood regulation. These monoamines are integral to limbic system circuits controlling emotion, sleep, and motivation.
9) A 45-year-old man with Parkinson’s disease has reduced activity in which dopaminergic pathway?
a) Mesolimbic
b) Mesocortical
c) Nigrostriatal
d) Tuberoinfundibular
Answer & Explanation: The correct answer is c) Nigrostriatal. The nigrostriatal pathway connects the substantia nigra to the striatum. Degeneration of dopaminergic neurons in this pathway causes motor symptoms of Parkinson’s disease, including tremor, rigidity, and bradykinesia. Dopamine replacement therapy helps restore function and reduce symptoms.
10) A 50-year-old woman with temporal lobe epilepsy experiences intense emotional auras before seizures. Which structure is involved?
a) Amygdala
b) Thalamus
c) Hypothalamus
d) Medulla
Answer & Explanation: The correct answer is a) Amygdala. The amygdala, located in the temporal lobe, is a major site of epileptic discharges causing emotional experiences such as fear or déjà vu. It connects with the hippocampus and hypothalamus, linking emotional processing to autonomic responses during seizure activity.
Chapter: Sensory Physiology; Topic: Special Senses; Subtopic: Taste Sensation (Gustation)
Keyword Definitions:
• Umami: One of the five basic taste sensations, producing a savory flavor mainly due to amino acids like glutamate.
• Taste Buds: Specialized sensory structures containing gustatory cells that detect different taste modalities.
• Gustatory Pathway: Neural pathway transmitting taste sensations via cranial nerves VII, IX, and X to the gustatory cortex.
• Glutamic Acid: An amino acid that activates umami receptors, enhancing the flavor of protein-rich foods.
Lead Question - 2014
Umami taste is evoked by ?
a) Glucose
b) Glutamic acid
c) Quinine
d) Sodium chloride
Answer & Explanation: The correct answer is b) Glutamic acid. The umami taste is produced by monosodium glutamate (MSG) and certain amino acids such as glutamic acid. It represents the savory taste associated with protein-rich foods. Umami receptors (mGluR4 and T1R1/T1R3) detect these compounds and transmit signals via the glossopharyngeal and facial nerves to the gustatory cortex.
1) Which papillae of the tongue contain the maximum number of taste buds?
a) Fungiform
b) Vallate
c) Foliate
d) Filiform
Answer & Explanation: The correct answer is b) Vallate. Vallate papillae are large structures located in a V-shape at the posterior tongue. Each papilla contains hundreds of taste buds and plays a vital role in detecting bitter and umami tastes. Filiform papillae, in contrast, are numerous but non-gustatory, serving mainly mechanical functions during chewing and swallowing.
2) Sweet taste is mediated through which receptor mechanism?
a) Direct ion channel
b) G-protein coupled receptor
c) Ligand-gated ion channel
d) Voltage-gated sodium channel
Answer & Explanation: The correct answer is b) G-protein coupled receptor. Sweet taste perception involves G-protein coupled receptors (T1R2 and T1R3). When sugars or artificial sweeteners bind, second messenger pathways activate gustatory nerves. This mechanism differs from salty or sour tastes, which rely on direct ion channels for sodium and hydrogen ions, respectively.
3) Which taste sensation is most sensitive to low threshold levels?
a) Sweet
b) Bitter
c) Salty
d) Umami
Answer & Explanation: The correct answer is b) Bitter. Bitter taste receptors are highly sensitive, enabling the detection of potentially toxic alkaloids at minimal concentrations. This protective mechanism triggers aversive responses, reducing ingestion of harmful substances. Bitter taste receptors (T2Rs) are predominantly located in the posterior tongue within the vallate papillae.
4) A patient with glossopharyngeal nerve injury would lose taste sensation mainly from:
a) Anterior two-thirds of the tongue
b) Posterior one-third of the tongue
c) Tip of the tongue
d) Palate
Answer & Explanation: The correct answer is b) Posterior one-third of the tongue. The glossopharyngeal nerve (cranial nerve IX) supplies taste fibers to the posterior one-third of the tongue. Its damage leads to loss of bitter and umami taste perception in that region. The chorda tympani branch of the facial nerve carries taste from the anterior tongue.
5) Which ion is primarily responsible for the perception of salty taste?
a) Sodium ion
b) Potassium ion
c) Calcium ion
d) Chloride ion
Answer & Explanation: The correct answer is a) Sodium ion. Salty taste is mainly due to the entry of sodium ions through epithelial sodium channels (ENaCs) in taste cells. This depolarizes the gustatory membrane, leading to neurotransmitter release. Thus, sodium concentration directly correlates with the intensity of perceived saltiness in foods.
6) A 40-year-old man complains of complete loss of taste following Bell’s palsy. The likely site of lesion is:
a) Facial nerve proximal to chorda tympani
b) Vagus nerve
c) Glossopharyngeal nerve
d) Hypoglossal nerve
Answer & Explanation: The correct answer is a) Facial nerve proximal to chorda tympani. The chorda tympani branch of the facial nerve conveys taste from the anterior two-thirds of the tongue. Damage near its origin leads to ageusia (loss of taste). Bell’s palsy involves facial nerve paralysis, which may also impair salivation and lacrimation functions.
7) Which cranial nerves are involved in carrying taste sensations?
a) VII, IX, X
b) V, VII, IX
c) IX, X, XI
d) V, IX, XII
Answer & Explanation: The correct answer is a) VII, IX, X. The facial nerve (VII) carries taste from the anterior tongue, the glossopharyngeal (IX) from the posterior tongue, and the vagus (X) from the epiglottis and pharynx. These fibers converge at the nucleus tractus solitarius before projecting to the gustatory cortex for taste perception.
8) In umami taste perception, which receptor pair is activated?
a) T1R1 and T1R3
b) T2R1 and T2R2
c) T1R2 and T1R3
d) T1R1 and T2R3
Answer & Explanation: The correct answer is a) T1R1 and T1R3. These metabotropic receptors are sensitive to amino acids like glutamate. When activated, they trigger G-protein (gustducin)-mediated pathways leading to neurotransmitter release. This signaling mechanism forms the molecular basis for umami taste perception, especially from protein-rich foods such as meat and cheese.
9) A patient taking cisplatin chemotherapy develops loss of taste. The likely cause is:
a) Damage to taste buds
b) Increased salivary secretion
c) Enhanced nerve conduction
d) Nasal congestion
Answer & Explanation: The correct answer is a) Damage to taste buds. Cisplatin and other chemotherapeutic drugs cause cytotoxic injury to rapidly regenerating taste bud cells, leading to dysgeusia or ageusia. Recovery occurs after regeneration of gustatory cells. The effect highlights the dependence of taste function on healthy epithelial renewal in the oral cavity.
10) The central taste pathway terminates in which cortical region?
a) Occipital cortex
b) Insular cortex
c) Temporal cortex
d) Parietal association cortex
Answer & Explanation: The correct answer is b) Insular cortex. The gustatory pathway ends in the insula and frontal operculum. These regions integrate taste with olfactory and somatosensory inputs to form flavor perception. Lesions here impair the conscious recognition of taste, even when peripheral taste receptor function remains intact.
Chapter: Respiratory Physiology Topic: Neural Regulation of Respiration Subtopic: Reflex Control of Breathing
Keyword Definitions:
• Hering-Breuer Reflex: A protective reflex that prevents overinflation of the lungs during inspiration by stimulating stretch receptors in the lungs.
• Inspiration: The process of air entering the lungs due to diaphragmatic and intercostal muscle contraction.
• Expiration: The passive process of air leaving the lungs due to elastic recoil of the lungs and chest wall.
• Stretch Receptors: Mechanoreceptors located in the smooth muscle of airways sensitive to lung inflation.
• Vagus Nerve: Cranial nerve X, responsible for transmitting sensory input from lungs to the respiratory center.
Lead Question – 2014 Hering-Breuer reflex is an increase in?
a) Duration of inspiration
b) Duration of expiration
c) Depth of inspiration
d) Depth of expiration
Answer & Explanation: Answer: b) Duration of expiration
The Hering-Breuer inflation reflex inhibits inspiration when the lungs are overstretched, preventing overinflation. Pulmonary stretch receptors send signals via the vagus nerve to the medulla, stopping inspiratory impulses. This reflex lengthens expiration, allowing the lungs to deflate before the next inspiration begins. It maintains rhythmic breathing and protects alveolar structure.
1) Which nerve mediates the Hering-Breuer reflex?
a) Glossopharyngeal nerve
b) Vagus nerve
c) Phrenic nerve
d) Intercostal nerve
Answer & Explanation: Answer: b) Vagus nerve
The vagus nerve carries afferent impulses from stretch receptors in lung tissue to the medullary respiratory center. It plays a vital role in terminating inspiration and initiating expiration during the Hering-Breuer reflex, thereby preventing excessive lung inflation and maintaining normal tidal volume rhythmically.
2) In which condition is the Hering-Breuer reflex more prominent?
a) During quiet breathing
b) During deep breathing
c) During sleep
d) During apnea
Answer & Explanation: Answer: b) During deep breathing
The reflex becomes more active during deep or forceful inspiration, as lung inflation stretches the alveoli more significantly. The activation of pulmonary stretch receptors at high lung volumes triggers the vagus nerve, halting inspiration and allowing expiration. In quiet breathing, its role is minimal.
3) The Hering-Breuer reflex mainly functions to:
a) Increase respiratory rate
b) Prevent alveolar collapse
c) Prevent overinflation of lungs
d) Stimulate chemoreceptors
Answer & Explanation: Answer: c) Prevent overinflation of lungs
The reflex serves as a protective mechanism to prevent alveolar damage caused by overdistension. It achieves this by terminating inspiration through vagal feedback to the medullary centers, ensuring controlled tidal volume and maintaining pulmonary elasticity.
4) Which receptors are involved in the Hering-Breuer reflex?
a) Irritant receptors
b) Chemoreceptors
c) Stretch receptors
d) J receptors
Answer & Explanation: Answer: c) Stretch receptors
Stretch receptors, located in smooth muscles of the bronchi and bronchioles, detect lung inflation. Their activation stimulates afferent vagal fibers that inhibit inspiratory neurons, ending inspiration and extending expiration. They play a crucial role in maintaining ventilatory control under high tidal volumes.
5) Which part of the brain integrates the Hering-Breuer reflex?
a) Pons
b) Medulla oblongata
c) Midbrain
d) Cerebellum
Answer & Explanation: Answer: b) Medulla oblongata
The medulla houses the dorsal respiratory group (DRG) that integrates vagal input from pulmonary stretch receptors. This feedback inhibits inspiratory signals, ensuring expiration follows after sufficient inflation. Thus, medullary centers orchestrate the rhythmic pattern of breathing via reflex feedback.
6) A patient on mechanical ventilation exhibits prolonged expiration; this is due to activation of:
a) J receptors
b) Pulmonary stretch receptors
c) Carotid body receptors
d) Central chemoreceptors
Answer & Explanation: Answer: b) Pulmonary stretch receptors
During mechanical ventilation, the lungs are inflated more than normal, activating pulmonary stretch receptors. These receptors send inhibitory impulses via the vagus nerve to suppress inspiration, causing prolonged expiration as part of the Hering-Breuer inflation reflex.
7) In which situation would the Hering-Breuer reflex be suppressed?
a) Bilateral vagotomy
b) Increased tidal volume
c) Hypercapnia
d) Pulmonary edema
Answer & Explanation: Answer: a) Bilateral vagotomy
The reflex depends on intact vagal afferent fibers. Bilateral vagotomy disrupts vagal transmission from lung stretch receptors, eliminating the reflex, resulting in prolonged inspiration and irregular breathing patterns. It highlights the crucial role of vagal feedback in breathing regulation.
8) Which receptor type mediates the opposite reflex — promoting inspiration after deflation?
a) Stretch receptors
b) Deflation receptors
c) J receptors
d) Baroreceptors
Answer & Explanation: Answer: b) Deflation receptors
Deflation receptors are stimulated during lung collapse, sending excitatory impulses to promote inspiration. This reflex ensures that breathing resumes after expiration, maintaining cyclic ventilation. It complements the Hering-Breuer reflex by balancing inflation and deflation cycles.
9) In neonates, the Hering-Breuer reflex:
a) Is absent
b) Helps establish rhythmic breathing
c) Causes apnea
d) Inhibits expiratory neurons
Answer & Explanation: Answer: b) Helps establish rhythmic breathing
In newborns, the reflex plays a vital role in stabilizing breathing patterns by regulating the depth and duration of inspiration. It prevents overexpansion of immature alveoli and supports the development of rhythmic ventilation cycles crucial for survival.
10) A patient with COPD shows diminished Hering-Breuer reflex due to:
a) Decreased lung compliance
b) Increased stretch receptor sensitivity
c) Increased vagal tone
d) Enhanced elastic recoil
Answer & Explanation: Answer: a) Decreased lung compliance
In COPD, chronic airway obstruction and loss of elastic recoil reduce lung compliance. As a result, pulmonary stretch receptors are less stimulated, weakening the Hering-Breuer reflex. This contributes to abnormal respiratory rhythm and inefficient ventilation observed in chronic obstructive diseases.
Chapter: Central Nervous System; Topic: Neurotransmission in Cardiovascular Regulation; Subtopic: Nucleus Tractus Solitarius (NTS) Neurotransmitters
Keyword Definitions:
Nucleus Tractus Solitarius (NTS): A key brainstem nucleus that receives afferent signals from baroreceptors and chemoreceptors, regulating cardiovascular and respiratory reflexes.
Neurotransmitter: A chemical messenger that transmits signals between neurons across synapses.
Glutamate: The principal excitatory neurotransmitter in the central nervous system, involved in most synaptic transmission processes.
Afferent Fibers: Nerve fibers that carry sensory information from the periphery to the central nervous system.
Cardiovascular Regulation: The process by which the body maintains blood pressure and heart rate through neural and hormonal mechanisms.
Lead Question – 2014
Major neurotransmitter in afferents in nucleus tractus solitarius to regulate cardiovascular system?
a) Serotonin
b) Glutamate
c) Glycine
d) Norepinephrine
Explanation: The major neurotransmitter in afferent fibers terminating in the nucleus tractus solitarius (NTS) is Glutamate. It acts as the primary excitatory neurotransmitter, mediating baroreceptor and chemoreceptor reflexes to regulate cardiovascular functions. Activation of glutamate receptors in the NTS results in changes in heart rate and blood pressure by modulating autonomic output. Thus, the correct answer is Glutamate (b).
1) Which neurotransmitter mediates excitatory synaptic transmission in the central nervous system?
a) GABA
b) Glutamate
c) Dopamine
d) Serotonin
Explanation: The correct answer is Glutamate. It is the main excitatory neurotransmitter in the CNS, playing a major role in learning, memory, and synaptic plasticity. It acts on NMDA, AMPA, and kainate receptors to propagate excitatory signals throughout neuronal circuits, crucial for brain function and cardiovascular regulation.
2) The inhibitory neurotransmitter responsible for reducing neuronal excitability in the brain is:
a) Glycine
b) Glutamate
c) GABA
d) Dopamine
Explanation: The answer is GABA. Gamma-Aminobutyric Acid is the main inhibitory neurotransmitter in the CNS. It acts through GABA-A and GABA-B receptors to reduce neuronal excitability, counterbalancing glutamatergic excitation, thereby maintaining CNS stability and preventing overactivation.
3) A patient with baroreceptor dysfunction may have an abnormality in which brainstem nucleus?
a) Nucleus ambiguus
b) Nucleus tractus solitarius
c) Red nucleus
d) Substantia nigra
Explanation: The correct answer is Nucleus tractus solitarius (NTS). It is the main center integrating sensory input from baroreceptors and chemoreceptors, thereby maintaining arterial pressure. Damage to NTS disrupts cardiovascular reflexes, causing blood pressure instability and altered autonomic responses.
4) Which of the following neurotransmitters is excitatory and acts primarily on NMDA receptors?
a) Glutamate
b) Glycine
c) GABA
d) Serotonin
Explanation: The correct answer is Glutamate. It acts on NMDA, AMPA, and kainate receptors. NMDA receptor activation plays a vital role in learning and memory by allowing calcium influx, critical for long-term potentiation and synaptic strengthening, especially in the brainstem and cortex.
5) A lesion of the Nucleus Tractus Solitarius affects which reflex the most?
a) Corneal reflex
b) Baroreceptor reflex
c) Pupillary reflex
d) Cough reflex
Explanation: The correct answer is Baroreceptor reflex. NTS is the central termination site of baroreceptor afferents from the carotid sinus and aortic arch. Damage leads to impaired reflex control of blood pressure and heart rate, causing labile hypertension or bradycardia.
6) A patient presents with labile blood pressure and impaired vagal tone. Dysfunction of which neurotransmitter in NTS is likely?
a) Glutamate
b) Dopamine
c) Acetylcholine
d) GABA
Explanation: The answer is Glutamate. Its release in NTS is critical for initiating reflex control of heart rate and blood pressure. Disruption in glutamatergic transmission diminishes baroreflex sensitivity, resulting in unstable cardiovascular responses and reduced parasympathetic activity.
7) Which neurotransmitter is co-released with norepinephrine in sympathetic postganglionic neurons?
a) Neuropeptide Y
b) Glutamate
c) Dopamine
d) Serotonin
Explanation: The correct answer is Neuropeptide Y. It is often co-released with norepinephrine from sympathetic terminals, enhancing vasoconstriction and contributing to long-lasting effects on blood vessels, complementing rapid adrenergic action during cardiovascular stress responses.
8) Which enzyme is essential for the synthesis of glutamate from α-ketoglutarate?
a) Glutamate dehydrogenase
b) Glutaminase
c) Glutamine synthetase
d) Monoamine oxidase
Explanation: The correct answer is Glutamate dehydrogenase. It catalyzes the reversible conversion between α-ketoglutarate and glutamate, linking amino acid metabolism with the Krebs cycle, thus maintaining neurotransmitter balance essential for neuronal excitability and synaptic transmission.
9) Clinical Case: A patient with acute brainstem ischemia develops severe bradycardia. Which neurotransmitter’s function in NTS is compromised?
a) Glutamate
b) Acetylcholine
c) GABA
d) Serotonin
Explanation: The correct answer is Glutamate. Brainstem ischemia can impair glutamatergic signaling in NTS, disrupting baroreceptor reflexes and causing autonomic imbalance. This leads to bradycardia and hypotension due to inadequate excitatory transmission to vagal efferents.
10) Clinical Case: A hypertensive patient with impaired baroreflex sensitivity shows reduced glutamate activity in NTS. Which response is expected?
a) Stable heart rate
b) Increased vagal output
c) Unstable blood pressure
d) Decreased sympathetic activity
Explanation: The correct answer is Unstable blood pressure. Reduced glutamatergic neurotransmission in NTS diminishes baroreflex control, impairing the buffering of blood pressure fluctuations. This results in autonomic instability with variable heart rate and pressure control failure, typical in chronic hypertension.
Chapter: Cerebral Circulation and Intracranial Dynamics; Topic: Blood Supply to the Brain; Subtopic: Cerebral Blood Flow (CBF) and Regulation
Keyword Definitions:
Cerebral Blood Flow (CBF): The volume of blood passing through 100 g of brain tissue per minute, normally about 50–55 ml/100 g/min.
Autoregulation: The brain’s ability to maintain constant blood flow despite changes in mean arterial pressure between 60–160 mmHg.
Circle of Willis: Arterial circle at the brain base ensuring collateral circulation between internal carotid and vertebral arteries.
Ischemia: Inadequate blood supply leading to tissue hypoxia and neuronal injury.
CO₂ sensitivity: CBF rises by 2–4% for every 1 mmHg increase in arterial CO₂ due to vasodilation.
Lead Question – 2014
Blood supply of brain is ?
a) 1500 ml/min
b) 2000 ml/min
c) 750 ml/min
d) 250 ml/min
Answer & Explanation: (c) 750 ml/min.
Cerebral blood flow averages about 750 ml/min, constituting nearly 15% of cardiac output. This flow ensures adequate oxygen and glucose delivery to brain tissue. It is tightly regulated by autoregulatory mechanisms responding to CO₂, O₂, and mean arterial pressure. Any disruption, such as hypoxia or ischemia, can impair neuronal activity and cause irreversible damage.
1. Normal cerebral blood flow per 100 g of brain tissue is:
a) 10 ml/min
b) 25 ml/min
c) 50 ml/min
d) 100 ml/min
Answer & Explanation: (c) 50 ml/min. The brain requires a continuous blood supply of 50 ml/100 g/min to meet its metabolic demands. Gray matter receives more blood than white matter due to higher neuronal activity. A fall below 20 ml/100 g/min can impair neuronal function, while values under 10 ml/100 g/min cause irreversible neuronal death.
2. Which artery supplies the visual cortex?
a) Anterior cerebral artery
b) Middle cerebral artery
c) Posterior cerebral artery
d) Basilar artery
Answer & Explanation: (c) Posterior cerebral artery. The visual cortex, located in the occipital lobe, is supplied by the posterior cerebral artery, a branch of the basilar artery. Occlusion of this artery causes contralateral homonymous hemianopia due to loss of visual field from both eyes corresponding to the affected hemisphere.
3. The Circle of Willis is formed by all except:
a) Anterior communicating artery
b) Posterior communicating artery
c) Internal carotid artery
d) External carotid artery
Answer & Explanation: (d) External carotid artery. The Circle of Willis provides collateral circulation between anterior and posterior cerebral systems, comprising internal carotid, anterior, middle, posterior cerebral arteries, and communicating branches. The external carotid supplies extracranial structures, not intracranial circulation.
4. Increased PaCO₂ causes cerebral:
a) Vasoconstriction
b) Vasodilation
c) Ischemia
d) No change
Answer & Explanation: (b) Vasodilation. Elevated CO₂ levels increase hydrogen ion concentration in the cerebrospinal fluid, leading to relaxation of cerebral arterioles. This enhances blood flow to maintain pH homeostasis. Conversely, hyperventilation reduces CO₂, causing vasoconstriction and lowering intracranial pressure temporarily in brain edema management.
5. Decreased cerebral blood flow is caused by:
a) Hypercapnia
b) Hypocapnia
c) Acidosis
d) Increased arterial CO₂
Answer & Explanation: (b) Hypocapnia. Low arterial CO₂ due to hyperventilation induces cerebral vasoconstriction, reducing blood flow and intracranial pressure. Although this can relieve pressure temporarily in head injury, prolonged hypocapnia may reduce oxygen delivery and aggravate ischemic neuronal damage.
6. Clinical-type: A patient with head injury shows reduced cerebral perfusion despite normal systemic BP. Likely cause:
a) Cerebral vasodilation
b) Loss of autoregulation
c) Hypoxia
d) Hypercapnia
Answer & Explanation: (b) Loss of autoregulation. Brain injury impairs autoregulatory mechanisms that maintain constant flow, making CBF pressure-dependent. Even normal blood pressure may not suffice to maintain perfusion. This leads to ischemic zones, explaining why cerebral perfusion pressure is closely monitored in neurosurgical patients.
7. Clinical-type: A 50-year-old hypertensive man develops sudden weakness on the right side. Likely artery affected:
a) Anterior cerebral artery
b) Middle cerebral artery
c) Posterior cerebral artery
d) Basilar artery
Answer & Explanation: (b) Middle cerebral artery. It supplies the motor and sensory cortex for the face and upper limb. Infarction leads to contralateral hemiplegia (face and arm), aphasia if the dominant hemisphere is affected, and sensory deficits. It is the most common site of cerebral infarction.
8. Clinical-type: A patient develops coma due to global hypoxia. Which area of the brain is most vulnerable?
a) Cerebellum
b) Hippocampus
c) Medulla
d) Hypothalamus
Answer & Explanation: (b) Hippocampus. The hippocampus is highly sensitive to hypoxia and ischemia due to its high metabolic activity. Neurons here undergo early necrosis in hypoxic conditions, explaining memory impairment and altered consciousness in global cerebral ischemia.
9. Clinical-type: During carotid endarterectomy, blood flow through the ipsilateral hemisphere is maintained via:
a) Anterior communicating artery
b) Posterior communicating artery
c) External carotid branches
d) Vertebral artery
Answer & Explanation: (a) Anterior communicating artery. It connects the anterior cerebral arteries from both sides, providing collateral flow if one internal carotid is occluded. Adequate Circle of Willis integrity ensures uninterrupted perfusion during vascular surgeries like carotid endarterectomy.
10. Clinical-type: A 60-year-old with chronic COPD develops confusion and drowsiness. Cause is likely:
a) Hypercapnia causing cerebral vasoconstriction
b) Hypocapnia causing vasoconstriction
c) Hypercapnia causing vasodilation and increased ICP
d) Hypoxia causing vasoconstriction
Answer & Explanation: (c) Hypercapnia causing vasodilation and increased ICP. Chronic CO₂ retention leads to cerebral vasodilation, increasing intracranial pressure and reducing neuronal function, resulting in confusion or CO₂ narcosis. This underscores the importance of controlled oxygen therapy in COPD patients to prevent respiratory drive suppression.
Chapter: Respiratory Physiology; Topic: Neural Control of Respiration; Subtopic: Respiratory Centers in the Medulla and Pons
Keyword Definitions:
• Pre-Botzinger Complex: Group of neurons in the medulla responsible for generating rhythmic breathing.
• Dorsal Respiratory Group (DRG): Controls inspiration and integrates sensory input.
• Ventral Respiratory Group (VRG): Controls forced expiration and is inactive during quiet breathing.
• Pneumotaxic Center: Located in pons; regulates rate and depth of respiration.
• Apneustic Center: Stimulates prolonged inspiration when unchecked.
Lead Question (2014):
Which of the following are inactive during normal respiration?
a) Pre-Botzinger complex
b) Dorsal group of neurons
c) Ventral VRG group of neurons
d) Pneumotaxic center
Answer & Explanation:
Answer: c) Ventral VRG group of neurons. The ventral respiratory group (VRG) is mainly involved in forced expiration and accessory inspiration. During quiet or normal breathing, expiration is passive due to elastic recoil, making the VRG inactive. Only the dorsal group and Pre-Botzinger complex remain active to maintain rhythmic inspiration.
1. The Pre-Botzinger complex is primarily responsible for:
a) Controlling voluntary breath-holding
b) Rhythmic generation of breathing
c) Detecting oxygen levels
d) Activating expiratory muscles
Answer & Explanation:
Answer: b) Rhythmic generation of breathing. The Pre-Botzinger complex, located in the medulla, acts as the respiratory rhythm generator. It creates spontaneous action potentials that regulate inspiration cycles. Lesions here can cause apnea or irregular respiration, highlighting its importance in automatic control of breathing independent of conscious effort.
2. Which center inhibits inspiration to prevent lung overinflation?
a) Apneustic center
b) Pneumotaxic center
c) Dorsal respiratory group
d) Pre-Botzinger complex
Answer & Explanation:
Answer: b) Pneumotaxic center. The pneumotaxic center, located in the upper pons, limits inspiration by sending inhibitory impulses to the medullary inspiratory center. It promotes rhythmic and controlled breathing. Overactivity of this center causes shallow breathing, while inactivity leads to deep, prolonged inspiration (apneusis).
3. The dorsal respiratory group receives afferent signals from:
a) Chemoreceptors and stretch receptors
b) Baroreceptors only
c) Cortex and hypothalamus
d) Inspiratory muscles
Answer & Explanation:
Answer: a) Chemoreceptors and stretch receptors. The dorsal respiratory group (DRG) receives inputs from peripheral chemoreceptors, baroreceptors, and pulmonary stretch receptors. These signals help modulate the inspiratory drive according to blood gas levels and lung inflation status, ensuring efficient and adaptive respiration.
4. In forced expiration, which neuronal group becomes active?
a) DRG
b) VRG
c) Apneustic center
d) Pneumotaxic center
Answer & Explanation:
Answer: b) VRG. The ventral respiratory group (VRG) activates during forced expiration and inspiration, stimulating accessory muscles such as internal intercostals and abdominal muscles. It remains inactive during quiet breathing since normal expiration relies on passive lung recoil rather than muscular contraction.
5. A patient with brainstem injury affecting the pons shows prolonged inspiration. Which center is damaged?
a) Apneustic center
b) Pneumotaxic center
c) VRG
d) DRG
Answer & Explanation:
Answer: b) Pneumotaxic center. Damage to the pneumotaxic center removes inhibitory control on inspiration, resulting in apneustic breathing characterized by prolonged inspiratory efforts. This indicates the crucial role of pontine centers in balancing inspiration and expiration phases during normal breathing cycles.
6. The Hering-Breuer reflex helps in:
a) Preventing alveolar collapse
b) Preventing lung overinflation
c) Maintaining acid-base balance
d) Increasing CO₂ sensitivity
Answer & Explanation:
Answer: b) Preventing lung overinflation. The Hering-Breuer inflation reflex, mediated by pulmonary stretch receptors, inhibits further inspiration when the lungs are inflated excessively. It protects against overdistension by sending inhibitory signals through the vagus nerve to the dorsal respiratory group, reducing inspiratory drive.
7. In which condition does the VRG show maximal activity?
a) Sleep
b) Forced breathing during exercise
c) Quiet breathing
d) Shallow breathing
Answer & Explanation:
Answer: b) Forced breathing during exercise. During exercise, the ventral respiratory group activates accessory muscles for both inspiration and expiration, enhancing ventilation. It ensures efficient CO₂ clearance and O₂ intake to meet metabolic demands. This activation is absent during quiet respiration, where expiration remains passive.
8. Lesion of Pre-Botzinger complex results in:
a) Apnea
b) Cheyne-Stokes respiration
c) Kussmaul breathing
d) Biot’s respiration
Answer & Explanation:
Answer: a) Apnea. The Pre-Botzinger complex generates the basic rhythm of breathing. Lesions here disrupt rhythmic firing of inspiratory neurons, leading to complete cessation of spontaneous respiration (apnea). This area acts as the “pacemaker” of respiration in the medulla oblongata.
9. Which center provides tonic excitation to inspiratory neurons of DRG?
a) Apneustic center
b) Pneumotaxic center
c) VRG
d) Pre-Botzinger complex
Answer & Explanation:
Answer: a) Apneustic center. The apneustic center located in the lower pons provides continuous stimulatory input to the dorsal respiratory group, promoting prolonged inspiration. It is normally inhibited by the pneumotaxic center to maintain balanced respiratory rhythm and prevent excessive inspiration.
10. During quiet breathing, expiration occurs mainly due to:
a) Contraction of expiratory muscles
b) Elastic recoil of lungs
c) Activation of VRG neurons
d) Inhibition by apneustic center
Answer & Explanation:
Answer: b) Elastic recoil of lungs. During quiet respiration, expiration is a passive process resulting from the elastic recoil of the lungs and chest wall. No active neuronal or muscular effort is required. The VRG neurons remain inactive, resuming activity only during forced expiration or labored breathing.
Chapter: Upper Limb Anatomy; Topic: Cutaneous Nerve Supply of Hand; Subtopic: Ulnar Nerve Distribution
Keyword Definitions:
• Hypothenar Eminence: The fleshy medial prominence of the palm formed by muscles controlling the little finger.
• Ulnar Nerve: A branch of the brachial plexus (C8–T1) that supplies the medial side of the hand and fingers.
• Radial Nerve: Supplies posterior arm, forearm, and dorsum of hand.
• Median Nerve: Supplies lateral palm and first three and a half fingers.
• Cutaneous Innervation: Distribution of sensory nerves to the skin for touch and pain sensation.
Lead Question (2014):
Skin over hypothenar eminence is supplied by?
a) Radial nerve
b) Median nerve
c) Anterior interosseous nerve
d) Ulnar nerve
Answer & Explanation:
Answer: d) Ulnar nerve. The skin over the hypothenar eminence, corresponding to the medial aspect of the palm, is supplied by the palmar branch of the ulnar nerve. This nerve carries sensory fibers from C8–T1 roots and also supplies the little finger and medial half of the ring finger. Injury leads to sensory loss over this region.
1. The ulnar nerve arises from which part of the brachial plexus?
a) Lateral cord
b) Medial cord
c) Posterior cord
d) Upper trunk
Answer & Explanation:
Answer: b) Medial cord. The ulnar nerve arises from the medial cord of the brachial plexus, containing fibers from the C8 and T1 spinal nerves. It runs along the medial side of the arm, passes behind the medial epicondyle, and supplies both motor and sensory fibers to the hand, especially the hypothenar region.
2. The hypothenar muscles are supplied by:
a) Median nerve
b) Radial nerve
c) Ulnar nerve
d) Musculocutaneous nerve
Answer & Explanation:
Answer: c) Ulnar nerve. The hypothenar muscles—abductor digiti minimi, flexor digiti minimi brevis, and opponens digiti minimi—are supplied by the deep branch of the ulnar nerve. These muscles facilitate the movement of the little finger and help in grip. Damage to the ulnar nerve weakens these movements significantly.
3. A patient presents with numbness over the medial one and a half fingers and hypothenar area. Which nerve is likely involved?
a) Median nerve
b) Ulnar nerve
c) Radial nerve
d) Axillary nerve
Answer & Explanation:
Answer: b) Ulnar nerve. Sensory loss over the hypothenar region and medial one and a half fingers is a classic sign of ulnar nerve lesion, commonly at the wrist (Guyon’s canal syndrome) or elbow (cubital tunnel syndrome). Clinical symptoms include paresthesia and weakness of intrinsic hand muscles.
4. Which branch of the ulnar nerve supplies the skin over hypothenar eminence?
a) Deep branch
b) Palmar cutaneous branch
c) Dorsal cutaneous branch
d) Superficial branch
Answer & Explanation:
Answer: b) Palmar cutaneous branch. The palmar cutaneous branch of the ulnar nerve arises in the forearm and supplies the medial palm, including the hypothenar eminence. It is unaffected in lesions at the wrist, distinguishing it from other branches like the superficial and deep branches.
5. In cubital tunnel syndrome, the earliest symptom is:
a) Pain over thenar area
b) Loss of wrist extension
c) Tingling in the ring and little fingers
d) Weakness of biceps
Answer & Explanation:
Answer: c) Tingling in the ring and little fingers. Cubital tunnel syndrome results from ulnar nerve compression at the elbow. The earliest symptom is paresthesia in the ring and little fingers, followed by muscle weakness and wasting in the hypothenar region if untreated. Diagnosis is clinical and confirmed by nerve conduction studies.
6. Which muscle is not supplied by the ulnar nerve?
a) Adductor pollicis
b) Flexor carpi ulnaris
c) First lumbrical
d) Abductor digiti minimi
Answer & Explanation:
Answer: c) First lumbrical. The first and second lumbricals are supplied by the median nerve, whereas the third and fourth lumbricals receive innervation from the ulnar nerve. Recognizing this mixed innervation is essential in assessing nerve injury patterns in the palm and finger flexion tests.
7. In Guyon’s canal syndrome, which function is preserved?
a) Abduction of little finger
b) Sensation over dorsum of hand
c) Flexion of distal phalanx of ring finger
d) Adduction of thumb
Answer & Explanation:
Answer: b) Sensation over dorsum of hand. In Guyon’s canal syndrome, only the palmar branches of the ulnar nerve are affected, sparing the dorsal cutaneous branch. Thus, sensation over the dorsal aspect of the medial hand remains intact, helping differentiate this condition from more proximal ulnar nerve lesions.
8. Injury to the ulnar nerve at wrist leads to:
a) Ape thumb deformity
b) Claw hand deformity
c) Wrist drop
d) Benediction sign
Answer & Explanation:
Answer: b) Claw hand deformity. Ulnar nerve injury at the wrist causes loss of lumbricals to the ring and little fingers, resulting in hyperextension at MCP joints and flexion at IP joints—producing the classical “claw hand.” The hypothenar muscles also atrophy, leading to flattening of the medial palm.
9. The dorsal cutaneous branch of the ulnar nerve arises in the:
a) Axilla
b) Mid-forearm
c) Wrist
d) Palm
Answer & Explanation:
Answer: b) Mid-forearm. The dorsal cutaneous branch of the ulnar nerve arises in the middle of the forearm, supplying the skin over the dorsal aspect of the medial hand and fingers. Its integrity helps differentiate between high and low lesions of the ulnar nerve during neurological testing.
10. Which test is used to assess ulnar nerve function clinically?
a) Froment’s sign
b) Phalen’s test
c) Tinel’s sign
d) Finkelstein’s test
Answer & Explanation:
Answer: a) Froment’s sign. Froment’s sign checks ulnar nerve function by asking the patient to grasp a paper between thumb and index finger. If the adductor pollicis (ulnar nerve) is weak, the patient compensates by flexing the thumb using the flexor pollicis longus (median nerve). This indicates ulnar neuropathy.
Chapter: Anatomy; Topic: Gastrointestinal System; Subtopic: Enteric Nervous System
Keyword Definitions:
Submucosal plexus: Also called Meissner’s plexus, located in the submucosa and regulates glandular secretion and blood flow.
Myenteric plexus: Also known as Auerbach’s plexus, lies between muscle layers and controls peristalsis.
Peristalsis: Coordinated contraction and relaxation of intestinal muscles that move food along the digestive tract.
Enteric nervous system: Intrinsic neural network of the gut functioning independently of the central nervous system.
Parasympathetic innervation: Enhances GI secretion and motility through vagal and sacral outflow.
Lead Question - 2014
Submucosal plexus is ?
a) Myenteric plexus
b) Auerbach's plexus
c) Meissner's plexus
d) Tympanic plexus
Explanation: The submucosal plexus, also known as Meissner’s plexus, lies in the submucosa of the gastrointestinal tract. It primarily controls glandular secretions and local blood flow of the mucosa. In contrast, Auerbach’s (myenteric) plexus regulates smooth muscle activity and peristalsis. Together they form the enteric nervous system, functioning semi-autonomously in digestion.
1) Myenteric plexus is located between:
a) Mucosa and submucosa
b) Circular and longitudinal muscle layers
c) Serosa and muscularis externa
d) Submucosa and serosa
Explanation: The myenteric (Auerbach’s) plexus lies between the circular and longitudinal muscle layers of the gut wall. It coordinates peristaltic activity, controlling rhythmic contractions of smooth muscles. Hence, the correct answer is Circular and longitudinal muscle layers. This plexus is essential for propulsive motility of the gastrointestinal tract and smooth coordination of digestive movements.
2) Meissner’s plexus controls mainly:
a) GI motility
b) Glandular secretion and local blood flow
c) Gastric emptying
d) Defecation reflex
Explanation: Meissner’s plexus, located in the submucosa, regulates glandular secretion and local mucosal blood flow. It doesn’t directly control motility, which is governed by the myenteric plexus. Hence, the correct answer is Glandular secretion and local blood flow. It ensures proper lubrication and enzymatic activity during digestion by modulating secretion rates and mucosal perfusion.
3) In Hirschsprung’s disease, which plexus is absent?
a) Myenteric plexus
b) Submucosal plexus
c) Both Auerbach’s and Meissner’s plexus
d) None of the above
Explanation: Hirschsprung’s disease (congenital aganglionic megacolon) results from the absence of both Auerbach’s and Meissner’s plexuses in affected bowel segments. This leads to a lack of peristalsis, causing functional obstruction and dilation of the proximal colon. Neural crest migration failure during embryogenesis underlies this pathology, leading to chronic constipation and abdominal distension.
4) Which of the following is part of the enteric nervous system?
a) Myenteric plexus
b) Submucosal plexus
c) Both a and b
d) None of the above
Explanation: The enteric nervous system consists of two interconnected plexuses: the myenteric (Auerbach’s) and submucosal (Meissner’s) plexuses. Together, they regulate motility, secretion, and blood flow of the gastrointestinal tract. Hence, the correct answer is Both a and b. They function semi-independently under modulation by parasympathetic and sympathetic inputs, often termed the “second brain” of the gut.
5) Which neurotransmitter is mainly involved in GI peristalsis?
a) Dopamine
b) Acetylcholine
c) Serotonin
d) GABA
Explanation: Acetylcholine is the principal neurotransmitter promoting gastrointestinal smooth muscle contraction and peristalsis. It enhances coordinated contractions mediated by the myenteric plexus. In contrast, sympathetic stimulation inhibits motility. Acetylcholine’s excitatory role ensures proper mixing and propulsion of food, a vital function controlled by enteric and vagal nerve coordination in digestion.
6) A patient with bowel obstruction shows loss of peristalsis. Which plexus is most affected?
a) Meissner’s plexus
b) Myenteric plexus
c) Both
d) None
Explanation: The myenteric (Auerbach’s) plexus is primarily responsible for peristalsis. Damage to this plexus impairs smooth muscle coordination, resulting in paralysis of gut motility. Meissner’s plexus affects secretion rather than movement. Hence, the correct answer is Myenteric plexus. Restoration of peristalsis depends on the integrity of enteric neuronal circuits and their cholinergic transmission efficiency.
7) Which of the following is true about Meissner’s plexus?
a) Lies between two muscle layers
b) Controls secretion and mucosal blood flow
c) Responsible for rhythmic peristalsis
d) Absent in colon
Explanation: Meissner’s plexus lies in the submucosa, not between muscle layers. It regulates mucosal gland secretion and local blood flow. Hence, the correct answer is Controls secretion and mucosal blood flow. It is present throughout the intestine and functions in coordination with the myenteric plexus to ensure optimal digestive enzyme release and mucosal lubrication.
8) A 3-year-old child presents with chronic constipation and distended abdomen. Which of the following is defective?
a) Meissner’s plexus
b) Auerbach’s plexus
c) Both Meissner’s and Auerbach’s plexuses
d) Muscular layer only
Explanation: In congenital aganglionic megacolon (Hirschsprung’s disease), both Meissner’s and Auerbach’s plexuses are absent. Hence, the correct answer is Both Meissner’s and Auerbach’s plexuses. Lack of ganglion cells leads to tonic contraction and functional obstruction. Surgical resection of the aganglionic segment restores normal bowel function in such cases.
9) Which region of the GI tract has the most prominent Meissner’s plexus?
a) Esophagus
b) Stomach
c) Small intestine
d) Rectum
Explanation: The small intestine has a well-developed Meissner’s plexus because of its high secretory and absorptive functions. It regulates secretions from Brunner’s glands and intestinal crypts, facilitating digestion. In contrast, the esophagus has minimal submucosal plexus development. Thus, Meissner’s plexus activity parallels the metabolic demand of mucosal secretion.
10) Injury to the enteric nervous system may lead to:
a) Hypersecretion of gastric acid
b) Loss of coordinated peristalsis
c) Increased intestinal absorption
d) Enhanced mucosal immunity
Explanation: Damage to the enteric nervous system, including Auerbach’s and Meissner’s plexuses, impairs neural regulation of motility and secretion. The most significant effect is loss of coordinated peristalsis. This results in intestinal stasis, distension, and malabsorption. Restoration requires functional neuronal circuits and intact parasympathetic regulation for normal gastrointestinal propulsion.
Chapter: Neuroanatomy; Topic: Brainstem; Subtopic: Developmental Origin of Medulla Oblongata
Keyword Definitions:
Medulla oblongata: The lower part of the brainstem responsible for autonomic functions like respiration, heart rate, and blood pressure.
Myelencephalon: The posterior part of the embryonic hindbrain that gives rise to the medulla oblongata.
Brainstem: The central trunk of the brain connecting the cerebrum with the spinal cord.
Embryonic vesicles: Developmental brain regions — prosencephalon, mesencephalon, and rhombencephalon — that differentiate into various brain structures.
Lead Question - 2014
Medulla oblongata is derived from ?
a) Telencephalon
b) Diencephalon
c) Mesencephalon
d) Myelencephalon
Explanation: The medulla oblongata develops from the myelencephalon, the caudal part of the rhombencephalon (hindbrain). During embryonic development, the rhombencephalon divides into the metencephalon (forming the pons and cerebellum) and the myelencephalon (forming the medulla). The medulla controls vital autonomic functions like heart rate, respiration, and reflexes such as coughing and vomiting.
1) The pons is derived from ?
a) Metencephalon
b) Myelencephalon
c) Diencephalon
d) Mesencephalon
Explanation: The pons originates from the metencephalon, the upper division of the rhombencephalon. It serves as a communication bridge between the cerebrum and cerebellum. Functionally, it assists in respiratory control and facial sensation. During embryogenesis, neural tube segmentation determines these structures' distinct developmental origins.
2) The midbrain originates from which embryonic vesicle?
a) Prosencephalon
b) Mesencephalon
c) Diencephalon
d) Rhombencephalon
Explanation: The midbrain arises from the mesencephalon. It contains important structures such as the superior and inferior colliculi, red nucleus, and substantia nigra. These regions regulate visual and auditory reflexes and coordinate motor control, linking sensory input to motor output through cranial nerve nuclei and tracts.
3) Clinical-type: A patient with lesions in the medulla shows respiratory arrest. Which embryonic structure was primarily affected?
a) Telencephalon
b) Myelencephalon
c) Mesencephalon
d) Metencephalon
Explanation: The myelencephalon forms the medulla oblongata, housing the respiratory and cardiac centers. Lesions here result in life-threatening respiratory and cardiovascular dysfunction. The medulla integrates autonomic signals and transmits them via cranial nerves IX, X, XI, and XII to maintain homeostasis.
4) The cerebellum develops from ?
a) Metencephalon
b) Myelencephalon
c) Mesencephalon
d) Diencephalon
Explanation: The cerebellum arises from the metencephalon during neural tube development. It plays a crucial role in balance, posture, and coordination. The cerebellar hemispheres form from dorsal outgrowths of the metencephalon, developing the vermis and cerebellar cortex for fine motor control.
5) Clinical-type: Damage to the olive of medulla affects which function?
a) Auditory processing
b) Coordination
c) Taste
d) Smell
Explanation: The inferior olivary nucleus in the medulla assists cerebellar coordination by relaying sensory and motor information. Lesions disrupt timing and precision of movements, causing ataxia. This nucleus originates embryologically within the myelencephalon, confirming its role in medullary circuitry.
6) The diencephalon gives rise to ?
a) Thalamus and hypothalamus
b) Pons and medulla
c) Cerebral cortex
d) Midbrain
Explanation: The diencephalon forms structures like the thalamus, hypothalamus, epithalamus, and subthalamus. These regulate sensory relay, endocrine control, and autonomic functions. Positioned between the midbrain and telencephalon, it plays a central role in sensory integration and emotional processing via hypothalamic connections.
7) Clinical-type: A newborn with Arnold–Chiari malformation shows medullary herniation. Which embryonic part was affected?
a) Myelencephalon
b) Metencephalon
c) Telencephalon
d) Mesencephalon
Explanation: Arnold–Chiari malformation involves herniation of medulla (derived from myelencephalon) into the foramen magnum, compressing brainstem structures. This congenital defect leads to respiratory and swallowing issues, reflecting the medulla’s role in autonomic and cranial nerve function.
8) Which of the following is not derived from the rhombencephalon?
a) Pons
b) Cerebellum
c) Medulla
d) Midbrain
Explanation: The midbrain develops from the mesencephalon, not the rhombencephalon. The rhombencephalon gives rise to the pons, cerebellum, and medulla, collectively forming the hindbrain responsible for coordination, autonomic control, and reflex activity.
9) Clinical-type: A patient with loss of gag reflex has lesion in which structure derived from myelencephalon?
a) Nucleus ambiguus
b) Red nucleus
c) Substantia nigra
d) Thalamus
Explanation: The nucleus ambiguus in the medulla, derived from the myelencephalon, controls muscles of the pharynx and larynx via cranial nerves IX and X. Lesions impair the gag reflex, swallowing, and phonation. This highlights the medulla’s role in cranial nerve integration.
10) The telencephalon develops into ?
a) Cerebral hemispheres
b) Pons
c) Midbrain
d) Cerebellum
Explanation: The telencephalon forms the cerebral hemispheres, basal ganglia, and olfactory bulbs. These structures handle higher cognitive, emotional, and voluntary motor functions. As the most anterior brain vesicle, it develops from the prosencephalon during week five of embryonic development.
Chapter: Head and Neck Anatomy; Topic: Oral Cavity and Salivary Glands; Subtopic: Submandibular Gland and Duct Relations
Keyword Definitions:
Submandibular Duct: Also known as Wharton’s duct, it opens at the sublingual papilla beside the lingual frenulum and carries saliva from the submandibular gland.
Lingual Nerve: A branch of the mandibular nerve (V3) that supplies general sensation to the anterior two-thirds of the tongue.
Hypoglossal Nerve: The twelfth cranial nerve, motor to intrinsic and extrinsic muscles of the tongue.
Recurrent Laryngeal Nerve: A branch of the vagus nerve that supplies intrinsic laryngeal muscles.
Lead Question (2014):
Nerve which loops around submandibular duct?
a) Mandibular nerve
b) Lingual nerve
c) Hypoglossal nerve
d) Recurrent laryngeal nerve
Explanation: The lingual nerve passes deep to the submandibular duct after looping beneath it. This unique anatomical relationship allows the lingual nerve to carry both general sensation and special taste fibers (via chorda tympani) to the anterior two-thirds of the tongue. Hence, the correct answer is (b) Lingual nerve. Injury during submandibular surgery may cause sensory loss in the tongue. (100 words)
1. The lingual nerve is a branch of which cranial nerve?
a) Facial nerve
b) Glossopharyngeal nerve
c) Trigeminal nerve
d) Hypoglossal nerve
Explanation: The lingual nerve is a branch of the mandibular division of the trigeminal nerve (CN V3). It carries general sensory fibers from the anterior two-thirds of the tongue and is joined by the chorda tympani, which adds taste and parasympathetic fibers. Therefore, the correct answer is (c) Trigeminal nerve. (100 words)
2. Which nerve carries taste fibers from the anterior two-thirds of the tongue?
a) Lingual nerve
b) Glossopharyngeal nerve
c) Chorda tympani
d) Vagus nerve
Explanation: The chorda tympani nerve, a branch of the facial nerve (CN VII), carries taste fibers from the anterior two-thirds of the tongue. It joins the lingual nerve to reach its destination. Thus, though taste fibers travel within the lingual nerve, they originate from the facial nerve. Hence, the correct answer is (c) Chorda tympani. (100 words)
3. The submandibular duct opens into the oral cavity at?
a) Base of tongue
b) Floor of mouth beside lingual frenulum
c) Near tonsillar fossa
d) Soft palate
Explanation: The submandibular duct (Wharton’s duct) opens into the floor of the mouth beside the lingual frenulum at the sublingual papilla. This location allows easy saliva drainage into the oral cavity. The correct answer is (b) Floor of mouth beside lingual frenulum. (100 words)
4. Which nerve is closely related to the submandibular ganglion?
a) Hypoglossal nerve
b) Lingual nerve
c) Facial nerve
d) Glossopharyngeal nerve
Explanation: The submandibular ganglion is suspended from the lingual nerve by small filaments. Preganglionic fibers from the chorda tympani synapse here, and postganglionic fibers supply the submandibular and sublingual glands. Thus, the correct answer is (b) Lingual nerve. (100 words)
5. (Clinical) During surgery of submandibular duct stones, which nerve is at greatest risk of injury?
a) Lingual nerve
b) Hypoglossal nerve
c) Mylohyoid nerve
d) Glossopharyngeal nerve
Explanation: The lingual nerve loops beneath the submandibular duct and is in close relation to it. Surgical removal of ductal calculi (sialolithiasis) may damage the lingual nerve, resulting in loss of general sensation and taste from the anterior two-thirds of the tongue. Thus, the correct answer is (a) Lingual nerve. (100 words)
6. (Clinical) A patient presents with loss of taste and general sensation in the anterior two-thirds of the tongue. The lesion likely involves?
a) Lingual nerve distal to chorda tympani junction
b) Hypoglossal nerve
c) Glossopharyngeal nerve
d) Recurrent laryngeal nerve
Explanation: The lingual nerve distal to the chorda tympani junction carries both general sensory and taste fibers. A lesion at this point causes combined sensory and taste loss in the anterior tongue, while proximal injury affects only general sensation. Therefore, the correct answer is (a) Lingual nerve distal to chorda tympani junction. (100 words)
7. (Clinical) Parasympathetic fibers to the submandibular gland originate from?
a) Facial nerve via chorda tympani
b) Glossopharyngeal nerve via lesser petrosal
c) Vagus nerve
d) Hypoglossal nerve
Explanation: The facial nerve gives parasympathetic fibers through the chorda tympani, which joins the lingual nerve and synapses in the submandibular ganglion. Postganglionic fibers innervate the submandibular and sublingual glands, controlling salivary secretion. Hence, the correct answer is (a) Facial nerve via chorda tympani. (100 words)
8. (Clinical) Hypoglossal nerve supplies which muscles of the tongue?
a) Palatoglossus
b) Styloglossus
c) Hyoglossus
d) Both b and c
Explanation: The hypoglossal nerve (CN XII) supplies all intrinsic and extrinsic muscles of the tongue except palatoglossus (which is supplied by the vagus nerve). Therefore, the correct answer is (d) Both b and c. (100 words)
9. (Clinical) A patient with right hypoglossal nerve palsy shows?
a) Tongue deviation to the left
b) Tongue deviation to the right
c) Loss of taste
d) Paralysis of soft palate
Explanation: In hypoglossal nerve palsy, the tongue deviates toward the side of the lesion due to paralysis of the genioglossus muscle. Over time, atrophy and fasciculations appear on the affected side. Hence, the correct answer is (b) Tongue deviation to the right. (100 words)
10. (Clinical) Which condition results from inflammation of the submandibular gland?
a) Sialolithiasis
b) Parotitis
c) Glossitis
d) Pharyngitis
Explanation: Sialolithiasis is the formation of calculi (stones) within the salivary ducts, commonly the submandibular duct due to its long upward course. It leads to pain and swelling during meals. Chronic obstruction can cause infection (sialadenitis). The correct answer is (a) Sialolithiasis. (100 words)
Chapter: Head and Neck Anatomy; Topic: Cranial Cavity and Meninges; Subtopic: Middle Meningeal Artery and Cranial Foramina
Keyword Definitions:
Middle Meningeal Artery: A branch of the maxillary artery that supplies the dura mater and the calvaria.
Foramen Spinosum: A small opening in the sphenoid bone through which the middle meningeal artery enters the cranial cavity.
Foramen Ovale: Transmits the mandibular nerve (V3) and accessory meningeal artery.
Foramen Rotundum: Transmits the maxillary nerve (V2).
Foramen Lacerum: A fibrocartilaginous structure at the skull base, not transmitting major arteries.
Lead Question (2014):
Middle meningeal artery passes through?
a) Foramen ovale
b) Foramen lacerum
c) Foramen rotundum
d) Foramen spinosum
Explanation: The middle meningeal artery, a branch of the maxillary artery, enters the cranial cavity through the foramen spinosum. It supplies the dura mater and calvarial bones. This artery runs between the dura and the skull and is clinically important because fractures of the temporal bone can rupture it, causing epidural hematoma. Hence, the correct answer is (d) Foramen spinosum. (100 words)
1. Which nerve passes through the foramen ovale?
a) Maxillary nerve
b) Mandibular nerve
c) Ophthalmic nerve
d) Abducent nerve
Explanation: The foramen ovale transmits the mandibular nerve (V3), the accessory meningeal artery, and sometimes the lesser petrosal nerve. It is an oval opening in the sphenoid bone connecting the middle cranial fossa with the infratemporal fossa. Therefore, the correct answer is (b) Mandibular nerve. (100 words)
2. The middle meningeal artery is a branch of?
a) External carotid artery
b) Internal carotid artery
c) Vertebral artery
d) Superficial temporal artery
Explanation: The middle meningeal artery arises from the first part of the maxillary artery, which is a branch of the external carotid artery. It enters the cranial cavity through the foramen spinosum and divides into anterior and posterior branches, supplying the meninges and skull. Hence, the answer is (a) External carotid artery. (100 words)
3. Which bone is most commonly fractured leading to middle meningeal artery rupture?
a) Temporal bone
b) Parietal bone
c) Occipital bone
d) Frontal bone
Explanation: The temporal bone, particularly the pterion region, is the thinnest part of the skull and lies directly over the middle meningeal artery. Fractures here can rupture the artery, causing an epidural hematoma. Thus, the correct answer is (a) Temporal bone. (100 words)
4. The anterior branch of the middle meningeal artery lies beneath which region?
a) Pterion
b) Lambda
c) Bregma
d) Asterion
Explanation: The anterior branch of the middle meningeal artery runs deep to the pterion — a junction where the frontal, parietal, temporal, and sphenoid bones meet. Because of this anatomical relationship, trauma at the pterion can rupture the artery. Therefore, the correct answer is (a) Pterion. (100 words)
5. The foramen spinosum is located in which bone?
a) Temporal bone
b) Sphenoid bone
c) Parietal bone
d) Frontal bone
Explanation: The foramen spinosum is a small circular opening in the greater wing of the sphenoid bone. It transmits the middle meningeal artery, vein, and meningeal branch of the mandibular nerve. Thus, the correct answer is (b) Sphenoid bone. (100 words)
6. (Clinical) A patient with a temporal bone fracture develops rapid loss of consciousness and a biconvex hematoma on CT. The likely cause is?
a) Rupture of middle meningeal artery
b) Rupture of superior sagittal sinus
c) Rupture of vertebral artery
d) Rupture of cortical vein
Explanation: A temporal bone fracture can tear the middle meningeal artery, leading to accumulation of blood between the skull and dura — an epidural hematoma. This condition shows a biconvex (lens-shaped) appearance on CT and requires emergency evacuation. The correct answer is (a) Rupture of middle meningeal artery. (100 words)
7. (Clinical) Which clinical sign suggests an epidural hematoma due to middle meningeal artery rupture?
a) Lucid interval
b) Continuous unconsciousness
c) Subdural bleed
d) Bilateral mydriasis
Explanation: In epidural hematoma, patients often experience a “lucid interval,” a temporary recovery period after head trauma followed by rapid deterioration as the hematoma enlarges. This is a hallmark of middle meningeal artery rupture. Hence, the correct answer is (a) Lucid interval. (100 words)
8. (Clinical) During a craniotomy, the surgeon identifies bleeding near the foramen spinosum. The source is most likely?
a) Middle meningeal artery
b) Inferior alveolar artery
c) Ophthalmic artery
d) Internal carotid artery
Explanation: The middle meningeal artery enters the skull via the foramen spinosum and lies close to the inner surface of the skull. Bleeding near this area during surgery typically indicates injury to this artery. Thus, the correct answer is (a) Middle meningeal artery. (100 words)
9. (Clinical) A patient develops right-sided weakness and anisocoria after head injury. Which artery is most likely ruptured?
a) Middle meningeal artery
b) Anterior cerebral artery
c) Vertebral artery
d) Posterior inferior cerebellar artery
Explanation: In a middle meningeal artery rupture, an epidural hematoma may compress the brain, causing uncal herniation. The herniated temporal lobe compresses the oculomotor nerve, leading to anisocoria, and affects the contralateral corticospinal tract, causing hemiparesis. Hence, the answer is (a) Middle meningeal artery. (100 words)
10. (Clinical) In epidural hematoma, blood collects between?
a) Dura mater and skull
b) Dura mater and arachnoid mater
c) Arachnoid and pia mater
d) Brain tissue and pia mater
Explanation: An epidural hematoma results from arterial bleeding between the dura mater and the inner surface of the skull. The dura is stripped from the bone by pressure of the expanding hematoma, leading to a biconvex shape on imaging. Hence, the correct answer is (a) Dura mater and skull. (100 words)
Chapter: Central Nervous System; Topic: Brainstem Anatomy; Subtopic: Floor of the Fourth Ventricle
Keyword Definitions:
Fourth Ventricle: A cavity in the hindbrain located between the pons and medulla, continuous with the central canal of the spinal cord.
Vagal Triangle: A triangular area on the floor of the fourth ventricle overlying the dorsal nucleus of the vagus nerve.
Infundibulum: A stalk connecting the hypothalamus to the pituitary gland.
Mammillary Body: Rounded structures involved in memory, located in the hypothalamus.
Tuber Cinereum: A gray matter area in the hypothalamus between the optic chiasma and mammillary bodies.
Lead Question (2014):
Floor of 4th ventricle has?
a) Infundibulum
b) Vagal triangle
c) Mammillary body
d) Tuber cinereum
Explanation: The floor of the fourth ventricle, also called the rhomboid fossa, contains several nuclei and tracts of cranial nerves. The vagal triangle represents the underlying dorsal nucleus of the vagus nerve, making it a prominent feature of the ventricular floor. The correct answer is (b) Vagal triangle. Other options belong to the hypothalamic region. (100 words)
1. The facial colliculus is formed by?
a) Fibers of facial nerve winding around abducens nucleus
b) Hypoglossal nucleus
c) Vagal nucleus
d) Vestibular nucleus
Explanation: The facial colliculus is an elevation in the floor of the fourth ventricle formed by the facial nerve fibers looping around the abducens nucleus. It represents an important landmark in the pontine tegmentum. Hence, the correct answer is (a) Fibers of facial nerve winding around abducens nucleus. (100 words)
2. Which structure forms the roof of the fourth ventricle?
a) Superior medullary velum
b) Inferior olivary nucleus
c) Medial lemniscus
d) Hypothalamus
Explanation: The roof of the fourth ventricle is formed by the superior and inferior medullary vela, and the cerebellum. The superior medullary velum forms the upper part of the roof, connecting the superior cerebellar peduncles. Therefore, the correct answer is (a) Superior medullary velum. (100 words)
3. The floor of the fourth ventricle corresponds to which structure of the brainstem?
a) Pons and medulla
b) Midbrain
c) Hypothalamus
d) Cerebellum
Explanation: The floor of the fourth ventricle is formed by the dorsal surfaces of the pons and medulla oblongata. It has several important nuclei and tracts that form visible elevations. The midbrain and hypothalamus lie above this region. Thus, the correct answer is (a) Pons and medulla. (100 words)
4. The hypoglossal triangle overlies which cranial nerve nucleus?
a) Hypoglossal nerve nucleus
b) Glossopharyngeal nucleus
c) Abducens nucleus
d) Facial motor nucleus
Explanation: The hypoglossal triangle is a small triangular area on the floor of the fourth ventricle overlying the hypoglossal nerve nucleus. It is located medially to the vagal triangle. Therefore, the correct answer is (a) Hypoglossal nerve nucleus. (100 words)
5. The area postrema is associated with which physiological function?
a) Vomiting reflex
b) Vision
c) Hearing
d) Taste perception
Explanation: The area postrema is located in the medulla near the obex and lacks a blood-brain barrier. It detects toxins in the blood and triggers vomiting. Hence, it is known as the vomiting center of the brain. Therefore, the correct answer is (a) Vomiting reflex. (100 words)
6. (Clinical) A patient presents with deviation of the tongue to one side and dysarthria. Lesion is likely at?
a) Hypoglossal triangle region
b) Facial colliculus
c) Vagal triangle
d) Trapezoid body
Explanation: The hypoglossal nerve controls tongue movement. Lesions in the hypoglossal triangle region of the floor of the fourth ventricle damage its nucleus, leading to tongue deviation toward the affected side and speech difficulty. Hence, the correct answer is (a) Hypoglossal triangle region. (100 words)
7. (Clinical) A lesion involving the vagal triangle may cause?
a) Dysphonia and dysphagia
b) Loss of facial sensation
c) Diplopia
d) Ataxia
Explanation: The vagal triangle overlies the dorsal nucleus of the vagus nerve, which supplies motor fibers to the laryngeal and pharyngeal muscles. Lesions here produce dysphonia and dysphagia. Therefore, the correct answer is (a) Dysphonia and dysphagia. (100 words)
8. (Clinical) A tumor compressing the floor of the fourth ventricle may cause?
a) Vomiting and ataxia
b) Blindness
c) Deafness
d) Memory loss
Explanation: Tumors of the fourth ventricle (e.g., medulloblastomas) compress the area postrema and cerebellar connections, causing vomiting and ataxia due to obstruction of CSF flow and vestibular imbalance. Hence, the correct answer is (a) Vomiting and ataxia. (100 words)
9. (Clinical) Lesion near the facial colliculus results in?
a) Facial paralysis and medial strabismus
b) Hearing loss
c) Taste loss
d) Ptosis
Explanation: The facial colliculus contains fibers of the facial nerve looping around the abducens nucleus. Lesion here causes ipsilateral facial paralysis and inability to abduct the eye (medial strabismus). The correct answer is (a) Facial paralysis and medial strabismus. (100 words)
10. (Clinical) A lesion at the obex region leads to?
a) Interference with vomiting reflex
b) Loss of smell
c) Visual disturbances
d) Hemiparesis
Explanation: The obex is the caudal tip of the fourth ventricle near the area postrema, which regulates vomiting. A lesion here impairs toxin detection and vomiting reflex. Hence, the correct answer is (a) Interference with vomiting reflex. (100 words)
Chapter: Cranial Nerves; Topic: Facial Nerve (VII Cranial Nerve); Subtopic: Vidian Nerve (Nerve of the Pterygoid Canal)
Keyword Definitions:
Vidian Nerve: Also known as the nerve of the pterygoid canal, it carries both parasympathetic and sympathetic fibers to the pterygopalatine ganglion.
Greater Petrosal Nerve: A branch of the facial nerve carrying preganglionic parasympathetic fibers from the superior salivatory nucleus.
Deep Petrosal Nerve: A sympathetic nerve derived from the internal carotid plexus.
Pterygopalatine Ganglion: A parasympathetic ganglion in the pterygopalatine fossa associated with facial nerve fibers for lacrimation and nasal secretions.
Lead Question – 2014
Vidian nerve is formed by union of?
a) Superficial petrosal nerve and deep petrosal nerve
b) Greater petrosal nerve and superficial petrosal nerve
c) Greater petrosal nerve and deep petrosal nerve
d) Greater petrosal nerve and external petrosal nerve
Explanation: The Vidian nerve (nerve of the pterygoid canal) is formed by the union of the greater petrosal nerve (carrying preganglionic parasympathetic fibers) and the deep petrosal nerve (carrying postganglionic sympathetic fibers). It passes through the pterygoid canal to reach the pterygopalatine ganglion, where parasympathetic fibers synapse and sympathetic fibers pass without synapse. Answer: Greater petrosal nerve and deep petrosal nerve.
1) The Vidian nerve passes through which bony canal?
a) Carotid canal
b) Pterygoid canal
c) Optic canal
d) Foramen rotundum
Explanation: The Vidian nerve traverses the pterygoid canal, which lies within the sphenoid bone. It connects the foramen lacerum region to the pterygopalatine fossa. The canal transmits the nerve along with accompanying vessels. Its course is important during surgical approaches to the skull base. Answer: Pterygoid canal.
2) The parasympathetic fibers of the Vidian nerve synapse in which ganglion?
a) Otic ganglion
b) Submandibular ganglion
c) Pterygopalatine ganglion
d) Ciliary ganglion
Explanation: The parasympathetic fibers from the greater petrosal component of the Vidian nerve synapse in the pterygopalatine ganglion. From there, postganglionic fibers supply the lacrimal gland, nasal mucosa, and palate glands, controlling secretion. The sympathetic fibers, however, pass through without synapsing. Answer: Pterygopalatine ganglion.
3) The sympathetic fibers in the Vidian nerve originate from?
a) Superior cervical ganglion
b) Middle cervical ganglion
c) Inferior cervical ganglion
d) Celiac ganglion
Explanation: The sympathetic fibers in the Vidian nerve arise from the superior cervical ganglion. They travel along the internal carotid plexus as the deep petrosal nerve, which later joins the greater petrosal nerve to form the Vidian nerve. These fibers are responsible for vasoconstriction in nasal mucosa. Answer: Superior cervical ganglion.
4) Which type of fibers are present in the Vidian nerve?
a) Only sympathetic
b) Only parasympathetic
c) Both sympathetic and parasympathetic
d) Only sensory
Explanation: The Vidian nerve carries a combination of sympathetic and parasympathetic fibers. Parasympathetic fibers arise from the facial nerve via the greater petrosal nerve, and sympathetic fibers originate from the internal carotid plexus via the deep petrosal nerve. Together, they regulate lacrimal and nasal secretions. Answer: Both sympathetic and parasympathetic.
5) The greater petrosal nerve arises from which part of the facial nerve?
a) Geniculate ganglion
b) Stylomastoid foramen
c) Chorda tympani
d) Internal acoustic meatus
Explanation: The greater petrosal nerve arises from the geniculate ganglion of the facial nerve. It carries preganglionic parasympathetic fibers destined for the lacrimal and nasal glands through the pterygopalatine ganglion. Its lesion leads to dry eyes and nasal dryness. Answer: Geniculate ganglion.
6) (Clinical) Injury to the Vidian nerve during endoscopic sinus surgery results in?
a) Loss of lacrimation and nasal dryness
b) Loss of taste sensation
c) Facial paralysis
d) Loss of hearing
Explanation: The Vidian nerve carries secretomotor fibers to lacrimal and nasal glands. Surgical injury leads to loss of lacrimation and nasal mucosal dryness. It does not affect facial motor function or taste, as those are carried by other facial branches. Answer: Loss of lacrimation and nasal dryness.
7) (Clinical) In a case of deep petrosal nerve damage, which function is lost?
a) Vasoconstriction of nasal mucosa
b) Salivation
c) Lacrimation
d) Taste sensation
Explanation: The deep petrosal nerve carries sympathetic fibers responsible for vasoconstriction in nasal mucosa. Injury leads to vasodilation, nasal congestion, and increased secretion. Other autonomic functions like lacrimation and salivation remain unaffected. Answer: Vasoconstriction of nasal mucosa.
8) (Clinical) Which symptom best indicates a lesion at the pterygopalatine ganglion?
a) Loss of lacrimation and nasal secretion
b) Loss of taste
c) Facial paralysis
d) Tinnitus
Explanation: The pterygopalatine ganglion receives parasympathetic fibers from the Vidian nerve. Lesions here lead to reduced lacrimation and nasal dryness but no facial weakness. This condition may follow trauma or skull base surgery. Answer: Loss of lacrimation and nasal secretion.
9) (Clinical) Which surgical nerve block relieves sphenopalatine neuralgia?
a) Vidian nerve block
b) Auriculotemporal nerve block
c) Lingual nerve block
d) Glossopharyngeal nerve block
Explanation: The Vidian nerve block can relieve sphenopalatine neuralgia by interrupting parasympathetic and sympathetic fibers that mediate pain and nasal secretions. It is used in refractory cases of cluster headache or chronic rhinitis. Answer: Vidian nerve block.
10) (Clinical) A lesion in the greater petrosal nerve proximal to its junction with deep petrosal nerve leads to?
a) Dry eyes and nasal mucosa
b) Excessive salivation
c) Loss of hearing
d) Facial muscle paralysis
Explanation: The greater petrosal nerve carries preganglionic parasympathetic fibers to the lacrimal and nasal glands. A lesion before its junction with the deep petrosal nerve results in dryness of eyes and nasal mucosa due to loss of secretomotor fibers. Answer: Dry eyes and nasal mucosa.
Chapter: Cranial Nerves; Topic: Facial Nerve (VII Cranial Nerve); Subtopic: Chorda Tympani Nerve
Keyword Definitions:
Chorda Tympani: A branch of the facial nerve that carries taste fibers from the anterior two-thirds of the tongue and preganglionic parasympathetic fibers to the submandibular and sublingual glands.
Preganglionic Parasympathetic Fibers: Nerve fibers that originate from the central nervous system and synapse in peripheral ganglia before reaching the target organ.
Facial Nerve: The seventh cranial nerve that supplies muscles of facial expression and carries special sensory and parasympathetic fibers.
Submandibular Ganglion: A parasympathetic ganglion associated with the chorda tympani and lingual nerve, supplying salivary glands.
Lead Question – 2014
What is true about chorda tympani?
a) Postganglionic sympathetic
b) Preganglionic sympathetic
c) Preganglionic parasympathetic
d) Postganglionic parasympathetic
Explanation: The chorda tympani carries preganglionic parasympathetic fibers originating from the superior salivatory nucleus of the facial nerve. These fibers join the lingual nerve and synapse in the submandibular ganglion, providing secretomotor fibers to the submandibular and sublingual glands. It also conveys taste fibers from the anterior two-thirds of the tongue. Answer: Preganglionic parasympathetic.
1) The chorda tympani nerve joins which branch of the trigeminal nerve?
a) Lingual nerve
b) Inferior alveolar nerve
c) Auriculotemporal nerve
d) Buccal nerve
Explanation: The chorda tympani joins the lingual nerve, a branch of the mandibular division of the trigeminal nerve. Through this connection, it carries taste fibers from the anterior two-thirds of the tongue and parasympathetic fibers to the submandibular and sublingual glands. This anatomical association is crucial for salivation and taste perception. Answer: Lingual nerve.
2) The chorda tympani nerve passes through which cavity of the ear?
a) Tympanic cavity
b) External auditory canal
c) Mastoid antrum
d) Eustachian tube
Explanation: The chorda tympani traverses the tympanic cavity, passing between the handle of the malleus and the long process of the incus. It then exits through the petrotympanic fissure to join the lingual nerve. Its proximity to middle ear structures explains taste disturbances after middle ear infections or surgeries. Answer: Tympanic cavity.
3) Which ganglion is associated with the chorda tympani nerve?
a) Submandibular ganglion
b) Otic ganglion
c) Pterygopalatine ganglion
d) Geniculate ganglion
Explanation: The chorda tympani carries preganglionic parasympathetic fibers to the submandibular ganglion. Postganglionic fibers from this ganglion innervate the submandibular and sublingual salivary glands. This connection plays a key role in salivary secretion under parasympathetic control. Answer: Submandibular ganglion.
4) Taste sensation from the anterior two-thirds of the tongue is carried by?
a) Chorda tympani
b) Glossopharyngeal nerve
c) Vagus nerve
d) Lingual nerve only
Explanation: The chorda tympani, a branch of the facial nerve, carries taste fibers from the anterior two-thirds of the tongue via the lingual nerve. The glossopharyngeal nerve serves the posterior third, and the vagus nerve supplies the epiglottis region. Hence, the major taste pathway anteriorly is through the chorda tympani. Answer: Chorda tympani.
5) Loss of taste in the anterior two-thirds of the tongue and decreased salivation may indicate a lesion of?
a) Chorda tympani
b) Greater petrosal nerve
c) Glossopharyngeal nerve
d) Auriculotemporal nerve
Explanation: Damage to the chorda tympani results in loss of taste sensation from the anterior two-thirds of the tongue and reduced secretion from submandibular and sublingual glands. This occurs in facial nerve injury distal to the geniculate ganglion but proximal to its junction with the lingual nerve. Answer: Chorda tympani.
6) (Clinical) A 25-year-old patient reports taste loss on the left anterior tongue after otitis media. Which nerve is affected?
a) Chorda tympani
b) Glossopharyngeal
c) Lingual
d) Hypoglossal
Explanation: The chorda tympani passes through the middle ear cavity and can be damaged during infection or surgery. Loss of taste on the anterior two-thirds of the tongue on the same side and decreased salivation confirm chorda tympani involvement. Answer: Chorda tympani.
7) (Clinical) A lesion at the stylomastoid foramen will cause which deficit?
a) Facial paralysis only
b) Loss of taste and salivation
c) Loss of lacrimation
d) Loss of smell
Explanation: The chorda tympani branches off the facial nerve proximal to the stylomastoid foramen. Therefore, a lesion at the stylomastoid foramen affects only motor fibers causing facial paralysis, without loss of taste or salivation. Answer: Facial paralysis only.
8) (Clinical) During parotid surgery, which nerve function remains unaffected by facial nerve injury?
a) Taste from anterior tongue
b) Motor supply to buccinator
c) Secretion from parotid gland
d) Movement of orbicularis oculi
Explanation: The parotid gland receives parasympathetic innervation from the glossopharyngeal nerve via the otic ganglion, not the facial nerve. Thus, parotid secretion remains intact even if the facial nerve (and chorda tympani) are injured. Answer: Secretion from parotid gland.
9) (Clinical) A patient presents with reduced salivation but normal tear secretion. The lesion is most likely proximal to which branch?
a) Chorda tympani
b) Greater petrosal nerve
c) Nerve to stapedius
d) Posterior auricular nerve
Explanation: The chorda tympani carries parasympathetic fibers to salivary glands. A lesion proximal to its origin but distal to the greater petrosal nerve causes loss of salivation without affecting lacrimation. Answer: Chorda tympani.
10) (Clinical) A tumor compressing the facial nerve at the internal acoustic meatus will result in all except?
a) Facial paralysis
b) Loss of lacrimation
c) Loss of taste
d) Loss of hearing
Explanation: The internal acoustic meatus contains both facial and vestibulocochlear nerves. Compression here leads to facial paralysis, loss of taste from chorda tympani, and decreased lacrimation. Hearing loss, however, occurs if the vestibulocochlear nerve is involved. If spared, hearing remains normal. Answer: Loss of hearing (if vestibulocochlear intact).
Chapter: Nervous System; Topic: Cranial Nerves; Subtopic: Parasympathetic Pathways to Lacrimal Gland
Keyword Definitions:
Lacrimal gland: Exocrine gland that secretes tears to lubricate and protect the eye.
Parasympathetic supply: Autonomic fibers responsible for secretory stimulation of glands.
Greater petrosal nerve: A branch of the facial nerve that carries parasympathetic fibers to the lacrimal gland.
Pterygopalatine ganglion: A parasympathetic ganglion where preganglionic fibers from the facial nerve synapse before reaching the lacrimal gland.
Lead Question - 2014
Parasympathetic supply to lacrimal glands are passed through?
a) Lesser petrosal nerve
b) Chorda tympani
c) Greater petrosal nerve
d) Lingual nerve
Explanation:
The parasympathetic fibers to the lacrimal gland originate from the superior salivatory nucleus of the facial nerve. They travel via the greater petrosal nerve, synapse in the pterygopalatine ganglion, and then reach the lacrimal gland through the zygomatic and lacrimal nerves. This pathway controls tear secretion. Hence, the correct answer is c) Greater petrosal nerve.
1) Which ganglion is involved in the parasympathetic pathway to the lacrimal gland?
a) Ciliary ganglion
b) Otic ganglion
c) Pterygopalatine ganglion
d) Submandibular ganglion
The pterygopalatine ganglion (also called sphenopalatine) is where the preganglionic parasympathetic fibers from the greater petrosal nerve synapse. Postganglionic fibers then reach the lacrimal gland via the zygomatic branch of the maxillary nerve. Hence, the correct answer is c) Pterygopalatine ganglion.
2) The greater petrosal nerve is a branch of which cranial nerve?
a) Trigeminal
b) Facial
c) Glossopharyngeal
d) Vagus
The greater petrosal nerve arises from the facial nerve (cranial nerve VII) at the geniculate ganglion. It carries preganglionic parasympathetic fibers to the pterygopalatine ganglion, which later innervate the lacrimal and nasal glands. Hence, the correct answer is b) Facial nerve.
3) Which nerve carries postganglionic fibers to the lacrimal gland?
a) Zygomatic nerve
b) Auriculotemporal nerve
c) Lingual nerve
d) Nasociliary nerve
After synapsing in the pterygopalatine ganglion, postganglionic fibers travel via the zygomatic nerve (branch of V2) and join the lacrimal nerve (branch of V1) to reach the lacrimal gland. Hence, the correct answer is a) Zygomatic nerve.
4) Which cranial nucleus gives origin to fibers supplying the lacrimal gland?
a) Inferior salivatory nucleus
b) Superior salivatory nucleus
c) Edinger-Westphal nucleus
d) Dorsal motor nucleus of vagus
The superior salivatory nucleus of the pons provides preganglionic parasympathetic fibers that travel through the facial nerve and greater petrosal nerve to reach the lacrimal gland, promoting tear secretion. Hence, the correct answer is b) Superior salivatory nucleus.
5) (Clinical) Damage to the greater petrosal nerve results in?
a) Loss of taste
b) Dryness of eyes
c) Hyperlacrimation
d) Facial palsy
Injury to the greater petrosal nerve disrupts parasympathetic innervation to the lacrimal gland, leading to dryness of the eye due to reduced tear secretion. This may occur in facial nerve lesions proximal to the geniculate ganglion. Hence, the correct answer is b) Dryness of eyes.
6) (Clinical) A lesion at the geniculate ganglion affects which functions?
a) Lacrimation and taste
b) Hearing only
c) Facial sensation
d) Eye movements
A lesion at the geniculate ganglion affects both lacrimation (greater petrosal nerve) and taste (chorda tympani). Patients may present with dry eyes and loss of taste on the anterior two-thirds of the tongue. Hence, the correct answer is a) Lacrimation and taste.
7) (Clinical) Which symptom indicates greater petrosal nerve damage in facial palsy?
a) Loss of corneal reflex
b) Decreased lacrimation
c) Hyperacusis
d) Vertigo
In facial palsy affecting the segment proximal to the greater petrosal nerve, decreased lacrimation occurs due to interruption of parasympathetic fibers to the lacrimal gland. Hence, the correct answer is b) Decreased lacrimation.
8) (Clinical) A patient with Bell’s palsy complains of dry eyes. Which structure is likely involved?
a) Chorda tympani
b) Greater petrosal nerve
c) Stapedius branch
d) Temporal branch of facial nerve
Dry eyes in Bell’s palsy suggest involvement of the greater petrosal nerve due to interruption of parasympathetic supply to the lacrimal gland. This differentiates proximal from distal facial nerve lesions. Hence, the correct answer is b) Greater petrosal nerve.
9) (Clinical) Which condition may cause excessive tearing (epiphora) despite normal nerve function?
a) Blocked nasolacrimal duct
b) Facial nerve palsy
c) Damage to pterygopalatine ganglion
d) Geniculate ganglion lesion
Excessive tearing, or epiphora, usually results from a blocked nasolacrimal duct, preventing proper tear drainage into the nasal cavity. The parasympathetic pathway remains intact. Hence, the correct answer is a) Blocked nasolacrimal duct.
10) (Clinical) Which condition is characterized by dry eyes and mouth due to autoimmune destruction of lacrimal and salivary glands?
a) Myasthenia gravis
b) Sjögren’s syndrome
c) Bell’s palsy
d) Sarcoidosis
Sjögren’s syndrome causes autoimmune destruction of lacrimal and salivary glands, leading to dry eyes (xerophthalmia) and dry mouth (xerostomia). Parasympathetic pathways remain intact, but glandular tissue is damaged. Hence, the correct answer is b) Sjögren’s syndrome.
Chapter: Neuroanatomy; Topic: Brainstem Circulation; Subtopic: Arterial Supply of the Medulla Oblongata
Keyword Definitions:
Medulla oblongata: The lowest part of the brainstem, controlling vital functions like breathing, heart rate, and reflexes such as swallowing and coughing.
Vertebral artery: A branch of the subclavian artery that ascends through the cervical vertebrae to supply the brainstem and cerebellum.
Anterior spinal artery: Formed by branches of the vertebral arteries, supplying the anterior two-thirds of the medulla and spinal cord.
Basilar artery: Formed by the union of the two vertebral arteries, supplying the pons and upper medulla.
Posterior cerebral artery: A branch of the basilar artery that supplies the midbrain and occipital lobes but not the medulla.
Lead Question – 2014
Medulla is supplied by all except?
a) Basilar artery
b) Anterior spinal artery
c) Vertebral artery
d) Posterior cerebral artery
Explanation:
The medulla oblongata is supplied mainly by the vertebral arteries, anterior spinal artery, and posterior inferior cerebellar artery (PICA). The basilar artery contributes minimally to the upper medulla. However, the posterior cerebral artery supplies the midbrain and occipital lobes but does not supply the medulla. Hence, the correct answer is d) Posterior cerebral artery. This vascular arrangement is vital for maintaining vital autonomic functions.
1) Which artery forms the anterior spinal artery?
a) Vertebral artery
b) Basilar artery
c) Posterior inferior cerebellar artery
d) Posterior spinal artery
Explanation: The anterior spinal artery is formed by branches of the vertebral arteries near their junction at the medulla. It runs along the anterior median fissure and supplies the anterior two-thirds of the spinal cord and medulla. Hence, the correct answer is a) Vertebral artery.
2) The posterior inferior cerebellar artery (PICA) is a branch of which artery?
a) Basilar artery
b) Vertebral artery
c) Posterior cerebral artery
d) Internal carotid artery
Explanation: The PICA arises from the vertebral artery before it merges into the basilar artery. It supplies the dorsolateral medulla and inferior part of the cerebellum. Its occlusion results in lateral medullary (Wallenberg’s) syndrome. The correct answer is b) Vertebral artery.
3) Occlusion of which artery causes medial medullary syndrome?
a) Anterior spinal artery
b) Posterior spinal artery
c) Basilar artery
d) Posterior inferior cerebellar artery
Explanation: Medial medullary syndrome occurs due to occlusion of the anterior spinal artery. It affects the pyramids, medial lemniscus, and hypoglossal nerve, leading to contralateral hemiplegia, loss of proprioception, and ipsilateral tongue paralysis. Hence, the correct answer is a) Anterior spinal artery.
4) Which artery does not directly contribute to the blood supply of the medulla?
a) Vertebral artery
b) Posterior cerebral artery
c) Basilar artery
d) Anterior spinal artery
Explanation: The posterior cerebral artery supplies the midbrain, thalamus, and occipital cortex, but not the medulla. The vertebral, basilar, and anterior spinal arteries all supply portions of the medulla. Hence, the correct answer is b) Posterior cerebral artery.
5) A patient presents with hoarseness, loss of gag reflex, and contralateral body sensory loss. Which artery is most likely affected?
a) Anterior spinal artery
b) Posterior inferior cerebellar artery
c) Basilar artery
d) Posterior cerebral artery
Explanation: These are classic features of lateral medullary (Wallenberg’s) syndrome due to occlusion of the PICA. It affects the nucleus ambiguus, spinal tract of the trigeminal nerve, and spinothalamic tract. Hence, the correct answer is b) Posterior inferior cerebellar artery.
6) The basilar artery is formed by the union of which arteries?
a) Internal carotid arteries
b) Vertebral arteries
c) Anterior spinal arteries
d) Posterior cerebral arteries
Explanation: The basilar artery is formed by the union of the two vertebral arteries at the level of the pontomedullary junction. It ascends on the ventral surface of the pons and gives off branches to the pons, cerebellum, and inner ear. Hence, the correct answer is b) Vertebral arteries.
7) A 60-year-old man presents with tongue deviation to one side and contralateral hemiplegia. The lesion involves which artery?
a) Basilar artery
b) Anterior spinal artery
c) Posterior inferior cerebellar artery
d) Posterior cerebral artery
Explanation: The symptoms indicate medial medullary syndrome due to occlusion of the anterior spinal artery. It damages the hypoglossal nerve, pyramid, and medial lemniscus. Tongue deviation occurs ipsilaterally, and contralateral weakness occurs due to corticospinal tract involvement. Correct answer: b) Anterior spinal artery.
8) Which of the following arteries supplies the dorsal medulla including the gracile and cuneate nuclei?
a) Posterior spinal artery
b) Anterior spinal artery
c) Vertebral artery
d) Basilar artery
Explanation: The posterior spinal arteries supply the dorsal medulla, particularly the gracile and cuneate nuclei. These arteries arise from either the vertebral arteries or posterior inferior cerebellar arteries. Their occlusion leads to sensory loss for fine touch and proprioception. Hence, the correct answer is a) Posterior spinal artery.
9) A stroke involving the vertebral artery may present with which of the following symptoms?
a) Contralateral limb paralysis and ipsilateral facial weakness
b) Dysphagia, hoarseness, and ataxia
c) Aphasia and hemianopia
d) Facial numbness and upper limb weakness
Explanation: Vertebral artery occlusion can cause lateral medullary syndrome with dysphagia, hoarseness, ataxia, and loss of pain and temperature sensation. These features result from involvement of the nucleus ambiguus and inferior cerebellar peduncle. The correct answer is b) Dysphagia, hoarseness, and ataxia.
10) Which artery is commonly affected in brainstem infarction at the pontomedullary junction?
a) Basilar artery
b) Posterior cerebral artery
c) Anterior spinal artery
d) Posterior inferior cerebellar artery
Explanation: The basilar artery supplies the pons and upper medulla at the pontomedullary junction. Infarction here may cause cranial nerve deficits (VI, VII) and contralateral motor weakness. The posterior cerebral artery does not supply this region. Hence, the correct answer is a) Basilar artery.
Chapter: Neuroanatomy; Topic: Cranial Nerves; Subtopic: Trochlear Nerve and its Unique Features
Keyword Definitions:
Cranial nerves: Twelve pairs of nerves that arise directly from the brain and brainstem, responsible for sensory and motor innervation to the head and neck.
Brainstem: The central part of the brain consisting of the midbrain, pons, and medulla oblongata, controlling vital reflexes.
Trochlear nerve (CN IV): The only cranial nerve emerging dorsally from the brainstem and the smallest in size. It supplies the superior oblique muscle of the eye.
Dorsal exit: Refers to a nerve emerging from the posterior (back) aspect of the brainstem.
Superior oblique muscle: Muscle responsible for intorsion and depression of the eyeball, controlled by CN IV.
Lead Question – 2014
Which is the only nerve which exits the brainstem on dorsal side?
a) Facial
b) Trigeminal
c) Trochlear
d) Abducent
Explanation:
The trochlear nerve (CN IV) is unique as it exits from the dorsal aspect of the brainstem, just below the inferior colliculus. It also decussates before emerging, supplying the contralateral superior oblique muscle. This dorsal emergence distinguishes it from all other cranial nerves, which exit ventrally or laterally. Hence, the correct answer is Trochlear (c).
1) The trochlear nerve supplies which muscle?
a) Superior rectus
b) Superior oblique
c) Inferior oblique
d) Lateral rectus
Explanation: The trochlear nerve supplies the superior oblique muscle, responsible for intorsion and depression of the eyeball. It helps move the eye downward and inward. Its lesion leads to vertical diplopia, especially when looking downward, such as reading or descending stairs.
2) Which cranial nerve has the longest intracranial course?
a) Trochlear
b) Abducent
c) Trigeminal
d) Facial
Explanation: The trochlear nerve has the longest intracranial course and is most prone to injury during head trauma. It travels around the midbrain before entering the orbit through the superior orbital fissure. The correct answer is Trochlear (a).
3) Which cranial nerve nucleus lies in the midbrain at the level of the inferior colliculus?
a) Oculomotor
b) Trochlear
c) Abducent
d) Facial
Explanation: The trochlear nucleus lies in the midbrain at the level of the inferior colliculus. The nerve fibers decussate before emerging dorsally. This unique crossing explains contralateral superior oblique muscle innervation. The correct answer is Trochlear (b).
4) A patient presents with vertical diplopia, worsened when looking down. Which nerve is likely affected?
a) Oculomotor
b) Trochlear
c) Abducent
d) Optic
Explanation: Lesion of the trochlear nerve causes vertical diplopia due to paralysis of the superior oblique muscle. The patient tilts the head away from the affected side to compensate. The correct answer is Trochlear (b).
5) Which cranial nerve decussates before emerging from the brainstem?
a) Optic
b) Trochlear
c) Abducent
d) Facial
Explanation: The trochlear nerve is the only cranial nerve to fully cross (decussate) within the brainstem before exiting dorsally. Each nucleus thus controls the contralateral superior oblique muscle. The correct answer is Trochlear (b).
6) During head trauma, which cranial nerve is most likely injured due to its long intracranial course?
a) Oculomotor
b) Trochlear
c) Abducent
d) Facial
Explanation: Because the trochlear nerve travels the longest intracranial path, it is highly susceptible to shearing forces in head injury. Patients experience difficulty looking down, particularly when reading. The correct answer is Trochlear (b).
7) The trochlear nerve exits the brainstem from which structure?
a) Medulla
b) Pons
c) Midbrain
d) Cerebellum
Explanation: The trochlear nerve arises from the dorsal aspect of the midbrain at the level of the inferior colliculus. It then wraps around the brainstem anteriorly to enter the cavernous sinus. The correct answer is Midbrain (c).
8) A patient cannot look down when walking downstairs. Which muscle is weak?
a) Superior rectus
b) Superior oblique
c) Inferior rectus
d) Lateral rectus
Explanation: Weakness of the superior oblique muscle due to trochlear nerve palsy causes inability to look downward when the eye is adducted. This results in vertical diplopia. The correct answer is Superior oblique (b).
9) Which cranial nerve controls the downward gaze of the eyeball?
a) Oculomotor
b) Trochlear
c) Abducent
d) Optic
Explanation: The trochlear nerve controls the superior oblique muscle, which assists in downward gaze, especially when the eye is adducted. Lesion leads to upward deviation and diplopia. The correct answer is Trochlear (b).
10) A patient presents with head tilt to the right and vertical diplopia. Which nerve is likely affected?
a) Right oculomotor
b) Left trochlear
c) Right trochlear
d) Left abducent
Explanation: A lesion of the right trochlear nerve causes the right eye to deviate upward due to superior oblique paralysis. The patient tilts the head to the left to compensate for diplopia. The correct answer is Right trochlear (c).
Chapter: Neuroanatomy; Topic: Cranial Nerve Pathways; Subtopic: Trigeminal Ganglion and Meckel’s Cave
Keyword Definitions:
• Meckel’s Cave: A dural pouch in the middle cranial fossa housing the trigeminal ganglion.
• Trigeminal Ganglion: Sensory ganglion of cranial nerve V located within Meckel’s cave.
• Otic Ganglion: A small parasympathetic ganglion located below the foramen ovale, linked to the glossopharyngeal nerve.
• Pterygopalatine Ganglion: Parasympathetic ganglion in the pterygopalatine fossa associated with facial nerve fibers.
• Submandibular Ganglion: Parasympathetic ganglion associated with chorda tympani and lingual nerve supplying submandibular glands.
Lead Question – 2014
Meckel's cave is related to?
a) Submandibular ganglion
b) Trigeminal ganglion
c) Otic ganglion
d) Pterygopalatine ganglion
Explanation:
Meckel’s cave is a dural recess in the middle cranial fossa that contains the trigeminal ganglion (Gasserian ganglion). It provides cushioning and protection to the ganglion and its divisions. The space is filled with cerebrospinal fluid and communicates with the subarachnoid space. Therefore, the correct answer is trigeminal ganglion.
1. Meckel’s cave is located in which cranial fossa?
a) Anterior
b) Middle
c) Posterior
d) None
2. The trigeminal ganglion lies within:
a) Cavernous sinus
b) Meckel’s cave
c) Foramen ovale
d) Internal auditory meatus
3. Trigeminal ganglion gives rise to how many main divisions?
a) One
b) Two
c) Three
d) Four
4. Which structure passes through the foramen rotundum?
a) Maxillary nerve
b) Mandibular nerve
c) Ophthalmic nerve
d) Facial nerve
5. Which cranial nerve is most likely affected if there is lesion in Meckel’s cave?
a) CN IV
b) CN V
c) CN VI
d) CN VII
6. A patient presents with loss of corneal reflex and facial pain. The lesion most likely involves:
a) Otic ganglion
b) Trigeminal ganglion
c) Geniculate ganglion
d) Pterygopalatine ganglion
7. The dural covering of Meckel’s cave is derived from:
a) Pia mater
b) Arachnoid mater
c) Dura mater
d) Endosteum
8. A tumor compressing Meckel’s cave may lead to which symptom?
a) Anosmia
b) Trigeminal neuralgia
c) Vertigo
d) Diplopia
9. Pterygopalatine ganglion is associated with which cranial nerve?
a) Facial nerve
b) Glossopharyngeal nerve
c) Trigeminal nerve
d) Vagus nerve
10. Otic ganglion is related to which nerve fiber type?
a) Sensory fibers of trigeminal nerve
b) Parasympathetic fibers from glossopharyngeal nerve
c) Sympathetic fibers from carotid plexus
d) Somatic motor fibers
11. During skull base surgery, damage to Meckel’s cave may result in loss of sensation from:
a) Ear and scalp
b) Cornea and face
c) Tongue and palate
d) Jaw and larynx
Explanation:
Meckel’s cave houses the trigeminal ganglion and is an important structure in skull base anatomy. Compression of this space can lead to trigeminal neuralgia, facial pain, or sensory loss in the face. It is located in the middle cranial fossa, posterior to the cavernous sinus, lined by dura mater. Thus, the correct answer is trigeminal ganglion.
Topic: Reflex Arcs; Subtopic: Cremasteric Reflex
Keyword Definitions:
Cremasteric Reflex: A superficial reflex that causes elevation of the testis on stroking the inner thigh.
Genitofemoral Nerve: A mixed nerve from L1–L2 spinal segments, with genital and femoral branches.
Afferent Limb: The sensory pathway carrying impulse to spinal cord.
Efferent Limb: The motor pathway carrying impulse to muscle.
Lead Question - 2014
True about cremasteric reflex?
a) Afferent: genital branch of genitofemoral nerve
b) Efferent: genital branch of genitofemoral nerve
c) Efferent: femoral branch of genitofemoral nerve
d) Afferent: pudendal nerve
Explanation: The cremasteric reflex has an afferent limb through the femoral branch of the genitofemoral nerve and the ilioinguinal nerve, and an efferent limb through the genital branch of the genitofemoral nerve causing contraction of the cremaster muscle. Answer: b) Efferent: genital branch of genitofemoral nerve.
1. Absence of cremasteric reflex indicates lesion at which spinal level?
a) T10–T11
b) L1–L2
c) S1–S2
d) C5–C6
The cremasteric reflex is mediated through L1–L2 spinal segments. Its absence suggests a lesion involving these levels, as in spinal cord injury or testicular torsion. Answer: b) L1–L2.
2. Cremaster muscle is derived from which layer of the abdominal wall?
a) External oblique
b) Internal oblique
c) Transversus abdominis
d) Fascia transversalis
The cremaster muscle is a continuation of the internal oblique muscle fibers and forms part of the spermatic cord. Its contraction elevates the testis. Answer: b) Internal oblique.
3. Which nerve carries the sensory component of the cremasteric reflex?
a) Femoral branch of genitofemoral nerve
b) Genital branch of genitofemoral nerve
c) Pudendal nerve
d) Iliohypogastric nerve
The sensory (afferent) limb of the cremasteric reflex is carried mainly by the femoral branch of the genitofemoral nerve and partially by the ilioinguinal nerve. Answer: a) Femoral branch of genitofemoral nerve.
4. In testicular torsion, cremasteric reflex is:
a) Exaggerated
b) Absent
c) Normal
d) Delayed
In testicular torsion, the cremasteric reflex is typically absent on the affected side due to compromised nerve supply and pain inhibition. This is a key clinical diagnostic feature. Answer: b) Absent.
5. Which muscle contraction is responsible for the cremasteric reflex?
a) Dartos muscle
b) Cremaster muscle
c) External oblique
d) Transversus abdominis
The cremasteric reflex involves contraction of the cremaster muscle, causing elevation of the testis. This muscle is innervated by the genital branch of the genitofemoral nerve. Answer: b) Cremaster muscle.
6. Clinical case: A 25-year-old male presents after trauma to the upper thigh with absent cremasteric reflex. Which nerve is likely injured?
a) Ilioinguinal nerve
b) Genitofemoral nerve
c) Pudendal nerve
d) Obturator nerve
Damage to the genitofemoral nerve disrupts both the afferent and efferent limbs of the cremasteric reflex, leading to its absence. Answer: b) Genitofemoral nerve.
7. A newborn has undescended testes and absent cremasteric reflex. What is the most likely cause?
a) Cryptorchidism
b) Hydrocele
c) Hernia
d) Varicocele
In cryptorchidism, the testis fails to descend into the scrotum, leading to absent cremasteric reflex due to abnormal nerve and muscle development. Answer: a) Cryptorchidism.
8. Afferent fibers of cremasteric reflex travel through:
a) Femoral branch of genitofemoral nerve
b) Ilioinguinal nerve
c) Both a and b
d) None
Both the femoral branch of the genitofemoral nerve and ilioinguinal nerve contribute sensory input from the inner thigh to the spinal cord. Answer: c) Both a and b.
9. Which reflex is mediated at the spinal level S1–S2?
a) Cremasteric reflex
b) Anal reflex
c) Abdominal reflex
d) Plantar reflex
The anal reflex, not the cremasteric reflex, is mediated at the S1–S2 level. The cremasteric reflex is L1–L2. Answer: b) Anal reflex.
10. Clinical case: A man with spinal cord injury above L1–L2 shows absent cremasteric reflex but intact anal reflex. Which statement is correct?
a) Reflex arc of cremasteric is intact
b) Reflex arc of cremasteric is interrupted
c) Pudendal nerve involved
d) Reflex mediated by S2–S4
The cremasteric reflex arc is interrupted in lesions above L1–L2, abolishing the reflex, while the anal reflex (S2–S4) remains intact. Answer: b) Reflex arc of cremasteric is interrupted.
Subtopic: Nerves of Thorax – Vagus and Phrenic Nerves
Keyword Definitions:
Vagus nerve: The tenth cranial nerve supplying parasympathetic fibers to thoracic and abdominal viscera.
Phrenic nerve: Arises from C3–C5 spinal roots, providing motor supply to the diaphragm.
Arch of aorta: The curved portion of the aorta giving rise to major arteries of the upper body.
Thoracic cavity: The chest space containing lungs, heart, and major vessels.
Lead Question (2014):
At the level of Arch of aorta, the relationship of left vagus nerve and left phrenic nerve?
a) Phrenic nerve anterior, vagus nerve posterior
b) Phrenic nerve posterior, vagus nerve anterior
c) Both in same plane anteroposteriorly
d) Variable in relationship
Explanation:
At the level of the aortic arch, the left phrenic nerve lies anterior to the left vagus nerve. Answer: a) Phrenic nerve anterior, vagus nerve posterior. The vagus nerve gives off the left recurrent laryngeal branch here, looping under the arch near the ligamentum arteriosum.
1)
The left recurrent laryngeal nerve hooks around which structure in the thorax?
a) Right subclavian artery
b) Arch of aorta
c) Pulmonary trunk
d) Left subclavian vein
The left recurrent laryngeal nerve hooks under the arch of the aorta, close to the ligamentum arteriosum. Answer: b) Arch of aorta. This explains why enlargement of mediastinal nodes or aneurysm can cause hoarseness due to recurrent laryngeal nerve compression.
2)
Which of the following nerves carries parasympathetic fibers to the thoracic and abdominal organs?
a) Phrenic nerve
b) Vagus nerve
c) Intercostal nerve
d) Sympathetic trunk
The vagus nerve provides parasympathetic supply to most thoracic and abdominal viscera. Answer: b) Vagus nerve. It slows heart rate, enhances gastrointestinal motility, and modulates glandular secretions through its autonomic fibers descending through the thorax into the abdomen.
3)
A 60-year-old man with hoarseness of voice is found to have an aortic aneurysm. Which nerve is most likely compressed?
a) Right phrenic
b) Left phrenic
c) Left recurrent laryngeal
d) Right vagus
An aortic arch aneurysm can compress the left recurrent laryngeal nerve as it loops beneath the arch. Answer: c) Left recurrent laryngeal. This causes hoarseness due to paralysis of the left vocal cord supplied by this branch of the vagus nerve.
4)
Phrenic nerve arises from which spinal segments?
a) C1–C3
b) C2–C4
c) C3–C5
d) C4–C6
The phrenic nerve originates mainly from cervical spinal nerves C3, C4, and C5. Answer: c) C3–C5. Its mnemonic “C3, 4, and 5 keep the diaphragm alive” emphasizes its importance in diaphragmatic contraction and maintenance of respiratory function.
5)
In thoracic surgery, which nerve must be carefully preserved to avoid diaphragmatic paralysis?
a) Vagus
b) Intercostal
c) Phrenic
d) Recurrent laryngeal
The phrenic nerve supplies the diaphragm motor fibers. Injury during surgery may cause hemidiaphragm paralysis and breathing difficulty. Answer: c) Phrenic. Its anterior location on the pericardium makes it vulnerable in cardiac or mediastinal procedures.
6)
The right vagus nerve passes posterior to which thoracic structure?
a) Right pulmonary root
b) Left pulmonary root
c) Arch of aorta
d) Pulmonary trunk
The right vagus nerve passes posterior to the right pulmonary root, whereas the left vagus lies anterior to the left root. Answer: a) Right pulmonary root. These relations are clinically important during lung and mediastinal surgeries to avoid nerve injury.
7)
A trauma patient presents with left hemidiaphragmatic paralysis. Which nerve is most likely injured?
a) Left vagus
b) Left phrenic
c) Right vagus
d) Left intercostal
Diaphragmatic paralysis on one side indicates phrenic nerve damage. Answer: b) Left phrenic. The phrenic nerve runs along the pericardium and can be injured by penetrating trauma or mediastinal compression, leading to elevation of the hemidiaphragm on chest X-ray.
8)
Which branch of the vagus nerve contributes to the cardiac plexus?
a) Superior cardiac branch
b) Recurrent laryngeal branch
c) Cervical cardiac branch
d) Pulmonary branch
The cervical cardiac branches of the vagus nerve descend to form part of the cardiac plexus, supplying parasympathetic fibers to the heart. Answer: c) Cervical cardiac branch. These fibers help decrease heart rate and modulate conduction through the atrioventricular node.
9)
A mediastinal tumor compressing the left phrenic nerve would cause which symptom?
a) Hoarseness
b) Dysphagia
c) Dyspnea on exertion
d) Loss of cough reflex
Compression of the left phrenic nerve results in paralysis of the left diaphragm, producing breathlessness on exertion. Answer: c) Dyspnea on exertion. This occurs because the affected hemidiaphragm fails to contract effectively, reducing lung expansion during inspiration.
10)
The vagus nerve forms which plexus on the esophagus before entering the abdomen?
a) Pulmonary plexus
b) Cardiac plexus
c) Esophageal plexus
d) Gastric plexus
Before passing through the diaphragm, the vagus nerve forms the esophageal plexus around the esophagus. Answer: c) Esophageal plexus. These fibers then reorganize into anterior and posterior vagal trunks that continue into the abdomen to supply visceral organs.
Topic: Lumbar Plexus
Subtopic: Nerve Branches
Keyword Definitions:
Lumbar plexus: A network of nerves in the posterior abdominal wall formed by ventral rami of L1–L4 spinal nerves.
Iliohypogastric nerve: Branch of lumbar plexus supplying abdominal wall and skin above pubis.
Ilioinguinal nerve: Branch of lumbar plexus supplying groin and upper medial thigh skin.
Obturator nerve: Nerve from lumbar plexus supplying medial thigh muscles.
Subcostal nerve: A branch of T12 nerve, not part of the lumbar plexus.
Lead Question - 2014
All are branches of lumbar plexus except?
a) Iliohypogastric nerve
b) Ilioinguinal nerve
c) Obturator nerve
d) Subcostal nerve
Explanation: The lumbar plexus arises from L1–L4 spinal nerves. It gives branches such as iliohypogastric, ilioinguinal, genitofemoral, femoral, and obturator nerves. The subcostal nerve is from T12 and not part of the lumbar plexus. Hence, the correct answer is d) Subcostal nerve.
1. The lumbar plexus is formed by which spinal segments?
a) L1–L3
b) L1–L4
c) L2–L5
d) T12–L3
Explanation: The lumbar plexus is formed by ventral rami of L1–L4, with a contribution from T12 in some cases. It supplies abdominal wall, thigh, and pelvic regions. Correct answer: b) L1–L4.
2. Which nerve of the lumbar plexus supplies the skin of the lateral thigh?
a) Genitofemoral
b) Lateral femoral cutaneous
c) Ilioinguinal
d) Obturator
Explanation: The lateral femoral cutaneous nerve, a branch of the lumbar plexus (L2–L3), supplies sensation to the lateral thigh. Lesion causes meralgia paresthetica. Correct answer: b) Lateral femoral cutaneous.
3. A patient presents with difficulty adducting the thigh. Which nerve is affected?
a) Obturator nerve
b) Femoral nerve
c) Sciatic nerve
d) Iliohypogastric nerve
Explanation: The obturator nerve supplies adductor muscles of the medial thigh. Injury leads to loss of thigh adduction and sensory loss in medial thigh. Correct answer: a) Obturator nerve.
4. The femoral nerve arises from which roots?
a) L2–L4
b) L1–L3
c) L3–L5
d) T12–L2
Explanation: The femoral nerve is the largest branch of the lumbar plexus and arises from L2–L4 roots. It supplies anterior thigh muscles and skin over anterior thigh and medial leg. Correct answer: a) L2–L4.
5. During hernia repair, which nerve is most at risk of injury in the inguinal canal?
a) Femoral
b) Ilioinguinal
c) Obturator
d) Genitofemoral
Explanation: The ilioinguinal nerve passes through the inguinal canal and is commonly at risk during hernia surgeries. Its injury causes numbness over groin and upper medial thigh. Correct answer: b) Ilioinguinal.
6. Which lumbar plexus nerve supplies the cremaster muscle in males?
a) Ilioinguinal
b) Genitofemoral
c) Obturator
d) Femoral
Explanation: The genital branch of the genitofemoral nerve supplies the cremaster muscle and scrotal skin in males. It is tested by the cremasteric reflex. Correct answer: b) Genitofemoral.
7. A patient has anesthesia over the anterior thigh and medial leg. Which nerve is most likely damaged?
a) Obturator
b) Femoral
c) Ilioinguinal
d) Genitofemoral
Explanation: The femoral nerve supplies sensation to the anterior thigh and via the saphenous branch to medial leg. Damage results in anesthesia and weakness of knee extension. Correct answer: b) Femoral nerve.
8. Which nerve is NOT a branch of the lumbar plexus?
a) Genitofemoral
b) Pudendal
c) Iliohypogastric
d) Lateral femoral cutaneous
Explanation: The pudendal nerve arises from the sacral plexus (S2–S4) and is not a branch of the lumbar plexus. Correct answer: b) Pudendal.
9. A patient develops burning pain over the lateral thigh after tight clothing. Which nerve is compressed?
a) Obturator
b) Lateral femoral cutaneous
c) Genitofemoral
d) Femoral
Explanation: Compression of the lateral femoral cutaneous nerve (L2–L3) causes meralgia paresthetica, presenting with burning pain and numbness over lateral thigh. Correct answer: b) Lateral femoral cutaneous.
10. Which of the following supplies the quadriceps femoris muscle?
a) Femoral nerve
b) Obturator nerve
c) Lateral femoral cutaneous
d) Genitofemoral
Explanation: The quadriceps femoris is the chief extensor of the knee, innervated by the femoral nerve (L2–L4). Correct answer: a) Femoral nerve.
11. A pelvic fracture injures the obturator nerve. Which action is lost?
a) Hip flexion
b) Hip extension
c) Hip adduction
d) Knee extension
Explanation: The obturator nerve supplies adductor muscles. Injury results in loss of thigh adduction with sensory deficit over medial thigh. Correct answer: c) Hip adduction.
Topic: Nerve Supply of Foot Muscles
Subtopic: Lumbricals of Foot
Keyword Definitions:
Lumbricals (foot): Four small intrinsic muscles of the foot, flex metatarsophalangeal joints and extend interphalangeal joints.
Medial plantar nerve: Branch of tibial nerve, supplies abductor hallucis, flexor digitorum brevis, flexor hallucis brevis, and 1st lumbrical.
Lateral plantar nerve: Branch of tibial nerve, supplies most intrinsic muscles of the foot including 2nd–4th lumbricals.
Tibial nerve: Main posterior leg nerve, parent of medial and lateral plantar nerves.
Peroneal nerve: Supplies anterior and lateral compartments of leg, not lumbricals.
Lead Question - 2014
3rd and 4th lumbrical (lateral two lumbricals) of foot are supplied by?
a) Medial plantar nerve
b) Lateral plantar nerve
c) Peroneal nerve
d) None of the above
Explanation: The 1st lumbrical of the foot is supplied by the medial plantar nerve. The 2nd, 3rd, and 4th lumbricals are supplied by the lateral plantar nerve. Thus, the 3rd and 4th lumbricals specifically receive innervation from the lateral plantar nerve. Correct answer: Lateral plantar nerve.
1) Which lumbrical of the foot is supplied by medial plantar nerve?
a) 1st
b) 2nd
c) 3rd
d) 4th
Explanation: Only the first lumbrical of the foot is supplied by the medial plantar nerve. The remaining lumbricals (2nd to 4th) are supplied by the lateral plantar nerve. Correct answer is 1st.
2) A patient presents with weakness in toe flexion at metatarsophalangeal joints. Which muscles are primarily involved?
a) Lumbricals
b) Interossei
c) Extensors
d) Plantar aponeurosis
Explanation: Lumbricals flex the metatarsophalangeal joints and extend the interphalangeal joints. Weakness in these movements is indicative of lumbrical muscle dysfunction. Correct answer is Lumbricals.
3) Which nerve injury leads to loss of function of lateral three lumbricals of foot?
a) Medial plantar nerve
b) Lateral plantar nerve
c) Deep peroneal nerve
d) Superficial peroneal nerve
Explanation: The lateral plantar nerve supplies the 2nd, 3rd, and 4th lumbricals. Injury to this nerve results in loss of function of the lateral three lumbricals. Correct answer is Lateral plantar nerve.
4) Which compartment of foot contains lumbricals?
a) Medial
b) Lateral
c) Central
d) Interosseous
Explanation: The lumbricals are located in the central compartment of the foot, along with flexor digitorum brevis and tendons of flexor digitorum longus. Correct answer is Central.
5) Lumbricals of the foot act on which joints?
a) Only ankle
b) Only knee
c) Metatarsophalangeal and interphalangeal joints
d) Tarsal joints
Explanation: Lumbricals flex the metatarsophalangeal joints and extend interphalangeal joints, thus balancing flexors and extensors during walking. Correct answer is Metatarsophalangeal and interphalangeal joints.
6) A football player has tibial nerve injury at ankle. Which lumbrical remains functional?
a) 1st
b) 2nd
c) 3rd
d) None
Explanation: Both medial and lateral plantar nerves are branches of tibial nerve. Injury at ankle compromises all lumbricals. Hence, no lumbrical remains functional. Correct answer is None.
7) Which intrinsic foot muscles are supplied by medial plantar nerve along with 1st lumbrical?
a) Flexor hallucis brevis, abductor hallucis, flexor digitorum brevis
b) Interossei
c) Adductor hallucis
d) Lateral two lumbricals
Explanation: Medial plantar nerve supplies abductor hallucis, flexor hallucis brevis, flexor digitorum brevis, and 1st lumbrical. The rest are supplied by the lateral plantar nerve. Correct answer is Flexor hallucis brevis, abductor hallucis, flexor digitorum brevis.
8) In clawing of toes due to lumbrical paralysis, what happens?
a) Hyperextension of MTP, flexion of IP
b) Flexion of MTP, extension of IP
c) Both extended
d) Both flexed
Explanation: Lumbricals normally flex the MTP joints and extend the IP joints. Paralysis leads to opposite deformity: hyperextension at MTP and flexion at IP joints, causing claw toe deformity. Correct answer is Hyperextension of MTP, flexion of IP.
9) Which tendon gives origin to lumbricals of foot?
a) Flexor digitorum longus
b) Flexor digitorum brevis
c) Extensor digitorum longus
d) Extensor digitorum brevis
Explanation: All four lumbricals of the foot arise from tendons of flexor digitorum longus. Correct answer is Flexor digitorum longus.
10) A patient with lateral plantar nerve injury is most likely to lose which movement?
a) Flexion of great toe
b) Extension of toes
c) Flexion at MTP and extension at IP of lateral toes
d) Inversion of foot
Explanation: Lateral plantar nerve supplies lateral three lumbricals. Their paralysis causes inability to flex MTP and extend IP joints of lateral toes. Correct answer is Flexion at MTP and extension at IP of lateral toes.
Topic: Upper Limb
Subtopic: Brachial Artery and Radial Nerve Relations
Keyword Definitions:
Profunda brachii artery: Deep branch of brachial artery, runs in the spiral groove of humerus, supplies posterior compartment of arm.
Spiral groove: Groove on posterior aspect of humerus where radial nerve and profunda brachii artery travel together.
Radial nerve: Nerve of posterior compartment of arm, supplies triceps and forearm extensors, runs with profunda brachii artery in spiral groove.
Ulnar nerve: Runs medial arm, behind medial epicondyle, unrelated to profunda brachii artery.
Median nerve: Runs with brachial artery in anterior compartment of arm.
Lead Question - 2014
Nerve running along with profunda brachii artery, in spiral groove?
a) Ulnar
b) Median
c) Radial
d) None
Explanation: The radial nerve runs in the spiral groove of humerus alongside the profunda brachii artery, supplying the posterior compartment of the arm. Injury here can lead to wrist drop. Correct answer is Radial.
Guessed Questions
1. Nerve vulnerable in mid-shaft humerus fracture?
a) Median
b) Radial
c) Ulnar
d) Musculocutaneous
Explanation: Mid-shaft humerus fractures can injure the radial nerve as it lies in the spiral groove along with the profunda brachii artery. This can cause wrist drop and sensory deficits over dorsum of hand. Correct answer is Radial.
2. Profunda brachii artery is a branch of?
a) Axillary artery
b) Brachial artery
c) Subclavian artery
d) Radial artery
Explanation: The profunda brachii artery is the deep branch of the brachial artery, running in the posterior compartment of arm alongside the radial nerve in the spiral groove. Correct answer is Brachial artery.
3. Muscle supplied by nerve in spiral groove?
a) Biceps brachii
b) Triceps brachii
c) Brachialis
d) Pronator teres
Explanation: The radial nerve running in the spiral groove supplies the triceps brachii muscle of posterior compartment. Injury at this site affects elbow extension. Correct answer is Triceps brachii.
4. Sensory area of radial nerve in arm?
a) Medial forearm
b) Dorsum of lateral hand
c) Palm of hand
d) Posterior arm only
Explanation: The radial nerve provides sensory innervation to the dorsum of lateral hand and posterior arm/forearm. Mid-shaft injury in spiral groove can cause sensory deficits here. Correct answer is Dorsum of lateral hand.
5. Complication of radial nerve injury in spiral groove?
a) Claw hand
b) Wrist drop
c) Foot drop
d) Ape hand
Explanation: Injury to the radial nerve in the spiral groove leads to wrist drop due to paralysis of wrist extensors, while elbow extension may be preserved. Correct answer is Wrist drop.
6. Spiral groove is located on?
a) Anterior humerus
b) Posterior humerus
c) Medial epicondyle
d) Lateral epicondyle
Explanation: The spiral groove is a groove on the posterior aspect of the humerus where the radial nerve and profunda brachii artery travel together. Correct answer is Posterior humerus.
7. Nerve supplying forearm extensors?
a) Median
b) Radial
c) Ulnar
d) Musculocutaneous
Explanation: The radial nerve, after running in spiral groove with profunda brachii artery, continues to innervate forearm extensors. Injury leads to weakness in wrist and finger extension. Correct answer is Radial.
8. Profunda brachii artery enters posterior compartment via?
a) Triangular interval
b) Quadrangular space
c) Cubital fossa
d) Medial intermuscular septum
Explanation: The profunda brachii artery enters the posterior compartment of arm through the triangular interval, running with the radial nerve in spiral groove. Correct answer is Triangular interval.
9. Clinical test for radial nerve injury?
a) Elbow flexion test
b) Wrist extension test
c) Thumb opposition test
d) Finger abduction test
Explanation: The wrist extension test assesses function of the radial nerve. Injury in spiral groove causes wrist drop, inability to extend wrist. Correct answer is Wrist extension test.
10. Surgical landmark for profunda brachii artery?
a) Lateral epicondyle
b) Spiral groove of humerus
c) Medial epicondyle
d) Olecranon
Explanation: The spiral groove of humerus is the surgical landmark where the profunda brachii artery runs with radial nerve. Knowledge helps prevent iatrogenic injury. Correct answer is Spiral groove of humerus.
Topic: Upper Limb
Subtopic: Cubital Fossa Structures
Keyword Definitions:
Bicipital aponeurosis: A broad fibrous expansion from biceps brachii tendon crossing cubital fossa, protecting underlying structures.
Cubital fossa: Triangular area anterior to elbow joint, containing important nerves and vessels.
Brachial artery: Main artery of upper arm continuing from axillary artery to cubital fossa.
Ulnar nerve: Nerve passing medial to cubital fossa, supplying intrinsic hand muscles.
Anterior interosseous artery: Branch of ulnar artery running along forearm interosseous membrane.
Lead Question - 2014
Bicipital aponeurosis lies over which structure in cubital fossa?
a) Ulnar nerve
b) Radial nerve
c) Brachial artery
d) Anterior interosseous artery
Explanation: The bicipital aponeurosis is a fibrous expansion from the biceps tendon that lies superficial to the brachial artery in the cubital fossa, providing protection to this major vessel during venipuncture or trauma. Correct answer is Brachial artery.
Guessed Questions
1. Ulnar nerve lies medial to which structure in cubital fossa?
a) Brachial artery
b) Bicipital aponeurosis
c) Median nerve
d) Radial artery
Explanation: The ulnar nerve passes medial to the cubital fossa, lying behind the medial epicondyle and not under the bicipital aponeurosis. Correct answer is Brachial artery lies anterior to it, but ulnar nerve is medial to cubital fossa structures.
2. Median nerve in cubital fossa is located?
a) Lateral to brachial artery
b) Medial to brachial artery
c) Superficial to bicipital aponeurosis
d) Posterior to brachial artery
Explanation: The median nerve lies medial to the brachial artery in cubital fossa and deep to the bicipital aponeurosis, ensuring protection during venipuncture. Correct answer is Medial to brachial artery.
3. Bicipital aponeurosis prevents injury to?
a) Radial nerve
b) Brachial artery
c) Ulnar artery
d) Median cubital vein
Explanation: The bicipital aponeurosis covers the brachial artery and median nerve in cubital fossa, protecting them from superficial lacerations during venipuncture or trauma. Correct answer is Brachial artery.
4. Cephalic vein in cubital fossa lies relative to bicipital aponeurosis?
a) Superficial
b) Deep
c) Lateral
d) Medial
Explanation: The cephalic vein lies superficial to the bicipital aponeurosis in the cubital fossa, allowing easy access for venipuncture without injuring deeper structures. Correct answer is Superficial.
5. Anterior interosseous artery is branch of?
a) Radial artery
b) Brachial artery
c) Ulnar artery
d) Median artery
Explanation: The anterior interosseous artery arises from the ulnar artery, passes along the interosseous membrane supplying deep flexor muscles of forearm. It is not covered by bicipital aponeurosis. Correct answer is Ulnar artery.
6. Structures passing deep to bicipital aponeurosis?
a) Median cubital vein
b) Brachial artery
c) Cephalic vein
d) Superficial radial nerve
Explanation: Brachial artery and median nerve lie deep to the bicipital aponeurosis in cubital fossa, providing protection from superficial trauma. Superficial veins lie above the aponeurosis. Correct answer is Brachial artery.
7. Radial nerve in cubital fossa lies?
a) Medial to biceps tendon
b) Lateral and deep to brachioradialis
c) Superficial to brachialis
d) Under bicipital aponeurosis
Explanation: The radial nerve passes laterally, deep to brachioradialis, and does not lie under the bicipital aponeurosis. Correct answer is Lateral and deep to brachioradialis.
8. Median cubital vein is located?
a) Superficial to bicipital aponeurosis
b) Deep to bicipital aponeurosis
c) Deep to brachial artery
d) Between brachial artery and ulnar nerve
Explanation: Median cubital vein is superficial to the bicipital aponeurosis, connecting cephalic and basilic veins in cubital fossa, commonly used for venipuncture. Correct answer is Superficial to bicipital aponeurosis.
9. Clinical importance of bicipital aponeurosis?
a) Protects brachial artery
b) Guides venipuncture
c) Can be injured in lacerations
d) All of the above
Explanation: The bicipital aponeurosis protects brachial artery and median nerve, provides a landmark for venipuncture, and may be injured in trauma. Correct answer is All of the above.
10. Injury under bicipital aponeurosis may affect?
a) Radial nerve
b) Median nerve
c) Cephalic vein
d) Basilic vein
Explanation: Structures deep to bicipital aponeurosis include brachial artery and median nerve. Injury here can cause hemorrhage and loss of hand function. Radial nerve lies laterally and cephalic vein superficially. Correct answer is Median nerve.
Topic: Upper Limb
Subtopic: Hand Muscles Innervation
Keyword Definitions:
Small muscles of hand: Intrinsic hand muscles including thenar, hypothenar, lumbricals, and interossei, responsible for fine motor movements.
Innervation: Nerve supply to muscles, determining motor function.
C5-T1 roots: Spinal nerve roots forming the brachial plexus supplying upper limb muscles.
Median nerve: Nerve supplying thenar muscles and lateral two lumbricals.
Ulnar nerve: Nerve supplying hypothenar muscles, interossei, and medial two lumbricals.
Lead Question - 2014
Small muscles of hand are supplied by:
a) C3
b) C4
c) C6
d) C5-C7 , C8-T1
Explanation: The small intrinsic muscles of the hand are supplied by nerves originating from C5-C7 (median nerve) and C8-T1 (ulnar nerve). These roots form the brachial plexus, allowing fine motor control. Correct answer is C5-C7 , C8-T1.
Guessed Questions
1. Thenar muscles are supplied by?
a) Ulnar nerve
b) Median nerve
c) Radial nerve
d) Musculocutaneous nerve
Explanation: Thenar muscles controlling thumb movements are innervated by the median nerve, derived from C5-C7 roots of the brachial plexus. Ulnar nerve does not supply thenar muscles except adductor pollicis. Correct answer is Median nerve.
2. Hypothenar muscles are supplied by?
a) Median nerve
b) Ulnar nerve
c) Radial nerve
d) Axillary nerve
Explanation: Hypothenar muscles (flexor digiti minimi, abductor digiti minimi, opponens digiti minimi) controlling little finger movements are supplied by the ulnar nerve arising from C8-T1 roots. Correct answer is Ulnar nerve.
3. Lumbrical muscles innervation?
a) Median for all
b) Ulnar for all
c) Median for lateral two, Ulnar for medial two
d) Radial nerve
Explanation: Lumbricals 1 and 2 (lateral) are supplied by median nerve, lumbricals 3 and 4 (medial) by ulnar nerve. This allows coordinated finger flexion at MCP joints and extension at IP joints. Correct answer is Median for lateral two, Ulnar for medial two.
4. Interossei muscles are supplied by?
a) Median nerve
b) Ulnar nerve
c) Radial nerve
d) Axillary nerve
Explanation: Dorsal and palmar interossei of the hand, responsible for finger abduction and adduction, are supplied by ulnar nerve (C8-T1). Median nerve does not supply interossei. Correct answer is Ulnar nerve.
5. Injury to C8-T1 roots affects?
a) Shoulder abduction
b) Elbow flexion
c) Intrinsic hand muscles
d) Wrist extension
Explanation: C8-T1 nerve roots supply the intrinsic hand muscles via ulnar and median nerves. Damage results in weakness of fine motor control, claw hand deformity. Shoulder and elbow muscles are supplied by higher roots. Correct answer is Intrinsic hand muscles.
6. Claw hand is due to injury of?
a) Median nerve
b) Ulnar nerve
c) Both median and ulnar nerves
d) Radial nerve
Explanation: Claw hand deformity occurs when ulnar nerve is injured, leading to hyperextension at MCP and flexion at IP joints. Combined median and ulnar nerve lesions worsen intrinsic hand function. Correct answer is Both median and ulnar nerves.
7. Ape hand deformity is due to?
a) Median nerve injury
b) Ulnar nerve injury
c) Radial nerve injury
d) Axillary nerve injury
Explanation: Ape hand results from median nerve injury, causing loss of thumb opposition, atrophy of thenar muscles, and flattening of thenar eminence. Correct answer is Median nerve injury.
8. Median nerve arises from which roots?
a) C5-C6
b) C5-C7
c) C8-T1
d) C7-T1
Explanation: Median nerve originates from the brachial plexus, receiving contributions from C5-C7 (lateral cord) and C8-T1 (medial cord). Supplies most thenar muscles and lateral lumbricals. Correct answer is C5-C7.
9. Ulnar nerve arises from?
a) Lateral cord
b) Medial cord
c) Posterior cord
d) Musculocutaneous nerve
Explanation: Ulnar nerve arises from medial cord of brachial plexus (C8-T1), supplying hypothenar muscles, medial lumbricals, and interossei. Correct answer is Medial cord.
10. Fine motor control of hand depends on?
a) Only extrinsic muscles
b) Only intrinsic muscles
c) Intrinsic and extrinsic muscles
d) Only wrist muscles
Explanation: Fine hand movements require coordinated action of intrinsic muscles (thenar, hypothenar, interossei, lumbricals) and extrinsic muscles (flexors/extensors of forearm). Both median and ulnar nerves contribute. Correct answer is Intrinsic and extrinsic muscles.
Topic: Reflexes
Subtopic: Supinator (Brachioradialis) Reflex
Keyword Definitions:
Supinator jerk: Also called brachioradialis reflex, elicited by tapping brachioradialis tendon, leading to forearm flexion and supination.
Reflex arc: Neural pathway that mediates a reflex action, involving sensory input, spinal cord integration, and motor output.
C5, C6 roots: Cervical spinal nerve roots contributing to supinator (brachioradialis) reflex.
Neurological examination: Assessment of reflexes helps localize lesions in spinal cord or peripheral nerves.
Clinical relevance: Supinator jerk tests integrity of C5-C6 nerve roots and musculocutaneous/ radial nerves.
Lead Question - 2014
Root value of supinator jerk?
a) C3-C4
b) C4-C5
c) C5-C6
d) C8-T1
Explanation: The supinator (brachioradialis) jerk involves tapping the tendon of brachioradialis, causing flexion and supination of the forearm. This reflex is mediated by the C5-C6 spinal nerve roots. Correct answer is C5-C6.
Guessed Questions
1. Biceps jerk tests which nerve roots?
a) C5-C6
b) C6-C7
c) C7-C8
d) C8-T1
Explanation: The biceps jerk is elicited by tapping the biceps tendon, causing forearm flexion. It tests integrity of C5-C6 nerve roots and musculocutaneous nerve. Correct answer is C5-C6.
2. Triceps jerk assesses?
a) C5-C6
b) C6-C7
c) C7-C8
d) C8-T1
Explanation: The triceps reflex is elicited by tapping the triceps tendon, causing elbow extension. It tests C7-C8 roots and radial nerve function. Correct answer is C7-C8.
3. Supinator jerk is mediated by which nerve?
a) Median nerve
b) Musculocutaneous nerve
c) Radial nerve
d) Ulnar nerve
Explanation: The supinator jerk reflex involves contraction of brachioradialis, mediated by the radial nerve, with sensory input from C5-C6 roots. Correct answer is Radial nerve.
4. Forearm supination in supinator reflex tests integrity of?
a) Muscles only
b) Spinal cord only
c) C5-C6 roots and radial nerve
d) Peripheral nerves only
Explanation: Supinator jerk causes forearm supination, testing C5-C6 nerve roots and radial nerve. Both peripheral nerve and spinal roots must be intact. Correct answer is C5-C6 roots and radial nerve.
5. Absence of supinator jerk suggests?
a) Normal reflex
b) Upper motor neuron lesion
c) Lower motor neuron lesion at C5-C6
d) Cerebellar lesion
Explanation: Absence or diminution of supinator jerk indicates lower motor neuron lesion affecting C5-C6 nerve roots or radial nerve. Upper motor neuron lesions usually cause hyperreflexia. Correct answer is Lower motor neuron lesion at C5-C6.
6. Reinforcement technique for supinator reflex?
a) Jendrassik maneuver
b) Deep breath
c) Leg crossing
d) Valsalva maneuver
Explanation: Jendrassik maneuver (clenching teeth or interlocking fingers) enhances supinator reflex by increasing central excitability. Correct answer is Jendrassik maneuver.
7. Supinator reflex is classified as?
a) Superficial reflex
b) Deep tendon reflex
c) Pathological reflex
d) Cranial reflex
Explanation: The supinator jerk is a deep tendon reflex, elicited by tapping the tendon, involving monosynaptic reflex arc and testing spinal nerve root integrity. Correct answer is Deep tendon reflex.
8. Clinical significance of exaggerated supinator jerk?
a) LMN lesion
b) UMN lesion
c) Peripheral neuropathy
d) Muscle rupture
Explanation: Exaggerated or hyperactive supinator jerk indicates upper motor neuron lesion above C5-C6 level, causing hyperreflexia. Correct answer is UMN lesion.
9. Supinator reflex primarily tests which muscle?
a) Biceps brachii
b) Brachioradialis
c) Triceps brachii
d) Supinator muscle
Explanation: Supinator jerk is elicited by tapping the tendon of brachioradialis, causing forearm flexion and supination. Though supinator muscle participates in supination, the primary muscle tested is brachioradialis. Correct answer is Brachioradialis.
10. Supinator jerk is decreased in which condition?
a) Cervical radiculopathy C5-C6
b) Carpal tunnel syndrome
c) Cubital tunnel syndrome
d) Rotator cuff tear
Explanation: Supinator reflex is reduced or absent in cervical radiculopathy involving C5-C6 because the nerve roots and radial nerve contribution are impaired. Peripheral neuropathies in distal nerves do not affect this reflex. Correct answer is Cervical radiculopathy C5-C6.
Topic: Pectoral Region
Subtopic: Clavipectoral Fascia
Keyword Definitions:
Clavipectoral fascia: Deep fascia beneath pectoralis major, enclosing subclavius and pectoralis minor, extending from clavicle to axilla.
Lateral pectoral nerve: Nerve supplying pectoralis major, passes through clavipectoral fascia.
Median pectoral nerve: Nerve supplying pectoralis minor and part of major, pierces fascia near axilla.
Thoracoacromial vessels: Artery and vein branching from axillary vessels, pierce fascia near pectoralis minor.
Cephalic vein: Superficial vein of upper limb, runs in deltopectoral groove but does not pierce fascia.
Lead Question - 2014
Clavipectoral fascia is pierced by all except?
a) Lateral pectoral nerve
b) Median pectoral nerve
c) Thoracoacromial vessels
d) Cephalic vein
Explanation: The clavipectoral fascia is pierced by the lateral pectoral nerve, median pectoral nerve, and thoracoacromial vessels. However, the cephalic vein runs superficial in the deltopectoral groove and does not pierce the fascia. Therefore, the correct answer is Cephalic vein.
Guessed Questions
1. Lateral pectoral nerve primarily supplies?
a) Pectoralis minor
b) Pectoralis major
c) Subclavius
d) Serratus anterior
Explanation: The lateral pectoral nerve mainly supplies the pectoralis major muscle, especially its clavicular part. It passes through the clavipectoral fascia near pectoralis minor but does not innervate minor or other muscles. Correct answer is Pectoralis major.
2. Median pectoral nerve supplies?
a) Pectoralis major only
b) Pectoralis minor only
c) Pectoralis minor and part of major
d) Deltoid
Explanation: The median pectoral nerve pierces the clavipectoral fascia to supply pectoralis minor and the lower fibers of pectoralis major. It does not supply deltoid. Correct answer is Pectoralis minor and part of major.
3. Thoracoacromial artery branches into all except?
a) Pectoral branch
b) Acromial branch
c) Clavicular branch
d) Lateral thoracic branch
Explanation: The thoracoacromial artery pierces the clavipectoral fascia and branches into pectoral, acromial, clavicular, and deltoid branches. Lateral thoracic artery is separate from axillary artery. Correct answer is Lateral thoracic branch.
4. Cephalic vein drains into?
a) Axillary vein
b) Subclavian vein
c) Basilic vein
d) Brachial vein
Explanation: The superficial cephalic vein runs in deltopectoral groove and drains into the axillary vein near clavipectoral fascia. It does not pierce the fascia. Correct answer is Axillary vein.
5. Clavipectoral fascia encloses which muscles?
a) Pectoralis major
b) Pectoralis minor and subclavius
c) Deltoid
d) Serratus anterior
Explanation: The clavipectoral fascia lies deep to pectoralis major and encloses pectoralis minor and subclavius muscles. It does not enclose deltoid or serratus anterior. Correct answer is Pectoralis minor and subclavius.
6. Deltopectoral triangle is bounded by?
a) Clavicle, deltoid, pectoralis major
b) Scapula, deltoid, trapezius
c) Clavicle, trapezius, pectoralis minor
d) Deltoid, biceps, coracoid
Explanation: The deltopectoral triangle is formed by clavicle superiorly, deltoid laterally, and pectoralis major medially. It contains the cephalic vein and deltopectoral lymph nodes. Correct answer is Clavicle, deltoid, pectoralis major.
7. Clavipectoral fascia attaches inferiorly to?
a) First rib
b) Sternum
c) Axillary fascia
d) Coracoid process
Explanation: The clavipectoral fascia descends from clavicle and attaches inferiorly to the axillary fascia and encases subclavius and pectoralis minor. Correct answer is Axillary fascia.
8. Lateral pectoral nerve communicates with?
a) Medial pectoral nerve
b) Axillary nerve
c) Musculocutaneous nerve
d) Radial nerve
Explanation: The lateral pectoral nerve communicates with medial pectoral nerve around clavipectoral fascia, forming a nerve loop to pectoralis major. It does not communicate with axillary, musculocutaneous, or radial nerves. Correct answer is Medial pectoral nerve.
9. Piercing of clavipectoral fascia by vessels allows?
a) Superficial drainage
b) Nerve passage
c) Communication between axilla and pectoral region
d) Fat deposition
Explanation: Piercing of the fascia by thoracoacromial vessels and nerves allows communication between axilla and pectoral region and passage of nerves and vessels to superficial muscles. Correct answer is Communication between axilla and pectoral region.
10. Subclavius muscle pierces clavipectoral fascia?
a) Yes
b) No
c) Partially
d) Only medial fibers
Explanation: The subclavius muscle lies enclosed within the clavipectoral fascia; it does not pierce it. Only nerves and vessels pierce the fascia. Correct answer is No.
Topic: Upper Limb Nerves
Subtopic: Median Nerve and Branches
Keyword Definitions:
Anterior interosseous nerve: A branch of the median nerve, supplying deep flexors of the forearm and pronator quadratus.
Median nerve: Formed by medial and lateral cords, supplies most forearm flexors and hand muscles.
Radial nerve: A major nerve of the posterior arm, supplying extensors of the forearm and hand.
Ulnar nerve: Supplies intrinsic hand muscles and some forearm flexors.
Axillary nerve: Supplies deltoid and teres minor muscles, and shoulder sensation.
Lead Question - 2014
Anterior interosseous nerve is a branch of?
a) Radial nerve
b) Median nerve
c) Ulnar nerve
d) Axillary nerve
Explanation: The anterior interosseous nerve arises from the median nerve just below the elbow. It supplies flexor pollicis longus, lateral half of flexor digitorum profundus, and pronator quadratus. It does not provide cutaneous innervation. Therefore, the correct answer is Median nerve, which gives this important motor branch.
Guessed Questions
1. The anterior interosseous nerve supplies all except?
a) Flexor pollicis longus
b) Pronator quadratus
c) Flexor digitorum superficialis
d) Flexor digitorum profundus (lateral half)
Explanation: The anterior interosseous nerve supplies FPL, pronator quadratus, and the lateral half of FDP. Flexor digitorum superficialis is supplied by the median nerve but not its anterior interosseous branch. Thus, the correct answer is Flexor digitorum superficialis.
2. A patient with anterior interosseous nerve injury is unable to?
a) Flex distal phalanx of thumb
b) Flex proximal phalanx of thumb
c) Extend wrist
d) Abduct thumb
Explanation: Anterior interosseous nerve injury causes weakness in flexor pollicis longus, leading to inability to flex the distal phalanx of the thumb. Proximal flexion is intact, wrist extension involves radial nerve, and thumb abduction involves radial/median nerves. Thus, the correct answer is Flex distal phalanx of thumb.
3. A clinical sign of anterior interosseous nerve palsy is?
a) Ape thumb deformity
b) Hand of benediction
c) Pinch sign
d) Claw hand
Explanation: In anterior interosseous nerve palsy, patients cannot form a tip-to-tip pinch between thumb and index finger due to loss of FPL and FDP function. Instead, they approximate pads of fingers. This is called the Pinch sign, characteristic of AIN injury.
4. The median nerve in the forearm gives rise to?
a) Anterior interosseous nerve
b) Posterior interosseous nerve
c) Musculocutaneous nerve
d) Lateral pectoral nerve
Explanation: The median nerve gives off the anterior interosseous nerve below the elbow. The posterior interosseous is a branch of the radial nerve, musculocutaneous comes from the lateral cord, and lateral pectoral is from the lateral cord. Correct answer is Anterior interosseous nerve.
5. A patient with deep forearm pain and weakness of pinch grip but no sensory loss most likely has -
a) Median nerve lesion
b) Ulnar nerve lesion
c) Anterior interosseous nerve lesion
d) Radial nerve lesion
Explanation: Anterior interosseous nerve is purely motor. Its lesion causes deep forearm pain, loss of pinch grip, but no cutaneous sensory loss. Median and ulnar nerve lesions include sensory changes, radial nerve causes wrist drop. Correct answer is Anterior interosseous nerve lesion.
6. Flexor digitorum profundus is supplied by -
a) Median nerve alone
b) Ulnar nerve alone
c) Median and ulnar nerves
d) Radial nerve
Explanation: Flexor digitorum profundus has dual innervation. Lateral half (index and middle fingers) by anterior interosseous nerve (median), medial half (ring and little fingers) by ulnar nerve. Hence, correct answer is Median and ulnar nerves.
7. The anterior interosseous nerve runs along which artery?
a) Radial artery
b) Anterior interosseous artery
c) Posterior interosseous artery
d) Ulnar artery
Explanation: The anterior interosseous nerve runs on the anterior surface of the interosseous membrane, accompanying the anterior interosseous artery, a branch of the ulnar artery. Correct answer is Anterior interosseous artery.
8. Injury to anterior interosseous nerve affects which movement?
a) Thumb extension
b) Index finger DIP flexion
c) Wrist flexion
d) Elbow extension
Explanation: The anterior interosseous nerve supplies the lateral half of FDP, flexing DIP of index and middle fingers. Injury impairs DIP flexion of index finger. Wrist flexion is preserved by FCR, thumb extension by radial nerve, elbow extension by radial nerve. Correct answer is Index finger DIP flexion.
9. The "OK sign" test is used to diagnose -
a) Ulnar nerve palsy
b) Radial nerve palsy
c) Anterior interosseous nerve palsy
d) Axillary nerve palsy
Explanation: In anterior interosseous nerve palsy, the patient cannot form a circle using thumb and index finger tips, producing a flat "OK sign." This is diagnostic of Anterior interosseous nerve palsy. Other nerve lesions present with different clinical signs.
10. A 40-year-old with forearm fracture develops inability to flex thumb IP and index DIP joints, but no sensory loss. Which nerve is injured?
a) Ulnar nerve
b) Anterior interosseous nerve
c) Radial nerve
d) Musculocutaneous nerve
Explanation: Loss of thumb IP and index DIP flexion with no sensory loss indicates anterior interosseous nerve injury, as it supplies FPL and FDP lateral half. Ulnar nerve causes sensory loss, radial nerve affects extensors, musculocutaneous supplies arm flexors. Correct answer is Anterior interosseous nerve.
Subtopic: Neurotransmitter Receptors
Keyword Definitions:
• Ionic receptors: Ligand-gated ion channels that mediate fast synaptic transmission.
• NMDA: A subtype of glutamate receptor functioning as an ion channel.
• Kainate: Ionotropic glutamate receptor subtype controlling sodium influx.
• mGluR: Metabotropic glutamate receptor linked to G-proteins, not ionotropic.
• AMPA: Ionotropic glutamate receptor mediating fast excitatory transmission.
Lead Question - 2013
Ionic receptors are all except ?
a) NMDA
b) Kainate
c) mGluR
d) AMPA
Explanation: Ionotropic receptors are ligand-gated ion channels such as NMDA, Kainate, and AMPA, which mediate rapid excitatory synaptic transmission. Metabotropic glutamate receptors (mGluRs) are G-protein coupled receptors, slower in action and not ionic. Hence, the correct answer is c) mGluR.
1) Guess Question:
Which receptor subtype is blocked by magnesium at rest?
a) AMPA
b) NMDA
c) Kainate
d) mGluR
Explanation: NMDA receptors are blocked by magnesium ions at resting potential. Depolarization removes the block, allowing calcium and sodium influx. This mechanism is crucial for synaptic plasticity. Answer: b) NMDA.
2) Guess Question:
Which receptor subtype mediates most fast excitatory neurotransmission in CNS?
a) NMDA
b) AMPA
c) GABA-A
d) Glycine
Explanation: AMPA receptors mediate the majority of fast excitatory synaptic responses in the central nervous system by allowing sodium influx when glutamate binds. Answer: b) AMPA.
3) Guess Question:
Which neurotransmitter activates NMDA, AMPA, and Kainate receptors?
a) Acetylcholine
b) Glutamate
c) GABA
d) Glycine
Explanation: Glutamate is the major excitatory neurotransmitter in the CNS. It binds to NMDA, AMPA, and Kainate receptors, mediating excitatory neurotransmission and plasticity. Answer: b) Glutamate.
4) Guess Question:
A 65-year-old patient with Alzheimer’s disease benefits from memantine because it blocks:
a) GABA-A
b) NMDA
c) AMPA
d) Kainate
Explanation: Memantine is an NMDA receptor antagonist that reduces excitotoxicity caused by excessive glutamate activity in Alzheimer’s disease. Answer: b) NMDA.
5) Guess Question:
Which receptor subtype is a G-protein coupled receptor (GPCR)?
a) NMDA
b) AMPA
c) Kainate
d) mGluR
Explanation: Metabotropic glutamate receptors (mGluRs) are GPCRs that activate second messenger systems. They are not ionotropic receptors. Answer: d) mGluR.
6) Guess Question:
Inhibition of which receptor improves seizure control in epilepsy?
a) NMDA
b) AMPA
c) Kainate
d) All of the above
Explanation: Excitatory glutamate receptors like NMDA, AMPA, and Kainate are involved in seizure generation. Blocking them reduces excitability, making d) All of the above correct.
7) Guess Question:
A patient with excessive glutamate release may develop:
a) Excitotoxicity
b) Bradycardia
c) Hypoglycemia
d) Alkalosis
Explanation: Excessive glutamate activates ionotropic receptors like NMDA, leading to calcium overload and neuronal damage called excitotoxicity, common in stroke and trauma. Answer: a) Excitotoxicity.
8) Guess Question:
Which ion is mainly conducted by AMPA receptors?
a) Calcium
b) Sodium
c) Potassium
d) Chloride
Explanation: AMPA receptors mainly conduct sodium ions into the neuron, causing depolarization. Some AMPA subtypes may also allow calcium entry. Answer: b) Sodium.
9) Guess Question:
NMDA receptor activation requires binding of:
a) Glutamate only
b) Glycine only
c) Glutamate and glycine
d) GABA
Explanation: NMDA receptors require co-agonist binding of glutamate and glycine for activation. This dual requirement ensures controlled calcium influx. Answer: c) Glutamate and glycine.
10) Guess Question:
A patient on phencyclidine (PCP) shows psychotic symptoms due to blockade of:
a) NMDA receptors
b) AMPA receptors
c) Kainate receptors
d) mGluRs
Explanation: PCP blocks NMDA receptors, impairing glutamate transmission and causing dissociative and psychotic symptoms. Answer: a) NMDA receptors.
Chapter: Cerebral Circulation
Topic: Regulation of Cerebral Blood Flow
Subtopic: Effect of Exercise
Keyword Definitions:
• Cerebral blood flow: Volume of blood passing through 100g of brain tissue per minute, normally about 50 ml/100g/min.
• Autoregulation: Brain maintains constant blood flow despite blood pressure variations.
• Moderate exercise: Physical activity that increases heart rate moderately without excessive oxygen debt.
• Hypercapnia: Increased CO₂ levels, a strong regulator of cerebral blood flow.
• Hypoxia: Low oxygen, also increases cerebral blood flow.
Lead Question - 2013
What is the effect of moderate exercise on cerebral blood flow?
a) Does not change
b) Increases
c) Decreases
d) Initially decreases then increases
Explanation: During moderate exercise, cerebral blood flow remains unchanged due to autoregulation. Increased blood pressure and cardiac output are balanced by cerebral vasoconstriction, keeping flow constant. Correct answer: Does not change.
1) Which factor has the most potent effect on cerebral blood flow?
a) Oxygen
b) Carbon dioxide
c) Hydrogen ions
d) Nitric oxide
Explanation: Cerebral blood flow is most strongly influenced by arterial carbon dioxide concentration. Even slight increases in PaCO₂ cause marked vasodilation. Correct answer: Carbon dioxide.
2) A patient with head injury develops hypoventilation. What happens to cerebral blood flow?
a) Decreases
b) Increases
c) No change
d) Biphasic response
Explanation: Hypoventilation leads to hypercapnia, which dilates cerebral vessels and increases cerebral blood flow, raising intracranial pressure. Correct answer: Increases.
3) Normal cerebral blood flow is approximately?
a) 25 ml/100g/min
b) 35 ml/100g/min
c) 50 ml/100g/min
d) 75 ml/100g/min
Explanation: Normal cerebral blood flow is about 50 ml/100g/min in adults. This ensures adequate oxygen and glucose supply to neurons. Correct answer: 50 ml/100g/min.
4) During severe hypoxia, cerebral blood flow?
a) Increases
b) Decreases
c) Remains constant
d) Initially decreases then stabilizes
Explanation: Hypoxia stimulates cerebral vasodilation to maintain oxygen supply, leading to increased cerebral blood flow. Correct answer: Increases.
5) A patient undergoing hyperventilation during neurosurgery will have?
a) Increased cerebral blood flow
b) Decreased cerebral blood flow
c) No change
d) Fluctuating response
Explanation: Hyperventilation reduces PaCO₂ (hypocapnia), causing vasoconstriction and decreased cerebral blood flow, useful in lowering intracranial pressure. Correct answer: Decreased cerebral blood flow.
6) Which artery supplies the motor cortex controlling leg movement?
a) Anterior cerebral artery
b) Middle cerebral artery
c) Posterior cerebral artery
d) Basilar artery
Explanation: The anterior cerebral artery supplies the medial aspect of cerebral hemispheres, including the motor cortex area for lower limb control. Correct answer: Anterior cerebral artery.
7) Which brain region is most sensitive to hypoxia?
a) Hippocampus
b) Thalamus
c) Cerebellum
d) Medulla
Explanation: Hippocampal neurons are highly sensitive to hypoxia and ischemia, making them vulnerable to injury in low oxygen states. Correct answer: Hippocampus.
8) A patient with ischemic stroke due to middle cerebral artery occlusion will present with?
a) Hemiplegia sparing face
b) Hemiplegia involving face and arm more
c) Hemiplegia involving leg more
d) Ataxia only
Explanation: Middle cerebral artery occlusion causes contralateral hemiplegia involving face and upper limb more than lower limb. Correct answer: Hemiplegia involving face and arm more.
9) Cerebral perfusion pressure is calculated as?
a) Mean arterial pressure + intracranial pressure
b) Mean arterial pressure - intracranial pressure
c) Systolic blood pressure - intracranial pressure
d) Diastolic blood pressure + intracranial pressure
Explanation: Cerebral perfusion pressure = MAP – ICP. Adequate CPP is vital for brain oxygenation. Correct answer: Mean arterial pressure - intracranial pressure.
10) A patient with subarachnoid hemorrhage develops vasospasm. This leads to?
a) Increased cerebral blood flow
b) Decreased cerebral blood flow
c) Normal cerebral blood flow
d) Fluctuating cerebral blood flow
Explanation: Vasospasm after subarachnoid hemorrhage narrows cerebral arteries, reducing cerebral blood flow and causing ischemia. Correct answer: Decreased cerebral blood flow.
11) Autoregulation of cerebral blood flow is effective between which mean arterial pressures?
a) 30-80 mmHg
b) 50-150 mmHg
c) 70-200 mmHg
d) 90-220 mmHg
Explanation: Cerebral autoregulation maintains constant blood flow between MAP 50–150 mmHg. Outside this range, flow varies directly with pressure. Correct answer: 50-150 mmHg.
Topic: Autonomic Nervous System
Subtopic: Sympathetic Nervous System
Keyword Definitions:
- Sympathetic Noradrenergic Fibers: Sympathetic nerve fibers that release norepinephrine to activate target organs, primarily involved in the 'fight or flight' response.
- Blood Vessels: Vessels transporting blood throughout the body, regulated by sympathetic nerves for vasoconstriction and vasodilation.
- Sweat Gland: Glands producing sweat, involved in thermoregulation, uniquely innervated by sympathetic cholinergic fibers.
- Heart: Muscular organ pumping blood, controlled by sympathetic noradrenergic fibers increasing heart rate and contractility.
- Eye: Organ of vision, with sympathetic innervation controlling pupil dilation (mydriasis).
Lead Question - 2013
Which of the following does not have sympathetic noradrenergic fibers?
a) Blood vessels
b) Sweat gland
c) Heart
d) Eye
Answer and Explanation:
The correct answer is b) Sweat gland. Unlike other sympathetic target organs that use noradrenaline, sweat glands are uniquely innervated by sympathetic cholinergic fibers. This allows acetylcholine to mediate sweat secretion during thermoregulation. Blood vessels, heart, and eye receive sympathetic noradrenergic fibers, facilitating vasoconstriction, increased cardiac output, and pupil dilation.
Guessed Questions for NEET PG:
1. Sympathetic innervation of sweat glands uses:
a) Noradrenaline
b) Acetylcholine
c) Dopamine
d) Serotonin
Explanation: The correct answer is b) Acetylcholine. Sweat glands are innervated by sympathetic cholinergic fibers, which release acetylcholine, distinguishing them from most other sympathetic targets that use noradrenaline.
2. Which of the following is a function of sympathetic noradrenergic fibers?
a) Increase salivation
b) Decrease heart rate
c) Vasoconstriction
d) Pupil constriction
Explanation: The correct answer is c) Vasoconstriction. Sympathetic noradrenergic fibers release norepinephrine, causing blood vessels to constrict, thereby increasing blood pressure during stress.
3. Sympathetic noradrenergic fibers in the eye control:
a) Lens accommodation
b) Mydriasis
c) Eyelid closure
d) Tear secretion
Explanation: The correct answer is b) Mydriasis. Sympathetic noradrenergic fibers stimulate radial muscles of the iris to dilate the pupil, aiding vision in low light.
4. Which gland uses cholinergic fibers despite being sympathetic?
a) Adrenal medulla
b) Sweat gland
c) Salivary gland
d) Lacrimal gland
Explanation: The correct answer is b) Sweat gland. Sweat glands are unique as they are innervated by sympathetic cholinergic fibers, releasing acetylcholine for thermoregulation.
5. In sympathetic stimulation, the heart responds by:
a) Decreasing rate
b) Increasing rate and contractility
c) Vasodilation
d) Secreting hormones
Explanation: The correct answer is b) Increasing rate and contractility. Sympathetic noradrenergic fibers release norepinephrine, increasing heart rate and force of contraction during stress.
6. Blood vessel constriction during sympathetic activation occurs via:
a) Cholinergic fibers
b) Adrenergic fibers
c) Dopaminergic fibers
d) Serotonergic fibers
Explanation: The correct answer is b) Adrenergic fibers. Sympathetic noradrenergic fibers release norepinephrine to stimulate vasoconstriction, increasing blood pressure during 'fight or flight' response.
7. Which of the following does not use norepinephrine in its sympathetic innervation?
a) Heart
b) Sweat gland
c) Blood vessels
d) Eye
Explanation: The correct answer is b) Sweat gland. Sweat glands use acetylcholine despite being sympathetic targets, whereas the heart, blood vessels, and eye use norepinephrine.
8. Adrenergic fibers primarily release:
a) Acetylcholine
b) Noradrenaline
c) Dopamine
d) GABA
Explanation: The correct answer is b) Noradrenaline. Adrenergic fibers, part of the sympathetic nervous system, release norepinephrine to stimulate target organs during stress responses.
9. Sympathetic control of pupil dilation involves:
a) Sphincter pupillae muscle
b) Ciliary muscle
c) Radial muscle of iris
d) Orbicularis oculi
Explanation: The correct answer is c) Radial muscle of iris. Sympathetic noradrenergic fibers stimulate the radial muscle, causing pupil dilation (mydriasis) for improved vision in dim light.
10. Sympathetic noradrenergic fibers originate from:
a) Parasympathetic ganglia
b) Sympathetic ganglia
c) Spinal cord gray matter
d) Dorsal root ganglion
Explanation: The correct answer is b) Sympathetic ganglia. Postganglionic sympathetic noradrenergic fibers arise from sympathetic ganglia and innervate target organs, releasing norepinephrine.
Topic: Autonomic Nervous System
Subtopic: Reflexes and Intracranial Pressure Regulation
Keyword Definitions:
- Cushing Reflex: A physiological nervous system response to increased intracranial pressure leading to hypertension, bradycardia, and irregular respiration.
- Intracranial Pressure (ICP): The pressure exerted by fluids such as cerebrospinal fluid (CSF) inside the skull.
- Bradycardia: A slower than normal heart rate, typically less than 60 beats per minute.
- Tachypnoea: Abnormally rapid breathing.
Lead Question - 2013
Cushing reflex is associated with all except ?
a) Hypotension
b) Increased intracranial pressure
c) Bradycardia
d) Tachyponea
Answer and Explanation:
The correct answer is a) Hypotension. The Cushing reflex is a protective physiological response to increased intracranial pressure characterized by hypertension, bradycardia, and irregular or tachypnoea respiratory patterns. Hypotension is not associated with Cushing reflex. Instead, elevated ICP leads to increased systemic blood pressure to maintain cerebral perfusion.
Guessed Questions for NEET PG:
1. Which of the following is not a feature of Cushing reflex?
a) Hypertension
b) Bradycardia
c) Tachypnoea
d) Hypotension
Explanation: The correct answer is d) Hypotension. Cushing reflex results from increased intracranial pressure and leads to hypertension, bradycardia, and tachypnoea to maintain cerebral perfusion. Hypotension is not part of the reflex response.
2. The Cushing reflex is primarily a response to:
a) Hypoxia
b) Hypercapnia
c) Increased intracranial pressure
d) Dehydration
Explanation: The correct answer is c) Increased intracranial pressure. Cushing reflex activates to maintain cerebral perfusion in the face of elevated intracranial pressure by increasing systemic blood pressure, inducing bradycardia and irregular breathing.
3. Clinical manifestation of the Cushing reflex includes:
a) Tachycardia
b) Hypotension
c) Bradycardia
d) Hyperthermia
Explanation: The correct answer is c) Bradycardia. The Cushing reflex features bradycardia, hypertension, and irregular breathing, as a compensatory mechanism to maintain cerebral perfusion pressure during raised ICP.
4. Which cranial nerve is mainly involved in mediating bradycardia during Cushing reflex?
a) Trigeminal nerve
b) Vagus nerve
c) Hypoglossal nerve
d) Facial nerve
Explanation: The correct answer is b) Vagus nerve. Increased ICP stimulates the vagus nerve leading to bradycardia, as part of the Cushing reflex to balance elevated systemic blood pressure.
5. A patient with severe head injury shows high blood pressure, slow pulse, and irregular respiration. This is indicative of:
a) Cushing reflex
b) Mydriasis
c) Horner's syndrome
d) Meniere's disease
Explanation: The correct answer is a) Cushing reflex. The triad of hypertension, bradycardia, and irregular respiration signifies the body's response to high intracranial pressure to maintain cerebral blood flow.
6. A 40-year-old patient with traumatic brain injury develops bradycardia and hypertension. This reflex helps:
a) Reduce ICP
b) Increase cerebral perfusion
c) Decrease cerebral perfusion
d) Cause vasodilation
Explanation: The correct answer is b) Increase cerebral perfusion. The Cushing reflex increases systemic arterial pressure to counteract elevated intracranial pressure, thereby preserving cerebral perfusion.
7. Tachypnoea in Cushing reflex is due to:
a) Metabolic acidosis
b) Medullary ischemia
c) Hyperventilation
d) Hypoxia
Explanation: The correct answer is b) Medullary ischemia. Increased ICP compresses the medulla, leading to irregular respiratory patterns or tachypnoea as part of the Cushing reflex.
8. In Cushing reflex, systemic hypertension occurs to:
a) Increase cardiac output
b) Reduce heart rate
c) Maintain cerebral perfusion
d) Stimulate sweat glands
Explanation: The correct answer is c) Maintain cerebral perfusion. Systemic hypertension compensates for high ICP, ensuring adequate oxygen and nutrient delivery to the brain despite compression.
9. Which part of the brain senses increased ICP in Cushing reflex?
a) Hypothalamus
b) Medulla oblongata
c) Cerebellum
d) Thalamus
Explanation: The correct answer is b) Medulla oblongata. The medulla senses increased intracranial pressure, triggering autonomic responses like bradycardia and hypertension to maintain cerebral blood flow.
10. Which of the following is not part of Cushing's triad?
a) Hypertension
b) Bradycardia
c) Tachypnoea
d) Hypothermia
Explanation: The correct answer is d) Hypothermia. Cushing's triad includes hypertension, bradycardia, and irregular or tachypnoea respiration. Hypothermia is unrelated to the reflex response.
Topic: Hypothalamic Regulation
Subtopic: Orexigenic Neurons
Keyword Definitions:
Orexigenic Neurons: Neurons that stimulate appetite and increase food intake.
Dorsal Raphae: Nucleus in brainstem involved in serotonin production.
Locus Coeruleus: Brainstem nucleus involved in noradrenaline production.
Lateral Hypothalamic Area: Hypothalamic region promoting feeding behavior.
Hippocampus: Brain structure involved in memory formation.
Lead Question - 2013
Cell bodies of orexigenic neurons are present in?
a) Dorsal raphae
b) Locus coeruleus
c) Lateral hypothalamic area
d) Hippocampus
Answer & Explanation:
Answer: c) Lateral hypothalamic area.
Orexigenic neurons, which promote appetite, are primarily located in the lateral hypothalamic area. These neurons stimulate food intake by releasing neuropeptides that act on various brain regions. Dysfunction in this area may lead to eating disorders such as anorexia or obesity. The correct answer is lateral hypothalamic area.
1. Guessed Question
Which neurotransmitter is primarily involved in stimulating orexigenic neurons?
a) Dopamine
b) Norepinephrine
c) Neuropeptide Y
d) Serotonin
Answer & Explanation:
Answer: c) Neuropeptide Y.
Neuropeptide Y (NPY) is a powerful orexigenic neurotransmitter that stimulates food intake and reduces energy expenditure. It is synthesized in the hypothalamus and acts on various brain regions to promote hunger. Increased NPY activity is associated with increased appetite and obesity, while decreased levels relate to anorexia.
2. Guessed Question
Destruction of the lateral hypothalamic area causes:
a) Hyperphagia
b) Aphagia
c) Polydipsia
d) Polyuria
Answer & Explanation:
Answer: b) Aphagia.
Destruction of the lateral hypothalamic area leads to aphagia, which is the inability or refusal to eat. This area is crucial for hunger signaling. Damage to this region results in severe anorexia and weight loss. Therefore, the correct answer is aphagia, indicating its role in promoting food intake.
3. Guessed Question
Which of the following is NOT a function of the hypothalamus?
a) Regulation of hunger
b) Thermoregulation
c) Visual processing
d) Water balance regulation
Answer & Explanation:
Answer: c) Visual processing.
The hypothalamus regulates hunger, thirst, body temperature, and circadian rhythms but does not participate directly in visual processing. Visual information is processed by the occipital lobe and visual cortex. Therefore, visual processing is not a function of the hypothalamus, making it the correct choice for this question.
4. Guessed Question
Which hormone released by the hypothalamus stimulates appetite?
a) Leptin
b) Ghrelin
c) Insulin
d) Cortisol
Answer & Explanation:
Answer: b) Ghrelin.
Ghrelin, secreted by the stomach, acts on the hypothalamus to stimulate appetite and promote food intake. It increases before meals and decreases after food consumption. High ghrelin levels are associated with increased hunger and potential weight gain, while low levels relate to appetite suppression.
5. Guessed Question
Orexigenic neurons are activated during:
a) Postprandial state
b) Fasting state
c) Sleep
d) Physical exercise
Answer & Explanation:
Answer: b) Fasting state.
During fasting, orexigenic neurons in the lateral hypothalamic area become activated, increasing the secretion of hunger-promoting neuropeptides like NPY and AgRP. This stimulates appetite and encourages food-seeking behavior, aiming to restore energy balance. Therefore, the correct answer is fasting state.
6. Guessed Question
Which peptide inhibits orexigenic neurons?
a) Agouti-related peptide (AgRP)
b) Leptin
c) Neuropeptide Y (NPY)
d) Melanin-concentrating hormone (MCH)
Answer & Explanation:
Answer: b) Leptin.
Leptin, secreted by adipose tissue, inhibits orexigenic neurons and promotes satiety. High leptin levels signal sufficient energy stores, decreasing appetite. In obesity, leptin resistance may develop, impairing this regulation. Therefore, leptin is the key hormone that suppresses orexigenic neuronal activity.
7. Guessed Question
Orexigenic neurons primarily release which of the following?
a) Dopamine
b) Agouti-related peptide (AgRP)
c) Acetylcholine
d) GABA
Answer & Explanation:
Answer: b) Agouti-related peptide (AgRP).
Orexigenic neurons release Agouti-related peptide (AgRP) and neuropeptide Y (NPY), which stimulate appetite by antagonizing melanocortin receptors in the hypothalamus. These peptides promote feeding and reduce energy expenditure. Hence, AgRP is correctly identified as a primary mediator of orexigenic neuron action.
8. Guessed Question
Which clinical condition is associated with damage to the lateral hypothalamic area?
a) Obesity
b) Anorexia
c) Hypertension
d) Diabetes insipidus
Answer & Explanation:
Answer: b) Anorexia.
Damage to the lateral hypothalamic area causes severe anorexia and aphagia, leading to weight loss. This area is critical for hunger signaling. Absence of its function impairs the body's ability to initiate feeding behavior, resulting in decreased food intake and energy imbalance.
9. Guessed Question
Which of the following statements is true about orexigenic neurons?
a) Inhibit food intake
b) Located in hippocampus
c) Promote feeding behavior
d) Located in dorsal raphae
Answer & Explanation:
Answer: c) Promote feeding behavior.
Orexigenic neurons stimulate appetite and promote food intake by releasing neuropeptides such as NPY and AgRP. Located in the lateral hypothalamic area, they are activated during energy deficit states. Their primary role is to encourage feeding, contrasting with anorexigenic neurons that suppress appetite.
10. Guessed Question
Leptin resistance is commonly observed in:
a) Anorexia nervosa
b) Obesity
c) Hypothyroidism
d) Addison's disease
Answer & Explanation:
Answer: b) Obesity.
Leptin resistance occurs when high leptin levels fail to suppress appetite, commonly observed in obesity. This condition prevents adequate feedback to the hypothalamus, causing continued food intake despite sufficient energy stores. It is a key factor in the pathophysiology of obesity, leading to further weight gain.
Topic: Visual System
Subtopic: Retina and Photoreceptors
Keywords:
• Retina: Light-sensitive layer at the back of the eye.
• Cones: Photoreceptor cells responsible for color vision and visual acuity.
• Photoreceptor: Specialized cell that responds to light.
• Visual System: Structures and pathways involved in vision.
Lead Question - 2013 (September 2008)
Number of cones in Retina?
a) 3-5 millions
b) 10-20 millions
c) 25-50 millions
d) 50-100 millions
Answer and Explanation:
Correct answer is a) 3-5 millions. The human retina contains approximately 3 to 5 million cone photoreceptors concentrated in the central region called the fovea. These cones enable high-resolution color vision under bright light (photopic) conditions and are essential for tasks requiring fine visual detail. (50 words)
1. Rod cells are responsible for:
a) Color vision
b) Low-light vision
c) High-resolution vision
d) Motion detection
Explanation:
Rod cells are specialized for low-light (scotopic) vision, providing black and white images in dim conditions, with high sensitivity but low spatial resolution. (Answer: b)
2. Fovea centralis contains predominantly:
a) Rods
b) Cones
c) Bipolar cells
d) Ganglion cells
Explanation:
The fovea centralis contains the highest concentration of cone cells, essential for sharp central vision and color discrimination in bright light. (Answer: b)
3. Cone cells are most sensitive to which type of light?
a) Dim light
b) Bright light
c) Infrared light
d) Ultraviolet light
Explanation:
Cone cells function optimally in bright light conditions, enabling high acuity and color perception. (Answer: b)
4. Total number of rod cells in human retina is approximately:
a) 120 million
b) 6 million
c) 3 million
d) 1 million
Explanation:
The human retina contains around 120 million rod cells, which mediate vision in low-light conditions and are more numerous than cone cells. (Answer: a)
5. The three types of cones are sensitive to:
a) Red, Green, Blue wavelengths
b) Ultraviolet, Infrared, Visible
c) Alpha, Beta, Gamma
d) Rod, Cone, Bipolar
Explanation:
Cone cells are categorized into three types based on spectral sensitivity to red (long), green (medium), and blue (short) wavelengths, enabling color vision. (Answer: a)
6. Clinical condition related to cone dysfunction is called:
a) Night blindness
b) Color blindness
c) Glaucoma
d) Cataract
Explanation:
Color blindness is caused by defective or absent cone cells, impairing color discrimination, typically inherited and most commonly affecting red-green perception. (Answer: b)
7. Which layer of retina contains photoreceptors?
a) Ganglion cell layer
b) Inner nuclear layer
c) Outer nuclear layer
d) Plexiform layer
Explanation:
The photoreceptors, including rods and cones, are located in the outer nuclear layer of the retina, where they transduce light into neural signals. (Answer: c)
8. Cone density is maximum at:
a) Optic disc
b) Peripheral retina
c) Fovea centralis
d) Macula lutea
Explanation:
Cone density peaks in the fovea centralis, the central region of the retina, responsible for sharp and detailed central vision. (Answer: c)
9. Cone cells mediate which type of vision?
a) Scotopic
b) Photopic
c) Mesopic
d) None
Explanation:
Cone cells mediate photopic vision, functioning under bright light conditions, essential for color perception and high visual acuity. (Answer: b)
10. Cone dysfunction may lead to which of the following disorders?
a) Glaucoma
b) Achromatopsia
c) Retinitis pigmentosa
d) Optic neuritis
Explanation:
Achromatopsia is a congenital condition caused by cone dysfunction, resulting in color blindness, poor visual acuity, and photophobia. (Answer: b)
Topic: Motor System
Subtopic: Corticospinal Tract and Precentral Gyrus
Keywords:
• Precentral Gyrus: Brain region responsible for voluntary motor control.
• Corticospinal Tract: Major pathway transmitting motor commands from brain to spinal cord.
• Vision: Sensory perception of light and images.
• Olfaction: Sense of smell.
• Auditory: Related to hearing.
• Voluntary Movement: Conscious control of skeletal muscles.
Lead Question - 2013 (September 2008)
Precentral gyrus & corticospinal tract are essential for?
a) Vision
b) Olfaction
c) Auditory
d) Voluntary movement
Answer and Explanation:
Correct answer is d) Voluntary movement. The precentral gyrus (primary motor cortex) initiates voluntary movements, and the corticospinal tract transmits these motor signals from the cortex to the spinal motor neurons, enabling conscious control of muscle activity. This system is critical for purposeful and coordinated bodily movements. (50 words)
1. The primary function of the precentral gyrus is:
a) Sensory perception
b) Voluntary motor control
c) Balance regulation
d) Speech comprehension
Explanation:
The precentral gyrus, part of the frontal lobe, controls voluntary skeletal muscle movements by sending motor commands through the corticospinal tract. (Answer: b)
2. Corticospinal tract primarily carries:
a) Sensory signals
b) Autonomic signals
c) Voluntary motor commands
d) Reflex arcs
Explanation:
The corticospinal tract transmits voluntary motor commands from the precentral gyrus to spinal cord neurons, enabling conscious muscle control. (Answer: c)
3. Lesion of precentral gyrus causes:
a) Blindness
b) Loss of voluntary movement
c) Loss of smell
d) Hearing loss
Explanation:
Damage to the precentral gyrus results in paralysis or weakness of voluntary muscles contralateral to the lesion, as it is the primary motor cortex. (Answer: b)
4. Corticospinal tract decussation occurs at:
a) Midbrain
b) Medulla
c) Pons
d) Spinal cord
Explanation:
The corticospinal tract crosses (decussates) at the medullary pyramids, ensuring contralateral control of voluntary motor functions. (Answer: b)
5. Damage to corticospinal tract leads to:
a) Sensory loss
b) Muscle atrophy
c) Spastic paralysis
d) Loss of consciousness
Explanation:
Lesions of the corticospinal tract often produce spastic paralysis, characterized by increased muscle tone and exaggerated reflexes. (Answer: c)
6. Voluntary movement requires integration of:
a) Sensory input and motor commands
b) Reflex actions only
c) Autonomic responses
d) Visual stimuli alone
Explanation:
Voluntary movements involve integrating sensory inputs and motor commands from the precentral gyrus and other brain regions to execute precise actions. (Answer: a)
7. Corticospinal tract is also known as:
a) Pyramidal tract
b) Extrapyramidal tract
c) Spinothalamic tract
d) Vestibulospinal tract
Explanation:
The corticospinal tract is called the pyramidal tract due to its passage through the medullary pyramids and its critical role in voluntary movement control. (Answer: a)
8. Clinical sign of upper motor neuron lesion is:
a) Flaccid paralysis
b) Spasticity and hyperreflexia
c) Muscle fasciculations
d) Decreased tone
Explanation:
Upper motor neuron lesions affecting the corticospinal tract typically cause spasticity and hyperreflexia, due to loss of inhibitory control. (Answer: b)
9. Which lobe contains the precentral gyrus?
a) Parietal
b) Temporal
c) Frontal
d) Occipital
Explanation:
The precentral gyrus is located in the frontal lobe and houses the primary motor cortex responsible for voluntary motor control. (Answer: c)
10. The primary neurotransmitter of corticospinal neurons is:
a) Dopamine
b) Acetylcholine
c) Norepinephrine
d) GABA
Explanation:
Corticospinal neurons use glutamate as the primary excitatory neurotransmitter, but acetylcholine is released at the neuromuscular junction by lower motor neurons. (Answer: b)
Topic: Reflexes
Subtopic: Conditioned Reflex
Keywords:
• Conditioned Reflex: A learned response to a previously neutral stimulus.
• Reinforcement: Process of strengthening a conditioned response.
• Habituation: Decreased response to a repeated benign stimulus.
• Innate Reflex: Inborn automatic response to a stimulus.
Lead Question - 2013 (September 2008)
Salivation of dog when food is given along with bell is?
a) Conditioned reflex
b) Reinforcement
c) Habituation
d) Innate reflex
Answer and Explanation:
Correct answer is a) Conditioned reflex. Pavlov’s experiment demonstrated that a neutral stimulus (bell) when paired with food becomes a conditioned stimulus, leading to salivation as a conditioned reflex. This is a classic example of learned behavior where the dog salivates to the bell alone after conditioning. (50 words)
1. Classical conditioning was first demonstrated by?
a) Skinner
b) Pavlov
c) Watson
d) Thorndike
Explanation:
Ivan Pavlov first demonstrated classical conditioning through experiments with dogs, showing that a neutral stimulus paired with food can trigger a conditioned reflex. (Answer: b)
2. In conditioned reflex, the stimulus-response association is:
a) Innate
b) Learned
c) Genetic
d) Random
Explanation:
Conditioned reflex involves a learned association between a neutral stimulus and a biologically significant stimulus, resulting in a new behavioral response. (Answer: b)
3. Reinforcement in conditioning is used to:
a) Weaken response
b) Strengthen conditioned response
c) Remove stimulus
d) Introduce habit
Explanation:
Reinforcement strengthens the conditioned reflex by increasing the probability that the conditioned response occurs following the conditioned stimulus. (Answer: b)
4. Habituation leads to:
a) Increased response
b) Unchanged response
c) Decreased response to repeated stimulus
d) Conditioned reflex formation
Explanation:
Habituation is a process where the organism gradually decreases its response to a harmless stimulus when presented repeatedly over time. (Answer: c)
5. Example of innate reflex is:
a) Pavlov's dog salivation
b) Knee jerk reflex
c) Learning to read
d) Phobia formation
Explanation:
An innate reflex, like the knee jerk reflex, is a pre-programmed automatic response not dependent on prior learning or experience. (Answer: b)
6. Which part of the brain is critical for conditioned reflex?
a) Cerebellum
b) Hypothalamus
c) Cerebral Cortex
d) Medulla
Explanation:
The cerebral cortex is primarily responsible for conditioned reflex formation by integrating sensory input and associating new stimulus-response patterns. (Answer: c)
7. Pavlov’s experiment demonstrated which type of learning?
a) Operant conditioning
b) Classical conditioning
c) Observational learning
d) Habituation
Explanation:
Pavlov’s dog experiment is a seminal example of classical conditioning, where a neutral stimulus paired with an unconditioned stimulus elicits a conditioned response. (Answer: b)
8. A neutral stimulus becomes a conditioned stimulus when:
a) It naturally triggers a response
b) It is repeatedly paired with an unconditioned stimulus
c) It is presented alone
d) It loses its effect
Explanation:
A neutral stimulus becomes conditioned by being repeatedly paired with an unconditioned stimulus until it elicits the response on its own. (Answer: b)
9. Conditioned reflex differs from innate reflex because:
a) Both are genetically programmed
b) Conditioned reflex is learned, innate is inborn
c) Innate reflex involves reinforcement
d) Conditioned reflex is involuntary
Explanation:
Conditioned reflex is acquired through learning and experience, whereas innate reflexes are inborn and do not require learning or practice. (Answer: b)
10. An example of conditioned reflex in humans is:
a) Sneezing
b) Salivating to the smell of food
c) Knee jerk
d) Cough reflex
Explanation:
Salivating at the smell or sight of food is a conditioned reflex in humans, where a neutral stimulus becomes associated with food anticipation. (Answer: b)
Topic: Cerebellum
Subtopic: Cerebellar Nuclei Functions
Keywords:
• Cerebellar Nucleus: A deep collection of nerve cells in the cerebellum that processes input and output signals.
• Caudate Nucleus: Part of basal ganglia involved in motor control and learning.
• Subthalamic Nucleus: Basal ganglia structure involved in movement regulation.
• Fastigial Nucleus: A deep cerebellar nucleus involved in balance and posture control.
• Putamen: Basal ganglia nucleus important for motor skills.
Lead Question - 2013 (September 2008)
Which of the following is a cerebellar nucleus?
a) Caudate nucleus
b) Subthalamic nucleus
c) Fastigial nucleus
d) Putamen
Answer and Explanation:
Correct answer is c) Fastigial nucleus. The fastigial nucleus is one of the deep cerebellar nuclei involved in the coordination of balance and posture. It processes input from the cerebellar cortex and integrates vestibular and proprioceptive signals. Other choices are part of the basal ganglia, not cerebellum. (50 words)
1. The dentate nucleus is associated with:
a) Muscle tone regulation
b) Planning of voluntary movements
c) Sensory perception
d) Reflex actions
Explanation:
The dentate nucleus, largest of the cerebellar nuclei, is involved in planning, initiation, and control of voluntary movements by connecting the cerebellum with motor areas of the cerebral cortex. (Answer: b)
2. Which is not a cerebellar deep nucleus?
a) Fastigial
b) Globus pallidus
c) Interposed
d) Dentate
Explanation:
Globus pallidus is part of the basal ganglia, not cerebellar nuclei. Cerebellar deep nuclei include fastigial, interposed, and dentate nuclei, important for motor control. (Answer: b)
3. Lesion in fastigial nucleus may lead to:
a) Ataxia
b) Loss of fine touch
c) Aphasia
d) Visual field defect
Explanation:
Lesions in the fastigial nucleus affect posture and balance, causing ataxia due to disrupted vestibulocerebellar connections. (Answer: a)
4. Interposed nucleus controls:
a) Muscle tone
b) Limb movement coordination
c) Visual processing
d) Auditory perception
Explanation:
Interposed nucleus (globose and emboliform) plays a role in regulating limb movement coordination and fine motor control via cerebellar pathways. (Answer: b)
5. The primary function of cerebellar nuclei is to:
a) Generate motor commands
b) Relay and modulate signals from cerebellar cortex
c) Process sensory input
d) Regulate consciousness
Explanation:
Cerebellar nuclei relay and modulate signals from the cerebellar cortex to motor areas, integrating sensory and motor information to coordinate movement. (Answer: b)
6. Cerebellar damage presents clinically with:
a) Hemiplegia
b) Dysmetria
c) Aphasia
d) Hemianopia
Explanation:
Cerebellar damage leads to dysmetria—impaired control of movement range and force—due to disrupted coordination, especially from deep nuclei dysfunction. (Answer: b)
7. Which is true about dentate nucleus?
a) It is involved in posture regulation
b) Largest cerebellar nucleus
c) Receives input from vestibular apparatus
d) Controls spinal reflexes
Explanation:
The dentate nucleus is the largest cerebellar nucleus and is primarily involved in the planning and initiation of voluntary movements. (Answer: b)
8. Cerebellar nuclei send output via:
a) Spinothalamic tract
b) Superior cerebellar peduncle
c) Inferior cerebellar peduncle
d) Medial lemniscus
Explanation:
Cerebellar nuclei send efferent output via the superior cerebellar peduncle, mainly targeting thalamus and motor cortex to regulate movement. (Answer: b)
9. Damage to subthalamic nucleus causes:
a) Hemiballismus
b) Ataxia
c) Aphasia
d) Hypotonia
Explanation:
Lesions in the subthalamic nucleus cause hemiballismus—a unilateral, involuntary flinging movement of limbs—due to basal ganglia dysfunction, not cerebellar. (Answer: a)
10. The cerebellar vermis is involved in:
a) Cognitive processing
b) Eye movement control
c) Body posture and locomotion
d) Hearing pathways
Explanation:
Cerebellar vermis contributes to control of axial muscles, body posture, and locomotion, integrating sensory inputs and motor coordination via cerebellar nuclei. (Answer: c)
Topic: Thalamus and Basal Ganglia
Subtopic: Thalamic Nuclei Functions
Keywords:
• Thalamic Nuclei: Relay centers in the brain transmitting sensory and motor signals.
• Basal Ganglia: Brain structures involved in movement control and coordination.
• Lateral Dorsal Nucleus: Thalamic relay nucleus involved in limbic system connections.
• Pulvinar: Thalamic nucleus involved in visual processing.
• Ventral Anterior Nucleus: Thalamic nucleus linked with motor control circuits.
• Intralaminar Nuclei: Group of thalamic nuclei involved in arousal, awareness, and basal ganglia function.
Lead Question - 2013 (September 2008)
Which thalamic nuclei can produce basal ganglia symptoms?
a) Lateral dorsal
b) Pulvinar
c) Ventral anterior
d) Intralaminar
Answer and Explanation:
Correct answer is d) Intralaminar. The intralaminar thalamic nuclei are involved in connecting the basal ganglia with the cortex and play a critical role in modulating motor function and awareness. Lesions in these nuclei can lead to symptoms mimicking basal ganglia dysfunction such as movement disorders or altered consciousness. (50 words)
1. The basal ganglia includes all except:
a) Caudate nucleus
b) Putamen
c) Thalamus
d) Globus pallidus
Explanation:
Thalamus is not part of the basal ganglia but acts as a relay station, whereas caudate, putamen, and globus pallidus are core basal ganglia structures involved in motor control. (Answer: c)
2. Lesion in ventral anterior nucleus causes:
a) Sensory loss
b) Movement disorder
c) Visual impairment
d) Hearing loss
Explanation:
Ventral anterior nucleus is part of the motor thalamus and its lesion typically results in movement disorders due to disrupted motor control pathways. (Answer: b)
3. Which pathway connects basal ganglia to cortex via thalamus?
a) Corticospinal tract
b) Corticothalamic pathway
c) Pallidothalamic tract
d) Spinothalamic tract
Explanation:
Pallidothalamic tract connects globus pallidus of basal ganglia to the thalamus, modulating motor signals relayed to the cortex. (Answer: c)
4. Lesions in intralaminar nuclei cause:
a) Aphasia
b) Basal ganglia symptoms
c) Cerebellar ataxia
d) Visual field defects
Explanation:
Intralaminar nuclei are connected to basal ganglia circuits; damage can cause movement disorders resembling basal ganglia disease. (Answer: b)
5. Pulvinar nucleus mainly affects:
a) Motor function
b) Visual processing
c) Sensory perception
d) Speech production
Explanation:
Pulvinar nucleus is primarily involved in visual processing and higher visual functions, not basal ganglia symptoms. (Answer: b)
6. Lateral dorsal nucleus connects primarily with:
a) Motor cortex
b) Limbic system
c) Visual cortex
d) Brainstem
Explanation:
Lateral dorsal nucleus is part of the limbic thalamic group, involved in emotional and memory functions, not basal ganglia symptoms. (Answer: b)
7. Basal ganglia symptoms include:
a) Bradykinesia
b) Aphasia
c) Hemianopia
d) Anosmia
Explanation:
Bradykinesia, or slowed movement, is a cardinal feature of basal ganglia dysfunction due to impaired modulation of motor circuits. (Answer: a)
8. Intralaminar nuclei receive input from:
a) Cerebral cortex
b) Brainstem reticular formation
c) Cerebellum
d) Peripheral nerves
Explanation:
Intralaminar nuclei receive extensive input from brainstem reticular formation and basal ganglia, crucial for arousal and motor coordination. (Answer: b)
9. Thalamic syndrome includes:
a) Motor weakness
b) Sensory loss with pain
c) Visual field defects
d) Aphasia
Explanation:
Thalamic syndrome is typically characterized by contralateral sensory loss with severe pain due to lesions in thalamic sensory relay nuclei. (Answer: b)
10. Basal ganglia is primarily involved in:
a) Memory processing
b) Voluntary motor control
c) Visual acuity
d) Hearing perception
Explanation:
Basal ganglia play a key role in voluntary motor control, movement initiation, and coordination, not in sensory processing like vision or hearing. (Answer: b)
Topic: Sympathetic Nervous System
Subtopic: Control of Sweat Glands
Keywords:
• Sweating: Process of producing sweat to regulate body temperature.
• Norepinephrine: Neurotransmitter involved in sympathetic nervous responses.
• Epinephrine: Hormone and neurotransmitter involved in fight or flight response.
• Acetylcholine: Neurotransmitter in both parasympathetic and sympathetic nervous systems.
• Histamine: Chemical involved in allergic reactions and inflammation.
Lead Question - 2013 (September 2008)
Sweating is mediated by ?
a) Norepinephrine
b) Epinephrine
c) Acetylcholine
d) Histamine
Answer and Explanation:
Correct answer is c) Acetylcholine. Unlike most sympathetic postganglionic neurons that release norepinephrine, sweat glands are activated by acetylcholine acting on muscarinic receptors. This unique exception allows thermoregulatory sweating to occur, especially in response to increased body temperature or emotional stress. (50 words)
1. Sympathetic preganglionic fibers release:
a) Acetylcholine
b) Norepinephrine
c) Epinephrine
d) Dopamine
Explanation:
Sympathetic preganglionic fibers always release acetylcholine at synapses with postganglionic neurons, stimulating them to release norepinephrine or acetylcholine depending on the target tissue. (Answer: a)
2. Which receptor type mediates sweating?
a) Alpha-1 adrenergic
b) Beta-2 adrenergic
c) Muscarinic cholinergic
d) Nicotinic cholinergic
Explanation:
Sweating is mediated by muscarinic cholinergic receptors on sweat glands activated by acetylcholine, differing from typical adrenergic sympathetic responses. (Answer: c)
3. In hyperhidrosis, excessive sweating is primarily due to:
a) Increased norepinephrine
b) Increased acetylcholine
c) Decreased dopamine
d) Decreased epinephrine
Explanation:
Hyperhidrosis is commonly caused by overactivity of cholinergic sympathetic fibers, leading to excessive acetylcholine release and profuse sweating. (Answer: b)
4. Sweat glands controlled by sympathetic system are called:
a) Apocrine glands
b) Eccrine glands
c) Sebaceous glands
d) Ceruminous glands
Explanation:
Eccrine sweat glands, responsible for thermoregulation, are controlled by the sympathetic nervous system via acetylcholine release onto muscarinic receptors. (Answer: b)
5. Which is not a function of sweating?
a) Temperature regulation
b) Excretion of toxins
c) Lubrication of skin
d) pH regulation
Explanation:
Sweating primarily regulates body temperature and eliminates waste; however, lubrication of skin is a function of sebaceous glands, not sweat glands. (Answer: c)
6. Emotional sweating is mediated by which part of nervous system?
a) Parasympathetic
b) Somatic
c) Sympathetic
d) Central
Explanation:
Emotional sweating is mediated by the sympathetic nervous system via cholinergic stimulation of eccrine glands, independent of thermoregulatory needs. (Answer: c)
7. Histamine acts primarily in:
a) Sweat gland stimulation
b) Vasodilation and allergic reactions
c) Muscle contraction
d) Thermoregulation
Explanation:
Histamine is involved in allergic responses and vasodilation but does not play a direct role in mediating sweat gland activity. (Answer: b)
8. Which is true regarding acetylcholine's role in the ANS?
a) Only released in parasympathetic pathways
b) Released in both sympathetic and parasympathetic pathways
c) Only released in somatic motor neurons
d) Not involved in ANS
Explanation:
Acetylcholine is the neurotransmitter at all preganglionic synapses and at parasympathetic postganglionic synapses, as well as for sympathetic postganglionic innervation of sweat glands. (Answer: b)
9. Dysfunction in acetylcholine receptors causes which condition?
a) Myasthenia gravis
b) Hyperthyroidism
c) Diabetes mellitus
d) Hypotension
Explanation:
Myasthenia gravis is an autoimmune disorder targeting acetylcholine receptors, leading to muscle weakness and impaired neuromuscular transmission. (Answer: a)
10. Eccrine sweat gland stimulation is predominantly:
a) Adrenergic
b) Cholinergic
c) Dopaminergic
d) Serotonergic
Explanation:
Eccrine sweat glands are stimulated by cholinergic fibers in the sympathetic nervous system, releasing acetylcholine onto muscarinic receptors. (Answer: b)
Topic: Cerebrospinal Fluid (CSF) Composition
Subtopic: Electrolyte Concentrations in CSF and Plasma
Keywords:
• Cerebrospinal fluid (CSF): Clear fluid surrounding brain and spinal cord, providing protection.
• Plasma: The liquid component of blood carrying cells and proteins.
• Electrolytes: Ions like Na+, K+, Cl-, Ca2+, important for cell function.
• Glucose: Primary energy source in body fluids.
• Bicarbonate (HCO3-): Important for pH buffering in body fluids.
Lead Question - 2013 (September 2008)
Which of the following has same concentration in CSF and plasma ?
a) Ca2+
b) HCO3
c) Glucose
d) Cl-
Answer and Explanation:
Correct answer is d) Cl-. Chloride ion concentration in CSF is approximately equal to plasma because it crosses the blood-brain barrier more freely compared to other ions. Ca2+, HCO3 and glucose are regulated and have different concentrations in CSF vs plasma to maintain CNS homeostasis. (50 words)
1. The main function of CSF is:
a) Oxygen transport
b) Nutrient delivery
c) Mechanical protection and chemical stability
d) Hormone transport
Explanation:
The primary function of CSF is to cushion the brain and spinal cord, maintain chemical stability, and remove waste products. It provides mechanical protection and optimal ionic environment for neuronal function. (Answer: c)
2. CSF is produced mainly by:
a) Arachnoid villi
b) Choroid plexus
c) Ependymal cells
d) Subarachnoid space
Explanation:
The majority of CSF is produced by the choroid plexus within the ventricles of the brain through selective filtration and active secretion mechanisms. (Answer: b)
3. Glucose concentration in CSF compared to plasma is generally:
a) Equal
b) Higher
c) Lower
d) Zero
Explanation:
CSF glucose concentration is typically about two-thirds of plasma glucose concentration due to restricted transport across the blood-brain barrier. (Answer: c)
4. In bacterial meningitis, CSF glucose level is usually:
a) Increased
b) Normal
c) Decreased
d) Unchanged
Explanation:
In bacterial meningitis, CSF glucose level typically decreases because bacteria consume glucose and inflammation impairs glucose transport across the blood-brain barrier. (Answer: c)
5. The blood-brain barrier allows easy passage of which ion into CSF?
a) K+
b) Na+
c) Cl-
d) Protein
Explanation:
Chloride ions (Cl-) can pass relatively freely across the blood-brain barrier, hence their concentrations in plasma and CSF are approximately equal. (Answer: c)
6. Bicarbonate concentration in CSF compared to plasma is:
a) Same
b) Lower
c) Higher
d) Zero
Explanation:
Bicarbonate concentration in CSF is slightly lower than in plasma due to selective transport, contributing to pH regulation in the CNS. (Answer: b)
7. Calcium concentration in CSF is generally:
a) Equal to plasma
b) Higher than plasma
c) Lower than plasma
d) Zero
Explanation:
Calcium concentration in CSF is lower than in plasma because tight regulation prevents excess calcium influx, protecting neurons from excitotoxicity. (Answer: c)
8. In which condition would CSF protein be markedly elevated?
a) Viral meningitis
b) Bacterial meningitis
c) Hypoglycemia
d) Hypocalcemia
Explanation:
CSF protein levels are markedly elevated in bacterial meningitis due to increased permeability of the blood-brain barrier and inflammation. (Answer: b)
9. The osmolarity of CSF compared to plasma is:
a) Lower
b) Higher
c) Equal
d) Variable
Explanation:
Osmolarity of CSF is nearly equal to plasma, maintained by regulated ion transport to preserve CNS homeostasis and prevent neuronal dysfunction. (Answer: c)
10. Which of the following is not a function of CSF?
a) Mechanical cushioning
b) Nutrient supply
c) Waste removal
d) Blood oxygen transport
Explanation:
CSF provides mechanical cushioning, supplies nutrients, and removes waste, but oxygen transport is primarily carried out by blood, not CSF. (Answer: d)
Topic: Reflex Actions
Subtopic: Withdrawal Reflex
Keywords:
• Withdrawal reflex: An automatic response to a painful stimulus to protect the body.
• Flexion: Bending a limb towards the body.
• Extension: Straightening a limb away from the body.
• Polysynaptic reflex: Reflex involving multiple synapses.
• Protective reflex: Prevents injury by removing body part from harm.
Lead Question - 2013 (September 2008)
What is seen in withdrawal reflex ?
a) Extension
b) Flexion
c) Extension followed by flexion
d) None of the above
Answer and Explanation:
Correct answer is b) Flexion. The withdrawal reflex is a protective polysynaptic reflex resulting in the flexion of a limb when exposed to a painful stimulus. This action helps rapidly remove the body part from harm. It involves sensory neurons, interneurons, and motor neurons. (50 words)
1. The withdrawal reflex primarily involves which type of neurons?
a) Sensory and motor only
b) Sensory, interneurons, and motor
c) Only motor neurons
d) Only sensory neurons
Explanation:
The withdrawal reflex is polysynaptic and involves sensory neurons detecting the painful stimulus, interneurons in the spinal cord transmitting the signal, and motor neurons triggering the flexor muscles to contract and withdraw the limb. (Answer: b)
2. Clinically, exaggerated withdrawal reflex indicates:
a) Normal response
b) Upper motor neuron lesion
c) Lower motor neuron lesion
d) Spinal shock
Explanation:
An exaggerated withdrawal reflex may suggest an upper motor neuron lesion, indicating disinhibition of spinal reflexes. It points toward a lack of supraspinal modulation. (Answer: b)
3. In the withdrawal reflex, the contralateral limb shows:
a) Flexion
b) Extension
c) No response
d) Tremors
Explanation:
During the withdrawal reflex, the crossed extensor reflex causes the contralateral limb to extend, providing balance and support when the affected limb withdraws from the painful stimulus. (Answer: b)
4. The withdrawal reflex is mediated at the level of the:
a) Brain
b) Spinal cord
c) Brainstem
d) Peripheral nerve
Explanation:
The withdrawal reflex is primarily mediated at the spinal cord level, allowing for rapid response without cortical involvement, which ensures speed and efficiency in protecting the body from harmful stimuli. (Answer: b)
5. In clinical testing, absence of the withdrawal reflex may indicate:
a) Upper motor neuron lesion
b) Lower motor neuron lesion
c) Normal finding
d) Hyperreflexia
Explanation:
An absent withdrawal reflex may suggest a lower motor neuron lesion, indicating damage to peripheral nerves or the spinal cord segment responsible for reflex arc, impairing the protective response. (Answer: b)
6. The speed of the withdrawal reflex is typically:
a) Very slow
b) Intermediate
c) Rapid
d) Variable
Explanation:
The withdrawal reflex is rapid to promptly remove the body part from painful stimuli, preventing injury. Its speed is enhanced by short reflex pathways and minimal synapses involved. (Answer: c)
7. Flexor muscles involved in withdrawal reflex are activated via:
a) Direct sensory neuron connections
b) Interneurons
c) Brain control
d) Hormonal signaling
Explanation:
Flexor muscles in the withdrawal reflex are activated via interneurons in the spinal cord, which transmit the pain signal from sensory neurons to motor neurons, causing the necessary contraction. (Answer: b)
8. Painful stimulus applied to the foot initiates the withdrawal reflex causing:
a) Flexion of leg
b) Extension of leg
c) No movement
d) Opposite limb movement only
Explanation:
A painful stimulus on the foot activates sensory receptors, initiating the withdrawal reflex. This leads to flexion of the affected leg to withdraw from harm, often coupled with contralateral limb extension for support. (Answer: a)
9. The withdrawal reflex is classified as:
a) Monosynaptic reflex
b) Polysynaptic reflex
c) Cranial reflex
d) Voluntary reflex
Explanation:
The withdrawal reflex is a polysynaptic reflex involving multiple interneurons and synapses, unlike monosynaptic reflexes like the knee jerk. This complexity allows for integration and modulation of the response. (Answer: b)
10. Which neurotransmitter is primarily involved in the withdrawal reflex?
a) Acetylcholine
b) Dopamine
c) Glutamate
d) GABA
Explanation:
Glutamate is the primary excitatory neurotransmitter involved in the withdrawal reflex, mediating signal transmission between sensory neurons, interneurons, and motor neurons in the spinal cord to activate muscle contraction. (Answer: c)
Topic: Cerebral Circulation
Subtopic: Cerebral Blood Flow (CBF)
Keywords:
• Cerebral blood flow (CBF): The volume of blood passing through the brain per unit time.
• ml/min: Milliliters per minute, unit of flow rate.
• Autoregulation: Brain's mechanism to maintain constant CBF despite changes in blood pressure.
• Ischemia: Insufficient blood flow causing tissue damage.
• Hyperemia: Increased blood flow to tissues.
Lead Question - 2013 (September 2008)
Normal cerebral blood flow in ml/min ?
a) 55
b) 150
c) 750
d) 1000
Answer and Explanation:
Correct answer is c) 750. Normal cerebral blood flow in an adult is approximately 750 ml/min, supplying essential oxygen and nutrients to brain tissue. Autoregulatory mechanisms maintain this flow across a range of systemic blood pressures. Insufficient CBF leads to ischemia, while excessive flow may cause hyperemia or edema. (50 words)
1. Cerebral autoregulation maintains constant blood flow at what mean arterial pressure range?
a) 50-150 mmHg
b) 70-110 mmHg
c) 90-160 mmHg
d) 40-120 mmHg
Explanation:
Cerebral autoregulation preserves stable cerebral blood flow despite systemic blood pressure changes, within a mean arterial pressure range of approximately 50–150 mmHg. Outside this range, autoregulation fails, risking ischemia or hyperemia. (Answer: a)
2. Which condition decreases cerebral blood flow?
a) Hypercapnia
b) Hypotension
c) Fever
d) Exercise
Explanation:
Hypotension reduces cerebral perfusion pressure, impairing blood flow to the brain, risking ischemia. Hypercapnia and fever increase cerebral blood flow, while exercise generally increases systemic flow, also enhancing cerebral perfusion. (Answer: b)
3. The major factor increasing cerebral blood flow is:
a) Hypocapnia
b) Hypercapnia
c) Hypothermia
d) Hypoxia
Explanation:
Hypercapnia (elevated CO2 levels) dilates cerebral vessels, significantly increasing cerebral blood flow. Hypocapnia causes vasoconstriction and reduces flow. Hypoxia increases flow as compensatory, but hypercapnia remains the primary regulator. (Answer: b)
4. Clinical correlation: In head trauma, monitoring cerebral perfusion pressure is vital because:
a) Prevents seizures
b) Avoids ischemia
c) Controls temperature
d) Reduces intracranial pressure
Explanation:
Monitoring cerebral perfusion pressure ensures adequate blood flow, preventing ischemic injury in traumatic brain injury. Insufficient perfusion worsens neuronal damage, while elevated pressure risks herniation. Seizures and temperature control are secondary considerations. (Answer: b)
5. In ischemic stroke, cerebral blood flow is typically:
a) Increased
b) Normal
c) Severely reduced
d) Unchanged
Explanation:
Ischemic stroke results from arterial blockage, leading to a marked decrease in cerebral blood flow to the affected region, risking permanent neuronal damage unless promptly reperfused. (Answer: c)
6. The primary unit of cerebral blood flow measurement is:
a) ml/min
b) ml/100g/min
c) mmHg
d) L/min
Explanation:
Cerebral blood flow is typically expressed in ml/100g/min to account for brain mass differences, though absolute flow in ml/min is also used. (Answer: b)
7. Hyperemia in the brain is caused by:
a) Hypocapnia
b) Hypoxia
c) Low blood glucose
d) Low body temperature
Explanation:
Hyperemia results from vasodilation, triggered by hypoxia or increased metabolic demand. Hypocapnia causes vasoconstriction, reducing flow, while hypoglycemia and low temperature reduce metabolic rate and flow. (Answer: b)
8. Normal cerebral blood flow per 100 grams of brain tissue is approximately:
a) 20-25 ml/min
b) 50-55 ml/min
c) 10-15 ml/min
d) 60-70 ml/min
Explanation:
Normal cerebral blood flow is around 50-55 ml/100g/min, ensuring adequate oxygen and nutrient supply. Values below 20 ml/100g/min risk ischemia. (Answer: b)
9. Which area of the brain has the highest blood flow?
a) White matter
b) Basal ganglia
c) Gray matter
d) Brainstem
Explanation:
Gray matter has the highest cerebral blood flow due to high metabolic demands, approximately 80 ml/100g/min, compared to white matter. This supports active neuronal processing. (Answer: c)
10. In conditions of chronic hypertension, cerebral autoregulation curve:
a) Shifts left
b) Shifts right
c) Remains unchanged
d) Becomes steeper
Explanation:
Chronic hypertension shifts the autoregulatory curve rightward, meaning higher pressures are needed to maintain normal cerebral blood flow, increasing risk of hypoperfusion if aggressive BP lowering occurs. (Answer: b)
Topic: Motor Pathways
Subtopic: Spinal Tracts and Motor Control
Keywords:
• Spinal pathway: Neural routes in the spinal cord conducting motor and sensory signals.
• Fine motor activity: Precise movements involving small muscles, e.g., fingers.
• Rubrospinal tract: Motor pathway from red nucleus involved in fine motor control.
• Vestibulospinal tract: Maintains posture and balance.
• Reticulospinal tract: Coordinates locomotion and postural control.
Lead Question - 2013 (September 2008)
Spinal pathway mainly regulating fine motor activity ?
a) Anterior corticospinal tract
b) Rubrospinal tract
c) Vestibulospinal tract
d) Reticulospinal tract
Answer and Explanation:
Correct answer is b) Rubrospinal tract. The rubrospinal tract originates in the red nucleus of midbrain and primarily controls fine motor activities of upper limbs, especially hand and fingers. It complements corticospinal control, while vestibulospinal and reticulospinal tracts are involved in posture and gross movements. (50 words)
1. The primary role of vestibulospinal tract is:
a) Fine motor control
b) Postural stability
c) Pain modulation
d) Conscious sensation
Explanation:
The vestibulospinal tract originates from vestibular nuclei and facilitates postural stability and balance, particularly in standing and walking. It does not participate in fine motor control or sensory functions, which are mediated by other pathways like corticospinal tract and sensory tracts. (Answer: b)
2. Which tract is most involved in voluntary fine motor movement?
a) Rubrospinal tract
b) Corticospinal tract
c) Reticulospinal tract
d) Vestibulospinal tract
Explanation:
The corticospinal tract is the most important for voluntary fine motor movement, particularly in distal muscles of limbs, although the rubrospinal tract supports fine motor control. Reticulospinal and vestibulospinal tracts are mainly involved in posture and gross movements. (Answer: b)
3. Damage to the rubrospinal tract results in:
a) Loss of pain sensation
b) Impaired fine motor control
c) Loss of balance
d) Hyperreflexia
Explanation:
The rubrospinal tract modulates fine motor control, particularly in upper limbs. Lesions here impair precise hand and finger movements. It does not affect pain sensation, balance, or cause hyperreflexia directly, which involve different pathways. (Answer: b)
4. Reticulospinal tract primarily controls:
a) Fine motor activity
b) Reflex arcs
c) Postural control and locomotion
d) Visual processing
Explanation:
The reticulospinal tract plays a major role in regulating posture and locomotion by modulating motor neurons controlling axial and proximal limb muscles. It does not govern fine motor skills, visual processing, or direct reflex arcs. (Answer: c)
5. Which of the following is NOT a function of the anterior corticospinal tract?
a) Voluntary movement of axial muscles
b) Fine distal limb control
c) Postural muscle control
d) Gross motor movements
Explanation:
Anterior corticospinal tract mainly controls axial and proximal muscles and contributes to posture and gross movements. Fine distal limb control is primarily by the lateral corticospinal tract, not anterior. (Answer: b)
6. Clinical correlation: A patient with midbrain stroke exhibits loss of fine motor control in upper limbs. The likely affected tract is:
a) Corticospinal tract
b) Vestibulospinal tract
c) Rubrospinal tract
d) Reticulospinal tract
Explanation:
Midbrain stroke may damage the rubrospinal tract, impairing fine upper limb movements. The corticospinal tract is more affected in cortical or spinal lesions. Vestibulospinal and reticulospinal tracts control posture and gross movements, not fine motor skills. (Answer: c)
7. Rubrospinal tract decussates at:
a) Medulla
b) Pons
c) Midbrain
d) Spinal cord
Explanation:
The rubrospinal tract decussates in the midbrain at the level of the red nucleus before descending in the lateral funiculus of the spinal cord. This decussation enables contralateral control of upper limb fine motor functions. (Answer: c)
8. The primary neurotransmitter of rubrospinal tract is:
a) Acetylcholine
b) Glutamate
c) GABA
d) Dopamine
Explanation:
The rubrospinal tract primarily uses glutamate as its neurotransmitter to stimulate motor neurons involved in fine control of limb muscles. Acetylcholine is used in neuromuscular junctions, and dopamine is mainly modulatory in basal ganglia pathways. (Answer: b)
9. The role of corticospinal tract vs rubrospinal tract is:
a) Both control gross movements
b) Both control fine motor activity equally
c) Corticospinal dominates fine motor; rubrospinal supports
d) Neither involved in fine motor control
Explanation:
The corticospinal tract is the primary pathway for voluntary fine motor control, especially of distal limbs. The rubrospinal tract acts as an auxiliary pathway supporting upper limb fine motor function, especially when corticospinal pathways are compromised. (Answer: c)
10. A lesion affecting the red nucleus primarily impacts:
a) Pain sensation
b) Proprioception
c) Fine motor control
d) Temperature sensation
Explanation:
The red nucleus gives rise to the rubrospinal tract, which modulates fine motor control of upper limbs. A lesion here results in impaired hand and finger movements, not pain or temperature sensation. (Answer: c)
Topic: Sensory Physiology
Subtopic: Mechanoreceptors
Keywords:
• Ruffini end organ: Slowly adapting mechanoreceptors detecting sustained skin stretch and pressure.
• Sensation: The process by which sensory receptors detect stimuli and transmit them to the brain.
• Sustained pressure: Continuous mechanical force applied to the skin over time.
• Heat receptors: Thermoreceptors detecting temperature changes.
• Touch receptors: Mechanoreceptors detecting light touch and pressure.
Lead Question - 2013 (September 2008)
Ruffini end organ is associated with sensation of:
a) Sustained Pressure
b) Heat
c) Touch
d) None of the above
Answer and Explanation:
Correct answer is a) Sustained Pressure. Ruffini endings are slow-adapting mechanoreceptors located in the dermis and joint capsules. They respond to skin stretch and continuous pressure, contributing to proprioception and detection of object manipulation. Heat is detected by thermoreceptors and touch by Meissner or Merkel corpuscles.
1. Meissner’s corpuscles detect:
a) Vibration
b) Light touch
c) Sustained pressure
d) Pain
Explanation:
Meissner's corpuscles are rapidly adapting mechanoreceptors present in glabrous skin. They are highly sensitive to light touch and low-frequency vibration, not sustained pressure or pain. Ruffini endings detect sustained pressure, while pain is sensed by free nerve endings. (Answer: b)
2. Pacinian corpuscles are primarily sensitive to:
a) Temperature
b) Sustained pressure
c) High-frequency vibration
d) Pain
Explanation:
Pacinian corpuscles are large encapsulated mechanoreceptors detecting high-frequency vibration and deep pressure. They adapt quickly and are not involved in detecting sustained pressure or pain. Ruffini endings are responsible for sustained pressure. (Answer: c)
3. Clinical case: Loss of Ruffini endings results in impairment of:
a) Pain perception
b) Temperature sensation
c) Proprioception and sustained pressure sensation
d) Light touch sensation
Explanation:
Ruffini endings contribute to proprioception and sustained pressure detection. Their loss impairs object manipulation and joint position sense, affecting fine motor skills. Pain and temperature remain unaffected as they are mediated by free nerve endings and thermoreceptors. (Answer: c)
4. Thermoreceptors primarily detect:
a) Sustained pressure
b) Temperature changes
c) Vibration
d) Light touch
Explanation:
Thermoreceptors are specialized nerve endings that detect temperature changes, signaling warmth or cold. Ruffini endings detect sustained pressure, while Meissner and Pacinian corpuscles detect touch and vibration respectively. Free nerve endings detect both pain and temperature. (Answer: b)
5. Which receptor helps in detecting skin stretch and joint position?
a) Meissner corpuscle
b) Pacinian corpuscle
c) Ruffini end organ
d) Merkel disc
Explanation:
Ruffini endings are mechanoreceptors detecting sustained skin stretch and joint capsule deformation, contributing to proprioception. Meissner and Pacinian corpuscles detect touch and vibration. Merkel discs detect sustained light touch. (Answer: c)
6. Merkel discs are associated with:
a) Pain
b) Sustained light touch
c) Vibration
d) Temperature
Explanation:
Merkel discs are slow-adapting mechanoreceptors found in basal epidermis, responsible for detecting sustained light touch and pressure. They do not detect pain, temperature, or vibration. (Answer: b)
7. A patient with joint position sense defect likely has damaged:
a) Meissner corpuscles
b) Ruffini end organs
c) Free nerve endings
d) Pacinian corpuscles
Explanation:
Ruffini end organs in joint capsules are essential for proprioception, detecting joint position and skin stretch. Damage leads to impaired position sense, whereas Meissner and Pacinian corpuscles do not contribute significantly to proprioception. (Answer: b)
8. Which of the following is NOT a function of Ruffini end organ?
a) Detect sustained pressure
b) Detect temperature
c) Detect skin stretch
d) Aid proprioception
Explanation:
Ruffini end organs detect sustained pressure, skin stretch, and contribute to proprioception. Temperature detection is the function of thermoreceptors, not Ruffini endings. (Answer: b)
9. Clinical correlation: In diabetic neuropathy, which receptor's dysfunction leads to impaired fine touch?
a) Ruffini end organ
b) Merkel disc
c) Free nerve endings
d) Pacinian corpuscle
Explanation:
Merkel discs mediate fine touch and pressure. Diabetic neuropathy often impairs these receptors, resulting in decreased touch sensitivity. Ruffini endings are less commonly affected in early stages. (Answer: b)
10. Receptors involved in proprioception include:
a) Free nerve endings
b) Meissner corpuscle
c) Muscle spindles and Ruffini end organs
d) Pacinian corpuscles
Explanation:
Proprioception depends on inputs from muscle spindles and Ruffini end organs in joint capsules. These receptors detect muscle stretch and skin/joint position, providing essential feedback for movement coordination. Free nerve endings detect pain, not proprioception. (Answer: c)
Topic: Sensory Physiology
Subtopic: Pain Perception Mechanism
Keywords:
• Pain receptors: Specialized nerve endings that detect noxious stimuli causing pain sensation.
• Meissner's corpuscle: Mechanoreceptors sensitive to light touch, located in dermal papillae.
• Pacinian corpuscle: Mechanoreceptors detecting deep pressure and vibration.
• Free nerve endings: Unencapsulated nerve endings detecting pain, temperature, and crude touch.
• Merkel disc: Slowly adapting mechanoreceptors detecting sustained touch and pressure.
Lead Question - 2013 (September 2008)
Pain receptors are?
a) Meissners corpuscle
b) Pacinian corpuscle
c) Free nerve endings
d) Merkel disc
Answer and Explanation:
The correct answer is c) Free nerve endings. These are unencapsulated nerve endings distributed widely in skin, mucosa, and organs. They are responsible for detecting pain (nociception) by responding to mechanical, chemical, or thermal noxious stimuli. Other options detect touch or pressure, not pain directly.
1. Which receptor is primarily responsible for light touch sensation?
a) Pacinian corpuscle
b) Meissner's corpuscle
c) Free nerve endings
d) Merkel disc
Explanation:
Meissner's corpuscles are rapidly adapting mechanoreceptors located in dermal papillae of glabrous skin. They are highly sensitive to light touch and vibrations of low frequency. Free nerve endings do not primarily detect light touch but rather pain and temperature changes. (Answer: b)
2. Clinical relevance: Patient presents with loss of pain sensation. Which receptor type is most likely damaged?
a) Pacinian corpuscle
b) Merkel disc
c) Free nerve endings
d) Meissner's corpuscle
Explanation:
Free nerve endings are responsible for detecting pain and temperature changes. Damage to these fibers can result in analgesia, making patients unable to perceive harmful stimuli, thereby increasing injury risk. Pacinian corpuscles and Merkel discs are not directly involved in pain perception. (Answer: c)
3. Which receptor is NOT involved in mechanoreception?
a) Meissner's corpuscle
b) Pacinian corpuscle
c) Free nerve endings
d) Merkel disc
Explanation:
Free nerve endings are primarily responsible for detecting pain and temperature, not fine touch or pressure. Meissner's, Pacinian, and Merkel receptors are mechanoreceptors responsible for various touch and pressure sensations. Hence, free nerve endings are not involved in mechanoreception. (Answer: c)
4. In clinical practice, which receptor dysfunction causes numbness but preserves pain sensation?
a) Meissner's corpuscle
b) Free nerve endings
c) Pacinian corpuscle
d) Merkel disc
Explanation:
Meissner's corpuscles and Pacinian corpuscles dysfunction result in loss of fine touch and vibration perception, but pain and temperature sensation remain intact due to free nerve endings. Pure loss of fine touch without pain indicates mechanoreceptor-specific dysfunction. (Answer: a)
5. Which receptor type helps in proprioception, not pain sensation?
a) Meissner's corpuscle
b) Pacinian corpuscle
c) Free nerve endings
d) Muscle spindle
Explanation:
Muscle spindles are specialized mechanoreceptors that provide information on muscle stretch and proprioception. Free nerve endings detect pain, whereas Meissner’s and Pacinian corpuscles detect light touch and vibration, not position sense. (Answer: d)
6. Free nerve endings detect:
a) Light touch
b) Temperature and pain
c) Pressure
d) Vibration
Explanation:
Free nerve endings are unencapsulated sensory receptors widely distributed and specialized for detecting temperature variations and nociceptive (painful) stimuli. They are key in transmitting painful sensations from peripheral tissues to the CNS. (Answer: b)
7. Which receptor is encapsulated and sensitive to deep pressure?
a) Free nerve endings
b) Merkel disc
c) Pacinian corpuscle
d) Meissner's corpuscle
Explanation:
Pacinian corpuscles are large, encapsulated mechanoreceptors highly sensitive to deep pressure and high-frequency vibration. They do not respond to pain stimuli, which is the function of free nerve endings. (Answer: c)
8. Pathological loss of pain sensation indicates damage to?
a) Merkel disc
b) Pacinian corpuscle
c) Free nerve endings
d) Meissner's corpuscle
Explanation:
Pathological loss of pain sensation, termed analgesia, typically occurs when free nerve endings are damaged. These receptors detect harmful mechanical, chemical, or thermal stimuli. Damage to other receptor types affects touch and pressure, not pain. (Answer: c)
9. Merkel discs are primarily responsible for detecting:
a) Pain
b) Fine touch
c) Vibration
d) Temperature
Explanation:
Merkel discs are slowly adapting mechanoreceptors located in the basal epidermis, responsible for detecting sustained light touch and pressure. They do not participate in pain sensation, which is mediated by free nerve endings. (Answer: b)
10. Clinical case: Patient with small-fiber neuropathy shows impaired pain sensation. Which fibers are affected?
a) Group Aβ fibers
b) Group Aα fibers
c) Free nerve endings
d) Meissner's corpuscle
Explanation:
Small-fiber neuropathy typically affects unmyelinated or thinly myelinated fibers, including free nerve endings responsible for pain and temperature. Group Aβ and Aα fibers mediate touch and proprioception, not pain. (Answer: c)
Topic: Autonomic Nervous System
Subtopic: Nerve Fiber Classification
Keywords:
• Group B nerve fibers: Myelinated fibers with medium diameter, conducting impulses at 3-15 m/s.
• Sympathetic preganglionic fibers: Nerve fibers that originate from the spinal cord and synapse in ganglia.
• Parasympathetic preganglionic fibers: Nerve fibers from brainstem or sacral spinal cord synapsing in ganglia near target organs.
• Postganglionic fibers: Nerve fibers that emerge from autonomic ganglia to innervate target tissues.
Lead Question - 2013 (September 2008)
Group B nerve fibers are?
a) Sympathetic preganglionic
b) Sympathetic postganglionic
c) Parasympathetic preganglionic
d) Parasympathetic postganglionic
Answer and Explanation:
The correct answer is a) Sympathetic preganglionic. Group B nerve fibers are myelinated with a moderate diameter and conduct impulses at speeds between 3 to 15 meters per second. They are predominantly found in the autonomic nervous system as preganglionic fibers, which transmit signals from the spinal cord to ganglia before reaching the target organ.
1. In NEET PG exams, which nerve fiber type is classified as Group A?
a) Preganglionic sympathetic
b) Postganglionic sympathetic
c) Motor fibers to skeletal muscle
d) Sensory fibers from skin
Explanation:
Group A fibers are large-diameter, myelinated nerve fibers conducting impulses rapidly at over 15 m/s. These fibers include somatic motor fibers that innervate skeletal muscles and sensory fibers transmitting touch and proprioception. They differ from Group B fibers, which are preganglionic autonomic fibers transmitting signals to autonomic ganglia. (Answer: c)
2. Clinical significance of damaged Group B fibers?
a) Loss of skeletal muscle movement
b) Impaired autonomic preganglionic transmission
c) Loss of pain sensation
d) None
Explanation:
Damage to Group B fibers impairs autonomic nervous system preganglionic transmission, leading to dysregulation of autonomic functions like heart rate, digestion, and temperature control. Skeletal muscle movement remains unaffected because Group A fibers mediate voluntary muscle control. Pain sensation is conveyed via different sensory fibers. (Answer: b)
3. Which statement is false regarding Group B fibers?
a) Myelinated
b) High conduction velocity
c) Found in autonomic pathways
d) Preganglionic
Explanation:
Group B fibers are myelinated and preganglionic but have moderate, not high, conduction velocity compared to Group A fibers. They conduct impulses at 3–15 m/s, appropriate for autonomic functions but slower than somatic motor fibers. This moderate speed suits their role in autonomic signal transmission. (Answer: b)
4. Parasympathetic preganglionic fibers are classified as?
a) Group A
b) Group B
c) Group C
d) None
Explanation:
Parasympathetic preganglionic fibers belong to Group B fibers. These are myelinated with moderate diameter and conduct impulses at 3–15 m/s. They project from the CNS to parasympathetic ganglia near target organs. Postganglionic fibers in the parasympathetic system are Group C fibers, unmyelinated and slower. (Answer: b)
5. Sympathetic postganglionic fibers are?
a) Group A
b) Group B
c) Group C
d) Group D
Explanation:
Sympathetic postganglionic fibers are classified as Group C fibers. These are unmyelinated and conduct impulses slowly (0.5–2 m/s). They connect the autonomic ganglia to target organs, regulating functions such as vasoconstriction and sweat gland activation, making them distinct from myelinated preganglionic Group B fibers. (Answer: c)
6. Clinical scenario: A patient with autonomic dysfunction shows reduced preganglionic activity. Which fibers are affected?
a) Group A
b) Group B
c) Group C
d) None
Explanation:
Autonomic dysfunction due to reduced preganglionic activity implicates Group B fibers. These fibers transmit signals from the CNS to autonomic ganglia. Dysfunction here causes impaired control of heart rate, digestion, and other involuntary processes. Postganglionic fibers (Group C) are not primarily affected. (Answer: b)
7. Which fibers are responsible for fast pain transmission?
a) Group Aδ
b) Group B
c) Group C
d) Group Aβ
Explanation:
Group Aδ fibers transmit fast pain signals. They are myelinated and conduct impulses at around 12–30 m/s, providing rapid sharp pain perception. Group B fibers do not transmit pain signals; they are preganglionic autonomic fibers. Group C fibers transmit slow, dull pain. (Answer: a)
8. In a case of diabetic autonomic neuropathy, which fibers are commonly damaged?
a) Group A
b) Group B
c) Group C
d) None
Explanation:
Diabetic autonomic neuropathy primarily affects Group C fibers—unmyelinated postganglionic fibers responsible for autonomic regulation. Group B fibers are generally less vulnerable but may also be involved. Damage leads to symptoms like orthostatic hypotension, gastroparesis, and abnormal sweating. (Answer: c)
9. Which fibers connect CNS to autonomic ganglia?
a) Group A
b) Group B
c) Group C
d) Group D
Explanation:
Group B fibers connect the central nervous system to autonomic ganglia. These are myelinated fibers with moderate diameter and conduction speed (3–15 m/s). They serve as preganglionic fibers in both sympathetic and parasympathetic divisions. Postganglionic fibers are mainly Group C. (Answer: b)
10. Clinical case: Patient has impaired heart rate regulation due to preganglionic fiber damage. Which group is involved?
a) Group A
b) Group B
c) Group C
d) None
Explanation:
Heart rate regulation impairment due to preganglionic fiber damage involves Group B fibers. These myelinated fibers transmit autonomic signals from the CNS to autonomic ganglia, influencing cardiac function. Postganglionic (Group C) fibers affect target organ response but do not transmit CNS signals to ganglia. (Answer: b)
Topic: Autonomic Nervous System
Subtopic: Parasympathetic Nervous System
Keyword Definitions:
- Post-ganglionic fibers: Nerve fibers extending from autonomic ganglia to target organs.
- Parasympathetic Nervous System: Part of the autonomic system responsible for "rest and digest" responses.
- A fibers: Myelinated fibers classified by diameter and conduction velocity.
- C fibers: Small diameter, unmyelinated fibers with slow conduction velocity.
Lead Question - 2013
Post-ganglionic parasympathetic fibers are - (September 2008)
a) A a
b) A (3
c) A 7
d) C
Answer & Explanation:
Correct Answer: d) C
Post-ganglionic parasympathetic fibers are predominantly small-diameter unmyelinated C fibers. These fibers conduct impulses slowly and are responsible for transmitting signals from autonomic ganglia to effector organs, regulating functions such as secretion, smooth muscle contraction, and vasodilation in the "rest and digest" state.
1. Which fibers have the fastest conduction velocity?
a) A fibers
b) B fibers
c) C fibers
d) D fibers
Answer & Explanation:
Correct Answer: a) A fibers
A fibers are myelinated and have large diameters, giving them the fastest conduction velocity among nerve fibers. This is crucial for rapid reflex actions and motor control, whereas C fibers are slow due to their unmyelinated nature.
2. The main neurotransmitter released by post-ganglionic parasympathetic fibers is:
a) Norepinephrine
b) Acetylcholine
c) Dopamine
d) Serotonin
Answer & Explanation:
Correct Answer: b) Acetylcholine
Post-ganglionic parasympathetic fibers release acetylcholine at their synapses with effector organs. Acetylcholine acts on muscarinic receptors to produce parasympathetic responses such as reduced heart rate and increased glandular secretion.
3. Clinical example of parasympathetic dysfunction:
a) Hypertension
b) Dry mouth
c) Tachycardia
d) Hyperhidrosis
Answer & Explanation:
Correct Answer: b) Dry mouth
Parasympathetic dysfunction can result in reduced glandular secretion, leading to symptoms such as dry mouth (xerostomia). It may occur due to nerve injury, diseases, or drug side effects affecting cholinergic pathways.
4. A fibers are primarily:
a) Unmyelinated
b) Large myelinated
c) Medium unmyelinated
d) Small myelinated
Answer & Explanation:
Correct Answer: b) Large myelinated
A fibers are large-diameter, myelinated fibers that conduct impulses rapidly. They are responsible for carrying motor commands and touch sensations, unlike C fibers which are slow and unmyelinated.
5. B fibers are mainly associated with:
a) Sensory pathways
b) Motor pathways
c) Preganglionic autonomic fibers
d) Postganglionic autonomic fibers
Answer & Explanation:
Correct Answer: c) Preganglionic autonomic fibers
B fibers are myelinated, preganglionic autonomic fibers with intermediate diameter and conduction speed, transmitting signals from the central nervous system to autonomic ganglia.
6. Parasympathetic nervous system is also known as:
a) Thoracolumbar system
b) Craniosacral system
c) Somatic system
d) Sympathetic system
Answer & Explanation:
Correct Answer: b) Craniosacral system
The parasympathetic system is called the craniosacral system because its preganglionic neurons originate in the brainstem (cranial nerves) and sacral spinal cord, responsible for conserving energy and promoting digestion.
7. C fibers are primarily responsible for transmitting:
a) Touch sensation
b) Pain and temperature
c) Proprioception
d) Visual input
Answer & Explanation:
Correct Answer: b) Pain and temperature
C fibers, unmyelinated and slow-conducting, are involved in transmitting pain (nociceptive) and temperature sensations from peripheral tissues to the central nervous system.
8. Damage to post-ganglionic parasympathetic fibers may cause:
a) Increased heart rate
b) Hypotension
c) Bradycardia
d) Hypersecretion
Answer & Explanation:
Correct Answer: a) Increased heart rate
Damage to post-ganglionic parasympathetic fibers impairs acetylcholine release, reducing vagal influence on the heart and resulting in unopposed sympathetic activity, thereby increasing heart rate (tachycardia).
9. The primary function of post-ganglionic parasympathetic fibers is to:
a) Prepare body for fight or flight
b) Maintain homeostasis during rest
c) Detect muscle stretch
d) Stimulate pain pathways
Answer & Explanation:
Correct Answer: b) Maintain homeostasis during rest
Post-ganglionic parasympathetic fibers mediate "rest and digest" activities, promoting digestion, energy storage, and reduced heart rate. Their slow conduction is adequate for these sustained, non-urgent functions.
10. Clinical test to assess parasympathetic function:
a) Pupillary light reflex
b) Deep tendon reflex
c) Pinprick sensation
d) Babinski sign
Answer & Explanation:
Correct Answer: a) Pupillary light reflex
Pupillary light reflex tests parasympathetic function via the oculomotor nerve (cranial nerve III). Light-induced pupil constriction reflects intact parasympathetic pathways, while absence indicates dysfunction in post-ganglionic parasympathetic fibers.
Topic: Sensory Receptors
Subtopic: Thermoreceptors
Keyword Definitions:
- Thermoreceptors: Sensory receptors that respond to temperature changes.
- CMR-1: Cold and menthol receptor-1, activated by moderate cold temperatures.
- VR1: Vanilloid receptor type 1, activated by high heat and capsaicin.
- VRL-1: Vanilloid receptor-like 1, involved in noxious heat sensing.
- VR2: Vanilloid receptor type 2, primarily involved in deep tissue temperature sensing.
Lead Question - 2013
Which receptor get stimulated in moderate cold? (September 2008)
a) CMR-1
b) VR1
c) VRL-1
d) VR2
Answer & Explanation:
Correct Answer: a) CMR-1
CMR-1 (Cold and Menthol Receptor-1) is primarily activated by moderate cold temperatures. It plays a critical role in sensing environmental temperature changes and contributes to the body's thermoregulatory mechanisms. This receptor differs from VR1 and VRL-1, which are activated by heat or chemical stimuli.
1. The primary function of thermoreceptors is:
a) Pain perception
b) Temperature detection
c) Pressure detection
d) Muscle stretch detection
Answer & Explanation:
Correct Answer: b) Temperature detection
Thermoreceptors are specialized sensory receptors that detect changes in temperature, allowing the body to maintain homeostasis. They are classified into cold and warm receptors, contributing to thermoregulation and behavioral responses to temperature changes.
2. VR1 receptor is activated by:
a) Menthol
b) Moderate cold
c) Capsaicin and high heat
d) Low pressure
Answer & Explanation:
Correct Answer: c) Capsaicin and high heat
VR1 (Vanilloid Receptor 1) is activated by capsaicin, the active component in chili peppers, and high temperatures (>43°C). It plays a key role in pain sensation and detecting harmful heat, not in sensing moderate cold.
3. CMR-1 is also known as:
a) TRPV1
b) TRPM8
c) TRPV2
d) TRPA1
Answer & Explanation:
Correct Answer: b) TRPM8
CMR-1 is also known as TRPM8 (Transient Receptor Potential Melastatin 8), activated by menthol and moderate cold temperatures (~15-30°C). It helps detect environmental cold stimuli and is important for cold sensation.
4. Which receptor does NOT respond to temperature?
a) CMR-1
b) VR1
c) VRL-1
d) Meissner corpuscle
Answer & Explanation:
Correct Answer: d) Meissner corpuscle
Meissner corpuscles are mechanoreceptors responsible for detecting light touch and texture, not temperature. Thermoreceptors like CMR-1 and VR1 detect cold and heat stimuli, respectively, and contribute to thermosensation.
5. Moderate cold is sensed at approximately:
a) 5-15°C
b) 15-30°C
c) 30-40°C
d) >40°C
Answer & Explanation:
Correct Answer: b) 15-30°C
CMR-1 (TRPM8) is activated by moderate cold in the range of 15–30°C. This range triggers appropriate physiological and behavioral responses to maintain thermal homeostasis and comfort.
6. Activation of CMR-1 leads to:
a) Pain sensation
b) Cold sensation
c) Heat sensation
d) Vibration sensation
Answer & Explanation:
Correct Answer: b) Cold sensation
Activation of CMR-1 receptors induces a cold sensation in the brain. These receptors detect cooling stimuli, enabling the body to react to environmental temperature changes and maintain internal temperature balance.
7. VRL-1 receptor is primarily activated by:
a) Moderate cold
b) High heat
c) Menthol
d) Light touch
Answer & Explanation:
Correct Answer: b) High heat
VRL-1 is primarily activated by high temperatures and plays a role in detecting noxious heat stimuli above normal body temperature. It is not involved in cold sensation.
8. Which is a chemical activator of CMR-1?
a) Capsaicin
b) Menthol
c) Bradykinin
d) Histamine
Answer & Explanation:
Correct Answer: b) Menthol
Menthol activates CMR-1 (TRPM8), simulating the sensation of cold. This chemical is often used in topical analgesics for a cooling effect and pain relief.
9. VR2 receptor primarily senses:
a) Surface temperature
b) Deep tissue temperature
c) Pain
d) Touch
Answer & Explanation:
Correct Answer: b) Deep tissue temperature
VR2 receptors are involved in sensing deep tissue temperature changes, differing from CMR-1 and VR1 that respond to surface temperature and noxious heat, respectively.
10. Clinical relevance of CMR-1:
a) Assessing pain threshold
b) Diagnosing heat stroke
c) Managing cold allodynia
d) Measuring vibration sense
Answer & Explanation:
Correct Answer: c) Managing cold allodynia
CMR-1 receptors are important in cold allodynia, where non-painful cold stimuli cause pain. Understanding its role helps in treating neuropathic conditions and developing targeted therapies to alleviate abnormal cold sensitivity.
Topic: Neuromuscular System
Subtopic: Cerebellar Function and Disorders
Keyword Definitions:
- Cerebellum: Part of the brain responsible for coordination, balance, and fine motor control.
- Dysmetria: Inability to judge distance or scale of movement.
- Hypertonia: Increased muscle tone leading to stiffness.
- Ataxia: Lack of voluntary coordination of muscle movements.
- Past-pointing: A cerebellar sign where a patient overshoots a target when attempting to touch it.
Lead Question - 2013
Cerebellar damage causes all except ? (September 2008)
a) Dysmetria
b) Hypertonia
c) Ataxia
d) Past-pointing
Answer & Explanation:
Correct Answer: b) Hypertonia
Cerebellar damage typically results in symptoms like dysmetria, ataxia, and past-pointing due to disrupted coordination and balance. However, hypertonia is not characteristic of cerebellar lesions but is more associated with upper motor neuron damage. Cerebellar dysfunction primarily causes hypotonia and impaired fine motor control.
1. Most common cause of cerebellar ataxia ?
a) Stroke
b) Peripheral neuropathy
c) Cerebellar degeneration
d) Muscle dystrophy
Answer & Explanation:
Correct Answer: c) Cerebellar degeneration
Cerebellar ataxia commonly results from cerebellar degeneration due to chronic alcoholism, genetic disorders, or paraneoplastic syndromes. It impairs coordination, balance, and fine motor control. Early recognition helps in managing symptoms and preventing complications by initiating targeted rehabilitation and therapy.
2. Dysmetria test involves:
a) Finger-to-nose test
b) Babinski reflex
c) Plantar reflex
d) Deep tendon reflex
Answer & Explanation:
Correct Answer: a) Finger-to-nose test
Dysmetria is assessed using the finger-to-nose test, where the patient’s inability to touch their nose accurately indicates cerebellar dysfunction. Overshooting or undershooting the target suggests impaired proprioception and coordination, hallmark features of cerebellar damage.
3. Ataxia affects:
a) Voluntary muscle strength
b) Coordination of movements
c) Sensory perception
d) Reflexes only
Answer & Explanation:
Correct Answer: b) Coordination of movements
Ataxia reflects impaired coordination of voluntary movements due to cerebellar dysfunction. Patients display unsteady gait, difficulty with fine motor tasks, and poor balance, but muscle strength remains generally intact. Recognizing ataxia is vital for early intervention.
4. Past-pointing indicates:
a) Involuntary muscle contraction
b) Overshooting a target
c) Loss of sensation
d) Enhanced reflexes
Answer & Explanation:
Correct Answer: b) Overshooting a target
Past-pointing is a cerebellar sign where patients overshoot the target when attempting to touch it. This demonstrates impaired proprioception and motor planning due to cerebellar dysfunction. It is assessed during coordination tests to evaluate cerebellar integrity.
5. Hypotonia in cerebellar lesions is due to:
a) Increased reflex activity
b) Loss of inhibitory control
c) Impaired motor planning
d) Decreased sensory input
Answer & Explanation:
Correct Answer: c) Impaired motor planning
Hypotonia in cerebellar lesions results from disrupted motor planning and coordination. Unlike hypertonia seen in upper motor neuron lesions, cerebellar damage leads to decreased muscle tone and poor posture control, contributing to unsteady movements and balance difficulties.
6. Common clinical feature of cerebellar lesion:
a) Muscle rigidity
b) Muscle atrophy
c) Intention tremor
d) Flaccid paralysis
Answer & Explanation:
Correct Answer: c) Intention tremor
An intention tremor is a hallmark feature of cerebellar lesions, where the tremor worsens during voluntary movement toward a target. This contrasts with resting tremors of Parkinson’s disease. It signifies defective cerebellar coordination of purposeful actions.
7. Cerebellar lesion affects which side of the body ?
a) Ipsilateral
b) Contralateral
c) Bilateral
d) Randomly
Answer & Explanation:
Correct Answer: a) Ipsilateral
Cerebellar lesions affect the ipsilateral side of the body because cerebellar pathways do not decussate. Thus, damage to the right cerebellar hemisphere results in motor deficits on the right side, an important diagnostic consideration.
8. Intention tremor is worsened during:
a) Rest
b) Voluntary movement
c) Sleep
d) Reflex action
Answer & Explanation:
Correct Answer: b) Voluntary movement
Intention tremor worsens during voluntary movement toward a target, reflecting impaired cerebellar coordination. It differentiates cerebellar from extrapyramidal tremors and helps localize neurological damage, enabling targeted therapeutic interventions.
9. Cerebellar dysfunction assessment includes:
a) MRI scan only
b) Blood tests
c) Clinical coordination tests
d) Muscle biopsy
Answer & Explanation:
Correct Answer: c) Clinical coordination tests
Assessment of cerebellar dysfunction involves clinical coordination tests like finger-to-nose and heel-to-shin tests. These reveal ataxia, dysmetria, and intention tremor, essential for diagnosis. Imaging aids in further evaluation, but clinical examination is crucial for early detection.
10. Therapeutic approach for cerebellar damage includes:
a) Antibiotics
b) Physical therapy
c) Antivirals
d) Chemotherapy
Answer & Explanation:
Correct Answer: b) Physical therapy
Physical therapy is key in managing cerebellar damage, focusing on improving coordination, balance, and muscle tone. Structured rehabilitation helps patients adapt, promoting functional independence. Early therapy initiation enhances outcomes and mitigates long-term disability.
Topic: Neuromuscular Reflexes
Subtopic: Primitive Reflexes
Keyword Definitions:
- Asymmetric Tonic Neck Reflex (ATNR): A primitive reflex in infants where turning the head causes limb extension on the same side.
- Muscle tone: The continuous and passive partial contraction of muscles.
- Primitive reflexes: Reflex actions present at birth and integrated during normal development.
Lead Question - 2013
Significance of absence of loss of asymmetric tonic neck reflex in 9 months ? (September 2008)
a) Decreased muscle tone
b) Increased muscle tone
c) Normal phenomenon
d) None of the above
Answer & Explanation:
Correct Answer: b) Increased muscle tone
Persistence of the asymmetric tonic neck reflex (ATNR) beyond 6 months, especially at 9 months, is abnormal and suggests increased muscle tone. This may indicate neurological disorders like cerebral palsy, where primitive reflexes are not properly inhibited, affecting voluntary motor control. Early detection aids in managing developmental delays.
1. ATNR normally integrates by which age ?
a) 1 month
b) 3 months
c) 6 months
d) 12 months
Answer & Explanation:
Correct Answer: c) 6 months
ATNR normally integrates by 6 months as the infant’s brain matures. Persistence beyond this age is clinically significant, often pointing to neurological disorders. Proper integration indicates healthy neurological development and progression from involuntary to voluntary motor control, essential in developmental assessment.
2. Clinical significance of persistent ATNR ?
a) Sign of neurological health
b) Normal in all infants
c) Indicator of neurological disorder
d) Sign of cardiovascular issue
Answer & Explanation:
Correct Answer: c) Indicator of neurological disorder
A persistent ATNR suggests an underlying neurological disorder, such as cerebral palsy. It reflects abnormal maturation of the central nervous system where primitive reflexes are not inhibited. This clinical sign helps early diagnosis and intervention planning to support motor and cognitive development.
3. ATNR assessment helps evaluate:
a) Respiratory function
b) Cardiovascular health
c) Neurological maturation
d) Digestive function
Answer & Explanation:
Correct Answer: c) Neurological maturation
ATNR assessment is a key indicator of neurological maturation in infants. Proper integration by 6 months signifies healthy central nervous system development, while persistence raises suspicion of disorders. Early assessment enables prompt diagnosis and intervention, supporting healthy developmental outcomes.
4. Persistence of ATNR is most commonly associated with:
a) Asthma
b) Cerebral palsy
c) Diabetes
d) Congenital heart disease
Answer & Explanation:
Correct Answer: b) Cerebral palsy
Persistent ATNR is commonly linked to cerebral palsy, reflecting improper neurological inhibition of primitive reflexes. It signals impaired central nervous system development, warranting early therapeutic intervention. Timely identification aids in managing motor dysfunction and improving patient outcomes.
5. ATNR helps in early:
a) Respiratory regulation
b) Muscle coordination
c) Digestive enzyme release
d) Hormonal balance
Answer & Explanation:
Correct Answer: b) Muscle coordination
ATNR plays a role in early muscle coordination development, aiding the infant in learning hand-eye coordination and spatial awareness. Its disappearance by 6 months marks a transition to voluntary control, crucial for further motor development and normal growth patterns.
6. Abnormal persistence of ATNR may result in:
a) Improved reflex control
b) Difficulty in voluntary movements
c) Enhanced muscle strength
d) Normal development
Answer & Explanation:
Correct Answer: b) Difficulty in voluntary movements
Persistent ATNR disrupts voluntary movement development, causing challenges in posture, coordination, and motor planning. This abnormality typically indicates neurological issues like cerebral palsy. Early detection facilitates interventions focused on improving voluntary motor control and overall functional development.
7. ATNR is also termed:
a) Symmetrical tonic neck reflex
b) Asymmetrical tonic neck reflex
c) Stretch reflex
d) Postural reflex
Answer & Explanation:
Correct Answer: b) Asymmetrical tonic neck reflex
The correct term for the tonic neck reflex is the Asymmetrical Tonic Neck Reflex (ATNR), characterized by turning the infant's head to one side, causing the ipsilateral limbs to extend and the contralateral limbs to flex. This primitive reflex is crucial for early coordination development.
8. ATNR integrates due to maturation of:
a) Spinal cord only
b) Brainstem only
c) Cerebral cortex
d) Peripheral nerves
Answer & Explanation:
Correct Answer: c) Cerebral cortex
Integration of ATNR occurs as the cerebral cortex matures, allowing voluntary motor control to override primitive reflexes. This reflects healthy neurological development, with cortical inhibition progressively replacing reflex-driven responses by around 6 months of age.
9. Clinical test for ATNR involves:
a) Stroking the sole
b) Turning the infant's head to one side
c) Pulling the infant upward
d) Stimulating the palm
Answer & Explanation:
Correct Answer: b) Turning the infant's head to one side
The clinical test for ATNR involves gently turning the infant's head to one side, eliciting extension of the ipsilateral arm and leg, and flexion of the opposite limbs. This helps assess neurological development and detect persistence indicating potential disorders.
10. Importance of assessing ATNR at 9 months is to:
a) Check muscle tone
b) Assess feeding habits
c) Evaluate neurological development
d) Check cardiovascular health
Answer & Explanation:
Correct Answer: c) Evaluate neurological development
Assessing ATNR at 9 months helps evaluate neurological development. Its persistence suggests developmental delay or neurological disorders like cerebral palsy, prompting further investigation. Timely assessment is critical for early diagnosis and therapeutic intervention, aiming to improve developmental outcomes.
Topic: Neuromuscular Reflexes
Subtopic: Primitive Reflexes
Keyword Definitions:
- Tonic neck reflex: A primitive reflex in infants causing head-turning to extend limbs on the same side.
- Disappearance Age: Age at which primitive reflexes naturally vanish during development.
- Primitive Reflexes: Reflex actions originating in the central nervous system, present at birth.
Lead Question - 2013
Tonic neck reflex disappears at what age ? (September 2008)
a) 1 month
b) 2 months
c) 3 months
d) 6 months
Answer & Explanation:
Correct Answer: c) 3 months
Tonic neck reflex typically disappears around 3 months of age as the infant's nervous system matures. Persistence beyond this age may suggest neurological abnormalities. This reflex aids early development of hand-eye coordination, but its disappearance is crucial for voluntary movement control and indicates normal neurological progression in infants.
1. What type of reflex is the tonic neck reflex ?
a) Spinal reflex
b) Primitive reflex
c) Postural reflex
d) Vestibular reflex
Answer & Explanation:
Correct Answer: b) Primitive reflex
The tonic neck reflex is classified as a primitive reflex. These reflexes are present at birth and typically disappear as the brain matures. Their presence or persistence is used as a clinical indicator of neurological development and potential abnormalities, helping clinicians assess infant development.
2. At what age do most primitive reflexes disappear ?
a) 1 month
b) 3-6 months
c) 1 year
d) 2 years
Answer & Explanation:
Correct Answer: b) 3-6 months
Primitive reflexes, including the tonic neck reflex, generally disappear by 3 to 6 months of age. This indicates proper central nervous system development. Persistent primitive reflexes beyond this window may suggest neurological disorders and require further evaluation to rule out underlying pathologies.
3. Clinical importance of disappearance of tonic neck reflex ?
a) Indicates respiratory health
b) Reflects neurological development
c) Shows gastrointestinal health
d) Indicates cardiac maturity
Answer & Explanation:
Correct Answer: b) Reflects neurological development
The disappearance of the tonic neck reflex reflects proper neurological development and maturation of the central nervous system. It signifies progression from involuntary reflexes to voluntary motor control. Persistence may indicate cerebral palsy or other neurological disorders, making it a critical assessment in pediatric exams.
4. Tonic neck reflex involves movement of:
a) Head only
b) Limbs only
c) Head and ipsilateral limbs
d) Contralateral limbs only
Answer & Explanation:
Correct Answer: c) Head and ipsilateral limbs
When the infant’s head turns to one side, the arm and leg on the same side extend while the opposite limbs flex. This helps early development of hand-eye coordination but should disappear as voluntary motor control develops to prevent abnormal persistence.
5. Persistence of tonic neck reflex beyond normal age suggests?
a) Normal variation
b) Neurological disorder
c) Cardiac anomaly
d) Respiratory infection
Answer & Explanation:
Correct Answer: b) Neurological disorder
Persistence of the tonic neck reflex beyond 3-6 months suggests a possible neurological disorder such as cerebral palsy. It indicates a delay in central nervous system maturation and warrants further clinical investigation to assess the infant’s neurological status and rule out pathological causes.
6. Which structure integrates the tonic neck reflex?
a) Cerebral cortex
b) Brainstem
c) Spinal cord
d) Cerebellum
Answer & Explanation:
Correct Answer: b) Brainstem
The brainstem serves as the integration center for the tonic neck reflex. It manages primitive reflexes by coordinating sensory input and motor responses. Proper brainstem function is crucial for reflex development and overall neurological health, making its assessment vital in pediatric examinations.
7. Tonic neck reflex is also known as:
a) Moro reflex
b) Asymmetrical tonic neck reflex (ATNR)
c) Palmar grasp reflex
d) Startle reflex
Answer & Explanation:
Correct Answer: b) Asymmetrical tonic neck reflex (ATNR)
The tonic neck reflex is specifically termed the Asymmetrical Tonic Neck Reflex (ATNR). It involves turning the infant's head to one side, causing the ipsilateral limbs to extend. It's a primitive reflex essential for early coordination but should fade with neurological maturation.
8. Clinical test for tonic neck reflex involves:
a) Stroking the sole
b) Turning the head to one side
c) Touching the palm
d) Pulling the infant upward
Answer & Explanation:
Correct Answer: b) Turning the head to one side
The clinical test for the tonic neck reflex involves gently turning the infant’s head to one side. This elicits extension of the arm and leg on the same side (ipsilateral) and flexion of the opposite limbs, assessing the reflex's presence and neurological development status.
9. Which condition is commonly associated with persistent ATNR?
a) Cerebral palsy
b) Asthma
c) Diabetes
d) Hypertension
Answer & Explanation:
Correct Answer: a) Cerebral palsy
Persistent Asymmetrical Tonic Neck Reflex (ATNR) beyond the expected developmental period is often associated with cerebral palsy. It suggests neurological impairment where primitive reflexes do not integrate properly, highlighting the need for early intervention and developmental support.
10. When does normal integration of ATNR typically occur ?
a) By 6 months
b) By 1 year
c) By 2 years
d) At birth
Answer & Explanation:
Correct Answer: a) By 6 months
Normal integration of the Asymmetrical Tonic Neck Reflex (ATNR) typically occurs by around 6 months of age. This reflects the maturation of the infant’s nervous system. If persistence is observed beyond this period, it may indicate developmental delays or neurological disorders requiring clinical attention. Topic: Sensory Pathways
Subtopic: Proprioception and Vibration Sense
Keywords:
Joint position sense: Ability to perceive the position of a joint in space.
Vibration sense: Ability to perceive oscillatory stimuli applied to the skin.
Proprioception: The body's ability to perceive its own position in space.
Aβ fibers: Large, myelinated nerve fibers that transmit proprioceptive and vibration information.
Lead Question - 2013 (September 2008)
Joint position & vibration sense is carried by?
a) Act
b) A(3)
c) Aβ
d) B
Answer: c) Aβ
Explanation: Joint position and vibration sense are carried by Aβ fibers, which are large, myelinated fibers. These fibers are responsible for transmitting proprioceptive and fine touch information rapidly to the central nervous system, particularly important for balance and coordinated movements.
1. Which tract primarily carries joint position and vibration sense to the brain?
a) Corticospinal tract
b) Spinothalamic tract
c) Dorsal column-medial lemniscus pathway
d) Spinocerebellar tract
Answer: c) Dorsal column-medial lemniscus pathway
Explanation: The dorsal column-medial lemniscus pathway transmits fine touch, vibration, and proprioceptive information from the periphery to the cerebral cortex. Aβ fibers synapse in the dorsal column nuclei, ascend, and cross in the medulla before reaching the thalamus and cortex.
2. Damage to Aβ fibers results in?
a) Loss of pain sensation
b) Loss of joint position and vibration sense
c) Loss of temperature sensation
d) Loss of motor function
Answer: b) Loss of joint position and vibration sense
Explanation: Aβ fiber damage disrupts the transmission of joint position and vibration sense, leading to impaired proprioception and balance. Clinically, patients may present with ataxia, especially when visual input is removed (positive Romberg sign).
3. The proprioceptive signals are integrated primarily in which brain region?
a) Cerebellum
b) Hippocampus
c) Prefrontal cortex
d) Thalamus
Answer: a) Cerebellum
Explanation: The cerebellum integrates proprioceptive signals for coordination of movement and balance. Inputs from muscle spindles and joint receptors travel via the dorsal column and spinocerebellar tracts to the cerebellum for fine-tuning motor activity.
4. Clinical test for vibration sense involves?
a) Pinprick test
b) Tuning fork application
c) Light touch with cotton
d) Reflex hammer percussion
Answer: b) Tuning fork application
Explanation: The tuning fork is applied over bony prominences to assess vibration sense. Reduced or absent vibration sense suggests a defect in large myelinated fibers (Aβ) or dorsal column pathology, as seen in conditions like tabes dorsalis.
5. Joint position sense is most impaired in:
a) Peripheral neuropathy
b) Alzheimer’s disease
c) Migraine
d) Bell's palsy
Answer: a) Peripheral neuropathy
Explanation: Peripheral neuropathy damages large myelinated fibers (Aβ), impairing proprioceptive signals and resulting in unsteady gait and balance problems. Patients may show positive Romberg sign, particularly when deprived of visual input.
6. The dorsal column is composed of:
a) Fasciculus gracilis and fasciculus cuneatus
b) Spinothalamic and spinocerebellar tracts
c) Corticospinal and rubrospinal tracts
d) Medial lemniscus only
Answer: a) Fasciculus gracilis and fasciculus cuneatus
Explanation: The dorsal column consists of fasciculus gracilis (lower body) and fasciculus cuneatus (upper body). They transmit fine touch, vibration, and proprioceptive information via Aβ fibers to the brain.
7. Romberg's test evaluates:
a) Muscle strength
b) Proprioceptive function
c) Pain sensation
d) Visual acuity
Answer: b) Proprioceptive function
Explanation: Romberg's test assesses proprioceptive function. A positive Romberg sign indicates impaired joint position sense or dorsal column dysfunction, causing instability when eyes are closed.
8. Vibration sense testing is best done over:
a) Soft tissue
b) Bony prominences
c) Skin surface
d) Muscle belly
Answer: b) Bony prominences
Explanation: Bony prominences are ideal for vibration sense testing because they transmit vibrations better. Tuning fork application on these areas assesses Aβ fiber function and dorsal column integrity.
9. Proprioceptive deficit presents as:
a) Muscle weakness
b) Ataxia
c) Hyperreflexia
d) Tremors
Answer: b) Ataxia
Explanation: Loss of proprioception leads to ataxia, characterized by uncoordinated movements, especially evident when visual input is removed. This occurs due to impaired joint position and vibration sense carried by Aβ fibers.
10. Which sensory fiber type carries touch, pressure, and proprioception?
a) Aα
b) Aβ
c) Aδ
d) C fibers
Answer: b) Aβ
Explanation: Aβ fibers are large, myelinated fibers responsible for transmitting touch, pressure, proprioceptive, and vibration sensations rapidly to the central nervous system, ensuring precise perception of body position and movement.
Chapter: Central Nervous System
Topic: Limbic System
Subtopic: Papez Circuit
Keywords:
Papez Circuit: Neural circuit involved in controlling emotional expression.
Nucleus: Collection of neurons in the central nervous system responsible for specific functions.
Thalamus: Part of the brain that relays motor and sensory signals to the cerebral cortex.
Anterior Nucleus of Thalamus: Thalamic nucleus involved in memory and part of the limbic system.
Lead Question - 2013
The nucleus involved in Papez circuit is:
a) Pulvinar
b) Intralaminar
c) VPL nucleus
d) Anterior nucleus of Thalamus
Answer: d) Anterior nucleus of Thalamus
Explanation: The anterior nucleus of the thalamus plays a key role in the Papez circuit, which is critical for regulating emotions and memory. It receives input from the mammillary bodies via the mammillothalamic tract and projects to the cingulate cortex, forming an essential part of this limbic pathway.
1. Which of the following is not part of the Papez circuit?
a) Hippocampus
b) Anterior nucleus of thalamus
c) Cerebellum
d) Cingulate gyrus
Answer: c) Cerebellum
Explanation: The cerebellum is not part of the Papez circuit. The circuit includes the hippocampus, anterior nucleus of the thalamus, mammillary bodies, cingulate gyrus, and entorhinal cortex, and plays a major role in processing emotions and memory functions.
2. Lesion in the anterior nucleus of the thalamus can cause:
a) Ataxia
b) Korsakoff’s syndrome
c) Hemiplegia
d) Aphasia
Answer: b) Korsakoff’s syndrome
Explanation: Damage to the anterior nucleus of the thalamus, as seen in Korsakoff’s syndrome, disrupts memory formation and leads to confabulation and memory loss. It is commonly associated with chronic alcoholism and thiamine deficiency affecting the limbic system.
3. The Papez circuit is primarily associated with:
a) Motor coordination
b) Emotional regulation
c) Language processing
d) Visual perception
Answer: b) Emotional regulation
Explanation: The Papez circuit, consisting of interconnected brain regions such as the hippocampus, anterior thalamic nuclei, and cingulate gyrus, plays a critical role in regulating emotions and memory processes, making it essential in the limbic system’s function.
4. The mammillothalamic tract connects:
a) Mammillary body to anterior nucleus of thalamus
b) Hippocampus to cingulate gyrus
c) Pulvinar to entorhinal cortex
d) Amygdala to hypothalamus
Answer: a) Mammillary body to anterior nucleus of thalamus
Explanation: The mammillothalamic tract connects the mammillary body to the anterior nucleus of the thalamus. It plays a crucial role in the Papez circuit by relaying information essential for emotional and memory processing within the limbic system.
5. Clinical dysfunction in the Papez circuit primarily results in:
a) Ataxia
b) Memory impairment
c) Aphasia
d) Seizures
Answer: b) Memory impairment
Explanation: Dysfunction in the Papez circuit, such as lesions affecting the anterior nucleus of the thalamus or hippocampus, leads to memory deficits. This occurs because the circuit is central to memory consolidation and emotional regulation, linking various structures in the limbic system.
6. The Papez circuit connects to the cerebral cortex via:
a) Internal capsule
b) Fornix
c) Corpus callosum
d) Mammillary body
Answer: b) Fornix
Explanation: The fornix is the major fiber tract connecting the hippocampus to the mammillary bodies, which further projects to the anterior nucleus of the thalamus. This anatomical pathway is a crucial component of the Papez circuit, essential for memory and emotion processing.
7. Damage to the anterior nucleus of the thalamus results in?
a) Wernicke's aphasia
b) Korsakoff’s psychosis
c) Huntington's chorea
d) Parkinson's disease
Answer: b) Korsakoff’s psychosis
Explanation: The anterior nucleus of the thalamus is a critical relay in the Papez circuit. Its damage leads to Korsakoff’s psychosis, characterized by severe memory loss, confabulation, and learning difficulties. This is commonly associated with thiamine deficiency in chronic alcoholics.
8. Anterior nucleus of thalamus is part of which brain system?
a) Reticular formation
b) Limbic system
c) Basal ganglia
d) Extrapyramidal system
Answer: b) Limbic system
Explanation: The anterior nucleus of the thalamus is part of the limbic system, which is involved in emotion regulation, memory formation, and integration of autonomic responses. It acts as a relay between the mammillary bodies and cingulate gyrus.
9. Which structure does not participate in the Papez circuit?
a) Hippocampus
b) Amygdala
c) Cingulate gyrus
d) Anterior nucleus of thalamus
Answer: b) Amygdala
Explanation: The amygdala, although involved in emotion processing, is not a component of the Papez circuit. The circuit primarily involves the hippocampus, anterior nucleus of the thalamus, mammillary bodies, and cingulate gyrus.
10. Damage to the Papez circuit most likely leads to:
a) Visual disturbances
b) Severe memory impairment
c) Motor weakness
d) Loss of smell
Answer: b) Severe memory impairment
Explanation: The Papez circuit is integral for memory consolidation and emotional processing. Damage, particularly to the anterior nucleus of the thalamus or hippocampus, impairs memory formation, leading to conditions like amnesia and confabulation, especially seen in Korsakoff’s syndrome.
Topic: Reflexes
Subtopic: Righting Reflex
Righting Reflex: Reflex that helps maintain normal posture.
Postural Reflex: Reflex that helps maintain body posture against gravity.
Stretch Reflex: Reflex in response to muscle stretching.
Spinal Reflex: Reflex controlled by spinal cord without brain involvement.
Ocular Reflex: Reflex related to eye movements.
Lead Question - 2013
Righting reflex is a ? (September 2008)
a) Stretch reflex
b) Postural reflex
c) Spinal reflex
d) Ocular reflex
Explanation: The righting reflex is a postural reflex that helps maintain body posture and balance by repositioning the head and body when they are displaced. It is primarily mediated through the vestibular system and helps in stabilizing the body against gravity. The correct answer is (b) Postural reflex.
Which of the following reflexes helps maintain posture against gravity?
a) Stretch reflex
b) Postural reflex
c) Ocular reflex
d) Gag reflex
Explanation: Postural reflex helps in maintaining body posture and balance by adjusting the body position relative to gravity. This reflex is vital for standing and walking. It primarily involves signals from the vestibular system and proprioceptors. The correct answer is (b) Postural reflex.
What type of reflex is mediated without brain involvement?
a) Ocular reflex
b) Postural reflex
c) Spinal reflex
d) Righting reflex
Explanation: A spinal reflex is mediated by the spinal cord without brain involvement, allowing for quick responses to stimuli, such as the knee-jerk reaction. This enables immediate reaction to maintain posture and protect the body. The correct answer is (c) Spinal reflex.
Damage to which system impairs righting reflex?
a) Cochlear system
b) Vestibular system
c) Visual system
d) Motor cortex
Explanation: The vestibular system provides essential sensory input for the righting reflex, helping the body maintain equilibrium. Damage to this system can impair the righting reflex, leading to balance problems and falls. The correct answer is (b) Vestibular system.
Which reflex corrects the position of the head in space?
a) Stretch reflex
b) Ocular reflex
c) Righting reflex
d) Flexor reflex
Explanation: The righting reflex helps correct the position of the head and body in space, ensuring proper posture and balance by detecting deviations from the upright position. It primarily involves vestibular input. The correct answer is (c) Righting reflex.
Righting reflex is essential for patients recovering from:
a) Stroke
b) Cochlear damage
c) Myocardial infarction
d) Asthma
Explanation: In stroke patients, the righting reflex may be impaired due to damage to the vestibular or central nervous systems, leading to poor balance and posture. Rehabilitation focuses on restoring this reflex to improve functional recovery. The correct answer is (a) Stroke.
Which nucleus integrates the righting reflex?
a) Suprachiasmatic nucleus
b) Vestibular nucleus
c) Preoptic nucleus
d) Paraventricular nucleus
Explanation: The vestibular nucleus integrates sensory inputs from the vestibular apparatus and coordinates the righting reflex. It helps maintain posture and equilibrium by processing signals related to head and body position. The correct answer is (b) Vestibular nucleus.
Clinical implication of impaired righting reflex includes:
a) Hypotension
b) Ataxia
c) Bradycardia
d) Anemia
Explanation: Ataxia refers to lack of coordination in movement due to impaired righting reflex. This can result from vestibular damage or neurological disorders, causing difficulty in maintaining balance and posture. The correct answer is (b) Ataxia.
Primary sensory input for righting reflex comes from:
a) Cochlea
b) Muscle spindles
c) Vestibular apparatus
d) Retina
Explanation: The primary sensory input for the righting reflex is from the vestibular apparatus, which detects changes in head position and motion, providing critical information to maintain balance and posture. The correct answer is (c) Vestibular apparatus.
Righting reflex is impaired in which condition?
a) Meniere's disease
b) Myocardial infarction
c) Diabetes mellitus
d) Hypertension
Explanation: Meniere's disease affects the inner ear and impairs the vestibular system, leading to impaired righting reflex, dizziness, and balance disturbances. The correct answer is (a) Meniere's disease.
Righting reflex is classified as:
a) Simple reflex
b) Spinal reflex
c) Postural reflex
d) Autonomic reflex
Explanation: The righting reflex is classified as a postural reflex because it helps maintain body posture and balance by detecting deviations from the upright position. The correct answer is (c) Postural reflex.
Topic: Reflex Arcs
Subtopic: Righting Reflex
Righting reflex: Reflex that helps maintain upright posture by correcting body position automatically in response to sensory inputs.
Cochlear reflex: Reflex related to auditory function, not involved in body posture control.
Spinal reflex: Simple reflexes mediated by the spinal cord, not involving higher centers like the midbrain for complex posture control.
Vestibular reflex: Reflex originating from the vestibular system in the inner ear, critical for maintaining balance and posture.
Lead Question - 2013 (September 2008)
Righting reflex is a ?
a) Cochlear reflex
b) Spinal reflex
c) Vestibular reflex
d) None of the above
Answer: c) Vestibular reflex
Explanation: The righting reflex is a vestibular reflex that helps maintain body posture by processing vestibular input from the inner ear in the midbrain. It automatically adjusts the body and head position in response to changes in orientation, preventing falls and aiding balance during movements.
1. Guessed Question
The primary sensory organ involved in righting reflex is:
a) Retina
b) Cochlea
c) Vestibular apparatus
d) Olfactory bulb
Answer: c) Vestibular apparatus
Explanation: The vestibular apparatus in the inner ear detects changes in head position and motion. It sends signals to the midbrain to adjust body posture via the righting reflex, maintaining balance and preventing falls automatically.
2. Guessed Question
Loss of righting reflex suggests lesion in:
a) Cerebellum
b) Vestibular pathways
c) Auditory nerve
d) Olfactory nerve
Answer: b) Vestibular pathways
Explanation: Lesions in the vestibular pathways, including the vestibular nerve or midbrain centers, impair the righting reflex. This prevents proper integration of sensory inputs needed for maintaining posture, resulting in imbalance and a tendency to fall.
3. Guessed Question
Which brain structure integrates the righting reflex?
a) Medulla
b) Midbrain
c) Pons
d) Spinal cord
Answer: b) Midbrain
Explanation: The midbrain integrates inputs from the vestibular apparatus and visual system to control the righting reflex. It rapidly adjusts postural muscle tone and body orientation without conscious effort, ensuring balance during movements or displacement.
4. Guessed Question
Righting reflex mainly helps in:
a) Hearing improvement
b) Postural correction
c) Voluntary limb movement
d) Respiratory rate regulation
Answer: b) Postural correction
Explanation: The righting reflex automatically adjusts head and body position in space by processing vestibular inputs. It helps prevent falls and maintain an upright posture, acting without conscious control, essential for normal daily activities and movement stability.
5. Guessed Question
Clinical test of righting reflex is useful to assess:
a) Liver function
b) Vestibular integrity
c) Visual acuity
d) Motor strength
Answer: b) Vestibular integrity
Explanation: Testing the righting reflex clinically helps assess the integrity of vestibular pathways. Impairment in the reflex suggests damage in the vestibular system or midbrain, indicating balance disorders or neurological deficits, aiding diagnosis and treatment planning.
6. Guessed Question
Righting reflex is important for:
a) Digestive enzyme release
b) Conscious decision-making
c) Maintaining upright posture
d) Hormonal regulation
Answer: c) Maintaining upright posture
Explanation: The righting reflex ensures automatic maintenance of upright posture. It quickly compensates for head and body displacement using midbrain integration of sensory inputs, enabling balance without voluntary effort or conscious awareness, essential for everyday stability.
7. Guessed Question
The righting reflex adjusts:
a) Limb length
b) Muscle tone
c) Hair growth
d) Joint structure
Answer: b) Muscle tone
Explanation: The righting reflex adjusts muscle tone in response to vestibular inputs, allowing rapid postural corrections. This ensures head and body alignment during changes in position, preventing falls and providing stability during voluntary and involuntary movements.
8. Guessed Question
Which reflex is NOT related to posture control?
a) Righting reflex
b) Vestibulospinal reflex
c) Patellar reflex
d) Labyrinthine reflex
Answer: c) Patellar reflex
Explanation: The patellar reflex is a simple spinal reflex that regulates knee extension in response to muscle stretch. It is not directly involved in posture control like the righting, vestibulospinal, or labyrinthine reflexes, which maintain body orientation and balance automatically.
9. Guessed Question
Righting reflex failure is commonly seen in:
a) Vestibular neuronitis
b) Diabetes mellitus
c) Hypertension
d) Migraine
Answer: a) Vestibular neuronitis
Explanation: Vestibular neuronitis causes inflammation of the vestibular nerve, impairing the righting reflex. This results in severe imbalance and vertigo, demonstrating the essential role of the vestibular system and midbrain in postural control and automatic body orientation adjustments.
Topic: Reflex Arcs
Subtopic: Righting Reflex
Righting reflex: Reflex that helps maintain upright posture by correcting body position.
Pons: Part of the brainstem involved in motor control and sensory analysis but not primary for righting reflex.
Spinal cord: Facilitates simple reflexes but higher centers are involved in complex postural reflexes.
Cortex: Controls voluntary movements but not responsible for reflex integration.
Midbrain: Higher center integrating vestibular inputs, critical for righting reflex and posture control.
Lead Question - 2013 (September 2008)
Higher center for righting reflex?
a) Pons
b) Spinal cord
c) Cortex
d) Midbrain
Answer: d) Midbrain
Explanation: The midbrain acts as the higher center for the righting reflex, integrating vestibular and visual inputs to help maintain posture and correct body position after displacement. This reflex helps in stabilizing the head and body in space, maintaining upright posture without conscious effort.
1. Guessed Question
Righting reflex primarily helps in:
a) Regulating heart rate
b) Maintaining upright posture
c) Digestive motility
d) Hormone secretion
Answer: b) Maintaining upright posture
Explanation: The righting reflex is crucial for maintaining upright posture. It corrects head and body orientation automatically through midbrain processing of sensory inputs. This ensures balance during movement and prevents falls, especially in response to sudden shifts in body position.
2. Guessed Question
The righting reflex is mediated by which sensory input?
a) Auditory
b) Vestibular
c) Olfactory
d) Gustatory
Answer: b) Vestibular
Explanation: Vestibular input from the inner ear provides critical information for the righting reflex. The midbrain processes this input to adjust body and head posture, enabling automatic correction of balance when the body is displaced, preventing instability or falls.
3. Guessed Question
Lesion of the midbrain would impair:
a) Pain sensation
b) Righting reflex
c) Voluntary speech
d) Visual acuity
Answer: b) Righting reflex
Explanation: A lesion in the midbrain disrupts the righting reflex by impairing integration of vestibular and visual inputs necessary for postural correction. This leads to inability to maintain upright posture, resulting in imbalance, falls, and coordination difficulties.
4. Guessed Question
Which of the following is NOT involved in righting reflex?
a) Midbrain
b) Vestibular apparatus
c) Auditory cortex
d) Visual system
Answer: c) Auditory cortex
Explanation: The auditory cortex is unrelated to the righting reflex. The midbrain integrates vestibular and visual inputs to correct posture and maintain balance. Auditory pathways do not contribute to this reflex, which is crucial for body orientation and stability.
5. Guessed Question
Righting reflex becomes prominent at what stage of development?
a) Adulthood
b) Neonatal period
c) Infancy
d) Old age
Answer: c) Infancy
Explanation: The righting reflex is most prominent in infancy when postural control is immature. It allows the infant to correct body position in response to head displacement automatically. As voluntary motor control develops, reliance on the reflex decreases, indicating normal neurological development.
6. Guessed Question
The righting reflex primarily prevents:
a) Seizures
b) Falls
c) Headaches
d) Muscle atrophy
Answer: b) Falls
Explanation: The righting reflex prevents falls by automatically adjusting body posture in response to changes in head or body position. Vestibular inputs to the midbrain trigger compensatory muscle tone adjustments, ensuring stability without conscious effort.
7. Guessed Question
Damage to which structure leads to loss of righting reflex?
a) Cerebellum
b) Medulla
c) Midbrain
d) Thalamus
Answer: c) Midbrain
Explanation: The midbrain is the primary integration center for the righting reflex. Damage to the midbrain disrupts its ability to process vestibular and visual signals required to adjust posture automatically, leading to impaired balance and loss of the righting reflex.
8. Guessed Question
Righting reflex adjusts muscle tone via:
a) Corticospinal tract
b) Vestibulospinal tract
c) Spinothalamic tract
d) Rubrospinal tract
Answer: b) Vestibulospinal tract
Explanation: The vestibulospinal tract conveys signals from the vestibular apparatus to spinal motor neurons, adjusting muscle tone automatically as part of the righting reflex. This enables rapid postural corrections without cortical involvement, ensuring balance during dynamic body movements.
9. Guessed Question
Which clinical condition may show loss of righting reflex?
a) Parkinson's disease
b) Vestibular neuronitis
c) Alzheimer's disease
d) Migraine
Answer: b) Vestibular neuronitis
Explanation: Vestibular neuronitis causes inflammation of the vestibular nerve, impairing sensory input to the midbrain. This leads to loss of the righting reflex, resulting in severe imbalance and dizziness. It exemplifies the crucial role of vestibular pathways in maintaining postural stability automatically.
Topic: Reflex Arcs
Subtopic: Tonic Labyrinthine Reflex
Tonic labyrinthine reflex: A primitive reflex involving body posture regulation through vestibular inputs.
Integration center: Neural structure where sensory inputs are processed to generate reflex motor output.
Spinal cord: Part of the CNS transmitting neural signals but not the main integrator for tonic labyrinthine reflex.
Medulla: Brainstem structure involved in autonomic and reflexive functions but not primary for this reflex.
Midbrain: Part of brainstem integrating vestibular signals, crucial for postural reflex control.
Cerebral cortex: Higher brain area responsible for voluntary motor control, not reflex integration.
Lead Question - 2013 (September 2008)
Integration center of tonic labyrinthine reflex is?
a) Spinal cord
b) Medulla
c) Midbrain
d) Cerebral cortex
Answer: c) Midbrain
Explanation: The tonic labyrinthine reflex integrates at the midbrain level, where vestibular inputs about head position influence postural muscle tone. It helps maintain balance by adjusting limb muscle tone in response to head movements. The spinal cord and cortex play roles in other reflexes and voluntary actions, respectively.
1. Guessed Question
Which structure integrates the tonic labyrinthine reflex?
a) Cerebellum
b) Midbrain
c) Spinal cord
d) Thalamus
Answer: b) Midbrain
Explanation: The midbrain serves as the primary integration center for the tonic labyrinthine reflex. It processes vestibular inputs from the inner ear, influencing posture and muscle tone. This reflex is vital for maintaining balance during head position changes, distinguishing it from higher cortical or spinal reflexes.
2. Guessed Question
The tonic labyrinthine reflex helps in maintaining?
a) Digestive function
b) Posture and balance
c) Cognitive function
d) Visual processing
Answer: b) Posture and balance
Explanation: The tonic labyrinthine reflex stabilizes posture by adjusting muscle tone in response to head movements, mediated by the midbrain. This reflex helps maintain equilibrium during activities like standing or walking and is especially prominent in infants before voluntary postural control matures.
3. Guessed Question
Which sensory input primarily activates the tonic labyrinthine reflex?
a) Visual input
b) Proprioceptive input
c) Vestibular input
d) Auditory input
Answer: c) Vestibular input
Explanation: Vestibular inputs from the inner ear are crucial for the tonic labyrinthine reflex. These inputs inform the midbrain about head position changes, prompting automatic adjustments in muscle tone and posture to maintain balance and equilibrium, essential for upright stance and coordinated movement.
4. Guessed Question
The tonic labyrinthine reflex is most prominent in:
a) Elderly adults
b) Healthy adults
c) Infants
d) Athletes
Answer: c) Infants
Explanation: In infants, the tonic labyrinthine reflex is dominant, aiding in early postural control before higher cortical mechanisms develop. It disappears with maturation as voluntary control takes over. Persistence of this reflex in adults may indicate neurological dysfunction.
5. Guessed Question
The primary role of the midbrain in reflexes is:
a) Memory processing
b) Visual perception
c) Integration of vestibular reflexes
d) Hormone secretion
Answer: c) Integration of vestibular reflexes
Explanation: The midbrain integrates vestibular reflexes, including the tonic labyrinthine reflex. It processes head movement and position data to maintain muscle tone and posture. This ensures equilibrium and coordinated movement during head position changes, without needing cortical input or conscious awareness.
6. Guessed Question
Lesion in the midbrain may result in disruption of:
a) Voluntary movement
b) Tonic labyrinthine reflex
c) Hormone secretion
d) Sensory perception
Answer: b) Tonic labyrinthine reflex
Explanation: Damage to the midbrain impairs integration of the tonic labyrinthine reflex, leading to postural instability and abnormal muscle tone. This results in poor balance and coordination, indicating the critical role of the midbrain in automatic postural adjustments driven by vestibular input.
7. Guessed Question
The tonic labyrinthine reflex adjusts muscle tone in response to?
a) Muscle fatigue
b) Head position changes
c) Visual stimuli
d) Pain sensation
Answer: b) Head position changes
Explanation: The tonic labyrinthine reflex modifies muscle tone in response to changes in head position. The midbrain processes vestibular input from the inner ear, regulating limb and axial muscle tone to maintain body balance and upright posture, especially during movements like tilting the head.
8. Guessed Question
Failure of the tonic labyrinthine reflex leads to:
a) Improved posture
b) Loss of equilibrium
c) Enhanced reflexes
d) Increased muscle strength
Answer: b) Loss of equilibrium
Explanation: Dysfunction in the tonic labyrinthine reflex disrupts posture regulation, leading to balance issues and potential falls. The reflex is critical for adjusting muscle tone in response to vestibular input. Its impairment suggests midbrain or vestibular system pathology, often seen in neurological disorders.
9. Guessed Question
Which pathway transmits signals for the tonic labyrinthine reflex?
a) Corticospinal tract
b) Vestibulospinal tract
c) Spinothalamic tract
d) Dorsal column
Answer: b) Vestibulospinal tract
Explanation: The vestibulospinal tract conveys signals from the vestibular apparatus to the spinal cord to modulate muscle tone and posture as part of the tonic labyrinthine reflex. This pathway bypasses higher cortical centers, enabling rapid reflexive responses to maintain equilibrium.
Topic: Reflex Arcs
Subtopic: Sensory Nerve Fibers
Stretch Impulse: Signals generated from muscle spindles in response to stretch.
Ia Fibers: Large-diameter, myelinated fibers transmitting stretch signals to the spinal cord.
Ib Fibers: Sensory fibers from Golgi tendon organs sensing tension in tendons.
B Fibers: Myelinated fibers transmitting autonomic preganglionic impulses.
C Fibers: Unmyelinated fibers carrying pain and temperature signals at slow conduction velocity.
Lead Question - 2013 (September 2008)
Stretch impulse is carried by?
a) Ia
b) Ib
c) B
d) C
Answer: a) Ia
Explanation: Stretch impulses generated by muscle spindle receptors are transmitted to the spinal cord via large-diameter myelinated Ia afferent fibers. These fibers are highly sensitive to muscle length changes and conduction is rapid, facilitating immediate reflex responses such as the stretch reflex to maintain muscle tone and posture.
1. Guessed Question
Which sensory fiber carries Golgi tendon organ signals?
a) Ia
b) Ib
c) B
d) C
Answer: b) Ib
Explanation: The Golgi tendon organ senses tension in the tendon during muscle contraction and sends impulses via Ib afferent fibers. These myelinated fibers conduct signals rapidly to the spinal cord, contributing to the autogenic inhibition reflex that prevents excessive force and protects muscles from injury.
2. Guessed Question
Which fiber type is unmyelinated and transmits pain?
a) Ia
b) Ib
c) B
d) C
Answer: d) C
Explanation: C fibers are unmyelinated sensory fibers responsible for transmitting dull, aching pain and temperature sensations. Their slow conduction velocity ensures prolonged pain sensation. They are crucial in chronic pain and inflammatory responses, contributing to body’s protective mechanisms and nociceptive signaling.
3. Guessed Question
B fibers are involved in carrying impulses to?
a) Skeletal muscle
b) Autonomic ganglia
c) Sensory cortex
d) Spinal cord
Answer: b) Autonomic ganglia
Explanation: B fibers are lightly myelinated fibers that transmit preganglionic autonomic signals from the central nervous system to autonomic ganglia. These fibers conduct impulses moderately fast, participating in autonomic regulation of visceral organs, such as heart rate, digestion, and glandular secretion.
4. Guessed Question
Muscle spindle stretch reflex helps maintain?
a) Blood pressure
b) Body temperature
c) Muscle tone and posture
d) Respiratory rate
Answer: c) Muscle tone and posture
Explanation: The stretch reflex, mediated by Ia afferent fibers from muscle spindles, plays a critical role in maintaining muscle tone and body posture. Stretch of the muscle activates spindle receptors, producing reflex contraction that stabilizes joints, preventing falls or injury during movement.
5. Guessed Question
Which receptor is primarily responsible for detecting muscle length?
a) Golgi tendon organ
b) Muscle spindle
c) Free nerve endings
d) Ruffini endings
Answer: b) Muscle spindle
Explanation: Muscle spindles are specialized stretch receptors located within skeletal muscles. They detect changes in muscle length and velocity of stretch, sending impulses through Ia afferent fibers to the spinal cord. This feedback maintains muscle tone and coordinates smooth voluntary movements.
6. Guessed Question
Ib afferent fibers regulate which reflex?
a) Stretch reflex
b) Flexor withdrawal reflex
c) Autogenic inhibition reflex
d) Crossed extensor reflex
Answer: c) Autogenic inhibition reflex
Explanation: The autogenic inhibition reflex prevents excessive muscle tension. Ib afferent fibers from Golgi tendon organs sense high tension and inhibit alpha motor neurons, reducing muscle contraction force. This protects muscles and tendons from damage during intense contraction or load lifting.
7. Guessed Question
Primary role of Ia afferent fibers in reflex arc?
a) Transmit pain
b) Transmit temperature
c) Conduct stretch information
d) Relay visual signals
Answer: c) Conduct stretch information
Explanation: Ia afferent fibers conduct stretch information from muscle spindles to the spinal cord. They are crucial in the monosynaptic stretch reflex, where muscle stretch leads to reflex contraction, maintaining posture and muscle tone. Their high conduction velocity enables rapid responses essential for movement control.
8. Guessed Question
Ib fibers originate from which organ?
a) Muscle spindle
b) Golgi tendon organ
c) Pacinian corpuscle
d) Meissner's corpuscle
Answer: b) Golgi tendon organ
Explanation: Ib fibers originate from Golgi tendon organs located at muscle-tendon junctions. They detect changes in muscle tension and relay this information to the spinal cord, playing a protective role by inhibiting excessive contraction and preventing tendon rupture during high-load activities.
9. Guessed Question
Which fibers have the fastest conduction velocity?
a) C fibers
b) B fibers
c) Ib fibers
d) Ia fibers
Answer: d) Ia fibers
Explanation: Ia fibers possess the fastest conduction velocity due to large diameter and heavy myelination. This facilitates rapid transmission of stretch impulses, essential for immediate reflex responses like the stretch reflex, contributing to quick adjustments in muscle activity and joint stability.
Topic: Neuroanatomy
Subtopic: Development of Brain Structures
Keywords:
Medulla Oblongata: The lower portion of the brainstem, responsible for autonomic functions such as breathing, heart rate, and reflex actions.
Prosencephalon: The embryonic forebrain, which develops into the cerebral hemispheres and diencephalon.
Rhombencephalon: The embryonic hindbrain, which gives rise to the pons, cerebellum, and medulla oblongata.
Mesencephalon: The embryonic midbrain, involved in motor movement and auditory/visual processing.
Lead Question - 2013:
Medulla oblongata arises from?
a) Prosencephalon
b) Rhombencephalon
c) Mesencephalon
d) None
Answer & Explanation:
Correct answer: b) Rhombencephalon.
Explanation: The medulla oblongata is derived from the embryonic rhombencephalon (hindbrain). It regulates vital autonomic functions such as breathing, heart rate, and reflexes like coughing and vomiting. Understanding its developmental origin helps in diagnosing congenital malformations and understanding brainstem pathologies.
MCQ 1:
Which part of the brainstem is directly continuous with the spinal cord?
a) Midbrain
b) Pons
c) Medulla oblongata
d) Thalamus
Answer & Explanation:
Correct answer: c) Medulla oblongata.
Explanation: The medulla oblongata forms the lowest part of the brainstem and directly continues with the spinal cord. It houses vital autonomic centers that regulate heart rate, respiration, and reflexes, making it essential for life-sustaining functions.
MCQ 2:
The medulla oblongata contains which important center?
a) Visual center
b) Respiratory center
c) Auditory center
d) Olfactory center
Answer & Explanation:
Correct answer: b) Respiratory center.
Explanation: The medulla oblongata contains the respiratory center, which controls the rate and depth of breathing. It responds to chemical and neural signals, ensuring homeostasis of blood gases, and is critical in cases of respiratory dysfunction or brainstem injury.
MCQ 3 (Clinical):
Medullary syndrome (Wallenberg syndrome) is caused by occlusion of?
a) Anterior spinal artery
b) Posterior inferior cerebellar artery (PICA)
c) Middle cerebral artery
d) Basilar artery
Answer & Explanation:
Correct answer: b) Posterior inferior cerebellar artery (PICA).
Explanation: Wallenberg syndrome occurs due to PICA occlusion, causing ipsilateral loss of pain and temperature in the face, contralateral body loss, vertigo, ataxia, and dysphagia. It highlights the medulla’s role in sensory pathways and autonomic functions.
MCQ 4:
Which cranial nerve nuclei are located in the medulla oblongata?
a) CN III, IV
b) CN V, VI
c) CN IX, X, XII
d) CN II, III
Answer & Explanation:
Correct answer: c) CN IX, X, XII.
Explanation: The medulla oblongata houses the nuclei of cranial nerves IX (Glossopharyngeal), X (Vagus), and XII (Hypoglossal), which control swallowing, cardiovascular function, and tongue movements. Damage leads to dysphagia, hoarseness, and tongue deviation.
MCQ 5 (Clinical):
A lesion in the medulla oblongata can cause?
a) Hemiplegia
b) Loss of proprioception
c) Respiratory failure
d) All of the above
Answer & Explanation:
Correct answer: d) All of the above.
Explanation: Medullary lesions can affect pyramidal tracts (causing hemiplegia), sensory pathways (causing proprioceptive loss), and the respiratory center, leading to life-threatening respiratory failure. Comprehensive assessment is vital in brainstem strokes or trauma.
MCQ 6:
The pyramidal decussation is located at?
a) Midbrain
b) Pons
c) Medulla oblongata
d) Cerebellum
Answer & Explanation:
Correct answer: c) Medulla oblongata.
Explanation: The pyramidal decussation occurs in the medulla oblongata, where most corticospinal fibers cross to the contralateral side. This explains why the left cerebral hemisphere controls the right body side, a fundamental concept in neuroanatomy and clinical neurology.
MCQ 7 (Clinical):
Which syndrome is caused by medullary infarction?
a) Horner's syndrome
b) Locked-in syndrome
c) Weber syndrome
d) Wallenberg syndrome
Answer & Explanation:
Correct answer: d) Wallenberg syndrome.
Explanation: Medullary infarction, specifically of the lateral medulla due to PICA occlusion, causes Wallenberg syndrome with symptoms like vertigo, dysphagia, and sensory deficits. Timely identification prevents long-term complications and guides treatment.
MCQ 8:
The area postrema located in the medulla oblongata is responsible for?
a) Respiratory control
b) Vomiting reflex
c) Temperature regulation
d) Sleep cycle
Answer & Explanation:
Correct answer: b) Vomiting reflex.
Explanation: The area postrema, located in the medulla oblongata, detects toxins in the blood and triggers the vomiting reflex. It lacks a blood-brain barrier, making it sensitive to emetogenic substances, and is targeted in antiemetic drug development.
MCQ 9:
Which artery primarily supplies the medulla oblongata?
a) Basilar artery
b) Vertebral artery
c) Anterior cerebral artery
d) Posterior cerebral artery
Answer & Explanation:
Correct answer: b) Vertebral artery.
Explanation: The vertebral arteries supply the medulla oblongata via branches such as the anterior and posterior spinal arteries and PICA. Vascular compromise leads to medullary infarcts, manifesting as life-threatening syndromes like Wallenberg syndrome or respiratory arrest.
MCQ 10 (Clinical):
Key function of medulla oblongata?
a) Visual processing
b) Autonomic control
c) Language comprehension
d) Hormone secretion
Answer & Explanation:
Correct answer: b) Autonomic control.
Explanation: The medulla oblongata regulates essential autonomic functions like heart rate, respiration, blood pressure, and reflex actions such as coughing and vomiting. Lesions can cause life-threatening autonomic dysfunction, highlighting its clinical importance.
Topic: Neuroanatomy
Subtopic: Thalamic Nuclei and Cerebral Cortex Connections
Keywords:
Thalamic Nuclei: Clusters of neurons in the thalamus relaying sensory and motor signals to the cerebral cortex.
Neocortex: The part of the cerebral cortex involved in higher-order brain functions such as sensory perception, cognition, and motor control.
Pulvinar Nucleus: Largest thalamic nucleus involved in visual attention and connects to association areas of neocortex.
Intralaminar Nuclei: Involved in arousal, attention, and pain perception, projecting diffusely to cortex.
Anterior Nucleus: Connected to limbic system and involved in memory processing, projecting to cingulate gyrus.
Lead Question - 2013:
Which thalamic nuclei connects with neocortex?
a) Pulvinar
b) Intralaminar
c) Anterior
d) All
Answer & Explanation:
Correct answer: d) All.
Explanation: All listed thalamic nuclei—Pulvinar, Intralaminar, and Anterior—connect with the neocortex. Pulvinar is important in visual attention, Intralaminar nuclei help in arousal and attention, and the Anterior nucleus participates in memory functions. Their integration ensures complex cortical processing and behavior regulation.
MCQ 1:
The main role of pulvinar nucleus is?
a) Motor coordination
b) Visual attention
c) Auditory processing
d) Hormonal regulation
Answer & Explanation:
Correct answer: b) Visual attention.
Explanation: The pulvinar nucleus plays a major role in visual attention by connecting with visual association areas in the neocortex. It helps in filtering and prioritizing visual information, which is vital for focused attention and perception of complex stimuli.
MCQ 2:
Which thalamic nuclei is primarily involved in arousal and alertness?
a) Anterior nucleus
b) Pulvinar nucleus
c) Intralaminar nuclei
d) Medial geniculate nucleus
Answer & Explanation:
Correct answer: c) Intralaminar nuclei.
Explanation: Intralaminar nuclei are diffusely connected to the neocortex and are crucial in arousal, attention, and awareness. They are often implicated in disorders of consciousness and are a focus of research in disorders like coma and vegetative states.
MCQ 3:
Anterior nucleus of the thalamus projects to?
a) Visual cortex
b) Prefrontal cortex
c) Cingulate gyrus
d) Auditory cortex
Answer & Explanation:
Correct answer: c) Cingulate gyrus.
Explanation: The anterior nucleus of the thalamus connects to the cingulate gyrus, playing a role in memory and emotional processing. Lesions can lead to memory disturbances and behavioral changes due to its link with the limbic system.
MCQ 4 (Clinical):
Lesion in pulvinar nucleus may cause:
a) Visual neglect
b) Motor weakness
c) Hearing loss
d) Memory loss
Answer & Explanation:
Correct answer: a) Visual neglect.
Explanation: Pulvinar nucleus lesions can lead to visual neglect, especially in the contralateral visual field, due to its role in visual attention and integration. It demonstrates the clinical importance of thalamic pathways in higher cortical functions.
MCQ 5:
Which nuclei is least involved in direct sensory relay?
a) Anterior nucleus
b) Lateral geniculate nucleus
c) Medial geniculate nucleus
d) Ventral posterior nucleus
Answer & Explanation:
Correct answer: a) Anterior nucleus.
Explanation: The anterior nucleus primarily connects with the limbic system for memory and emotion, not directly involved in primary sensory relay, unlike the lateral and medial geniculate nuclei and ventral posterior nucleus that relay visual, auditory, and somatosensory information respectively.
MCQ 6 (Clinical):
Thalamic stroke involving which nuclei causes significant arousal deficit?
a) Ventral posterior nucleus
b) Intralaminar nuclei
c) Lateral geniculate nucleus
d) Anterior nucleus
Answer & Explanation:
Correct answer: b) Intralaminar nuclei.
Explanation: Intralaminar nuclei are critical for arousal and consciousness. Thalamic strokes affecting these nuclei can cause coma or severe arousal deficits, emphasizing their importance in consciousness and attentional mechanisms in clinical neurology.
MCQ 7:
The medial geniculate nucleus of the thalamus connects to?
a) Auditory cortex
b) Visual cortex
c) Motor cortex
d) Somatosensory cortex
Answer & Explanation:
Correct answer: a) Auditory cortex.
Explanation: The medial geniculate nucleus relays auditory information from the inferior colliculus to the auditory cortex, enabling hearing perception and auditory processing. Damage to this pathway can cause hearing deficits or auditory agnosia.
MCQ 8 (Clinical):
Damage to anterior thalamic nucleus causes?
a) Memory deficits
b) Visual loss
c) Hearing impairment
d) Motor weakness
Answer & Explanation:
Correct answer: a) Memory deficits.
Explanation: The anterior nucleus is part of the Papez circuit and plays a crucial role in memory processing. Lesions lead to amnesia and disorientation, frequently observed in thalamic stroke syndromes affecting cognition and memory function.
MCQ 9:
Which thalamic nuclei relays somatosensory information?
a) Anterior nucleus
b) Medial geniculate nucleus
c) Ventral posterior nucleus
d) Pulvinar nucleus
Answer & Explanation:
Correct answer: c) Ventral posterior nucleus.
Explanation: The ventral posterior nucleus of the thalamus is the main relay station for somatosensory information from the body and face to the primary somatosensory cortex, crucial in sensory perception and clinical examination.
MCQ 10 (Clinical):
Thalamic pain syndrome (Dejerine-Roussy) is associated with damage to?
a) Anterior nucleus
b) Pulvinar nucleus
c) Ventral posterior nucleus
d) Medial geniculate nucleus
Answer & Explanation:
Correct answer: c) Ventral posterior nucleus.
Explanation: Dejerine-Roussy syndrome occurs due to damage to the ventral posterior nucleus, causing chronic contralateral pain, sensory disturbances, and allodynia. Recognition is important for diagnosis and management of post-stroke pain syndromes.
Topic: Ear Anatomy
Subtopic: Stapedius Nerve
Keywords:
Stapedius Nerve: A small branch of the facial nerve that innervates the stapedius muscle in the middle ear.
Trigeminal Nerve (CN V): Supplies motor function to muscles of mastication and sensory to face.
Facial Nerve (CN VII): Provides motor innervation to facial muscles and carries taste and parasympathetic fibers.
Vagus Nerve (CN X): Supplies parasympathetic fibers to thoracic and abdominal organs and sensory/motor fibers to larynx and pharynx.
Lead Question - 2013:
Stapedius nerve is a branch of?
a) Trigeminal nerve
b) Facial nerve
c) Vagus nerve
d) None
Answer & Explanation:
Correct answer: b) Facial nerve.
Explanation: The stapedius nerve is a branch of the facial nerve (CN VII). It innervates the stapedius muscle in the middle ear, which stabilizes the stapes bone to dampen loud sounds. Dysfunction can lead to hyperacusis, highlighting the nerve's clinical importance in auditory protection and sound modulation.
MCQ 1:
The stapedius muscle helps to:
a) Amplify sound
b) Stabilize the stapes
c) Transmit sound to the cochlea
d) Open the Eustachian tube
Answer & Explanation:
Correct answer: b) Stabilize the stapes.
Explanation: The stapedius muscle dampens excessive vibrations of the stapes bone to protect the inner ear from loud sounds. Innervated by the stapedius nerve (branch of facial nerve), its dysfunction causes hyperacusis. Understanding this helps in diagnosing auditory sensitivity disorders after middle ear pathology or surgery.
MCQ 2:
Damage to the facial nerve can cause which auditory symptom?
a) Sensorineural hearing loss
b) Conductive hearing loss
c) Hyperacusis
d) Tinnitus
Answer & Explanation:
Correct answer: c) Hyperacusis.
Explanation: Damage to the stapedius nerve branch of the facial nerve results in paralysis of the stapedius muscle. This reduces damping of sound vibrations, leading to hyperacusis—an increased sensitivity to normal sounds. Recognizing this is essential in facial nerve palsy evaluations to assess auditory system involvement.
MCQ 3:
The stapedius nerve emerges from which part of the facial nerve?
a) Intracranial segment
b) Geniculate ganglion
c) Tympanic segment
d) Mastoid segment
Answer & Explanation:
Correct answer: c) Tympanic segment.
Explanation: The stapedius nerve branches from the tympanic segment of the facial nerve. It passes through the middle ear to innervate the stapedius muscle. Knowledge of this anatomy is critical during middle ear surgery to avoid damaging the nerve, which would cause hyperacusis and affect sound modulation.
MCQ 4 (Clinical):
A patient with Bell's palsy reports sensitivity to loud sounds. Which nerve is likely involved?
a) Auriculotemporal nerve
b) Vestibulocochlear nerve
c) Stapedius nerve
d) Glossopharyngeal nerve
Answer & Explanation:
Correct answer: c) Stapedius nerve.
Explanation: In Bell's palsy, the facial nerve is inflamed or compressed, potentially affecting the stapedius nerve. This leads to hyperacusis due to lack of stapes stabilization. Identifying this clinical sign aids in diagnosis and indicates the extent of facial nerve involvement.
MCQ 5:
The primary function of the stapedius muscle is to:
a) Transmit sound to the cochlea
b) Amplify high-frequency sounds
c) Stabilize the stapes during loud noise
d) Regulate middle ear pressure
Answer & Explanation:
Correct answer: c) Stabilize the stapes during loud noise.
Explanation: The stapedius muscle contracts in response to loud sounds, stabilizing the stapes to dampen excessive vibration and protect the inner ear. This reflex, known as the acoustic reflex, is essential for auditory system protection and is mediated via the facial nerve pathway.
MCQ 6 (Clinical):
Which condition may result from stapedius muscle paralysis?
a) Otosclerosis
b) Hyperacusis
c) Conductive hearing loss
d) Vertigo
Answer & Explanation:
Correct answer: b) Hyperacusis.
Explanation: Paralysis of the stapedius muscle due to stapedius nerve damage removes its dampening effect on stapes movement. This leads to hyperacusis, where normal environmental sounds are perceived as excessively loud and uncomfortable. Understanding this helps differentiate auditory hypersensitivity disorders from other hearing impairments.
MCQ 7:
The stapedius muscle attaches to which auditory ossicle?
a) Malleus
b) Incus
c) Stapes
d) Tympanic membrane
Answer & Explanation:
Correct answer: c) Stapes.
Explanation: The stapedius muscle attaches to the neck of the stapes, the smallest bone in the human body. Its contraction stabilizes the stapes to modulate sound transmission. Injury to the stapedius nerve affects this function, emphasizing the importance of precise anatomical knowledge during middle ear surgeries.
MCQ 8 (Clinical):
During middle ear surgery, which nerve is at risk of damage affecting auditory sensitivity?
a) Facial nerve
b) Auditory nerve
c) Chorda tympani
d) Stapedius nerve
Answer & Explanation:
Correct answer: d) Stapedius nerve.
Explanation: The stapedius nerve traverses the middle ear cavity, making it vulnerable during procedures like stapedectomy. Damage results in stapedius muscle paralysis and hyperacusis. Surgeons must carefully navigate the middle ear anatomy to preserve this nerve and prevent postoperative auditory sensitivity disorders.
MCQ 9:
The stapedius reflex is primarily a protective mechanism against:
a) Low-frequency sounds
b) High-intensity sounds
c) Continuous ambient noise
d) Vestibular imbalance
Answer & Explanation:
Correct answer: b) High-intensity sounds.
Explanation: The stapedius reflex, mediated by the stapedius nerve, contracts the stapedius muscle in response to high-intensity sounds. This reduces the transmission of sound energy to the inner ear, protecting cochlear hair cells. Clinical tests of this reflex assess middle ear and facial nerve integrity.
MCQ 10 (Clinical):
A patient with congenital absence of the stapedius muscle experiences:
a) Complete deafness
b) Hyperacusis
c) Vertigo
d) Tinnitus
Answer & Explanation:
Correct answer: b) Hyperacusis.
Explanation: Congenital absence of the stapedius muscle prevents normal damping of stapes vibrations, leading to hyperacusis. Patients perceive everyday sounds as painfully loud. This emphasizes the stapedius muscle’s protective role and the importance of assessing auditory reflexes in patients with unexplained sound sensitivity.
Chapter: Anatomy
Topic: Nervous System
Subtopic: Nerve Anastomosis
Keywords:
Galen's Anastomosis: A nerve connection between the internal and external laryngeal nerves.
Recurrent Laryngeal Nerve: Branch of the vagus nerve supplying motor function to intrinsic laryngeal muscles.
Internal Laryngeal Nerve: Provides sensation to the laryngeal mucosa above the vocal cords.
External Laryngeal Nerve: Supplies the cricothyroid muscle responsible for pitch modulation.
Lead Question - 2013:
Galen's anastomosis is between ?
a) Recurrent laryngeal nerve and external laryngeal nerve
b) Recurrent laryngeal nerve and internal laryngeal nerve
c) Internal laryngeal nerve and external laryngeal nerve
d) None of the above
Answer & Explanation:
Correct answer: c) Internal laryngeal nerve and external laryngeal nerve.
Explanation: Galen's anastomosis refers to the communication between the internal and external laryngeal nerves. This connection is significant for overlapping sensory and motor innervation in the larynx, ensuring coordinated function. Understanding this anastomosis is important in surgeries to avoid vocal complications.
MCQ 1
Which nerve provides sensation above the vocal cords?
a) Recurrent laryngeal nerve
b) Internal laryngeal nerve
c) External laryngeal nerve
d) Hypoglossal nerve
Answer & Explanation:
Correct answer: b) Internal laryngeal nerve.
Explanation: The internal laryngeal nerve supplies sensory innervation to the laryngeal mucosa above the vocal cords. This nerve plays a crucial role in the cough reflex, preventing aspiration. Damage can lead to loss of sensation and predispose to aspiration pneumonia in clinical practice.
MCQ 2
The cricothyroid muscle is innervated by which nerve?
a) Recurrent laryngeal nerve
b) Internal laryngeal nerve
c) External laryngeal nerve
d) Accessory nerve
Answer & Explanation:
Correct answer: c) External laryngeal nerve.
Explanation: The external laryngeal nerve innervates the cricothyroid muscle, which adjusts vocal cord tension and modulates pitch. Clinically, damage may result in hoarseness. Understanding this is vital during thyroid surgeries to prevent vocal changes.
MCQ 3
Which statement about the recurrent laryngeal nerve is true?
a) It supplies the cricothyroid muscle.
b) It provides sensation above the vocal cords.
c) It supplies all intrinsic laryngeal muscles except the cricothyroid.
d) It is a branch of the external carotid artery.
Answer & Explanation:
Correct answer: c) It supplies all intrinsic laryngeal muscles except the cricothyroid.
Explanation: The recurrent laryngeal nerve provides motor supply to all intrinsic laryngeal muscles except the cricothyroid. This anatomical detail is crucial during neck surgeries, as injury can cause vocal cord paralysis and hoarseness.
MCQ 4 (Clinical):
A patient presents with hoarseness post-thyroidectomy. Which nerve is most likely injured?
a) Internal laryngeal nerve
b) External laryngeal nerve
c) Recurrent laryngeal nerve
d) Hypoglossal nerve
Answer & Explanation:
Correct answer: c) Recurrent laryngeal nerve.
Explanation: The recurrent laryngeal nerve is prone to injury during thyroidectomy due to its anatomical proximity. Injury leads to hoarseness or loss of voice, as it innervates the intrinsic laryngeal muscles except cricothyroid. Early recognition is essential to manage vocal dysfunction.
MCQ 5
What is the function of the external laryngeal nerve?
a) Sensory innervation above vocal cords
b) Motor supply to cricothyroid muscle
c) Motor supply to all intrinsic laryngeal muscles
d) Sensory innervation below vocal cords
Answer & Explanation:
Correct answer: b) Motor supply to cricothyroid muscle.
Explanation: The external laryngeal nerve provides motor innervation to the cricothyroid muscle, essential for modulating voice pitch. Injury affects high-pitched voice production. Knowledge of this nerve’s function aids clinicians in assessing voice disorders after surgery.
MCQ 6 - (Clinical)
Which anastomosis ensures backup sensory innervation if one laryngeal nerve is damaged?
a) Galen's anastomosis
b) Beclard's anastomosis
c) Thyrohyoid anastomosis
d) Ansell’s anastomosis
Answer & Explanation:
Correct answer: a) Galen's anastomosis.
Explanation: Galen's anastomosis connects the internal and external laryngeal nerves, providing backup sensory innervation. This redundancy ensures partial preservation of laryngeal sensation if one nerve is injured, reducing clinical deficits like aspiration.
MCQ 7
Which nerve loops around the aortic arch on the left side?
a) External laryngeal nerve
b) Internal laryngeal nerve
c) Left recurrent laryngeal nerve
d) Right recurrent laryngeal nerve
Answer & Explanation:
Correct answer: c) Left recurrent laryngeal nerve.
Explanation: The left recurrent laryngeal nerve loops around the aortic arch, while the right loops around the subclavian artery. This anatomical variation is important for surgeons to avoid inadvertent injury during mediastinal procedures.
MCQ 8 - (Clinical):
A patient has loss of high-pitched voice but normal cough reflex. Which nerve is likely damaged?
a) Recurrent laryngeal nerve
b) Internal laryngeal nerve
c) External laryngeal nerve
d) Hypoglossal nerve
Answer & Explanation:
Correct answer: c) External laryngeal nerve.
Explanation: The external laryngeal nerve controls the cricothyroid muscle, affecting pitch modulation. Its damage leads to loss of high-pitched voice without affecting the cough reflex, which depends on the internal laryngeal nerve.
MCQ 9
Which nerve is primarily responsible for cough reflex initiation?
a) External laryngeal nerve
b) Internal laryngeal nerve
c) Recurrent laryngeal nerve
d) Glossopharyngeal nerve
Answer & Explanation:
Correct answer: b) Internal laryngeal nerve.
Explanation: The internal laryngeal nerve provides sensory input to the larynx above vocal cords and is critical for initiating the cough reflex. Damage to this nerve impairs protective reflexes, leading to aspiration risk in patients.
MCQ 10 - (Clinical):
During surgery, accidental cutting of which nerve affects voice pitch but not vocal cord movement?
a) Internal laryngeal nerve
b) External laryngeal nerve
c) Recurrent laryngeal nerve
d) Vagus nerve
Answer & Explanation:
Correct answer: b) External laryngeal nerve.
Explanation: The external laryngeal nerve innervates the cricothyroid muscle, controlling voice pitch. Injury does not paralyze vocal cords but impairs pitch modulation, leading to monotonous speech. Surgeons must carefully preserve this nerve during thyroid and neck surgeries.
Topic: Nerve Supply of Larynx
Subtopic: Innervation of Intrinsic Laryngeal Muscles
Keyword Definitions:
Recurrent Laryngeal Nerve: Branch of the vagus nerve supplying all intrinsic laryngeal muscles except the cricothyroid.
Cricothyroid Muscle: Muscle that adjusts tension of vocal cords, supplied by the external branch of superior laryngeal nerve.
Vocalis Muscle: Intrinsic laryngeal muscle controlling fine tension of the vocal cords.
Thyroarytenoid Muscle: Muscle relaxing vocal cords, aiding in sound modulation.
Interarytenoid Muscle: Muscle that adducts arytenoid cartilages, closing the posterior laryngeal inlet.
Lead Question - 2013
Which muscle of larynx is not supplied by recurrent laryngeal nerve?
a) Vocalis
b) Thyroarytenoid
c) Cricothyroid
d) Interarytenoid
Explanation: The cricothyroid muscle is the only intrinsic laryngeal muscle not supplied by the recurrent laryngeal nerve. Instead, it receives innervation from the external branch of the superior laryngeal nerve and is responsible for tensing the vocal cords to modulate voice pitch. Correct answer is c) Cricothyroid.
Guessed Question 2
Lesion of recurrent laryngeal nerve causes:
a) Loss of voice pitch modulation
b) Hoarseness or aphonia
c) Loss of sensation above vocal cords
d) Loss of sensation below vocal cords
Explanation: A lesion of the recurrent laryngeal nerve affects motor supply to all intrinsic laryngeal muscles except the cricothyroid, resulting in hoarseness, voice fatigue, or even aphonia. Sensory loss below the vocal cords may also occur. Correct answer is b) Hoarseness or aphonia.
Guessed Question 3
External branch of superior laryngeal nerve innervates:
a) Vocalis
b) Cricothyroid
c) Thyroarytenoid
d) Interarytenoid
Explanation: The external branch of the superior laryngeal nerve provides motor innervation to the cricothyroid muscle, which is crucial for regulating vocal cord tension and pitch. Other intrinsic muscles are supplied by the recurrent laryngeal nerve. Correct answer is b) Cricothyroid.
Guessed Question 4
Internal branch of superior laryngeal nerve provides:
a) Motor supply to cricothyroid
b) Sensory to larynx above vocal cords
c) Sensory to larynx below vocal cords
d) Motor supply to interarytenoid
Explanation: The internal branch of the superior laryngeal nerve supplies sensory innervation to the laryngeal mucosa above the vocal cords. It does not have a motor function. Recurrent laryngeal nerve supplies motor innervation below vocal cords. Correct answer is b) Sensory to larynx above vocal cords.
Guessed Question 5
Recurrent laryngeal nerve arises from the vagus nerve at which level on the right side?
a) Arch of aorta
b) Subclavian artery
c) Common carotid artery
d) Carotid sheath
Explanation: On the right side, the recurrent laryngeal nerve arises from the vagus nerve at the level of the subclavian artery, looping around it before ascending toward the larynx. This anatomical pathway makes it vulnerable during surgery. Correct answer is b) Subclavian artery.
Guessed Question 6
Which structure is at risk of injury in thyroid surgery?
a) Internal laryngeal nerve
b) Recurrent laryngeal nerve
c) External carotid artery
d) Vagus nerve trunk
Explanation: The recurrent laryngeal nerve is at risk during thyroid surgeries because of its close anatomical relationship to the thyroid gland. Injury leads to vocal cord paralysis, hoarseness, or airway obstruction. Careful dissection is required to avoid this complication. Correct answer is b) Recurrent laryngeal nerve.
Guessed Question 7
Which of the following is NOT an intrinsic laryngeal muscle?
a) Thyroarytenoid
b) Cricothyroid
c) Sternohyoid
d) Vocalis
Explanation: The sternohyoid muscle is an extrinsic laryngeal muscle involved in depressing the hyoid bone and larynx. The intrinsic muscles, such as thyroarytenoid, cricothyroid, and vocalis, control vocal cord tension and position. Correct answer is c) Sternohyoid.
Guessed Question 8
Which nerve carries parasympathetic fibers to the larynx?
a) Glossopharyngeal
b) Vagus
c) Hypoglossal
d) Spinal accessory
Explanation: The vagus nerve (CN X) carries parasympathetic fibers to the larynx, in addition to its sensory and motor functions. It plays a role in regulating secretions and blood flow in the larynx. Correct answer is b) Vagus.
Guessed Question 9
The internal branch of superior laryngeal nerve enters the larynx through which structure?
a) Cricothyroid membrane
b) Thyrohyoid membrane
c) Jugular foramen
d) Carotid canal
Explanation: The internal branch of the superior laryngeal nerve penetrates the thyrohyoid membrane to provide sensory innervation to the larynx above the vocal cords. This passage is clinically important during procedures like laryngeal blocks. Correct answer is b) Thyrohyoid membrane.
Guessed Question 10
External laryngeal nerve is a branch of which nerve?
a) Vagus nerve
b) Glossopharyngeal nerve
c) Hypoglossal nerve
d) Facial nerve
Explanation: The external laryngeal nerve is a branch of the vagus nerve (CN X) and provides motor innervation to the cricothyroid muscle. Its damage affects pitch modulation, leading to monotone voice. Correct answer is a) Vagus nerve.
Topic: Nerve Supply of Larynx
Subtopic: Innervation of Larynx
Keyword Definitions:
Larynx: An organ located in the neck involved in breathing, producing sound, and protecting the trachea against food aspiration.
Superior Laryngeal Nerve: Branch of the vagus nerve (CN X) that provides sensation above the vocal cords and motor supply to the cricothyroid muscle.
Recurrent Laryngeal Nerve: Branch of the vagus nerve looping around major arteries; provides motor supply to intrinsic laryngeal muscles and sensation below the vocal cords.
Glossopharyngeal Nerve: Cranial nerve IX, involved in taste, sensory supply to pharynx, and carotid body functions, not directly involved in laryngeal innervation.
External Laryngeal Nerve: Branch of the superior laryngeal nerve supplying the cricothyroid muscle.
Lead Question - 2013
Nerve supply of larynx above level of vocal cord?
a) Superior laryngeal
b) Recurrent laryngeal
c) Glossopharyngeal
d) External laryngeal
Explanation: The larynx above the vocal cords receives sensory innervation from the internal branch of the superior laryngeal nerve, which is a branch of the vagus nerve (CN X). This nerve is essential for protective reflexes and sensation. Recurrent laryngeal supplies below the vocal cords. Correct answer is a) Superior laryngeal.
Guessed Question 2
Which nerve supplies the cricothyroid muscle?
a) Recurrent laryngeal
b) Internal laryngeal
c) External laryngeal
d) Glossopharyngeal
Explanation: The external branch of the superior laryngeal nerve supplies the cricothyroid muscle, responsible for tensing the vocal cords and modulating pitch. Damage causes monotone voice. Recurrent laryngeal nerve supplies other intrinsic laryngeal muscles. Correct answer is c) External laryngeal.
Guessed Question 3
Recurrent laryngeal nerve provides motor supply to:
a) Cricothyroid
b) All intrinsic laryngeal muscles except cricothyroid
c) Only vocalis muscle
d) None of the above
Explanation: The recurrent laryngeal nerve supplies motor innervation to all intrinsic muscles of the larynx except the cricothyroid, which is supplied by the external laryngeal nerve. This is clinically significant in thyroid surgeries. Correct answer is b) All intrinsic laryngeal muscles except cricothyroid.
Guessed Question 4
Sensory supply below vocal cords is provided by?
a) Glossopharyngeal nerve
b) Internal laryngeal nerve
c) Recurrent laryngeal nerve
d) External laryngeal nerve
Explanation: The recurrent laryngeal nerve provides sensory innervation to the larynx below the vocal cords and motor supply to the intrinsic muscles, except the cricothyroid. This is important for cough reflex and surgical risk. Correct answer is c) Recurrent laryngeal nerve.
Guessed Question 5
Injury to superior laryngeal nerve results in:
a) Loss of sensation below vocal cords
b) Loss of high-pitched voice
c) Complete voice loss
d) Horner's syndrome
Explanation: Injury to the superior laryngeal nerve, particularly its external branch, affects the cricothyroid muscle, resulting in loss of pitch modulation, especially high-pitched sounds. The recurrent laryngeal nerve injury causes hoarseness but not high-pitched voice loss specifically. Correct answer is b) Loss of high-pitched voice.
Guessed Question 6
Internal branch of superior laryngeal nerve provides:
a) Motor to intrinsic laryngeal muscles
b) Sensory above vocal cords
c) Sensory below vocal cords
d) Motor to cricothyroid
Explanation: The internal branch of superior laryngeal nerve provides sensory innervation to the mucosa of the larynx above the vocal cords, important in protective airway reflexes like cough. It does not supply motor fibers. Correct answer is b) Sensory above vocal cords.
Guessed Question 7
Which nerve is most at risk during thyroid surgery?
a) Internal laryngeal
b) Recurrent laryngeal
c) External laryngeal
d) Glossopharyngeal
Explanation: The recurrent laryngeal nerve is particularly vulnerable during thyroidectomy, as it courses close to the thyroid gland. Injury leads to vocal cord paralysis, hoarseness, or airway obstruction. Careful dissection avoids damage. Correct answer is b) Recurrent laryngeal.
Guessed Question 8
The superior laryngeal nerve is a branch of which cranial nerve?
a) CN IX
b) CN X
c) CN XI
d) CN XII
Explanation: The superior laryngeal nerve is a branch of the vagus nerve (Cranial Nerve X). It divides into internal and external branches, providing sensory and motor innervation of the larynx. Correct answer is b) CN X.
Guessed Question 9
Function of cricothyroid muscle is to:
a) Adduct vocal cords
b) Abduct vocal cords
c) Tense vocal cords
d) Relax vocal cords
Explanation: The cricothyroid muscle, innervated by the external branch of the superior laryngeal nerve, tenses the vocal cords, increasing pitch. It is essential for modulating voice frequency. Injury leads to monotone voice. Correct answer is c) Tense vocal cords.
Guessed Question 10
Lesion of internal laryngeal nerve results in:
a) Loss of cough reflex
b) Hoarseness
c) Loss of taste
d) Difficulty swallowing
Explanation: Lesion of the internal laryngeal nerve results in loss of sensation above the vocal cords, impairing the cough reflex and increasing the risk of aspiration pneumonia. It does not cause motor deficits. Correct answer is a) Loss of cough reflex.
Topic: Female Reproductive System
Subtopic: Uterine Anomalies
Keyword Definitions:
Cochleate uterus: Uterus with acute retroflexion, where the body of the uterus is sharply bent backward over the cervix.
Anteflexion: Forward bending of the uterine body over the cervix.
Retroflexion: Backward bending of the uterine body over the cervix.
Uterine anomalies: Variations in uterine shape or structure that can affect fertility or cause symptoms like pain.
Clinical significance: Cochleate uterus may lead to menstrual disorders or infertility due to abnormal uterine positioning.
Cervix: The lower part of the uterus connecting to the vagina, important in childbirth and menstrual flow.
Lead Question - 2013
Cochleate uterus is ?
a) Large uterus
b) Acute anteflexion
c) Acute retroflexion
d) Large cervix
Explanation: Cochleate uterus refers to an abnormal uterine position where the uterine body is sharply retroflexed over the cervix, leading to acute retroflexion. This condition can cause infertility, menstrual irregularities, and pelvic pain. It is not related to large size or anteflexion. Correct answer is c) Acute retroflexion.
Guessed Question 2
Which condition describes forward bending of the uterus?
a) Anteflexion
b) Retroflexion
c) Prolapse
d) Inversion
Explanation: Anteflexion refers to the normal forward bending of the uterus over the cervix. It is a common anatomical variant and not usually associated with clinical problems. Acute retroflexion (cochleate uterus) is the opposite condition. Correct answer is a) Anteflexion.
Guessed Question 3
Major clinical symptom of cochleate uterus may be?
a) Menstrual irregularities
b) Vaginal discharge
c) Asymptomatic
d) Pelvic infection
Explanation: Cochleate uterus may lead to menstrual irregularities and infertility due to mechanical hindrance in menstrual flow or sperm transport. Pelvic pain may also be present due to the abnormal uterine position. Correct answer is a) Menstrual irregularities.
Guessed Question 4
Uterine prolapse is best described as:
a) Downward displacement of uterus
b) Forward bending of uterus
c) Enlargement of uterus
d) Acute retroflexion of uterus
Explanation: Uterine prolapse refers to the downward displacement of the uterus into the vaginal canal, often due to weakening of pelvic support structures. This is different from cochleate uterus, which is a retroflexion. Correct answer is a) Downward displacement of uterus.
Guessed Question 5
Which imaging modality is useful to diagnose uterine position anomalies?
a) Ultrasound
b) X-ray
c) CT Scan
d) PET Scan
Explanation: Ultrasound is the primary imaging modality used to evaluate uterine position and detect anomalies such as cochleate uterus. It is non-invasive and provides real-time anatomical details. Correct answer is a) Ultrasound.
Guessed Question 6
Cochleate uterus may result in difficulty during:
a) Micturition
b) Childbirth
c) Digestion
d) Respiration
Explanation: Cochleate uterus, due to its acute retroflexion, can mechanically obstruct the birth canal and lead to difficulty during childbirth. It is not directly related to digestion or respiration. Correct answer is b) Childbirth.
Guessed Question 7
The cervix connects the uterus to:
a) Rectum
b) Vagina
c) Bladder
d) Peritoneum
Explanation: The cervix is the lower, narrow part of the uterus that connects the uterine cavity with the vagina, allowing passage of menstrual flow and sperm. It does not connect directly to bladder or rectum. Correct answer is b) Vagina.
Guessed Question 8
Which is NOT a common symptom of uterine position anomalies?
a) Dyspareunia
b) Infertility
c) Frequent urination
d) Asthma
Explanation: Uterine position anomalies such as cochleate uterus commonly cause dyspareunia (pain during intercourse), infertility, and urinary symptoms due to pressure effects. Asthma is unrelated. Correct answer is d) Asthma.
Guessed Question 9
Cochleate uterus is best managed by:
a) Hormonal therapy
b) Physiotherapy
c) Surgical correction
d) Antibiotics
Explanation: Severe cases of cochleate uterus with symptoms such as infertility or pain may require surgical correction to reposition the uterus. Hormonal therapy or physiotherapy alone is usually insufficient. Correct answer is c) Surgical correction.
Guessed Question 10
Which of the following is TRUE about retroflexed uterus?
a) Always pathological
b) May be normal variant
c) Causes immediate infertility
d) Must be surgically corrected
Explanation: A retroflexed uterus can be a normal anatomical variant and is often asymptomatic. Not all cases require surgical correction unless associated with symptoms like infertility or pain. Correct answer is b) May be normal variant.
Chapter: Anatomy
Topic: Pelvic Nerves
Subtopic: Pain Pathways in Pelvis
Keyword Definitions:
Pudendal nerve: Somatic nerve supplying perineum, external genitalia, and sphincters.
Sciatic nerve: Largest nerve of body, supplies lower limb but not pelvic viscera.
Autonomic nerves: Sympathetic and parasympathetic nerves controlling involuntary pelvic organ function and mediating visceral pain.
Pelvic pain: Often mediated by autonomic nerve fibers transmitting nociceptive signals from pelvic organs.
Clinical significance: Understanding pain pathways aids diagnosis of pelvic disorders like endometriosis or pelvic inflammatory disease.
Visceral vs Somatic pain: Visceral pain is dull, poorly localized, mediated by autonomic fibers; somatic pain is sharp, well localized.
Lead Question - 2013
Pelvic pain is mediated by ?
a) Pudendal nerve
b) Sciatic nerve
c) Autonomic nerves
d) None of the above
Explanation: Pelvic pain, especially from internal pelvic organs like uterus, bladder, or rectum, is primarily mediated by autonomic (sympathetic and parasympathetic) nerves. These nerves carry visceral afferent fibers responsible for dull, poorly localized pain sensations. Pudendal and sciatic nerves are somatic and do not mediate visceral pelvic pain. Correct answer is c) Autonomic nerves.
Guessed Question 2
Which nerve supplies the external genitalia and perineum?
a) Pudendal nerve
b) Sciatic nerve
c) Pelvic splanchnic nerve
d) Obturator nerve
Explanation: The pudendal nerve provides motor and sensory innervation to the external genitalia and perineum, important in voluntary control of sphincters. It does not mediate visceral pelvic pain. Correct answer is a) Pudendal nerve.
Guessed Question 3
Visceral pain in pelvis is typically described as?
a) Sharp and localized
b) Dull and poorly localized
c) Burning
d) Electric shock-like
Explanation: Visceral pain in the pelvis is usually dull and poorly localized due to autonomic nerve mediation. It contrasts with somatic pain, which is sharp and well localized. Correct answer is b) Dull and poorly localized.
Guessed Question 4
Which of the following nerves carries parasympathetic fibers to pelvic organs?
a) Pudendal nerve
b) Pelvic splanchnic nerve
c) Hypogastric nerve
d) Sciatic nerve
Explanation: The pelvic splanchnic nerves (S2-S4) carry parasympathetic fibers to pelvic organs, regulating functions such as bladder contraction and genital erection. These are involved in autonomic control but not in direct somatic pain transmission. Correct answer is b) Pelvic splanchnic nerve.
Guessed Question 5
Sympathetic fibers mediating pelvic pain originate from which spinal segments?
a) T1-T5
b) T10-L2
c) S2-S4
d) L4-L5
Explanation: Sympathetic fibers mediating pelvic visceral pain primarily originate from spinal segments T10 to L2, traveling via hypogastric plexus to pelvic organs. These pathways carry nociceptive signals in disease states. Correct answer is b) T10-L2.
Guessed Question 6
Which of the following is true regarding somatic pelvic pain?
a) Mediated by autonomic nerves
b) Poorly localized
c) Sharp and well localized
d) Does not involve nerve endings
Explanation: Somatic pelvic pain is sharp and well localized because it is mediated by somatic nerves like the pudendal nerve, which carries precise sensory input from skin, muscles, and deeper somatic structures. Correct answer is c) Sharp and well localized.
Guessed Question 7
Pudendal nerve block is used for:
a) Pain relief during childbirth
b) Lower limb surgery
c) Appendectomy
d) Hernia repair
Explanation: Pudendal nerve block is commonly used for analgesia during childbirth, targeting somatic nerves supplying the perineum and external genitalia. It does not block autonomic-mediated visceral pain. Correct answer is a) Pain relief during childbirth.
Guessed Question 8
Which nerve is NOT involved in transmitting pelvic visceral pain?
a) Pelvic splanchnic nerve
b) Hypogastric nerve
c) Pudendal nerve
d) Vagus nerve
Explanation: The pudendal nerve is primarily somatic, serving perineum and external genitalia, and does not carry visceral afferents. Visceral pain is mediated by pelvic splanchnic, hypogastric, and vagus nerves. Correct answer is c) Pudendal nerve.
Guessed Question 9
Visceral pelvic pain is aggravated by:
a) Movement
b) Organ distension
c) Skin irritation
d) Muscle contraction
Explanation: Visceral pelvic pain is often aggravated by organ distension or ischemia due to stretch-sensitive receptors in autonomic nerves. Movement or somatic stimuli typically affect somatic pain. Correct answer is b) Organ distension.
Guessed Question 10
Which of the following is NOT a function of autonomic nerves in pelvis?
a) Regulate bladder contraction
b) Mediate erection
c) Innervate external anal sphincter
d) Transmit visceral pain
Explanation: Autonomic nerves regulate involuntary functions like bladder contraction, genital erection, and visceral pain. The external anal sphincter is innervated by somatic pudendal nerve, not autonomic nerves. Correct answer is c) Innervate external anal sphincter.
Chapter: Anatomy
Topic: Pelvis and Perineum
Subtopic: Sacrotuberous Ligament and Related Structures
Keyword Definitions:
Sacrotuberous Ligament: Strong ligament connecting sacrum to ischial tuberosity, stabilizing the sacroiliac joint.
Perforating Cutaneous Nerve: Small branch of sacral plexus that pierces sacrotuberous ligament to supply buttock skin.
Posterior Femoral Cutaneous Nerve: Nerve supplying posterior thigh and gluteal skin, passing beneath gluteus maximus.
Sciatic Nerve: Largest nerve of the body, leaves pelvis via greater sciatic foramen below piriformis.
Superior Gluteal Nerve: Nerve passing through greater sciatic foramen above piriformis to supply gluteus medius, minimus, and tensor fascia lata.
Clinical Relevance: Injury to sacrotuberous ligament region can cause entrapment neuropathy or gluteal pain syndromes.
Lead Question - 2013
Sacrotuberous ligament is pierced by
a) Perforating cutaneous nerve
b) Posterior femoral cutaneous
c) Superior gluteal nerve
d) Sciatic nerve
Explanation: The sacrotuberous ligament is pierced only by the perforating cutaneous nerve, which arises from the sacral plexus and supplies the medial gluteal region skin. Other nerves pass through sciatic foramina. The correct answer is a) Perforating cutaneous nerve.
Guessed Question 2
Which ligament forms the lower boundary of the lesser sciatic foramen?
a) Sacrospinous ligament
b) Sacrotuberous ligament
c) Inguinal ligament
d) Obturator membrane
Explanation: The sacrotuberous ligament extends from sacrum to ischial tuberosity, forming the lower boundary of lesser sciatic foramen. The sacrospinous ligament forms its upper boundary. The correct answer is b) Sacrotuberous ligament.
Guessed Question 3
Which nerve passes below the piriformis through the greater sciatic foramen?
a) Sciatic nerve
b) Superior gluteal nerve
c) Obturator nerve
d) Femoral nerve
Explanation: The sciatic nerve passes through the greater sciatic foramen below piriformis, running deep to gluteus maximus. Compression here may cause sciatica. The correct answer is a) Sciatic nerve.
Guessed Question 4
Which ligament along with sacrospinous converts sciatic notch into foramen?
a) Sacrotuberous ligament
b) Iliofemoral ligament
c) Pubofemoral ligament
d) Ischiofemoral ligament
Explanation: Sacrotuberous and sacrospinous ligaments transform greater and lesser sciatic notches into foramina through which pelvic nerves and vessels pass. The correct answer is a) Sacrotuberous ligament.
Guessed Question 5
In piriformis syndrome, which nerve is compressed as it passes beneath piriformis?
a) Sciatic nerve
b) Pudendal nerve
c) Posterior femoral cutaneous nerve
d) Inferior gluteal nerve
Explanation: In piriformis syndrome, the sciatic nerve gets compressed as it passes below piriformis, producing pain radiating to posterior thigh and leg. The correct answer is a) Sciatic nerve.
Guessed Question 6
Which nerve leaves pelvis through the lesser sciatic foramen?
a) Pudendal nerve
b) Superior gluteal nerve
c) Posterior femoral cutaneous nerve
d) Sciatic nerve
Explanation: The pudendal nerve exits through greater sciatic foramen, hooks around ischial spine and sacrospinous ligament, and re-enters through lesser sciatic foramen into the perineum. The correct answer is a) Pudendal nerve.
Guessed Question 7
Which structure passes between sacrotuberous and sacrospinous ligaments?
a) Pudendal nerve
b) Obturator nerve
c) Femoral nerve
d) Genitofemoral nerve
Explanation: Pudendal nerve, internal pudendal vessels, and nerve to obturator internus pass between sacrospinous and sacrotuberous ligaments to enter lesser sciatic foramen. The correct answer is a) Pudendal nerve.
Guessed Question 8
Damage to superior gluteal nerve affects which function?
a) Hip abduction
b) Hip extension
c) Hip adduction
d) Knee flexion
Explanation: Superior gluteal nerve supplies gluteus medius and minimus. Damage weakens hip abduction and causes positive Trendelenburg sign due to failure of pelvic support. The correct answer is a) Hip abduction.
Guessed Question 9
Which ligament resists posterior rotation of sacrum?
a) Sacrotuberous ligament
b) Anterior sacroiliac ligament
c) Iliofemoral ligament
d) Inguinal ligament
Explanation: Sacrotuberous ligament helps resist posterior rotation of sacrum at sacroiliac joint, stabilizing pelvis against body weight. The correct answer is a) Sacrotuberous ligament.
Guessed Question 10
Which nerve supplies posterior thigh skin but does not pierce sacrotuberous ligament?
a) Posterior femoral cutaneous nerve
b) Perforating cutaneous nerve
c) Obturator nerve
d) Pudendal nerve
Explanation: Posterior femoral cutaneous nerve passes below gluteus maximus to supply posterior thigh skin but does not pierce sacrotuberous ligament. The correct answer is a) Posterior femoral cutaneous nerve.
Guessed Question 11
Entrapment of perforating cutaneous nerve causes pain in which region?
a) Medial gluteal region
b) Posterior thigh
c) Perineum
d) Lateral leg
Explanation: The perforating cutaneous nerve, piercing sacrotuberous ligament, supplies medial gluteal region. Entrapment or irritation causes localized gluteal pain. The correct answer is a) Medial gluteal region.
Keyword Definitions
• Flexor retinaculum (ankle) – Fibrous band over medial ankle forming the roof of the tarsal tunnel.
• Tarsal tunnel – Space deep to flexor retinaculum transmitting tendons, vessels, and nerve into the foot.
• Posterior tibial artery – Major artery passing through tarsal tunnel to supply plantar foot; palpable as posterior tibial pulse.
• Tibialis anterior tendon – Anterior compartment tendon crossing dorsum of foot; does NOT pass under flexor retinaculum.
• Peroneus tertius – Anterior-lateral tendon inserting on dorsum of 5th metatarsal; not in tarsal tunnel.
• Long saphenous (great saphenous) vein – Superficial vein running anterior to medial malleolus, superficial to retinaculum.
• Tom, Dick And Very Nervous Harry – Mnemonic for structures deep to flexor retinaculum: Tibialis posterior, flexor Digitorum longus, posterior tibial Artery, posterior tibial Vein, tibial Nerve, flexor Hallucis longus.
• Tinel’s sign (at ankle) – Tapping over flexor retinaculum producing tingling in tunnel distribution, suggests tibial nerve entrapment.
• Posterior tibial pulse – Palpated just posterior to medial malleolus deep to flexor retinaculum; important in vascular exam.
• Clinical relevance – Tarsal tunnel syndrome results from compression of structures under flexor retinaculum causing plantar numbness/pain.
Chapter: Anatomy / Lower Limb
Topic: Ankle Region
Subtopic: Flexor Retinaculum (Tarsal Tunnel) Contents
Lead Question – 2013
Structure passing deep to flexor retinaculum is ?
a) Posterior tibial artery
b) Long saphenous vein
c) Tibialis anterior tendon
d) Peroneus tertius
Explanation: The posterior tibial artery passes deep to the flexor retinaculum within the tarsal tunnel alongside tendons and the tibial nerve. The long saphenous vein is superficial, tibialis anterior and peroneus tertius are anterior tendons. Correct answer: (a) Posterior tibial artery. Clinically the posterior tibial pulse is palpable here.
Guessed Questions for NEET PG
1) The tarsal tunnel contains all EXCEPT:
a) Tibialis posterior tendon
b) Flexor digitorum longus tendon
c) Peroneus longus tendon
d) Posterior tibial nerve
Explanation: The peroneus longus runs laterally and passes under the cuboid (peroneal groove), not through the tarsal tunnel. The tunnel contains tibialis posterior, FDL, posterior tibial vessels, tibial nerve, and FHL. Correct answer: Peroneus longus. Tarsal tunnel syndrome spares lateral tendons.
2) Posterior tibial pulse is best palpated:
a) Anterior to lateral malleolus
b) Posterior to medial malleolus beneath flexor retinaculum
c) On dorsum of foot lateral to EHL tendon
d) In popliteal fossa only
Explanation: The posterior tibial artery runs deep to flexor retinaculum posterior to the medial malleolus; its pulse is palpated there. Correct answer: Posterior to medial malleolus beneath flexor retinaculum. Loss of this pulse suggests distal arterial compromise.
3) Tinel’s sign at the tarsal tunnel tests for entrapment of which nerve?
a) Superficial peroneal nerve
b) Deep peroneal nerve
c) Tibial nerve (posterior tibial nerve in tunnel)
d) Sural nerve
Explanation: Tinel’s tapping over flexor retinaculum reproduces paresthesia in tibial nerve distribution (sole) when entrapped. Correct answer: Tibial nerve. Positive test aids diagnosis of tarsal tunnel syndrome which presents with plantar numbness and burning pain.
4) Which tendon is most medial under the flexor retinaculum (medial to lateral order)?
a) Flexor hallucis longus
b) Tibialis posterior
c) Flexor digitorum longus
d) Peroneus brevis
Explanation: Medial-to-lateral order in tarsal tunnel is tibialis posterior, flexor digitorum longus, posterior tibial vessels/nerve, then flexor hallucis longus more laterally. Peroneal tendons are lateral. Correct answer: Tibialis posterior. Important during surgical decompression.
5) Compression of posterior tibial nerve in the tarsal tunnel causes loss of sensation over:
a) Dorsum of foot only
b) Plantar surface of foot and toes
c) Lateral calf only
d) Medial thigh
Explanation: Tibial nerve supplies plantar cutaneous nerves; entrapment in tarsal tunnel causes plantar burning, numbness, and possible intrinsic muscle weakness. Correct answer: Plantar surface of foot and toes. Tarsal tunnel mimics plantar fasciitis clinically sometimes.
6) A patient with rupture of posterior tibial artery in ankle trauma will most likely present with:
a) Loss of dorsalis pedis pulse only
b) Absent posterior tibial pulse, ischemic plantar changes
c) Isolated foot drop
d) Loss of great saphenous waveform only
Explanation: Rupture of posterior tibial artery abolishes its palpable pulse and can compromise plantar circulation causing ischemic changes. Dorsalis pedis may be maintained via anterior tibial flow. Correct answer: Absent posterior tibial pulse, ischemic plantar changes. Urgent vascular assessment required.
7) The mnemonic “Tom, Dick, And Very Nervous Harry” lists structures in tarsal tunnel in which order?
a) Tibialis posterior, flexor Digitorum longus, posterior tibial Artery, posterior tibial Vein, tibial Nerve, flexor Hallucis longus
b) Tibialis anterior, extensor Digitorum longus, anterior tibial Artery…
c) Peroneus longus, peroneus brevis, sural nerve…
d) Flexor hallucis longus first then others
Explanation: The mnemonic correctly orders tibialis posterior, FDL, posterior tibial artery, posterior tibial vein, tibial nerve, and flexor hallucis longus (from medial to lateral). Correct answer: (a). Surgeons use this to identify structures during decompression.
8) Which vessel provides collateral supply to plantar arch if posterior tibial artery is occluded?
a) Anterior tibial (via dorsalis pedis and perforating branches)
b) Great saphenous vein
c) Peroneal artery exclusively without connections
d) Small saphenous vein
Explanation: Anterior tibial continues as dorsalis pedis and via perforating branches can contribute to plantar arches, providing collateral flow when posterior tibial artery is occluded. Correct answer: Anterior tibial (via dorsalis pedis). Clinical: Important in planning bypass and assessing ischemia.
9) Long saphenous vein at the ankle is located relative to flexor retinaculum as:
a) Deep to retinaculum within tarsal tunnel
b) Superficial to retinaculum anterior to medial malleolus
c) Passing through lateral retinaculum
d) Within tarsal tunnel posterior to tibial nerve
Explanation: The great saphenous vein is superficial on the medial ankle, anterior to the medial malleolus and superficial to the flexor retinaculum, used for venous cutdown access. Correct answer: Superficial to retinaculum anterior to medial malleolus. Preserve saphenous nerve during harvest.
10) Surgical decompression for tarsal tunnel syndrome requires incision of which structure?
a) Flexor retinaculum (tarsal tunnel roof)
b) Extensor retinaculum
c) Plantar aponeurosis only
d) Lateral ankle ligament complex
Explanation: Tarsal tunnel release involves incising the flexor retinaculum to decompress tibial nerve and associated structures. Correct answer: Flexor retinaculum. Timing is important as chronic compression can cause irreversible neuropathy and intrinsic foot muscle atrophy.
Keyword Definitions
• Anterior interosseous nerve (AIN) – A pure motor branch of the median nerve that supplies deep forearm flexors.
• Flexor pollicis longus (FPL) – Muscle of thumb flexion at interphalangeal joint, supplied by AIN.
• Flexor digitorum profundus (FDP) – Medial part supplied by ulnar nerve, lateral part by AIN.
• Flexor digitorum superficialis (FDS) – Flexes proximal interphalangeal joints, supplied by median nerve proper (not AIN).
• Flexor carpi ulnaris (FCU) – Flexes and adducts wrist, supplied by ulnar nerve.
• Brachioradialis – Forearm flexor in mid-pronation, supplied by radial nerve.
• Abductor pollicis brevis (APB) – Thenar muscle, supplied by recurrent branch of median nerve.
• Kiloh–Nevin syndrome – Clinical syndrome due to anterior interosseous nerve palsy.
• Froment’s sign – Indicates ulnar nerve palsy when adductor pollicis is weak.
• Nerve lesions – Important in differential diagnosis of anterior compartment weakness.
Chapter: Anatomy / Upper Limb
Topic: Nerve Supply
Subtopic: Anterior Interosseous Nerve
Lead Question – 2013
All are supplied by anterior interosseous nerve except –
a) Flexor carpi ulnaris
b) Brachioradialis
c) Abductor pollicis brevis
d) Flexor pollicis longus
e) Flexor digitorum superficialis
Explanation: The anterior interosseous nerve supplies flexor pollicis longus, pronator quadratus, and the lateral half of flexor digitorum profundus. Muscles like FCU (ulnar nerve), brachioradialis (radial nerve), APB (recurrent median), and FDS (median nerve proper) are not supplied by AIN. Correct answer: All except FPL.
Guessed Questions for NEET PG
1) Anterior interosseous nerve is a branch of:
a) Ulnar nerve
b) Radial nerve
c) Median nerve
d) Musculocutaneous nerve
Explanation: The anterior interosseous nerve is a motor branch of the median nerve that arises in the proximal forearm. It runs along the interosseous membrane supplying deep flexors. Correct answer: Median nerve. It carries no cutaneous fibers, making its lesions purely motor.
2) Kiloh–Nevin syndrome refers to:
a) Ulnar nerve palsy
b) Radial nerve entrapment
c) Anterior interosseous nerve palsy
d) Posterior interosseous nerve palsy
Explanation: Kiloh–Nevin syndrome is anterior interosseous nerve palsy, presenting with inability to make the “OK sign” due to weakness of FPL and FDP (index finger). Correct answer: Anterior interosseous nerve palsy. It is often due to compression or neuritis.
3) Inability to flex thumb IP joint is seen in lesion of:
a) Median nerve at wrist
b) Ulnar nerve at elbow
c) Anterior interosseous nerve
d) Radial nerve in spiral groove
Explanation: The flexor pollicis longus, innervated by AIN, flexes thumb IP joint. Its palsy causes inability to flex the thumb tip. Correct answer: Anterior interosseous nerve. This finding is a diagnostic clue for AIN syndrome.
4) Which muscle is NOT supplied by AIN?
a) Pronator quadratus
b) FPL
c) FDP (lateral half)
d) FDS
Explanation: The anterior interosseous nerve supplies pronator quadratus, FPL, and lateral half of FDP. The FDS is supplied by the main trunk of the median nerve, not AIN. Correct answer: FDS. This helps localize nerve lesions in clinical practice.
5) Patient unable to flex index finger DIP joint likely has lesion in:
a) Radial nerve
b) Ulnar nerve
c) AIN
d) Musculocutaneous nerve
Explanation: The lateral part of flexor digitorum profundus (index and middle fingers) is supplied by AIN. Inability to flex DIP of index suggests AIN palsy. Correct answer: AIN. Ulnar supplies medial part for ring and little fingers.
6) Which test detects AIN palsy?
a) Phalen’s test
b) Froment’s sign
c) Pinch “OK” sign test
d) Tinel’s sign
Explanation: In AIN palsy, patient cannot make a round “O” with thumb and index, instead forming a triangular pinch due to weakness of FPL and FDP. Correct answer: Pinch “OK” sign test. This is diagnostic of AIN syndrome.
7) Which nerve supplies pronator quadratus?
a) Radial
b) Ulnar
c) Anterior interosseous
d) Posterior interosseous
Explanation: Pronator quadratus, a deep forearm pronator, is supplied exclusively by the anterior interosseous nerve. Correct answer: Anterior interosseous. Lesion impairs pronation, especially when forearm is flexed, and reduces grip strength.
8) A forearm fracture with isolated motor palsy (no sensory loss) indicates lesion of:
a) Ulnar nerve
b) Radial nerve
c) Anterior interosseous nerve
d) Median nerve proper
Explanation: Since the anterior interosseous nerve is a pure motor branch without cutaneous innervation, its injury causes motor weakness only. Correct answer: Anterior interosseous nerve. This differentiates it from other mixed nerves.
9) Which thenar muscle is NOT supplied by anterior interosseous nerve?
a) Abductor pollicis brevis
b) FPL
c) Opponens pollicis
d) Adductor pollicis
Explanation: Abductor pollicis brevis and other thenar muscles are supplied by the recurrent branch of the median nerve. Adductor pollicis is supplied by ulnar. Only FPL is under AIN supply. Correct answer: Abductor pollicis brevis.
10) A patient with supracondylar fracture develops inability to flex thumb IP and index DIP joints. Likely involved nerve is:
a) Radial
b) Ulnar
c) AIN
d) Musculocutaneous
Explanation: This classic presentation is due to AIN palsy following trauma, causing paralysis of FPL and FDP (index). Correct answer: AIN. Distinguishing feature is pure motor deficit with preserved cutaneous sensation.
Keyword Definitions
• Musculocutaneous nerve – Terminal branch of lateral cord of brachial plexus, supplies flexors of arm.
• Brachial plexus – Nerve network supplying upper limb.
• Flexor compartment of arm – Contains biceps brachii, brachialis, and coracobrachialis.
• Biceps brachii – Flexor of elbow, supinator of forearm.
• Coracobrachialis – Flexes and adducts the arm.
• Brachialis – Primary flexor of elbow.
• Radial nerve – Supplies extensor compartment of arm.
• Median nerve – Supplies forearm and hand, not arm flexors.
• Ulnar nerve – Supplies intrinsic hand muscles and part of forearm.
• Clinical test – Elbow flexion and cutaneous sensation of lateral forearm test musculocutaneous nerve.
• Upper limb injuries – Trauma, fractures, or entrapment may affect musculocutaneous nerve function.
Chapter: Anatomy / Upper Limb
Topic: Brachial Plexus
Subtopic: Musculocutaneous Nerve and Arm Flexors
Lead Question – 2013
Nerve supply to the muscles of flexor compartment of arm?
a) Radial nerve
b) Median nerve
c) Musculocutaneous nerve
d) Ulnar nerve
Explanation: The flexor compartment of the arm (biceps brachii, brachialis, coracobrachialis) is innervated by the musculocutaneous nerve, a branch of the lateral cord of the brachial plexus. Correct answer: (c) Musculocutaneous nerve. Clinical: Injury leads to weak elbow flexion and sensory loss over lateral forearm.
Guessed Questions for NEET PG
1) Which nerve continues as the lateral cutaneous nerve of forearm?
a) Radial
b) Median
c) Musculocutaneous
d) Ulnar
Explanation: Musculocutaneous nerve ends as lateral cutaneous nerve of forearm. Correct answer: Musculocutaneous nerve. Clinical: Injury causes sensory loss in lateral forearm.
2) Which muscle is pierced by musculocutaneous nerve?
a) Biceps brachii
b) Coracobrachialis
c) Brachialis
d) Deltoid
Explanation: Musculocutaneous nerve pierces coracobrachialis before supplying flexor compartment. Correct answer: Coracobrachialis. Clinical: Landmark for nerve tracing.
3) Which muscle in flexor compartment also receives supply from radial nerve?
a) Biceps brachii
b) Coracobrachialis
c) Brachialis
d) None
Explanation: Brachialis is mainly supplied by musculocutaneous nerve, but radial nerve gives additional innervation. Correct answer: Brachialis. Clinical: Explains preserved flexion in musculocutaneous injury.
4) Elbow flexion against resistance tests mainly?
a) Median nerve
b) Musculocutaneous nerve
c) Ulnar nerve
d) Axillary nerve
Explanation: Biceps brachii and brachialis, innervated by musculocutaneous nerve, are prime elbow flexors. Correct answer: Musculocutaneous nerve. Clinical: Used to check function.
5) Which branch of brachial plexus gives rise to musculocutaneous nerve?
a) Lateral cord
b) Posterior cord
c) Medial cord
d) Upper trunk
Explanation: Musculocutaneous nerve arises from the lateral cord (C5–C7 roots). Correct answer: Lateral cord. Clinical: Knowledge useful in brachial plexus blocks.
6) Sensory loss over lateral forearm is due to injury of?
a) Radial nerve
b) Musculocutaneous nerve
c) Median nerve
d) Ulnar nerve
Explanation: Lateral cutaneous nerve of forearm (continuation of musculocutaneous) supplies skin here. Correct answer: Musculocutaneous nerve. Clinical: Sensory deficit confirms diagnosis.
7) Which movement is most affected in musculocutaneous nerve injury?
a) Shoulder abduction
b) Elbow flexion
c) Wrist extension
d) Finger flexion
Explanation: Musculocutaneous injury impairs elbow flexion due to paralysis of biceps and brachialis. Correct answer: Elbow flexion. Clinical: Weak supination also observed.
8) A patient with injury to musculocutaneous nerve will show weakness of?
a) Forearm pronation
b) Elbow flexion
c) Finger extension
d) Thumb opposition
Explanation: Injury leads to loss of flexors of arm, causing weak elbow flexion. Correct answer: Elbow flexion. Clinical: Supination also affected due to biceps involvement.
9) Which muscle is absent in flexor compartment if musculocutaneous nerve is injured?
a) Deltoid
b) Coracobrachialis
c) Triceps
d) Supinator
Explanation: Coracobrachialis is supplied by musculocutaneous nerve and loses function in its injury. Correct answer: Coracobrachialis. Clinical: Shoulder adduction and flexion weakened.
10) Which of the following is NOT supplied by musculocutaneous nerve?
a) Biceps brachii
b) Brachialis
c) Coracobrachialis
d) Triceps brachii
Explanation: Triceps brachii belongs to extensor compartment, supplied by radial nerve. Correct answer: Triceps brachii. Clinical: Preserved triceps function rules out musculocutaneous injury.
Keyword Definitions
• Axillary nerve – Branch of posterior cord of brachial plexus, supplies deltoid and teres minor.
• Quadrangular space – Anatomical space transmitting axillary nerve and posterior circumflex humeral artery.
• Deltoid paralysis – Clinical feature of axillary nerve injury, causing loss of shoulder abduction.
• Humeral surgical neck – Common fracture site leading to axillary nerve damage.
• Circumflex humeral arteries – Branches of axillary artery encircling humerus.
• Teres minor – Rotator cuff muscle innervated by axillary nerve.
• Shoulder dislocation – Can injure axillary nerve.
• Posterior circumflex humeral artery – Runs with axillary nerve in quadrangular space.
• Clinical test – Abduction and sensation over regimental badge area for axillary nerve integrity.
• Brachial plexus – Nerve network supplying upper limb.
• Surgical relevance – Axillary nerve at risk during deltoid intramuscular injections.
Chapter: Anatomy / Upper Limb
Topic: Brachial Plexus
Subtopic: Axillary Nerve and Vessels
Lead Question – 2013
Axillary nerve is accompanied by which artery?
a) Axillary
b) Subscapular
c) Anterior circumflex humeral
d) Posterior circumflex humeral
Explanation: Axillary nerve passes through the quadrangular space along with the posterior circumflex humeral artery. This anatomical relationship is clinically important during humeral neck fractures or shoulder dislocations. Correct answer: (d) Posterior circumflex humeral artery. Clinical: Injury leads to deltoid weakness and sensory loss over regimental badge area.
Guessed Questions for NEET PG
1) Which space transmits axillary nerve and posterior circumflex humeral artery?
a) Triangular space
b) Quadrangular space
c) Cubital fossa
d) Axilla
Explanation: Axillary nerve and posterior circumflex humeral artery pass through quadrangular space. Correct answer: Quadrangular space. Clinical: Compression here can cause axillary neuropathy.
2) Fracture of surgical neck of humerus most likely injures?
a) Radial nerve
b) Axillary nerve
c) Median nerve
d) Ulnar nerve
Explanation: Surgical neck fracture endangers axillary nerve and posterior circumflex humeral artery. Correct answer: Axillary nerve. Clinical: Presents with deltoid atrophy and shoulder abduction weakness.
3) Which muscle is NOT supplied by axillary nerve?
a) Deltoid
b) Teres minor
c) Teres major
d) Skin over regimental badge
Explanation: Teres major is supplied by subscapular nerve, not axillary nerve. Correct answer: Teres major. Clinical: Differentiates axillary nerve palsy from broader plexus injury.
4) Loss of sensation over regimental badge area indicates injury to?
a) Radial nerve
b) Axillary nerve
c) Suprascapular nerve
d) Musculocutaneous nerve
Explanation: Axillary nerve injury causes sensory deficit over regimental badge area. Correct answer: Axillary nerve. Clinical: Pathognomonic for axillary neuropathy.
5) Which rotator cuff muscle is innervated by axillary nerve?
a) Supraspinatus
b) Infraspinatus
c) Teres minor
d) Subscapularis
Explanation: Teres minor is the only rotator cuff muscle supplied by axillary nerve. Correct answer: Teres minor. Clinical: Weakness in external rotation occurs in axillary nerve injury.
6) During deltoid intramuscular injection, which nerve is at risk?
a) Radial nerve
b) Median nerve
c) Axillary nerve
d) Musculocutaneous nerve
Explanation: Axillary nerve runs deep to deltoid; incorrect needle placement may injure it. Correct answer: Axillary nerve. Clinical: Presents with deltoid weakness.
7) Posterior circumflex humeral artery is a branch of?
a) Brachial artery
b) Axillary artery
c) Subclavian artery
d) Radial artery
Explanation: Posterior circumflex humeral artery arises from the 3rd part of axillary artery. Correct answer: Axillary artery. Clinical: Injured in humeral neck fractures.
8) Which movement is most affected in axillary nerve injury?
a) Elbow flexion
b) Shoulder abduction
c) Wrist extension
d) Thumb opposition
Explanation: Deltoid paralysis impairs shoulder abduction beyond 15 degrees. Correct answer: Shoulder abduction. Clinical: Differentiates from supraspinatus injury which initiates abduction.
9) Which clinical test best evaluates axillary nerve function?
a) Flexion of elbow
b) Abduction of shoulder against resistance
c) Extension of wrist
d) Pronation of forearm
Explanation: Abduction of shoulder against resistance tests deltoid function supplied by axillary nerve. Correct answer: Shoulder abduction against resistance. Clinical: Standard examination method.
10) Anterior dislocation of shoulder commonly injures?
a) Radial nerve
b) Axillary nerve
c) Median nerve
d) Ulnar nerve
Explanation: Axillary nerve lies close to shoulder joint and is frequently injured in anterior dislocation. Correct answer: Axillary nerve. Clinical: Presents with deltoid atrophy and regimental badge anesthesia.
Keyword Definitions
• Dual nerve supply – Muscle receiving motor innervation from two different nerves.
• Subscapularis – Supplied by upper and lower subscapular nerves.
• Pectoralis major – Supplied by medial and lateral pectoral nerves.
• Pronator teres – Supplied by median nerve only.
• Flexor digitorum profundus – Medial half by ulnar nerve, lateral half by anterior interosseous branch of median nerve.
• Brachial plexus – Network of nerves supplying upper limb, roots C5–T1.
• Median nerve – Formed by medial and lateral cords, supplies most forearm flexors.
• Ulnar nerve – Arises from medial cord, supplies intrinsic hand muscles and medial FDP.
• Subscapular nerves – Branches of posterior cord, innervate subscapularis.
• Clinical correlation – Knowledge of dual supply important in nerve lesions and recovery.
• Muscle palsy – Weakness pattern helps localize lesion to specific nerve or part of plexus.
Chapter: Anatomy / Upper Limb
Topic: Brachial Plexus and Muscle Innervation
Subtopic: Dual nerve supply of upper limb muscles
Lead Question – 2013
All of the following muscles have dual nerve supply except?
a) Subscapularis
b) Pectoralis major
c) Pronator teres
d) Flexor digitorum profundus
Explanation: Subscapularis has dual supply (upper and lower subscapular nerves). Pectoralis major has dual supply (medial and lateral pectoral nerves). FDP has dual supply (median and ulnar nerves). Pronator teres has single supply (median nerve). Correct answer: (c) Pronator teres. Clinical: Isolated median injury can paralyze pronator teres completely.
Guessed Questions for NEET PG
1) Which muscle among the following is supplied by both ulnar and median nerves?
a) Flexor pollicis longus
b) Flexor carpi radialis
c) Flexor digitorum profundus
d) Pronator quadratus
Explanation: Flexor digitorum profundus has dual supply – medial half by ulnar, lateral half by anterior interosseous (median). Correct answer: FDP. Clinical: Explains partial preservation in isolated lesions.
2) Subscapularis is supplied by?
a) Upper and lower subscapular nerves
b) Thoracodorsal nerve
c) Lateral pectoral nerve
d) Axillary nerve
Explanation: Subscapularis is innervated by both upper and lower subscapular nerves from posterior cord. Correct answer: Upper and lower subscapular nerves. Clinical: Injury leads to weak internal rotation.
3) Which muscle receives innervation from both medial and lateral pectoral nerves?
a) Pectoralis major
b) Pectoralis minor
c) Subclavius
d) Serratus anterior
Explanation: Pectoralis major is supplied by medial and lateral pectoral nerves. Correct answer: Pectoralis major. Clinical: Paralysis leads to weak adduction and internal rotation.
4) A patient with ulnar nerve lesion at wrist retains partial flexion of DIP of ring finger due to?
a) Median nerve supply
b) Radial nerve supply
c) Musculocutaneous nerve supply
d) Axillary nerve supply
Explanation: Lateral half of FDP (index and middle fingers) supplied by median, medial half (ring and little fingers) by ulnar. Correct answer: Median nerve supply. Clinical: Explains incomplete loss in ulnar palsy.
5) Which of the following has single nerve supply?
a) FDP
b) Pectoralis major
c) Pronator teres
d) Subscapularis
Explanation: Pronator teres is solely supplied by median nerve. Others have dual innervation. Correct answer: Pronator teres. Clinical: Useful in lesion localization.
6) Which nerve supplies medial half of flexor digitorum profundus?
a) Ulnar
b) Median
c) Radial
d) Musculocutaneous
Explanation: Medial half (ring and little fingers) of FDP is innervated by ulnar nerve. Correct answer: Ulnar nerve. Clinical: Explains weakness of DIP flexion in ulnar palsy.
7) Damage to lateral pectoral nerve causes weakness in?
a) Shoulder abduction
b) Arm adduction
c) Elbow flexion
d) Wrist extension
Explanation: Lateral pectoral nerve innervates pectoralis major, main action is adduction and internal rotation of arm. Correct answer: Arm adduction. Clinical: Loss of powerful adduction in lesion.
8) A patient with lesion of posterior cord affecting both upper and lower subscapular nerves shows weakness in?
a) Internal rotation
b) External rotation
c) Abduction
d) Supination
Explanation: Subscapularis performs internal rotation of humerus, supplied by both upper and lower subscapular nerves. Correct answer: Internal rotation. Clinical: Shoulder stability is also reduced.
9) Median nerve injury at elbow spares which of the following?
a) Pronator teres
b) Flexor digitorum profundus (medial half)
c) Flexor digitorum superficialis
d) Flexor pollicis longus
Explanation: Medial half of FDP is supplied by ulnar nerve, hence spared in median nerve injury at elbow. Correct answer: FDP (medial half). Clinical: Explains partial preservation of finger flexion.
10) Which of the following combinations represent dual innervation correctly?
a) FDP – Median & Ulnar
b) Pectoralis major – Medial & Lateral pectoral
c) Subscapularis – Upper & Lower subscapular
d) All of the above
Explanation: All mentioned muscles are examples of dual innervation. Correct answer: All of the above. Clinical: Important for understanding muscle function in partial nerve injuries.
Keyword Definitions
• Profunda brachii artery – Deep artery of arm, branch of brachial artery, runs in spiral groove.
• Spiral groove – Shallow groove on posterior humerus, occupied by radial nerve and profunda brachii artery.
• Radial nerve – Continuation of posterior cord of brachial plexus, supplies extensor compartment.
• Ulnar nerve – Arises from medial cord, passes behind medial epicondyle, supplies intrinsic hand muscles.
• Median nerve – Formed from medial and lateral cords, passes through carpal tunnel, major flexor nerve.
• Humeral shaft fracture – Common injury damaging radial nerve in spiral groove.
• Wrist drop – Clinical sign of radial nerve injury, due to loss of extensor muscle function.
• Saturday night palsy – Radial nerve compression neuropathy in spiral groove.
• Deep brachial artery – Synonym for profunda brachii artery, accompanies radial nerve.
• Extensor compartment – Muscles of posterior arm and forearm controlled by radial nerve.
• Clinical correlation – Spiral groove relation important in fractures and compressive neuropathies.
Chapter: Anatomy / Upper Limb
Topic: Arm and Brachial Plexus
Subtopic: Radial nerve and profunda brachii artery in spiral groove
Lead Question – 2013
Nerve running along with profunda brachii artery, in spiral groove?
a) Ulnar
b) Median
c) Radial
d) None
Explanation: The radial nerve runs along with the profunda brachii artery in the spiral groove of the humerus. This relationship is clinically significant as humeral shaft fractures can injure both structures. Correct answer: (c) Radial. Clinical: Injury causes wrist drop and sensory loss over dorsum of hand.
Guessed Questions for NEET PG
1) A mid-shaft fracture of humerus most commonly injures?
a) Median nerve
b) Radial nerve
c) Ulnar nerve
d) Musculocutaneous nerve
Explanation: Mid-shaft humeral fractures frequently damage the radial nerve as it lies in the spiral groove. Correct answer: Radial nerve. Clinical: Presents with wrist drop and loss of finger extension.
2) Which muscle is first affected in radial nerve palsy at spiral groove?
a) Triceps
b) Anconeus
c) Brachioradialis
d) Extensor carpi radialis longus
Explanation: Triceps is spared in spiral groove lesions. Brachioradialis and wrist extensors are first affected. Correct answer: Brachioradialis. Clinical: Weak elbow flexion in mid-pronation position.
3) Saturday night palsy refers to?
a) Ulnar nerve compression
b) Radial nerve compression
c) Median nerve compression
d) Axillary nerve compression
Explanation: Saturday night palsy occurs when prolonged compression damages the radial nerve in spiral groove during deep sleep or intoxication. Correct answer: Radial nerve compression. Clinical: Wrist drop with sensory loss.
4) Sensory loss in radial nerve injury at spiral groove involves?
a) Thenar eminence
b) Dorsum of first web space
c) Medial forearm
d) Palmar little finger
Explanation: Spiral groove injury spares triceps but causes sensory loss over dorsum of hand, particularly first web space. Correct answer: Dorsum of first web space. Clinical: Important diagnostic clue.
5) Which branch of radial nerve supplies triceps?
a) Posterior cutaneous nerve
b) Muscular branches
c) Deep branch
d) Superficial branch
Explanation: Muscular branches of radial nerve supply triceps before entering spiral groove. Correct answer: Muscular branches. Clinical: Triceps preserved in spiral groove lesions.
6) Which artery is at risk with humeral shaft fracture along with radial nerve?
a) Brachial artery
b) Profunda brachii artery
c) Radial artery
d) Ulnar artery
Explanation: Profunda brachii artery accompanies radial nerve in spiral groove, making it vulnerable in shaft fractures. Correct answer: Profunda brachii artery. Clinical: Bleeding complicates fracture management.
7) Which test best detects radial nerve palsy?
a) Asking patient to oppose thumb
b) Asking patient to extend wrist
c) Asking patient to flex DIP of index
d) Asking patient to abduct little finger
Explanation: Wrist extension is controlled by radial nerve. In palsy, patient cannot extend wrist, producing wrist drop. Correct answer: Wrist extension test. Clinical: Pathognomonic finding.
8) Wrist drop occurs due to paralysis of?
a) Flexor muscles
b) Extensor muscles
c) Pronator muscles
d) Intrinsic hand muscles
Explanation: Radial nerve injury paralyzes extensor muscles of forearm, causing wrist drop. Correct answer: Extensor muscles. Clinical: Patient presents with inability to extend wrist and fingers.
9) Which part of triceps is usually spared in spiral groove lesion?
a) Long head
b) Lateral head
c) Medial head
d) All heads affected
Explanation: Radial nerve supplies long and lateral heads of triceps before entering spiral groove, hence spared. Medial head may be affected. Correct answer: Long and lateral heads spared. Clinical: Partial triceps weakness only.
10) In radial nerve injury, supination is preserved due to action of?
a) Biceps brachii
b) Supinator
c) Pronator teres
d) Brachialis
Explanation: Supination is performed by supinator (radial nerve) and biceps brachii (musculocutaneous nerve). Even if radial nerve is injured, biceps maintains supination. Correct answer: Biceps brachii. Clinical: Supination relatively preserved in palsy.
Keyword Definitions
• Anatomical snuff box – Triangular depression on lateral wrist, important surface landmark.
• Abductor pollicis longus (APL) – Forms lateral boundary of snuff box.
• Extensor pollicis brevis (EPB) – Lateral boundary with APL.
• Extensor pollicis longus (EPL) – Forms medial boundary.
• Extensor carpi ulnaris (ECU) – Not a boundary of snuff box, lies more medially.
• Radial artery – Runs through floor of snuff box, pulse palpable.
• Scaphoid bone – Floor of snuff box, common fracture site.
• Cephalic vein – Originates near snuff box region.
• Superficial branch of radial nerve – Crosses over snuff box, provides cutaneous innervation.
• Clinical importance – Site for palpating scaphoid fracture tenderness.
• Wrist injuries – Tenderness in snuff box suggests scaphoid fracture.
Chapter: Anatomy / Upper Limb
Topic: Wrist and Hand
Subtopic: Anatomical snuff box and relations
Lead Question – 2013
Boundaries of anatomical snuff box are all except
a) APL
b) EPL
c) EPB
d) ECU
Explanation: The anatomical snuff box is bounded laterally by abductor pollicis longus and extensor pollicis brevis, medially by extensor pollicis longus. Extensor carpi ulnaris is not a boundary. Correct answer: (d) ECU. Clinical: Snuff box tenderness is diagnostic of scaphoid fracture.
Guessed Questions for NEET PG
1) Which structure forms the floor of anatomical snuff box?
a) Capitate
b) Lunate
c) Scaphoid
d) Pisiform
Explanation: The scaphoid forms the main floor of the anatomical snuff box, along with trapezium. Correct answer: Scaphoid. Clinical: Scaphoid fractures are suspected if snuff box tenderness is present after a fall on an outstretched hand.
2) Which artery passes through anatomical snuff box?
a) Ulnar artery
b) Radial artery
c) Brachial artery
d) Interosseous artery
Explanation: The radial artery runs through the floor of the snuff box before entering the palm. Correct answer: Radial artery. Clinical: Radial pulse can be palpated here in lean individuals.
3) Injury to scaphoid bone presents with?
a) Swelling of thenar eminence
b) Pain in anatomical snuff box
c) Loss of thumb extension
d) Tingling of little finger
Explanation: Scaphoid fracture commonly presents with pain and tenderness in anatomical snuff box. Correct answer: Pain in anatomical snuff box. Clinical: Delayed diagnosis risks avascular necrosis of proximal scaphoid.
4) Which tendon crosses the floor of snuff box?
a) Flexor carpi radialis
b) Extensor carpi radialis longus
c) Palmaris longus
d) Flexor digitorum profundus
Explanation: Extensor carpi radialis longus and brevis tendons form part of floor of anatomical snuff box. Correct answer: Extensor carpi radialis longus. Clinical: Palpable tendon aids in anatomical landmarking.
5) Which nerve crosses superficial to anatomical snuff box?
a) Ulnar nerve
b) Median nerve
c) Superficial radial nerve
d) Deep radial nerve
Explanation: The superficial branch of radial nerve crosses superficial to snuff box, supplying cutaneous sensation. Correct answer: Superficial radial nerve. Clinical: Injury causes numbness over dorsum of hand near thumb.
6) Which muscle tendon forms medial boundary of snuff box?
a) EPL
b) EPB
c) APL
d) ECU
Explanation: The medial boundary of the snuff box is formed by extensor pollicis longus tendon. Correct answer: EPL. Clinical: Prominent during thumb extension testing.
7) A patient with fall on outstretched hand and tenderness in snuff box most likely has?
a) Colles fracture
b) Scaphoid fracture
c) Hamate fracture
d) Lunate dislocation
Explanation: Snuff box tenderness is classic for scaphoid fracture. Correct answer: Scaphoid fracture. Clinical: Requires urgent immobilization to prevent avascular necrosis.
8) Which carpal bone is most prone to avascular necrosis after fracture?
a) Lunate
b) Pisiform
c) Scaphoid
d) Capitate
Explanation: The scaphoid bone is prone to avascular necrosis due to retrograde blood supply. Correct answer: Scaphoid. Clinical: Missed fractures can cause chronic wrist pain.
9) In wrist examination, tenderness in anatomical snuff box is tested to rule out?
a) Radial head fracture
b) Scaphoid fracture
c) Ulna styloid fracture
d) Capitate fracture
Explanation: Snuff box tenderness specifically indicates scaphoid fracture. Correct answer: Scaphoid fracture. Clinical: Common in young adults after fall on outstretched hand.
10) Which vein originates near anatomical snuff box?
a) Basilic vein
b) Cephalic vein
c) Median cubital vein
d) Radial vein
Explanation: The cephalic vein begins from venous plexus near the anatomical snuff box. Correct answer: Cephalic vein. Clinical: Important for venous access in upper limb procedures.
Keyword Definitions
• Median nerve – Major nerve of forearm and hand, supplies palmar aspect of lateral 3½ fingers.
• Ulnar nerve – Supplies medial 1½ fingers and most intrinsic hand muscles.
• Radial nerve – Provides sensation to dorsum of hand and motor supply to extensor compartment.
• Digital nerves – Terminal branches of median and ulnar nerves, supplying fingers and nail beds.
• Nail bed – Specialized skin beneath nail plate, richly innervated for fine sensation.
• Palmar digital branches – Arise from median nerve, innervate palmar surfaces of lateral fingers.
• Dorsal digital branches – Arise from radial and ulnar nerves, supply dorsum of fingers.
• Clinical surface anatomy – Nail bed sensation is a key test in digital nerve injuries.
• Carpal tunnel – Narrow passage in wrist transmitting median nerve and tendons, site of compression.
• Sensory testing – Performed with pinprick or light touch to assess nerve integrity in trauma.
• Hand dominance – Important in recovery and surgical repair of nerve injuries.
Chapter: Anatomy / Upper Limb
Topic: Hand
Subtopic: Nerve supply of digits and nail bed
Lead Question – 2013
The nerve supply of nail bed of index finger is?
a) Superficial br of radial nerve
b) Deep br of radial nerve
c) Median nerve
d) Ulnar nerve
Explanation: The nail bed of index finger is supplied by the palmar digital branches of the median nerve. The superficial branch of radial nerve supplies dorsum of hand but not the nail bed of index. Correct answer: (c) Median nerve. Clinical: Sensory loss here suggests median nerve injury.
Guessed Questions for NEET PG
1) Sensory loss over nail bed of middle finger indicates injury to?
a) Ulnar nerve
b) Median nerve
c) Radial nerve
d) Musculocutaneous nerve
Explanation: The nail bed of middle finger, like index, is innervated by the median nerve. Injury to the nerve proximal to wrist causes loss of sensation here. Correct answer: Median nerve. Clinical: Important in diagnosing carpal tunnel syndrome.
2) Which nerve supplies nail bed of little finger?
a) Radial nerve
b) Median nerve
c) Ulnar nerve
d) Musculocutaneous nerve
Explanation: The little finger is supplied by the palmar digital branches of the ulnar nerve. Correct answer: Ulnar nerve. Clinical: Injury to ulnar nerve in Guyon’s canal affects sensation of little finger.
3) Which nerve injury is suspected when thumb, index, and middle finger nail beds lose sensation?
a) Radial nerve
b) Median nerve
c) Ulnar nerve
d) Axillary nerve
Explanation: Loss of sensation in thumb, index, and middle finger nail beds indicates median nerve injury. Correct answer: Median nerve. Clinical: Often seen in carpal tunnel syndrome and supracondylar fractures.
4) The superficial branch of radial nerve supplies?
a) Palmar surface of index finger
b) Dorsum of thumb
c) Nail bed of index finger
d) Palmaris brevis muscle
Explanation: The superficial branch of radial nerve supplies dorsum of thumb and hand but does not reach nail beds of index finger. Correct answer: Dorsum of thumb. Clinical: Injury leads to sensory deficit in dorsum of hand.
5) Injury at wrist producing loss of sensation in nail bed of ring finger (lateral half) indicates?
a) Ulnar nerve
b) Median nerve
c) Radial nerve
d) Posterior interosseous nerve
Explanation: The lateral half of ring finger is supplied by median nerve digital branches. Correct answer: Median nerve. Clinical: Important in mixed finger innervation assessment.
6) Which nerve supplies motor innervation to thenar muscles along with nail bed sensation of index finger?
a) Ulnar nerve
b) Median nerve
c) Radial nerve
d) Axillary nerve
Explanation: The median nerve supplies both thenar muscles (except adductor pollicis and deep head of FPB) and sensation of index finger nail bed. Correct answer: Median nerve. Clinical: Injury causes thenar atrophy and sensory loss.
7) A patient with carpal tunnel syndrome will typically complain of numbness over?
a) Little finger nail bed
b) Index and middle finger nail beds
c) Medial palm
d) Dorsum of hand
Explanation: Carpal tunnel compression of median nerve affects sensation over nail beds of thumb, index, middle, and radial half of ring finger. Correct answer: Index and middle finger nail beds. Clinical: Classic diagnostic sign.
8) Which nerve injury is tested by checking sensation at tip of little finger?
a) Median nerve
b) Radial nerve
c) Ulnar nerve
d) Axillary nerve
Explanation: Sensation at tip of little finger is supplied by ulnar nerve digital branches. Correct answer: Ulnar nerve. Clinical: Simple bedside test to isolate ulnar nerve damage.
9) Which nerve provides sensory supply to dorsum of index finger proximal phalanx?
a) Ulnar nerve
b) Median nerve
c) Superficial radial nerve
d) Musculocutaneous nerve
Explanation: The dorsum of proximal index finger is supplied by superficial branch of radial nerve. Correct answer: Superficial radial nerve. Clinical: Differentiate radial vs median injury.
10) Following supracondylar fracture, patient develops loss of sensation in nail bed of index finger. Which nerve is likely injured?
a) Median nerve
b) Ulnar nerve
c) Radial nerve
d) Musculocutaneous nerve
Explanation: Supracondylar fracture often injures median nerve, causing sensory loss in index nail bed. Correct answer: Median nerve. Clinical: Needs urgent assessment due to risk of Volkmann’s ischemic contracture.
Keyword Definitions
• Ulnar nerve – Terminal branch of medial cord of brachial plexus, supplies intrinsic hand muscles and some forearm flexors.
• Flexor carpi ulnaris (FCU) – Forearm muscle, flexes and adducts wrist, supplied by ulnar nerve.
• Flexor digitorum profundus (FDP) – Deep flexor of fingers, medial half supplied by ulnar nerve, lateral half by median nerve.
• Forearm flexors – Muscles anterior to radius/ulna, flex wrist and fingers.
• Medial cord – Branch of brachial plexus giving rise to ulnar nerve.
• Cubital tunnel – Anatomical passage for ulnar nerve at elbow, common entrapment site.
• Claw hand – Deformity caused by ulnar nerve injury, hyperextension at MCP, flexion at IP joints.
• Guyon’s canal – Ulnar nerve compression site at wrist.
• Sensory supply – Ulnar nerve supplies medial 1½ fingers and corresponding palm/dorsum.
• Motor supply – Ulnar nerve supplies FCU, medial FDP, and most intrinsic hand muscles.
• Clinical localization – Identifying site of nerve injury based on selective muscle/sensory involvement.
Chapter: Anatomy / Upper Limb
Topic: Brachial Plexus
Subtopic: Ulnar Nerve in Arm and Forearm
Lead Question – 2013
In arm ulnar nerve gives muscular branch to which muscle ?
a) FCU
b) FDP
c) Both
d) None
Explanation: In the arm, the ulnar nerve does not supply any muscle. It simply travels down medially without branches. FCU and FDP receive branches in the forearm, not arm. Correct answer: None. Clinical: important in localizing lesions since proximal arm injuries do not affect muscle action directly.
Guessed Questions for NEET PG
1) Which muscle is supplied by ulnar nerve in the forearm?
a) Pronator teres
b) Flexor carpi ulnaris
c) Flexor pollicis longus
d) Palmaris longus
Explanation: Ulnar nerve supplies FCU and medial half of FDP in the forearm. Flexor carpi ulnaris is a key muscle supplied by it. Correct answer: Flexor carpi ulnaris. Clinical: tested by resisted wrist flexion and adduction.
2) A patient with ulnar nerve lesion at elbow will have weakness of:
a) Pronation
b) Wrist flexion and adduction
c) Wrist extension
d) Supination
Explanation: Elbow lesion of ulnar nerve affects FCU and medial FDP. This weakens wrist flexion/adduction and finger flexion. Correct answer: Wrist flexion and adduction. Clinical: combined with sensory loss over medial hand.
3) Which deformity is caused by distal ulnar nerve lesion at wrist?
a) Wrist drop
b) Claw hand
c) Ape thumb
d) Benediction sign
Explanation: Distal ulnar nerve lesion causes paralysis of lumbricals/interossei leading to claw hand deformity. Correct answer: Claw hand. Clinical: more severe when lesion is distal because FDP is spared.
4) Which nerve is compressed in Guyon’s canal syndrome?
a) Radial
b) Median
c) Ulnar
d) Musculocutaneous
Explanation: Ulnar nerve passes through Guyon’s canal near wrist. Compression here produces sensory and motor loss in ulnar distribution without affecting forearm muscles. Correct answer: Ulnar nerve. Clinical: common in cyclists ("handlebar palsy").
5) Which intrinsic hand muscle is not supplied by ulnar nerve?
a) Adductor pollicis
b) First dorsal interosseous
c) Lateral two lumbricals
d) Palmar interossei
Explanation: Median nerve supplies lateral two lumbricals and thenar muscles (except adductor pollicis). Ulnar supplies all others. Correct answer: Lateral two lumbricals. Clinical: important for fine finger movements.
6) Injury to ulnar nerve at elbow spares which muscle?
a) FCU
b) FDP (medial half)
c) FDP (lateral half)
d) Palmar interossei
Explanation: Lateral half of FDP is supplied by median nerve, not ulnar nerve. Hence spared in elbow lesion. Correct answer: FDP (lateral half). Clinical: helps differentiate median vs ulnar nerve contributions.
7) Sensory supply of ulnar nerve includes:
a) Lateral 3½ fingers
b) Medial 1½ fingers
c) Entire palm
d) Thenar eminence
Explanation: Ulnar nerve supplies skin of medial 1½ fingers and adjacent palm/dorsum. Correct answer: Medial 1½ fingers. Clinical: loss of sensation here is diagnostic.
8) Froment’s sign is positive in lesion of:
a) Radial nerve
b) Median nerve
c) Ulnar nerve
d) Axillary nerve
Explanation: Froment’s sign indicates weakness of adductor pollicis supplied by ulnar nerve. Thumb flexion occurs due to compensation by flexor pollicis longus. Correct answer: Ulnar nerve. Clinical: classic bedside test.
9) A patient with difficulty in finger abduction most likely has injury to:
a) Median nerve
b) Ulnar nerve
c) Radial nerve
d) Musculocutaneous nerve
Explanation: Interossei supplied by ulnar nerve abduct/adduct fingers. Injury impairs abduction. Correct answer: Ulnar nerve. Clinical: seen in claw hand cases.
10) Which test checks for integrity of ulnar nerve?
a) Phalen’s test
b) Card test
c) Tinel’s sign
d) O’Brien’s test
Explanation: Card test involves holding paper between fingers using interossei. Ulnar nerve lesion causes inability to hold paper. Correct answer: Card test. Clinical: simple bedside diagnostic tool.
Keyword Definitions
• Radial nerve – Largest branch of brachial plexus, supplies posterior compartment of arm and forearm.
• Spiral groove – Groove on humerus where radial nerve travels, common injury site in fractures.
• Triceps brachii – Muscle with three heads (long, lateral, medial), main extensor of elbow.
• Extensor carpi radialis longus (ECRL) – Radial nerve branch above spiral groove, extends wrist.
• Wrist drop – Inability to extend wrist due to radial nerve lesion.
• Posterior interosseous nerve – Deep radial branch, supplies finger extensors.
• Saturday night palsy – Radial nerve compression in axilla leading to wrist drop.
• Humeral shaft fracture – Common cause of radial nerve injury at spiral groove.
• Supinator canal – Site of posterior interosseous nerve entrapment.
• Clinical localization – Identifying nerve injury by selective motor/sensory loss.
• Dorsal web space – Sensory area supplied by superficial radial nerve.
Chapter: Anatomy / Upper Limb
Topic: Brachial Plexus
Subtopic: Radial Nerve at Spiral Groove
Lead Question – 2013
Which muscle will be paralyzed when radial nerve is injured in just below the spiral groove ?
a) Lateral head of triceps
b) Medial head of triceps
c) Long head of triceps
d) ECRL
Explanation: Radial nerve gives branches to triceps before entering spiral groove, sparing it in distal lesions. ECRL is also supplied above the groove. Injury just below spiral groove paralyzes medial head of triceps. Correct answer: Medial head of triceps. Clinical: elbow extension weak but not absent.
Guessed Questions for NEET PG
1) In a mid-shaft humeral fracture, which motor deficit is most expected?
a) Loss of elbow extension
b) Wrist drop
c) Loss of pronation
d) Finger flexion weakness
Explanation: Radial nerve in spiral groove is injured, sparing triceps but affecting wrist/finger extensors. This causes wrist drop while elbow extension remains intact. Correct answer: Wrist drop. Clinical: selective extensor weakness helps localize lesion.
2) A patient with radial nerve injury in axilla will present with:
a) Wrist drop only
b) Elbow and wrist extension loss
c) Only sensory loss
d) Finger abduction loss
Explanation: Axillary radial nerve lesion affects triceps, wrist extensors, and sensory branches. This produces loss of elbow and wrist extension with sensory loss. Correct answer: Elbow and wrist extension loss. Clinical: classic in crutch palsy.
3) Which muscle is spared in radial nerve injury at spiral groove?
a) Extensor digitorum
b) Extensor carpi radialis longus
c) Extensor pollicis longus
d) Extensor indicis
Explanation: ECRL is supplied before spiral groove. Thus, wrist extension is weak but not lost. Correct answer: Extensor carpi radialis longus. Clinical: partial wrist drop seen instead of complete.
4) Loss of thumb extension is seen in injury to:
a) Median nerve
b) Ulnar nerve
c) Posterior interosseous nerve
d) Musculocutaneous nerve
Explanation: Posterior interosseous nerve supplies extensor pollicis longus and brevis. Its injury causes inability to extend thumb. Correct answer: Posterior interosseous nerve. Clinical: selective thumb drop without wrist involvement.
5) A patient cannot adduct fingers but wrist extension is normal. Likely nerve injured?
a) Radial
b) Median
c) Ulnar
d) Axillary
Explanation: Ulnar nerve supplies interossei for finger adduction. Radial nerve intact preserves wrist extension. Correct answer: Ulnar nerve. Clinical: card test positive.
6) Injury to radial nerve just above wrist affects:
a) Motor only
b) Sensory only
c) Both motor and sensory
d) Neither
Explanation: At wrist, radial nerve is superficial and purely sensory. Injury here causes sensory loss in dorsum of first web space. Correct answer: Sensory only. Clinical: no motor deficit seen.
7) Which nerve is tested by sensation over dorsal first web space?
a) Median
b) Ulnar
c) Radial
d) Musculocutaneous
Explanation: Radial nerve superficial branch supplies skin of first web space dorsally. Correct answer: Radial nerve. Clinical: useful in localizing high radial lesions.
8) Which nerve injury produces “claw hand”?
a) Median
b) Ulnar
c) Radial
d) Axillary
Explanation: Ulnar nerve injury at wrist causes paralysis of medial lumbricals leading to claw hand deformity. Correct answer: Ulnar nerve. Clinical: worsens with distal lesions.
9) Inability to supinate forearm after fracture of proximal radius is due to injury of:
a) Median
b) Radial
c) Musculocutaneous
d) Ulnar
Explanation: Supinator is innervated by posterior interosseous nerve, branch of radial. Injury near proximal radius affects supination. Correct answer: Radial nerve. Clinical: partial supination possible via biceps if intact.
10) A patient develops wrist drop after sleeping with arm compressed over chair. This condition is called:
a) Crutch palsy
b) Saturday night palsy
c) Honeymoon palsy
d) Arcade syndrome
Explanation: Compression of radial nerve in axilla during deep sleep causes Saturday night palsy. Correct answer: Saturday night palsy. Clinical: wrist drop with sensory loss over dorsum hand.
Keyword Definitions
• Radial nerve – Largest branch of brachial plexus, supplies extensors of arm and forearm.
• Posterior interosseous nerve – Deep terminal branch of radial nerve, supplies finger extensors.
• Median nerve – Supplies most anterior forearm muscles and thenar muscles.
• Ulnar nerve – Supplies intrinsic hand muscles and medial forearm muscles.
• Wrist drop – Inability to extend wrist due to radial nerve injury.
• Spiral groove – Location of radial nerve on humerus, commonly injured in fractures.
• Crutch palsy – Radial nerve injury due to axillary compression from crutches.
• Dorsal digital branch – Radial nerve branch supplying skin over first web space.
• Clinical localization – Identifying nerve injuries based on motor and sensory deficits.
• Supinator canal – Site where posterior interosseous nerve may be compressed.
• Extensor compartment – Posterior muscles of forearm responsible for finger/wrist extension.
Chapter: Anatomy / Upper Limb
Topic: Brachial Plexus
Subtopic: Posterior Interosseous Nerve
Lead Question – 2013
A person had injury to right upper limb he is not able to extend fingers but able to extend wrist and elbow. Nerve injured is ?
a) Radial
b) Median
c) Ulnar
d) Posterior interosseous
Explanation: Finger extension is controlled by posterior interosseous nerve, a branch of radial nerve. Wrist and elbow extension are preserved because proximal radial nerve branches are intact. Correct answer: Posterior interosseous nerve. Clinical: injury produces finger drop without wrist drop.
Guessed Questions for NEET PG
1) Which nerve is injured in humeral shaft fracture leading to wrist drop?
a) Median
b) Radial
c) Ulnar
d) Axillary
Explanation: Radial nerve travels in the spiral groove of humerus and is vulnerable in shaft fractures. Injury leads to wrist drop and sensory loss in dorsum of hand. Correct answer: Radial nerve. Clinical: triceps often spared due to proximal innervation.
2) A patient with wrist extension preserved but inability to extend thumb likely has lesion of:
a) Median nerve
b) Radial nerve
c) Posterior interosseous nerve
d) Ulnar nerve
Explanation: Posterior interosseous nerve specifically supplies thumb extensors. Wrist extension is preserved via intact extensor carpi radialis longus. Correct answer: Posterior interosseous nerve. Clinical: selective finger drop is hallmark.
3) Inability to oppose thumb is due to injury of:
a) Ulnar nerve
b) Median nerve
c) Radial nerve
d) Musculocutaneous nerve
Explanation: Median nerve supplies thenar muscles including opponens pollicis. Its injury prevents thumb opposition. Correct answer: Median nerve. Clinical: seen in carpal tunnel syndrome or wrist lacerations.
4) Which nerve is injured in “Saturday night palsy”?
a) Radial
b) Ulnar
c) Median
d) Axillary
Explanation: Prolonged pressure in axilla compresses radial nerve, leading to wrist drop. Correct answer: Radial nerve. Clinical: common in unconscious patients with arm hanging over chair.
5) Loss of sensation in first dorsal web space occurs in:
a) Median nerve lesion
b) Ulnar nerve lesion
c) Radial nerve lesion
d) Axillary nerve lesion
Explanation: The radial nerve supplies skin over dorsum of first web space. Correct answer: Radial nerve. Clinical: this sensory loss confirms radial nerve lesion.
6) Which nerve passes through supinator canal and may be compressed there?
a) Ulnar
b) Median
c) Posterior interosseous
d) Axillary
Explanation: Posterior interosseous nerve passes through supinator canal (Arcade of Frohse) where entrapment can occur. Correct answer: Posterior interosseous nerve. Clinical: presents with finger drop but preserved wrist extension.
7) A patient cannot extend elbow. The nerve involved is:
a) Median
b) Radial
c) Ulnar
d) Musculocutaneous
Explanation: Radial nerve supplies triceps brachii responsible for elbow extension. Injury proximal to triceps branches leads to loss of elbow extension. Correct answer: Radial nerve.
8) Inability to adduct fingers is due to lesion of:
a) Median nerve
b) Ulnar nerve
c) Radial nerve
d) Axillary nerve
Explanation: Ulnar nerve supplies interossei muscles responsible for finger adduction and abduction. Correct answer: Ulnar nerve. Clinical: test by asking patient to hold a card between fingers (card test).
9) Which nerve is commonly injured during axillary lymph node dissection?
a) Long thoracic
b) Radial
c) Axillary
d) Median
Explanation: Long thoracic and thoracodorsal nerves are at risk, but not radial. Correct answer: Long thoracic nerve. Clinical: its injury leads to winging of scapula due to serratus anterior paralysis.
10) A patient with inability to pronate forearm likely has injury of:
a) Radial nerve
b) Median nerve
c) Ulnar nerve
d) Axillary nerve
Explanation: Median nerve supplies pronator teres and pronator quadratus. Injury causes loss of pronation. Correct answer: Median nerve. Clinical: pronation deficit helps localize lesion.
Keyword Definitions
• Brachial plexus – Network of nerves formed by anterior rami of C5–T1 supplying upper limb.
• Radial nerve – Largest branch of brachial plexus, supplies extensor compartment of arm and forearm.
• Ulnar nerve – Arises from medial cord, supplies intrinsic hand muscles and medial forearm.
• Median nerve – Formed by medial and lateral cords, supplies anterior forearm and thenar muscles.
• Axillary nerve – Branch of posterior cord, supplies deltoid and teres minor.
• Posterior cord – Formed by posterior divisions of all trunks of brachial plexus.
• Musculocutaneous nerve – Arises from lateral cord, supplies anterior arm muscles.
• Clinical correlation – Radial nerve injury leads to wrist drop and sensory loss in dorsum of hand.
• Surgical relevance – Axillary dissection may endanger nerves such as thoracodorsal and long thoracic.
• Root value – Spinal segmental origin of a peripheral nerve, important in localization of lesions.
• Extensor compartment – Posterior arm and forearm muscles controlled by radial nerve.
Chapter: Anatomy / Upper Limb
Topic: Brachial Plexus
Subtopic: Radial Nerve
Lead Question – 2013
Largest branch of brachial plexus is
a) Ulnar
b) Median
c) Radial
d) Axillary
Explanation: The radial nerve is the largest branch of the brachial plexus. It arises from the posterior cord (C5–T1) and supplies the extensor compartments of the arm and forearm. Correct answer: Radial nerve. Clinically, its injury causes wrist drop and weak hand grip due to loss of extensors.
Guessed Questions for NEET PG
1) Root value of radial nerve is:
a) C5–C6
b) C5–T1
c) C7–T1
d) C6–C8
Explanation: Radial nerve is derived from the posterior cord of the brachial plexus with root values C5–T1. Correct answer: C5–T1. Clinically, knowledge of root value helps in diagnosing radiculopathies presenting with upper limb weakness.
2) Nerve injured in mid-shaft fracture of humerus:
a) Median
b) Ulnar
c) Radial
d) Axillary
Explanation: Radial nerve runs in the spiral groove of the humerus and is commonly injured in mid-shaft fractures. Correct answer: Radial nerve. This results in wrist drop due to paralysis of wrist extensors.
3) Which nerve supplies triceps brachii?
a) Axillary
b) Radial
c) Median
d) Musculocutaneous
Explanation: Triceps brachii, the main extensor of the elbow, is innervated by the radial nerve. Correct answer: Radial nerve. Clinical: injury above triceps branches causes loss of elbow extension along with wrist drop.
4) Nerve supply of supinator muscle is:
a) Median
b) Radial (deep branch)
c) Ulnar
d) Musculocutaneous
Explanation: The deep branch of the radial nerve, also called the posterior interosseous nerve, innervates the supinator muscle. Correct answer: Radial (deep branch). Clinical: weakness in supination if injured.
5) Wrist drop is due to injury of:
a) Median nerve
b) Ulnar nerve
c) Radial nerve
d) Musculocutaneous nerve
Explanation: Wrist drop occurs due to loss of extensor muscles supplied by the radial nerve. Correct answer: Radial nerve. Clinical: seen in humeral shaft fractures or compressive neuropathy ("Saturday night palsy").
6) Which nerve is closely related to spiral groove of humerus?
a) Axillary
b) Median
c) Radial
d) Ulnar
Explanation: The radial nerve courses through the spiral groove of the humerus. Correct answer: Radial nerve. Clinical: susceptible to injury in humeral shaft fractures leading to sensory and motor deficits.
7) Posterior interosseous nerve is a branch of:
a) Median
b) Ulnar
c) Radial
d) Axillary
Explanation: Posterior interosseous nerve is the terminal deep branch of radial nerve after passing through supinator. Correct answer: Radial nerve. Clinical: supplies most extensor muscles of forearm.
8) Which nerve supplies skin over dorsum of first web space?
a) Median
b) Ulnar
c) Radial
d) Musculocutaneous
Explanation: The dorsal digital branch of radial nerve supplies skin of first web space between thumb and index finger. Correct answer: Radial nerve. Clinical: sensory loss here confirms radial nerve lesion.
9) Nerve injured in improper use of crutches (“crutch palsy”):
a) Median
b) Radial
c) Ulnar
d) Axillary
Explanation: Radial nerve injury occurs in axilla due to compression from crutches or prolonged pressure. Correct answer: Radial nerve. Clinical: causes wrist drop and weakness of grip.
10) Which nerve is tested by extension of wrist against resistance?
a) Median
b) Ulnar
c) Radial
d) Axillary
Explanation: Radial nerve integrity is tested by checking wrist extension against resistance. Correct answer: Radial nerve. Clinical: inability indicates lesion of radial nerve or its branches.
Keyword Definitions
• Thoracodorsal nerve – Branch of posterior cord, supplies latissimus dorsi.
• Root value – Spinal nerves contributing fibers to a peripheral nerve.
• Brachial plexus – Formed by ventral rami of C5–T1, supplies upper limb.
• Latissimus dorsi – Muscle aiding extension, adduction, and medial rotation of humerus.
• Posterior cord – Division of brachial plexus formed by posterior divisions of all trunks.
• Axillary nerve – Terminal branch of posterior cord, innervates deltoid and teres minor.
• Long thoracic nerve – Arises from C5–C7 roots, supplies serratus anterior.
• Suprascapular nerve – Arises from upper trunk, supplies supraspinatus and infraspinatus.
• Clinical correlation – Injury to thoracodorsal nerve impairs arm adduction, weakens shoulder extension.
• Surgical relevance – Preserved during axillary clearance to maintain latissimus dorsi flap viability.
Chapter: Anatomy / Upper Limb
Topic: Brachial Plexus
Subtopic: Thoracodorsal Nerve
Lead Question – 2013
Root value of thoracodorsal nerve?
a) C5, C6, C7
b) C8, T1
c) C6, C7, C8
d) T1, T2
Explanation: Thoracodorsal nerve arises from the posterior cord of brachial plexus with root value C6, C7, and C8. It innervates latissimus dorsi, which is important in climbing and swimming. Correct answer: C6, C7, C8. Clinically preserved during axillary dissections to prevent functional loss of shoulder movements.
Guessed Questions for NEET PG
1) Nerve supply of latissimus dorsi is:
a) Thoracodorsal nerve
b) Dorsal scapular nerve
c) Axillary nerve
d) Long thoracic nerve
Explanation: Latissimus dorsi is supplied by the thoracodorsal nerve (C6–C8). Correct answer: Thoracodorsal nerve. Clinical: important for forceful adduction and extension of the arm, also preserved in reconstructive flap surgeries.
2) Root value of long thoracic nerve is:
a) C5–C7
b) C7–C9
c) C8–T1
d) C5–C6
Explanation: Long thoracic nerve arises from C5, C6, and C7 roots and supplies serratus anterior. Correct answer: C5–C7. Clinical: injury produces winged scapula due to paralysis of serratus anterior.
3) Which nerve is closely related to axillary lymph node dissection?
a) Thoracodorsal nerve
b) Median nerve
c) Ulnar nerve
d) Radial nerve
Explanation: Thoracodorsal nerve lies in the axilla and is at risk during axillary dissection. Correct answer: Thoracodorsal nerve. Clinical: its injury leads to weakness in shoulder extension and loss of latissimus dorsi flap viability.
4) Nerve injured in surgical neck fracture of humerus:
a) Axillary nerve
b) Radial nerve
c) Median nerve
d) Thoracodorsal nerve
Explanation: Axillary nerve winds around the surgical neck of humerus with posterior circumflex humeral artery. Correct answer: Axillary nerve. Clinical: injury causes inability to abduct shoulder beyond 15° and loss of sensation over deltoid patch.
5) Function of latissimus dorsi is:
a) Flexion and lateral rotation
b) Extension, adduction, medial rotation
c) Abduction and supination
d) Flexion and pronation
Explanation: Latissimus dorsi is a powerful extensor, adductor, and medial rotator of the arm. Correct answer: Extension, adduction, medial rotation. Clinical: active in climbing and swimming movements.
6) Root value of axillary nerve:
a) C5–C6
b) C7–C8
c) C8–T1
d) C5–C7
Explanation: Axillary nerve arises from posterior cord with root value C5 and C6. Correct answer: C5–C6. Clinical: injured in humeral fractures causing deltoid paralysis and sensory loss on upper arm.
7) Which nerve supplies serratus anterior?
a) Long thoracic nerve
b) Thoracodorsal nerve
c) Dorsal scapular nerve
d) Axillary nerve
Explanation: Serratus anterior is innervated by the long thoracic nerve (C5–C7). Correct answer: Long thoracic nerve. Clinical: injury produces winged scapula, especially during axillary lymph node dissection.
8) Root value of suprascapular nerve:
a) C5–C6
b) C6–C7
c) C8–T1
d) C5–C7
Explanation: Suprascapular nerve arises from the upper trunk of brachial plexus with root value C5–C6. Correct answer: C5–C6. Clinical: supplies supraspinatus and infraspinatus, important in shoulder abduction and external rotation.
9) Which nerve injury causes wrist drop?
a) Ulnar nerve
b) Median nerve
c) Radial nerve
d) Thoracodorsal nerve
Explanation: Radial nerve injury leads to paralysis of wrist extensors causing wrist drop. Correct answer: Radial nerve. Clinical: commonly seen in mid-shaft humerus fractures.
10) Klumpke’s palsy involves injury to:
a) Upper trunk (C5–C6)
b) Lower trunk (C8–T1)
c) Posterior cord
d) Medial cord
Explanation: Klumpke’s palsy occurs due to lower trunk (C8–T1) injury. Correct answer: Lower trunk (C8–T1). Clinical: causes claw hand deformity and weakness of intrinsic hand muscles.
Keyword Definitions
• Brachial plexus – A network of nerves formed by ventral rami of C5–T1, supplying upper limb.
• Infraclavicular branches – Nerves arising below clavicle from cords of brachial plexus.
• Supraclavicular branches – Nerves arising above clavicle, mainly from roots and trunks.
• Ulnar nerve – Terminal branch of medial cord; motor to intrinsic hand muscles and sensory to medial hand.
• Long thoracic nerve – Supraclavicular branch from roots (C5–C7); supplies serratus anterior.
• Axillary nerve – Terminal branch of posterior cord; supplies deltoid and teres minor.
• Thoracodorsal nerve – Branch of posterior cord; supplies latissimus dorsi.
• Cords of brachial plexus – Named medial, lateral, posterior according to relation with axillary artery.
• Clinical correlation – Injury to long thoracic nerve causes winging of scapula.
• Fascial compartments – Axilla contains cords of plexus, vessels, and lymph nodes surrounded by sheath.
Chapter: Anatomy / Upper Limb
Topic: Brachial Plexus
Subtopic: Infraclavicular vs Supraclavicular Branches
Lead Question – 2013
All are infraclavicular branches of brachial plexus except?
a) Ulnar nerve
b) Long thoracic nerve
c) Axillary nerve
d) Thoracodorsal nerve
Explanation: Infraclavicular branches arise from cords of brachial plexus. Ulnar, axillary, and thoracodorsal nerves are infraclavicular. Long thoracic nerve arises from roots above the clavicle, hence it is supraclavicular. Correct answer: Long thoracic nerve. Clinically, injury to this nerve produces winged scapula due to serratus anterior paralysis.
Guessed Questions for NEET PG
1) Which nerve is injured in winged scapula?
a) Axillary nerve
b) Long thoracic nerve
c) Thoracodorsal nerve
d) Dorsal scapular nerve
Explanation: Winged scapula occurs due to paralysis of serratus anterior muscle from long thoracic nerve injury. Correct answer: Long thoracic nerve. Clinical: commonly injured in axillary dissections or trauma to lateral thoracic wall.
2) Axillary nerve supplies which muscle?
a) Latissimus dorsi
b) Teres major
c) Teres minor
d) Pectoralis minor
Explanation: Axillary nerve innervates deltoid and teres minor. Correct answer: Teres minor. Clinical: Injury causes inability to abduct shoulder beyond 15° and loss of sensation over deltoid patch.
3) Thoracodorsal nerve supplies:
a) Pectoralis major
b) Latissimus dorsi
c) Subscapularis
d) Serratus anterior
Explanation: Thoracodorsal nerve (middle subscapular nerve) arises from posterior cord and supplies latissimus dorsi. Correct answer: Latissimus dorsi. Clinical: important in flap surgeries like latissimus dorsi flap for reconstruction.
4) Ulnar nerve lesion at wrist causes:
a) Wrist drop
b) Claw hand
c) Ape thumb
d) Foot drop
Explanation: Ulnar nerve injury at wrist leads to claw hand due to loss of intrinsic hand muscles. Correct answer: Claw hand. Clinical: common in fractures of hook of hamate or lacerations.
5) Which is a supraclavicular branch of brachial plexus?
a) Musculocutaneous nerve
b) Long thoracic nerve
c) Median nerve
d) Ulnar nerve
Explanation: Long thoracic nerve arises from roots (C5–C7) above the clavicle, making it a supraclavicular branch. Correct answer: Long thoracic nerve. Clinical: vulnerable during axillary lymph node dissection.
6) Which cord gives rise to median nerve?
a) Lateral cord only
b) Medial cord only
c) Both medial and lateral cords
d) Posterior cord
Explanation: Median nerve arises from contributions of both medial and lateral cords. Correct answer: Both medial and lateral cords. Clinical: median nerve lesions cause loss of thumb opposition and ape thumb deformity.
7) Posterior cord of brachial plexus gives rise to:
a) Axillary and radial nerves
b) Ulnar and radial nerves
c) Median and ulnar nerves
d) Musculocutaneous and median nerves
Explanation: Posterior cord terminates as axillary and radial nerves. Correct answer: Axillary and radial nerves. Clinical: injuries affect shoulder abduction and wrist extension respectively.
8) Klumpke’s palsy involves which roots?
a) C5–C6
b) C7
c) C8–T1
d) C5–C7
Explanation: Klumpke’s palsy occurs due to injury to C8–T1 roots, affecting intrinsic hand muscles. Correct answer: C8–T1. Clinical: causes claw hand deformity and sensory loss in medial forearm and hand.
9) Erb’s palsy involves paralysis of:
a) Flexors of forearm
b) Extensors of wrist
c) Abductors and lateral rotators of shoulder
d) Intrinsic muscles of hand
Explanation: Erb’s palsy occurs due to C5–C6 root lesion, affecting deltoid, supraspinatus, infraspinatus, and biceps. Correct answer: Abductors and lateral rotators of shoulder. Clinical: arm hangs medially rotated, extended, pronated (“waiter’s tip”).
10) Which nerve accompanies posterior circumflex humeral artery?
a) Musculocutaneous nerve
b) Radial nerve
c) Axillary nerve
d) Median nerve
Explanation: Axillary nerve travels with posterior circumflex humeral artery through quadrangular space. Correct answer: Axillary nerve. Clinical: injured in surgical neck fractures of humerus.
Chapter: Central Nervous System
Topic: Cerebrospinal Fluid
Subtopic: Properties and Clinical Importance of CSF
Keyword Definitions:
CSF: Clear fluid cushioning brain and spinal cord, circulating within ventricles and subarachnoid space.
Arachnoid Villi: Microscopic projections of arachnoid membrane into venous sinuses, allowing CSF absorption.
Intracranial Pressure: Pressure inside the skull regulated by CSF volume and brain compliance.
CSF pH: Slightly lower (7.33) than plasma (7.40).
Dural Tap: Lumbar puncture procedure for diagnostic/therapeutic collection of CSF.
Lead Question – 2012
Which of the following is NOT TRUE about CSF?
a) Removal of CSF during dural tap causes intense intracranial headache
b) Normally contain no neutrophils
c) Formed by arachnoid villi within the ventricles
d) pH is less than that of plasma
Explanation: The false statement is (c). CSF is formed by the choroid plexus, not arachnoid villi. Arachnoid villi function in absorption into venous circulation. CSF removal during tap causes headache due to traction on meninges. Normal CSF has no neutrophils and a slightly lower pH than plasma. Correct answer: c.
Guessed Question 1
Which structure produces most CSF in adults?
a) Choroid plexus
b) Arachnoid villi
c) Dural sinuses
d) Astrocytes
Explanation: The majority of CSF is secreted by the choroid plexus in lateral, third, and fourth ventricles. Arachnoid villi only absorb it. Production rate is about 500 ml/day. Answer: a.
Guessed Question 2
Which condition is associated with elevated neutrophils in CSF?
a) Viral meningitis
b) Bacterial meningitis
c) Tuberculous meningitis
d) Fungal meningitis
Explanation: Presence of neutrophils in CSF indicates bacterial meningitis. Viral causes lymphocytic predominance, while TB and fungal show mononuclear cells. Thus, neutrophils are highly suggestive of bacterial etiology. Answer: b.
Guessed Question 3
A patient develops severe headache after lumbar puncture. The mechanism is?
a) Rise in CSF pressure
b) Fall in CSF pressure with meningeal traction
c) Dural nerve irritation
d) Arachnoid inflammation
Explanation: Post-lumbar puncture headache results from fall in CSF pressure causing traction on meninges and intracranial structures when upright. It improves on lying down. Answer: b.
Guessed Question 4
Normal CSF pressure in an adult lying on the side is?
a) 30–50 mm H₂O
b) 60–150 mm H₂O
c) 180–250 mm H₂O
d) 300–400 mm H₂O
Explanation: Normal CSF pressure in lateral decubitus position is 60–150 mm H₂O. Above this suggests raised intracranial pressure. Answer: b.
Guessed Question 5
Which of the following is true about CSF circulation?
a) Flows from subarachnoid space to ventricles
b) Produced in arachnoid villi
c) Passes from lateral to third ventricle via foramen of Monro
d) Absorbed in spinal cord
Explanation: CSF flows from lateral ventricles to third ventricle via foramen of Monro. It is produced in choroid plexus and absorbed by arachnoid villi into venous sinuses. Answer: c.
Guessed Question 6
A patient with suspected subarachnoid hemorrhage but normal CT should undergo?
a) EEG
b) Lumbar puncture
c) MRI
d) Skull X-ray
Explanation: In suspected subarachnoid hemorrhage, if CT scan is negative, lumbar puncture to detect xanthochromia in CSF is diagnostic. Answer: b.
Guessed Question 7
CSF glucose is normally?
a) Equal to plasma glucose
b) Two-thirds of plasma glucose
c) Half of plasma glucose
d) Higher than plasma glucose
Explanation: Normal CSF glucose is about two-thirds of plasma glucose. Low glucose levels suggest bacterial, TB, or fungal meningitis. Answer: b.
Guessed Question 8
In tuberculous meningitis, CSF typically shows?
a) High neutrophils, high glucose
b) Lymphocytes, low glucose, high protein
c) Neutrophils, high glucose, low protein
d) Normal findings
Explanation: Tuberculous meningitis is characterized by lymphocytic pleocytosis, low glucose, and high protein. This pattern helps distinguish it from viral or bacterial causes. Answer: b.
Guessed Question 9
Which of the following substances does NOT normally cross the blood-CSF barrier easily?
a) CO₂
b) Oxygen
c) Glucose
d) Plasma proteins
Explanation: The blood-CSF barrier prevents entry of large proteins while allowing gases and glucose. Hence plasma proteins do not cross easily. Answer: d.
Guessed Question 10
Increased opening pressure during lumbar puncture is seen in?
a) Intracranial hypertension
b) Normal pressure hydrocephalus
c) Spinal anesthesia
d) Hypovolemia
Explanation: Raised opening pressure during lumbar puncture is diagnostic of intracranial hypertension. This may result from tumors, hemorrhage, meningitis, or venous sinus thrombosis. Answer: a.
Chapter: Neurology
Topic: Peripheral Nerve Injuries
Subtopic: Types of Nerve Damage
Keyword Definitions:
• Neuropraxia – Temporary conduction block without axonal damage.
• Axonotmesis – Axonal disruption with intact connective tissue sheaths, recovery possible.
• Neurotmesis – Complete nerve transection with poor prognosis.
• Motor march – Sequential return of motor power in regenerating nerves.
• Nerve regeneration – Repair process after injury involving axonal sprouting.
Lead Question - 2012
Motor march is seen in ?
a) Axonotmesis
b) Neurotmesis
c) Neuropraxia
d) Nerve regeneration
Explanation:
Motor march refers to the sequential recovery of muscles supplied by a regenerating nerve, starting from proximal to distal. It is characteristic of nerve regeneration, particularly following axonotmesis. Correct answer: Nerve regeneration.
Guessed Questions for NEET PG
1. In neuropraxia, which of the following is true?
a) Axons are disrupted
b) Myelin sheath conduction block
c) Endoneurium destroyed
d) Recovery not possible
Explanation:
Neuropraxia is the mildest form of nerve injury with myelin sheath block but intact axons. Conduction resumes within weeks. Correct answer: Myelin sheath conduction block.
2. Wallerian degeneration occurs in?
a) Neuropraxia
b) Axonotmesis
c) Myasthenia gravis
d) Multiple sclerosis
Explanation:
When axons are disrupted (axonotmesis or neurotmesis), the distal segment undergoes Wallerian degeneration. It does not occur in neuropraxia. Correct answer: Axonotmesis.
3. Clinical: A patient with complete transection of the radial nerve shows no recovery after months. Likely diagnosis?
a) Neuropraxia
b) Neurotmesis
c) Axonotmesis
d) Neuritis
Explanation:
Neurotmesis is complete nerve transection with disruption of connective tissue sheaths. Spontaneous recovery is poor without surgical repair. Correct answer: Neurotmesis.
4. First sign of nerve regeneration is?
a) Return of sensation
b) Motor march
c) Tinel’s sign
d) Muscle hypertrophy
Explanation:
Tinel’s sign, tingling on percussion over the regenerating nerve, indicates axonal sprouting and is the earliest sign of regeneration. Correct answer: Tinel’s sign.
5. Clinical: A young man with wrist drop after humeral fracture recovers completely in 6 weeks. Likely nerve injury?
a) Neuropraxia
b) Neurotmesis
c) Axonotmesis
d) Axonopathy
Explanation:
Rapid, complete recovery without surgery suggests neuropraxia, a conduction block without axonal disruption. Correct answer: Neuropraxia.
6. Axonotmesis differs from neurotmesis because?
a) Axons preserved
b) Connective tissue sheath preserved
c) No Wallerian degeneration
d) Requires surgical repair
Explanation:
In axonotmesis, axons are destroyed but connective tissue sheaths remain intact, allowing axonal regrowth. Neurotmesis disrupts both. Correct answer: Connective tissue sheath preserved.
7. Rate of peripheral nerve regeneration is approximately?
a) 1 cm/day
b) 1 mm/day
c) 1 mm/week
d) 1 cm/week
Explanation:
Peripheral nerve regeneration typically occurs at a rate of about 1 mm per day, depending on patient age and injury site. Correct answer: 1 mm/day.
8. Clinical: Recovery of biceps before finger flexors after musculocutaneous nerve injury is due to?
a) Proximal-first regeneration
b) Motor march
c) Collateral sprouting
d) Sensory recovery faster
Explanation:
Motor march explains sequential recovery from proximal to distal muscles in nerve regeneration. Correct answer: Motor march.
9. Sunderland classification grade V corresponds to?
a) Neuropraxia
b) Axonotmesis
c) Neurotmesis
d) Demyelination only
Explanation:
Grade V injury in Sunderland classification represents neurotmesis, complete transection of axons and sheaths. Correct answer: Neurotmesis.
10. In nerve regeneration, Schwann cells mainly provide?
a) Structural myelin
b) Pathway for axonal growth
c) Motor endplate repair
d) Sensory transduction
Explanation:
Schwann cells form bands of Büngner, guiding regenerating axons along preserved endoneurial tubes. Correct answer: Pathway for axonal growth.
Keyword Definitions:
Resting Membrane Potential: Electrical potential difference across a cell membrane at rest.
Potassium Equilibrium: Balance between inward and outward K+ movement.
Surface Electrodes: External electrodes that measure global, not intracellular potentials.
Action Potential: Rapid depolarization and repolarization event in excitable tissue.
Sodium-Potassium Pump: Active transport maintaining high intracellular K+ and low Na+.
Lead Question - 2012
Resting membrane potential in nerve fibre
a) Is equal to the potential of ventricular muscle fibre
b) Can be measured by surface electrodes
c) Increases as extracellular K+ increases
d) Depends upon K+ equilibrium
Explanation: Resting membrane potential in nerve fibres is around –70 mV, determined mainly by K+ equilibrium across the membrane. It cannot be measured by surface electrodes, only by microelectrodes. Increased extracellular K+ reduces negativity, not increases it. Correct answer: d) Depends upon K+ equilibrium.
MCQ 2
A patient with hyperkalemia develops reduced resting membrane potential. What mechanism explains this?
a) Increased Na+ conductance
b) Decreased K+ gradient
c) Increased chloride influx
d) Enhanced Na+-K+ ATPase activity
Explanation: Hyperkalemia reduces the concentration gradient for potassium, lowering efflux and reducing negativity of resting potential. Correct answer: b) Decreased K+ gradient.
MCQ 3
Which ion movement contributes most to resting membrane potential?
a) Active calcium transport
b) Sodium influx
c) Potassium efflux
d) Chloride trapping
Explanation: Resting potential is mostly due to passive potassium efflux through leak channels. Na+ and Cl– have smaller roles. Correct answer: c) Potassium efflux.
MCQ 4
What is the approximate value of neuronal resting membrane potential?
a) +30 mV
b) 0 mV
c) –70 mV
d) –120 mV
Explanation: Resting membrane potential in neurons averages –70 mV, reflecting high K+ permeability and Na+/K+ pump activity. Correct answer: c) –70 mV.
MCQ 5
Resting membrane potential is measured using?
a) Surface electrodes
b) Patch clamp or microelectrodes
c) ECG leads
d) EMG surface electrodes
Explanation: Intracellular recording with glass microelectrodes or patch clamp measures true resting potential, not surface electrodes. Correct answer: b) Patch clamp or microelectrodes.
MCQ 6
A patient has hypokalemia. What happens to resting membrane potential?
a) Becomes less negative
b) Becomes more negative
c) No change
d) Oscillates
Explanation: In hypokalemia, extracellular K+ falls, increasing the gradient, making resting potential more negative (hyperpolarized). Correct answer: b) Becomes more negative.
MCQ 7
The Na+/K+ ATPase contributes to resting potential by?
a) Pumping 3 Na+ out and 2 K+ in
b) Pumping 2 Na+ in and 3 K+ out
c) Pumping equal Na+ and K+
d) Passive diffusion of ions
Explanation: The pump actively moves 3 Na+ out for every 2 K+ in, contributing directly to negativity of resting potential. Correct answer: a) Pumping 3 Na+ out and 2 K+ in.
MCQ 8
In ischemia, resting membrane potential decreases. Why?
a) Excess chloride influx
b) Pump failure due to ATP depletion
c) Enhanced potassium efflux
d) Excessive sodium extrusion
Explanation: ATP depletion in ischemia stops Na+/K+ ATPase, causing Na+ retention, K+ loss, and reduced resting potential. Correct answer: b) Pump failure due to ATP depletion.
MCQ 9
A nerve cell with –70 mV resting potential is depolarized to –50 mV. This means?
a) Membrane becomes more negative
b) Membrane becomes less negative
c) No change in excitability
d) Cell is in refractory period
Explanation: Depolarization reduces negativity, moving closer to threshold and increasing excitability. Correct answer: b) Membrane becomes less negative.
MCQ 10
Which disease involves abnormal resting potential due to ion channel defect?
a) Myasthenia gravis
b) Hyperkalemic periodic paralysis
c) Parkinson’s disease
d) Huntington’s disease
Explanation: In hyperkalemic periodic paralysis, Na+ channel mutations alter resting potential stability, causing episodic weakness. Correct answer: b) Hyperkalemic periodic paralysis.
MCQ 11
Resting membrane potential is closest to equilibrium potential of which ion?
a) Sodium
b) Potassium
c) Chloride
d) Calcium
Explanation: Because of high membrane permeability to K+, resting potential is closest to potassium equilibrium potential. Correct answer: b) Potassium.
Keyword Definitions
• Taste transduction: Mechanisms by which chemical stimuli are converted to neural signals.
• ENaC: Epithelial sodium channels mediating salty taste via Na+ influx.
• Gustducin: Taste G-protein involved in sweet, umami, and bitter signalling cascades.
• PKD2L1: Proton-sensitive channel implicated in sour taste transduction.
• Chorda tympani: Facial nerve branch (VII) carrying anterior two-thirds taste.
• Nucleus tractus solitarius (NTS): Medullary gustatory relay receiving cranial nerve afferents.
• VPM nucleus: Thalamic relay for facial/gustatory sensory information to cortex.
• Dysgeusia: Distorted taste perception commonly seen in disease or after drugs.
• Ageusia: Loss of taste sensation.
• Taste cell turnover: Continuous replacement of taste receptor cells from basal progenitors.
Chapter: Neurophysiology
Topic: Gustation (Taste Physiology)
Subtopic: Taste Transduction Mechanisms and Clinical Correlates
Lead Question – 2012
Salty taste is due to?
a) Ca+2 channels
b) Na+ channels
c) G-protein
d) H+ channels
Explanation: Salt taste transduction primarily occurs via epithelial sodium channels on taste receptor cells which allow Na+ influx, depolarizing the cell and triggering neurotransmitter release to gustatory nerves. This receptor mechanism explains perception of salty stimuli. Answer: b) Na+ channels. Clinically, sodium channel blockers reduce salty perception in experimental settings commonly.
Question 2
Sour taste transduction is primarily mediated by?
a) G-protein coupled receptors
b) Voltage-gated Ca2+ channels
c) ENaC channels
d) H+ (proton) channels
Explanation: Sour taste arises from H+ ions entering taste receptor cells through proton-sensitive channels (e.g., PKD2L1 or HCN modulation), causing depolarization and neurotransmitter release to gustatory afferents. This transduction distinguishes acidity in foods and guides ingestion or rejection. Answer: d) H+ channels. Clinically, sour detection may be reduced in zinc deficiency.
Question 3
Bitter taste receptors transduce signals via which mechanism?
a) ENaC-mediated depolarization
b) Ionotropic glutamate receptors
c) G-protein coupled T2R receptors
d) Direct H+ gating
Explanation: Bitter taste perception relies on G-protein–coupled T2R receptors activating gustducin and PLCβ2, increasing intracellular calcium and neurotransmitter release, signaling potential toxins and causing aversive responses. Genetic variability affects sensitivity to bitter compounds clinically. Answer: c) G-protein. Pharmacologic blockade of these pathways blunts bitter detection during drug therapy in some patients.
Question 4
Sweet taste transduction occurs through?
a) ENaC channels
b) Ionotropic receptors only
c) G-protein coupled T1R receptors
d) H+ channels
Explanation: Sweet taste is mediated by heterodimeric T1R2/T1R3 G-protein–coupled receptors activating gustducin and second messenger cascades, increasing intracellular calcium to depolarize taste cells and signal pleasant carbohydrate-rich nutrition. Artificial sweeteners selectively activate these receptors. Answer: c) G-protein. Clinical disorders like diabetes may alter sweet perception via receptor and central processing changes.
Question 5
Umami (savory) taste primarily uses which receptors?
a) ENaC only
b) Ionotropic serotonin receptors
c) T1R1/T1R3 G-protein receptors or mGluRs
d) Voltage-gated Na+ channels
Explanation: Umami taste responds to L-glutamate via T1R1/T1R3 G-protein–coupled receptors or mGluR receptors, enhancing savory flavor perception and signaling protein-rich foods. Monosodium glutamate exemplifies this transduction. Answer: c) G-protein. Clinical taste disturbances in chemotherapy can reduce umami sensitivity, contributing to anorexia and weight loss requiring dietary counseling often for patient recovery.
Question 6
Taste from anterior two-thirds of tongue is carried by?
a) Chorda tympani (facial nerve VII)
b) Glossopharyngeal nerve (IX)
c) Vagus nerve (X)
d) Hypoglossal nerve (XII)
Explanation: Taste from anterior two-thirds of tongue is transmitted by the chorda tympani branch of facial nerve (VII), carrying modality-specific signals to nucleus of solitary tract. Lesions produce ipsilateral ageusia and dysgeusia impacting appetite and nutrition. Answer: a) Chorda tympani (facial nerve). Testing assesses gustatory function clinically and guides management decisions.
Question 7
First central relay for taste afferents is?
a) Ventral posterolateral nucleus (VPL)
b) Nucleus tractus solitarius (NTS) in medulla
c) Insular cortex directly
d) Hypothalamus
Explanation: Primary central relay for gustatory afferents is the nucleus of the solitary tract in the medulla, receiving inputs from facial, glossopharyngeal, and vagus nerves and projecting to thalamus and cortex for taste perception and reflexes like salivation. Answer: b) Nucleus tractus solitarius (NTS). Lesions impair gustatory reflexes and taste perception.
Question 8
Which pharmacologic agent reduces salty taste by blocking ENaC experimentally?
a) Lidocaine
b) Amiloride
c) Ondansetron
d) Scopolamine
Explanation: Epithelial sodium channel (ENaC) blockers such as amiloride reduce perception of salty taste by inhibiting Na+ entry into taste cells, demonstrating pharmacologic modulation of taste transduction. This informs pathophysiology and potential therapies for dysgeusia. Answer: b) Na+ channels. Clinical trials assess amiloride's effect on taste alteration in various disorders today.
Question 9
Which condition commonly causes ipsilateral anterior two-thirds taste loss?
a) Bell's palsy
b) Stroke of occipital lobe
c) Otitis externa only
d) Myasthenia gravis
Explanation: Bell's palsy commonly causes ipsilateral anterior two-thirds taste loss due to chorda tympani fiber involvement within the facial nerve; patients report dysgeusia and ageusia with associated facial weakness. Distinguishing peripheral facial palsy from central lesions guides prognosis and therapy, often including corticosteroids. Answer: a) Bell's palsy improving recovery in many.
Question 10
Approximate turnover time of taste receptor cells is?
a) Several years
b) Ten to fourteen days
c) Months
d) Hours
Explanation: Taste receptor cells have a high turnover, regenerating from basal progenitors approximately every ten to fourteen days, maintaining gustatory sensitivity and repair after injury; disrupted regeneration from chemotherapy or aging contributes to chronic dysgeusia and nutritional problems. Answer: b) ~10 days. Monitoring taste during chemotherapy aids patient counseling and management.
Question 11
Thalamic relay for taste to cortex is which nucleus?
a) VPL nucleus
b) VPM nucleus
c) Lateral geniculate nucleus
d) Medial geniculate nucleus
Explanation: Gustatory signals ascend to the ventral posteromedial nucleus of the thalamus, which relays taste information to the insular and frontal opercular cortex for conscious perception, discrimination, and hedonic valuation, integrating with olfactory input for flavor. Lesions produce contralateral taste deficits. Answer: b) VPM nucleus. Taste testing aids localization of lesions.
Keyword Definitions
• Vomiting centre: Brainstem nuclei coordinating emesis reflex integrating multisource inputs.
• Area postrema: Chemoreceptor trigger zone at floor of fourth ventricle, outside BBB.
• Nucleus tractus solitarius (NTS): Primary visceral sensory nucleus relaying vagal afferents.
• Chemoreceptor trigger zone (CTZ): Detects blood-borne emetic agents and drugs.
• Vestibular nuclei: Brainstem centers mediating motion and balance inputs causing motion sickness.
• Reticular formation: Medullary network housing central pattern generator for vomiting.
• Ondansetron: 5-HT3 receptor antagonist used for chemotherapy and postoperative nausea.
• Scopolamine: Antimuscarinic antiemetic effective for motion sickness via vestibular blockade.
• Apomorphine: Dopamine agonist that stimulates CTZ and induces vomiting pharmacologically.
• Projectile vomiting: Forceful vomiting often from raised intracranial pressure or posterior fossa lesions.
• NK1 antagonist (Aprepitant): Blocks substance P to prevent chemotherapy-induced nausea and vomiting.
Chapter: Neurophysiology
Topic: Brainstem Reflexes
Subtopic: Vomiting Mechanisms and Clinical Correlates
Lead Question – 2012
Vomiting centre is situated in the: (September 2008)
a) Hypothalamus
b) Midbrain
c) Pons
d) Medulla
Explanation: Vomiting is coordinated by a reflex center located in the medulla oblongata, integrating signals from chemoreceptor trigger zone, vestibular system, GI tract, and higher centers. Lesions or irritants trigger emesis via medullary nuclei. Answer: d) Medulla. This clinically explains why brainstem lesions often produce persistent vomiting and autonomic disturbances.
Question 2
Which structure functions as the chemoreceptor trigger zone for emesis?
a) Nucleus ambiguus
b) Nucleus tractus solitarius
c) Area postrema
d) Dorsal motor nucleus of vagus
Explanation: The chemoreceptor trigger zone lies in the area postrema of the dorsal medulla at the floor of the fourth ventricle, outside the blood-brain barrier, detecting blood-borne emetic agents and drugs. It relays to the vomiting center to initiate emesis. Answer: c) Area postrema. This localization explains sensitivity to chemotherapeutic agents.
Question 3
Visceral afferents from the gastrointestinal tract synapse primarily in which nucleus relevant to vomiting?
a) Hypoglossal nucleus
b) Nucleus tractus solitarius
c) Inferior olivary nucleus
d) Dorsal motor nucleus of vagus
Explanation: Visceral afferents from gastrointestinal tract travel via vagus and glossopharyngeal nerves to the nucleus tractus solitarius in the medulla, which integrates sensory input and projects to the vomiting center and reticular formation. This pathway mediates reflex emesis from gastric irritation or inflammation. Answer: b) Nucleus tractus solitarius clinically important centrally.
Question 4
Which antiemetic class blocks serotonin-mediated vagal and CTZ signals effectively?
a) Dopamine antagonists
b) Anticholinergics
c) 5-HT3 receptor antagonists
d) NK1 receptor antagonists
Explanation: 5-HT3 receptor antagonists such as ondansetron block serotonin-mediated stimulation of vagal afferents and the chemoreceptor trigger zone, effectively preventing chemotherapy-induced and postoperative nausea. They act centrally and peripherally with good efficacy and tolerability and are first-line antiemetics in many protocols. Answer: a) Ondansetron. Widely used clinically for nausea control effectively.
Question 5
Motion sickness and vestibular-induced vomiting primarily involve which structures?
a) Area postrema only
b) Vestibular nuclei
c) Cerebellar vermis only
d) Hypothalamus exclusively
Explanation: Vestibular apparatus and vestibular nuclei in brainstem detect motion and send signals to vomiting center and cerebellum; conflicts between visual and vestibular input provoke motion sickness and emesis via connections to nucleus tractus solitarius and area postrema. Antihistamines reduce vestibular input. Answer: b) Vestibular nuclei. Used antiemetics target this pathway.
Question 6
Which drug class is known to directly stimulate the CTZ and produce emesis historically?
a) Anticholinergics
b) Serotonin antagonists
c) Dopamine agonists (e.g., apomorphine)
d) NK1 antagonists
Explanation: Apomorphine, a dopamine agonist, stimulates D2 receptors in the chemoreceptor trigger zone (area postrema), provoking profound emesis. Historically used as an emetic, it demonstrates pharmacologic activation of vomiting circuits. Dopamine antagonists block this reflex therapeutically. Answer: c) Apomorphine. Now replaced by safer antiemetics in teaching.
Question 7
Strong risk factors for postoperative nausea and vomiting include which of the following?
a) Male sex and smoking
b) Female sex and opioid use
c) Elderly age exclusively
d) Short duration surgeries only
Explanation: Risk factors for postoperative nausea and vomiting include female sex, history of motion sickness or prior PONV, nonsmoking status, use of volatile anesthetics or opioids, lengthy surgery, and younger age. Multimodal prophylaxis reduces incidence by targeting multiple pathways. Answer: b) Female sex and opioid exposure are significant risk contributors clinically.
Question 8
Best prophylactic agent for motion sickness is?
a) Ondansetron
b) Metoclopramide
c) Domperidone
d) Scopolamine
Explanation: Anticholinergic scopolamine applied transdermally blocks muscarinic receptors in vestibular nuclei and central vomiting pathways, preventing motion-induced nausea and vomiting. Histamine H1 antagonists like promethazine also help. Side effects include dry mouth, blurred vision, and sedation. Answer: d) Scopolamine. It is recommended prophylactically for susceptible patients before travel or procedures commonly.
Question 9
The central pattern generator coordinating emesis resides in which region?
a) Medullary reticular formation
b) Hypothalamus
c) Midbrain periaqueductal gray
d) Pontine tegmentum exclusively
Explanation: Emesis results from activation of a central pattern generator located in the medullary reticular formation and adjacent reticular nuclei, coordinating respiratory, upper GI, and pharyngeal muscles to produce vomiting. This network receives multisensory inputs including chemoreceptor trigger zone and vestibular signals. Answer: a) Medullary reticular formation critical for protective reflexes.
Question 10
Projectile vomiting without nausea often indicates which pathology?
a) Gastroenteritis
b) Metabolic alkalosis only
c) Posterior fossa lesion compressing medulla
d) Peripheral vestibular neuritis
Explanation: Projectile vomiting without preceding nausea suggests increased intracranial pressure or posterior fossa lesion compressing medullary centers. Vomiting may be forceful and predominantly nocturnal. Early recognition mandates neuroimaging to identify obstructive hydrocephalus or tumor. Answer: c) Posterior fossa lesion causing medullary compression requiring urgent neurosurgical decompression to prevent herniation and death.
Question 11
Which agent is recommended to prevent both acute and delayed chemotherapy-induced emesis when combined with others?
a) Ondansetron alone
b) Aprepitant (NK1 antagonist)
c) Metoclopramide alone
d) Scopolamine patch
Explanation: Aprepitant, an NK1 receptor antagonist, blocks substance P signaling in vomiting pathways, reducing acute and delayed chemotherapy-induced nausea and vomiting when combined with 5-HT3 antagonists and corticosteroids. It improves control of emesis after highly emetogenic chemotherapy. Answer: b) Aprepitant. Recommended in guidelines for high emetic risk regimens to reduce vomiting.
Keyword Definitions
• Spinocerebellar tract: Pathways conveying unconscious proprioception to cerebellum for coordination.
• Dorsal spinocerebellar: Ipsilateral lower limb proprioceptive tract entering via inferior peduncle.
• Ventral spinocerebellar: Tract that double-crosses and conveys integrated movement signals to cerebellum.
• Cuneocerebellar: Upper limb equivalent of dorsal spinocerebellar, via accessory cuneate nucleus.
• Clarke’s column: Nucleus dorsalis (T1–L2) origin of dorsal spinocerebellar fibres.
• Inferior cerebellar peduncle: Major cerebellar input for dorsal spinocerebellar and cuneocerebellar tracts.
• Unconscious proprioception: Automatic sensory feedback used to adjust movement without awareness.
• Dysmetria: Overshoot or undershoot of target during voluntary movement, sign of cerebellar dysfunction.
• Intention tremor: Tremor appearing during voluntary movement, characteristic of cerebellar disease.
• Romberg sign: Sway or fall on eye closure from proprioceptive loss (dorsal column), not cerebellar typically.
• Heel-to-shin: Bedside test for lower limb cerebellar coordination and spinocerebellar function.
Chapter: Neurophysiology
Topic: Cerebellar Systems
Subtopic: Spinocerebellar Tracts and Function
Lead Question – 2012
True about spinocerebellar tract is?
a) Equilibrium
b) Smoothens and coordinates movement
c) Learning induced by change in vestibulo ocular reflex
d) Planning and programming
Explanation: Spinocerebellar tracts carry unconscious proprioceptive information from muscles and joints to the cerebellum, enabling real-time adjustment of ongoing movements and posture. They assist coordination and timing rather than motor planning or voluntary initiation. Therefore they smooth and coordinate movement. Answer: b) Smoothens and coordinates movement for precise limb control continuously.
Question 2
Dorsal spinocerebellar tract originates from which nucleus?
a) Clarke’s column
b) Accessory cuneate nucleus
c) Inferior olivary nucleus
d) Red nucleus
Explanation: The dorsal spinocerebellar tract arises from Clarke’s column (nucleus dorsalis) in spinal segments T1 to L2, conveying ipsilateral proprioceptive information from lower limbs to the cerebellum via the inferior cerebellar peduncle. It does not decussate. Answer: a) Clarke’s column. This tract is essential for unconscious proprioception and limb coordination clinically.
Question 3
A lesion of spinocerebellar tract produces which clinical sign?
a) Ipsilateral limb ataxia
b) Contralateral weakness
c) Loss of vibration sense only
d) Sensory level with aneasthesia
Explanation: Spinocerebellar tract lesions produce ipsilateral limb ataxia because most cerebellar afferents enter the cerebellum without crossing or double-cross, preserving same-side representation. Patients show dysmetria, decomposition of movement, and intention tremor on the affected side. Answer: a) Ipsilateral limb ataxia. Coordination deficits worsen with eyes closed and during rapid alternating movements.
Question 4
Which is true about ventral spinocerebellar tract?
a) It never crosses
b) It conveys conscious proprioception
c) It double-crosses
d) It terminates in thalamus
Explanation: The ventral spinocerebellar tract transmits integrated proprioceptive and interneuronal activity related to ongoing limb movement. It decussates twice: once in the spinal cord and again within the cerebellum, resulting in ipsilateral cerebellar representation ultimately. Answer: c) It double-crosses to reach the cerebellum. This anatomical feature explains localization of cerebellar signs.
Question 5
Primary modality carried by spinocerebellar tracts is?
a) Conscious proprioception
b) Unconscious proprioception
c) Pain and temperature
d) Fine tactile discrimination
Explanation: Spinocerebellar pathways convey unconscious proprioception from muscle spindles and Golgi tendon organs to cerebellar cortex, enabling automatic postural adjustments and gait coordination. They are distinct from dorsal columns that mediate conscious proprioception. Answer: b) Unconscious proprioception. Clinical lesions produce ataxia yet preserve conscious position sense; coordination testing reveals deficits often.
Question 6
Finger-to-nose test primarily assesses which system?
a) Cerebellar coordination including spinocerebellar input
b) Dorsal column conscious proprioception only
c) Spinothalamic tract function
d) Pyramidal tract strength
Explanation: Finger-to-nose test evaluates cerebellar coordination and proprioceptive integration including spinocerebellar inputs. Dysmetria, intention tremor, and decomposition of movement during this test indicate cerebellar dysfunction. It does not differentiate conscious from unconscious proprioception but assesses functional output of cerebellum. Answer: a) True. Clinically helps localize lesion to hemisphere or vermis region.
Question 7
Which hereditary disease affects spinocerebellar tracts prominently?
a) Multiple sclerosis
b) Amyotrophic lateral sclerosis
c) Friedreich ataxia
d) Myasthenia gravis
Explanation: Friedreich ataxia causes degeneration of spinocerebellar tracts, dorsal columns, and corticospinal tracts due to frataxin deficiency. Patients present with progressive gait ataxia, loss of vibration and proprioception, areflexia, and cardiomyopathy. Genetic testing confirms GAA repeat expansion. Answer: c) Friedreich ataxia. Onset usually adolescence; progression causes severe disability needing supportive care.
Question 8
Dorsal spinocerebellar fibres enter cerebellum via which peduncle?
a) Inferior cerebellar peduncle
b) Middle cerebellar peduncle
c) Superior cerebellar peduncle
d) None of the above
Explanation: Dorsal spinocerebellar fibers ascend ipsilaterally and enter cerebellum through the inferior cerebellar peduncle, carrying lower limb proprioceptive information. Ventral spinocerebellar fibers primarily enter via superior peduncle after double crossing. Knowledge of peduncle entry assists lesion localization. Answer: a) Inferior cerebellar peduncle. Distinguishing peduncle entry aids accurate lesion localization clinically rapidly.
Question 9
Romberg sign in spinocerebellar or cerebellar lesions is usually?
a) Positive only with eyes open
b) Negative (does not depend on vision)
c) Positive only with vibration loss
d) Always bilateral sensory level
Explanation: Romberg sign becomes positive when proprioceptive input via dorsal columns is lost, causing increased sway with eye closure. Cerebellar or spinocerebellar lesions produce ataxia independent of visual input, so patients remain unstable with eyes open and closed; Romberg is typically negative. Answer: b) Negative. Clinical testing distinguishes lesion location effectively.
Question 10
Best bedside test for lower limb spinocerebellar function is?
a) Romberg test alone
b) Vibration at toe only
c) Rapid alternating foot movements only
d) Heel-to-shin test
Explanation: Heel-to-shin maneuver tests lower limb coordination and cerebellar integration of proprioceptive input including spinocerebellar feedback. Patients with spinocerebellar tract or cerebellar hemisphere lesions exhibit dysmetria and inability to maintain a smooth, straight movement along the shin. Answer: d) Heel-to-shin test. It detects ipsilateral coordination deficits and helps lateralize lesions accurately.
Question 11
Unconscious proprioception from the upper limb is conveyed by?
a) Dorsal spinocerebellar tract
b) Cuneocerebellar tract
c) Ventral spinothalamic tract
d) Lateral corticospinal tract
Explanation: Cuneocerebellar tract carries unconscious proprioceptive input from upper limbs via accessory cuneate nucleus to the cerebellum through inferior peduncle, analogous to dorsal spinocerebellar tract for lower limbs. Lesions impair ipsilateral upper limb coordination and contribute to ataxia. Answer: b) Cuneocerebellar tract. Clinically causes dysmetria and intention tremor during voluntary tasks.
Keyword Definitions
• Proprioception: Sense of position and movement from muscles and joints.
• Vibration sense: Perception of oscillatory stimuli via large myelinated fibers.
• Dorsal columns: Ascending pathway for fine touch, vibration, and proprioception.
• Medial lemniscus: Brainstem tract formed by decussated dorsal column fibers.
• Fasciculus gracilis: Dorsal column for lower limb and trunk below T6.
• Fasciculus cuneatus: Dorsal column for upper limb and trunk above T6.
• VPL nucleus: Thalamic relay for body somatosensation to cortex.
• Romberg sign: Instability on eye closure indicating proprioceptive loss.
• Brown-Séquard syndrome: Hemisection causing ipsilateral dorsal column loss, contralateral pain loss.
• Tabes dorsalis: Neurosyphilis causing posterior column degeneration and sensory ataxia.
• Syringomyelia: Central canal cavity causing bilateral pain/temperature loss with spared dorsal columns.
• Asterognosis: Inability to recognize objects by touch despite intact basic sensation.
• Large-fiber neuropathy: Peripheral nerve disorder affecting vibration and position sense early.
Chapter: Neurophysiology
Topic: Somatosensory Pathways
Subtopic: Dorsal Column–Medial Lemniscus System
Lead Question – 2012
Loss of proprioception & fine touch ?
a) Anterior spinothalamic tract
b) Lateral spinothalamic tract
c) Dorsal column
d) Corticospinal tract
Explanation: Loss of proprioception, vibration, and fine discriminative touch indicates dorsal column–medial lemniscus pathway dysfunction. Large myelinated fibers ascend ipsilaterally to gracile and cuneate nuclei, then decussate to medial lemniscus and VPL. Answer: c) Dorsal column. Spinothalamic tracts carry pain and temperature; corticospinal mediates voluntary movement, not somatosensory modalities, primarily pathways.
Question 2
A 55-year-old with numb feet, gait unsteadiness, and positive Romberg has impaired vibration at toes. Which pathway is damaged?
a) Dorsal columns
b) Lateral corticospinal tract
c) Spinothalamic tract
d) Vestibulospinal tract
Explanation: Subacute combined degeneration from vitamin B12 deficiency damages large myelinated posterior column fibers, producing sensory ataxia, impaired vibration, and proprioceptive loss in legs. Answer: a) Dorsal columns. Lateral corticospinal damage causes weakness and spasticity; spinothalamic lesions impair pain and temperature; vestibulospinal tracts mediate balance reflexes without conveying discriminative touch signals.
Question 3
After a knife injury causing right T10 hemisection, which deficit occurs on the right below the lesion?
a) Loss of pain and temperature
b) Loss of vibration and proprioception
c) Flaccid paralysis below lesion
d) Bilateral pain loss
Explanation: Brown-Séquard hemisection causes ipsilateral loss of dorsal column modalities below the lesion from uncrossed ascent, and contralateral pain/temperature loss after spinothalamic decussation. Answer: b) Loss of vibration and proprioception. Flaccid paralysis occurs at the level from anterior horn involvement, not below. Bilateral pain loss suggests central cord lesions, not hemisection.
Question 4
Which thalamic nucleus relays body fine touch, vibration, and proprioception to cortex?
a) VPL nucleus
b) VPM nucleus
c) Lateral geniculate nucleus
d) Medial geniculate nucleus
Explanation: The ventral posterolateral thalamic nucleus relays somatosensory information from body—both medial lemniscus and spinothalamic—to the primary somatosensory cortex. Lesions impair discriminative touch, vibration, and proprioception contralaterally. Answer: a) VPL nucleus. VPM handles facial sensation; LGN vision; MGN audition. Precise localization of body sensation depends critically on intact VPL relay neurons.
Question 5
A patient with lightning pains, wide-based gait, and positive Romberg likely has damage to which structure?
a) Dorsal columns
b) Spinocerebellar tracts
c) Ventral horn cells
d) Substantia nigra
Explanation: Tabes dorsalis from neurosyphilis degenerates dorsal columns and roots, causing lightning pains, sensory ataxia, impaired vibration, and positive Romberg sign. Answer: a) Dorsal columns. Spinocerebellar tract disease produces limb ataxia without Romberg positivity; ventral horn disease causes lower motor neuron weakness; substantia nigra degeneration produces Parkinsonism, not sensory ataxia, classically.
Question 6
Fine touch from the right hand ascends initially in which tract?
a) Spinothalamic tract
b) Fasciculus gracilis
c) Fasciculus cuneatus
d) Dorsal spinocerebellar tract
Explanation: Fine touch and proprioceptive signals from the upper limb ascend in the ipsilateral fasciculus cuneatus to synapse in the cuneate nucleus before crossing as internal arcuate fibers. Answer: c) Fasciculus cuneatus. Fasciculus gracilis conveys lower limb input; spinothalamic carries pain and temperature; dorsal spinocerebellar conveys unconscious proprioception, not discriminative touch.
Question 7
A patient cannot identify a key by touch in the left hand, yet basic touch is intact. Likely lesion?
a) Cerebellar hemisphere
b) Postcentral gyrus
c) Precentral gyrus
d) Dorsal horn
Explanation: Inability to recognize objects by touch with intact primary modalities indicates cortical sensory loss—astereognosis—from a contralateral parietal lesion, usually postcentral gyrus (primary somatosensory cortex). Answer: b) Postcentral gyrus. Precentral gyrus is motor; cerebellum coordinates movement but not stereognosis; dorsal columns carry signals, yet cortical interpretation is required for object recognition.
Question 8
Bilateral loss of pain and temperature over shoulders with preserved vibration suggests which tract is spared?
a) Spinothalamic tract
b) Lateral corticospinal tract
c) Anterior horn cells
d) Dorsal columns
Explanation: Syringomyelia damages decussating anterior commissural spinothalamic fibers in the cervical cord, causing bilateral cape-like pain and temperature loss while sparing dorsal column modalities. Answer: d) Dorsal columns are spared. Thus vibration and proprioception remain intact. Lateral corticospinal may be affected later causing weakness; dorsal spinocerebellar mediates unconscious proprioception effectively overall.
Question 9
A medial medullary infarct damaging the medial lemniscus causes which deficit?
a) Ipsilateral pain and temperature loss
b) Loss of contralateral discriminative touch
c) Ipsilateral loss of vibration
d) Bilateral pain loss
Explanation: A medial medullary lesion involving the medial lemniscus produces contralateral loss of fine touch, vibration, and proprioception from body due to disruption of dorsal column fibers. Answer: b) Loss of contralateral discriminative touch. Pain and temperature are carried by spinothalamic tract located laterally; hypoglossal involvement would cause ipsilateral tongue weakness.
Question 10
Which modality is typically earliest impaired in large-fiber diabetic neuropathy?
a) Impaired vibration sense
b) Spasticity
c) Hyperalgesia
d) Nystagmus
Explanation: Large-fiber peripheral neuropathy in diabetes affects vibration and position sense earliest, causing positive Romberg and sensory ataxia. Answer: a) Impaired vibration sense. Pain and temperature rely on small fibers; strength may be preserved; hyperreflexia suggests upper motor neuron disease, not peripheral neuropathy, which typically shows reduced or absent ankle reflexes.
Question 11
Which bedside test best assesses dorsal column proprioception?
a) Graphesthesia on palm
b) Vibration at medial malleolus
c) Great toe position sense
d) Hot/cold discrimination
Explanation: Testing joint position at the great toe assesses conscious proprioception via dorsal columns and medial lemniscus. Eyes are closed to remove visual cues. Answer: c) Great toe position sense. Tuning fork tests vibration, not position; pinprick examines spinothalamic pain; plantar response assesses corticospinal integrity, unrelated to dorsal column proprioceptive function.
Keyword Definitions
• Dopamine: Catecholamine neurotransmitter crucial for motor control, motivation, and reward pathways.
• Nigrostriatal pathway: Dopaminergic pathway projecting from substantia nigra to striatum, vital for movement regulation.
• Serotonin: Neurotransmitter involved in mood, sleep, and appetite regulation.
• Cholinergic neurons: Use acetylcholine, important in learning, memory, and motor circuits.
• Adrenergic neurons: Release norepinephrine, essential for arousal, vigilance, and autonomic functions.
• Substantia nigra: Midbrain nucleus producing dopamine, degeneration causes Parkinsonism.
• Basal ganglia: Group of nuclei modulating movement initiation and suppression.
• Extrapyramidal system: Motor system controlling posture, tone, and coordination.
• Parkinson’s disease: Neurodegenerative disorder due to dopaminergic loss in nigrostriatal pathway.
• Dyskinesia: Involuntary abnormal movements due to neurotransmitter imbalance.
• Levodopa: Dopamine precursor used as therapy in Parkinson’s disease.
Chapter: Neurophysiology
Topic: Basal Ganglia
Subtopic: Nigrostriatal Pathway
Lead Question – 2012
Neurotransmitter involved in nigrostriatal pathway is?
a) Serotonin
b) Dopamine
c) Cholinergic
d) Adrenergic
Explanation: The nigrostriatal pathway is a dopaminergic tract connecting substantia nigra pars compacta with the striatum. It regulates voluntary movement by balancing excitatory and inhibitory signals in basal ganglia circuits. Loss of dopamine here causes Parkinsonism. Answer: b) Dopamine. Other neurotransmitters modulate but dopamine is the principal one involved.
Question 2
Which brain structure degenerates in Parkinson’s disease?
a) Substantia nigra pars compacta
b) Globus pallidus externa
c) Red nucleus
d) Subthalamic nucleus
Explanation: Parkinson’s disease arises from dopaminergic neuronal loss in substantia nigra pars compacta, leading to striatal dopamine deficiency. This impairs basal ganglia modulation, causing bradykinesia, rigidity, and tremor. Answer: a) Substantia nigra pars compacta. Subthalamic nucleus lesions cause hemiballismus, while pallidal and red nucleus lesions show different deficits.
Question 3
Which dopamine receptor subtype facilitates the direct pathway in basal ganglia?
a) D1 receptors
b) D2 receptors
c) D3 receptors
d) D4 receptors
Explanation: D1 receptors in striatum stimulate the direct pathway, enhancing movement by exciting striatal neurons projecting to internal globus pallidus. Dopamine binding here increases activity, disinhibiting thalamus and promoting cortical excitation. Answer: a) D1 receptors. D2 receptors inhibit indirect pathway, while D3 and D4 are extrastriatal predominantly.
Question 4
Which clinical feature is not typical of Parkinson’s disease?
a) Rest tremor
b) Bradykinesia
c) Rigidity
d) Spastic paralysis
Explanation: Parkinson’s disease is characterized by rest tremor, bradykinesia, rigidity, and postural instability. Spastic paralysis occurs in upper motor neuron lesions, not basal ganglia dysfunction. Answer: d) Spastic paralysis. Distinguishing Parkinsonism from pyramidal tract damage clinically relies on absence of spasticity and hyperreflexia, despite motor difficulties and tremors.
Question 5
Which neurotransmitter imbalance causes Huntington’s disease?
a) Loss of GABA and acetylcholine
b) Excess dopamine
c) Loss of dopamine
d) Increased serotonin
Explanation: Huntington’s disease features degeneration of striatal GABAergic and cholinergic neurons, combined with relative dopaminergic overactivity. This imbalance produces choreiform hyperkinetic movements. Answer: a) Loss of GABA and acetylcholine. Dopamine blockade may reduce symptoms, unlike Parkinson’s disease where dopamine replacement is therapeutic and symptomatically beneficial clinically.
Question 6
Which drug increases brain dopamine by crossing blood-brain barrier?
a) Levodopa
b) Dopamine
c) Carbidopa
d) Bromocriptine
Explanation: Dopamine itself cannot cross the blood-brain barrier. Levodopa, its precursor, is converted to dopamine in brain. Carbidopa prevents peripheral breakdown, enhancing central availability. Answer: a) Levodopa. Bromocriptine is a dopamine agonist, while dopamine injection only acts peripherally without improving Parkinson’s motor symptoms effectively within CNS.
Question 7
Which basal ganglia lesion produces hemiballismus?
a) Subthalamic nucleus
b) Putamen
c) Caudate nucleus
d) Globus pallidus interna
Explanation: Hemiballismus, a flinging hyperkinetic movement disorder, occurs due to contralateral subthalamic nucleus lesion. Subthalamus normally excites globus pallidus interna, inhibiting thalamus. Its damage reduces inhibition, causing excessive cortical motor output. Answer: a) Subthalamic nucleus. Other basal ganglia nuclei lesions cause Parkinsonism or chorea, not violent ballistic movements.
Question 8
Which dopaminergic pathway is associated with reward and addiction?
a) Nigrostriatal pathway
b) Mesolimbic pathway
c) Tuberoinfundibular pathway
d) Mesocortical pathway
Explanation: Mesolimbic pathway projects from ventral tegmental area to nucleus accumbens, mediating reward, reinforcement, and addiction. Answer: b) Mesolimbic pathway. Nigrostriatal controls movement, mesocortical regulates cognition and emotion, and tuberoinfundibular inhibits prolactin secretion. Dopamine thus has multiple distinct functional pathways in the central nervous system overall.
Question 9
Blockade of which dopamine pathway leads to drug-induced Parkinsonism?
a) Mesolimbic
b) Mesocortical
c) Nigrostriatal
d) Tuberoinfundibular
Explanation: Antipsychotic drugs blocking D2 receptors in the nigrostriatal pathway cause extrapyramidal symptoms resembling Parkinson’s disease. Answer: c) Nigrostriatal. Mesolimbic blockade improves psychosis, mesocortical blockade causes cognitive dulling, and tuberoinfundibular blockade elevates prolactin. Understanding pathway selectivity helps minimize antipsychotic side effects clinically during patient treatment overall effectively.
Question 10
Stimulation of which dopamine receptor subtype inhibits indirect pathway activity?
a) D1
b) D2
c) D3
d) D5
Explanation: D2 receptors inhibit striatal neurons of the indirect pathway, reducing thalamic suppression and promoting movement. Answer: b) D2. D1 receptors activate direct pathway, D3 and D5 have roles in limbic and cortical areas. Balanced D1/D2 signaling ensures smooth motor control within basal ganglia circuits clinically.
Question 11
Which hypothalamic hormone secretion is inhibited by tuberoinfundibular dopamine pathway?
a) Growth hormone
b) Cortisol
c) Prolactin
d) Thyroxine
Explanation: Dopaminergic neurons of tuberoinfundibular pathway inhibit prolactin release from anterior pituitary lactotrophs. Blockade or damage increases prolactin, causing galactorrhea and infertility. Answer: c) Prolactin. Dopamine agonists treat hyperprolactinemia, while antagonists may induce it. Other hypothalamic hormones are regulated differently by respective hypothalamic releasing factors clinically overall.
Keyword Definitions
• Purkinje fibres: Large inhibitory neurons of cerebellar cortex using GABA as neurotransmitter.
• Deep cerebellar nuclei: Primary output centres of cerebellum receiving inhibitory Purkinje input.
• Climbing fibres: Excitatory inputs from inferior olivary nucleus synapsing on Purkinje cells.
• Mossy fibres: Excitatory afferents from spinal cord and brainstem projecting to granule cells.
• Basket cells: Inhibitory interneurons forming axo-somatic synapses on Purkinje cells.
• Stellate cells: Inhibitory interneurons acting on Purkinje dendrites in molecular layer.
• Spinocerebellar tracts: Convey unconscious proprioceptive information from muscles and joints.
• Granule cells: Excitatory interneurons giving rise to parallel fibres synapsing on Purkinje cells.
• GABA: Gamma-aminobutyric acid, main inhibitory neurotransmitter in CNS.
• Cerebellar cortex: Three-layered structure modulating motor coordination and balance.
• Motor learning: Cerebellar mechanism for adapting and fine-tuning skilled movements.
Chapter: Neurophysiology
Topic: Cerebellum
Subtopic: Purkinje Cell Function
Lead Question – 2012
Purkinje fibres are inhibitory for?
a) Deep cerebellar nuclei
b) Climbing fibre
c) Basket cells
d) Spinocerebellar tracts
Explanation: Purkinje cells are GABAergic neurons that project inhibitory signals to deep cerebellar nuclei, regulating motor output precision. They receive excitatory input from climbing and mossy fibres, while interneurons like basket and stellate cells refine their activity. Answer: a) Deep cerebellar nuclei. This inhibitory control ensures smooth coordination and balance.
Question 2
Which neurotransmitter is released by Purkinje cells?
a) Acetylcholine
b) Dopamine
c) GABA
d) Glutamate
Explanation: Purkinje cells are the sole output of the cerebellar cortex. They are inhibitory neurons releasing gamma-aminobutyric acid (GABA). This neurotransmitter suppresses activity of deep cerebellar nuclei, ensuring controlled modulation of motor output. Answer: c) GABA. Excitatory neurotransmitters like glutamate act via mossy and climbing fibre inputs.
Question 3
A patient has loss of coordination but preserved strength. Which structure is primarily affected?
a) Cerebellum
b) Basal ganglia
c) Motor cortex
d) Spinal cord anterior horn
Explanation: The cerebellum coordinates timing, precision, and smoothness of movement but does not initiate voluntary force generation. Lesions cause ataxia, dysmetria, and intention tremor without significant weakness. Answer: a) Cerebellum. Motor cortex lesions reduce strength, while basal ganglia dysfunction causes rigidity or tremor, not incoordination.
Question 4
Climbing fibres originate from which source?
a) Inferior olivary nucleus
b) Red nucleus
c) Vestibular nuclei
d) Pontine nuclei
Explanation: Climbing fibres arise exclusively from the inferior olivary nucleus and form powerful excitatory synapses directly on Purkinje cells. They regulate motor learning and coordination through long-term depression at parallel fibre synapses. Answer: a) Inferior olivary nucleus. Mossy fibres, instead, arise from pontine and spinal inputs.
Question 5
Damage to Purkinje cells would primarily result in?
a) Spastic paralysis
b) Ataxia
c) Rigidity
d) Hyporeflexia
Explanation: Purkinje cell loss disrupts cerebellar inhibitory control, impairing coordination of movement. This produces ataxia with unsteady gait, dysdiadochokinesia, and intention tremor. Answer: b) Ataxia. Spasticity results from corticospinal damage, rigidity from basal ganglia disease, and hyporeflexia from lower motor neuron lesions.
Question 6
Which interneuron inhibits Purkinje cells in cerebellar cortex?
a) Basket cells
b) Golgi cells
c) Pyramidal cells
d) Oligodendrocytes
Explanation: Basket cells provide inhibitory axo-somatic synapses directly on Purkinje neurons, limiting their firing. Stellate cells also inhibit dendrites. Golgi cells regulate granule cells. Answer: a) Basket cells. These interneurons fine-tune Purkinje output before it reaches deep cerebellar nuclei, optimizing cerebellar motor coordination and timing.
Question 7
Which cerebellar lesion leads to truncal ataxia and swaying while standing?
a) Vermis
b) Hemisphere
c) Flocculonodular lobe
d) Dentate nucleus
Explanation: Lesions in cerebellar vermis cause truncal ataxia with broad-based gait and inability to maintain upright posture. Answer: a) Vermis. Hemisphere lesions cause limb ataxia, flocculonodular lesions cause balance and nystagmus, while dentate involvement produces dysmetria and decomposition of movement. Clinical localization depends on specific cerebellar subdivisions.
Question 8
Mossy fibres synapse first on?
a) Purkinje cells
b) Basket cells
c) Granule cells
d) Stellate cells
Explanation: Mossy fibres relay information from spinal cord and brainstem. They terminate on granule cells in cerebellar cortex, which then give rise to parallel fibres. These parallel fibres excite Purkinje dendrites. Answer: c) Granule cells. Thus, mossy fibres indirectly influence Purkinje output by granule cell activation.
Question 9
Which symptom is most typical of cerebellar disease?
a) Rest tremor
b) Intention tremor
c) Hypokinesia
d) Spasticity
Explanation: Cerebellar lesions produce intention tremor, which appears during voluntary movement and worsens as target is approached. Answer: b) Intention tremor. Rest tremor suggests Parkinsonism, hypokinesia occurs in basal ganglia disease, and spasticity arises from pyramidal tract lesions. Cerebellum chiefly impairs timing and coordination of motor actions.
Question 10
Which cerebellar output nucleus is largest and projects to motor cortex via thalamus?
a) Fastigial nucleus
b) Globose nucleus
c) Dentate nucleus
d) Emboliform nucleus
Explanation: The dentate nucleus is the largest deep cerebellar nucleus, projecting to contralateral motor cortex via the ventrolateral thalamus. It coordinates planning, timing, and fine motor execution. Answer: c) Dentate nucleus. Fastigial controls posture, globose and emboliform modulate intermediate motor activities of limbs.
Question 11
Which tract conveys unconscious proprioception from muscles to cerebellum?
a) Corticospinal tract
b) Spinothalamic tract
c) Spinocerebellar tract
d) Rubrospinal tract
Explanation: Spinocerebellar tracts carry proprioceptive input from muscle spindles and Golgi tendon organs to cerebellum. This unconscious sensory feedback helps adjust movement in real time. Answer: c) Spinocerebellar tract. Corticospinal controls voluntary movement, spinothalamic transmits pain/temperature, rubrospinal influences flexor tone but not proprioception.
Keyword Definitions
• Two-point discrimination: Ability to distinguish two separate simultaneous tactile stimuli.
• Tactile acuity: Sharpness of touch perception depending on receptor density.
• Receptive field: Area of skin innervated by a single sensory neuron.
• Merkel cells: Slowly adapting mechanoreceptors specialized for shape and edges.
• Meissner corpuscles: Rapidly adapting mechanoreceptors detecting flutter and low-frequency vibration.
• Pacinian corpuscles: Rapidly adapting mechanoreceptors specialized for high-frequency vibration.
• Dorsal columns: Pathways carrying touch, vibration, and proprioception.
• Medial lemniscus: Brainstem tract formed by decussated dorsal column fibers.
• Somatosensory cortex: Postcentral gyrus area processing tactile information.
• Cortical magnification: Enlarged cortical representation of regions like lips and fingertips.
• Astereognosis: Inability to identify objects by touch.
Chapter: General Physiology
Topic: Sensory Physiology
Subtopic: Two-point Discrimination
Lead Question – 2012
The distance by which two touch stimuli must be separated to be perceived as two separate stimuli is greatest at?
a) The lips
b) The palm of the hand
c) The back of scapula
d) The dorsum of the hand
Explanation: Two-point discrimination threshold is largest where receptive fields are big and cortical representation is small. Proximal trunk regions have poorest tactile acuity. Therefore, the greatest minimum separable distance is on the back of the scapula. Answer: c) The back of scapula.
Question 2
Which receptor type contributes most to high-resolution two-point discrimination on fingertips?
a) Pacinian corpuscles
b) Merkel discs
c) Ruffini endings
d) Free nerve endings
Explanation: Edges and fine form are encoded by slowly adapting type I mechanoreceptors with small receptive fields. Merkel cell–neurite complexes provide the highest spatial resolution for static touch and contribute most to two-point discrimination on fingertips and lips. Answer: b) Merkel discs.
Question 3
A patient with a hemisection of the spinal cord loses two-point discrimination below the lesion. Which tract is involved?
a) Spinothalamic tract
b) Dorsal column pathway
c) Spinocerebellar tract
d) Corticospinal tract
Explanation: A hemisection damaging dorsal columns impairs ipsilateral discriminative touch, vibration sense, and conscious proprioception below the lesion. Two-point discrimination on the affected side is markedly reduced, while pain and temperature may remain spared. Answer: b) Dorsal column pathway.
Question 4
Where is the two-point discrimination threshold smallest in the body?
a) Fingertips
b) Palm
c) Back
d) Abdomen
Explanation: Two-point thresholds are smallest where receptor density is highest and receptive fields are tiniest. Fingertips have abundant Merkel and Meissner endings plus strong cortical magnification, enabling exquisite spatial acuity. Therefore, minimum separable distance is least at fingertips. Answer: a) Fingertips.
Question 5
Which pathway carries discriminative touch and vibration sense to the brain?
a) Spinothalamic tract
b) Corticospinal tract
c) Dorsal column–medial lemniscus pathway
d) Spinoreticular tract
Explanation: Discriminative touch, vibration, and conscious proprioception ascend ipsilaterally in the dorsal columns to nucleus gracilis and cuneatus, then decussate as internal arcuate fibers to form the medial lemniscus. They project to thalamic VPL and somatosensory cortex. Answer: c) Dorsal column–medial lemniscus pathway.
Question 6
In which cortical region is two-point discrimination primarily resolved?
a) Prefrontal cortex
b) Primary somatosensory cortex
c) Insular cortex
d) Cerebellum
Explanation: Two-point discrimination is ultimately resolved in the primary somatosensory cortex on the postcentral gyrus, especially area 3b, exhibiting cortical magnification for hands and lips. Lesions there cause astereognosis and impaired tactile acuity contralaterally. Answer: b) Primary somatosensory cortex.
Question 7
A patient has loss of two-point discrimination on the left hand. Lesion is most likely in?
a) Right primary somatosensory cortex
b) Left motor cortex
c) Right cerebellum
d) Left dorsal root ganglion
Explanation: Somatosensory pathways decussate before reaching the cortex. Loss of discriminative touch from the left hand arises with a lesion in the contralateral somatosensory cortex. Answer: a) Right primary somatosensory cortex.
Question 8
Which phenomenon sharpens spatial resolution in two-point discrimination by inhibiting neighboring neurons?
a) Rebound excitation
b) Temporal summation
c) Lateral inhibition
d) Referred sensation
Explanation: Lateral inhibition enhances sensory contrast by suppressing responses in adjacent receptive fields. This improves spatial acuity and is fundamental for resolving two-point discrimination. Answer: c) Lateral inhibition.
Question 9
Which clinical sign indicates impaired cortical processing of tactile stimuli despite intact primary sensory pathways?
a) Hyperalgesia
b) Allodynia
c) Astereognosis
d) Hyperreflexia
Explanation: Patients with cortical lesions affecting parietal sensory areas cannot identify objects by touch despite preserved basic tactile sensation. This condition is astereognosis. Answer: c) Astereognosis.
Question 10
A lesion in which thalamic nucleus impairs two-point discrimination from the contralateral body?
a) VPL nucleus
b) Medial geniculate nucleus
c) VPM nucleus
d) Lateral geniculate nucleus
Explanation: The ventral posterolateral (VPL) nucleus of the thalamus receives medial lemniscus inputs carrying discriminative touch, vibration, and proprioception from the contralateral body. Lesions here impair two-point discrimination. Answer: a) VPL nucleus.
Question 11
Two-point discrimination is impaired but pain sensation remains intact. Which tract remains unaffected?
a) Spinothalamic tract
b) Dorsal column–medial lemniscus pathway
c) Corticospinal tract
d) Reticulospinal tract
Explanation: Preservation of pain sensation indicates an intact spinothalamic tract. Impairment of two-point discrimination indicates dorsal column dysfunction. Answer: a) Spinothalamic tract.
Chapter: Central Nervous System Physiology | Topic: Neuronal Membrane & Action Potential | Subtopic: Voltage-Gated Sodium Channels
Keywords
Voltage-gated sodium channels — proteins that open on depolarization allowing Na⁺ influx to initiate action potentials.
Axon initial segment (AIS) / Axon hillock — region where action potentials are usually initiated due to high Na⁺ channel density.
Nodes of Ranvier — gaps in myelin with concentrated Na⁺ channels enabling saltatory conduction.
Soma — neuronal cell body; integrates synaptic inputs but has lower Na⁺ channel density than AIS.
Dendrites — receive inputs and may have Na⁺ channels for back-propagation, but fewer than AIS.
Action potential threshold — lowest depolarization required to open sufficient Na⁺ channels to trigger spike.
Saltatory conduction — rapid impulse propagation between nodes of Ranvier in myelinated axons.
Ankyrin-G — scaffold protein essential for clustering Na⁺ channels at the axon initial segment.
Local anaesthetics — block voltage-gated Na⁺ channels to prevent action-potential propagation.
Channelopathies — disorders caused by mutations in sodium channel genes affecting excitability and causing seizures or paralysis.
Lead Question - 2012
Sodium channels are maximum in which part of neuron ?
a) Soma
b) Axon hillock
c) Dendrites
d) Axon
Explanation: The axon hillock (axon initial segment) has the highest density of voltage-gated sodium channels and is the usual trigger zone for action potentials. This high channel concentration, organized by ankyrin-G and associated scaffolds, lowers the threshold for spike initiation. Correct answer: b) Axon hillock.
Q1. Where along myelinated axons are sodium channels highly concentrated to enable saltatory conduction?
a) Internodal myelin
b) Nodes of Ranvier
c) Soma membrane
d) Dendritic spines
Explanation: Nodes of Ranvier are unmyelinated gaps densely populated with voltage-gated sodium channels. Action potentials are regenerated at these nodes, allowing rapid saltatory conduction down the axon. This arrangement increases conduction velocity and metabolic efficiency. Correct answer: b) Nodes of Ranvier.
Q2. Which scaffolding protein is essential for clustering sodium channels at the axon initial segment?
a) Ankyrin-G
b) Tubulin
c) Actin
d) Spectrin
Explanation: Ankyrin-G anchors voltage-gated sodium channels and other membrane proteins to the axon initial segment cytoskeleton, maintaining high local channel density necessary for action-potential initiation. Disruption of ankyrin-G disperses channels and reduces neuronal excitability. Correct answer: a) Ankyrin-G.
Q3. Local anaesthetics such as lidocaine produce analgesia primarily by blocking which channels?
a) Voltage-gated potassium channels
b) Voltage-gated sodium channels
c) Voltage-gated calcium channels
d) Ligand-gated chloride channels
Explanation: Local anaesthetics bind to and block voltage-gated sodium channels, preventing initiation and propagation of action potentials in sensory fibers. Small nociceptive fibers are preferentially blocked, producing loss of pain and temperature sensation. Correct answer: b) Voltage-gated sodium channels.
Q4. A mutation that reduces sodium-channel availability in the axon hillock would most likely cause:
a) Increased neuronal firing
b) Decreased excitability and possible weakness
c) Faster action potentials
d) Enhanced synaptic transmission
Explanation: Reduced sodium-channel availability at the axon initial segment raises the threshold for spike initiation, decreasing neuronal excitability and impairing signal transmission. Clinically this may cause muscle weakness, conduction block, or epileptic phenotypes depending on neuronal population affected. Correct answer: b) Decreased excitability and possible weakness.
Q5. Dendritic sodium channels support which physiological process relevant to synaptic plasticity?
a) Action-potential back-propagation
b) Neurotransmitter synthesis
c) Axonal myelination
d) Vesicle recycling
Explanation: Voltage-gated sodium channels in dendrites allow action potentials to back-propagate into the dendritic tree, modulating calcium entry and synaptic strength. This back-propagation contributes to spike-timing-dependent plasticity and learning. Correct answer: a) Action-potential back-propagation.
Q6. During the relative refractory period, why is a larger stimulus required to elicit an action potential?
a) All Na⁺ channels are permanently removed
b) Many Na⁺ channels are inactivated and K⁺ conductance is increased
c) Membrane potential is more positive than threshold
d) Synaptic inputs are inhibited
Explanation: After an action potential, a subset of Na⁺ channels remains inactivated and K⁺ channels remain open, hyperpolarizing the membrane; a stronger depolarizing input is thus required to reach threshold. This defines the relative refractory period. Correct answer: b) Many Na⁺ channels are inactivated and K⁺ conductance is increased.
Q7. Which feature increases axonal conduction velocity most effectively?
a) Decreasing axon diameter
b) Increasing myelination and axon diameter
c) Reducing Na⁺ channel density at nodes
d) Increasing internodal capacitance
Explanation: Larger axon diameter and increased myelination raise conduction velocity by reducing internal resistance and membrane capacitance. Adequate sodium-channel density at nodes is also required. Correct answer: b) Increasing myelination and axon diameter.
Q8. Which antiepileptic medication exerts effects by stabilizing the inactivated state of sodium channels?
a) Phenytoin
b) Levodopa
c) Fluoxetine
d) Propranolol
Explanation: Phenytoin binds voltage-gated sodium channels, prolonging their inactivated state and limiting repetitive firing of neurons. This mechanism reduces seizure propagation in many epilepsy syndromes. Correct answer: a) Phenytoin.
Q9. Which region is the most common site for initiation of spontaneous epileptic discharges due to high excitability?
a) Axon hillock / initial segment
b) Distal axon terminals
c) Soma nucleus
d) Myelin sheath
Explanation: The axon initial segment’s high density of sodium channels and low threshold make it a frequent locus for abnormal spontaneous discharges in epilepsy. Pathologic increases in excitability here can produce paroxysmal firing. Correct answer: a) Axon hillock / initial segment.
Q10. Which pathological process directly reduces sodium-channel clustering at the AIS leading to reduced excitability?
a) Mutation or loss of ankyrin-G
b) Increased myelination
c) Enhanced Na⁺ channel synthesis
d) Elevated extracellular potassium only
Explanation: Loss or dysfunction of ankyrin-G disrupts anchoring of sodium channels at the AIS, dispersing them and impairing action-potential initiation. This reduces neuronal excitability and can contribute to neurological disease. Correct answer: a) Mutation or loss of ankyrin-G.
Chapter: Central Nervous System Physiology | Topic: Hypothalamic Control | Subtopic: Thermoregulation and Shivering
Keywords
Thermoregulation — physiological processes that maintain core body temperature.
Preoptic area (POA) — hypothalamic region sensing temperature and coordinating heat-loss responses.
Posterior hypothalamus — activates heat-production mechanisms including shivering and sympathetic vasoconstriction.
Shivering — involuntary rhythmic skeletal muscle contractions generating heat under hypothalamic drive.
Warm-sensitive neurons — in POA; stimulate heat-loss (sweating, vasodilation).
Cold-sensitive pathways — activate posterior hypothalamus to produce heat via shivering and autonomic output.
Pyrogens — raise hypothalamic set point producing fever (distinct from hyperthermia).
Thermal effector organs — skeletal muscle (shivering), skin vessels (vasomotor), sweat glands.
Behavioral responses — seeking shelter/clothing under hypothalamic and cortical influence.
Clinical relevance — hypothalamic lesions can produce hypothermia or hyperthermia and loss of shivering.
Lead Question - 2012
Shivering is controlled by: (also in September 2012, March 2013)
a) Dorsomedial nucleus
b) Posterior hypothalamus
c) Perifornical nucleus
d) Lateral hypothalamic area
Explanation: Shivering—involuntary rhythmic skeletal muscle contractions that generate heat—is driven by cold-sensitive pathways activating the posterior hypothalamus. The posterior hypothalamic area orchestrates motor and sympathetic outputs for heat production. Therefore the correct answer is b) Posterior hypothalamus, responsible for shivering and thermogenic responses.
Q1. Lesion of the preoptic area typically causes:
a) Hypothermia
b) Hyperthermia
c) Diabetes insipidus
d) Hyperphagia
Explanation: The preoptic area contains warm-sensitive neurons initiating heat-loss responses. Lesioning it abolishes heat-dissipation, producing uncontrolled rise in body temperature (hyperthermia). Thus the correct answer is b) Hyperthermia. This differs from DI or appetite disturbances tied to other hypothalamic nuclei.
Q2. Fever differs from hyperthermia because fever results from:
a) Ambient heat overload
b) Raised hypothalamic set point due to pyrogens
c) Failure of sweating
d) Posterior hypothalamic lesion
Explanation: Fever arises when pyrogens raise the hypothalamic thermoregulatory set point, causing the body to conserve and generate heat until the new set point is reached. This distinguishes fever from hyperthermia, which is failure of heat loss. Correct answer: b).
Q3. Which hypothalamic area promotes heat production when activated by cold?
a) Anterior hypothalamus
b) Posterior hypothalamus
c) Suprachiasmatic nucleus
d) Arcuate nucleus
Explanation: Cold signals activate cold-sensitive afferents that stimulate the posterior hypothalamus to increase thermogenesis by shivering and sympathetic activation. The anterior (preoptic) area mediates heat loss. Correct answer: b) Posterior hypothalamus, which triggers heat-generating mechanisms.
Q4. Which effector mediates most rapid heat production in humans?
a) Brown adipose tissue
b) Shivering (skeletal muscle activity)
c) Increased thyroid secretion
d) Skin vasodilation
Explanation: Shivering produces immediate heat via rhythmic skeletal muscle contractions under posterior hypothalamic control, providing rapid thermogenesis. Brown adipose tissue contributes in infants, while thyroid changes and vasomotor adjustments are slower. Correct answer: b) Shivering.
Q5. Which sign indicates activation of heat-loss mechanisms?
a) Vasoconstriction
b) Shivering
c) Sweating and cutaneous vasodilation
d) Piloerection
Explanation: Heat-loss responses include sweating and cutaneous vasodilation mediated by preoptic area signals. These lower core temperature by evaporative cooling and increased skin blood flow. Correct answer: c) Sweating and cutaneous vasodilation, opposite to shivering which produces heat.
Q6. A patient with impaired shivering after hypothalamic surgery most likely had damage to:
a) Ventromedial nucleus
b) Posterior hypothalamus
c) Suprachiasmatic nucleus
d) Lateral hypothalamus
Explanation: Surgical damage to the posterior hypothalamus abolishes cold-induced shivering and some sympathetic thermogenic responses. Therefore impaired shivering after hypothalamic surgery suggests posterior hypothalamic injury. Correct answer: b) Posterior hypothalamus.
Q7. Which autonomic response accompanies shivering to preserve core temperature?
a) Cutaneous vasodilation
b) Cutaneous vasoconstriction
c) Diaphoresis
d) Increased salivation
Explanation: To conserve heat during shivering, sympathetic-mediated cutaneous vasoconstriction reduces blood flow to the skin, minimizing heat loss. This complements muscular heat production. Correct answer: b) Cutaneous vasoconstriction.
Q8. Which hypothalamic nucleus is the master clock for circadian temperature rhythm?
a) Suprachiasmatic nucleus (SCN)
b) Paraventricular nucleus
c) Dorsomedial nucleus
d) Lateral hypothalamus
Explanation: The suprachiasmatic nucleus (SCN) entrains circadian rhythms including daily fluctuations in body temperature by signaling other hypothalamic areas. Lesions disrupt rhythmic temperature variations. Correct answer: a) Suprachiasmatic nucleus (SCN).
Q9. Which pharmacologic agent can reduce shivering by central action?
a) Acetaminophen (paracetamol)
b) Meperidine (pethidine)
c) Epinephrine
d) Dobutamine
Explanation: Meperidine centrally suppresses shivering via opioid and α2-adrenergic effects in the hypothalamus and brainstem. It is used to treat postoperative shivering. Correct answer: b) Meperidine (pethidine).
Q10. In hypothermia, which behavioral response is initiated by cortical and hypothalamic centers?
a) Removing clothing
b) Seeking warmth and adding clothing
c) Inducing sweat
d) Increasing water intake
Explanation: Behavioral thermoregulation includes seeking warmth and adding clothing to reduce heat loss; these actions are driven by hypothalamic signals integrated with cortical decision-making. Correct answer: b) Seeking warmth and adding clothing.
Chapter: Central Nervous System Physiology | Topic: Hypothalamic Functions | Subtopic: Preoptic Area & Homeostasis
Keyword Definitions
Preoptic nucleus — hypothalamic region with warm-sensitive neurons controlling heat-loss responses.
Thermoregulation — physiological processes maintaining core temperature via autonomic and behavioral responses.
Hyperthermia — abnormally high body temperature due to failed heat dissipation or excessive heat production.
Hyperphagia — excessive eating driven by hypothalamic or metabolic disturbances.
Hyperdipsia — excessive thirst and fluid intake, often osmotic or hypothalamic in origin.
Homeostasis — coordinated regulation of internal milieu (temperature, thirst, hunger, endocrine balance).
Autonomic output — hypothalamic control of sympathetic and parasympathetic tone influencing temperature and metabolism.
POA lesions — can disrupt fever responses, thermoregulatory set points, and heat-loss mechanisms.
Fever vs hyperthermia — fever raises set point via pyrogens; hyperthermia is failure of dissipation.
Clinical relevance — hypothalamic injury, stroke, tumors can produce dysautonomia and temperature dysregulation.
Lead Question - 2012
Lesion of preoptic nucleus of hypothalamus causes?
a) Hyperphagia
b) Hyperdypsia
c) Hyperthermia
d) Hyperglycemia
Explanation: The preoptic area contains warm-sensitive neurons that initiate heat-loss responses (vasodilation, sweating). Lesioning these neurons abolishes heat-loss mechanisms, producing uncontrolled rise in body temperature (hyperthermia). This is not primarily a feeding or thirst center. Correct answer: c) Hyperthermia.
Q1. Damage to the lateral hypothalamic area typically causes:
a) Anorexia
b) Hyperphagia
c) Polydipsia
d) Hypothermia
Explanation: The lateral hypothalamus is the feeding (hunger) center; lesions produce anorexia and weight loss, while stimulation causes hyperphagia. Thus damage causes lack of eating rather than increased appetite. Correct answer: a) Anorexia (lesion → anorexia; stimulation → hyperphagia).
Q2. A lesion of the supraoptic nucleus would most likely produce:
a) Diabetes insipidus (polyuria, polydipsia)
b) Cushing’s syndrome
c) Hyperthermia
d) Adipsia
Explanation: The supraoptic nucleus produces vasopressin (ADH); damage causes central diabetes insipidus with polyuria and compensatory polydipsia. This is a classic hypothalamic endocrine deficit. Correct answer: a) Diabetes insipidus (polyuria, polydipsia).
Q3. Which hypothalamic lesion produces hyperphagia and obesity in animals?
a) Ventromedial nucleus lesion
b) Lateral hypothalamic lesion
c) Preoptic lesion
d) Suprachiasmatic lesion
Explanation: The ventromedial hypothalamus is a satiety center; lesions remove restraining signals leading to hyperphagia and obesity. Lateral lesions cause anorexia. Correct answer: a) Ventromedial nucleus lesion.
Q4. Destruction of the suprachiasmatic nucleus (SCN) leads to:
a) Loss of circadian rhythms
b) Hyperthermia
c) Polyphagia
d) Diabetes insipidus
Explanation: The SCN is the master circadian pacemaker; lesions disrupt sleep-wake, hormonal, and temperature rhythms. This abolishes regular circadian patterns but does not directly cause fever or thirst disorders. Correct answer: a) Loss of circadian rhythms.
Q5. Fever (pyrogen-mediated) differs from hyperthermia because fever involves:
a) Raised hypothalamic set point
b) Failure of heat dissipation
c) Ambient heat overload
d) Direct injury to preoptic neurons
Explanation: Fever results from pyrogens raising the hypothalamic thermostat (set point), inducing chills and thermoregulatory defenses to reach the new set point. Hyperthermia is failure of heat loss without set-point change. Correct answer: a) Raised hypothalamic set point.
Q6. A patient with hypothalamic lesion presents with persistent hyperphagia and rage; which nucleus is likely affected?
a) Ventromedial nucleus
b) Preoptic nucleus
c) Paraventricular nucleus
d) Lateral hypothalamic area
Explanation: Ventromedial nucleus lesions remove satiety signals causing hyperphagia and aggression (sham rage). The lateral hypothalamus promotes feeding when stimulated. Paraventricular lesions affect autonomic and endocrine outputs. Correct answer: a) Ventromedial nucleus.
Q7. Lesion of preoptic area interferes with which autonomic thermoregulatory response?
a) Sweating and cutaneous vasodilation
b) Salivation
c) Pupillary constriction
d) Gastrointestinal motility
Explanation: The preoptic area triggers heat-loss responses such as sweating and vasodilation. Lesions abolish these mechanisms, leading to impaired heat dissipation and hyperthermia. Other autonomic functions are mediated by different hypothalamic regions. Correct answer: a) Sweating and cutaneous vasodilation.
Q8. Paraventricular nucleus (PVN) lesions primarily affect:
a) Oxytocin and CRH release influencing endocrine and autonomic functions
b) Visual processing
c) Primary motor control
d) Vestibular reflexes
Explanation: PVN neurons produce CRH and oxytocin and modulate sympathetic outflow; lesions disrupt HPA axis regulation and autonomic balance. This leads to endocrine and autonomic dysfunction rather than direct motor or visual deficits. Correct answer: a).
Q9. Central fever after hypothalamic hemorrhage is due to:
a) Disruption of preoptic heat-loss neurons
b) Bacterial infection
c) Peripheral inflammation only
d) Increased sweating
Explanation: Hypothalamic injury can cause central fever by damaging preoptic/POA neurons that mediate heat loss and set-point regulation; this produces sustained hyperthermia without infection. Correct answer: a) Disruption of preoptic heat-loss neurons.
Q10. A lesion of arcuate nucleus would most likely cause:
a) Disordered appetite regulation and altered GnRH/release control
b) Loss of temperature sensation
c) Loss of visual fields
d) Cerebellar ataxia
Explanation: The arcuate nucleus integrates peripheral metabolic signals (leptin, ghrelin) and influences appetite, prolactin and GnRH modulation. Lesions disrupt feeding and reproductive hormone regulation. It is not primarily involved in temperature sensation or motor coordination. Correct answer: a).
Chapter: Sensory Physiology | Topic: Visual Transduction | Subtopic: Photoreceptor Proteins
Keywords
Transducin — a heterotrimeric G-protein in photoreceptor cells involved in phototransduction.
Rhodopsin — light-sensitive pigment in rod outer segments that activates transducin.
Phototransduction — process converting photon absorption into electrical signals in retina.
cGMP phosphodiesterase — enzyme activated by transducin to lower cGMP and hyperpolarize photoreceptors.
Rods and cones — retinal photoreceptors for scotopic and photopic vision respectively.
Retinal — chromophore (11-cis-retinal) that changes conformation on photon absorption.
Hyperpolarization — photoreceptor response to light due to decreased cGMP-gated current.
Dark current — inward Na⁺/Ca²⁺ current in photoreceptors maintained by cGMP.
Visual cycle — enzymatic regeneration of 11-cis-retinal in retinal pigment epithelium.
G-protein coupled receptor (GPCR) — rhodopsin is a GPCR that activates transducin.
Lead Question - 2012
Transducin is a protein found in:
a) Glomerulus
b) Retina
c) Skeletal muscle
d) Adrenal medulla
Explanation: Transducin is a G-protein located in photoreceptor outer segments of the retina; it couples activated rhodopsin to cGMP phosphodiesterase. Upon photon capture transducin activates PDE, lowers cGMP, closes cGMP-gated channels and hyperpolarizes the photoreceptor. Correct answer: b) Retina.
Q1. Which pigment initiates phototransduction by activating transducin?
a) Hemoglobin
b) Melanin
c) Rhodopsin
d) Opsin in kidney
Explanation: Rhodopsin in rod outer segments absorbs photons, isomerizes 11-cis-retinal to all-trans-retinal, and activates the GPCR rhodopsin which then activates transducin, initiating the phototransduction cascade. Correct answer: c) Rhodopsin.
Q2. Activation of transducin leads directly to activation of which enzyme?
a) Adenylate cyclase
b) cGMP phosphodiesterase
c) Na⁺/K⁺ ATPase
d) Phospholipase C
Explanation: Activated transducin (Gαt) stimulates cGMP phosphodiesterase, decreasing cytoplasmic cGMP, closing cGMP-gated cation channels and hyperpolarizing photoreceptors. This is central to light signal transduction. Correct answer: b) cGMP phosphodiesterase.
Q3. Photoreceptor response to light is a:
a) Depolarization
b) Hyperpolarization
c) Action potential firing
d) No change
Explanation: Light activation leads to reduced cGMP, closure of cGMP-gated Na⁺ channels, decreased inward dark current and membrane hyperpolarization of photoreceptors. This graded hyperpolarization reduces glutamate release. Correct answer: b) Hyperpolarization.
Q4. Which retinal cells regenerate 11-cis-retinal as part of the visual cycle?
a) Müller glia
b) Retinal pigment epithelium (RPE)
c) Ganglion cells
d) Bipolar cells
Explanation: The retinal pigment epithelium (RPE) performs enzymatic steps to convert all-trans-retinal back to 11-cis-retinal, replenishing the chromophore for photopigments. This visual cycle is essential for sustained phototransduction. Correct answer: b) Retinal pigment epithelium (RPE).
Q5. Which photoreceptors primarily use transducin in their signal cascade?
a) Rods
b) Cones
c) Ganglion photoreceptors only
d) Both rods and cones
Explanation: Both rods and cones possess phototransduction cascades that employ G-proteins homologous to transducin (rod transducin Gαt1, cone transducins Gαt2), so both use transducin-like proteins to activate PDE. Correct answer: d) Both rods and cones.
Q6. A defect in transducin function would most likely cause:
a) Color blindness only
b) Night blindness and impaired phototransduction
c) Loss of accommodation
d) Elevated intraocular pressure
Explanation: Impaired transducin prevents effective activation of PDE, blunting photoreceptor hyperpolarization and reducing sensitivity, particularly affecting scotopic (rod-mediated) vision, causing night blindness and phototransduction defects. Correct answer: b) Night blindness and impaired phototransduction.
Q7. Which event follows activation of cGMP phosphodiesterase in photoreceptors?
a) Increased intracellular cGMP
b) Closure of cGMP-gated cation channels
c) Increased glutamate release
d) Depolarization
Explanation: PDE lowers cGMP levels, resulting in closure of cGMP-gated Na⁺/Ca²⁺ channels, decreased inward current and reduced glutamate release due to photoreceptor hyperpolarization. Correct answer: b) Closure of cGMP-gated cation channels.
Q8. Which molecule directly undergoes photoisomerization to start phototransduction?
a) 11-cis-retinal
b) Opsin protein backbone
c) cGMP
d) Transducin
Explanation: The chromophore 11-cis-retinal within rhodopsin photoisomerizes to all-trans-retinal upon photon absorption, changing rhodopsin conformation and activating transducin, initiating the cascade. Correct answer: a) 11-cis-retinal.
Q9. Which test assesses rod (scotopic) function most directly?
a) Photopic visual acuity
b) Dark adaptation test
c) Color vision test
d) Pupillary light reflex
Explanation: Dark adaptation measures recovery of visual sensitivity in low light, reflecting rod photoreceptor and transducin-PDE cascade function. Delayed or impaired dark adaptation suggests rod/transducin pathway dysfunction. Correct answer: b) Dark adaptation test.
Q10. Which class of receptors does rhodopsin belong to?
a) Ligand-gated ion channel
b) Tyrosine kinase receptor
c) G-protein coupled receptor (GPCR)
d) Nuclear receptor
Explanation: Rhodopsin is a GPCR embedded in photoreceptor membranes; upon photon-induced conformational change it activates transducin (a G-protein), classifying rhodopsin as a light-activated GPCR. Correct answer: c) G-protein coupled receptor (GPCR).
Chapter: Central Nervous System Physiology
Topic: Nerve Fibers and Conduction
Subtopic: Sensitivity to Pressure and Hypoxia
Keyword Definitions:
• Nerve Fibers: Axons classified based on diameter, conduction velocity, and function (A, B, C fibers).
• A Fibers: Large, myelinated fibers with rapid conduction, subdivided into alpha, beta, gamma, delta.
• B Fibers: Small, myelinated preganglionic autonomic fibers.
• C Fibers: Small, unmyelinated fibers, slow conduction, pain and temperature transmission.
• Hypoxia Sensitivity: Vulnerability of fibers to oxygen deprivation.
• Pressure Sensitivity: Susceptibility of fibers to mechanical compression.
• Neuropraxia: Temporary conduction block often due to compression.
• Paresthesia: Abnormal tingling or numb sensation due to nerve dysfunction.
Lead Question - 2012
A man slept with head over forearm, next morning he complains of tingling, numbness over forearm. It is caused by?
a) Sensitivity to hypoxia is A > B > C
b) Sensitivity to pressure is A > B > C
c) Sensitivity to hypoxia is C > B > A
d) Sensitivity to pressure is B > A > C
Explanation: Large myelinated A fibers are more susceptible to compression due to their size and myelin sheath. This explains temporary numbness or tingling after sleeping on a limb. Small unmyelinated C fibers are more resistant. Hence, sensitivity to pressure is A > B > C. Answer: b) Sensitivity to pressure is A > B > C
--- Guessed Question 1
Which nerve fibers are most sensitive to hypoxia?
a) A fibers
b) B fibers
c) C fibers
d) All equally
Explanation: Unmyelinated C fibers require continuous metabolic support and are highly sensitive to hypoxia. In oxygen deprivation, C fibers lose function first, causing early loss of pain and temperature sensation. Answer: c) C fibers
--- Guessed Question 2
Temporary conduction block without axonal damage due to compression is termed:
a) Axonotmesis
b) Neurotmesis
c) Neuropraxia
d) Wallerian degeneration
Explanation: Neuropraxia is a transient conduction block due to mechanical compression, as in the case of sleeping on an arm. Recovery is complete within days to weeks as no axonal damage occurs. Answer: c) Neuropraxia
--- Guessed Question 3
Which type of fibers mediate burning pain?
a) A alpha
b) A beta
c) A delta
d) C fibers
Explanation: Burning, dull, poorly localized pain is transmitted by unmyelinated C fibers. A delta fibers carry sharp, pricking pain. Thus, chronic tingling or burning after compression is mediated by C fibers. Answer: d) C fibers
--- Guessed Question 4
Which fibers are blocked earliest by local anesthetics?
a) A alpha
b) A delta
c) B fibers
d) C fibers
Explanation: Small, myelinated B fibers (preganglionic autonomic) are most sensitive to local anesthetics, followed by C fibers, then A delta, and lastly large motor A alpha fibers. Answer: c) B fibers
--- Guessed Question 5
Patient develops numbness after tight bandage. Most likely affected fibers are:
a) A fibers
b) B fibers
c) C fibers
d) None
Explanation: Mechanical compression preferentially affects large myelinated A fibers, leading to numbness and weakness. Unmyelinated C fibers remain relatively preserved. Answer: a) A fibers
--- Guessed Question 6
Which sensation is first affected during hypoxia?
a) Pain
b) Touch
c) Autonomic functions
d) Vibration
Explanation: Pain is mediated by C fibers, which are highly hypoxia-sensitive. Thus, pain sensation is often first impaired in hypoxic conditions. Answer: a) Pain
--- Guessed Question 7
Compression of radial nerve during deep sleep leads to:
a) Wrist drop
b) Foot drop
c) Claw hand
d) Facial palsy
Explanation: Radial nerve palsy due to compression in sleep ("Saturday night palsy") leads to weakness of wrist extensors, manifesting as wrist drop. Answer: a) Wrist drop
--- Guessed Question 8
Which nerve fiber type has the slowest conduction velocity?
a) A alpha
b) A beta
c) B fibers
d) C fibers
Explanation: Unmyelinated C fibers have the slowest conduction velocity (~0.5–2 m/s) compared to fast-conducting A alpha fibers (~100 m/s). Answer: d) C fibers
--- Guessed Question 9
Which fibers are most pressure-sensitive clinically leading to tingling?
a) A fibers
b) B fibers
c) C fibers
d) All equally
Explanation: Large, heavily myelinated A fibers are most pressure-sensitive, hence tingling and numbness are due to their dysfunction. Answer: a) A fibers
--- Guessed Question 10
Loss of touch and vibration but preserved pain after compression indicates damage to:
a) A alpha and beta fibers
b) A delta fibers
c) C fibers
d) B fibers
Explanation: Touch and vibration are carried by large A alpha and A beta fibers, which are most pressure-sensitive. C fibers carrying pain remain intact initially, explaining preserved pain sensation. Answer: a) A alpha and beta fibers
Chapter: Peripheral Nerve Physiology | Topic: Nociception & Pain Fibres | Subtopic: Sensory Fiber Types
Keywords
Peripheral nociceptors — sensory receptors that signal tissue-damaging stimuli.
Aδ fibres — thin myelinated fibres conducting fast sharp pain.
C fibres — small unmyelinated fibres conducting slow burning/dull pain.
Aβ fibres — large myelinated fibres for touch and vibration, not primary nociception.
Conduction velocity — speed of action potential propagation determined by diameter and myelination.
Spinothalamic tract — ascending pathway transmitting pain and temperature to thalamus.
Substance P / CGRP — neuropeptides released by nociceptors mediating pain and neurogenic inflammation.
Gate control theory — modulation of pain by non-nociceptive afferents at spinal level.
Central sensitization — heightened dorsal horn responsiveness after persistent nociceptive input.
Local anaesthetics — block sodium channels, preferentially affecting small diameter fibres first.
Lead Question - 2012
Burning pain is carried by which type of fibres ?
a) A alpha
b) A delta
c) A beta
d) C
Explanation: Burning, slow, poorly localized pain is typically transmitted by small unmyelinated C fibres that conduct at low velocity and carry polymodal nociceptive input. Aδ fibres convey fast sharp pain. Therefore the correct answer is d) C. C-fibre activity also mediates neurogenic inflammation via Substance P and CGRP.
Q1. Fast, well-localized sharp pain (first pain) is carried mainly by:
a) A alpha
b) A delta
c) C fibres
d) A beta
Explanation: First, sharp pain is mediated by thinly myelinated Aδ fibres that have higher conduction velocity than C fibres and project through the spinothalamic tract to somatosensory cortex, producing rapid, localized pain sensations. Hence the correct answer is b) A delta.
Q2. Which fibres primarily transmit touch and vibration?
a) A beta
b) C fibres
c) A delta
d) A gamma
Explanation: Large myelinated Aβ fibres carry discriminative touch, pressure, and vibration information via the dorsal column–medial lemniscal pathway. They are not primary nociceptors. Correct answer: a) A beta. Activation of Aβ fibres can modulate pain via gate control mechanisms in the dorsal horn.
Q3. Which ascending pathway carries pain and temperature to the brain?
a) Dorsal columns
b) Spinothalamic tract
c) Corticospinal tract
d) Spinocerebellar tract
Explanation: The anterolateral system, chiefly the spinothalamic tract, transmits nociceptive and thermoreceptive signals from spinal cord to thalamus and cortex. Dorsal columns carry vibration and proprioception. Correct answer: b) Spinothalamic tract.
Q4. Which neuropeptide released from nociceptors contributes to neurogenic inflammation?
a) GABA
b) Substance P
c) Dopamine
d) Serotonin
Explanation: Substance P and CGRP released from peripheral terminals of C fibres promote vasodilation, plasma extravasation, and immune cell recruitment, producing neurogenic inflammation and sensitization. This augments pain. Correct answer: b) Substance P.
Q5. Local anaesthetics block which channels to prevent nociception?
a) Calcium channels
b) Sodium channels
c) Potassium channels
d) Chloride channels
Explanation: Local anaesthetics inhibit voltage-gated sodium channels, preventing action potential initiation and propagation in sensory fibres. Small-diameter unmyelinated C and thin myelinated Aδ fibres are blocked preferentially, producing analgesia. Correct answer: b) Sodium channels.
Q6. Which clinical sign suggests small fibre (C/Aδ) neuropathy?
a) Loss of vibration sense
b) Burning distal pain with preserved reflexes
c) Pure motor weakness
d) Loss of proprioception
Explanation: Small-fibre neuropathy causes burning, shooting pain and dysesthesias in a distal stocking distribution with relatively preserved muscle strength and large-fibre modalities like vibration. Reflexes may be normal early. Correct answer: b) Burning distal pain with preserved reflexes.
Q7. Gate control theory proposes that activation of which fibres inhibits pain transmission?
a) C fibres
b) A beta fibres
c) A delta fibres
d) Sympathetic efferents
Explanation: Large-diameter Aβ fibres carrying touch input activate inhibitory interneurons in dorsal horn, reducing transmission from nociceptive Aδ/C fibres to projection neurons. This underlies analgesic effects of rubbing or TENS. Correct answer: b) A beta fibres.
Q8. Central sensitization results in which clinical phenomenon?
a) Hypoalgesia
b) Allodynia (pain to non-painful stimuli)
c) Loss of reflexes
d) Improved proprioception
Explanation: Persistent nociceptive input induces dorsal horn hyperexcitability and synaptic plasticity, producing allodynia and hyperalgesia where innocuous stimuli become painful. This is central sensitization seen in chronic pain syndromes. Correct answer: b) Allodynia.
Q9. Which fibre type has the slowest conduction velocity?
a) A alpha
b) A beta
c) A delta
d) C fibres
Explanation: Unmyelinated C fibres have the smallest diameter and slowest conduction velocity (~0.5–2 m/s), mediating slow burning pain and autonomic reflexes. Aα/Aβ are fastest. Correct answer: d) C fibres.
Q10. Which analgesic mechanism involves opioid receptors in the dorsal horn?
a) NSAID inhibition of COX
b) Activation of μ-opioid receptors reducing neurotransmitter release
c) Local anaesthetic sodium channel block
d) TRPV1 activation
Explanation: Opioids bind μ receptors on presynaptic nociceptive terminals and postsynaptic dorsal horn neurons, inhibiting substance P release and hyperpolarizing neurons, reducing pain transmission centrally. This is a principal mechanism for strong analgesics. Correct answer: b) Activation of μ-opioid receptors.
Chapter: Central Nervous System Physiology
Topic: Cerebrospinal Fluid (CSF)
Subtopic: Biochemical Properties of CSF
Keyword Definitions:
• CSF: Clear fluid in brain and spinal cord providing cushioning and nutrient exchange.
• Plasma: Liquid component of blood carrying cells, proteins, and nutrients.
• CSF/Plasma Glucose Ratio: A diagnostic marker comparing CSF glucose levels to plasma glucose.
• Blood-Brain Barrier: Selective barrier regulating passage of substances into CSF.
• Meningitis: Inflammation of meninges often causing altered CSF glucose.
• Hypoglycorrhachia: Abnormally low CSF glucose, seen in infections and tumors.
• Hyperglycemia: High plasma glucose leading to proportionally higher CSF glucose.
• CSF Analysis: Laboratory test to evaluate neurological diseases.
Lead Question - 2012
CSF/plasma glucose ratio is ?
a) 0.2 - 0.4
b) 0.6 - 0.8
c) 1.2 - 1.6
d) 1.6 - 2.2
Explanation: The normal CSF/plasma glucose ratio is about 0.6 to 0.8. CSF glucose is typically two-thirds of plasma glucose. Reduced ratios occur in bacterial and TB meningitis, while viral meningitis usually maintains normal values. Answer: b) 0.6 - 0.8
--- Guessed Question 1
In bacterial meningitis, the CSF/plasma glucose ratio is usually:
a) Normal
b) Increased
c) Decreased
d) Unchanged
Explanation: Bacterial meningitis reduces CSF glucose due to bacterial metabolism. The CSF/plasma glucose ratio often falls below 0.4. This is a key diagnostic indicator. Answer: c) Decreased
--- Guessed Question 2
Which CSF finding is most suggestive of tuberculous meningitis?
a) Normal CSF glucose
b) CSF glucose ↓
c) CSF protein normal
d) No pleocytosis
Explanation: Tuberculous meningitis shows markedly reduced glucose, elevated proteins, and lymphocytic pleocytosis. This pattern helps distinguish it from viral meningitis. Answer: b) CSF glucose ↓
--- Guessed Question 3
A patient with viral meningitis is most likely to show which CSF/plasma glucose ratio?
a) 0.6 - 0.8
b) c) >1.0
d) 0.2
Explanation: Viral meningitis does not significantly affect CSF glucose, so the CSF/plasma ratio remains in the normal range of 0.6–0.8. Answer: a) 0.6 - 0.8
--- Guessed Question 4
Which condition most commonly causes low CSF glucose with elevated protein and lymphocytes?
a) Viral meningitis
b) Tuberculous meningitis
c) Subarachnoid hemorrhage
d) Normal pressure hydrocephalus
Explanation: Tuberculous meningitis typically presents with low glucose, high protein, and lymphocytic predominance. Answer: b) Tuberculous meningitis
--- Guessed Question 5
Increased CSF glucose compared to plasma is seen in:
a) Hyperglycemia
b) Hypoglycemia
c) Meningitis
d) None
Explanation: CSF glucose reflects plasma levels. In hyperglycemia, CSF glucose increases but more slowly, keeping the ratio near normal. Answer: a) Hyperglycemia
--- Guessed Question 6
CSF/plasma glucose ratio helps primarily in diagnosis of:
a) Epilepsy
b) Stroke
c) Meningitis
d) Brain tumor
Explanation: The ratio is mainly valuable in meningitis. Bacterial and TB meningitis reduce the ratio, while viral meningitis keeps it normal. Answer: c) Meningitis
--- Guessed Question 7
A lumbar puncture shows CSF glucose 20 mg/dl, plasma glucose 100 mg/dl. What is the CSF/plasma ratio?
a) 0.2
b) 0.5
c) 0.8
d) 1.2
Explanation: Ratio = 20/100 = 0.2, which is abnormally low. This strongly suggests bacterial or TB meningitis. Answer: a) 0.2
--- Guessed Question 8
In which condition is CSF glucose usually normal?
a) Viral meningitis
b) Bacterial meningitis
c) TB meningitis
d) Fungal meningitis
Explanation: Viral meningitis maintains normal CSF glucose, unlike bacterial, TB, or fungal meningitis which lower glucose levels. Answer: a) Viral meningitis
--- Guessed Question 9
Which barrier controls glucose entry into CSF?
a) Meninges
b) Blood-brain barrier
c) Pia mater
d) Dural venous sinuses
Explanation: The blood-brain barrier regulates glucose entry into CSF by selective transport, ensuring stable CSF composition. Answer: b) Blood-brain barrier
--- Guessed Question 10
A patient with suspected bacterial meningitis has a CSF/plasma ratio of 0.3. What additional CSF finding is expected?
a) Low protein
b) Neutrophilic pleocytosis
c) Eosinophilia
d) Normal cell count
Explanation: Bacterial meningitis typically shows low glucose, high protein, and neutrophilic pleocytosis. This triad is highly diagnostic. Answer: b) Neutrophilic pleocytosis
Chapter: Central Nervous System | Topic: Visual Recognition | Subtopic: Face Perception & Agnosias
Keywords
Prosopagnosia — inability to recognize familiar faces despite intact vision.
Fusiform face area (FFA) — region in inferior temporal cortex specialized for face recognition.
Associative visual agnosia — failure to assign meaning to perceived objects despite intact perception.
Apperceptive agnosia — impaired object perception (shape/form) with intact elementary vision.
Right occipitotemporal cortex — often dominant for facial recognition tasks.
Visual associative cortex — links visual percepts to memory and meaning.
Alexia — loss of reading; can be visual-associative if cortex involved.
Topographic agnosia — inability to recognize familiar places/landmarks.
CFD (capgras) — delusional misidentification where face is recognized but person is believed to be impostor.
Face processing stream — ventral occipitotemporal pathway for object/face identification.
Lead Question - 2012
Pt. is able to recognise person by name but not by face. Lesion is in ?
a) Post parietal region
b) Occipital
c) Frontal lobe
d) Temporal lobe
Explanation: Inability to recognize faces (prosopagnosia) with preserved verbal identification indicates damage to the face-processing region — the fusiform face area in the inferior temporal (occipitotemporal) cortex, typically the right temporal lobe. Thus the correct answer is (d) Temporal lobe. This spares name retrieval via language networks.
Q2. Classic prosopagnosia most commonly results from lesion in which area?
a) Dorsal parietal cortex
b) Fusiform gyrus (inferior temporal)
c) Primary visual cortex (V1)
d) Broca’s area
Explanation: Acquired prosopagnosia typically follows lesions in the fusiform gyrus (inferior temporal/occipitotemporal region), especially on the right. The FFA processes holistic face information; damage disrupts face identity recognition while leaving basic vision and language intact. Correct answer: (b) Fusiform gyrus.
Q3. A patient sees an object but cannot name it though can describe its use. This suggests:
a) Apperceptive visual agnosia
b) Associative visual agnosia
c) Cortical blindness
d) Visual neglect
Explanation: Associative visual agnosia occurs when perceptual processing is adequate but the link to semantic knowledge is disrupted, so patients can describe object use but cannot name it. This localizes to higher-order ventral stream cortical areas. Correct answer: (b) Associative visual agnosia.
Q4. Which deficit indicates right occipitotemporal dysfunction rather than primary visual loss?
a) Hemianopia with macular sparing
b) Prosopagnosia with normal acuity
c) Complete blindness
d) Visual field neglect
Explanation: Prosopagnosia with preserved visual acuity and fields suggests cortical processing impairment in the right occipitotemporal region (FFA) rather than primary visual cortex damage. Hemianopia indicates V1 lesions; neglect implicates parietal cortex. Correct answer: (b).
Q5. Developmental (congenital) prosopagnosia is best characterized by:
a) Later-life stroke causing face blindness
b) Lifelong difficulty recognizing faces with normal IQ
c) Visual acuity loss from birth
d) Progressive dementia
Explanation: Developmental prosopagnosia presents from early life as a selective deficit in face recognition despite normal vision, intelligence, and no structural lesion. Patients rely on voice or context. Correct answer: (b). It reflects functional differences in face-processing networks.
Q6. Capgras syndrome differs from prosopagnosia because patients:
a) Cannot see faces
b) Recognize faces but believe they are impostors
c) Name faces accurately
d) Have primary visual loss
Explanation: In Capgras delusion, patients perceive faces and may name them but lack the normal affective familiarity, leading to belief that a known person is an impostor — a disconnection between recognition and emotional response. Correct answer: (b).
Q7. Which test best assesses prosopagnosia clinically?
a) Visual acuity chart
b) Benton Facial Recognition Test
c) Pupillary light reflex
d) Snellen chart
Explanation: The Benton Facial Recognition Test evaluates face perception and matching without requiring naming, useful to detect prosopagnosia. Visual acuity tests do not assess identity recognition. Correct answer: (b). Neuropsychological testing localizes processing deficits.
Q8. A lesion producing alexia without agraphia involves the visual word form area and typically spares:
a) Right inferior temporal lobe
b) Language output areas (e.g., Broca’s)
c) Primary visual cortex exclusively
d) Auditory comprehension
Explanation: Alexia without agraphia results from left occipitotemporal (visual word form area) lesions with intact language cortex, so patients can write but not read. It demonstrates modality-specific visual-associative deficits. Correct answer: (b).
Q9. Which hemisphere is more commonly dominant for face recognition in most right-handed people?
a) Left hemisphere
b) Right hemisphere
c) Both equally
d) Brainstem
Explanation: The right occipitotemporal region is typically dominant for holistic face processing in right-handed individuals; right-sided lesions more often produce prosopagnosia. Left-sided lesions can impair aspects of facial recognition but less commonly. Correct answer: (b) Right hemisphere.
Q10. Which rehabilitation strategy may help a patient with prosopagnosia?
a) Visual acuity correction only
b) Training to use non-face cues (voice, gait, context)
c) Surgical removal of fusiform gyrus
d) High-dose steroids
Explanation: Compensation training teaches reliance on non-facial cues (voice, clothing, context, unique features) to identify people, improving functioning despite persistent cortical deficit. There is no surgical or steroid cure for most acquired prosopagnosia. Correct answer: (b).
Q11. In a patient who recognizes names but not faces, which imaging finding is most likely?
a) Infarct in right inferior occipitotemporal cortex
b) Bilateral frontal lobe atrophy
c) Lesion of primary visual cortex
d) Left cerebellar infarct
Explanation: MRI showing a focal lesion or infarct in the right inferior occipitotemporal (fusiform) cortex fits classical acquired prosopagnosia with preserved language-based name recognition. Primary V1 lesions cause field defects, while frontal or cerebellar lesions produce different syndromes. Correct answer: (a).
Chapter: Autonomic Nervous System | Topic: Peripheral Autonomic Ganglia | Subtopic: Sympathetic Ganglia
Keywords
Sympathetic ganglia — clusters of postganglionic neuronal cell bodies in the sympathetic chain and prevertebral plexuses.
Multipolar neuron — neuron with one axon and multiple dendrites; typical of autonomic ganglia.
Preganglionic fiber — myelinated B fiber from thoracolumbar spinal cord that synapses in ganglia.
Postganglionic fiber — unmyelinated C fiber projecting to effector organs.
Neurotransmitters — acetylcholine (preganglionic); norepinephrine (postganglionic) for most sympathetic targets.
Paravertebral chain — bilateral sympathetic trunk alongside vertebral column.
Prevertebral ganglia — celiac, superior and inferior mesenteric ganglia supplying abdominal viscera.
Visceral reflexes — autonomic reflex arcs involving pre- and postganglionic neurons.
Chromaffin cells — adrenal medulla cells acting like sympathetic postganglionic neurons (release catecholamines into blood).
Autonomic dysreflexia — exaggerated sympathetic response seen with high spinal cord injury.
Lead Question - 2012
Neurons in sympathetic ganglia are ?
a) Unipolar
b) Bipolar
c) Pseudounipolar
d) Multipolar
Explanation: Sympathetic (autonomic) ganglia contain multipolar neurons with several dendrites and a single axon that receive preganglionic cholinergic input. These neurons form synapses within the ganglion and project postganglionic fibers to effectors. Therefore the correct answer is d) Multipolar.
Q1. Preganglionic sympathetic fibers originate from which spinal segments?
a) Cervical only
b) Thoracolumbar (T1–L2)
c) Sacral only
d) Craniosacral
Explanation: Preganglionic sympathetic neurons arise from the intermediolateral cell column of the spinal cord segments T1–L2 (thoracolumbar outflow). These fibers synapse in paravertebral or prevertebral ganglia. Correct answer: b) Thoracolumbar (T1–L2).
Q2. Postganglionic sympathetic fibers predominantly release which neurotransmitter at effector organs?
a) Acetylcholine
b) Norepinephrine
c) Dopamine
d) GABA
Explanation: Most sympathetic postganglionic neurons release norepinephrine acting on α and β receptors at target organs. Exceptions include sweat glands (sympathetic cholinergic) and adrenal medulla (releases epinephrine/norepinephrine into blood). Correct answer: b) Norepinephrine.
Q3. Which ganglia form the sympathetic chain alongside the vertebral column?
a) Paravertebral ganglia
b) Prevertebral ganglia
c) Dorsal root ganglia
d) Cranial parasympathetic ganglia
Explanation: Paravertebral (sympathetic chain) ganglia lie bilaterally along the vertebral column and connect segmentally. They mediate sympathetic distribution to body wall and limbs. Prevertebral ganglia are anterior near abdominal vessels. Correct answer: a) Paravertebral ganglia.
Q4. The adrenal medulla acts like a sympathetic ganglion because its chromaffin cells:
a) Are derived from neural crest and release catecholamines into blood
b) Contain multipolar neurons with axons
c) Release acetylcholine at distant organs
d) Form synapses with skeletal muscle
Explanation: Adrenal medullary chromaffin cells are neural-crest-derived and respond to preganglionic cholinergic input by secreting epinephrine and norepinephrine into the circulation, functioning as endocrine equivalents of postganglionic sympathetic neurons. Correct answer: a).
Q5. Which of the following is a feature of sympathetic ganglia histology?
a) Presence of synaptic boutons between ganglionic neurons
b) Pseudounipolar neuronal soma
c) No satellite cells
d) Myelinated postganglionic fibers only
Explanation: Sympathetic ganglia display multipolar neuronal somata receiving preganglionic synapses; satellite cells are present. Postganglionic fibers are typically unmyelinated. The distinguishing feature is intraganglionic synapses; correct answer: a) Presence of synaptic boutons between ganglionic neurons.
Q6. A lesion of the sympathetic chain at T1 (Horner’s syndrome) causes which features ipsilaterally?
a) Ptosis, miosis, anhidrosis
b) Mydriasis and hyperhidrosis
c) Flaccid paralysis
d) Loss of taste
Explanation: Interruption of sympathetic outflow to the face produces Horner’s syndrome: ipsilateral ptosis (levator palpebrae dysfunction), miosis (unopposed parasympathetic), and anhidrosis. Correct answer: a) Ptosis, miosis, anhidrosis.
Q7. Which embryologic origin is shared by sympathetic ganglion neurons?
a) Neural tube
b) Neural crest
c) Endoderm
d) Mesoderm
Explanation: Sympathetic ganglion neurons originate from migrating neural crest cells that differentiate into peripheral neurons and glia. Neural tube gives rise to CNS structures. Correct answer: b) Neural crest.
Q8. Which receptor type predominates on vascular smooth muscle mediating sympathetic vasoconstriction?
a) β1-adrenergic
b) α1-adrenergic
c) Muscarinic M2
d) Nicotinic
Explanation: Sympathetic vasoconstriction is mediated mainly by norepinephrine acting on α1-adrenergic receptors on vascular smooth muscle, causing increased intracellular calcium and contraction. Correct answer: b) α1-adrenergic.
Q9. Which pharmacologic agent blocks transmission at autonomic ganglia (both sympathetic and parasympathetic)?
a) Atropine
b) Hexamethonium
c) Propranolol
d) Phenylephrine
Explanation: Ganglionic blockers like hexamethonium antagonize nicotinic receptors at autonomic ganglia, interrupting both sympathetic and parasympathetic transmission. Atropine blocks muscarinic receptors at effector sites. Correct answer: b) Hexamethonium.
Q10. Which statement about sympathetic postganglionic fibers is correct?
a) They are myelinated and fast conducting
b) They are unmyelinated and form varicosities over targets
c) They synapse onto skeletal muscle endplates
d) They release GABA at target organs
Explanation: Postganglionic sympathetic fibers are typically thin, unmyelinated C fibers that form en passant varicosities along target tissues, releasing neurotransmitter diffusely. They do not innervate skeletal muscle motor endplates. Correct answer: b).
Q11. Which clinical condition results from excessive sympathetic activity causing sustained vasoconstriction?
a) Orthostatic hypotension
b) Raynaud’s phenomenon
c) Myasthenia gravis
d) Guillain–Barré syndrome
Explanation: Raynaud’s phenomenon involves exaggerated sympathetic-mediated vasoconstriction of digital arterioles in response to cold or stress, causing pallor and ischemia. Orthostatic hypotension is due to inadequate sympathetic compensation. Correct answer: b) Raynaud’s phenomenon.
Chapter: Central Nervous System
Topic: Sensory Pathways
Subtopic: Dorsal Root Ganglion (DRG) Neurons
Keywords
- Dorsal root ganglion (DRG): Cluster of primary sensory neuron cell bodies in intervertebral foramina.
- Pseudounipolar neuron: Single process bifurcating into peripheral and central branches; soma is not in the conduction path.
- Satellite (capsular) cells: Glia enveloping DRG neuronal soma, providing metabolic support.
- Neural crest: Embryologic origin of DRG neurons and satellite cells/Schwann cells.
- Lipofuscin: Yellow-brown wear-and-tear pigment accumulating with age in long-lived neurons.
- Nissl substance: Rough endoplasmic reticulum in neuronal soma/dendrites; disperses during chromatolysis.
- T-junction: DRG axonal bifurcation reducing ectopic transmission; important in sensory signaling.
- Aδ and C fibers: Small-diameter nociceptive fibers with somata in DRG (sharp and dull pain).
- Herpes zoster: Varicella-zoster virus latency/reactivation in DRG causing dermatomal rash and pain.
- Autonomic ganglion: Multipolar neurons with synapses; contrasts with DRG which lacks intraganglionic synapses.
Lead Question – 2012
All the following features are seen in neurons from dorsal root ganglia, EXCEPT:
a) They are multipolar
b) They contain lipofuscin granules
c) They have centrally located nuclei
d) They are derived from neural crest cells
Explanation: DRG neurons are characteristically pseudounipolar (not multipolar), with large round soma, centrally located nuclei, prominent nucleoli, frequent lipofuscin, and neural crest origin. They lack synapses within the ganglion and are wrapped by satellite cells. Therefore, the false statement is a) They are multipolar.
Guessed Questions
1) The typical morphological type of a DRG neuron is:
a) Multipolar
b) Bipolar
c) Pseudounipolar
d) Pyramidal
Explanation: Primary sensory neurons in DRG are pseudounipolar with a single process that splits into peripheral and central branches at a T-junction. This design allows rapid transmission without synapsing on the soma. Correct answer: c) Pseudounipolar. Multipolar neurons are characteristic of autonomic ganglia and many CNS nuclei.
2) The glial cells that directly envelope DRG neuronal somata are:
a) Oligodendrocytes
b) Astrocytes
c) Satellite (capsular) cells
d) Microglia
Explanation: Satellite (capsular) cells form a continuous sheath around DRG neuronal soma, regulating the microenvironment and participating in pain modulation. Oligodendrocytes myelinate CNS axons, while Schwann cells (not listed) myelinate PNS axons. Correct answer: c) Satellite (capsular) cells.
3) A biopsy from a paraspinal ganglion shows large neurons with central nuclei, Nissl substance, lipofuscin, and no synapses between neurons. The structure is:
a) Sympathetic chain ganglion
b) Dorsal root ganglion
c) Ciliary ganglion
d) Enteric plexus
Explanation: Lack of intraganglionic synapses with centrally placed nuclei favors DRG. Autonomic ganglia (sympathetic/parasympathetic) contain multipolar neurons receiving synapses. DRG neurons are sensory and pseudounipolar. Correct answer: b) Dorsal root ganglion.
4) Embryologic origin of DRG neurons is:
a) Neural tube
b) Notochord
c) Neural crest
d) Mesoderm
Explanation: DRG neurons, Schwann cells, and satellite cells arise from neural crest, which migrates from the dorsal neural tube to form peripheral sensory ganglia. The neural tube forms CNS neurons and glia. Correct answer: c) Neural crest.
5) After transection of a peripheral branch of a DRG neuron, the soma shows chromatolysis. Which change is most typical?
a) Nuclear hyperchromasia and centralization
b) Nissl dispersal with eccentric nucleus and cell body swelling
c) Condensed Nissl with shrunken soma
d) Apoptotic bodies immediately
Explanation: Chromatolysis features dissolution of Nissl substance, cell body swelling, and eccentric displacement of the nucleus due to upregulated protein synthesis for axonal repair. Correct answer: b) Nissl dispersal with eccentric nucleus and cell body swelling.
6) Sharp, well-localized first pain from a pinprick is carried by fibers whose cell bodies lie in the DRG. These fibers are:
a) Aβ fibers
b) Aδ fibers
c) C fibers
d) Ia spindle afferents
Explanation: Aδ fibers are small, thinly myelinated afferents mediating fast, sharp pain and cold. Their somata reside in DRG. C fibers mediate slow, dull pain; Aβ carry touch/vibration; Ia are muscle spindle afferents. Correct answer: b) Aδ fibers.
7) A 65-year-old with dermatomal vesicular rash and burning pain over T6 likely has reactivation of varicella-zoster virus in the:
a) Ventral horn
b) Dorsal root ganglion
c) Sympathetic chain ganglion
d) Spinal cord dorsal column
Explanation: Herpes zoster lies dormant in DRG neurons and reactivates to cause dermatomal neuritis and rash. This localizes to the sensory ganglion at the affected level. Correct answer: b) Dorsal root ganglion.
8) Myelination of the peripheral process of a DRG neuron is performed by:
a) Oligodendrocytes
b) Schwann cells
c) Astrocytes
d) Microglia
Explanation: In the peripheral nervous system, Schwann cells myelinate individual axonal internodes, including the peripheral branch of DRG neurons. Oligodendrocytes myelinate CNS axons (multiple internodes). Correct answer: b) Schwann cells.
9) Functionally, the DRG neuron’s soma primarily:
a) Actively conducts action potentials
b) Serves trophic and metabolic roles while the impulse bypasses the soma
c) Generates synaptic potentials with neighboring DRG neurons
d) Integrates dendritic inputs from spinal interneurons
Explanation: In pseudounipolar neurons, the action potential travels from peripheral to central process across a T-junction, largely bypassing the soma. The soma provides metabolic support; DRG lacks interneuronal synapses. Correct answer: b) Serves trophic and metabolic roles….
10) Which feature best distinguishes a sympathetic ganglion from a DRG histologically?
a) Presence of satellite cells
b) Multipolar neurons receiving synapses within the ganglion
c) Central nuclei in neurons
d) Lipofuscin granules
Explanation: Autonomic (sympathetic) ganglia contain multipolar neurons receiving preganglionic synapses; DRG neurons are pseudounipolar with no intraganglionic synapses. Both have satellite cells and may show lipofuscin. Nuclei are often eccentric in autonomic ganglia. Correct answer: b) Multipolar neurons receiving synapses within the ganglion.
11) A patient with radicular pain from L5 nerve root compression has primary sensory neuron cell bodies located in the:
a) Dorsal horn lamina II
b) Dorsal root ganglion at L5
c) Ventral root
d) Sympathetic chain at L5
Explanation: Primary sensory neuron somata reside in the DRG at the corresponding spinal level (here, L5). Their central processes enter the dorsal root to synapse in dorsal horn or ascend in dorsal columns. Correct answer: b) Dorsal root ganglion at L5.
Chapter: Central Nervous System | Topic: Motor System | Subtopic: Motor Planning & Postural Set
Keywords
Supplementary motor area (SMA) — medial frontal region involved in planning, initiating complex and bimanual movements and setting postural tone.
Premotor cortex — lateral frontal area organizing movements in response to external cues and sensory guidance.
Primary motor cortex (M1) — executes voluntary movements; somatotopically organized.
Postural set — anticipatory adjustment of posture before voluntary movement to maintain balance.
Motor planning — preparation and sequencing of movement components prior to execution.
Apraxia — impairment of learned purposeful movements despite intact strength and comprehension.
Basal ganglia — selection/gating of motor programs and modulation of movement initiation.
Cerebellum — timing, coordination, and adaptive control of movements and posture.
Feedforward control — anticipatory motor adjustments based on predicted outcomes.
Pharmacologic/lesion effects — focal lesions produce specific deficits in planning or execution.
Lead Question - 2012
Setting posture before planned movement ?
a) Premotor cortex
b) Motor cortex
c) Frontal
d) Supplementary motor cortex
Explanation: The supplementary motor area (SMA) is primarily responsible for internally generated movement planning and anticipatory postural adjustments (postural set) before voluntary actions, especially for bimanual and sequential tasks. Lesions cause impaired initiation and postural preparation. Correct answer: d) Supplementary motor cortex.
Q2. A patient cannot initiate a self-paced sequence of movements but moves normally to external cues. Which area is likely affected?
a) Primary motor cortex
b) Premotor cortex
c) Supplementary motor area
d) Somatosensory cortex
Explanation: SMA lesions impair internally generated, self-initiated sequences while externally cued movements remain relatively preserved due to premotor circuits. Patients show akinesia or difficulty initiating learned sequences. Correct answer: c) Supplementary motor area. Rehabilitation uses external cues to bypass SMA deficits.
Q3. Which structure provides timing and coordination for anticipatory postural adjustments?
a) Cerebellum
b) Broca’s area
c) Hippocampus
d) Occipital lobe
Explanation: The cerebellum refines timing and coordination of both movement execution and anticipatory postural adjustments by integrating sensory inputs and motor plans. Cerebellar lesions produce dysmetria and impaired postural control. Correct answer: a) Cerebellum. It works with SMA for smooth movement onset.
Q4. A lesion in the lateral premotor cortex most likely impairs:
a) Internally-generated bimanual sequencing
b) Response to external sensory cues
c) Primary muscle strength
d) Long-term memory
Explanation: The lateral premotor area is important for selecting and preparing movements guided by external sensory cues. Lesions reduce cue-driven responses and impair reaching based on visual instructions. Correct answer: b) Response to external sensory cues; strength (M1) and memory are not primarily affected.
Q5. Which clinical sign suggests SMA dysfunction?
a) Contralateral spastic weakness only
b) Difficulty performing learned sequences without cues
c) Pure sensory loss
d) Visual field defect
Explanation: SMA damage causes difficulty initiating learned motor sequences and impaired bimanual coordination; actions may be performed with external prompting. While M1 lesions cause weakness, SMA lesions mainly affect initiation and sequencing. Correct answer: b). Assess by asking patient to perform self-initiated tasks.
Q6. Deep brain structures that gate motor programs and influence postural set include:
a) Basal ganglia
b) Optic chiasm
c) Medulla oblongata only
d) Corpus callosum
Explanation: Basal ganglia circuits select and facilitate appropriate motor programs and modulate posture and initiation. Dysfunction causes bradykinesia, rigidity, and impaired preparatory postural adjustments. Correct answer: a) Basal ganglia. They interact with SMA and premotor areas for smooth motor control.
Q7. Which test assesses anticipatory postural adjustments related to SMA function?
a) Ask patient to raise one arm quickly while observing weight shift
b) Visual acuity chart
c) Pure tone audiometry
d) Deep tendon reflex testing
Explanation: Observing compensatory weight shift when a patient rapidly raises an arm evaluates anticipatory postural adjustments. SMA dysfunction yields inadequate preparatory shifts and imbalance. Correct answer: a). This bedside test reveals impaired feedforward control of posture.
Q8. A patient with SMA lesion may exhibit which of the following during bimanual tasks?
a) Improved coordination
b) Apraxia for learned bimanual sequences
c) Enhanced reflexes only
d) Loss of primary sensation
Explanation: SMA lesions impair planning and coordination of bimanual and sequential movements, causing apraxia for learned tasks. Reflexes and primary sensation are not the primary deficits. Correct answer: b). Rehabilitation focuses on externally cued retraining to bypass SMA.
Q9. Which neurotransmitter system in basal ganglia influences movement initiation that complements SMA function?
a) Dopaminergic
b) Cholinergic only
c) GABAergic only
d) Serotonergic only
Explanation: Dopaminergic input from the substantia nigra modulates basal ganglia output and facilitates movement initiation, interacting with SMA planning. Dopamine deficiency (Parkinson’s) causes impaired initiation and reduced anticipatory postural adjustments. Correct answer: a) Dopaminergic.
Q10. Lesion affecting the supplementary motor area bilaterally may cause:
a) Difficulty initiating voluntary movements (akinetic mutism)
b) Pure sensory loss
c) Hyperreflexia only
d) Visual agnosia
Explanation: Bilateral SMA damage can cause severe akinesia and reduced spontaneous movement, sometimes akinetic mutism, and impaired postural preparation. Sensory and visual functions are spared. Correct answer: a). Management may include dopaminergic and rehabilitative strategies.
Q11. Which cortical area is mainly responsible for execution of a precise voluntary finger movement?
a) Primary motor cortex (M1)
b) Supplementary motor area
c) Occipital cortex
d) Broca’s area
Explanation: Primary motor cortex (M1) contains the final corticospinal output neurons that execute fine, precise voluntary movements such as individual finger actions. SMA plans and primes posture; M1 performs the actual contraction. Correct answer: a) Primary motor cortex (M1).
Chapter: Central Nervous System | Topic: Somatosensory System | Subtopic: Cortical Sensory Functions
Keywords
Primary somatosensory cortex — postcentral gyrus; processes discriminative touch and proprioception.
Stereognosis — recognition of objects by touch; cortical function.
Two-point discrimination — spatial acuity mediated by S1.
Graphesthesia — recognizing writing on skin; cortical sensory test.
Astereognosis — inability to identify objects by touch; indicates cortical lesion.
Thalamus — relay station for somatic sensory pathways to cortex.
Spinothalamic tract — transmits pain and temperature to thalamus/cortex.
Proprioception — limb position sense; dorsal columns and cortex.
Neglect — parietal lobe dysfunction causing inattention to one side.
Localization — identifying site of tactile stimulus; a cortical discriminative ability.
Lead Question - 2012
Somatosensory cortex lesion will cause ?
a) Pain
b) Temperature
c) Localization
d) Vibration
Explanation: The primary somatosensory cortex is essential for discriminative touch tasks such as localization, two-point discrimination, stereognosis and graphesthesia. A cortical lesion impairs the ability to localize stimuli despite preserved basic pain and temperature sensation. Correct answer is c) Localization.
Q2. A patient cannot identify an object by touch but can feel it; this sign is called:
a) Anosmia
b) Astereognosis
c) Hemianopia
d) Agraphia
Explanation: Astereognosis is the inability to recognize objects by touch despite intact primary sensation, indicating contralateral parietal or postcentral gyrus dysfunction. It reflects impaired cortical processing of tactile information. Correct answer: b) Astereognosis. This helps localize lesions to somatosensory cortex.
Q3. Loss of two-point discrimination on the right hand suggests lesion in:
a) Left primary somatosensory cortex
b) Right dorsal column
c) Left cerebellum
d) Peripheral nerve only
Explanation: Two-point discrimination is a cortical function processed in the contralateral primary somatosensory cortex. Loss on the right hand indicates a lesion in the left postcentral gyrus. Peripheral lesions can reduce sensation but cortical loss specifically impairs spatial discrimination. Correct answer: a).
Q4. In a pure postcentral gyrus infarct, which sensation is most likely preserved?
a) Stereognosis
b) Pain perception
c) Two-point discrimination
d) Graphesthesia
Explanation: Basic pain perception often remains because spinothalamic pathways reach thalamus and brainstem centers; however discriminative tasks (stereognosis, two-point discrimination, graphesthesia) require cortical processing and are impaired. Thus pain perception may be relatively preserved; correct answer: b).
Q5. A lesion of the secondary somatosensory cortex most affects:
a) Conscious pain detection
b) Integration and recognition of complex tactile patterns
c) Spinal reflexes
d) Muscle tone
Explanation: Secondary somatosensory cortex integrates tactile information for higher functions like texture discrimination and object recognition. Lesions impair complex tactile perception but not basic detection. Correct answer: b). Clinical tests include stereognosis and graphesthesia to detect such deficits.
Q6. Hemineglect (inattention to one side) most commonly results from lesion in:
a) Dominant parietal lobe
b) Nondominant parietal lobe
c) Occipital lobe
d) Brainstem
Explanation: Hemineglect is typically due to damage of the nondominant (usually right) parietal cortex, causing inattention to the contralateral side. It affects awareness more than primary sensation. Correct answer: b). This is tested by cancellation tasks and sensory stimulation.
Q7. Graphesthesia testing assesses which function?
a) Pain localization
b) Ability to recognize writing on skin
c) Vibration sense
d) Proprioception
Explanation: Graphesthesia is recognizing letters or numbers traced on the skin and depends on intact cortical sensory areas. Loss suggests parietal lobe dysfunction. Correct answer: b). It complements stereognosis and two-point discrimination in cortical assessment.
Q8. A thalamic stroke most commonly causes which sensory deficit?
a) Pure motor weakness only
b) Contralateral hemisensory loss including pain and temperature
c) Ipsilateral loss of vibration only
d) Loss of smell
Explanation: The thalamus relays most somatic sensory modalities to cortex; a thalamic stroke causes contralateral hemisensory loss affecting pain, temperature, touch and proprioception. Correct answer: b). Thalamic pain syndrome can follow with chronic dysesthesia.
Q9. Dissociation of pain and touch (lost pain but preserved touch) indicates lesion in:
a) Dorsal columns
b) Spinothalamic tract
c) Peripheral nerves only
d) Primary motor cortex
Explanation: Loss of pain and temperature with preserved dorsal column modalities suggests spinal cord lesion affecting the spinothalamic tract (e.g., syringomyelia). Correct answer: b). Cortical lesions typically affect discriminative touch rather than abolish pain selectively.
Q10. A patient with cortical sensory loss will most likely fail which bedside test?
a) Reflex hammer tendon reflex
b) Two-point discrimination
c) Muscle strength testing
d) Pupillary light reflex
Explanation: Two-point discrimination depends on cortical sensory processing; cortical lesions impair this test. Tendon reflexes and muscle strength reflect spinal and motor pathways. Pupillary reflex is brainstem mediated. Correct answer: b). This helps distinguish cortical from peripheral sensory loss.
Q11. Which sign suggests parietal lobe (cortical) sensory dysfunction rather than peripheral neuropathy?
a) Stocking-glove numbness
b) Astereognosis
c) Diminished ankle reflex
d) Burning pain in feet
Explanation: Astereognosis (inability to recognize objects by touch) indicates cortical parietal dysfunction. Peripheral neuropathies produce distal symmetric sensory loss and reflex changes. Correct answer: b). Identifying astereognosis localizes lesion to somatosensory cortex.
Chapter: Central Nervous System
Topic: Basal Ganglia
Subtopic: Striatum and Memory Functions
Keywords
- Striatum: Part of basal ganglia involved in motor control and habit learning.
- Procedural memory: Memory for skills and habits, dependent on basal ganglia.
- Explicit memory: Conscious recall of facts and events, mediated by hippocampus.
- Short-term memory: Temporary information storage lasting seconds to minutes.
- Long-term memory: Stable memory stored for days to years.
- Parkinsonism: Clinical syndrome due to basal ganglia dysfunction.
Lead Question – 2012
Striatum damage affects primarily?
a) Procedural memory
b) Short term memory
c) Long term memory
d) Explicit memory
Explanation: The striatum, part of the basal ganglia, regulates procedural memory linked to skills and habits. Unlike hippocampus-driven explicit memory, striatal injury impairs motor learning such as riding a bicycle or typing, while short- and long-term declarative memories remain intact. Correct answer: (a) Procedural memory.
Guessed Questions
1) A Parkinson’s patient struggles with buttoning his shirt. Which memory type is impaired?
a) Procedural memory
b) Explicit memory
c) Short term memory
d) Episodic memory
Explanation: Parkinson’s disease impairs basal ganglia circuits, particularly the striatum, which controls learned motor skills. Patients lose smooth execution of habitual actions. Declarative and episodic recall remain intact. Correct answer: (a) Procedural memory.
2) A stroke involving the hippocampus mainly affects:
a) Habit learning
b) Procedural skills
c) Explicit memory
d) Reflex conditioning
Explanation: The hippocampus is essential for consolidation of declarative memories, including facts and events. Stroke injury here produces severe anterograde amnesia for explicit memory, but motor habits and reflex learning remain intact. Correct answer: (c) Explicit memory.
3) Which memory remains unaffected in striatal lesions?
a) Motor habits
b) Procedural skills
c) Explicit memory
d) Skill learning
Explanation: Striatal lesions disrupt motor habits and procedural learning. Explicit memory, mediated by hippocampus and temporal cortex, stays preserved in such patients. They can recall events but struggle with habit execution. Correct answer: (c) Explicit memory.
4) A patient can recall his wedding day but cannot play the piano anymore after basal ganglia injury. This reflects loss of:
a) Explicit memory
b) Procedural memory
c) Short term memory
d) Semantic memory
Explanation: Piano playing is a learned skill relying on procedural memory, dependent on striatal circuits. Episodic recall, like remembering wedding details, involves hippocampus and remains unaffected. Correct answer: (b) Procedural memory.
5) In Huntington’s disease, degeneration of the striatum causes early:
a) Loss of semantic memory
b) Loss of episodic recall
c) Impaired procedural memory
d) Impaired short-term storage
Explanation: Huntington’s disease selectively damages the caudate and putamen (striatum). Patients show impaired motor learning and skill execution due to loss of procedural memory, while semantic and episodic memory are relatively intact early. Correct answer: (c) Impaired procedural memory.
6) Procedural memory is best tested by:
a) Word recall
b) Mirror tracing task
c) Object naming
d) Sentence repetition
Explanation: Procedural memory is assessed through tasks requiring skill learning, such as mirror tracing or rotary pursuit. Word recall and object naming depend on explicit memory and language centers. Correct answer: (b) Mirror tracing task.
7) A patient retains ability to solve arithmetic problems but forgets new motor skills after basal ganglia injury. Which is intact?
a) Procedural memory
b) Habit learning
c) Explicit memory
d) Reflex motor learning
Explanation: Arithmetic skills and fact-based knowledge depend on explicit memory stored in the cortex and hippocampus. These remain intact despite striatal injury. Procedural learning is impaired. Correct answer: (c) Explicit memory.
8) Which brain structure primarily encodes explicit long-term memory?
a) Striatum
b) Hippocampus
c) Amygdala
d) Cerebellum
Explanation: The hippocampus consolidates explicit long-term memory such as facts and events. The striatum mediates procedural memory, the amygdala processes emotional memory, and the cerebellum handles motor coordination. Correct answer: (b) Hippocampus.
9) Striatal injury spares which of the following?
a) Habit learning
b) Skill acquisition
c) Declarative recall
d) Motor sequence learning
Explanation: Declarative recall (explicit memory) is dependent on hippocampal circuits and not the striatum. Therefore, patients with striatal lesions can still recall facts but fail to perform acquired habits. Correct answer: (c) Declarative recall.
10) A medical student learns to perform venipuncture. Which memory system is reinforced?
a) Procedural memory
b) Short term memory
c) Semantic memory
d) Episodic memory
Explanation: Skill-based learning such as venipuncture is stored in procedural memory, relying on striatal and cerebellar circuits. Over time, repetition strengthens habit execution independent of conscious recall. Correct answer: (a) Procedural memory.
11) A patient with amnesia can still play the guitar skillfully. Which memory is preserved?
a) Explicit memory
b) Procedural memory
c) Episodic memory
d) Semantic memory
Explanation: Amnesia typically impairs explicit (declarative) memory while sparing procedural memory. Thus, the patient cannot recall past events but continues to perform learned motor skills such as guitar playing. Correct answer: (b) Procedural memory.
Chapter: Neuroanatomy
Topic: Cerebellum
Subtopic: Cerebellar Cortex
Keywords:
Cerebellar cortex: Three-layered structure of cerebellum responsible for motor coordination.
Molecular layer: Outer layer containing stellate and basket cells.
Purkinje cells: Principal output neurons of cerebellar cortex.
Granule cells: Smallest excitatory neurons forming parallel fibers.
Golgi cells: Inhibitory interneurons regulating granule cell activity.
Clinical relevance: Damage to cerebellar cortex causes ataxia, tremor, and dysmetria.
Lead Question - 2012
What are the cellular contents of cerebellar cortex?
a) Cortical cells
b) Glomus cells
c) Principle cells
d) Intercalated cells
Explanation: The cerebellar cortex has three layers containing five neuron types: Purkinje cells, granule cells, basket cells, stellate cells, and Golgi cells. Purkinje cells are the principal output neurons. Correct answer is Principle cells referring to Purkinje cells, essential for inhibitory output to deep cerebellar nuclei.
Guessed Question 1
Which is the only excitatory neuron of the cerebellar cortex?
a) Purkinje cells
b) Granule cells
c) Basket cells
d) Golgi cells
Explanation: Among cerebellar neurons, granule cells are the only excitatory type, releasing glutamate. Purkinje, basket, stellate, and Golgi cells are inhibitory GABAergic neurons. Correct answer is Granule cells.
Guessed Question 2
Purkinje cells exert their effect through:
a) Excitatory input to thalamus
b) Inhibitory output to deep cerebellar nuclei
c) Excitatory connections with cortex
d) Inhibitory effect on spinal cord
Explanation: Purkinje cells are GABAergic and provide the sole output from the cerebellar cortex, inhibiting deep cerebellar nuclei. This helps fine-tune motor control. Correct answer is Inhibitory output to deep cerebellar nuclei.
Guessed Question 3
Which cells in molecular layer form synapses on Purkinje cell dendrites?
a) Basket and stellate cells
b) Granule cells
c) Golgi cells
d) Glial cells
Explanation: The molecular layer contains basket and stellate cells which are inhibitory interneurons. They synapse on Purkinje cell dendrites and modulate their activity. Correct answer is Basket and stellate cells.
Guessed Question 4
Which neurotransmitter is released by Purkinje cells?
a) Glutamate
b) GABA
c) Dopamine
d) Glycine
Explanation: Purkinje cells are large inhibitory neurons of cerebellum releasing GABA. This inhibition regulates output from deep cerebellar nuclei and maintains motor coordination. Correct answer is GABA.
Guessed Question 5
Damage to Purkinje cells results in:
a) Ataxia
b) Rigidity
c) Hemiballismus
d) Tremor at rest
Explanation: Purkinje cell loss impairs inhibitory regulation, causing cerebellar ataxia with uncoordinated gait, intention tremor, and dysmetria. Unlike Parkinsonism, rigidity and resting tremor are absent. Correct answer is Ataxia.
Guessed Question 6
Which layer of cerebellar cortex contains Purkinje cell bodies?
a) Molecular layer
b) Purkinje cell layer
c) Granular layer
d) White matter
Explanation: Purkinje cells lie in a single row between the molecular and granular layers, forming the Purkinje cell layer. Their dendrites extend into molecular layer. Correct answer is Purkinje cell layer.
Guessed Question 7
Golgi cells provide inhibitory input to:
a) Purkinje cells
b) Basket cells
c) Granule cells
d) Stellate cells
Explanation: Golgi cells are inhibitory interneurons located in the granular layer. They regulate granule cell activity through inhibitory synapses, modulating parallel fiber output. Correct answer is Granule cells.
Guessed Question 8
Which afferent fibers excite Purkinje cells via climbing fibers?
a) Mossy fibers
b) Corticospinal fibers
c) Vestibulospinal fibers
d) Olivocerebellar fibers
Explanation: Climbing fibers arise from inferior olivary nucleus and directly synapse on Purkinje cell dendrites with strong excitatory input. Mossy fibers excite granule cells instead. Correct answer is Olivocerebellar fibers.
Guessed Question 9
Mossy fibers primarily synapse with:
a) Purkinje cells
b) Granule cells
c) Golgi cells
d) Stellate cells
Explanation: Mossy fibers form excitatory synapses with granule cells in the cerebellar granular layer. These granule cells then send parallel fibers to excite Purkinje cells indirectly. Correct answer is Granule cells.
Guessed Question 10
Clinical feature of midline cerebellar lesion (vermis) is:
a) Truncal ataxia
b) Intention tremor
c) Hemiballismus
d) Resting tremor
Explanation: Lesions in vermis affect axial muscles leading to truncal ataxia, wide-based gait, and imbalance. Intention tremor is seen in hemispheric lesions. Correct answer is Truncal ataxia.
Chapter: Neuroanatomy
Topic: Cerebral Hemispheres
Subtopic: Corpus Callosum
Keywords:
Corpus callosum: Largest commissural fiber tract connecting right and left cerebral hemispheres.
Hemispheric connection: Corpus callosum unites the two hemispheres for integration.
Frontal lobe connection: Anterior part (genu) connects the two frontal lobes.
Commissural fibers: White matter tracts connecting similar cortical areas of both hemispheres.
Split-brain syndrome: Condition after corpus callosotomy leading to loss of hemispheric integration.
Lead Question - 2012
True about Corpus callosum :
a) Unite far area of two sides of brain
b) Connect two frontal lobe
c) Unite two hemisphere
d) All
Explanation: Corpus callosum is the major commissural bundle uniting the two hemispheres. It connects frontal lobes (via genu), parietal lobes (body), and occipital lobes (splenium). It enables interhemispheric communication. Hence, all the given statements are true, making the correct answer All.
Guessed Question 1
A lesion in the corpus callosum can lead to:
a) Split-brain syndrome
b) Horner’s syndrome
c) Parkinsonism
d) Hemiballismus
Explanation: Lesions in corpus callosum disrupt interhemispheric communication leading to split-brain syndrome. Patients may fail to name objects in the left visual field due to disconnection between hemispheres. Correct answer is Split-brain syndrome.
Guessed Question 2
Anterior part of corpus callosum (genu) primarily connects:
a) Occipital lobes
b) Frontal lobes
c) Temporal lobes
d) Thalamus
Explanation: The genu of corpus callosum bends forward and connects the two frontal lobes, enabling coordinated executive and motor function across hemispheres. Correct answer is Frontal lobes.
Guessed Question 3
The splenium of the corpus callosum connects:
a) Frontal lobes
b) Parietal lobes
c) Occipital lobes
d) Temporal lobes
Explanation: The posterior part of corpus callosum is the splenium, which connects occipital lobes and allows integration of visual information between hemispheres. Correct answer is Occipital lobes.
Guessed Question 4
Which imaging modality best visualizes corpus callosum abnormalities?
a) CT scan
b) MRI
c) Ultrasound
d) PET scan
Explanation: MRI is the most sensitive imaging modality for detecting corpus callosum malformations, agenesis, or demyelination. It clearly shows its structure and fiber connections. Correct answer is MRI.
Guessed Question 5
Congenital absence of corpus callosum is associated with:
a) Agenesis syndromes
b) Huntington’s disease
c) Amyotrophic lateral sclerosis
d) Multiple sclerosis
Explanation: Agenesis of corpus callosum is a congenital malformation, often linked with developmental delay, seizures, and midline anomalies. Correct answer is Agenesis syndromes.
Guessed Question 6
Which part of corpus callosum connects parietal lobes?
a) Genu
b) Body
c) Splenium
d) Rostrum
Explanation: The central part or body of corpus callosum mainly connects parietal lobes, allowing somatosensory integration across hemispheres. Correct answer is Body.
Guessed Question 7
Damage to corpus callosum may impair:
a) Interhemispheric communication
b) Reflex arcs
c) Spinal cord conduction
d) Basal ganglia circuits
Explanation: Corpus callosum is essential for communication between hemispheres. Its damage leads to disconnection syndromes, affecting coordinated tasks involving both sides of the body. Correct answer is Interhemispheric communication.
Guessed Question 8
Which condition is treated by partial corpus callosotomy?
a) Intractable epilepsy
b) Multiple sclerosis
c) Meningitis
d) Brain abscess
Explanation: In refractory epilepsy, partial callosotomy prevents seizure spread between hemispheres, reducing drop attacks. Correct answer is Intractable epilepsy.
Guessed Question 9
Which fibers are carried by corpus callosum?
a) Commissural fibers
b) Association fibers
c) Projection fibers
d) Reticular fibers
Explanation: Corpus callosum carries commissural fibers connecting identical cortical areas of both hemispheres. Association fibers connect areas within the same hemisphere, while projection fibers connect cortex with lower centers. Correct answer is Commissural fibers.
Guessed Question 10
In MRI, corpus callosum appears as:
a) Hypointense on T1, Hyperintense on T2
b) Hyperintense on T1, Hypointense on T2
c) Isointense on both T1 and T2
d) Variable with pathology
Explanation: On MRI, corpus callosum is typically hypointense on T1 and hyperintense on T2 due to its myelinated white matter. Signal changes vary with demyelination or agenesis. Correct answer is Hypointense on T1, Hyperintense on T2.
Chapter: Neuroanatomy
Topic: Cranial Nerves
Subtopic: Oculomotor Nerve (III)
Keywords:
Oculomotor nerve: The third cranial nerve, motor to most extraocular muscles.
Parasympathetic fibers: Carried to ciliary ganglion for pupil constriction.
Inferior oblique muscle: Supplied by oculomotor nerve, elevates eye in adduction.
Superior orbital fissure: Route by which oculomotor enters orbit.
Pupil constriction: Mediated by sphincter pupillae via oculomotor parasympathetic supply.
Lead Question - 2012
All the following are characteristics of oculomotor nerve except:
a) Carries parasympathetic nerve fibres
b) Supplies inferior oblique muscle
c) Enters orbit through the inferior orbital fissure
d) Causes constriction of pupil
Explanation: The oculomotor nerve enters the orbit through the superior orbital fissure, not the inferior orbital fissure. It supplies extraocular muscles and parasympathetics to sphincter pupillae, causing pupillary constriction. Thus, the incorrect statement is Enters orbit through the inferior orbital fissure.
Guessed Question 1
A lesion of the oculomotor nerve results in:
a) Ptosis, mydriasis, eye down and out
b) Loss of corneal reflex
c) Diplopia only in horizontal gaze
d) Isolated nystagmus
Explanation: Oculomotor palsy produces ptosis (levator palpebrae), mydriasis (parasympathetics), and eye deviation down and out (unopposed lateral rectus and superior oblique). Correct answer is Ptosis, mydriasis, eye down and out.
Guessed Question 2
Parasympathetic fibers of oculomotor nerve synapse in:
a) Ciliary ganglion
b) Pterygopalatine ganglion
c) Otic ganglion
d) Submandibular ganglion
Explanation: The oculomotor nerve carries preganglionic parasympathetics that synapse in the ciliary ganglion, from where short ciliary nerves innervate sphincter pupillae and ciliary muscle. Correct answer is Ciliary ganglion.
Guessed Question 3
A patient with diabetes develops acute third nerve palsy but pupil is spared. This suggests:
a) Compressive lesion
b) Ischemic neuropathy
c) Tumor invasion
d) Inflammatory neuritis
Explanation: In ischemic oculomotor palsy (like in diabetes), central fibers are affected while peripheral parasympathetic fibers are spared, preserving pupillary function. Compressive lesions usually affect the pupil. Correct answer is Ischemic neuropathy.
Guessed Question 4
Which extraocular muscle is NOT supplied by oculomotor nerve?
a) Superior rectus
b) Inferior rectus
c) Superior oblique
d) Inferior oblique
Explanation: Oculomotor supplies all extraocular muscles except lateral rectus (abducens) and superior oblique (trochlear). Thus, correct answer is Superior oblique.
Guessed Question 5
The nucleus of oculomotor nerve is located in:
a) Midbrain at superior colliculus level
b) Midbrain at inferior colliculus level
c) Pons
d) Medulla
Explanation: The oculomotor nucleus is located in the midbrain at the level of the superior colliculus, ventral to the aqueduct. Correct answer is Midbrain at superior colliculus level.
Guessed Question 6
A compressive aneurysm of posterior communicating artery typically causes:
a) Pupil-involving oculomotor palsy
b) Pupil-sparing oculomotor palsy
c) Isolated superior oblique weakness
d) Bilateral ptosis without diplopia
Explanation: Aneurysms compress the superficial parasympathetic fibers of oculomotor nerve, leading to early pupillary involvement along with third nerve palsy. Correct answer is Pupil-involving oculomotor palsy.
Guessed Question 7
Which clinical sign is most specific for oculomotor nerve palsy?
a) Ptosis
b) Miosis
c) Lateral gaze palsy
d) Nystagmus
Explanation: Ptosis caused by paralysis of levator palpebrae superioris is a hallmark feature of oculomotor nerve palsy, often seen with diplopia and pupil dilation. Correct answer is Ptosis.
Guessed Question 8
The Edinger–Westphal nucleus provides fibers for:
a) Accommodation and pupillary constriction
b) Extraocular muscle contraction
c) Taste perception
d) Facial sensation
Explanation: The Edinger–Westphal nucleus, part of oculomotor complex, gives parasympathetic fibers that control accommodation reflex and pupillary constriction via ciliary ganglion. Correct answer is Accommodation and pupillary constriction.
Guessed Question 9
A patient presents with eye deviated down and out with dilated pupil. The most likely cause is:
a) Oculomotor nerve palsy
b) Trochlear nerve palsy
c) Abducens nerve palsy
d) Optic nerve lesion
Explanation: Down-and-out eye with dilated pupil indicates oculomotor palsy as lateral rectus and superior oblique act unopposed. Correct answer is Oculomotor nerve palsy.
Guessed Question 10
Which clinical test best assesses oculomotor function?
a) Ask patient to look laterally
b) Check convergence and accommodation
c) Assess corneal reflex
d) Test jaw movement
Explanation: Convergence and accommodation require action of medial rectus and ciliary muscle, both supplied by oculomotor nerve. Hence, testing accommodation is an effective way to assess its function. Correct answer is Check convergence and accommodation.
Chapter: Neuroanatomy
Topic: Cerebral Cortex
Subtopic: Sulci and Gyri
Keywords:
Operculated sulcus: A sulcus covered partially by cortical opercula (frontal, parietal, temporal lobes).
Calcarine sulcus: Found in occipital lobe, related to primary visual cortex.
Collateral sulcus: Separates fusiform gyrus from parahippocampal gyrus.
Lunate sulcus: Seen in some primates, rare in humans.
Central sulcus: Separates frontal and parietal lobes.
Lead Question - 2012
Which of the following is an operculated sulcus ?
a) Calcarine
b) Collateral
c) Lunate
d) Central
Explanation: The central sulcus is an operculated sulcus, as parts of the frontal, parietal, and temporal lobes form opercula around the insula. Operculation is a covering phenomenon seen around the insula. Hence, the correct answer is Central sulcus.
Guessed Question 1
A lesion of the calcarine sulcus results in loss of:
a) Auditory perception
b) Visual field
c) Language comprehension
d) Motor function
Explanation: The calcarine sulcus contains the primary visual cortex (Brodmann area 17). A lesion leads to contralateral homonymous hemianopia. Thus, the answer is Visual field.
Guessed Question 2
The collateral sulcus is located in:
a) Occipital lobe
b) Temporal lobe
c) Parietal lobe
d) Frontal lobe
Explanation: The collateral sulcus is found in the temporal lobe, separating fusiform gyrus from parahippocampal gyrus. It plays a role in higher-order visual processing. Answer is Temporal lobe.
Guessed Question 3
Which sulcus separates motor and sensory cortices?
a) Calcarine
b) Central
c) Lunate
d) Collateral
Explanation: The central sulcus separates the precentral gyrus (motor cortex) from the postcentral gyrus (sensory cortex). This functional division is crucial in neurosurgery. Answer is Central sulcus.
Guessed Question 4
Lunate sulcus is considered a remnant of evolution, more prominent in:
a) Humans
b) Primates
c) Amphibians
d) Birds
Explanation: The lunate sulcus is more prominent in primates like apes and monkeys, rarely observed in humans. It demarcates visual areas in non-human primates. Answer is Primates.
Guessed Question 5
The insula is hidden deep to which sulcus?
a) Central
b) Lateral
c) Calcarine
d) Collateral
Explanation: The insula is buried deep within the lateral sulcus, covered by opercula of adjacent lobes. It is involved in autonomic and visceral functions. Answer is Lateral sulcus.
Guessed Question 6
Which artery supplies the region around the central sulcus?
a) Middle cerebral artery
b) Anterior cerebral artery
c) Posterior cerebral artery
d) Basilar artery
Explanation: The central sulcus is supplied by both anterior and middle cerebral arteries. The anterior cerebral supplies medial aspect, and MCA supplies lateral aspect. Answer is Anterior cerebral artery.
Guessed Question 7
A stroke involving the calcarine sulcus territory typically spares:
a) Central vision
b) Peripheral vision
c) Entire vision
d) Color vision
Explanation: A stroke involving posterior cerebral artery affecting calcarine cortex causes contralateral homonymous hemianopia with macular sparing, as macular area has dual supply. Answer is Central vision.
Guessed Question 8
The collateral sulcus is most closely associated with:
a) Olfactory processing
b) Visual memory
c) Auditory signals
d) Motor control
Explanation: The collateral sulcus is related to the fusiform and parahippocampal gyri, involved in recognition and visual memory processes. Answer is Visual memory.
Guessed Question 9
The central sulcus is also called:
a) Fissure of Rolando
b) Fissure of Sylvius
c) Fissure of Broca
d) Fissure of Wernicke
Explanation: The central sulcus is known as the fissure of Rolando, a landmark separating frontal and parietal lobes. Answer is Fissure of Rolando.
Guessed Question 10
Which sulcus lies in the occipital lobe and is crucial for vision?
a) Central
b) Calcarine
c) Collateral
d) Sylvian
Explanation: The calcarine sulcus in the occipital lobe contains the primary visual cortex. It is essential for processing vision. Answer is Calcarine sulcus.
Chapter: Neuroanatomy
Topic: Dorsal Column Nuclei & Medial Lemniscus Pathway
Subtopic: Nucleus Gracilis/Cuneatus (aka “fasciculate” nuclei), Sensory Decussation & Clinical Correlates
Keyword Definitions
Fasciculus gracilis — Medial dorsal column carrying fine touch, vibration, proprioception from lower limb/trunk.
Fasciculus cuneatus — Lateral dorsal column carrying similar modalities from upper limb/upper trunk.
Nucleus gracilis — Relay nucleus in caudal medulla for fasciculus gracilis; gives internal arcuate fibers.
Nucleus cuneatus — Relay nucleus in caudal medulla for fasciculus cuneatus; projects to medial lemniscus.
Internal arcuate fibers — Axons from dorsal column nuclei that decussate in caudal medulla (sensory decussation).
Medial lemniscus — Ascending tract after decussation to VPL thalamus conveying touch, vibration, proprioception.
Accessory cuneate nucleus — Lateral to cuneate; origin of cuneocerebellar tract (ipsilateral proprioception to cerebellum).
Proprioception — Sense of joint position/movement; tested by joint position sense, Romberg.
Romberg test — Instability with eyes closed suggests proprioceptive deficit (e.g., dorsal column disease).
Tabes dorsalis — Neurosyphilis causing dorsal column degeneration; impaired vibration/position sense, positive Romberg.
Lead Question - 2012
Nucleus fasciculatus is seen in?
a) Frontal lobe
b) Medulla
c) Temporal lobe
d) Midbrain
Explanation: The dorsal column nuclei (gracilis and cuneatus—sometimes collectively termed “fasciculate” nuclei) lie in the caudal medulla. They receive input from the fasciculi gracilis and cuneatus and send internal arcuate fibers that decussate to form the medial lemniscus. Answer: b) Medulla.
1. Primary sensory modality conveyed by dorsal columns includes:
a) Pain and temperature
b) Fine touch, vibration, proprioception
c) Crude touch only
d) Auditory input
Explanation: Dorsal columns (gracilis, cuneatus) transmit fine touch, vibration, and proprioception. Pain and temperature ascend in the spinothalamic tract. Lesions produce loss of vibration/position sense and positive Romberg sign. Answer: b) Fine touch, vibration, proprioception.
2. Site of sensory decussation for the dorsal column–medial lemniscus pathway:
a) Spinal cord anterior white commissure
b) Caudal medulla (internal arcuate fibers)
c) Pons tegmentum
d) Midbrain colliculi
Explanation: Second-order neurons from nucleus gracilis and cuneatus cross midline as internal arcuate fibers in the caudal medulla, forming the medial lemniscus. Spinothalamic fibers decussate in the spinal cord. Answer: b) Caudal medulla (internal arcuate fibers).
3. The medial lemniscus terminates primarily in:
a) VPL nucleus of thalamus
b) VPM nucleus of thalamus
c) Pulvinar
d) LGB (lateral geniculate body)
Explanation: The medial lemniscus carrying body sensation projects to the VPL thalamus. Face tactile sensation via trigeminal lemniscus reaches VPM. From VPL, third-order neurons ascend to primary somatosensory cortex. Answer: a) VPL nucleus of thalamus.
4. A lesion limited to fasciculus gracilis causes deficits most prominent in the:
a) Ipsilateral upper limb
b) Ipsilateral lower limb
c) Contralateral upper limb
d) Contralateral lower limb
Explanation: Fasciculus gracilis carries input from the ipsilateral lower limb and lower trunk (below T6). Spinal cord dorsal column lesions are ipsilateral to deficits because crossing occurs in the medulla, not the cord. Answer: b) Ipsilateral lower limb.
5. Accessory cuneate nucleus projects to cerebellum via the:
a) Dorsal spinocerebellar tract
b) Cuneocerebellar tract
c) Ventral spinocerebellar tract
d) Rubrospinal tract
Explanation: Proprioceptive input from the upper limb reaches the accessory cuneate nucleus and ascends ipsilaterally via the cuneocerebellar tract to the inferior cerebellar peduncle, informing cerebellar coordination. Answer: b) Cuneocerebellar tract.
6. Clinical: A patient with sensory ataxia, positive Romberg, and impaired vibration in feet most likely has pathology in:
a) Lateral spinothalamic tract
b) Dorsal columns
c) Corticospinal tract only
d) Vestibular nuclei
Explanation: Loss of vibration/position sense with positive Romberg points to dorsal column disease (e.g., B12 deficiency, tabes dorsalis). Spinothalamic lesions cause pain/temperature loss instead. Answer: b) Dorsal columns.
7. In the caudal medulla, the gracile and cuneate tubercles on dorsal surface correspond to:
a) Inferior olivary nuclei
b) Nucleus gracilis and nucleus cuneatus
c) Hypoglossal nucleus only
d) Spinal trigeminal nucleus
Explanation: The dorsal elevations, gracile and cuneate tubercles, overlie the respective dorsal column nuclei, landmarks for the site where internal arcuate fibers originate. Answer: b) Nucleus gracilis and nucleus cuneatus.
8. Stroke of VPL thalamus most characteristically produces:
a) Ipsilateral facial analgesia
b) Contralateral loss of body vibration and proprioception
c) Ipsilateral limb ataxia from cerebellar lesion
d) Bilateral anosmia
Explanation: VPL relays contralateral body somatosensation (including dorsal column modalities). VPM serves face; cerebellar lesions cause ipsilateral ataxia via peduncles. Thus VPL stroke → contralateral tactile/vibration deficits. Answer: b) Contralateral loss of body vibration and proprioception.
9. Subacute combined degeneration (B12 deficiency) initially affects:
a) Anterior horn cells only
b) Dorsal columns and lateral corticospinal tracts
c) Medial longitudinal fasciculus
d) Optic radiations
Explanation: B12 deficiency causes demyelination of dorsal columns (sensory ataxia) and lateral corticospinal tracts (UMN signs). Early recognition prevents irreversible deficits. Answer: b) Dorsal columns and lateral corticospinal tracts.
10. A hemisection (Brown-Séquard) at T10 produces which dorsal column deficit below the lesion?
a) Contralateral loss of vibration/proprioception
b) Ipsilateral loss of vibration/proprioception
c) Bilateral pain loss only
d) No sensory deficit
Explanation: Dorsal column fibers ascend ipsilaterally and cross in the medulla; thus spinal hemisection causes ipsilateral loss of vibration and proprioception below the lesion, with contralateral pain/temperature loss (spinothalamic). Answer: b) Ipsilateral loss of vibration/proprioception.
11. Graphesthesia (recognizing numbers traced on skin) mainly tests integrity of:
a) Dorsal column–medial lemniscus pathway and cortical processing
b) Spinothalamic tract only
c) Vestibulospinal tract
d) Rubrospinal tract
Explanation: Graphesthesia requires intact dorsal column–medial lemniscus input to somatosensory cortex and association areas. Lesions in dorsal columns or parietal cortex impair stereognosis/graphesthesia despite normal strength. Answer: a) Dorsal column–medial lemniscus pathway and cortical processing.
Chapter: Neurophysiology
Topic: Nerve Action Potentials & Nerve Conduction
Subtopic: Compound (Biphasic) Action Potential of Mixed Nerve
Keyword Definitions
Compound action potential (CAP) — Sum of many axons firing; extracellular recording from a mixed nerve; amplitude is graded.
Biphasic recording — Surface electrodes record a negative then positive deflection as the wave passes between and beyond them.
Monophasic recording — One electrode over injured/isoelectric region and one active; produces single main deflection.
All-or-none law — Property of single axons and muscle fibers; not of CAP, which is graded with recruitment.
Refractory period — Time after an AP when axons cannot (absolute) or need stronger stimuli (relative) to fire.
Mixed nerve — Contains myelinated and unmyelinated, motor and sensory fibers with different velocities and thresholds.
Recruitment — Progressive activation of more/larger fibers as stimulus strength increases, enlarging CAP.
Conduction velocity — Distance/time of AP propagation; higher in large, myelinated fibers (saltatory conduction).
Stimulus artifact — Brief deflection from the stimulus itself that precedes the true CAP.
Rheobase/Chronaxie — Minimal current for long duration (rheobase); and duration at 2× rheobase (chronaxie) describing excitability.
Lead Question - 2012
Biphasic action potential of mixed nerve except?
a) All or none phenomenon
b) Two or more positive peaks
c) Refractory period
d) Recorded on surface
Explanation: A compound (biphasic) action potential from a mixed nerve is recorded on the surface and often shows multiple peaks from groups of fibers with differing velocities. Although individual axons obey refractoriness, the CAP as a whole is graded and not all-or-none. Answer: a) All or none phenomenon.
1. CAP amplitude increases with stimulus strength primarily due to:
a) Bigger APs in each axon
b) Recruitment of additional axons
c) Shorter refractory period
d) Lower extracellular resistance
Explanation: Single axon AP size is constant (all-or-none). Increasing stimulus strength excites more axons (especially larger, lower-threshold myelinated fibers first), summating extracellularly to a larger CAP. This is recruitment, not bigger unit APs. Answer: b) Recruitment of additional axons.
2. Multiple peaks in a CAP reflect:
a) Temporal dispersion of fiber groups
b) Alternating depolarization and repolarization of one axon
c) Electrode artifact only
d) Hyperkalemia
Explanation: Mixed nerves contain fast large myelinated and slower small fibers. Their different conduction velocities cause temporal dispersion, producing distinct CAP peaks (e.g., Aα/β then Aδ). It is physiological, not merely artifact. Answer: a) Temporal dispersion of fiber groups.
3. Clinical: In demyelinating neuropathy, a motor nerve study most likely shows:
a) Increased conduction velocity
b) Temporal dispersion and conduction block
c) Higher CAP amplitude with faster latency
d) Normal distal latency
Explanation: Demyelination slows propagation, broadens the CAP (temporal dispersion), and may cause conduction block with reduced amplitude across segments. Distal latency is prolonged; velocity is reduced. Answer: b) Temporal dispersion and conduction block.
4. An extracellular biphasic CAP turns monophasic if:
a) Stimulus intensity is halved
b) One recording electrode is placed over an inactive/lesioned segment
c) Temperature increases
d) Ground electrode is removed
Explanation: Making one electrode indifferent (over electrically silent tissue) converts the recording to monophasic, producing a single main deflection. This is a classic teaching-lab maneuver (crush or ischemic block under one electrode). Answer: b) One recording electrode is placed over an inactive/lesioned segment.
5. Which statement about refractory period in nerve is TRUE?
a) CAP has no refractory behavior at any interval
b) Individual axons have absolute and relative refractory periods
c) Refractoriness depends only on extracellular K+
d) Refractory period occurs only in muscle
Explanation: Each axon shows absolute then relative refractoriness due to Na+ channel inactivation and K+ efflux. CAP refractoriness is less obvious because different axons recover at different times but can be demonstrated with double stimuli. Answer: b) Individual axons have absolute and relative refractory periods.
6. Local anesthetics reduce CAP amplitude by:
a) Blocking voltage-gated Na+ channels
b) Opening Cl− channels
c) Stimulating Na+/K+ pump
d) Inhibiting Ca2+ release from SR
Explanation: Local anesthetics (e.g., lignocaine) bind and block voltage-gated Na+ channels preferentially in small, myelinated pain fibers first, reducing the number of conducting axons and hence CAP amplitude and pain transmission. Answer: a) Blocking voltage-gated Na+ channels.
7. Strength–duration curve: a low chronaxie indicates:
a) Low tissue excitability
b) High tissue excitability
c) Only demyelination
d) Only temperature effect
Explanation: Chronaxie is the pulse duration needed at twice rheobase to excite tissue. Lower chronaxie signifies greater excitability (e.g., large myelinated axons). It is used to compare nerve/muscle excitability in disease and rehabilitation. Answer: b) High tissue excitability.
8. In a nerve conduction lab trace, the earliest deflection after the stimulus is most likely:
a) CAP negative peak
b) Stimulus artifact
c) F-wave
d) H-reflex
Explanation: The stimulus artifact is a brief, non-physiologic deflection due to the stimulus pulse that precedes the true CAP. F-waves and H-reflexes are later, long-loop responses. Recognizing artifact avoids misinterpretation. Answer: b) Stimulus artifact.
9. Cooling a nerve segment will:
a) Increase conduction velocity
b) Decrease conduction velocity and broaden CAP
c) Abolish refractory period
d) Convert biphasic to monophasic
Explanation: Lower temperature slows ion channel kinetics, reducing conduction velocity and increasing temporal dispersion, broadening the CAP. Extreme cooling can block conduction; it does not inherently change recording polarity. Answer: b) Decrease conduction velocity and broaden CAP.
10. Clinical: A patient with acute carpal tunnel shows reduced median motor CAP amplitude distally. This most likely reflects:
a) Primary demyelination only
b) Axonal loss or conduction block at the carpal tunnel
c) Pure muscle disease
d) Increased recruitment
Explanation: Reduced distal CMAP amplitude suggests fewer functioning axons reaching the muscle due to axonal loss or focal conduction block across the entrapment. Demyelination alone typically slows and prolongs latency with relative amplitude preservation. Answer: b) Axonal loss or conduction block at the carpal tunnel.
11. Antidromic sensory study records a larger CAP when:
a) The distance is shorter and fibers are well myelinated
b) The distance is longer only
c) Stimulation is subthreshold
d) Recording is across a joint in extreme flexion
Explanation: Shorter distances reduce temporal dispersion; intact myelination preserves synchrony, yielding larger sensory CAPs. Subthreshold stimulation fails to recruit fibers; joint extremes can compress nerves and reduce amplitude. Answer: a) The distance is shorter and fibers are well myelinated.
Chapter: Neuroanatomy
Topic: Cranial Nerves
Subtopic: Accessory Nerve
Keyword Definitions:
Accessory Nerve: Cranial nerve XI, with spinal and cranial parts.
Cranial Part: Joins vagus nerve to supply palatal, pharyngeal, and laryngeal muscles.
Tensor Veli Palatini: Supplied by mandibular nerve, not accessory nerve.
Palatoglossus: Supplied by cranial part of accessory via vagus.
Palatopharyngeus: Receives motor supply from accessory via vagus.
Tensor Veli Tympani: Supplied by mandibular nerve branch.
Lead Question – 2012
Cranial part of accessory nerve supplies all palatal muscles, EXCEPT?
a) Palatoglossus
b) Palatopharyngeus
c) Tensor veli palatini
d) Tensor veli tympani
Explanation: The cranial part of the accessory nerve joins the vagus nerve to supply most palatal muscles except tensor veli palatini, which is supplied by the mandibular division of the trigeminal nerve. Correct answer: c) Tensor veli palatini.
Question 2. A patient presents with nasal regurgitation after surgery. The most likely injured muscle supplied by cranial accessory nerve is?
a) Palatoglossus
b) Tensor veli palatini
c) Palatopharyngeus
d) Stylopharyngeus
Explanation: Palatopharyngeus elevates the pharynx and closes the nasopharynx during swallowing. Injury leads to nasal regurgitation. Correct answer: c) Palatopharyngeus.
Question 3. Which palatal muscle prevents food from entering the nasopharynx?
a) Tensor veli palatini
b) Palatopharyngeus
c) Palatoglossus
d) Levator veli palatini
Explanation: Levator veli palatini elevates the soft palate to prevent food from entering the nasopharynx during swallowing. Correct answer: d) Levator veli palatini.
Question 4. Which nerve supplies the stylopharyngeus muscle?
a) Glossopharyngeal
b) Accessory
c) Vagus
d) Mandibular
Explanation: Stylopharyngeus is the only muscle supplied by glossopharyngeal nerve (CN IX). Correct answer: a) Glossopharyngeal.
Question 5. In lesion of cranial accessory nerve, which symptom is seen?
a) Hoarseness
b) Tongue deviation
c) Shoulder droop
d) Loss of gag reflex
Explanation: Lesion affects muscles of larynx and pharynx via vagus, leading to hoarseness and dysphagia. Correct answer: a) Hoarseness.
Question 6. A tumor compressing jugular foramen damages which nerves?
a) IX, X, XI
b) VII, IX, XI
c) IX, X, XII
d) X, XI, XII
Explanation: The jugular foramen transmits glossopharyngeal, vagus, and accessory nerves. Correct answer: a) IX, X, XI.
Question 7. A 45-year-old patient has palatal droop and uvula deviation to the right. The lesion is on?
a) Right accessory nerve
b) Left vagus nerve
c) Right glossopharyngeal nerve
d) Left hypoglossal nerve
Explanation: Uvula deviates to the opposite side of lesion due to vagus involvement. Correct answer: b) Left vagus nerve.
Question 8. Which palatal muscle is supplied by mandibular nerve?
a) Tensor veli palatini
b) Levator veli palatini
c) Palatoglossus
d) Palatopharyngeus
Explanation: Tensor veli palatini is the only palatal muscle supplied by mandibular nerve (V3). Correct answer: a) Tensor veli palatini.
Question 9. Injury to spinal accessory nerve leads to weakness of?
a) Sternocleidomastoid and trapezius
b) Masseter and temporalis
c) Levator scapulae and rhomboid
d) Palatoglossus and tensor veli palatini
Explanation: The spinal part of accessory nerve supplies sternocleidomastoid and trapezius. Injury causes shoulder droop and weak head rotation. Correct answer: a) Sternocleidomastoid and trapezius.
Question 10. Which nerve provides motor supply to intrinsic laryngeal muscles (except cricothyroid)?
a) External branch of superior laryngeal nerve
b) Recurrent laryngeal nerve
c) Glossopharyngeal nerve
d) Spinal accessory nerve
Explanation: Recurrent laryngeal nerve, branch of vagus, supplies all intrinsic laryngeal muscles except cricothyroid. Correct answer: b) Recurrent laryngeal nerve.
Question 11. A patient with vagus nerve lesion develops aspiration during swallowing. Which muscle is most affected?
a) Palatopharyngeus
b) Cricopharyngeus
c) Tensor veli palatini
d) Stylopharyngeus
Explanation: Cricopharyngeus (upper esophageal sphincter) controlled by vagus is impaired, leading to aspiration. Correct answer: b) Cricopharyngeus.
Chapter: Head and Neck Anatomy
Topic: Autonomic Nervous System
Subtopic: Otic Ganglion – Location and Relations
Keyword Definitions
Otic ganglion: A parasympathetic ganglion located in the infratemporal fossa, associated with glossopharyngeal nerve.
Foramen ovale: Opening in sphenoid bone transmitting mandibular nerve.
Tensor veli palatini: Muscle of soft palate, medial to otic ganglion.
Mandibular nerve: Main trunk passes lateral to otic ganglion.
Middle meningeal artery: Artery from 1st part of maxillary artery, medial to otic ganglion.
Auriculotemporal nerve: Branch of mandibular nerve carrying secretomotor fibers from otic ganglion to parotid gland.
Glossopharyngeal nerve: Provides preganglionic parasympathetic fibers via lesser petrosal nerve to otic ganglion.
Parasympathetic ganglion: Collection of postganglionic neurons controlling glandular secretion.
Lead Question - 2012
All of the following are true about location of otic ganglia except:
a) Inferior to foramen ovale
b) Lateral to tensor veli palatini
c) Lateral to mandibular nerve
d) Anterior to middle meningeal artery
Explanation: The otic ganglion lies just inferior to foramen ovale, medial to mandibular nerve, and lateral to tensor veli palatini. It is medial, not anterior, to middle meningeal artery. Therefore, the incorrect statement is d) Anterior to middle meningeal artery.
Guessed Question 1
Preganglionic fibers reaching otic ganglion are derived from:
a) Auriculotemporal nerve
b) Chorda tympani
c) Lesser petrosal nerve
d) Facial nerve
Explanation: Preganglionic fibers from glossopharyngeal nerve travel via tympanic branch and lesser petrosal nerve to synapse at otic ganglion. Postganglionic fibers reach parotid gland through auriculotemporal nerve. Correct answer: c) Lesser petrosal nerve.
Guessed Question 2
Secretomotor fibers to parotid gland are carried by:
a) Lingual nerve
b) Buccal nerve
c) Auriculotemporal nerve
d) Inferior alveolar nerve
Explanation: Postganglionic parasympathetic fibers from otic ganglion are carried to the parotid gland by auriculotemporal nerve, a branch of mandibular nerve. Correct answer: c) Auriculotemporal nerve.
Guessed Question 3
A lesion in lesser petrosal nerve would affect secretion of:
a) Sublingual gland
b) Lacrimal gland
c) Submandibular gland
d) Parotid gland
Explanation: Lesser petrosal nerve carries parasympathetic fibers from glossopharyngeal nerve to otic ganglion, essential for parotid gland secretion. Lesion results in dry mouth due to parotid dysfunction. Correct answer: d) Parotid gland.
Guessed Question 4
The otic ganglion is functionally associated with:
a) Facial nerve
b) Hypoglossal nerve
c) Glossopharyngeal nerve
d) Vagus nerve
Explanation: Although anatomically attached to mandibular nerve, the otic ganglion is functionally linked with glossopharyngeal nerve via lesser petrosal branch. Correct answer: c) Glossopharyngeal nerve.
Guessed Question 5
Middle meningeal artery lies in relation to otic ganglion as:
a) Lateral
b) Medial
c) Anterior
d) Posterior
Explanation: The otic ganglion is located medial to the mandibular nerve and lateral to tensor veli palatini, with the middle meningeal artery lying medial to it. Correct answer: b) Medial.
Guessed Question 6
Which nerve carries postganglionic sympathetic fibers through otic ganglion without synapse?
a) Lesser petrosal nerve
b) Glossopharyngeal nerve
c) Auriculotemporal nerve
d) Facial nerve
Explanation: Sympathetic fibers from external carotid plexus pass through otic ganglion without synapsing and reach parotid gland via auriculotemporal nerve. Correct answer: c) Auriculotemporal nerve.
Guessed Question 7
The otic ganglion is located in which fossa?
a) Pterygopalatine fossa
b) Parapharyngeal space
c) Infratemporal fossa
d) Temporal fossa
Explanation: The otic ganglion lies in the infratemporal fossa, just below foramen ovale, closely related to mandibular nerve. Correct answer: c) Infratemporal fossa.
Guessed Question 8
Damage to auriculotemporal nerve after otic ganglion lesion would impair:
a) Lacrimal gland secretion
b) Parotid gland secretion
c) Submandibular gland secretion
d) Sublingual gland secretion
Explanation: Postganglionic fibers from otic ganglion to parotid are transmitted by auriculotemporal nerve. Injury leads to reduced parotid secretion. Correct answer: b) Parotid gland secretion.
Guessed Question 9
Which muscle lies medial to otic ganglion?
a) Masseter
b) Buccinator
c) Tensor veli palatini
d) Temporalis
Explanation: The otic ganglion lies lateral to tensor veli palatini muscle, which forms part of its medial relation. Correct answer: c) Tensor veli palatini.
Guessed Question 10
Which foramen transmits lesser petrosal nerve to otic ganglion?
a) Foramen rotundum
b) Foramen spinosum
c) Foramen ovale
d) Jugular foramen
Explanation: The lesser petrosal nerve passes through foramen ovale to reach otic ganglion in the infratemporal fossa. Correct answer: c) Foramen ovale.
Chapter: Neuroanatomy
Topic: Brainstem nuclei
Subtopic: Nucleus ambiguus
Keyword Definitions
Nucleus ambiguus: Motor nucleus in medulla supplying muscles of pharynx, larynx, and soft palate via CN IX, X, XI.
Cranial nerve IX (Glossopharyngeal): Provides motor fibers to stylopharyngeus and contributes to pharyngeal plexus.
Cranial nerve X (Vagus): Provides motor innervation to laryngeal and pharyngeal muscles.
Cranial nerve XI (Accessory): Cranial root joins vagus to supply pharyngeal and laryngeal muscles.
Cranial nerve XII (Hypoglossal): Purely motor, supplies intrinsic and extrinsic muscles of tongue, not nucleus ambiguus.
Lead Question - 2012
Nucleus ambiguus is not associated with which cranial nerve:
a) X
b) XI
c) IX
d) XII
Explanation: The nucleus ambiguus provides motor innervation via cranial nerves IX, X, and cranial part of XI, controlling swallowing and phonation. Cranial nerve XII (hypoglossal) arises from a separate hypoglossal nucleus, supplying tongue muscles. Hence, the correct answer is d) XII.
Guessed Question 1
A lesion of nucleus ambiguus leads to:
a) Loss of taste
b) Dysphagia
c) Loss of tongue movement
d) Loss of vision
Explanation: Nucleus ambiguus damage causes dysphagia due to paralysis of pharyngeal muscles. Taste is controlled by nucleus solitarius, and tongue movement by hypoglossal nucleus. Therefore, the correct answer is b) Dysphagia.
Guessed Question 2
The cranial root of accessory nerve joins which cranial nerve?
a) IX
b) X
c) XI
d) XII
Explanation: The cranial root of accessory nerve merges with the vagus nerve (X) after emerging from medulla and contributes to pharyngeal and laryngeal innervation. Hence, the correct answer is b) X.
Guessed Question 3
A patient with hoarseness and difficulty swallowing most likely has a lesion affecting:
a) Hypoglossal nucleus
b) Nucleus solitarius
c) Nucleus ambiguus
d) Oculomotor nucleus
Explanation: Hoarseness and dysphagia point to pharyngeal and laryngeal muscle involvement, which are supplied via motor fibers from nucleus ambiguus. Correct answer is c) Nucleus ambiguus.
Guessed Question 4
Stylopharyngeus muscle receives motor fibers from:
a) CN IX
b) CN X
c) CN XI
d) CN XII
Explanation: Stylopharyngeus is the only muscle supplied directly by glossopharyngeal nerve (IX), receiving motor input from nucleus ambiguus. Correct answer is a) CN IX.
Guessed Question 5
Which cranial nerve nucleus is purely motor and controls tongue muscles?
a) Nucleus ambiguus
b) Hypoglossal nucleus
c) Solitary nucleus
d) Dorsal motor nucleus of vagus
Explanation: The hypoglossal nucleus (CN XII) exclusively supplies intrinsic and extrinsic tongue muscles, unlike nucleus ambiguus. Correct answer is b) Hypoglossal nucleus.
Guessed Question 6
Nucleus ambiguus contributes to which reflex?
a) Gag reflex motor limb
b) Light reflex
c) Corneal reflex
d) Pupillary accommodation reflex
Explanation: The motor limb of gag reflex is mediated by vagus via nucleus ambiguus, while the sensory limb is by glossopharyngeal. Correct answer is a) Gag reflex motor limb.
Guessed Question 7
A lesion of vagus nerve involving nucleus ambiguus may cause:
a) Tongue deviation
b) Uvula deviation
c) Jaw deviation
d) Facial palsy
Explanation: Vagus supplies soft palate muscles. Unilateral lesion causes uvula to deviate away from affected side. Correct answer is b) Uvula deviation.
Guessed Question 8
Which muscle is not innervated by nucleus ambiguus?
a) Cricothyroid
b) Pharyngeal constrictors
c) Intrinsic laryngeal muscles
d) Soft palate muscles
Explanation: Cricothyroid is supplied by external branch of superior laryngeal nerve (CN X) but motor nucleus is not nucleus ambiguus. Correct answer is a) Cricothyroid.
Guessed Question 9
Damage to nucleus ambiguus bilaterally can cause:
a) Complete anarthria and aspiration
b) Diplopia
c) Ataxia
d) Loss of smell
Explanation: Bilateral lesions result in paralysis of pharynx and larynx, leading to anarthria and aspiration pneumonia risk. Correct answer is a) Complete anarthria and aspiration.
Guessed Question 10
Motor fibers of vagus nerve arise from:
a) Nucleus solitarius
b) Nucleus ambiguus
c) Dorsal motor nucleus
d) Spinal trigeminal nucleus
Explanation: Motor fibers for pharynx, larynx, and palate in vagus nerve originate from nucleus ambiguus. Dorsal motor nucleus carries parasympathetic fibers. Correct answer is b) Nucleus ambiguus.
Chapter: Neuroanatomy
Topic: Cranial Nerves
Subtopic: Mandibular Nerve (V3) – Branches and Supply
Keyword definitions
Mandibular nerve (V3) — Third division of trigeminal nerve, mixed in function, carrying motor and sensory fibers.
Anterior division — Predominantly motor except for buccal nerve; supplies muscles of mastication (except medial pterygoid).
Posterior division — Mainly sensory, except nerve to mylohyoid which is motor.
Temporalis — Large fan-shaped muscle elevating and retracting mandible; supplied by deep temporal nerves (V3 anterior division).
Masseter — Powerful muscle for mastication; supplied by masseteric nerve (V3 anterior division).
Lateral pterygoid — Protracts mandible; supplied by nerve to lateral pterygoid (V3 anterior division).
Medial pterygoid — Elevates mandible; supplied by nerve to medial pterygoid (from main trunk of V3, not anterior division).
Buccal nerve — Sensory branch from anterior division of V3 supplying cheek mucosa and skin.
Nerve to mylohyoid — Motor branch from inferior alveolar nerve (posterior division) supplying mylohyoid and anterior digastric.
Otic ganglion — Small parasympathetic ganglion functionally related to V3, carrying fibers to parotid gland.
Lead Question - 2012
Which of the following is not supplied by the anterior division of mandibular nerve (V3)?
a) Temporalis
b) Medial pterygoid
c) Lateral pterygoid
d) Masseter
Explanation (50 words): The anterior division of V3 supplies temporalis, masseter, and lateral pterygoid muscles. The medial pterygoid, however, is supplied by the nerve to medial pterygoid, which arises directly from the main trunk of V3 before its anterior and posterior divisions. Answer: b) Medial pterygoid.
1. Which branch of the anterior division of mandibular nerve is purely sensory?
a) Deep temporal nerve
b) Buccal nerve
c) Masseteric nerve
d) Nerve to lateral pterygoid
Explanation (50 words): The buccal nerve is the only sensory branch from the anterior division of V3. It supplies sensation to the mucosa and skin of the cheek, while other branches are motor to muscles of mastication. Answer: b) Buccal nerve.
2. A patient with inability to protract the mandible likely has damage to:
a) Temporalis
b) Masseter
c) Lateral pterygoid
d) Medial pterygoid
Explanation (50 words): The lateral pterygoid is the only muscle of mastication responsible for protraction of the mandible. Damage to its nerve supply (nerve to lateral pterygoid from anterior division of V3) impairs forward movement of the jaw. Answer: c) Lateral pterygoid.
3. Which of the following muscles receives innervation from the nerve to medial pterygoid?
a) Tensor veli palatini
b) Lateral pterygoid
c) Temporalis
d) Masseter
Explanation (50 words): The nerve to medial pterygoid not only supplies the medial pterygoid muscle but also gives twigs to tensor tympani and tensor veli palatini. Answer: a) Tensor veli palatini.
4. Lesion of mandibular nerve at foramen ovale will produce all except:
a) Loss of sensation anterior 2/3 tongue (general)
b) Paralysis of temporalis
c) Loss of taste anterior 2/3 tongue
d) Paralysis of masseter
Explanation (50 words): Mandibular nerve carries general sensation but not taste from the anterior two-thirds of tongue (taste carried by chorda tympani via facial nerve). Hence taste is spared in V3 lesion. Answer: c) Loss of taste anterior 2/3 tongue.
5. The masseteric nerve passes through which structure to reach the masseter?
a) Foramen ovale
b) Mandibular notch
c) Pterygopalatine fossa
d) Infratemporal crest
Explanation (50 words): The masseteric nerve, branch of anterior division of V3, passes laterally through the mandibular notch to enter and supply the masseter muscle. Answer: b) Mandibular notch.
6. A fracture of mandibular condyle damaging the auriculotemporal nerve will cause:
a) Loss of taste
b) Loss of salivary secretion from parotid
c) Paralysis of masseter
d) Inability to protrude mandible
Explanation (50 words): Auriculotemporal nerve carries postganglionic parasympathetic secretomotor fibers from otic ganglion to the parotid gland. Injury reduces parotid secretion, not motor paralysis, as it is a sensory and secretomotor nerve. Answer: b) Loss of salivary secretion from parotid.
7. Which muscle of mastication lies in the temporal fossa and is innervated by deep temporal nerves?
a) Masseter
b) Temporalis
c) Lateral pterygoid
d) Medial pterygoid
Explanation (50 words): Temporalis is a large fan-shaped muscle in the temporal fossa, supplied by anterior and posterior deep temporal nerves from anterior division of V3. Answer: b) Temporalis.
8. Injury to lingual nerve before it is joined by chorda tympani will result in:
a) Loss of general and taste sensation
b) Loss of general sensation only
c) Loss of taste sensation only
d) Loss of salivary secretion only
Explanation (50 words): The lingual nerve carries general sensation from anterior 2/3 tongue. Before joining chorda tympani, it has no taste fibers. Therefore injury causes loss of general sensation only. Answer: b) Loss of general sensation only.
9. The mandibular nerve exits the skull through:
a) Foramen spinosum
b) Foramen ovale
c) Jugular foramen
d) Stylomastoid foramen
Explanation (50 words): The mandibular nerve passes through the foramen ovale to enter the infratemporal fossa where it divides into anterior and posterior divisions. Answer: b) Foramen ovale.
10. Which nerve supplies the mylohyoid and anterior belly of digastric?
a) Lingual nerve
b) Nerve to medial pterygoid
c) Nerve to mylohyoid
d) Buccal nerve
Explanation (50 words): The nerve to mylohyoid is a motor branch of the inferior alveolar nerve (posterior division of V3). It supplies the mylohyoid and anterior belly of digastric muscles. Answer: c) Nerve to mylohyoid.
Chapter: Upper Limb
Topic: Nerve supply of hand
Subtopic: Cutaneous innervation of palm
Keyword definitions
Median nerve — supplies lateral palm, palmar aspect of lateral 3½ fingers, and dorsal tips of the same fingers.
Ulnar nerve — supplies medial 1½ fingers and medial palm and dorsum of hand.
Radial nerve — supplies dorsum of lateral hand, but no palmar branches.
Palmar cutaneous branch — arises from median and ulnar nerves, supplying central and medial palm.
Musculocutaneous nerve — supplies lateral forearm, not palm.
Carpal tunnel — passage for median nerve and flexor tendons beneath flexor retinaculum.
Guyon’s canal — fibro-osseous tunnel for ulnar nerve and artery at wrist.
Dermatomes — C6 supplies thumb, C7 middle finger, C8 little finger.
Thenar eminence — supplied by recurrent branch of median nerve.
Hypothenar eminence — supplied by deep branch of ulnar nerve.
Lead Question - 2012
Sensory supply of the palm is from which nerves -
a) Median nerve and Radial nerve
b) Radial nerve and Ulnar nerve
c) Ulnar nerve and Median nerve
d) Musculocutaneous nerve and Radial nerve
Explanation (50 words): The palm receives sensory innervation mainly from the median and ulnar nerves. Median nerve supplies the lateral 3½ digits and central palm, while ulnar nerve supplies the medial 1½ digits and medial palm. Radial nerve contributes dorsally only. Answer: c) Ulnar nerve and Median nerve.
1. Which nerve is compressed in carpal tunnel syndrome?
a) Ulnar nerve
b) Radial nerve
c) Median nerve
d) Musculocutaneous nerve
Explanation (50 words): Carpal tunnel syndrome results from compression of the median nerve beneath the flexor retinaculum. It causes tingling, numbness, and weakness in the lateral 3½ digits and thenar muscles. Ulnar and radial nerves are unaffected at this site. Answer: c) Median nerve.
2. Sensory loss over hypothenar eminence indicates damage to?
a) Median nerve
b) Ulnar nerve
c) Radial nerve
d) Axillary nerve
Explanation (50 words): The hypothenar eminence is supplied by the superficial branch of the ulnar nerve. Injury results in sensory loss over medial palm and little finger. Median nerve affects thenar eminence, radial nerve affects dorsum, and axillary nerve supplies shoulder region. Answer: b) Ulnar nerve.
3. Loss of sensation in thumb and index finger palmar aspect indicates lesion of?
a) Median nerve
b) Radial nerve
c) Ulnar nerve
d) Musculocutaneous nerve
Explanation (50 words): The median nerve supplies palmar aspect of thumb, index, middle finger, and radial half of ring finger. A lesion, especially at the wrist, produces sensory loss in these areas. Ulnar supplies medial fingers, radial dorsum, musculocutaneous forearm. Answer: a) Median nerve.
4. Which nerve injury causes sensory loss over dorsum of first web space?
a) Median nerve
b) Radial nerve
c) Ulnar nerve
d) Axillary nerve
Explanation (50 words): The superficial branch of radial nerve supplies dorsum of the first web space. Injury causes sensory loss here without motor deficit. This finding is pathognomonic of radial nerve involvement. Median and ulnar nerves do not supply this area, axillary supplies shoulder. Answer: b) Radial nerve.
5. In claw hand, sensory loss occurs mainly over?
a) Lateral 3½ fingers
b) Medial 1½ fingers
c) Palm center
d) Dorsum first web
Explanation (50 words): Claw hand results from ulnar nerve palsy, producing sensory loss over the medial 1½ fingers and medial palm. Median nerve lesions cause ape thumb and lateral finger sensory loss. Radial lesions affect dorsum, not palm. Answer: b) Medial 1½ fingers.
6. A patient with wrist laceration near pisiform loses sensation over medial palm. Which nerve is cut?
a) Median
b) Ulnar
c) Radial
d) Musculocutaneous
Explanation (50 words): Near pisiform, the ulnar nerve enters the hand via Guyon’s canal. Laceration here damages its superficial branch, producing sensory loss over medial palm and digits. Median nerve supplies lateral palm, radial dorsum, musculocutaneous forearm. Answer: b) Ulnar.
7. Which nerve gives palmar cutaneous branch before carpal tunnel, sparing central palm in CTS?
a) Ulnar nerve
b) Radial nerve
c) Median nerve
d) Musculocutaneous nerve
Explanation (50 words): The median nerve gives off a palmar cutaneous branch proximal to the carpal tunnel. Thus, in carpal tunnel syndrome, the central palm sensation is preserved, though digits are affected. Ulnar and radial also give palmar branches, but this clinical distinction is key for median nerve. Answer: c) Median nerve.
8. A knife injury at medial wrist leads to loss of finger abduction and sensory loss of medial palm. Which nerve is injured?
a) Median
b) Ulnar
c) Radial
d) Axillary
Explanation (50 words): The ulnar nerve supplies interossei (responsible for finger abduction/adduction) and sensory supply to medial palm. Damage at wrist causes motor and sensory loss as described. Median nerve supplies thenar, radial dorsum is radial, axillary shoulder. Answer: b) Ulnar.
9. Which nerve injury leads to loss of precision grip due to loss of thenar sensation and motor supply?
a) Median nerve
b) Ulnar nerve
c) Radial nerve
d) Musculocutaneous nerve
Explanation (50 words): The median nerve supplies thenar muscles (via recurrent branch) and lateral palm. Loss of sensation and motor control causes impaired precision grip, a hallmark of median nerve injury at wrist. Ulnar affects power grip, radial wrist extension, musculocutaneous forearm. Answer: a) Median nerve.
10. Which nerve supplies both dorsal and palmar surfaces of the medial 1½ fingers?
a) Median
b) Radial
c) Ulnar
d) Axillary
Explanation (50 words): The ulnar nerve supplies sensory innervation to medial 1½ fingers on both palmar and dorsal aspects. Median nerve covers lateral fingers, radial dorsum, axillary upper arm. Clinical lesions produce characteristic sensory loss over these areas. Answer: c) Ulnar nerve.
Chapter: Neuroanatomy
Topic: Cranial Nerves
Subtopic: Facial Nerve – Chorda Tympani
Keyword Definitions:
Chorda tympani: Branch of facial nerve carrying taste and parasympathetic fibers.
Preganglionic parasympathetic: Autonomic fibers synapsing in ganglia before reaching target organs.
Postganglionic parasympathetic: Fibers arising from parasympathetic ganglia to supply target organs.
Preganglionic sympathetic: Fibers from spinal cord to sympathetic ganglia.
Postganglionic sympathetic: Fibers from sympathetic ganglia to target structures.
Submandibular ganglion: Parasympathetic ganglion controlling submandibular & sublingual glands.
Lingual nerve: Mandibular nerve branch carrying sensation & taste fibers.
Anterior two-thirds of tongue: Supplied by lingual nerve (general sensation) and chorda tympani (taste).
Facial nerve: 7th cranial nerve carrying motor, sensory, and parasympathetic fibers.
Petrotympanic fissure: Exit point of chorda tympani from middle ear.
Lead Question – 2012
1) What is true about chorda tympani?
a) Postganglionic sympathetic
b) Preganglionic sympathetic
c) Preganglionic parasympathetic
d) Postganglionic parasympathetic
Explanation: The chorda tympani carries preganglionic parasympathetic fibers from the facial nerve to the submandibular ganglion and taste fibers from the anterior two-thirds of the tongue. These parasympathetic fibers later supply submandibular and sublingual glands. Hence, the correct answer is c) Preganglionic parasympathetic.
2) A patient loses taste sensation in the anterior two-thirds of the tongue. The most likely injured structure is:
a) Glossopharyngeal nerve
b) Lingual nerve before joining chorda tympani
c) Chorda tympani
d) Hypoglossal nerve
Explanation: Taste sensation from the anterior two-thirds of the tongue is carried by the chorda tympani via the facial nerve. Loss of this function suggests chorda tympani damage. The glossopharyngeal nerve supplies posterior tongue, while hypoglossal carries only motor fibers. Correct answer: c) Chorda tympani.
3) Parasympathetic fibers from chorda tympani synapse in:
a) Otic ganglion
b) Submandibular ganglion
c) Pterygopalatine ganglion
d) Ciliary ganglion
Explanation: Chorda tympani carries preganglionic parasympathetic fibers that synapse in the submandibular ganglion. Postganglionic fibers then supply the submandibular and sublingual glands. Otic is for parotid, pterygopalatine for lacrimal/nasal, and ciliary for ocular muscles. Correct answer: b) Submandibular ganglion.
4) A lesion at the petrotympanic fissure affects which fibers?
a) General sensory
b) Taste and parasympathetic
c) Purely motor
d) Sympathetic
Explanation: The chorda tympani exits through the petrotympanic fissure, carrying taste and parasympathetic fibers. Damage here results in loss of taste (anterior 2/3 tongue) and reduced salivation. Correct answer: b) Taste and parasympathetic.
5) A patient with dry mouth due to loss of submandibular and sublingual gland secretion likely has a lesion in:
a) Auriculotemporal nerve
b) Lingual nerve after joining chorda tympani
c) Hypoglossal nerve
d) Glossopharyngeal nerve
Explanation: Chorda tympani joins the lingual nerve and carries parasympathetic fibers to submandibular ganglion. Lesion here reduces salivary secretion. Auriculotemporal is for parotid, glossopharyngeal supplies posterior tongue/parotid. Correct answer: b) Lingual nerve after joining chorda tympani.
6) Preganglionic parasympathetic fibers of chorda tympani originate from:
a) Superior salivatory nucleus
b) Inferior salivatory nucleus
c) Edinger–Westphal nucleus
d) Dorsal motor nucleus of vagus
Explanation: Preganglionic parasympathetic fibers of chorda tympani arise from the superior salivatory nucleus in the brainstem. They travel via the facial nerve to synapse in the submandibular ganglion. Correct answer: a) Superior salivatory nucleus.
7) Which nerve carries both taste fibers and parasympathetic fibers to glands?
a) Chorda tympani
b) Greater petrosal nerve
c) Auriculotemporal nerve
d) Hypoglossal nerve
Explanation: The chorda tympani uniquely carries taste (anterior 2/3 tongue) and parasympathetic fibers to submandibular/sublingual glands. Greater petrosal carries parasympathetic for lacrimal/nasal glands but no taste. Correct answer: a) Chorda tympani.
8) Which middle ear structure is closely related to the chorda tympani?
a) Stapes
b) Malleus
c) Incus
d) Tensor tympani
Explanation: The chorda tympani passes through the middle ear, running close to the medial surface of the malleus and incus before exiting. Hence, surgical damage to ossicles may injure it. Correct answer: b) Malleus.
9) A patient with facial nerve palsy sparing lacrimation but with loss of taste (anterior 2/3) has lesion distal to:
a) Geniculate ganglion
b) Stylomastoid foramen
c) Origin of greater petrosal nerve
d) Nucleus ambiguus
Explanation: Taste loss with preserved lacrimation indicates lesion distal to greater petrosal but proximal to chorda tympani origin. This localizes the lesion at/beyond the geniculate ganglion but before stylomastoid foramen. Correct answer: a) Geniculate ganglion.
10) Injury to chorda tympani in middle ear surgery causes:
a) Loss of general sensation posterior tongue
b) Loss of taste anterior tongue and reduced salivation
c) Loss of lacrimation
d) Paralysis of tongue muscles
Explanation: Middle ear surgery can damage chorda tympani, causing loss of taste (anterior 2/3) and decreased salivation from submandibular/sublingual glands. General sensation is lingual nerve, lacrimation via greater petrosal, tongue muscles via hypoglossal. Correct answer: b) Loss of taste anterior tongue and reduced salivation.
11) Which ganglion damage reproduces chorda tympani lesion effects?
a) Submandibular
b) Otic
c) Pterygopalatine
d) Ciliary
Explanation: Submandibular ganglion is the relay site for chorda tympani fibers. Damage here leads to reduced salivation and taste loss, mimicking chorda tympani lesion. Correct answer: a) Submandibular.
Keywords
* Posterior communicating artery (PCOM) — A vessel connecting the internal carotid artery to the posterior cerebral artery, part of the circle of Willis.
* Internal carotid artery (ICA) — Major intracranial artery that gives rise to the ophthalmic, posterior communicating, anterior cerebral, and middle cerebral branches.
* External carotid artery (ECA) — Supplies extracranial head and neck structures; not a primary intracranial circle of Willis branch.
* Middle cerebral artery (MCA) — Continuation of ICA supplying lateral cerebral convexity; important in stroke syndromes.
* Posterior inferior cerebellar artery (PICA) — Branch of vertebral artery supplying posteroinferior cerebellum; related to Wallenberg syndrome.
* Circle of Willis — Collateral arterial anastomotic ring at the base of the brain linking anterior and posterior circulations.
* PCOM aneurysm — Frequent site for saccular aneurysms; may compress oculomotor nerve causing ptosis and pupil changes.
* Oculomotor nerve palsy — Presents with ptosis, "down and out" eye, pupil involvement suggests compressive lesion (e.g., PCOM aneurysm).
* Subarachnoid hemorrhage (SAH) — Sudden severe headache; common presentation of ruptured intracranial saccular aneurysm including PCOM aneurysms.
* Cerebral angiography — Gold standard imaging for diagnosing aneurysms and arterial anatomy; CT angiography is commonly used as initial test.
Chapter: Neuroanatomy — Topic: Cerebral Circulation — Subtopic: Circle of Willis & Posterior Communicating Artery
Lead Question - 2012
Posterior communicating artery a branch of
a) Internal carotid
b) External carotid
c) Middle cerebral
d) Posterior superior cerebellar
Explanation & answer: The posterior communicating artery arises from the internal carotid artery and connects to the posterior cerebral artery, forming part of the circle of Willis. It is not a branch of the external carotid, MCA, or cerebellar arteries. Correct answer: (a) Internal carotid. This artery is clinically important for PCOM aneurysms and oculomotor palsy. (≈50 words)
1.A patient presents with acute third nerve palsy with pupil involvement. Which vascular lesion is most likely?
a) Posterior communicating artery aneurysm
b) Lacunar infarct in the internal capsule
c) Middle cerebral artery thrombosis
d) Superior sagittal sinus thrombosis
Explanation & answer: A compressive PCOM aneurysm classically produces oculomotor nerve palsy with pupil dilation due to parasympathetic fiber compression. Ischemic microvascular palsies typically spare the pupil. MCA stroke causes cortical deficits, not isolated pupil-involving third nerve palsy. Correct answer: (a) Posterior communicating artery aneurysm. (≈50 words)
2. Which artery completes the posterior circulation connection to the anterior circulation via the PCOM?
a) Posterior cerebral artery
b) Anterior communicating artery
c) Basilar artery branch to PICA
d) Superficial temporal artery
Explanation & answer: The PCOM links the internal carotid system anteriorly to the posterior cerebral artery, which arises from the basilar artery posteriorly; this forms part of the posterior-anterior collateral route in the circle of Willis. The anterior communicating artery links the two anterior cerebral arteries. Correct answer: (a) Posterior cerebral artery. (≈50 words)
3. Best noninvasive initial imaging to detect a suspected PCOM aneurysm after SAH is:
a) CT angiography (CTA)
b) Plain skull X-ray
c) Ultrasound Doppler of carotids only
d) Electroencephalogram (EEG)
Explanation & answer: After subarachnoid hemorrhage, CT angiography is a rapid, noninvasive test to visualize intracranial aneurysms including PCOM aneurysms. Digital subtraction cerebral angiography remains gold standard but CTA is commonly used initially for detection and surgical planning. Correct answer: (a) CT angiography (CTA). (≈50 words)
4. Which embryologic vessel contributes to formation of the posterior communicating artery?
a) Fetal carotid-basilar anastomosis
b) Stapedial artery
c) External maxillary artery
d) Vitelline artery
Explanation & answer: The PCOM represents persistence of embryologic carotid–basilar anastomoses connecting the internal carotid to the posterior circulation. These fetal connections normally regress as posterior communicating and posterior cerebral arteries mature. Stapedial and vitelline arteries are unrelated. Correct answer: (a) Fetal carotid-basilar anastomosis. (≈50 words)
5. A ruptured PCOM aneurysm typically causes SAH with blood deposition in which cistern most prominently?
a) Interpeduncular cistern
b) Cisterna magna only
c) Cavernous sinus
d) Sigmoid sinus
Explanation & answer: A PCOM aneurysm rupture often produces subarachnoid blood in the interpeduncular cistern and basal cisterns around the circle of Willis due to its location at the ICA–PCOM junction. Cavernous sinus or venous sinuses are not primary subarachnoid spaces. Correct answer: (a) Interpeduncular cistern. (≈50 words)
6. Which clinical sign suggests a compressive third nerve palsy rather than ischemic microvascular palsy?
a) Early pupil dilation (mydriasis)
b) Isolated finger weakness
c) Pure sensory loss in a dermatomal pattern
d) Pure cerebellar ataxia
Explanation & answer: Pupil-involving oculomotor palsy with early mydriasis points to compression of peripheral parasympathetic fibers, as in a PCOM aneurysm. Microvascular ischemic palsies typically spare the pupil because central somatic fibers are affected but peripheral parasympathetic fibers are preserved. Correct answer: (a) Early pupil dilation (mydriasis). (≈50 words)
7. Which artery is NOT a direct branch of the internal carotid artery in the intracranial segment?
a) Ophthalmic artery (intracranial origin)
b) Posterior communicating artery
c) Middle cerebral artery
d) External carotid artery
Explanation & answer: The external carotid artery is a separate extracranial terminal branch; it does not arise from the intracranial internal carotid. The ophthalmic artery, PCOM, and MCA are intracranial branches or continuations of the ICA. Correct answer: (d) External carotid artery. (≈50 words)
8. In surgical clipping of a PCOM aneurysm, which neural structure must be protected to avoid postoperative diplopia and ptosis?
a) Oculomotor nerve (III)
b) Facial nerve (VII) extracranial branch
c) Hypoglossal nerve (XII)
d) Vagus nerve (X) trunk
Explanation & answer: The oculomotor nerve runs adjacent to the PCOM and posterior cerebral artery; it can be compressed by aneurysms or injured during clipping, causing ptosis and extraocular movement deficits. Facial, hypoglossal, and vagus nerves are remote from the PCOM region. Correct answer: (a) Oculomotor nerve (III). (≈50 words)
9. Which anatomical variation increases risk of anterior circulation collateral failure if PCOM is hypoplastic?
a) Hypoplastic PCOM with inadequate posterior flow
b) Bilateral large PCOM vessels providing robust collateralization
c) Prominent anterior communicating artery bridging ACAs
d) Redundant ophthalmic artery branches
Explanation & answer: A hypoplastic PCOM limits posterior-to-anterior collateral flow, increasing risk of ischemia if ICA flow is compromised. Large bilateral PCOMs or a robust anterior communicating artery improve collateral resilience. Thus hypoplastic PCOM predisposes to collateral failure. Correct answer: (a) Hypoplastic PCOM with inadequate posterior flow. (≈50 words)
10. Which therapeutic option is commonly considered for a saccular PCOM aneurysm not suitable for clipping?
a) Endovascular coiling (with or without stent-assisted technique)
b) Oral anticoagulation alone
c) High-dose systemic corticosteroids only
d) Carotid endarterectomy
Explanation & answer: Endovascular coiling, sometimes stent-assisted, is a standard treatment for saccular intracranial aneurysms including PCOM aneurysms when clipping is unfeasible. Anticoagulation, steroids, or carotid endarterectomy are inappropriate as primary aneurysm treatments. Correct answer: (a) Endovascular coiling. (≈50 words)
Chapter: Autonomic Nervous System
Topic: Coeliac Plexus Block
Subtopic: Clinical Applications
Keywords:
Coeliac Plexus: A nerve plexus located in the upper abdomen around the origin of the celiac trunk, supplying abdominal viscera.
Pain Block: An injection that interrupts pain signals in a specific nerve plexus or pathway.
Retroperitoneal: Anatomical space behind the peritoneum where structures like kidneys and major vessels lie.
Hypotension: Abnormally low blood pressure due to vasodilation or decreased cardiac output.
Diarrhea: Increased frequency of loose stools caused by autonomic imbalance in intestines after nerve block.
Lead Question – 2012
Which of the following is true about coeliac plexus block?
a) Located retroperitoneally at the level of L3
b) Usually done unilaterally
c) Useful for the painful conditions of lower abdomen
d) Most common side effect is diarrhea and hypotension
Explanation: The coeliac plexus lies retroperitoneally at the level of T12–L1, not L3. It is performed bilaterally, not unilaterally. It is mainly useful for pain relief in upper abdominal malignancies such as pancreatic cancer. The most common side effects are diarrhea and hypotension. Answer: d)
Q2. Coeliac plexus block is most commonly indicated in:
a) Renal colic
b) Pancreatic cancer pain
c) Chronic appendicitis
d) Hernia pain
Explanation: Coeliac plexus block is primarily indicated for intractable upper abdominal pain, especially due to pancreatic cancer. It is not used for lower abdominal or somatic pain. Answer: b)
Q3. Which vertebral level corresponds to the coeliac plexus?
a) T6–T7
b) T10–T11
c) T12–L1
d) L3–L4
Explanation: The coeliac plexus is located around the origin of the celiac trunk, corresponding to the vertebral level T12–L1. This location allows innervation to abdominal viscera. Answer: c)
Q4. Which imaging technique is commonly used for coeliac plexus block guidance?
a) MRI
b) Ultrasound
c) CT Scan
d) X-ray
Explanation: CT scan guidance is often used for precise needle placement during coeliac plexus block, ensuring accurate delivery and minimizing complications. Answer: c)
Q5. Which artery is closely related to the coeliac plexus?
a) Superior mesenteric artery
b) Inferior mesenteric artery
c) Celiac trunk
d) Renal artery
Explanation: The coeliac plexus surrounds the origin of the celiac trunk, which arises from the abdominal aorta at T12. This anatomical relation is crucial for block procedure. Answer: c)
Q6. A 50-year-old male with pancreatic cancer undergoes coeliac plexus block. What is the likely immediate effect?
a) Relief of upper abdominal pain
b) Increased heart rate
c) Paralysis of lower limb
d) Relief of pelvic pain
Explanation: Coeliac plexus block specifically reduces visceral pain from upper abdominal organs such as the pancreas, liver, stomach. Answer: a)
Q7. Which is the most serious complication of coeliac plexus block?
a) Diarrhea
b) Hypotension
c) Vascular injury
d) Nausea
Explanation: Though diarrhea and hypotension are common, the most serious complication is vascular injury, which may cause bleeding or ischemia. Answer: c)
Q8. Which sympathetic fibers are mainly targeted in coeliac plexus block?
a) Thoracic splanchnic nerves
b) Lumbar splanchnic nerves
c) Sacral splanchnic nerves
d) Cervical sympathetic trunk
Explanation: The thoracic splanchnic nerves (greater, lesser, least) converge on the coeliac plexus, carrying visceral pain signals. Answer: a)
Q9. A patient develops severe hypotension after a coeliac plexus block. What is the immediate management?
a) IV atropine
b) IV fluids and vasopressors
c) Oral rehydration solution
d) Wait for spontaneous recovery
Explanation: Sudden hypotension results from sympathetic blockade. Management involves IV fluid resuscitation and vasopressors to restore hemodynamic stability. Answer: b)
Q10. Which of the following conditions is least likely to benefit from a coeliac plexus block?
a) Gastric cancer pain
b) Pancreatic cancer pain
c) Chronic pancreatitis pain
d) Acute appendicitis pain
Explanation: Coeliac plexus block is beneficial for chronic visceral pain of upper abdominal organs, but not for somatic pain like acute appendicitis. Answer: d)
Q11. After coeliac plexus block, a patient complains of persistent diarrhea. What is the cause?
a) Parasympathetic overactivity
b) Vagus nerve block
c) Somatic nerve injury
d) Loss of sympathetic inhibition
Explanation: Sympathetic blockade removes inhibitory control over the intestines, leading to unopposed parasympathetic activity and diarrhea. Answer: d)
Chapter: Pelvis and Perineum
Topic: Nerves and Vessels
Subtopic: Relations of Ischial Spine
Keywords:
Ischial spine: A bony projection on the ischium important as a landmark in pelvic anatomy.
Pudendal nerve: Main nerve of perineum, crosses posterior to ischial spine.
Internal pudendal vessels: Artery and vein accompanying pudendal nerve.
Nerve to obturator internus: Motor nerve crossing ischial spine with pudendal bundle.
Obturator nerve: Runs along lateral pelvic wall, does not cross ischial spine.
Lead Question - 2012
Structure crossing dorsal surface of ischial spine are A/E:
a) Internal pudendal vessel
b) Pudendal nerve
c) Obturator nerve
d) Nerve to obturator internus
Explanation: The pudendal nerve, internal pudendal vessels, and nerve to obturator internus cross the ischial spine. The obturator nerve runs along the pelvic sidewall and exits via obturator canal, not over the ischial spine. Answer: c) Obturator nerve
1) A 35-year-old female during childbirth suffered injury to the structure crossing the ischial spine. Which function is most likely affected?
a) Sensation of perineum
b) Hip adduction
c) Quadriceps contraction
d) Knee extension
Explanation: The pudendal nerve crosses ischial spine and provides sensory supply to perineum. Injury causes perineal sensory loss. Hip adduction involves obturator nerve which does not cross spine. Answer: a) Sensation of perineum
2) Which vessel accompanies pudendal nerve while crossing ischial spine?
a) Inferior gluteal artery
b) Internal pudendal artery
c) Superior gluteal artery
d) Obturator artery
Explanation: Pudendal nerve crosses ischial spine along with internal pudendal vessels. Inferior and superior gluteal arteries do not directly cross spine. Answer: b) Internal pudendal artery
3) A surgeon performing pudendal block locates which landmark near ischial spine?
a) Sacral promontory
b) Coccyx tip
c) Ischial spine via vaginal exam
d) Iliac crest
Explanation: Pudendal block is given by palpating ischial spine transvaginally where pudendal nerve passes. Coccyx and iliac crest are not related landmarks. Answer: c) Ischial spine via vaginal exam
4) Which nerve does not cross ischial spine?
a) Pudendal nerve
b) Nerve to obturator internus
c) Internal pudendal vessels
d) Obturator nerve
Explanation: Obturator nerve runs through obturator canal and does not cross ischial spine. Others cross spine dorsally. Answer: d) Obturator nerve
5) Pudendal nerve block provides anesthesia for which procedure?
a) Episiotomy
b) Appendectomy
c) Inguinal hernia repair
d) Cholecystectomy
Explanation: Pudendal block is mainly used in obstetrics for episiotomy and perineal repair by anesthetizing pudendal nerve at ischial spine. Answer: a) Episiotomy
6) Nerve to obturator internus after crossing ischial spine enters which region?
a) Perineum
b) Gluteal region
c) Obturator canal
d) Femoral canal
Explanation: The nerve to obturator internus crosses the ischial spine dorsally and enters the gluteal region before supplying obturator internus muscle. Answer: b) Gluteal region
7) Which muscle acts as close relation of structures crossing ischial spine?
a) Piriformis
b) Coccygeus
c) Obturator externus
d) Psoas major
Explanation: Coccygeus muscle lies in relation to ischial spine, providing support to crossing pudendal bundle. Piriformis is higher, obturator externus and psoas are not related. Answer: b) Coccygeus
8) Clinical feature of pudendal nerve entrapment near ischial spine is:
a) Loss of knee reflex
b) Perineal pain and numbness
c) Foot drop
d) Loss of Achilles reflex
Explanation: Pudendal entrapment near ischial spine causes perineal pain, numbness, and sphincter dysfunction. Reflexes and foot drop are unrelated. Answer: b) Perineal pain and numbness
9) In pelvic fracture involving ischial spine, which function is spared?
a) Perineal sensation
b) Anal sphincter tone
c) Hip adduction
d) External urethral sphincter
Explanation: Hip adduction is mediated by obturator nerve, which does not cross ischial spine, hence spared. Pudendal functions are compromised. Answer: c) Hip adduction
10) The pudendal nerve is derived from which spinal segments?
a) L2-L4
b) L4-S1
c) S2-S4
d) S1-S3
Explanation: Pudendal nerve originates from sacral plexus segments S2, S3, S4. These supply perineum and external sphincters. Answer: c) S2-S4
Chapter: Abdomen
Topic: Autonomic Nervous System of Abdomen
Subtopic: Celiac Plexus Block
Keyword Definitions:
Celiac Plexus: A network of nerves around the abdominal aorta supplying abdominal viscera.
Retrocrural Approach: Classic posterior approach to reach the celiac plexus.
Visceral Pain: Pain arising from internal organs, often dull and poorly localized.
Hypotension: Decrease in blood pressure, common after sympathetic block.
Anesthesia vs Analgesia: Anesthesia blocks sensation including touch and pain, while analgesia only blocks pain.
Lead Question – 2012
Celiac plexus block all the following is true except?
a) Relieved pain from gastric malignancy
b) Cause hypotention
c) Can be used to provide anesthesia for intra abdominal surgery
d) Can be given only by retrocrural (classic) approach
Explanation: The celiac plexus block is useful for pain relief in gastric and pancreatic malignancy and may cause hypotension due to sympathetic block. It is not used to provide complete anesthesia for intra-abdominal surgery. Multiple approaches (retrocrural, anterior, endoscopic) exist, so option c is correct.
Guessed Questions for NEET PG:
1) Celiac plexus block is most commonly used for pain relief in:
a) Chronic appendicitis
b) Pancreatic cancer
c) Gallbladder stones
d) Peptic ulcer disease
Explanation: Pancreatic cancer produces severe intractable pain, which is effectively relieved by celiac plexus block. It reduces narcotic use and improves quality of life. Hence option b is correct.
2) Complication of celiac plexus block includes:
a) Hypertension
b) Hypotension
c) Tachycardia
d) None of the above
Explanation: The sympathetic fibers in the celiac plexus regulate vascular tone. Their block causes vasodilation and hypotension, not hypertension. Tachycardia may be compensatory, but main complication is hypotension. Answer: b.
3) Approach not used for celiac plexus block:
a) Retrocrural
b) Anterior transabdominal
c) Endoscopic ultrasound-guided
d) Transoral
Explanation: The block can be done by retrocrural (classic posterior), anterior percutaneous, or endoscopic methods. Transoral is not a described route. Hence option d is correct.
4) Which type of nerve fibers are blocked by celiac plexus block?
a) Parasympathetic
b) Sympathetic
c) Somatic sensory
d) Motor
Explanation: The celiac plexus contains sympathetic fibers supplying abdominal viscera. Blocking these fibers reduces visceral pain. Parasympathetic and somatic motor fibers remain unaffected. Hence option b is correct.
5) A patient with unresectable pancreatic carcinoma complains of severe epigastric pain. Best palliative option is:
a) Celiac plexus block
b) Cholecystectomy
c) Gastrectomy
d) Splenectomy
Explanation: Celiac plexus block provides long-term relief of upper abdominal visceral pain, especially in pancreatic carcinoma. Surgical options are not indicated. Answer: a.
6) Hypotension after celiac plexus block occurs due to:
a) Increased vagal tone
b) Sympathetic blockade and vasodilation
c) Blood loss
d) Reflex bradycardia
Explanation: Sympathetic blockade leads to vasodilation of splanchnic vessels, reducing systemic vascular resistance and causing hypotension. Hence option b is correct.
7) Duration of analgesia after celiac plexus block using alcohol is:
a) Few hours
b) Days
c) Weeks to months
d) Lifetime
Explanation: Neurolytic agents like alcohol or phenol produce long-lasting destruction of sympathetic fibers, giving pain relief for weeks to months. Answer: c.
8) Clinical sign of successful celiac plexus block:
a) Increased heart rate
b) Warmth and vasodilation in lower limbs
c) Warmth and vasodilation in upper abdomen
d) Sweating in face
Explanation: Blocking sympathetic fibers causes vasodilation in splanchnic circulation, presenting as warmth in the upper abdomen. Thus option c is correct.
9) Contraindication for celiac plexus block:
a) Coagulopathy
b) Chronic pain of pancreas
c) Severe malignancy pain
d) Endoscopic procedure planned
Explanation: Coagulopathy increases the risk of retroperitoneal hemorrhage, hence is a contraindication. Chronic pain and malignancy are indications. Answer: a.
10) A patient undergoing celiac plexus block develops sudden severe back pain with leg weakness. Most likely cause is:
a) Alcohol neurotoxicity to somatic nerves
b) Hypotension
c) Pneumothorax
d) Allergy
Explanation: Neurolytic agents may spread to somatic nerves like lumbar plexus, leading to back pain and transient leg weakness. Hence option a is correct.
Chapter: Anatomy
Topic: Nerves of Perineum
Subtopic: Ischiorectal Abscess and Nerve Supply
Keyword Definitions:
Ischiorectal Abscess: A pus collection in ischiorectal fossa, usually due to anal gland infection.
Inferior Rectal Nerve: Branch of pudendal nerve, supplies external anal sphincter and perianal skin.
Superior Rectal Nerve: Branch of inferior mesenteric plexus, supplies rectal mucosa.
Inferior Gluteal Nerve: Supplies gluteus maximus muscle.
Superior Gluteal Nerve: Supplies gluteus medius, minimus, tensor fascia lata.
Pudendal Nerve: Main nerve of perineum, gives inferior rectal branches.
Clinical Correlation: Abscess drainage can damage inferior rectal nerve causing anal incontinence.
Lead Question – 2012
During incision & drainage of ischiorectal abscess, which nerve is/are affected/injured:
a) Superior rectal nerve
b) Inferior rectal nerve
c) Superior gluteal nerve
d) Inferior gluteal nerve
Explanation: The ischiorectal fossa contains branches of pudendal nerve, mainly inferior rectal nerves. Incision and drainage can damage these nerves, leading to sensory loss in perianal skin and possible sphincter dysfunction. Correct answer: Inferior rectal nerve.
Guessed Questions for NEET PG
Q1. Which structure forms the medial boundary of the ischiorectal fossa?
a) Levator ani
b) Obturator internus
c) Gluteus maximus
d) Piriformis
Explanation: The ischiorectal fossa lies between levator ani medially and obturator internus laterally. Medial boundary is levator ani muscle, covered by anal fascia. Correct answer: Levator ani.
Q2. A patient with ischiorectal abscess drainage later develops perianal sensory loss. The most likely injured nerve is:
a) Superior hypogastric plexus
b) Inferior rectal nerve
c) Sciatic nerve
d) Coccygeal plexus
Explanation: Perianal sensory loss after ischiorectal abscess surgery indicates inferior rectal nerve injury, which supplies perianal skin and external sphincter. Correct answer: Inferior rectal nerve.
Q3. Inferior rectal nerve is a branch of:
a) Pudendal nerve
b) Obturator nerve
c) Sacral plexus directly
d) Coccygeal nerve
Explanation: The inferior rectal nerve arises from the pudendal nerve, providing motor supply to external anal sphincter and sensory supply to perianal skin. Correct answer: Pudendal nerve.
Q4. Which vessel accompanies the inferior rectal nerve in the ischiorectal fossa?
a) Superior rectal artery
b) Middle rectal artery
c) Inferior rectal artery
d) Internal pudendal artery
Explanation: The inferior rectal nerve is accompanied by inferior rectal vessels, branches of internal pudendal vessels, within the ischiorectal fossa. Correct answer: Inferior rectal artery.
Q5. A 40-year-old with ischiorectal abscess presents with fecal incontinence post-surgery. Which muscle is most likely affected?
a) Internal anal sphincter
b) External anal sphincter
c) Puborectalis
d) Levator ani
Explanation: The inferior rectal nerve supplies external anal sphincter. Injury causes sphincter weakness leading to incontinence. Correct answer: External anal sphincter.
Q6. The pudendal nerve exits the pelvis through:
a) Greater sciatic foramen
b) Lesser sciatic foramen
c) Obturator canal
d) Sacral hiatus
Explanation: Pudendal nerve exits pelvis via greater sciatic foramen, curves around sacrospinous ligament, and enters perineum through lesser sciatic foramen. Correct answer: Greater sciatic foramen.
Q7. Which nerve is spared in ischiorectal abscess incision?
a) Inferior rectal nerve
b) Superior gluteal nerve
c) Perineal branch of posterior femoral cutaneous
d) Pudendal nerve
Explanation: Superior gluteal nerve is high in gluteal region, away from perianal surgery. Hence, spared in ischiorectal abscess drainage. Correct answer: Superior gluteal nerve.
Q8. Clinical sign of inferior rectal nerve damage is:
a) Loss of rectal mucosal sensation
b) Weakness of internal sphincter
c) Fecal incontinence
d) Urinary incontinence
Explanation: Inferior rectal nerve supplies external sphincter; injury leads to fecal incontinence, not urinary. Correct answer: Fecal incontinence.
Q9. Which of the following is NOT located in the ischiorectal fossa?
a) Fat
b) Inferior rectal vessels
c) Inferior rectal nerve
d) Superior rectal nerve
Explanation: Superior rectal nerve is from inferior mesenteric plexus and lies higher, not in ischiorectal fossa. Correct answer: Superior rectal nerve.
Q10. During perianal surgery, surgeon avoids deep lateral dissection to prevent injury to:
a) Pudendal canal structures
b) Sciatic nerve
c) Inferior gluteal nerve
d) Coccygeal plexus
Explanation: Pudendal canal contains pudendal nerve and internal pudendal vessels; deep lateral dissection risks injury. Correct answer: Pudendal canal structures.
Topic: Cavernous Sinus
Subtopic: Structures within the Cavernous Sinus
Keyword Definitions:
Cavernous Sinus: A venous sinus located on either side of the pituitary fossa, containing cranial nerves and the internal carotid artery.
Internal Carotid Artery: Artery passing through cavernous sinus, giving off ophthalmic artery and sympathetic plexus.
Oculomotor Nerve (CN III): Passes through lateral wall of cavernous sinus, controls most extraocular muscles.
Trochlear Nerve (CN IV): Lies in lateral wall, innervates superior oblique muscle.
Ophthalmic Nerve (V1): Branch of trigeminal nerve in lateral wall, sensory to upper face and cornea.
Maxillary Nerve (V2): Passes in lateral wall, sensory to midface, upper teeth.
Mandibular Nerve (V3): Does not pass through cavernous sinus; exits skull via foramen ovale.
Abducens Nerve (CN VI): Runs through sinus near ICA, controls lateral rectus muscle.
Facial Nerve (CN VII): Not present in cavernous sinus; exits via stylomastoid foramen.
Clinical Relevance: Cavernous sinus thrombosis affects cranial nerves III, IV, V1, V2, VI and internal carotid artery.
Lead Question – 2012
Which of the following structures seen in the cavernous sinus?
a) Maxillary division of V nerve
b) Mandibular division of V nerve
c) Internal carotid artery
d) Facial nerve
Explanation: The internal carotid artery courses through the cavernous sinus along with cranial nerves III, IV, V1, V2 (maxillary), and VI. The mandibular nerve (V3) and facial nerve (CN VII) are not present. Therefore, the correct answer is c) Internal carotid artery. Understanding anatomy is critical in cavernous sinus thrombosis and aneurysms.
1. Which cranial nerve passes through the lateral wall of the cavernous sinus?
a) Abducens (CN VI)
b) Oculomotor (CN III)
c) Mandibular (V3)
d) Facial (CN VII)
Explanation: Cranial nerves III, IV, V1, and V2 pass in the lateral wall of the cavernous sinus. CN VI runs centrally near ICA. Mandibular (V3) and facial nerve do not traverse the sinus. Correct answer: b) Oculomotor (CN III).
2. Abducens nerve (CN VI) is located:
a) In lateral wall
b) Medial to ICA
c) Outside sinus
d) With facial nerve
Explanation: The abducens nerve (CN VI) runs adjacent to the internal carotid artery within the sinus. It is most vulnerable in cavernous sinus thrombosis. Correct answer: b) Medial to ICA.
3. Which trigeminal branch passes through cavernous sinus?
a) Ophthalmic (V1)
b) Maxillary (V2)
c) Mandibular (V3)
d) Both a and b
Explanation: V1 (ophthalmic) and V2 (maxillary) traverse the lateral wall of cavernous sinus, providing sensory innervation to face. V3 exits via foramen ovale, outside sinus. Correct answer: d) Both a and b.
4. Facial nerve (CN VII) exits skull through:
a) Foramen ovale
b) Jugular foramen
c) Stylomastoid foramen
d) Superior orbital fissure
Explanation: The facial nerve (CN VII) exits via stylomastoid foramen, not through cavernous sinus. Lesions in sinus do not affect CN VII. Correct answer: c) Stylomastoid foramen.
5. Cavernous sinus thrombosis can affect all EXCEPT:
a) CN III
b) CN IV
c) CN VII
d) CN VI
Explanation: Thrombosis of cavernous sinus involves CN III, IV, V1, V2, and VI. CN VII (facial nerve) is not affected because it is outside the sinus. Correct answer: c) CN VII.
6. Internal carotid artery in cavernous sinus gives off:
a) Ophthalmic artery
b) Maxillary artery
c) Middle cerebral artery
d) Vertebral artery
Explanation: The internal carotid artery within cavernous sinus gives rise to ophthalmic artery and branches to carotid sympathetic plexus. Middle cerebral and vertebral arteries arise more proximally. Correct answer: a) Ophthalmic artery.
7. Superior orbital fissure transmits which structures in relation to cavernous sinus?
a) CN III, IV, V1, VI
b) V2 and V3
c) CN VII and VIII
d) ICA only
Explanation: Cranial nerves III, IV, V1, VI leave the cavernous sinus to enter orbit through superior orbital fissure. This pathway explains clinical ophthalmoplegia in cavernous sinus lesions. Correct answer: a) CN III, IV, V1, VI.
8. Maxillary nerve (V2) exits skull via:
a) Foramen rotundum
b) Foramen ovale
c) Superior orbital fissure
d) Jugular foramen
Explanation: Maxillary nerve (V2) passes through lateral wall of cavernous sinus and exits via foramen rotundum to reach pterygopalatine fossa. Correct answer: a) Foramen rotundum.
9. Mandibular nerve (V3) is located:
a) Within cavernous sinus
b) Exits via foramen ovale
c) Lateral wall of sinus
d) With CN VI
Explanation: The mandibular nerve (V3) does not traverse the cavernous sinus. It exits skull via foramen ovale to supply lower face and muscles of mastication. Correct answer: b) Exits via foramen ovale.
10. Cavernous sinus is clinically important in:
a) Spread of facial infections
b) Thrombosis causing cranial nerve palsies
c) ICA aneurysms
d) All of the above
Explanation: The cavernous sinus is prone to thrombosis from facial infections (danger triangle), can compress CN III, IV, V1, V2, VI, and ICA aneurysms can occur. Clinical knowledge is crucial. Correct answer: d) All of the above.
Sympathetic Nervous System: Part of the autonomic nervous system responsible for 'fight or flight' responses.
Cardiac Sympathetic Supply: Nerves arising from thoracic spinal segments innervating the heart, increasing heart rate and contractility.
Spinal Segments: Sections of the spinal cord from which sympathetic fibers emerge (T1–L2).
SA Node: Sinoatrial node, pacemaker of the heart, receives sympathetic and parasympathetic innervation.
AV Node: Atrioventricular node, conduction relay, also influenced by autonomic inputs.
Thoracic Sympathetic Chain: Series of ganglia along thoracic vertebrae transmitting sympathetic fibers to organs including the heart.
Chapter: Cardiovascular Physiology
Topic: Autonomic Nervous Supply of Heart
Subtopic: Sympathetic Innervation
Lead Question 2012: Sympathetic supply to the heart arises from which of the following spinal segments?
a) T1 to T5
b) T2 to T6
c) T3 to T7
d) T4 to T8
Answer: a) T1 to T5
Explanation: Cardiac sympathetic fibers arise from the thoracic spinal segments T1 to T5. These preganglionic fibers synapse in the sympathetic chain and postganglionic fibers reach the SA node, AV node, and coronary vessels. They increase heart rate, contractility, and conduction velocity. Understanding this is crucial in autonomic cardiac physiology and clinical conditions like referred cardiac pain.
1. Which part of the autonomic nervous system increases heart rate?
a) Parasympathetic
b) Sympathetic
c) Enteric
d) Somatic
Answer: b) Sympathetic
Explanation: Sympathetic stimulation increases heart rate, contractility, and conduction through SA and AV nodes. Parasympathetic (vagal) stimulation decreases heart rate. This balance is essential for cardiac homeostasis and is clinically important in arrhythmias and heart failure.
2. Preganglionic sympathetic fibers to the heart synapse in which ganglia?
a) Cervical ganglia
b) Thoracic sympathetic chain
c) Dorsal root ganglia
d) Celiac ganglia
Answer: b) Thoracic sympathetic chain
Explanation: Preganglionic fibers from T1–T5 synapse in the thoracic sympathetic chain. Postganglionic fibers then reach cardiac structures. This pathway is clinically significant for cardiac sympathectomy and management of arrhythmias.
3. Sympathetic stimulation affects which part of the heart first?
a) SA node
b) AV node
c) Ventricular myocardium
d) Aorta
Answer: a) SA node
Explanation: Sympathetic fibers primarily influence the SA node, increasing heart rate. AV node conduction and ventricular contractility are also enhanced, demonstrating the widespread effect of sympathetic innervation on cardiac function.
4. Referred cardiac pain is transmitted through which spinal segments?
a) T1–T5
b) C3–C5
c) L1–L3
d) S1–S3
Answer: a) T1–T5
Explanation: Sympathetic afferents from the heart enter spinal cord segments T1–T5, producing referred pain in the chest and medial arm during myocardial ischemia. Knowledge of this pathway aids in diagnosing angina or myocardial infarction.
5. Which neurotransmitter is released by postganglionic sympathetic fibers at the heart?
a) Acetylcholine
b) Norepinephrine
c) Dopamine
d) Serotonin
Answer: b) Norepinephrine
Explanation: Postganglionic sympathetic fibers release norepinephrine, which binds to beta-1 adrenergic receptors, increasing heart rate and contractility. Pharmacologic modulation of this pathway is important in heart failure and arrhythmia management.
6. Sympathetic cardiac nerves also innervate which coronary structures?
a) Coronary arteries
b) Pulmonary veins
c) Aorta only
d) SA node exclusively
Answer: a) Coronary arteries
Explanation: Sympathetic nerves innervate coronary vessels, causing vasodilation via beta-2 receptors during increased cardiac activity, ensuring adequate myocardial perfusion during stress or exercise.
7. Beta-1 adrenergic receptors are predominantly located on:
a) SA and AV nodes
b) Smooth muscle of arteries
c) Lung alveoli
d) Skeletal muscle
Answer: a) SA and AV nodes
Explanation: Beta-1 receptors are located on nodal tissue and ventricular myocardium, mediating sympathetic effects like increased heart rate and conduction velocity. Beta-blockers act here to treat tachyarrhythmias and hypertension.
8. Cardiac sympathectomy may be performed to treat:
a) Refractory angina
b) GERD
c) Asthma
d) Liver cirrhosis
Answer: a) Refractory angina
Explanation: Cardiac sympathectomy reduces sympathetic input, lowering heart rate and oxygen demand in severe angina. T1–T5 segments are targeted to interrupt sympathetic supply while preserving parasympathetic tone.
9. Which spinal levels contribute to sympathetic innervation of both SA and AV nodes?
a) T1–T5
b) C1–C4
c) L1–L3
d) T6–T10
Answer: a) T1–T5
Explanation: Preganglionic fibers from T1–T5 synapse in the cervical and upper thoracic ganglia. Postganglionic fibers innervate both SA and AV nodes, coordinating cardiac rate and conduction, essential knowledge for cardiology exams.
10. Increased sympathetic activity to the heart results in:
a) Tachycardia and increased contractility
b) Bradycardia and decreased contractility
c) Vasodilation of systemic veins
d) No change in cardiac function
Answer: a) Tachycardia and increased contractility
Explanation: Sympathetic activation increases SA node firing rate, AV conduction, and myocardial contractility through norepinephrine action on beta-1 receptors. This response supports cardiac output during stress and is a core concept in autonomic cardiac physiology.
Chapter: Musculoskeletal Anatomy
Topic: Lower Limb Anatomy
Sub-topic: Gluteal Region – Muscles, Nerves, and Safe Injection Sites
Keyword Definitions:
Gluteus maximus: The largest muscle in the buttock, responsible for hip extension and external rotation.
Gluteus medius: A muscle important for hip abduction, covering part of the safe injection site.
Sciatic nerve: Major nerve running through the posterior thigh, can be injured by improper injection.
Safe injection site: The superolateral quadrant of the buttock to avoid nerve or vascular injury.
Trendelenburg gait: Abnormal gait caused by injury to the superior gluteal nerve.
Superior gluteal nerve: Nerve that supplies gluteus medius, gluteus minimus, and tensor fasciae latae.
Inferior gluteal nerve: Supplies the gluteus maximus muscle.
Intramuscular injection: Administration of drugs into the muscle tissue for rapid absorption.
Quadrants of buttock: Divisions used to identify safe anatomical injection sites.
Ischial tuberosity: Bony prominence of pelvis, landmark for hamstring muscle attachment.
1. Site of injection in the gluteus? (2012)
a) Inferomedial
b) Superomedial
c) Superolateral
d) Superomedial
Explanation: The correct answer is c) Superolateral. Intramuscular injections in the gluteal region should be given in the superolateral quadrant to avoid injury to the sciatic nerve and major vessels. This quadrant lies over the gluteus medius, which has good muscle bulk and minimal risk of nerve damage. Choosing other quadrants risks nerve injury or hematoma formation.
2. Which nerve is most at risk if a gluteal intramuscular injection is given in the inferomedial quadrant?
a) Femoral nerve
b) Sciatic nerve
c) Obturator nerve
d) Pudendal nerve
Explanation: The correct answer is b) Sciatic nerve. The sciatic nerve passes through the lower and medial portions of the gluteal region. Improper injections here can cause severe neuropathic pain, weakness in the leg, and even permanent paralysis in severe cases. Hence, the inferomedial quadrant is strictly avoided for injections.
3. A patient develops Trendelenburg gait after hip surgery. Which nerve was likely injured?
a) Superior gluteal nerve
b) Inferior gluteal nerve
c) Sciatic nerve
d) Femoral nerve
Explanation: The correct answer is a) Superior gluteal nerve. This nerve innervates gluteus medius and minimus. Injury leads to weakness in hip abduction, causing the pelvis to drop on the opposite side during walking. This is commonly tested clinically and can occur after injections or surgery near the greater sciatic foramen.
4. Which muscle is primarily used for safe gluteal intramuscular injections?
a) Gluteus maximus
b) Gluteus medius
c) Piriformis
d) Tensor fasciae latae
Explanation: The correct answer is b) Gluteus medius. The muscle has a thick bulk in the superolateral quadrant, providing a safe site for intramuscular injections while avoiding important neurovascular structures. Gluteus maximus is avoided due to the underlying sciatic nerve pathway.
5. The piriformis muscle exits the pelvis through which structure?
a) Lesser sciatic foramen
b) Greater sciatic foramen
c) Obturator canal
d) Inguinal canal
Explanation: The correct answer is b) Greater sciatic foramen. The piriformis muscle is a key landmark in the gluteal region, dividing the greater sciatic foramen into superior and inferior parts for neurovascular structures. Its relationship is clinically important in avoiding nerve injury during injections.
6. Which artery accompanies the sciatic nerve in the gluteal region?
a) Inferior gluteal artery
b) Superior gluteal artery
c) Obturator artery
d) Femoral artery
Explanation: The correct answer is a) Inferior gluteal artery. This artery, along with the sciatic nerve, exits the pelvis below the piriformis. Injury to this artery during wrong injection placement can cause significant hematoma formation.
7. Which nerve passes above the piriformis muscle?
a) Sciatic nerve
b) Superior gluteal nerve
c) Inferior gluteal nerve
d) Pudendal nerve
Explanation: The correct answer is b) Superior gluteal nerve. It exits the pelvis above piriformis and is at risk in superomedial injections, leading to Trendelenburg gait. This is why even the superomedial quadrant is avoided for injections.
8. A patient complains of buttock pain radiating down the thigh after a fall. Which nerve is most likely injured?
a) Sciatic nerve
b) Femoral nerve
c) Obturator nerve
d) Pudendal nerve
Explanation: The correct answer is a) Sciatic nerve. This nerve supplies the posterior thigh and all muscles below the knee. Injury causes pain, weakness, and sensory loss along its distribution. It is the most commonly injured nerve in the gluteal region from trauma or improper injections.
9. Which muscle is innervated by the inferior gluteal nerve?
a) Gluteus maximus
b) Gluteus medius
c) Gluteus minimus
d) Piriformis
Explanation: The correct answer is a) Gluteus maximus. This muscle is important for hip extension, especially in rising from sitting or climbing stairs. Injury to its nerve reduces hip extension power but does not affect walking significantly.
10. Which landmark is used to locate the superolateral gluteal injection site?
a) Anterior superior iliac spine and greater trochanter
b) Ischial tuberosity and coccyx
c) Pubic symphysis and femoral head
d) Sacrum and iliac crest
Explanation: The correct answer is a) Anterior superior iliac spine and greater trochanter. Drawing an imaginary line between these two landmarks and selecting the upper outer quadrant ensures safe injection placement.
Keyword Definitions:
Tibial Nerve: A branch of the sciatic nerve that innervates the posterior compartment of the leg and plantar surface of the foot.
Palsy: Weakness or paralysis of a muscle group due to nerve damage.
Plantar Flexion: Movement of the foot downward at the ankle joint.
Dorsiflexion: Upward movement of the foot at the ankle joint.
Anterior Compartment of Leg: Muscles responsible for dorsiflexion of the foot and extension of the toes.
Posterior Compartment of Leg: Muscles responsible for plantar flexion and toe flexion.
Sciatic Nerve: Largest nerve in the body, branching into tibial and common peroneal nerves.
Medial Plantar Nerve: Branch of the tibial nerve supplying the medial aspect of the sole.
Lateral Plantar Nerve: Branch of the tibial nerve supplying the lateral aspect of the sole.
Clinical Examination: Bedside evaluation of nerve injury using motor, sensory, and reflex testing.
Chapter: Neuroanatomy Topic: Peripheral Nervous System Subtopic: Tibial Nerve Injury
Lead Question (2012):
Tibial nerve injury/palsy causes:
a) Dorsiflexion of foot at ankle joint
b) Plantar flexion of the foot at ankle joint
c) Loss of sensation of dorsum of foot
d) Paralysis of muscles of anterior compartment of leg
Explanation: The tibial nerve supplies the posterior compartment of the leg, responsible for plantar flexion and toe flexion. Injury leads to loss of plantar flexion, weakened inversion, and sensory loss over the sole. Dorsiflexion is performed by the anterior compartment (deep peroneal nerve). The dorsum of the foot is supplied mainly by the superficial peroneal nerve.
Q1. Which of the following muscles is innervated by the tibial nerve?
a) Tibialis anterior
b) Gastrocnemius
c) Peroneus longus
d) Extensor digitorum longus
Explanation: The gastrocnemius muscle is part of the posterior compartment of the leg, innervated by the tibial nerve, and functions in plantar flexion. Tibialis anterior and extensor digitorum longus are supplied by the deep peroneal nerve. Peroneus longus is supplied by the superficial peroneal nerve.
Q2. Damage to the tibial nerve at the popliteal fossa results in loss of:
a) Dorsiflexion
b) Plantar flexion
c) Knee extension
d) Hip abduction
Explanation: Tibial nerve injury at the popliteal fossa causes loss of plantar flexion and toe flexion, as the posterior compartment muscles are denervated. Dorsiflexion is controlled by anterior compartment muscles. Knee extension is mediated by the femoral nerve, and hip abduction by the gluteal nerves.
Q3. Sensory loss in tibial nerve injury typically occurs over:
a) Dorsum of foot
b) Lateral leg
c) Sole of foot
d) Medial thigh
Explanation: The tibial nerve provides sensory innervation to the sole of the foot via the medial and lateral plantar nerves. The dorsum of the foot is innervated by the superficial peroneal nerve. Lateral leg sensation comes from the superficial peroneal and sural nerves; medial thigh is via femoral nerve branches.
Q4. Which reflex may be lost in tibial nerve injury?
a) Knee jerk
b) Ankle jerk
c) Biceps jerk
d) Plantar reflex
Explanation: The ankle jerk (Achilles tendon reflex) tests the S1-S2 roots via the tibial nerve. Tibial nerve injury abolishes this reflex. Knee jerk involves L2-L4 via femoral nerve, biceps jerk involves C5-C6 via musculocutaneous nerve, and plantar reflex tests corticospinal tract integrity.
Q5. The tibial nerve is a terminal branch of which nerve?
a) Femoral nerve
b) Obturator nerve
c) Sciatic nerve
d) Common peroneal nerve
Explanation: The sciatic nerve divides into the tibial and common peroneal nerves at the apex of the popliteal fossa. The tibial nerve continues downward to supply the posterior leg and sole of the foot. Femoral and obturator nerves are separate lumbar plexus branches.
Q6. In tarsal tunnel syndrome, which nerve is compressed?
a) Common peroneal nerve
b) Tibial nerve
c) Deep peroneal nerve
d) Sural nerve
Explanation: Tarsal tunnel syndrome is due to compression of the tibial nerve beneath the flexor retinaculum, causing pain, tingling, and numbness in the sole of the foot. The common and deep peroneal nerves are involved in anterior/lateral leg innervation; sural nerve supplies the posterolateral leg and lateral foot.
Q7. A patient with tibial nerve injury will have difficulty in:
a) Standing on toes
b) Standing on heels
c) Knee extension
d) Hip extension
Explanation: Standing on toes requires plantar flexion, which is mediated by the tibial nerve. Tibial nerve injury makes this action weak or impossible. Standing on heels requires dorsiflexion (deep peroneal nerve). Knee and hip extension are controlled by femoral and gluteal nerves respectively.
Q8. Which artery accompanies the tibial nerve in the posterior compartment of the leg?
a) Anterior tibial artery
b) Posterior tibial artery
c) Popliteal artery
d) Fibular artery
Explanation: The posterior tibial artery runs with the tibial nerve in the posterior compartment. Anterior tibial artery runs with deep peroneal nerve. Popliteal artery is above the split, and fibular artery is a branch of the posterior tibial artery.
Q9. Which muscle is spared in tibial nerve injury at the ankle?
a) Flexor hallucis longus
b) Flexor digitorum longus
c) Tibialis posterior
d) Gastrocnemius
Explanation: Injury at the ankle spares proximal muscles like gastrocnemius, tibialis posterior, and long toe flexors because they are innervated higher up. Only intrinsic foot muscles supplied by tibial nerve branches are affected.
Q10. Clinical feature of complete tibial nerve transection at popliteal fossa includes:
a) Foot drop
b) Clawing of toes
c) Loss of knee jerk
d) Loss of hip abduction
Explanation: Complete tibial nerve injury causes inability to plantar flex and invert foot, loss of sole sensation, absent ankle jerk, and clawing of toes due to intrinsic foot muscle paralysis. Foot drop is from common peroneal nerve injury. Knee jerk and hip abduction are unrelated to tibial nerve.
Cervical Esophagus: The uppermost part of the esophagus, extending from the cricopharyngeus to the thoracic inlet.
Vagus Nerve: Cranial nerve X, responsible for parasympathetic control of the heart, lungs, and digestive tract.
Recurrent Laryngeal Nerve: Branch of the vagus nerve that supplies motor function and sensation to the larynx and cervical esophagus.
Esophageal Innervation: Nerve supply that controls esophageal motility and sensation.
Parasympathetic Fibers: Nerve fibers that control involuntary functions like digestion and glandular activity.
Sympathetic Fibers: Nerve fibers involved in fight-or-flight responses, also affecting smooth muscle tone in the esophagus.
Cricopharyngeus Muscle: The upper esophageal sphincter controlling the entry of food into the esophagus.
Esophageal Motility Disorders: Conditions affecting the coordinated muscle contractions of the esophagus.
Leas Question 2012
Q1. (2012) Nerve supply of cervical esophagus?
a) Vagus
b) Left recurrent laryngeal nerve
c) Right recurrent laryngeal nerve
d) All of the above
Answer: d) All of the above
Explanation: The cervical esophagus receives motor innervation from both the right and left recurrent laryngeal nerves, which are branches of the vagus nerve. These nerves provide motor supply to striated muscles and sensory innervation to the mucosa. Sympathetic fibers from the cervical sympathetic chain also contribute, controlling muscle tone. Damage to these nerves can lead to dysphagia and aspiration.
Q2. Which muscle forms the upper esophageal sphincter?
a) Cricopharyngeus
b) Thyropharyngeus
c) Inferior constrictor
d) Stylopharyngeus
Answer: a) Cricopharyngeus
Explanation: The cricopharyngeus muscle is the principal component of the upper esophageal sphincter, regulating the passage of food into the cervical esophagus. It is innervated by the recurrent laryngeal nerve. Dysfunction may lead to Zenker’s diverticulum formation.
Q3. Injury to the left recurrent laryngeal nerve during thyroid surgery may cause?
a) Hoarseness
b) Dysphagia
c) Loss of gag reflex
d) Loss of taste sensation
Answer: a) Hoarseness
Explanation: The left recurrent laryngeal nerve supplies motor fibers to most intrinsic laryngeal muscles. Injury results in vocal cord paralysis, causing hoarseness, weak cough, and aspiration risk. This nerve also contributes to the cervical esophageal motor function.
Q4. Which nerve passes under the arch of the aorta before ascending to the larynx?
a) Left recurrent laryngeal nerve
b) Right recurrent laryngeal nerve
c) Vagus nerve
d) Glossopharyngeal nerve
Answer: a) Left recurrent laryngeal nerve
Explanation: The left recurrent laryngeal nerve loops under the aortic arch, while the right loops under the subclavian artery. This anatomical difference explains why left-sided nerve palsy may occur in thoracic diseases affecting the aorta or mediastinum.
Q5. Sympathetic innervation of the cervical esophagus arises from?
a) Stellate ganglion
b) Cervical sympathetic chain
c) Thoracic sympathetic chain
d) Both a and b
Answer: d) Both a and b
Explanation: Sympathetic fibers from the cervical sympathetic chain, including the stellate ganglion, provide vasomotor and smooth muscle tone control to the cervical esophagus. These fibers complement parasympathetic vagal input for coordinated swallowing.
Q6. Which cranial nerve is responsible for parasympathetic innervation of the cervical esophagus?
a) Glossopharyngeal
b) Vagus
c) Accessory
d) Hypoglossal
Answer: b) Vagus
Explanation: The vagus nerve (cranial nerve X) carries parasympathetic fibers to the esophagus, facilitating peristalsis and glandular secretions. Recurrent laryngeal branches of the vagus provide the direct motor supply to the cervical portion.
Q7. A tumor compressing the left recurrent laryngeal nerve can present with all EXCEPT?
a) Stridor
b) Hoarseness
c) Dysphagia
d) Hyperacusis
Answer: d) Hyperacusis
Explanation: Hyperacusis is due to facial nerve (CN VII) involvement. Compression of the left recurrent laryngeal nerve by tumors such as lung carcinoma or mediastinal masses leads to hoarseness, stridor, and dysphagia but not auditory hypersensitivity.
Q8. Clinical sign most suggestive of bilateral recurrent laryngeal nerve injury?
a) Aphonia
b) Inspiratory stridor
c) Dysphagia
d) Nasal regurgitation
Answer: b) Inspiratory stridor
Explanation: Bilateral injury to recurrent laryngeal nerves causes vocal cord adduction, narrowing the airway and producing inspiratory stridor. This is a surgical emergency often seen after thyroidectomy complications.
Q9. The cervical esophagus transitions from striated to smooth muscle at approximately what vertebral level?
a) C4
b) C6
c) T1
d) T4
Answer: b) C6
Explanation: At the C6 vertebral level, the pharyngoesophageal junction marks the start of the esophagus. The upper part contains striated muscle fibers, gradually transitioning to smooth muscle in the thoracic portion.
Q10. Which nerve is most vulnerable during anterior cervical spine surgery?
a) Hypoglossal
b) Recurrent laryngeal
c) Vagus
d) Glossopharyngeal
Answer: b) Recurrent laryngeal
Explanation: Anterior cervical spine approaches risk injury to the recurrent laryngeal nerve due to its proximity to the tracheoesophageal groove. Injury can lead to hoarseness, aspiration, and swallowing difficulties.
Q11. The esophageal plexus is primarily formed by?
a) Glossopharyngeal and hypoglossal
b) Vagus and sympathetic fibers
c) Facial and trigeminal
d) Phrenic and accessory
Answer: b) Vagus and sympathetic fibers
Explanation: The esophageal plexus is formed by branches from the vagus nerve and sympathetic chain, integrating parasympathetic and sympathetic control of motility and glandular secretions along the esophagus.